Chirag's Abdomen Review

50
Chi rag's Abdomen Review - Part 1 Location of Abdomen b/w the . thorax and pelvis boundaries = anterolateral musculocutaneous wall and posterior lumbar vertebrae and IV disks. Contains - Peritoneum (Parietal + Visceral) + Peritoneal cavity which surround theGIT contents and part of the urogenital system. Be able to identify both the regions (9) and quadrants of the as well as the important structures in each. May not be tested directly but. it will help you in life aka DPS *Sub Costal Plane @ nOj Transtubercular Plane @ LS; Umbilicus in a healthy individual is @ l3/L4 Abdominal layers Rectus abdominis Rectus sheath Quadratus lumborllm - latisstmus l10rsi Erect or spinae ..--. Sally StmllieIs Eats Indian Toddlers To Enlarge Peritoneum. Layers of Anterior Abdominal wall o Skin o Superficial (Campers (fatty) and Scarpas (subcutaneous) fascia) o External Oblique - (Hands In pocket - why?) o Internal Oblique o Transverse Abdominus o Transversalis Fascia o Extraperitoneal Tissue o Parietal Peritoneum

Transcript of Chirag's Abdomen Review

Page 1: Chirag's Abdomen Review

Chi rags Abdomen Review - Part 1

Location of Abdomen

bw thethorax and pelvis

boundaries =anterolateral musculocutaneous wall and posterior lumbar vertebrae and IV

disks

Contains - Peritoneum (Parietal + Visceral) + Peritoneal cavity which surround theGIT

contents and part of the urogenital system

Be able to identify both the regions (9) and quadrants of the a~domen as well as the

important structures in each May not be tested directly butit will help you in life aka DPS

Sub Costal Plane nOj Transtubercular Plane LS Umbilicus in a healthy individual is

l3L4

Abdominal layers Rectus abdominis Rectus sheath

Quadratus lumborllm -

latisstmus l10rsi ~

Erector spinae -~ --

Sally StmllieIs Eats Indian Toddlers To Enlarge Peritoneum

Layers of Anterior Abdominal wall

o Skin

o Superficial (Campers (fatty) and Scarpas (subcutaneous) fascia)

o External Oblique - (Hands In pocket - why)

o Internal Oblique

o Transverse Abdominus

o Transversalis Fascia

o Extraperitoneal Tissue

o Parietal Peritoneum

Cutaneous N - WETDRY Lab - Make sure to 10 these nerves

T7 - Skin over the Xiphoid Process

T8

T9

TlO - Skin over the umbilicus

Tli

Tl2 (Subcostal N) - innervates the skin superior to the pubic symphysis

Ll- skin over the pubic symphysis

Illiohypogastric N (LI)

IlIioinguinal N - Travels in Inguinal Ligament (Ll)

Genitofemoral N

Three Folds you have to know Dry LabWet LabClinical

1 Median Umbilical Fold (Midline inferior to the umbilicus) - Contains Urachus (obliterated

Allantoic Duct)

2 Medial Umbillical Fold (Lateral to median) - Contains obliterated umbilical artery

3 lateral Umbillical Fold (lateral to Medial) - Contains Inf Epigastric vessels

Lateral to Lateral Umb Fold = Deep Inguinal Ring

Question - Only umbilical fold containing structures that function in a normal adult

Answer - lateral Umbillical Fold - the other 2 have embryonic strucs that dont fxn in adults

Collateral Circulation

The Superior Epigastric and Inferior Epigastric vessels (within the Lateral Umbillical Fold)

anastomose within the rectus sheath How can these vessels function as collateral supply in

the event that either the IVC or the Aorta are occluded

ColLatral circulation lfthe abdominal aorta IS blo~kcd tllese i1rkri~ll anaShJm(hC~ (with origin) compensltlie I Intrnal tho mcidma mmary ubeinian) H superior epi~lstrii (internal thOG1Cl(

H infltIiol epil-strk ((kfnal ilinlt 2 Superiof pancalicodllouenal (celiaC trunk) H inferior pnllcreatkodu-odtTlltI1

(S~L~)

i Jidc11t colic (SIAl H-left c-olie EvJAJ 4- Supc-rior reclal ll[i H- middle feel] Ontcmallllilc)

The superficial Epigastric Veins anastamose around the umbilicus with branches of the lateral

thoracic vein (this is an important collateral blood supply bw the femoral and axillary vein)

If patients have obstruction of IVC or Portal vein (acute or chronic portal hypertension) - these

veins become engorged =caput medusae - This is first of 3 important Portal Caval

Anastamoses - Here are the other ones Guaranteed Questions

portosystemiC anastomoses AV-azygollS vein

EV--esophageal vein lEV-inferior epigastric vein 1M V--int-erior mesenteric vein IRV~~nferior rectal vein IVe-interiQr vena cava LGV--Ieurolt gastric vein PlN--praumhiliCal vein PV--portal vein RV- renal vein SEV- -sliperior epi9astric vein SMV-superior mesentaficveln SRV-SlJperior rectal vein SV--splenic vein

bull Portal venous o SysternicvJnOus

Site- ()f tnlstolTIogtis Clinical sign Portal H systemic 1 Eophagus EsophHgto31 middot lti(e~ Lft gaslric H (-opblgt~ J 1 Umbilicus C1Pllt medmae PlnHUllbilicol H mperlicilI

1 11d inferior cpiglstricshyIkmormoios Snp( ri of re( tal H middle

and inferior redal

-ilfi cf~ pf8ut butt and caput M( commonly $(efl with portll hyperknsiun fli$trtin~] porlo(Iv11 sb unt d i llve(middotn the plenil anJ le ft renal v(ins relieves portJJ

hypertcnlti(n bygtlllH1ting blood to thtgt ~stemic cire-ulation

Make sure you know

How the Portal Vein is formed

Based on the site of occlusion - which Portal Hypertension clinical sign will present

Portocaval Shunt - Clinically Portal vein directly to IVC is not likely - Splenic to left Renal V

3

INGUINAL LigamentCanalHernias Inguinal canal

In fetal life the gonads descend from the posterior abdominal wall In the case of males the

testes must descend all the way outside of the body in order to maintain an optimal

temperature for spermatogenesis to be maintained

What you must know about the inguinal region

Boundaries

o Ant - Ext Oblique Aponeurosis

o Post - Transversalis Fascia amp Int Obliq (Form a conjoint tendon - attached Pub Symph)

o Roof - Aponeurosis of Int Obliq amp Trans Abd

o Floor -Inguinallig amp lacunar lig

o External Ring - Formed by Ext Obliq Aponeurosis

o Internal Ring - formed by conjoined tendon

layers

o Ext Spermatic Fascia (EOmuscle)

o Cremasteric Fascia (IOmuscle)

o Internal Spermatic Fascia (Transversalis Fascia)

Contents Dunkin Donuts Tastes Awesome Please Sip a Caffeinated Grande Vanilla latte

o Ductus Deferens (snip snip =vasectomy)

o Testicular Artery (Called Testicular Torsion if this gets wound up)

o Pampiform Venous Plexus (Bag of Worms -left Hangs lower - why)

o Sympathetic Fibers (Point and Shoot)

o Cremasteric Muscle (Tickle Tickle)

o Genital Branch of genitofemoral N

o Vestige Process Vagina lis

o lymphatic Vessels

Ox of Indirect vs Direct Inguinal Hernias (Protrusions of peritoneum through an opening)

Indired inguinal CQes through the INternal (deep) ingllil1111 FeHowS Ih( path of Ule descent hernil dng external (stlperGeial inguin~l rillg and of the testes Cocred by ~ll1

INto the scrotum Enters internl hlguin31 ring 3 layets )f spermatic filscilt) lateral to inferiof ejgtigastrh artery OC-(gtl1f in INfants owing to failure ofpfo(e~~us vaginalts to dose rfu c h more (Qmmon in males

Direct inguinal Protrudes through the inguilllt11 (Hes~eJhaehs l MDs dont tIc hemi~l triangle Bu1ges directly through abdominal 1ledial to inferior epigastric

wall medial to inferior epi~~1Strilt artcrr alte-ry =Dired hClTlkl

GOtgtS through the external (Hlperfidal ~ Lateral to inferior epigastriC ingnilltl1 ling only Covered by transversalis MtelT =Indired he micentl

fascia Usually in older men

Lymphatics - Please dont get Testes and Scrotum mixed up

o Testes Lymph follows blood supply through inguinal canal back to abdominal aorta to

lumbar and preaortic lymph nodes Metastasis of Testicular Cancer - Livestrong

o Scrotum Lymph (As do lymphatics from all ectodermally derived structures in this

region) drain to superficial inguinal lymph nodes

GI blood supply and innervation Emhryonic Artery Pltlrasytnpathdilt VCTt-cbral Structnr(s suVP li-lt gut region inIlervation level

For(ut Cdi ~1 C Sgl5 T12Ll St)(fIl(b to pmxtll1li dll(llteJlllJl1middot lin-f gallbladder IMlliHa

Ll Dist1I dwXlentll11 Jo proximll -f of lra n ~Hrc colo n

I~lA Di~t~lll lgt of tmmt rsl cokm to upp-r portion [f fthllll _pknic Ilexuf is a wtlkrshtd ngiol1

Heart - Gastric and

duodenal regions

Cenae artery

~IY-~ Primordiuln of liver

Superior llli3Senteric artery 10 mido~n

I Interior mesenteric Hhidgut artecrj

Celia( trunk Bran ches of cd ial( [rnnK (ornm011 ]lmiddotpltic splc- nicleft gastric lhclt -ornpri~t tile main blood supply of the toma(h

Short Ilstric han pC gtOf

1lIltlSlo nlOc if ~pl ~ llic nrkry

is bloch-d Strollt anasbmOHS ltxist

betwe tn

-Left and right ga gtir(jt~plpJoj (s

-Left Lind right gastriCs

KNOW YOUR ANASTAMOSES

IIhlieunci

~plecn to JID$tEfipr i pleric artc~

Biliary structures

BE ABLE TO DRAW A FLOW CHART OF BILE FLOW

Important GI Iigaments

I

COnnect t

Ucr lcl n l)le rid~ abdominal i LlnmeJl1mh teresshy wall 1

I

if o(tal i ri~l c) he~atlc ~l rhry_ l1ay be olnptessed -1 Prin1 vein emiddotolllmn be~ween tlmmb 111d

t blle dud indd fin~d Pticed in cpk ploic farunen nfWinslow hJ ~ntt 1l1Jeecling

t

6n~e tHtre~te(Urid I crIt

middotbullbullmiddot -os

trHr to 1 liSef clltUliue middot1 Gastric m~eri~ 1 n ter and~epIt1tes ghtgrel

of stom~cll I les~rsac _~b be qt 9Brtng urger 1

~ i to ac(~3S le~(r sa

Grcater luC ltinind Ifa1t ofgreater 1)1I1tmiddotntllln

[ran fr colon 1

Lcreater curv ep~r1tCs left gre~br and t $pI eil middot I lesscr bllt$

qI

Splenorenal linil Vd~l 1 bull bull abdaininal JltaU

___o-J-___ ~~~~

~~----- ~-- ~~- ---~

Be able to visualize these ligaments know their embryonic origins and contents

Recognize relationship of structures in epiploic foramen (Portal Triad)

Splenorenal is high yield

Know which ligaments contribute to the greater and lesser momentum

Clinical- If the stomach ruptures posteriorly - contents are confined to the lesser sac but if

the stomach ruptures anteriorly - contents in the greater sac can spread as far down as the

pelvis

Know the relationship of structures within the portal triad - Hepatic Vein will be the trick

answer choice

L sgt-J[ offiEntumHpatic artY proPll

8iloduct bull

Stomach

Glstrapllmic ligament

Ornootnl-+----+r--___-----cT-- ~Jrarn n

IH---V~tal

petitOllllU-m

Splllllofffial figamgn

TXH

LIVER - ClinicalDryWet

Coronary Iigameot

Ewph3g~al

Impression

Lett tria09ular Hepatic veins

ligament

Fibrougt Supralerfal impreurossionappendix of

tiOer Hoparorenal portion ot coronary ligament

Garuic RighitrianQularl igameot impresgtion

(Common) bile duct

Common hepalic duel

Clstic duel

Renal Impression Caudate lobe

Papiilary proc~ss Quadrat~ lobe

allbladder

___ FalCltltorm ligament

Hepatic artery PlOP - Round Jigam~nt 01 liver

Hepatic portal veio ~-- Coli c impression

FisUH for ligamen1vm teres

Porta hepatis

Make sure know

Embryonic origin of the iigaments

Anatomical vs Functional lobes and relationship to the Gallbladder

Portal Triad Structures (VH DISSECTOR)

Impressions

CLINICALS -

Trac heoesophageal Almocml COll 11ction htteell esophalIs ll nd tmlthe1 fistula Jost lI)OUllul1 StLblypt is hlind up tr () hngtls with lOler esophagus tO)HHcted to

lrac hea RcsulJ in (3 11 $($ (ho tng anc vo mitin with ftt(~~lS lir bllblllpoundQll CXR and )Qlvl V -~-

I ~ i I Esophageal y i J atresia - - Trachea

----shy

Z

Congenital pyloric l1l perlrnphy (J f le pylurus camt~ obstruti I) Pa Ipablc uhve mass in epigl~tric n~ion stenosis and nOl1biliolts PfOiL mnit+rrg- - - wetk of ltl4C Tn)tment is suriltai ineisi n

Ottmi in 160(llit births ofteIl in I ~ t-bOfl malts --

Peptk ulcer djsease G~ltri( ulcer Pain can he greater vdth mellls--Welghtl - ft(H OCClIlS in older putient

a Mlori tnfedion in 70 chronic NSAID use also implicated Dm to llHue-oml protection a~lllns t gfltrh l id

Duodella) ulcer Pain Deu(QSe5 wilh menh--weighl ~aln_ Almost IOO have H priori infection Dut to 1~ gastfk acid cerct 11 f( 1 2011 inpmiddotr-Ell lson )~md rol11t i ( r L m(lcos~ ll

protction Hyprttophy of Bntnllcr glands Tend to ha ve ltIeall pullcheJ-ouf margim unlike the tais(dJirre~lllar margins of

cnCinOl1lH P-te1itiai eomplications include bl eedin~ penetrnu(n into pnnerelS perfomtion and obs tmctiQH (not intrinsltllly pr((JIlcerom) (see Image 11-+

Question - A 57 year old obese chronic alcoholic presents with an ulcer which has ruptured the cI~wnpastelgrly Surgical investigation reveals blood in the peritoneum Which of shy

the following arteries is most likely responsible for the bleeding

a Splenic Artery

b Gastroduodenal Artery

c R Gastric Artery

d L Gastric Artery

e L Gastroepiploic Artery

Appendicitis All age ~rol1ps most common indication for em er(nl ilxkminai)lJrger) in ehildrefL huLial diffmc periumbilical pain --7 localized pain JtMcBmm~ point N~ltlSC~l kvef

r)13)- perfor~te - pcntouitis Difftr~ntiI1 divertieuli[i (elderly ectopic pr(~gnanl to [3-hCG 10 rule ont)o

-----

PcmiddotrhtltllCf of the ) lcllille dud Of ~Olk sh31k ~br lontli n e( topic ~l cid-seefehllg gatric m 1(0]

andor pmcrcatic ii~~lle Lllost common c(lllgenital anomliy aftlle CT tract Cm CllHt hkfciing illtusm~(~pjjon Dlnllus or nbstrudion nelr the tcrrnin)l ileum Contraslwitll QIB~efic nmiddot = cvtk dilalaUon of ittllilC dJet

------~-

The ile 2$

2Jpound~11 11~ 2 feet frolll the iie( middoteCill vke l~ QfiJ~ at 1~

CIllIn nly prcsenfltgt III rll~2

llf~ of lifc- by ilwe 2 ty )ts of

epilheH8 19ls-trie- pal1elli(i

Hirschsplungs disease

Congenital tnt91(middotolon characterized by lack of Think of a gian ~pring that ganliml ttlLJcnJ(rk ~~gJeXllS~( (~lihs and lei ~sner plCxpstTlrlsgIllent on inbstinal biop y ()It to-iIure of U(middotural t~restpoundtU migration =

has s nl

Presents as ronic comtipnHoll tHly in life Dilllted pOltioll of the colon proximal 10 the aganiionic

segment resulting in a middot trmsjtillll ZQl1t rnvolt~

rectum [huany farlur( to P~$ meconium

High Yield WetDrylTheory List-

Suggestion - in your study group try to write a question for each of these points and then

exchange with a friend and try to answer each others questions

Abdomen Blood Supply - Reference viks picture posted on my google group - this is THE MOST HIGH

YIELD TOPIC IN ALL OF ANATOMY - expect 5 questions on your mini and 5-10 questions on your shelf

Make sure you can draw the blood supply and answer tertiary questions

Example - If the patient had an occlusion of the celiac trunk - which of the following areas would

experience ischemia

Portal hypertension - Know the 3 clinically relevant sites of portal caval anastamoses

Testes vs Scrotum lymph drainage

Where are paraumbillicai veins located

Omalophcele - failure of the gutto come back in (if in yolk sac - fatal)

Marginal Arteries

Superior Messenteric Artery is in front of 3d part of duodenum

Caput medusa (Distended paraumbillical veins secondary to portal hypertension)

Kidney Constrictors -1 Renal Pelvis 2 Crossing Pelvic Brim 3 Entering Urinary Bladder

Ureter - wet lab

Vagus is PIIJS supply up to 23 trans colon than pelvic splanch N up to the ass

Hirshsprungs disease - baby cant poop - dilated colon

Meckels Diverticulum - rule of 2s - 2 feet prox from ileocecal ju nc

Urachal Fistula - weeping belly

Gall stones - common bile duct

Jaundice relation to tumor of the head of the pancreas

Hepatopancreatic ampulla

Blood supply of renal gland - s superrenal art m s suprarenal a abd aorta inf suprarenal art

R Kidney - Tl2-L3 L Kidney - Tl1-L2

Renal Artery - L2

Epiploic foramen - know the borders and contents

Alantois diverticulum - urachus - medial umbilical

Lateral Medial and Median umbillical Folds (know the contents)

Directindirect hernia - know how to diagnose where they enter and exit the inguinal region and which

one is congenital

Anular pancreas - projectile vomiting

Pyloric Stenosis - projectile vomiting (non bilous)

Duodenal Atresia - projectile vomiting (bilous)

Dry Lab - know x rays

Vitteline Fistula - food out of umbilicus

Major duodenal papilla - junc of foregutmidgut

Arcuate line - relationship to rectus sheath

Mcburneys point -13 from ASIS bw umbilicus

Parietal pain - what is the nerve supply

Internal Oblique - cremasteric relationship

Know spermatic fasia

Processes Vaginalis - connection bw peritoneum and gubernaculums

Umbillicus - TlO dermatome

Deep Inguinal-l25 cm above mid inguinal ligament

Superficial Inguinal Ligament- superolateral to pubic symphysis

Variocele - veins engorged in scrotum (bag of worms)

bull

bull External spermatic fascia derived from external obliques EO II Cremasteric fascia ~ from internal obliques fO bull Internal spermatic fascia derived from fascia transversalis bull Tunica vaginalis derived from processes vaginalis directly rests on testes bull know order from testes out to skin

note reflex o ilioinguinal nerve o Efferent =genital branch of the genitofemoral nerve

--lt gt-- info important anastamoses which connects thorax to abdomen

bull Sup

o Sup epigastric branch of internal thoracic o Inf branch external iliac

Venous drainage o Above umbilicus aXillary v o Below umbilicus veins in triangle o At level of umbilicus Paraumbilical veins -gt drain into the portal V

II Important in Portal Caval Venous system Venous drainage of testes

o Clinical correlation Varicocele 11 vein drains into IVC 11 Left testicular vein ~ drains into left renal v

bag of rmlt

for lymph drainage T10 axillary lymph nodes

ill Below T10 superficial inguinal lymph nodes (lateral

Umbilical Folds

Lateral umbilical folds inferior vessels

Medial umbillcial folds umbilical (fetal remnant)

Median umbilical fold urachus (fetal remnant)

Between these folds fossas o Supervesical fossa between median and medial folds

11 bladder o hesselbachs between medial and I folds

II DIRECT HERNIAS HERE Borders

Medial semilunar line

Lateral info Epigastric

Inferior inguinallig o Lateral Inguinal Fossa beyond lateral fold

INDIRECT HERNIAS HERE II Deep inguinal ring (lateral to inferior epigastric a)

Indirect inguinal hernia o Lateral to inferior epigastric a o more common o When inserting finger in superficial inguinal ring will feel on tip of finger (since it goes

throueh ineuinal canall

----

Dry Lab - Label subcostal iliohypogastric Ll Ilioinguinal (Ll)

Horesshoe Kidney - stuck under IMA

Renal Agenesis -failure of the ureter bud to develop

Double Ureter

Unilateral Agenesis -1 kidney

Kidneys - Metanephros

Fetal kidneys are at sacral level

Look at 3rd part of duodenum

Some of this stuff is repeated I know just copied and pasted a bunch of stuff I had copy

Dermatomes

bull T4 nipples

bull no umbilicus v o Pain referred to no in appendicitis o Pain referred to T7ITS in gastritis ~

Inguinal ligament = external abdominal oblique aponeurosis

bull Inserts at anterior superior iliac spine to the pubic tubercle o Why important to know -7 visualizing this line allows us to properly diagnose a hernia

Below the inguinallig femoral hernia Above the inguinallig =inguinal hernia

Also to palpate the deep inguinal ring you go about 12Scm above the mid-inguinal

point

bull Modifications to ligament o Pectinate ligament o Lacunar ligament -7 cut this ligament to relieve strain i~ stran ul~tEd hernia

Inguinal canal

bull in males -7 transmits spermatic cord o important structures of spermatic cord ductus deferens testicular a genital branch of

the genitofemoral n pampiniform plexus of veins bull in females -7 transmits round ligament

Borders

bull Floor -7 inguinal ligament + lacunar ligament bull Anterior -7 aponeurosis of external oblique + internal oblique bull Roof -7 internal oblique and traverse abdominal bull Posterior -7 transverse abdominal + transversalis fascia

o Reinforced by conjoint tendon bull Aponeurosis of internal abdominal obliques and transverse abdominus bull Lies immediately behind the superficial inguinal ring in what would otherwise be

a weak point in the abdominal wall bull Innervated by ilioinguinal nerve (Ll) ~why important

bull In appendicitis Ll can be injured which will injure this nerve and in turn

the conjoint tendon With loss of innervation to this supportive structure the patient is now predisposed to a direct inguinal hernia

o Only hernia that can transverse the inguinal canal o Associated with congenital condition persistent tunica vaginalis

bull Direct inguinal hernia o Medial to inferior epigastric a o When inserting finger in superficial inguinal ring will feel on back of finger o Associated w old age or recent surgery

Muscles (Abdomen RECTUS SHEATH)

bull Arcuate line at level of ASISor 13rd distance between pubis and umbilicus bull Above arcuate line rectus abdominus is surrounded by a rectus sheath anteriorly and

posteriorly

o EO and 10 lie over rectus abdominus o 10 and TA lie behind rectus abdominus

bull Below arcuate line rectus abdominus has no rectus sheath posteriorly o EO 10 and TA lie over rectus abdominus o Transversalis fascia lies behind rectus abdominus o Inf EpIgastric vessels pierces the rectus sheath here

Peritoneum serous sac which encloses most of the abdominal structures

bull Ovary =only intraperitoneal organ o Oocyte ejected from ovary then captured by fallopian tubes o Why impt Women more prone to infection that can enter peritoneum

Peritoneum forms

bull Mesentery double layered fold of peritoneum formed as the organ was pulled in

bull Ligament between 2 organs in general bull Omentum between stomach and another organ bull Bare area area of no peritoneum bull

Viscera innervation

bull Pa rasympathetics 11 o Afferents sense hunger o Efferents l peristalsis relaxes sphincters gland secretion

bull Sympathetics o Efferents do opposite o Afferents CARRY PAIN SENSATION OF THE VISCERA (dull stretching pain)

bull PARASYMPATHETIC INNERVATION o Vagus nerve 7 _1l to 23rd unct ion of la rgej nte~tine oJ)elVrcspla~~~)~~ic~rYe~ IJiU- ~rd aJ~lpoteotiD~ IMPT

Gut Embryology

Gut ~ We say that the gut is derived from endoderm We often forget that when we say so we mean

that only the mucosa is derived from the endoderm The submucosa and the muscle layer is actually derived from the splanchnopleuric mesoderm and the serosa is derived from the visceral peritoneum

~ The main function of the gut is to digest the food which is done by the glands derived (and are) in the mucosa (endoderm) The only two exceptions in the Gut where glands though derived from the endoderm do not stay there but migrate down into the submucosa are esophagus and duodenum These glands however have their ducts opening to the swface of the mucosa

bull

~ Lungs liver amp gall bladder and pancreas are off-shoots from the foregut Esophagusshy~ The region of the tube from the laryngeal diverticulum to the beginning of the stomach elongates

to form the esophagus ~ The glands which form in the endoderm (mucosa) migrate down into the submucosa The path

whlch it took migrate becomes the duct of the glands which open to the mucosa ~ Achalasia Cardia - Failure of relaxation of the lower esophageal sphincter because of congenital

absence of ganglia at the sphincter (The ganglia when present releases VIP (Vaso-IntestinalshyPeptide) which relaxes the sphincter)

Mid-Gut Rotation ~ Because of the 90 degree rotation of the primitive stomach all of the following events occur ~ Lesser curvature comes to the right Therefore lesser omentum also comes to the right ~ Greater curvature comes to the left Therefore greater omentum also comes to the left ~ Right side vagal trunk becomes posterior vagal trunk ~ Left side vagal trunk becomes anterior vagal trunk ~ The left side peritoneal cavity comes to the anterior aspect of the stomach and will later be called

as the greater sac ~ The right side peritoneal cavity comes to the posterior aspect of the stomach and is (relatively a

small sac because the liver is on the right) called the lesser sacomental bursaepiploic sac ~ Epiploic foramen of Winslow (the lower free margin of the ventral mesentry) wiII be the

communication between the greater and lesser sac ~ The Liver moves to the right and therefore actually causes the 90 degree rotation of the stomach

The spleen comes to lie on the left side ~ Axis Antero-posterior axis around the superior mesenteric artery

bull Counterclockwise bull Approximately 270deg bull During herniation (about 90deg) bull During return (remaining 180deg)

Duodenum ~ Becomes retroperitoneal (except the first part which is still suspended by the hepato-duodenal

part of lesser omentum) ~ Glands (of Brunner) go submucosal ~ An imaginary line drawn below the opening of the major duodenal papilla represents the junction

between the foregut and midgut ~ Duodenal atresia in Downs syndrome Liver ~ 3rd week

bull liver bud grow bull into the septum bull transversum

~ 10th week bull hematopoietic bull function

bull 10 of the total bull body weight

~ 12th week bull bile is formed

Pancreas ~ In about 10 of cases the duct system fails to fuse and the original double system persists ~ 3rd month

bull pancreatic islets (Langerhans) ~ 5th month

bull Insulin secretion ~ Annular pancreas

bull The right portiCn of the ventralbud migrates along its normal route but the left migrates in the opposite direction

~ Complete obstruction of duodenum ~ Accessory pancreatic tissue Polyhydramnios (Amniotic fluidgt 1500-2000 ml)

~ Congenital defects including central nervous system disorders (eg Anencephaly) and gastrointestinal defects (atresias ego Duodenal esophageal) prevent the infant from swallowing the amniotic fluid (failure of recanalization)

Oligohydramnios (Amniotic fluid lt 400 mt) ~ Cl~ldberenal-agenesis

bull Midgut _-_

~ Primary Midgut intestinal loop gives rise to bull Distal duodenum bull Jejunum bull Ileum bull Ascending colon bull Transverse colon - proximal two-thirds of the bull Transverse colon with the distal third

~ Primary intestinaltoop bull ncephalic limb distal part of the duodenum the jejunum and part of the ileum bull ncaudal limb lower portion of the ileum the cecum the appendix the ascending colon and

the proximal two-thirds of the transverse colon bull 6th week

bull Rapid elongation of the cephalic limb bull Rapid growth of the liver bull Intestinal loops enter the extraembryonic cavity in the umbilical cord

bull 10th week bull loops begin to return bull regression of the mesonephric kidney reduced growth of the liver expansion of the

abdominal cavity bull Jejunum -left bull Loops - more to the right

bull Cecal bud -last part (temporarily below the right lobe of the liver) ~ qIDlthaloseJe (Structures COlHLoArts9V~1tion)

bull Through umbilical ring bull 6th to 10th weeks

bull Associated with a high rate of mortality (25) and severe malformations bull Associated with chromosome abnormalities

~ Gastroschisis (Structures coming out are not covered by Amnion) bull herniation through the body wall ----=---=-shybull Into the amniotic cavity bull Lateral right of the umbilicus bull Sometimes the inferior wall fails to develop as a result lower abdominal structures like the

bladder would be exposed to the exterior not associated with chromosome abnormalities ~ Abnormalities of the Mesenteries

bull Mobile cecum persistence of mesocolon bull Extreme form - long mesentery bull Volvulus

~ Distal third of the transverse colon ~ Descending colon ~ Sigmoid colon ~ Rectum ~ Upper part of the anal canal ~ Primitive anorectal canal

bull 7th week cloacal membrane ruptures bull Tip of the urorectal septum perineal body bull Pectinate line

~ Hindgut anamolies bull Rectoanal atresias and fistulas bull Imperforate anus bull Congenital megacolon (aganglionic megacolon Hirschsprung disease)

bull

bull Hindgut

Chirags Abdomen Review - Part 2

Understanding Embryo makes learning blood supply EASY

I I

I t

~ -

)

Table l1r-~ L Adult SUmiddotuctu~SDrj~l Froln Each of he Three Dhisions of be Pringttive GUl Tube t-middot-----middotmiddotmiddotmiddot-

Foregu(

I_ (Celiac Trunk)

Ir-slt-gtphgus

S101na(b

I h -= LiJ~r

Pancre=l S

bull 1 i Biliary apparntu5

Gall bladdshy

i Pha11~Cal pltgtuchcs

LullSS-I

Mjig ---- bull __ _- ----n--duct----~---middot-------l--n

(Superior Jldesen1eric Artery)_-1I-(I_~__ middot __ O-=-)_in_middoto_r_M_e_se_n_t_e_r_i_c_An__

Uuodenu rn 2nd_ 3 lt141h V4Tt

Jejunun-~

nc-un]

tCCUJ11

AppltgtndLX

Transver5e -o1on (p~oxiln1l1 ~O Tbird)

bull__hytgtid~ _ _ ______ L _ __

Tr-dn~llt~se colon (diStul h lTd) I

)

i

Aa ca-nal -( uppeT patt) i

I I

_____ __ _ _ _ ___ - - rhe~ a(t clcriVOkt iV(5 opound~lt prbn1rC ~ nlQC blft TlI)( 134tof r~ tIonoinf~ i 1 ~l l1rd c- P Cle

Now Lets see how much youve learned

Questions

1) A pt receives a general anesthetic in preparation for a c~t~~my A right subcostal incision is made which begins near the xyphoid process runs along and immediately beneath the costal margin to an anterior axillary line and transects the rectus abdominus muscle and rectus sheath At the level of the transpyloric plane the anterior wall of the

-~~-~=--- _eco---shysheath of the rectus abdominus muscle receives contributions from which of the following

a Aponeuroses of the in~ande~tef-Ilal o~ues

b Aponeuroses of the transversus abdominis and internal oblique muscles c Aponeuroses of the transversus abdominis and internal and external oblique

muscles d Transversalis fascia e Transversalis fascia and aponeurosis of the transversus abdominus muscle

A

2) The lat~raJJJ11QjJt~gLfgJlLoneach side of the inner surface of the anterior abdominal wall is created by which of the following structures

K Falx inguinalis (~) Inferior epigastric a

c Lateral border of the rectus sheath d Obliterated umbilical a e Urachus

B

3) A man the victim of several knife wounds to the abdomen during a brawl at the Lobster Shack subsequently developed a direct inguinal hernY Damage to which of the following nerves is most likely responsible for the predisposing weakness of the abdominal wall

~ Genitofemoral nerve ( b) Ilioinguinal nerve ~-t Tenth intercostal nerve

d Subcostal nerve e Pelvic splanchnic nerve

B

4) Which of the following statements concerning a direct inguinal hernia is correct a It is the most common type of abdominal hernia b It transverses the entire length of the inguinal canal c It contains all3 fascia layers of the spermatic cord d It exits the inguinal canal via the superficial ingeJinal ring e It protrudes through H~acb strJg e

~(

1fltbS w E

tl

5) The conjoint tendon is

a Important in preventing indirect inguinal hernias b The fused aponeurotic layers of internal abdominal oblique and transversus

abdominus muscles c Posterior to the deep inguinal ring

d Medial fibers of the inguinal ligament

B

6) A 25 year old male is brought in to the ER after being involved in a car accident in which he received a crushed internal injury in his abdomen Examination reveals a lesion of parasympathetic fibers in the vagJsnerve which interferes with glandular secretory or

smooth muscle functions in which of the foliowingorgans a Bladder b Transverse coloiW c Descending colOO d Prostrate gland e Rectum

B

7) The spermatic cord includes all of the following contents except a Il ioinguinal nerve b Pampin iform plexus of veins c Vas deferens d Genitofemoral nerve

A

8 Which abdominal structure gives rise to the internal spermatic fascia (muscle) following the descent of testes in development

a External abdominal oblique aponeurosis b Transversalis fascia c Transversus abdominis muscle d Peritoneum e Internal abdominal oblique

B

9 Which abdominal structure gives rise to the tunica vaginalis fotlowing the descent of testes during development shy

a External abdominal oblique aponeurosis b Transversalis fascia c Transversus abdominis muscle d Peritoneum e Internal abdominal oblique

D

10) The lesser omentum is a peritoneal fold which is su bdivided into the a Hepatogastric and gastrosplenic ligaments b Hepatoduodenal and gastroomentalligaments c Hepatoduodenal and gastrosplenic ligaments d Hepatogastric and hepatoduoden9-jrj igaments

D

11) A posteriorly perforating ulcer in the pyloric antrum of the stomach is most likely to produce initiallocalized peritonitis or abcess formation in which ofthS fQllowing

a Great-sac - -- -

b Paracolic recess

c Omental bursa

d Right subphrenic space

c

The inferior mesenteric artery arises from the abdominal aorta ilm_ediill~y_J-Qs1eriQLto which of the foowing org~ns A-F~t~filie duodenum B Head of the pan~eis C Neck of the pandeas

D Second part of the duodenum

E Third part of the duooenum_shylaquoshy

shy

The correct answer is E The inferior mesenteric artery arises from the anterior surface of the aorta at the level of the third lumbar vertebra The third part of the duodenum crosses the midline at the level of the third lumbar vertebra and passes anterior to the aorta at the origin of the inferior mesenteric artery The

first part of the duodenum (choice A) lies horizontally to the right of the midline at the level of the first

lumbar vertebra The head of the pancreas (choice B) is to the right of the midline and extends from the

level of the first lumbar vertebra to the third lumbar vertebra It lies within the concavity of the

duodenum The neck of the pancreas (choice C) lies in the midline at the level of the first lumbar

vertebra It lies on the anterior surface of the aorta at the origin of the superior mesenteric artery The second part of the duodenum (choice D) lies vertically to the right of the midline and extends from the

level of the first lumbar vertebra to the level of the third lumbar vertebra

The left adrenaLvein drains directly into which of the following veins A Hemiazygos vein

B Inferior vena cavaee C Left renal veiri -

D Splenic vein

E Superior mesenteric vein

a

The correct answer is C The left adrenal vein and the left gonadal vein (either testicular or ovarian) drain into the left renal vein TheTeft renal vein t~ains intothe- inferior vena cava In contrast the right

adrenal ~~inandnght gonadal veindrai~ gLr~ctJy iQtoJhe iilferiQ[ Vencava -- -

ThehemTazygoS7ein- (~h-~i-~ A)~~c~i~es the venous drainage from the body wall on the left side of the

thorax and abdomen No visceral organs drain directly to the azygos or hemiazygos veins The inferior vena cava (choice B) receives the direct venous drainage from the right adrenal vein but not

the left adrenal vein Remember the inferior vena cava is on the right side of the abdomen The splenic

vein (choice D) receives the venous drainage from the spleen and part of the pancreas and stomach The splenic vein is part of the portal venous system

The superior mesenteric vein (choice E) receives venous drainage from much of the intestinal tract It is part of the portal venous system and joins with the splenic vein to form the portal vein

A 43-year-old man presents complaining of pain in the groin On examination his physician palpates a

bulge in the region of the superficial inguinal ring which he diagnoses as a direct inguinal hernia The hernial sac most likely

A is covered by all three layers of the spennatic fascia B passes medial to the inferior epi gastric artery

C passes medial to the lateral border of the rectus abdominis muscle

D passes posterior to the inguinal ligament E passes through the deep inguinal ring

The correct answer is B Direct inguinal hernias enter the inguinal canal by tearing through the posterior

wall of that structure The typical location for this type of hernia is through the inguinal triangle bounded

laterally by the inferior epigastric artery medially by the lateral border of the rectus abdominis and

inferiorly by the inguinal ligament Direct inguinal hernias pass medial to the inferior epigastric artery

whereas indirect inguinal hernias pass lateral to the inferior epigastric artery because the deep inguinal

ring is lateral to the artery Indirect inguinal hernias are covered by all three layers of the spermatic fascia (choice A) Direct inguinal hernias are covered by fewer than all three layers because the direct inguinal

hernia tears through one or more layers of fascia as it emerges though the abdominal wall The lateral

border of the rectus abdominis muscle (choice C) forms the medial border of the inguinal triangle All

inguinal hernias pass lateral to the rectus abdominis Femoral hernias pass posterior to the inguinal ligament (choice D) Inguinal hernias emerge through the superficial inguinal ring which is superior to the inguinal ligament Inguinal hernias that descend below the inguinal ligament pass anterior to the

ligament Indirect inguinal hernias pass through the deep inguinal ring (choice H) direct inguinal hernias

do not Both types of inguinal hernias pass through the superficial inguinal ring

During a gastric resection in a patient with stomach cancer a surgeon wants to remove the lesser

omentum because of tumor extension into it Which of the following structures lie in the free edge of the

l~~g omentum and consequently must be dissected out in order to be preserved

A Common bile duct cystic duct and hepatic artery 6

B Cystic duct hepatic artery and hepatic vein

e Hepatic vein and cystic duct

Portal vein common bile duct and hepatic artery

E Portal vein hepatic artery and hepatic vein

The correct answer is D The free edge of the lesser omentum contains three important structures the

common bile duct the hepatic artery and the portal vein Nei ther the cystic duct (choices A B and C) nor the hepatic vein (choices B C and E) lies in the free

edge of the lesser omentum

A 55-year-old male patient with chronic liver disease has portal hypertension To relieve the pressure in the portal system a porto-caval shunt is performed Which of the following veins may by anastomosed to

accomplish this porto-caval shunt A Left renal vein-left testicular veingt

B Right renal vein-right suprarenal vein I shy

e Splenic vein -left renal vein J

D Superior mesenteric vein-inferior mesenteric vein E Superior mesenteric vein-splenic vein

The correct answer is C The splenic vein drains directly into the portal vein The left renal vein drains

directly into the inferior vena cava Anastomosis of these veins would allow blood from the portal vein to

drain retrograde though the splenic vein into the renal vein and then into the inferior vena cava The left

renal vein (choice A) drains directly into the inferior vena cava The left testicular vein drains directly into

the left renal vein Thus these veins are already in communication and neither vein is part of the portal venous system The right renal vein (choice B) drains directly into the inferior vena cava The right

suprarenal vein also drains directly into the inferior vena cava Thus neither vein is part of the portal

venous system The superior mesenteric vein (choice D) drains directly into the portal vein The inferior

mesenteric vein drains into the splenic vein which then drains into the portal vein Thus neither vein is

part of the caval venous system The superior mesenteric vein (choice E) drains directly into the portal

vein The splenic vein also drains directly into the portal vein Thus neither vein is part of the caval

venous system

A 12 year old boy has fever vomiting and para-umbilical pain After examining the patient the doctor

makes an initial diagnosis of appendicitis Appendicular pain which is initially referred to the umbilicus goes to the dorsal root ganglion of

a TI b TI2 c L1 d T7

(e I TIO

A 59-year-old male undergoes a neurological examination which reveals that when the abdominal wall is

stroked the muscles of the abdominal wall of the side of the body stimulated failed to contract Other

neurological tests appeared normal The likely region affected includes

a CI - C5 spinal segments b C6 - TI c T2-TI ~T8-T12

e Ll- L5

The surgery done to relive portal hypertension is done by connecting two veins Which of the following veins would be suitable for connection

a Inferior vena cava and portal vein b Superior vena cava and portal vein c Splenic vein and right renal vein d Splenic vein and left renal vein e Superior mesenteric vein and Inferior vena cava

A mother brings her 3-week-old infant to the pediatric clinic reporting a new scrotal bulge that she found -~-

while changing a diaper yesterday The infant is afebrile Physical examination reveals a palpable mass in

the scrotum while in the standing position resolution of the mass in the supine position and no

transillumination of the scrotal sac What is the most likely diagnOSiS

a Cryptorchidism b Direct inguinal hernia c Hydrocele d Indirect inguinal hernia ~ e varicocele

The Vagal trunks enter the abdomen by passing through which of the following openings in the

diaphragm

a Right crus b Esophageal hiatus ~ c Vena caval hiatus d Aortic hiatus e Left crus

2 The anterior boundary of the epiploic foramen of Winslow is bounded by

a) First part of duodenum b) Lesser curvature of stomach c) Liver d) Hepato-duodenalligament v ~

3 The ilio-inguinal nerve is derived from

a TI2 ry b LI c L2 d L3 e L23

15 Surgically the structure used to suspend the kidney to the diaphragm is

a) Renal fascia b) True capsule c) Perinephric fat d) Paranephric fat

6 If there is portal obstruction because of carcinoma affecting the pancreas which of these of the

following signs would be present

a Caput medusae b Esophageal varices c Rectal varices c

d Pulmonary edema

7 In a sliding hernia the gastro-esophageal junction lies

a) At its normal position b) Below the normal position c) Above the normal position V d) None of the above

8 Which of the following structures is retroperi toneal

A transverse colon B spleen IJ2f6 C ileum D descending colon v r 1pound1111111

9 The renal angle is fonned lgtetween the 12th rib and ______ muscle

a Psoas major -middotshyb Erector spinae c Quadratus Iumborum d Diaphragm

10 The anterior structure at the hilum of the kidney is

a) Renal vein ~

b) Renal artery I middot~ I

c) Ureter d) Accessory renal artery

11 Because of origin of the muscle from the lateral one third of the inguinal ligament it

could not fonn the anterior wall of the inguinal ligament

a) External oblique b) Internal oblique c) Transversus abdominis_ d) Rectus abdominis

12 A large tumor mass impinges on the splenic artery and its branches as the artery pass out from below

the greater curvature of the stomach Branches o(which of the following arteries would most likely to

effected by the pressure on the splenic artery

a Left gastric b Left gastro-epipJoic c Right gastric d Right gastro-epipoloic e Short gastric_

13 A new born baby has projectile vomiting after each feeding It is determined that there is obstruction

of the digestive tract as a result of annular pancreas Annular pancreas is as a result of an abnormality in which of the following process

a Rotation of the dorsal pancreatic bud around the first part of duodenum b Rotation of the dorsal pancreatic bud around the second part of duodenum c Rotation of the dorsal pancreatic bud around the third part of duodenum d Rotation of the ventral pancreatic bud around the first part of duodenum y Rotation of the ventral pancreatic bud around the second part of duodenum

14 As the liver bud enters the ventral mesogastrium the region of the mesogastrium stretching from the

liver to the anterior abdominal wall is called

a Lesser Omentum b Greater Omentum ~ Falcifrom ligament d Lacunar ligament e Ligamentum teres of liver

16 A patient has absence of his 12th rib In such a patient if the doctor makes an incision to approach his

kidney mistaking the 11 th rib for the 12t he would end up injuring

Which of the following arteries is a direct branch of the gastroduodenal artery The

A right gastric artery

B left gastric artery

C inferior pancreaticoduodenal artery D left gastroepiploic artery

i E)right gastroepiploic artery --

E x pI a nation The right gastric artery is typically a branch of the proper hepatic artery The left gastric artery is a direct

branch of the celiac trunk The right and left gastric arteries anastomose along the lesser curvature of the

stomach The inferior pancreaticoduodenal artery is a branch of the superior mesenteric artery it

anastomoses with the superior pancreaticoduodenal in the head of the pancreas The left gastroepiploic

artery is a branch of the splenic artery it anastomoses with the right gastroepiploic artery along the greater

curvature of the stomach The right gastroepiploic artery is a branch of the gastroduodenal artery The

other branch of the gastroduodenal artery is the superior pancreaticoduodenal artery

Which of the following pairs of veins join together to form the portal vein The

A superior mesenteric vein and inferior mesenteric vein

B inferior mesenteric vein and splenic vein

C superior mesenteric vein and splenic vein

Ip)splenic vein and left gastric vein E superior mesenteric vein and left gastric vein

Explanation

The portal vein is formed behind the neck of the pancreas by the union of the superior mesenteric vein

and the splenic vein The inferior mesenteric vein drains into the splenic vein The left gastric vein drains

directly into the portal vein After the portal vein forms it enters the hepatoduodenalligament of the

lesser omentum to reach the liver The portal vein is the most posterior structure in the hepatoduodenal

ligament

At which of the following vertebral levels does the duodenum pass anterior to the aorta - _- shy

All ~

B L2 7~

CL3 I

~DL4

E L5

Explanation

The duodenum begins at the pyloric sphincter at the level of Ll The second (or descending) portion of

the duodenum is to the right of the aorta and extends inferiorly from the level of Ll to the level of L3 The third part of the duodenum crosses the aorta from the right side to the left side at the level of L3 The

fourth (ascending) portion of the duodenum extends from the level of LJ to the level of L2 The

duodenum ends at the duodenojejunal flexure The superior mesenteric artery passes anterior to the

duodenum as the duodenum passes anterior to the aorta The duodenum can be constricted at this level

In which of the following locations will perforation of the digestive tract result in the spilling of luminal

contents into the - lesser peritoneal sac

A Anterior wall of the second portion of the duodenum B Posterior wall of the second portion of the duodenum

C Anterior wall of the stomach

~Posterior wall of the stomach E Posterior wall of the transverse colon

Explanation

The posterior wall of the stomach is related to the lesser peritoneal sac The anterior wall of the stomach is related to the greater peritoneal sac The anterior wall of the second portion of the duodenum is related to the greater peritoneal sac The posterior wall of the second portion of the duodenum is related to the retroperitoneal space The posterior wall of the transverse colon is related to the greater peritoneal sac

The ureter lies against the anterior surface of which of the following muscles shyA Crus oftne diaphragm B Quadratus lumborum

0 Psoas major D Transversus abdominis

E Iliacus

Explanation The ureter exits the renal pelvis at about the level of vertebra L2 As it descends along the posterior abdominal wall it lies on the anterior surface of the psoas major The psoas major muscle arises from the bodies of the lower lumbar vertebrae The psoas major muscle is joined by the iliacus to fonn the

iliopsoas muscle The iliopsoas muscle then attaches to the lesser trochanter of the femur and is the major

flexor of the hip

As the right ureter passes the pelvic brim it lies against the anterior surface of which of the following

blood vessels

A Gonadal artery B Inferiorvena cava C Internal iliac artery

rJ- External Iliac artery

E Inferior mesenteric artery

Explanation

The ureter lies in the extraperitoneal space in the posterior abdominal wall Alter leaving the kidney it

passes inferiorly on the anterior surface of the psoas major muscle At the pelvic brim the ureter passes

into the pelvis At this point the common iliac artery is dividing into the external and iliac arteries The

ureter lies on the anterior surface of the external iliac artery immediately distal to the bifurcation This is a useful landmark for a surgeon to locate the ureter

When extravasated urine passes from the superficial perineal space into the anterior abdominal wall it is

found immediately deep to which of the following layers of the anterior abdominal wall

-ltScarpas fascia

B External oblique muscle

C Internal oblique muscle D Transversus abdominis muscle

E Transversalis fascia

Explanation

The superficial perineal space is bound by Colles fascia the fibrous portion of the superficial fascia This

layer of fascia is continuous with Scarpas fascia the fibrous portion of the superficial fascia of the anterior abdominal wall Therefore urine that is deep to Colles fascia will remain deep to Scarpa s fascia The urine will spread in the plane between Scarpas fascia and the external oblique layer

When a horseshoe kidney develops the ascent of the kidney is restricted by the A internal iliac artery B external Iliac artery

C common iliac artery

inferior mesenteric artery

E superior mesenteric artery

Explanation

A horseshoe kidney develops when the inferior poles of the to kidneys fuse together as they ascend into

the abdomen from the pelvis The first anterior midline vessel that is encountered by the horseshoe kidney

is the inferior mesenteric artery This artery prevents the kidney from continuing its ascent

The left testicular vein drains into which of the following veins

A Left internal iliac vein B Left common iliac vein

bflnferior vena cava D Left renal vein I

E Left internal pudendal vein

Explanation

The left testicular vein drains into the left renal vein The right testicular ~i~[~nsltjectlY into the

inferior vena cava This difference in venous drainage is believed to explain the greater incidence of

varicocele on the left side than on the right The venous drainage from the penis is to the internal vein

which then drains into the internal Iliac vein

The spinal nerve that provides cutaneous branches to the skin around the umbilicus is

A TS B TW-shy

C TI2

DL2 EtA

Explanation

The tenth intercostal nerve is the anterior ramus of the TIO spinal nerve After passing through the tenth

intercostal space the nerve continues forward in the anterolateral abdominal wall in the plane between

the internal oblique muscle and the transversus abdominis muscle In the abdominal wall the nerve innervates to the abdominal wall muscles as well as the skin and the parietal peritoneum The umbilicus is

a useful landmark for the region of distribution of the tenth thoracic nerve

The ligament of the vertebral column that resists its extension is the Aligamentum flavum

B supraspinous ligament

C posterior longitudinal ligament

D anterior longitudinal ligament

E interspinous ligament

Explanation

The ligaments of the vertebral column that resist flexion of the column include the supraspinous ligament

interspinous ligament ligamentum fiavum and posterior longitudinal ligament The ligament that resists

extension is the anterior longitudinal ligament This longitudinal ligament is very broad and strong It

covers the anterior and anterolateral surfaces of the vertebral bodies and the intervertebral disks In

addition to resisting extension the anterior longitudinal ligament provides reinforcement to the anterior

and anterolateral surfaces of the intervertebral disk The posterior longitudinal ligament is relatively

narrow and covers the posterior surface of the vertebral bodies and the intervertebral disks This ligament

reinforces the posterior surface of the disk The posterolateral surface of the disk is not reinforced and it

is through this region that herniation of the nucleus pulposus usually occurs

A patient presents with epigastric and right upper quadrant pain The pain is most intense 2-4 hours after

eating and is reduced by the ingestion of antacids The patient states that he has passed black tarry stools

(melena) within the last week Fiberoptic endoscopy reveals a yellowish crater surrounded by a rim of

erythema that is 3 cm distal to the pylorus Accordingly an ulcer has been identified in the patients

A fundus

B antrum

C duodenum

D jejunum

E ileum

A number of physiologic genetic and other factors increase the risk of gastric (and duodenal) peptic

ulcers The evidence that H pylori plays a principle role is compelling Smoking and caffeine are known to adversely affect the morbidity mortality and healing rates of peptic ulcers In general first-degree

relatives of peptic ulcer patients as well as males have a threefold to fourfold increased risk of developing this disorder Paradoxically in gastric ulcer disease acid secretion is not elevated It is possible that

excess secreted hydrogen ion is reabsorbed across the injured gastric mucosa In general a defect in gastric mucosal defense is the more important local physiologic

A patient presents with symptoms of duodenal obstruction caused by an annular pancreas Annular pancreas is caused by

A rotation of the dorsal pancreatic bud into the ventral mesentery B rotation of the ventral pancreatic bud into the dorsal mesentery

fJ failure of the major and minor pancreatic ducts to fuse ~ ~ cleavage of the ventral pancreatic bud and rotation of the two portions in opposite directions around -the duodenum E formation of one pancreatic bud instead of two

Explanation Normally the ventral pancreatic bud rotates around the gut tube to reach the dorsal pancreatic bud The two buds fuse to form a single pancreas and the distal portions of the two ducts fuse The ventral pancreatic bud forms the inferior portion of the head of the pancreas the uncinate process and the major pancreatic duct (of Wirsung) The dorsal pancreatic bud forms the superior part of the head the neck body and tail and the minor pancreatic duct (of Santorini) Annular pancreas is the result of the ventral pancreatic bud dividing into two portions before it rotates into the dorsal mesentery Each portion rotates in opposite directions to get to the dorsal mesentery thus encircling the duodenum The presence of annular pancreas can constrict the duodenum thus obstructing its lumen

In n _ phranlc----

Gon ~l ----_1 Lum bltano

~~--- CornmQ1t bull ac

+-~4--- lnlllirnaJ ilic

xtem iliac

OBJECTIVE - Identify the blood supply to each of the structures listed in the table on the previous page

Ill give you a head start

FOREGUT - Supplied bV Celiac Tru nk (T12)

Proper hepatic

GastiooUod 13Jafter

1nferlor pancreaticoduodenal artery

Common epatlc

Lett gas ric iiirtery

Spfen artery

shy Gastroepiphgtic artery

~ Superior mesenteric 8rtfry

~

1 Esophagus is a derivative of the foregut so its blood supply originates from the celiac trunk

(T12) The predominant blood supply to abdominal portion of the esophagus is the Esophageal

A (Branch of L Gastric) The venous drainage of the esophagus is particularly important because

it is 1 of 3 clinically relevant sites of Portal Caval anastamoses The Portal Esophageal Vein

meets the Caval Azygos System Persistent bleeding manifests as Esophageal Varices - a fata I

condition

2 The Stomach is also a derivative of the foregut has EXTENSIVE blood supply and is very high

yield on anatomy exams The lesser curvature is supplied superiorly by the L Gastric A (1 of 3

major branches ofthe Celiac trunk) and inferiorly by the R Gastric A ( a branch ofthe proper

Hepatic A) The greater curvature is supplied superiorly by the L Gastroepiploic A (a major

branch of the splenic A) and inferiorly by the R Gastroepiploic A

The Short Gastric arteries (branches of Splenic Artery) supply the fundus of the stomach and

are referred to as EIID ARTERIES because they have no collateral blood supply Therefore if the

splenic artery were occluded (ex - increased pressure in the ommental bursa) - there would be

ischemia to the fundus of the stomach Venous drainage of the stomach is extensive via various

veins lead ing to the portal system Posterior to the stomach the IMV joins the splenic V which

joins the SMV to form the PORTAL VEIN ADAMS

3 Duodenum blood supply has high clinical relevance because it is the junction of the foregut and

midgut and therefore is the site of anastamoses between branches ofthe Celiac Trunk (main

foregut artery) and the Superior Messenteric Artery (main midgut artery) The Proper hepatic

artery gives off the gastroduodenal artery which travels behind the 1st part of the duodenum

This point has high clin ical relevance because duodenal ulcers are very common and a posterior

rupture of the 1st part of the duodenum could rupture the gastroduodenal artery causing

traumatic abdominal bleeding The Gastroduodenal artery first gives off the R Gastroepiploic A

(mentioned above) and proceeds as the Superior pancreatico duodenal artery (supplies the

pancreas and duodenum) which anastamoses with the inferior pancreatico duodenal A (branch

of the SMA) This is the junction of foregut and midgut and occurs near the opening of the

bil iary system into the duodenum (ampula of vater) Portal venous drainage here is responsible

for delivering nutrients from digestion to the liver for metabolism Appreciate that the Superior

mesenteric artery (artery of the midgut) branches from the aorta at Ll travels posterior to the

pancreas than moves anteriorly (at the jxn of the pancreatic headbody) and comes over the

3rd4th part of the duodenum Tumor of the head of the pancreas can compress the SMA

4 Jiver blood supply is via the common hepatic artery (major branch of the cel iac trunk) The

common hepatiC becomes the proper hepatic gives off the R gastric A and the Gastroduodenal

A and then joins the common bile duct and the portal vein in the portal triad Clinical- if a

patient were bleeding from the hepatic A a surgeon can stick his fingers in the epiplOic foramen

and squeeze the free edge of the hepatoduodenalligament in order to stop bleeding to the

area Please note that the hepatic a branches into Rand L hepatic A The Right hepatic artery

gives off the cystic artery which supplies the gallbladder Afferent venous supply is via the

Portal vein which is bringing nutrient rich blood to the liver After metabolism takes place

venous blood leaves the liver through the hepatic veins into the IVC PLEASE UNDERSTAND THE

RELATIONSHIP OF THESE STRUCTURES - ADAMSNETIERSNH Etc

5 Pancreas - Head is supplied via the superior and inferior pancreaticoduodenal arteries

(mentioned above) The tail (situated towards the hilum of the spleen) is supplied via the

pancreatic branches of the splenic artery (END ARTERIES) This blood supply is very important

because the endocrine Alpha and Beta Cells from the pancreatic islets of lagerhans are located

towards the tail This is where Insulin and Glucagon is released to the blood

Now complete this for mid and hindgut structures Make sure to note clinically relevant arterial

anastomoses as well as portal caval anastomoses FYI Appendix blood supply SMA + IMA

anastamoses marginal artery Portalcaval rectal veins fhemmorhoids) and periumbilical caput

medusa are high yield THE BUTT THE GUT and THE CAPUT

Abdominal Development

Liver

Ij1f

II wall b

oh liN ~ VltJrti n be- bull

Pancreas

Secondary Retroperitonealization e I~tl r 1 a v-mtrai m ellter

Rotations of the Gut I i Ij (lIl1UtIJ f~ l r tilt

()l td 10 me l-ft and he v

--~--- -~ -~-~

i

I AolaijonjoI~guf I

STOMACH BED (IDENTIFY IN ADAMS)- the structures posterior to the ommental bursa which

support the stomach in the supine position

Abdomnal JQrUI

Splnic vein

OmQ-oul tv~ ) O(s(Jroa)

Lojt(r o m nturrt (hpJtodu o d~n31 Hid

Gadrl)SplerH (g3stroll~nal) IIgam~nt

hiad h~~atogrtricent IIQdmiddotcrt~)

Lt Dome of Diaphragm (why left Look this up in Adams)

Spleen (What is the blood supply)

Left Kidney (What is the blood supply - AND how is it different from the R kidney)

Suprarenal Gland (What is the Arterial AND Venous Blood supply - how are they different)

Pancreas (How does supply differ from Head to Tail What is the SMA Relationship)

Transverse Mesocolon

liver - ADAMSWET - Make sure you look at the liver in wet lab

Left triangular nl1am~nt

ComoaDj ligamnt

Erophg~1 impre$ioo

Hepatio veins

In1erior -ifena middotr3)Ia

Fibrous appendix o-t

live

impr~j on

Heprorendl p~rtion of Q)(Qllary ligament

Righllri~n9ul r 1I~met

(Common) bile quol

Gr)mmCtr~ hepatic dlJct

Ccentic duct

Duodenal impression

GaJdate p-fr)~S

Hepatic artgtrl prop-f iiiiila - Faloiform ligament

_ - shy Round ligamen liver

~--F-- CoJio imprgt-ssi-on

Prta heptis

Identify the lobes impressions and embryonic remnants associated with the liver

Caudate Lobe Quadrate Lobe Right Lobe Left Lobe Round ligament Falciform Ligament

Ligamentum Venosum (what is its fxn in embryonic life) Hepatic Veins (NOT PART OF THE

PORTAL TRIAD) IVC PORTAL TRIAD - Contents relationship cross section etc Know the

Galbladder relationship to the lobes of the liver

Biliary Duct System - Make sure you understand the sequence of these structures - BE ABLE TO

DRAW A FLOW CHART

TPVd i

t

I t

1 __ Cm-(r

patk GlJet

I

J

Clinical = JAUNDICE is caused by anything that prevents delivery of bile to intestine Tumor of the

head of the pancreas Stones etc Patient will have pale stools and yellowish colored mucus

membranes

Clinical- Any scenario that tells you the patient has BILLOUS VOMIT means that the obstruction to

the flow of digestive contents is after the Ampulla of Vater (Site of Entry of Billiary system to the

duodenum) - ie Duodenal Atresia

Spleen -located posterior to the mid axillary line between ribs 9 and 11 Make sure you know that

the 10th rib is the main axis of the spleen and this organ is susceptible to injury (stab wound errant

thoracoce ntesis etc)

The spleen is derived from mesodermal cells - NOT THE GUT TUBE

The spleen rests on the left colic flexure associates with the tail of the pancreas Know the

structures entering the Hilum of the spleen

Sh rt O~-t~ic 1 0(0 10 rtiltSPIric Iloa nt

(cut)

Peritoneum - similar concept to Pleura - think of a fist in a balloon

Visceral Peritoneum - Layer of balloon touching your fist

Parietal Peritoneum - Layer of balloon not touching your fist

Your fist represents the organ your wrist is the hilum and your arm contains the blood supply

entering the organ

Appreciate that there will never be organs in the peritoneal cavity - rather these organs invaginate

the cavity Kaplan videos

RULES OF NOMENCLATUREshy

1 Organ completely surrounded by peritoneum - peritoneal organ

2 Organ partially surrounded by peritoneum- Retroperitoneal

3 Peritoneum surrounding peritoneal organ is VISCERAL peritoneum

4 Peritoneum surrounding retroperitoneal organ is PARIETAL peritoneum

5 Peritoneum connecting visceral to parietal is called messentary 2 messentaries in the

gut Dorsal (to the gut tube) and ventral (to the gut tube) messentary

Aorta is in Retro peritoneal position - but blood must reach peritoneal position - vessels travel through

messentary All peritoneal organs will have blood supply reaching through messentary

-Mesentery is a 2 layer peritoneum with a neurovascular communication between body wall and organ

- Ligament connects one organ with another or to the abdominal wall (Ommentum = ligament)

lesser Ommentum (attach lesser curvature of stomach and duodenum to liver) =Hepatoduodenal

Ligament and Hepatogastric Ligament

Has a Superior and Inferior Recess (Accumulation of Fluid in Ascites)

Communicates with the greater sac through the epiplic foramen (what structures pass through

this foramen)

Boundaries - you must be able to visualize this

o Anterior - stomach

o Posterior - parietal peritoneum pancreas

o Superior - superior recess (bw diaphragm and coronary ligament)

o Inferior -Inferior recess (bw layers or greater momentum

Greater Ommentum (attach greater curvature of stomach) Gastrophrenic ligament Gastrosplenic

ligament gastrocolic ligament

The greater omentum is the largest peritoneal fold It consists of a double sheet of peritoneum folded on itself so that it is made up of four layers The two layers which descend from the greater curvature of the stomach and commencement of the duodenum pass in front of the small intestines sometimes as low down as the pelvis they then turn upon themselves and ascend again as far as the transverse colon where they separate and enclose that part of the intestine

ABDOMINAL PAIN

Parietal Peritoneum - supplied by same vasculature lymphatics and nerves supplying body wall it

lines and diaphragm Sensitive to pain pressure heat cold well localized

Visceral Peritoneum - supplied by same vasculature lymphatics and somatic nerve of organ it covers

Insensitive to touch heat cold and laceration - referred to dermatome of spinal ganglia providing

sensory fibers Where does appendicitis refer to

Foregut pain - epigastric area (ie - cholycystitis)

Midgut pain - periumbilical area (ie - appendicitis)

Hindgut Pain - suprapubic area (ie - diverticulitis)

Extra ImagesConcepts

ll~_____-

FalifCtrm ligament oind r~ud ligamet f Ilver

Blood from splenio gastriC and inferiof rne$e-rteri v~ins

Ca-I tributaries

Lett gastrio Ifein

Posterior superior pan~reatioodul)denal vaihS

Lott gamo-om~nlal (9aropip lomiddotic) -in

Poq_~ tjol imerl-9-r panCJertlcorllJod-nal veiopound --amp----I- - ~J Right grtr~-omntal

Anwrior interi (gartroepiploic) Jjn

pan euaii cod vl)denal veins middot Inf~Ji (t r mesentric vein

Miqdle (olic vein

Right cl)licvein Sigmoid and rectosigml)id (ei ns

IhH)Collc(~io

--- Mi~dl laquooLJl gtjrltgt

PoM ca vl1 illasto)moses -----shyampoptoageal 2 Paraumbilie-lt11 Inferi or Fectal vei ns

3 Recial 4 REuoperHonea1

Know how the Portal vein is formed I 4 sites of portal caval anastamoses and 1 clinical shunt

Col li t ltt-~ otTl~tI ~nj pc~ 1lt1 turJoG

Ltf 14i1 tImiddot~ artoftl9 on tj phtAt$

L-oftqf 4t t~r 1=laquoIran d 1 bull shy~p l ci rj o fOOOts

Nerves follow the arteries - appreciate the splanchnic nervous system I

Uet~ric branch of left ~nal art

Ureterie branch of righi renal artelY

Left Zld lumbar in and co mlTlunication to as)erdin9 lumbar l(~in Hi ~ht tEZ1~~t~ t3r j t itn ~ nJ l1t- rlnd lfe i r1

Inferior me5nteri~ artery

Notice that the right testicular vein drains directly into the IVC and the right testicular artery drains

directly into the aorta However the left testicular vein drains into the L renal vein at a right angleshy

reason left testicle is lower and more susceptible to varicocele (bag of worms)

Also notice that the left renal vein has a longer course because the IVC is on the right side whereas

the right renal artery has a longer course because the aorta is on the left side

Appreciate the anterior to posterior relationship of structures in the hilum of the kidney - VAP - Vein

Artery Renal Pelvis (Ureter)

11____ __ L_ L_ n VJ __ _ _ t_L I I_ _ L __ L_ I -pound1 bull LI_~-I ____

Posterior View of Head of Pancreas in ( of Duodenum

Celiao hunk

Co mmon ~L~jJth art~ry

GastNduQdonal artrf (partilly in phantn)

P1)Sterior $Up~Jior panCflaticuduodfmal art~r~t

(Co mm on) bile duct

middot~1t~~t-1l---~-~- Right gshomiddotomental (gastoe plp lolc) 3rte (phantomost)

Grener paocre atic art-ry

1n1~rjor pancr-iatlc artery

Jtrifll supejo r pal)oreailcento)dJodenal artr1 (phantom)

Anastomotlo branch

POostetlor bJanch of jo f~ri of pan-reatir(lduodensl drttnj

Anterio r branch of i flferior palcreati~)duodenal art~(phan1om)

Notice the extensive blood supply to the pancreas and duodenum via the branches of the celiac trunk

Notice collateral supply from SMA branches - makes sense bc this is the jxn of foregutmidgut

Identify the vessels in this arteriogram

Hiltid i)f N~ck oi B)dvof Tail 01 pa nereas pan cent~as P-nmiddot-reas panCtCas

I nferie v~na cava

jHept1iic p(lrlai v~in

Port1 tnd H~pti lt a ftH prol

Comm on) bll duct

Ouodtnum

~ft colic (sio)Atta~ hmtrlt jt~xJr-ofha~elSe

muo(IIQn

Right ~lIc (h~j)tic)

il~gtture

In1triol m~oten lIein (rttr op~ritoMdO

SlJp efl or mes~n~fiC amrV and lipln

KNOW YOUR NEIGHBORHOOD

Questions

vVhiJh structure supplied by a bnmdlof the cclia( artery is not derivcd from foregut LemCJUCrITI

(A) Head of the pancte-a5

CD) Pyloric duolenum

Cystkduct

( Liver hepatocyt~~

~F) Body of the spleen

An infant presents with an omrhaJucele at birth -hi oJ the [oHm illg applies to his cM1-dition

(A) It is 31so seen ill p4titnts with aganghonic megacolon

(11) ft reuirs from a fal1ure of resorption of theviteUine d let

(C) It results from herniation at the-site of regression of the right umbilk vein

DJ It is caustd by faihtrc of recanalization of the midgut part of the duodenum

~ It ill camioo by a failuIt vf the midgul to return to the abGQminal uity after herniashytion in-n the urnbilk s l stalk

Ot er than the spleen occlusion Cif the spit-Ilk artery at its odgin wm most likely affect die blood supply to jllch st cnud

(A) Jejunum

(B) Body of th pal1~lltas

(C) LeSStT Cllmiddotlaturc of tl )toma-ch

(D Duodenum dista to the entrance of the Ornmou bile duct

E Fundus of the stomach

A 38-yeu-old batL~er with a history of heartburn suddenly experiences excluciating pain in the (plgastric region of th~ abdomeu SurgCry is perf~rme immediard y upon admisshysion to the 1IlcrgCJliy tuomh~re i~ evidence uf a ruptured ulcer in the posterior waU of the stomach Vhere will a surgeon first fi nd the stomach contenlSf

A) Greater p4ritoneal sac

rB) Cul~de-s~c of Douglas (--

C Omental bursa ~

--D) Paracolic gutter

rEj Between -he panttal perimltum and the posterior body wal1

At birth an infant presents with a st()ma~ rb~tbas~njJled jfltotb~diaplfagru 1A1ltre is the defect thatresulied iiitJle heini~t()n shy~tsophagealbiatus

7 - rH-- Hiatus for the inferior vena cava

( Pleuroperitoneal membrane -(0) Septum transvcrsum

(E) Right Crlt~

An infant born with DOVv7l syndrome presents with bili()u~ vomiting Ahat congenital defect does the infant have

(A) Pyloric stenosis

(B) Meckel diverticulum C) Ornphaloce1e

(D) Gastroschisis

( ~ ) Duodenal atresia y A patient with cirrhosis of the liver presents with ~ bacalvaricestnlreased retrograde pressure in which veins caused the varices

(A) Paraumuilical

(B) Splenic

(ct AzygltJus

(15))G~trk ( (-F) Superior mesemeric

A htaltby 3-year~old male patient experiences a hernial sa protruding from the anterior abdominal wall about halfway between me anterior superior ilia spine and the pubk tuberde Pulsations of al1 artery are palpated medial to the protrusion site through the abdominal walL Which layer of the anterior abdominal wall will first be traversed by the

1hctma

fA) Rectus sheath (B) External oblique aponeurosis

(C) Inguinal ligament

lD) Transversalis fusda

(E) Cremasteric fa~cia

After 5urgi(aj ffpair of a hernia the patient tXperienccs mtmlgtness in the skin on the anteshyrior aspect of the S(Totum_ Vhaf nerve may have been lesioned during thehemiorrhaphy

(A) Femoral

(B) Obturator

(C) Ilioinguinal

(D) lliohypogastrk

(E) Pudendal

A 23~year-LJld female secretary il1 good health ~-uddcn1) doubles over with pain in the a ea of the 1JmbRicu$ Sbe feels vartn and ltneasy and has no appetite That night the pain seems to have mQved to the tower right abdominal regjol1 and she calls her family doctor who then arranges for an ambulance to pk-k her up and take her to the hospitaL Wh ell ntn~ perceived in the area of the urnbilirus most Hkely carried lhe pairfu I sensations into the eNS

tA) Vagus nerves I~

V B)

) Lessersplanchnk nerves

tC) Pudendal nerves

(D) lIiohpogastrk nerves

(E) Greater splam ic l erves

A CT reveals carcinoma in the bOod of the ancreas Vhich blood vessel trut ourses ----~- - -bull ------ --shy

immediately poftterior to the body ofthe pancreas is the m~t likely to be oompressed

(A) Splenk artery

(B) Abdominal aorta (C) Portal vein

(1) Splenic vein

(E) Renal vein

A patient has a penrln1l1ng uker of the posterior wall ot the br~l part ot the (lUooenmn llkh blood vessel is subject to erosion

(A) Common hepatic artery

(B) Gastroouodenal artery

(C) Proper hevatic artery

(D) Celiac artery

(E) Anterior inferior 11amrelltlcoduodcnal attery

Your patient has been diagnosed -ith a carcinoma locallted to the head and l~e(k of the pancreas Another clinical sign would be

A esophageal varices

(8) hemorrhoids

C) a caput medusa

(D) increased pra Teuro n th~ hepatic veins

(E) enlarged right supra lavkular lymph nodes

Wltkh of the foUowing structures develops in the ventral mesentery

(A) Spleen

(B) Jeiunum (C) Head of1ht pancreas (D) Transverse colon (E) Stomach

ti l Uw ~ littwin~ f( S-t lil oai Imdge ~ hi(h or tbt la~)d J truetur tgt liJ llntn nl) he hl p UC iJd [IIi ell

c o

A) drains Ie tht infCrior a La aI

R t middot~nfl0 ~ill to th~ lunlgtn of h i dtlndCrlllfH

(e) m t bull JiJattd on tl l J n T ~H

D ) sup Lc O VSlt I Hlid bhtu l 1 li - -I un oid

( ) U~tpli(t tr j middottUh~ 1 v(( b~nt rfK n1ilc~Zm

ANSWERS AND EXPLANATIONS

Answer E The spleen is t hlttnopodicand lymph organ demlted from mesoderm

Answ~ R Al1 tlmphalocele is caused by it failure of the nlidgut to return to the ahdomir nat cavity after herniation into the umbiliau Stalk Choices Aand D maybe seen in infants with Down syndrome choice D ~s the specific CBuse ofduudcnal JtiCSitt Choice C is (ile cause of gclstrosbisis and Choice B nsults iu a Meurolktldivertku1-tlB

Answer B The fundus ofthe stomach is suppHed by soort gastric brunches of the splenic altery The splenic artery supplies the body and tail of the pancreas part of the greater curvature of the sttmla(h and the spleen Te jejunum part of the head of the pancreas and tht~ duodenum distal to the entrance of the commOll bile duct are supplied by the superior mesenterk artery clll~l ~be less r ctlt1ature cmd the pylQric antrum are supplied by the right and lei gastric art(ries

AnSWftt C Tbeomental bursa or lesser ~ritoneaj sac lies direcdy posterior to the proxshyimal part of the duodeTtlm and the stomach and would be the first site where stomach contents ~Ott1d be fpoundluncL

Answer C A defect in a llleuropcritoneal membrane (uswlly the left) is the typical site of i1 cc-ngenitlI diilphragluatic hemia llere the membr4ne fails to dose ()pound( of the perishycCirdiopcritulleal canals

Answer E DuoJenal atresia and aganglionic megacoion are congwitaI defects S~Il in patients with Dowmiddotnsyndrome

Answer D RulaTgemt~llt of and retrograde flow in g~lstrk vel_ns in particlJl~r the kft gas~ tricveins dilates the capillary bed in rhe wall of the esophagus in (ases of porta yper~

tension Blood flow would increase in and dilampte tribntarkgts of the (lZygOUS vein on the other side of the capiUary bed but flow in this vein is in the typical direction t()ward the superior vena cava Paraumbiii(ltU vein eilgorgement contributes to a caput medusH Splenic ~nlargement might prc~nt with 5plcnonlegaly and balt-kflow in to tlu superior m~~ntclic vein occurs but is asymptomatic

Answer D The patient hagt an indirect inguinal hernia whi~h emerges from the antt-rior abdominal wall through the deep inguinltilling Theeep ring is a fault in the transv~rshysaUs fascia this I~yer wiIJ be penetrated first by the hernia

An~Wer C The ilioinguinal nenc which provides sens~llion to the lnedlal thigh ltmclanteshytior SClotunl pass~lt th rough the 5uperfh_ial inguinal ring ind $subject to inj i1T) becaus-e

it is in the operatitm Held of the erniorrhapny

Auswer B The leMHr splanchnic nerves are sympathdic nerVlts that carry viscera l sensashytlltgtrogt ftom illtllt1m~d ()J stietched gust (itinteitinal ~tructures (in this case the pprndix) into tnt eNS Lesser splanchnic ntTYcsarisc from thmiddot T9--T12 spinal cord segments lt1nd provide sympathetic innenation tD rnidgut siruc1ures whiCh include CLe app~JldD Viscera] Pain arising from affecLed Inidgut ampt 1C1ure is referred over the same dl- matorne~ of spinal segrnertts v-hich provide the sympathetic Innervation n this G1SC of appendicitis the invohen~n t of the ltire) of t e unlhHku indud s the T 10 dermatome

Answer B Of the five choices onty the dscending olon is retroperiton~al aldwould be a lik ~ ( choice to be seen immediately a(~jilcent to t11e posterior abdominal middotn~L

Amwen D The SpltftlC ~-ein ourses posterior to the body of the panneas m its way tt drain into the superior mCSfttltlri( vein

Answcr B TILt glstrodllolticnal artery 1 direct hIamh of the comrootl hepatic artery courses immediately pt))iwri() to the duodenum and is slbject to erosion

Answer B Carcinoma of th pan middott3S in the 1tilt1 may compreampgt the portltil vein at irs orishygill The poTtai vcin is fomled when the splenic vein jQiaswith tfie superior meStllt eric vein The inferiot mesenteric vein joins the ~plenjc vein just priOT to tlli~ point at which the splenic joins the superior Jlleit1ltcri( vein Increescd venous presslu in the inferior mesenteric vein is a cause of emo hoid~

Answer C The- velltral pancreas wilich forms most of the head of the p ~ncr as develops in the ventral mes(ntery as antutgrowth of the hepatic diverticulum Th~ hepatic divershyticulull induding the biIJary appa~atus develops in tbe ventral mesentery of the foregut

Answer~ A The superior mesenteric ~in joins with the spienkvein to form the hepatic portal vciu

Answer D The structure at gttlK is the proper hepatic artery~ whkh suppUesoxygenated b middotood to the liver

MAKE SURE YOU KNOW the diff bw Rectus Sheath above and below the arcuate line

ABOVE

Aponeurosis of xiiltmal obllque musclo

Extemll f)biquw musde

Reotln ilbdomlnls musole S~in

Internal 9bliquQ mY~QI

AponeUfOsi$ of hJH$V~~S Lir9a a lb lbdolTlin~ musolo Tri OJV6 rUi

atldomlnis mUS(loe

Sub cutanlilous tiue (tatty ye r)

BElOW

A POrl lJfosis 01 etemal oblique muscle

Aponeul~)sis 01 Internal oblique mU$cl~

Anteriol lay~ of r~ltdus st~ath EXttom1 oblique rnu$cll

Rectus Jbdominis muscle Intoernal Aponeurc-sis of tra~fersU$ oblique muscle-

at-domlnis muscentl ~ Skio

Tra nsvitSus abdomioLs ml)ZClt

TralSVersaHs fascia Medial umQil iegtt1 1i9Jment -and folj

Uldchus Peritoneum (ir median Umbilj~al Suboutane ous

Extraprftone 11ascia

Ymbilimiddot~1 fold)

preu9poundiea1 fascia

tissue (fatty 4nd m~mbr3n(iUS layers)

o Above the arcuate line (A horizontal line 13 of the distance bw the umbilicus and the

pubic symphysis) -10 Aponeurosis divides into an AntPost Laminae

o The Ant Laminae joins EO and Post Laminae joins Trans Abdominis = Ant and Post

RECTUS SHEATH respectively

o BElOW the arcuate line - all 3 aponeurosis join ANTERIOR to rectus muscle to meet its

counterpart in the midline (linea Alba)

o Take away Msg - The abdomen is devoid of a posterior rectus sheath below the

arcuate line and is therefore more vulnerable to herniasinjuries

Question - A physician makes a deep incision in the patients midline immediately superior to

the pubic symphysis which of the following layers is his knife least likely to pass

Rectus Abdominis External Oblique Ant Rectus Sheath Posterior Rectus Sheath All of the

Above

Answer - All of the above None of the other answer choices are midline structures -LINEA

ALBA

Linea Alba has very poor blood supply - doesnt heal well after surgery Therefore this is a

common site for incisional hernias

a Spleen b Transverse colon c Descending colon d Stomach e Pleura

17 Meckels diverticulum is normally found 2 feet proximal from the

a Pyloric sphincter b Lower esophageal sphincter c Ileo-cecal valve d Middle valve of Huston e Anal valve

18 Ulcer in the posterior wall of the first part of the duodenum would erode ___ artery and would cause bleeding

a Left gastric b Right gastric c Hepatic artery proper d Gastroduodenal artery e Middle colic artery

19 An inflamed appendix is identified by a surgeon on the operation table by noting

a The appendicies epiploicae b The convergence of tenia c The artery of Drummond d The mesocolon e The mesosalphinx

20 The nerve which emerges through the psoas major is

a Femoral b Ilio-inguinal c Ilio-hypogastric d Pudendal e Subcostal

21 The right gonadal vein drains into the

a Azygos b Hemiazygos c Inferior Vena Cava d Right renal vein e Left renal vein

22 The hepatocytes in the liver is derived from

a Ectoderm b Endoderm c Mesoderm

d Neural ectoderm

23 Abscess in the lumbar vertebrae due to tuberculosis would spread to the adjacent muscle which is

a Psoas Major b Iliacus c Quadratus lumborum d Tranversus Abdominis

24 The anterior wall of the inguinal canal is formed by

a External oblique and transverses abdominis b External oblique and fascia transversalis c Internal oblique and external oblique d Internal oblique and transverses abdominis e Fascia transversalis and peritoneum

Meckels diverticulum is a result of which of the following developmental abnormalities shy

A Failure of the vitelline duct to close

B Failure of the herniated intestinal loop to retract into the abdomen

C Failure of the urachus to close

D Failure of the midgut to rotate

E Failure of the hepatic duct to close

Explanation

Meckels diverticulum is a result of the persistence of the proximal part of the vitelline duct This

diverticulum is usually found about 2 feet proximal to the ileocecal junction and is usually about 2 inches

long It is present in about 2 of the popUlation It may be the site of ectopic pancreatic tissue or gastric

mucosa and may develop inflammatory processes and ulcerations Acute Meckels diverticulitis

simulates appendicitis

Which of the following veins carries blood from the esophagus to the portal vein The

A right gastric vein

B left gastric vein c splenic vein D azygos vein

E left gastroepiploic vein

Explanation

The left gastric vein a direct branch of the portal vein drains blood from the lesser curvature of the

stomach and the inferior portion of the esophagus Because branches of the portal vein do not have

valves blood can flow in a retrograde path when there is an obstruction to flow through the portal system or liveL Rlooci Cln then flow from the nortl] vein thr()1Ph the left PRstric vein to the esonhlPlIS lno

through venous communications within the submucosa of the esophagus to esophageal veins that drain

into the azygos vein The increase in blood flow through the esophageal submucosal veins results in esophageal varices

On the posterior wall of the abdomen the celiac ganglion A contains cell bodies of postganglionic parasympathetic neurons B is synapsed upon by neurons in the posterior vagal trunk C is synapsed upon by neurons in the greater splanchnic nerve D contains sensory cell bodies of lumbar spinal nerves E contains cell bodies of neurons that cause an increase in the rate of peristasis

Explanation The celiac ganglion is one of the preaortic ganglia of the sympathetic nervous system It contains cell bodies of postganglionic sympathetic neurons The sympathetic splanchnic nerves contain preganglionic sympathetic neurons that pass through the sympathetic chain without synapsing These splanchnic nerves go to the preaortic ganglia to synapse The greater splanchnic nerve contains preganglionic neurons from spinal cord segments T5-T9 This nerve synapses in the celiac ganglion The nerve fibers in the vagal trunks are preganglionic parasympathetic fibers that go to the walls of the organs that they will innervate and synapse on postganglionic parasympathetic neurons in the walls of those organs Cell bodies of sensory neurons in the abdomen are found in the dorsal root ganglia or the sensory ganglia of the vagus nerve Sympathetic innervation decreases the rate of peristalsis parasympathetic innervation increases the rate of peristalsis

Which of the following pairs of arteries will allow blood to bypass an occlusion of the celiac trunk

A Left gastric artery-right gastric artery

B Left gastroepiploic artery-right gastroepiploic artery

C Superior pancreaticoduodenal artery-inferior pancreaticoduodenal artery

D Splenic artery-common hepatic artery

E Left gastric artery - proper hepatic artery

Explanation The anastoOlosis of a branch of the celiac trunk and a branch of the superior mesenteric artery will

provide collateral circulation around an occlusion of the celiac trunk Each of the other choices pair

branches of the celiac trunk therefore these will not provide collateral flow around the obstruction of the

celiac trunk The left gastric splenic and common hepatic arteries are direct branches of the celiac trunk

The right gastric artery is a branch of the proper hepatic artery which is a branch of the common hepatic artery The left gastroepiploic artery is a branch of the splenic artery The right gastroepiploic artery is a

branch of the gastroduodenal artery whlch is a branch of the common hepatic artery

Which of the following organs has appendices epiploica The

A sigmoid colon

Bjejunum

C duodenum

D stomach E esophagus

Explanation Appendices epiploica are characteristic of the colon Appendices epiploica are subserosal accumulations

of fat None of the organs of the gastrointestinal tract has appendices epiploica except the colon

Page 2: Chirag's Abdomen Review

Cutaneous N - WETDRY Lab - Make sure to 10 these nerves

T7 - Skin over the Xiphoid Process

T8

T9

TlO - Skin over the umbilicus

Tli

Tl2 (Subcostal N) - innervates the skin superior to the pubic symphysis

Ll- skin over the pubic symphysis

Illiohypogastric N (LI)

IlIioinguinal N - Travels in Inguinal Ligament (Ll)

Genitofemoral N

Three Folds you have to know Dry LabWet LabClinical

1 Median Umbilical Fold (Midline inferior to the umbilicus) - Contains Urachus (obliterated

Allantoic Duct)

2 Medial Umbillical Fold (Lateral to median) - Contains obliterated umbilical artery

3 lateral Umbillical Fold (lateral to Medial) - Contains Inf Epigastric vessels

Lateral to Lateral Umb Fold = Deep Inguinal Ring

Question - Only umbilical fold containing structures that function in a normal adult

Answer - lateral Umbillical Fold - the other 2 have embryonic strucs that dont fxn in adults

Collateral Circulation

The Superior Epigastric and Inferior Epigastric vessels (within the Lateral Umbillical Fold)

anastomose within the rectus sheath How can these vessels function as collateral supply in

the event that either the IVC or the Aorta are occluded

ColLatral circulation lfthe abdominal aorta IS blo~kcd tllese i1rkri~ll anaShJm(hC~ (with origin) compensltlie I Intrnal tho mcidma mmary ubeinian) H superior epi~lstrii (internal thOG1Cl(

H infltIiol epil-strk ((kfnal ilinlt 2 Superiof pancalicodllouenal (celiaC trunk) H inferior pnllcreatkodu-odtTlltI1

(S~L~)

i Jidc11t colic (SIAl H-left c-olie EvJAJ 4- Supc-rior reclal ll[i H- middle feel] Ontcmallllilc)

The superficial Epigastric Veins anastamose around the umbilicus with branches of the lateral

thoracic vein (this is an important collateral blood supply bw the femoral and axillary vein)

If patients have obstruction of IVC or Portal vein (acute or chronic portal hypertension) - these

veins become engorged =caput medusae - This is first of 3 important Portal Caval

Anastamoses - Here are the other ones Guaranteed Questions

portosystemiC anastomoses AV-azygollS vein

EV--esophageal vein lEV-inferior epigastric vein 1M V--int-erior mesenteric vein IRV~~nferior rectal vein IVe-interiQr vena cava LGV--Ieurolt gastric vein PlN--praumhiliCal vein PV--portal vein RV- renal vein SEV- -sliperior epi9astric vein SMV-superior mesentaficveln SRV-SlJperior rectal vein SV--splenic vein

bull Portal venous o SysternicvJnOus

Site- ()f tnlstolTIogtis Clinical sign Portal H systemic 1 Eophagus EsophHgto31 middot lti(e~ Lft gaslric H (-opblgt~ J 1 Umbilicus C1Pllt medmae PlnHUllbilicol H mperlicilI

1 11d inferior cpiglstricshyIkmormoios Snp( ri of re( tal H middle

and inferior redal

-ilfi cf~ pf8ut butt and caput M( commonly $(efl with portll hyperknsiun fli$trtin~] porlo(Iv11 sb unt d i llve(middotn the plenil anJ le ft renal v(ins relieves portJJ

hypertcnlti(n bygtlllH1ting blood to thtgt ~stemic cire-ulation

Make sure you know

How the Portal Vein is formed

Based on the site of occlusion - which Portal Hypertension clinical sign will present

Portocaval Shunt - Clinically Portal vein directly to IVC is not likely - Splenic to left Renal V

3

INGUINAL LigamentCanalHernias Inguinal canal

In fetal life the gonads descend from the posterior abdominal wall In the case of males the

testes must descend all the way outside of the body in order to maintain an optimal

temperature for spermatogenesis to be maintained

What you must know about the inguinal region

Boundaries

o Ant - Ext Oblique Aponeurosis

o Post - Transversalis Fascia amp Int Obliq (Form a conjoint tendon - attached Pub Symph)

o Roof - Aponeurosis of Int Obliq amp Trans Abd

o Floor -Inguinallig amp lacunar lig

o External Ring - Formed by Ext Obliq Aponeurosis

o Internal Ring - formed by conjoined tendon

layers

o Ext Spermatic Fascia (EOmuscle)

o Cremasteric Fascia (IOmuscle)

o Internal Spermatic Fascia (Transversalis Fascia)

Contents Dunkin Donuts Tastes Awesome Please Sip a Caffeinated Grande Vanilla latte

o Ductus Deferens (snip snip =vasectomy)

o Testicular Artery (Called Testicular Torsion if this gets wound up)

o Pampiform Venous Plexus (Bag of Worms -left Hangs lower - why)

o Sympathetic Fibers (Point and Shoot)

o Cremasteric Muscle (Tickle Tickle)

o Genital Branch of genitofemoral N

o Vestige Process Vagina lis

o lymphatic Vessels

Ox of Indirect vs Direct Inguinal Hernias (Protrusions of peritoneum through an opening)

Indired inguinal CQes through the INternal (deep) ingllil1111 FeHowS Ih( path of Ule descent hernil dng external (stlperGeial inguin~l rillg and of the testes Cocred by ~ll1

INto the scrotum Enters internl hlguin31 ring 3 layets )f spermatic filscilt) lateral to inferiof ejgtigastrh artery OC-(gtl1f in INfants owing to failure ofpfo(e~~us vaginalts to dose rfu c h more (Qmmon in males

Direct inguinal Protrudes through the inguilllt11 (Hes~eJhaehs l MDs dont tIc hemi~l triangle Bu1ges directly through abdominal 1ledial to inferior epigastric

wall medial to inferior epi~~1Strilt artcrr alte-ry =Dired hClTlkl

GOtgtS through the external (Hlperfidal ~ Lateral to inferior epigastriC ingnilltl1 ling only Covered by transversalis MtelT =Indired he micentl

fascia Usually in older men

Lymphatics - Please dont get Testes and Scrotum mixed up

o Testes Lymph follows blood supply through inguinal canal back to abdominal aorta to

lumbar and preaortic lymph nodes Metastasis of Testicular Cancer - Livestrong

o Scrotum Lymph (As do lymphatics from all ectodermally derived structures in this

region) drain to superficial inguinal lymph nodes

GI blood supply and innervation Emhryonic Artery Pltlrasytnpathdilt VCTt-cbral Structnr(s suVP li-lt gut region inIlervation level

For(ut Cdi ~1 C Sgl5 T12Ll St)(fIl(b to pmxtll1li dll(llteJlllJl1middot lin-f gallbladder IMlliHa

Ll Dist1I dwXlentll11 Jo proximll -f of lra n ~Hrc colo n

I~lA Di~t~lll lgt of tmmt rsl cokm to upp-r portion [f fthllll _pknic Ilexuf is a wtlkrshtd ngiol1

Heart - Gastric and

duodenal regions

Cenae artery

~IY-~ Primordiuln of liver

Superior llli3Senteric artery 10 mido~n

I Interior mesenteric Hhidgut artecrj

Celia( trunk Bran ches of cd ial( [rnnK (ornm011 ]lmiddotpltic splc- nicleft gastric lhclt -ornpri~t tile main blood supply of the toma(h

Short Ilstric han pC gtOf

1lIltlSlo nlOc if ~pl ~ llic nrkry

is bloch-d Strollt anasbmOHS ltxist

betwe tn

-Left and right ga gtir(jt~plpJoj (s

-Left Lind right gastriCs

KNOW YOUR ANASTAMOSES

IIhlieunci

~plecn to JID$tEfipr i pleric artc~

Biliary structures

BE ABLE TO DRAW A FLOW CHART OF BILE FLOW

Important GI Iigaments

I

COnnect t

Ucr lcl n l)le rid~ abdominal i LlnmeJl1mh teresshy wall 1

I

if o(tal i ri~l c) he~atlc ~l rhry_ l1ay be olnptessed -1 Prin1 vein emiddotolllmn be~ween tlmmb 111d

t blle dud indd fin~d Pticed in cpk ploic farunen nfWinslow hJ ~ntt 1l1Jeecling

t

6n~e tHtre~te(Urid I crIt

middotbullbullmiddot -os

trHr to 1 liSef clltUliue middot1 Gastric m~eri~ 1 n ter and~epIt1tes ghtgrel

of stom~cll I les~rsac _~b be qt 9Brtng urger 1

~ i to ac(~3S le~(r sa

Grcater luC ltinind Ifa1t ofgreater 1)1I1tmiddotntllln

[ran fr colon 1

Lcreater curv ep~r1tCs left gre~br and t $pI eil middot I lesscr bllt$

qI

Splenorenal linil Vd~l 1 bull bull abdaininal JltaU

___o-J-___ ~~~~

~~----- ~-- ~~- ---~

Be able to visualize these ligaments know their embryonic origins and contents

Recognize relationship of structures in epiploic foramen (Portal Triad)

Splenorenal is high yield

Know which ligaments contribute to the greater and lesser momentum

Clinical- If the stomach ruptures posteriorly - contents are confined to the lesser sac but if

the stomach ruptures anteriorly - contents in the greater sac can spread as far down as the

pelvis

Know the relationship of structures within the portal triad - Hepatic Vein will be the trick

answer choice

L sgt-J[ offiEntumHpatic artY proPll

8iloduct bull

Stomach

Glstrapllmic ligament

Ornootnl-+----+r--___-----cT-- ~Jrarn n

IH---V~tal

petitOllllU-m

Splllllofffial figamgn

TXH

LIVER - ClinicalDryWet

Coronary Iigameot

Ewph3g~al

Impression

Lett tria09ular Hepatic veins

ligament

Fibrougt Supralerfal impreurossionappendix of

tiOer Hoparorenal portion ot coronary ligament

Garuic RighitrianQularl igameot impresgtion

(Common) bile duct

Common hepalic duel

Clstic duel

Renal Impression Caudate lobe

Papiilary proc~ss Quadrat~ lobe

allbladder

___ FalCltltorm ligament

Hepatic artery PlOP - Round Jigam~nt 01 liver

Hepatic portal veio ~-- Coli c impression

FisUH for ligamen1vm teres

Porta hepatis

Make sure know

Embryonic origin of the iigaments

Anatomical vs Functional lobes and relationship to the Gallbladder

Portal Triad Structures (VH DISSECTOR)

Impressions

CLINICALS -

Trac heoesophageal Almocml COll 11ction htteell esophalIs ll nd tmlthe1 fistula Jost lI)OUllul1 StLblypt is hlind up tr () hngtls with lOler esophagus tO)HHcted to

lrac hea RcsulJ in (3 11 $($ (ho tng anc vo mitin with ftt(~~lS lir bllblllpoundQll CXR and )Qlvl V -~-

I ~ i I Esophageal y i J atresia - - Trachea

----shy

Z

Congenital pyloric l1l perlrnphy (J f le pylurus camt~ obstruti I) Pa Ipablc uhve mass in epigl~tric n~ion stenosis and nOl1biliolts PfOiL mnit+rrg- - - wetk of ltl4C Tn)tment is suriltai ineisi n

Ottmi in 160(llit births ofteIl in I ~ t-bOfl malts --

Peptk ulcer djsease G~ltri( ulcer Pain can he greater vdth mellls--Welghtl - ft(H OCClIlS in older putient

a Mlori tnfedion in 70 chronic NSAID use also implicated Dm to llHue-oml protection a~lllns t gfltrh l id

Duodella) ulcer Pain Deu(QSe5 wilh menh--weighl ~aln_ Almost IOO have H priori infection Dut to 1~ gastfk acid cerct 11 f( 1 2011 inpmiddotr-Ell lson )~md rol11t i ( r L m(lcos~ ll

protction Hyprttophy of Bntnllcr glands Tend to ha ve ltIeall pullcheJ-ouf margim unlike the tais(dJirre~lllar margins of

cnCinOl1lH P-te1itiai eomplications include bl eedin~ penetrnu(n into pnnerelS perfomtion and obs tmctiQH (not intrinsltllly pr((JIlcerom) (see Image 11-+

Question - A 57 year old obese chronic alcoholic presents with an ulcer which has ruptured the cI~wnpastelgrly Surgical investigation reveals blood in the peritoneum Which of shy

the following arteries is most likely responsible for the bleeding

a Splenic Artery

b Gastroduodenal Artery

c R Gastric Artery

d L Gastric Artery

e L Gastroepiploic Artery

Appendicitis All age ~rol1ps most common indication for em er(nl ilxkminai)lJrger) in ehildrefL huLial diffmc periumbilical pain --7 localized pain JtMcBmm~ point N~ltlSC~l kvef

r)13)- perfor~te - pcntouitis Difftr~ntiI1 divertieuli[i (elderly ectopic pr(~gnanl to [3-hCG 10 rule ont)o

-----

PcmiddotrhtltllCf of the ) lcllille dud Of ~Olk sh31k ~br lontli n e( topic ~l cid-seefehllg gatric m 1(0]

andor pmcrcatic ii~~lle Lllost common c(lllgenital anomliy aftlle CT tract Cm CllHt hkfciing illtusm~(~pjjon Dlnllus or nbstrudion nelr the tcrrnin)l ileum Contraslwitll QIB~efic nmiddot = cvtk dilalaUon of ittllilC dJet

------~-

The ile 2$

2Jpound~11 11~ 2 feet frolll the iie( middoteCill vke l~ QfiJ~ at 1~

CIllIn nly prcsenfltgt III rll~2

llf~ of lifc- by ilwe 2 ty )ts of

epilheH8 19ls-trie- pal1elli(i

Hirschsplungs disease

Congenital tnt91(middotolon characterized by lack of Think of a gian ~pring that ganliml ttlLJcnJ(rk ~~gJeXllS~( (~lihs and lei ~sner plCxpstTlrlsgIllent on inbstinal biop y ()It to-iIure of U(middotural t~restpoundtU migration =

has s nl

Presents as ronic comtipnHoll tHly in life Dilllted pOltioll of the colon proximal 10 the aganiionic

segment resulting in a middot trmsjtillll ZQl1t rnvolt~

rectum [huany farlur( to P~$ meconium

High Yield WetDrylTheory List-

Suggestion - in your study group try to write a question for each of these points and then

exchange with a friend and try to answer each others questions

Abdomen Blood Supply - Reference viks picture posted on my google group - this is THE MOST HIGH

YIELD TOPIC IN ALL OF ANATOMY - expect 5 questions on your mini and 5-10 questions on your shelf

Make sure you can draw the blood supply and answer tertiary questions

Example - If the patient had an occlusion of the celiac trunk - which of the following areas would

experience ischemia

Portal hypertension - Know the 3 clinically relevant sites of portal caval anastamoses

Testes vs Scrotum lymph drainage

Where are paraumbillicai veins located

Omalophcele - failure of the gutto come back in (if in yolk sac - fatal)

Marginal Arteries

Superior Messenteric Artery is in front of 3d part of duodenum

Caput medusa (Distended paraumbillical veins secondary to portal hypertension)

Kidney Constrictors -1 Renal Pelvis 2 Crossing Pelvic Brim 3 Entering Urinary Bladder

Ureter - wet lab

Vagus is PIIJS supply up to 23 trans colon than pelvic splanch N up to the ass

Hirshsprungs disease - baby cant poop - dilated colon

Meckels Diverticulum - rule of 2s - 2 feet prox from ileocecal ju nc

Urachal Fistula - weeping belly

Gall stones - common bile duct

Jaundice relation to tumor of the head of the pancreas

Hepatopancreatic ampulla

Blood supply of renal gland - s superrenal art m s suprarenal a abd aorta inf suprarenal art

R Kidney - Tl2-L3 L Kidney - Tl1-L2

Renal Artery - L2

Epiploic foramen - know the borders and contents

Alantois diverticulum - urachus - medial umbilical

Lateral Medial and Median umbillical Folds (know the contents)

Directindirect hernia - know how to diagnose where they enter and exit the inguinal region and which

one is congenital

Anular pancreas - projectile vomiting

Pyloric Stenosis - projectile vomiting (non bilous)

Duodenal Atresia - projectile vomiting (bilous)

Dry Lab - know x rays

Vitteline Fistula - food out of umbilicus

Major duodenal papilla - junc of foregutmidgut

Arcuate line - relationship to rectus sheath

Mcburneys point -13 from ASIS bw umbilicus

Parietal pain - what is the nerve supply

Internal Oblique - cremasteric relationship

Know spermatic fasia

Processes Vaginalis - connection bw peritoneum and gubernaculums

Umbillicus - TlO dermatome

Deep Inguinal-l25 cm above mid inguinal ligament

Superficial Inguinal Ligament- superolateral to pubic symphysis

Variocele - veins engorged in scrotum (bag of worms)

bull

bull External spermatic fascia derived from external obliques EO II Cremasteric fascia ~ from internal obliques fO bull Internal spermatic fascia derived from fascia transversalis bull Tunica vaginalis derived from processes vaginalis directly rests on testes bull know order from testes out to skin

note reflex o ilioinguinal nerve o Efferent =genital branch of the genitofemoral nerve

--lt gt-- info important anastamoses which connects thorax to abdomen

bull Sup

o Sup epigastric branch of internal thoracic o Inf branch external iliac

Venous drainage o Above umbilicus aXillary v o Below umbilicus veins in triangle o At level of umbilicus Paraumbilical veins -gt drain into the portal V

II Important in Portal Caval Venous system Venous drainage of testes

o Clinical correlation Varicocele 11 vein drains into IVC 11 Left testicular vein ~ drains into left renal v

bag of rmlt

for lymph drainage T10 axillary lymph nodes

ill Below T10 superficial inguinal lymph nodes (lateral

Umbilical Folds

Lateral umbilical folds inferior vessels

Medial umbillcial folds umbilical (fetal remnant)

Median umbilical fold urachus (fetal remnant)

Between these folds fossas o Supervesical fossa between median and medial folds

11 bladder o hesselbachs between medial and I folds

II DIRECT HERNIAS HERE Borders

Medial semilunar line

Lateral info Epigastric

Inferior inguinallig o Lateral Inguinal Fossa beyond lateral fold

INDIRECT HERNIAS HERE II Deep inguinal ring (lateral to inferior epigastric a)

Indirect inguinal hernia o Lateral to inferior epigastric a o more common o When inserting finger in superficial inguinal ring will feel on tip of finger (since it goes

throueh ineuinal canall

----

Dry Lab - Label subcostal iliohypogastric Ll Ilioinguinal (Ll)

Horesshoe Kidney - stuck under IMA

Renal Agenesis -failure of the ureter bud to develop

Double Ureter

Unilateral Agenesis -1 kidney

Kidneys - Metanephros

Fetal kidneys are at sacral level

Look at 3rd part of duodenum

Some of this stuff is repeated I know just copied and pasted a bunch of stuff I had copy

Dermatomes

bull T4 nipples

bull no umbilicus v o Pain referred to no in appendicitis o Pain referred to T7ITS in gastritis ~

Inguinal ligament = external abdominal oblique aponeurosis

bull Inserts at anterior superior iliac spine to the pubic tubercle o Why important to know -7 visualizing this line allows us to properly diagnose a hernia

Below the inguinallig femoral hernia Above the inguinallig =inguinal hernia

Also to palpate the deep inguinal ring you go about 12Scm above the mid-inguinal

point

bull Modifications to ligament o Pectinate ligament o Lacunar ligament -7 cut this ligament to relieve strain i~ stran ul~tEd hernia

Inguinal canal

bull in males -7 transmits spermatic cord o important structures of spermatic cord ductus deferens testicular a genital branch of

the genitofemoral n pampiniform plexus of veins bull in females -7 transmits round ligament

Borders

bull Floor -7 inguinal ligament + lacunar ligament bull Anterior -7 aponeurosis of external oblique + internal oblique bull Roof -7 internal oblique and traverse abdominal bull Posterior -7 transverse abdominal + transversalis fascia

o Reinforced by conjoint tendon bull Aponeurosis of internal abdominal obliques and transverse abdominus bull Lies immediately behind the superficial inguinal ring in what would otherwise be

a weak point in the abdominal wall bull Innervated by ilioinguinal nerve (Ll) ~why important

bull In appendicitis Ll can be injured which will injure this nerve and in turn

the conjoint tendon With loss of innervation to this supportive structure the patient is now predisposed to a direct inguinal hernia

o Only hernia that can transverse the inguinal canal o Associated with congenital condition persistent tunica vaginalis

bull Direct inguinal hernia o Medial to inferior epigastric a o When inserting finger in superficial inguinal ring will feel on back of finger o Associated w old age or recent surgery

Muscles (Abdomen RECTUS SHEATH)

bull Arcuate line at level of ASISor 13rd distance between pubis and umbilicus bull Above arcuate line rectus abdominus is surrounded by a rectus sheath anteriorly and

posteriorly

o EO and 10 lie over rectus abdominus o 10 and TA lie behind rectus abdominus

bull Below arcuate line rectus abdominus has no rectus sheath posteriorly o EO 10 and TA lie over rectus abdominus o Transversalis fascia lies behind rectus abdominus o Inf EpIgastric vessels pierces the rectus sheath here

Peritoneum serous sac which encloses most of the abdominal structures

bull Ovary =only intraperitoneal organ o Oocyte ejected from ovary then captured by fallopian tubes o Why impt Women more prone to infection that can enter peritoneum

Peritoneum forms

bull Mesentery double layered fold of peritoneum formed as the organ was pulled in

bull Ligament between 2 organs in general bull Omentum between stomach and another organ bull Bare area area of no peritoneum bull

Viscera innervation

bull Pa rasympathetics 11 o Afferents sense hunger o Efferents l peristalsis relaxes sphincters gland secretion

bull Sympathetics o Efferents do opposite o Afferents CARRY PAIN SENSATION OF THE VISCERA (dull stretching pain)

bull PARASYMPATHETIC INNERVATION o Vagus nerve 7 _1l to 23rd unct ion of la rgej nte~tine oJ)elVrcspla~~~)~~ic~rYe~ IJiU- ~rd aJ~lpoteotiD~ IMPT

Gut Embryology

Gut ~ We say that the gut is derived from endoderm We often forget that when we say so we mean

that only the mucosa is derived from the endoderm The submucosa and the muscle layer is actually derived from the splanchnopleuric mesoderm and the serosa is derived from the visceral peritoneum

~ The main function of the gut is to digest the food which is done by the glands derived (and are) in the mucosa (endoderm) The only two exceptions in the Gut where glands though derived from the endoderm do not stay there but migrate down into the submucosa are esophagus and duodenum These glands however have their ducts opening to the swface of the mucosa

bull

~ Lungs liver amp gall bladder and pancreas are off-shoots from the foregut Esophagusshy~ The region of the tube from the laryngeal diverticulum to the beginning of the stomach elongates

to form the esophagus ~ The glands which form in the endoderm (mucosa) migrate down into the submucosa The path

whlch it took migrate becomes the duct of the glands which open to the mucosa ~ Achalasia Cardia - Failure of relaxation of the lower esophageal sphincter because of congenital

absence of ganglia at the sphincter (The ganglia when present releases VIP (Vaso-IntestinalshyPeptide) which relaxes the sphincter)

Mid-Gut Rotation ~ Because of the 90 degree rotation of the primitive stomach all of the following events occur ~ Lesser curvature comes to the right Therefore lesser omentum also comes to the right ~ Greater curvature comes to the left Therefore greater omentum also comes to the left ~ Right side vagal trunk becomes posterior vagal trunk ~ Left side vagal trunk becomes anterior vagal trunk ~ The left side peritoneal cavity comes to the anterior aspect of the stomach and will later be called

as the greater sac ~ The right side peritoneal cavity comes to the posterior aspect of the stomach and is (relatively a

small sac because the liver is on the right) called the lesser sacomental bursaepiploic sac ~ Epiploic foramen of Winslow (the lower free margin of the ventral mesentry) wiII be the

communication between the greater and lesser sac ~ The Liver moves to the right and therefore actually causes the 90 degree rotation of the stomach

The spleen comes to lie on the left side ~ Axis Antero-posterior axis around the superior mesenteric artery

bull Counterclockwise bull Approximately 270deg bull During herniation (about 90deg) bull During return (remaining 180deg)

Duodenum ~ Becomes retroperitoneal (except the first part which is still suspended by the hepato-duodenal

part of lesser omentum) ~ Glands (of Brunner) go submucosal ~ An imaginary line drawn below the opening of the major duodenal papilla represents the junction

between the foregut and midgut ~ Duodenal atresia in Downs syndrome Liver ~ 3rd week

bull liver bud grow bull into the septum bull transversum

~ 10th week bull hematopoietic bull function

bull 10 of the total bull body weight

~ 12th week bull bile is formed

Pancreas ~ In about 10 of cases the duct system fails to fuse and the original double system persists ~ 3rd month

bull pancreatic islets (Langerhans) ~ 5th month

bull Insulin secretion ~ Annular pancreas

bull The right portiCn of the ventralbud migrates along its normal route but the left migrates in the opposite direction

~ Complete obstruction of duodenum ~ Accessory pancreatic tissue Polyhydramnios (Amniotic fluidgt 1500-2000 ml)

~ Congenital defects including central nervous system disorders (eg Anencephaly) and gastrointestinal defects (atresias ego Duodenal esophageal) prevent the infant from swallowing the amniotic fluid (failure of recanalization)

Oligohydramnios (Amniotic fluid lt 400 mt) ~ Cl~ldberenal-agenesis

bull Midgut _-_

~ Primary Midgut intestinal loop gives rise to bull Distal duodenum bull Jejunum bull Ileum bull Ascending colon bull Transverse colon - proximal two-thirds of the bull Transverse colon with the distal third

~ Primary intestinaltoop bull ncephalic limb distal part of the duodenum the jejunum and part of the ileum bull ncaudal limb lower portion of the ileum the cecum the appendix the ascending colon and

the proximal two-thirds of the transverse colon bull 6th week

bull Rapid elongation of the cephalic limb bull Rapid growth of the liver bull Intestinal loops enter the extraembryonic cavity in the umbilical cord

bull 10th week bull loops begin to return bull regression of the mesonephric kidney reduced growth of the liver expansion of the

abdominal cavity bull Jejunum -left bull Loops - more to the right

bull Cecal bud -last part (temporarily below the right lobe of the liver) ~ qIDlthaloseJe (Structures COlHLoArts9V~1tion)

bull Through umbilical ring bull 6th to 10th weeks

bull Associated with a high rate of mortality (25) and severe malformations bull Associated with chromosome abnormalities

~ Gastroschisis (Structures coming out are not covered by Amnion) bull herniation through the body wall ----=---=-shybull Into the amniotic cavity bull Lateral right of the umbilicus bull Sometimes the inferior wall fails to develop as a result lower abdominal structures like the

bladder would be exposed to the exterior not associated with chromosome abnormalities ~ Abnormalities of the Mesenteries

bull Mobile cecum persistence of mesocolon bull Extreme form - long mesentery bull Volvulus

~ Distal third of the transverse colon ~ Descending colon ~ Sigmoid colon ~ Rectum ~ Upper part of the anal canal ~ Primitive anorectal canal

bull 7th week cloacal membrane ruptures bull Tip of the urorectal septum perineal body bull Pectinate line

~ Hindgut anamolies bull Rectoanal atresias and fistulas bull Imperforate anus bull Congenital megacolon (aganglionic megacolon Hirschsprung disease)

bull

bull Hindgut

Chirags Abdomen Review - Part 2

Understanding Embryo makes learning blood supply EASY

I I

I t

~ -

)

Table l1r-~ L Adult SUmiddotuctu~SDrj~l Froln Each of he Three Dhisions of be Pringttive GUl Tube t-middot-----middotmiddotmiddotmiddot-

Foregu(

I_ (Celiac Trunk)

Ir-slt-gtphgus

S101na(b

I h -= LiJ~r

Pancre=l S

bull 1 i Biliary apparntu5

Gall bladdshy

i Pha11~Cal pltgtuchcs

LullSS-I

Mjig ---- bull __ _- ----n--duct----~---middot-------l--n

(Superior Jldesen1eric Artery)_-1I-(I_~__ middot __ O-=-)_in_middoto_r_M_e_se_n_t_e_r_i_c_An__

Uuodenu rn 2nd_ 3 lt141h V4Tt

Jejunun-~

nc-un]

tCCUJ11

AppltgtndLX

Transver5e -o1on (p~oxiln1l1 ~O Tbird)

bull__hytgtid~ _ _ ______ L _ __

Tr-dn~llt~se colon (diStul h lTd) I

)

i

Aa ca-nal -( uppeT patt) i

I I

_____ __ _ _ _ ___ - - rhe~ a(t clcriVOkt iV(5 opound~lt prbn1rC ~ nlQC blft TlI)( 134tof r~ tIonoinf~ i 1 ~l l1rd c- P Cle

Now Lets see how much youve learned

Questions

1) A pt receives a general anesthetic in preparation for a c~t~~my A right subcostal incision is made which begins near the xyphoid process runs along and immediately beneath the costal margin to an anterior axillary line and transects the rectus abdominus muscle and rectus sheath At the level of the transpyloric plane the anterior wall of the

-~~-~=--- _eco---shysheath of the rectus abdominus muscle receives contributions from which of the following

a Aponeuroses of the in~ande~tef-Ilal o~ues

b Aponeuroses of the transversus abdominis and internal oblique muscles c Aponeuroses of the transversus abdominis and internal and external oblique

muscles d Transversalis fascia e Transversalis fascia and aponeurosis of the transversus abdominus muscle

A

2) The lat~raJJJ11QjJt~gLfgJlLoneach side of the inner surface of the anterior abdominal wall is created by which of the following structures

K Falx inguinalis (~) Inferior epigastric a

c Lateral border of the rectus sheath d Obliterated umbilical a e Urachus

B

3) A man the victim of several knife wounds to the abdomen during a brawl at the Lobster Shack subsequently developed a direct inguinal hernY Damage to which of the following nerves is most likely responsible for the predisposing weakness of the abdominal wall

~ Genitofemoral nerve ( b) Ilioinguinal nerve ~-t Tenth intercostal nerve

d Subcostal nerve e Pelvic splanchnic nerve

B

4) Which of the following statements concerning a direct inguinal hernia is correct a It is the most common type of abdominal hernia b It transverses the entire length of the inguinal canal c It contains all3 fascia layers of the spermatic cord d It exits the inguinal canal via the superficial ingeJinal ring e It protrudes through H~acb strJg e

~(

1fltbS w E

tl

5) The conjoint tendon is

a Important in preventing indirect inguinal hernias b The fused aponeurotic layers of internal abdominal oblique and transversus

abdominus muscles c Posterior to the deep inguinal ring

d Medial fibers of the inguinal ligament

B

6) A 25 year old male is brought in to the ER after being involved in a car accident in which he received a crushed internal injury in his abdomen Examination reveals a lesion of parasympathetic fibers in the vagJsnerve which interferes with glandular secretory or

smooth muscle functions in which of the foliowingorgans a Bladder b Transverse coloiW c Descending colOO d Prostrate gland e Rectum

B

7) The spermatic cord includes all of the following contents except a Il ioinguinal nerve b Pampin iform plexus of veins c Vas deferens d Genitofemoral nerve

A

8 Which abdominal structure gives rise to the internal spermatic fascia (muscle) following the descent of testes in development

a External abdominal oblique aponeurosis b Transversalis fascia c Transversus abdominis muscle d Peritoneum e Internal abdominal oblique

B

9 Which abdominal structure gives rise to the tunica vaginalis fotlowing the descent of testes during development shy

a External abdominal oblique aponeurosis b Transversalis fascia c Transversus abdominis muscle d Peritoneum e Internal abdominal oblique

D

10) The lesser omentum is a peritoneal fold which is su bdivided into the a Hepatogastric and gastrosplenic ligaments b Hepatoduodenal and gastroomentalligaments c Hepatoduodenal and gastrosplenic ligaments d Hepatogastric and hepatoduoden9-jrj igaments

D

11) A posteriorly perforating ulcer in the pyloric antrum of the stomach is most likely to produce initiallocalized peritonitis or abcess formation in which ofthS fQllowing

a Great-sac - -- -

b Paracolic recess

c Omental bursa

d Right subphrenic space

c

The inferior mesenteric artery arises from the abdominal aorta ilm_ediill~y_J-Qs1eriQLto which of the foowing org~ns A-F~t~filie duodenum B Head of the pan~eis C Neck of the pandeas

D Second part of the duodenum

E Third part of the duooenum_shylaquoshy

shy

The correct answer is E The inferior mesenteric artery arises from the anterior surface of the aorta at the level of the third lumbar vertebra The third part of the duodenum crosses the midline at the level of the third lumbar vertebra and passes anterior to the aorta at the origin of the inferior mesenteric artery The

first part of the duodenum (choice A) lies horizontally to the right of the midline at the level of the first

lumbar vertebra The head of the pancreas (choice B) is to the right of the midline and extends from the

level of the first lumbar vertebra to the third lumbar vertebra It lies within the concavity of the

duodenum The neck of the pancreas (choice C) lies in the midline at the level of the first lumbar

vertebra It lies on the anterior surface of the aorta at the origin of the superior mesenteric artery The second part of the duodenum (choice D) lies vertically to the right of the midline and extends from the

level of the first lumbar vertebra to the level of the third lumbar vertebra

The left adrenaLvein drains directly into which of the following veins A Hemiazygos vein

B Inferior vena cavaee C Left renal veiri -

D Splenic vein

E Superior mesenteric vein

a

The correct answer is C The left adrenal vein and the left gonadal vein (either testicular or ovarian) drain into the left renal vein TheTeft renal vein t~ains intothe- inferior vena cava In contrast the right

adrenal ~~inandnght gonadal veindrai~ gLr~ctJy iQtoJhe iilferiQ[ Vencava -- -

ThehemTazygoS7ein- (~h-~i-~ A)~~c~i~es the venous drainage from the body wall on the left side of the

thorax and abdomen No visceral organs drain directly to the azygos or hemiazygos veins The inferior vena cava (choice B) receives the direct venous drainage from the right adrenal vein but not

the left adrenal vein Remember the inferior vena cava is on the right side of the abdomen The splenic

vein (choice D) receives the venous drainage from the spleen and part of the pancreas and stomach The splenic vein is part of the portal venous system

The superior mesenteric vein (choice E) receives venous drainage from much of the intestinal tract It is part of the portal venous system and joins with the splenic vein to form the portal vein

A 43-year-old man presents complaining of pain in the groin On examination his physician palpates a

bulge in the region of the superficial inguinal ring which he diagnoses as a direct inguinal hernia The hernial sac most likely

A is covered by all three layers of the spennatic fascia B passes medial to the inferior epi gastric artery

C passes medial to the lateral border of the rectus abdominis muscle

D passes posterior to the inguinal ligament E passes through the deep inguinal ring

The correct answer is B Direct inguinal hernias enter the inguinal canal by tearing through the posterior

wall of that structure The typical location for this type of hernia is through the inguinal triangle bounded

laterally by the inferior epigastric artery medially by the lateral border of the rectus abdominis and

inferiorly by the inguinal ligament Direct inguinal hernias pass medial to the inferior epigastric artery

whereas indirect inguinal hernias pass lateral to the inferior epigastric artery because the deep inguinal

ring is lateral to the artery Indirect inguinal hernias are covered by all three layers of the spermatic fascia (choice A) Direct inguinal hernias are covered by fewer than all three layers because the direct inguinal

hernia tears through one or more layers of fascia as it emerges though the abdominal wall The lateral

border of the rectus abdominis muscle (choice C) forms the medial border of the inguinal triangle All

inguinal hernias pass lateral to the rectus abdominis Femoral hernias pass posterior to the inguinal ligament (choice D) Inguinal hernias emerge through the superficial inguinal ring which is superior to the inguinal ligament Inguinal hernias that descend below the inguinal ligament pass anterior to the

ligament Indirect inguinal hernias pass through the deep inguinal ring (choice H) direct inguinal hernias

do not Both types of inguinal hernias pass through the superficial inguinal ring

During a gastric resection in a patient with stomach cancer a surgeon wants to remove the lesser

omentum because of tumor extension into it Which of the following structures lie in the free edge of the

l~~g omentum and consequently must be dissected out in order to be preserved

A Common bile duct cystic duct and hepatic artery 6

B Cystic duct hepatic artery and hepatic vein

e Hepatic vein and cystic duct

Portal vein common bile duct and hepatic artery

E Portal vein hepatic artery and hepatic vein

The correct answer is D The free edge of the lesser omentum contains three important structures the

common bile duct the hepatic artery and the portal vein Nei ther the cystic duct (choices A B and C) nor the hepatic vein (choices B C and E) lies in the free

edge of the lesser omentum

A 55-year-old male patient with chronic liver disease has portal hypertension To relieve the pressure in the portal system a porto-caval shunt is performed Which of the following veins may by anastomosed to

accomplish this porto-caval shunt A Left renal vein-left testicular veingt

B Right renal vein-right suprarenal vein I shy

e Splenic vein -left renal vein J

D Superior mesenteric vein-inferior mesenteric vein E Superior mesenteric vein-splenic vein

The correct answer is C The splenic vein drains directly into the portal vein The left renal vein drains

directly into the inferior vena cava Anastomosis of these veins would allow blood from the portal vein to

drain retrograde though the splenic vein into the renal vein and then into the inferior vena cava The left

renal vein (choice A) drains directly into the inferior vena cava The left testicular vein drains directly into

the left renal vein Thus these veins are already in communication and neither vein is part of the portal venous system The right renal vein (choice B) drains directly into the inferior vena cava The right

suprarenal vein also drains directly into the inferior vena cava Thus neither vein is part of the portal

venous system The superior mesenteric vein (choice D) drains directly into the portal vein The inferior

mesenteric vein drains into the splenic vein which then drains into the portal vein Thus neither vein is

part of the caval venous system The superior mesenteric vein (choice E) drains directly into the portal

vein The splenic vein also drains directly into the portal vein Thus neither vein is part of the caval

venous system

A 12 year old boy has fever vomiting and para-umbilical pain After examining the patient the doctor

makes an initial diagnosis of appendicitis Appendicular pain which is initially referred to the umbilicus goes to the dorsal root ganglion of

a TI b TI2 c L1 d T7

(e I TIO

A 59-year-old male undergoes a neurological examination which reveals that when the abdominal wall is

stroked the muscles of the abdominal wall of the side of the body stimulated failed to contract Other

neurological tests appeared normal The likely region affected includes

a CI - C5 spinal segments b C6 - TI c T2-TI ~T8-T12

e Ll- L5

The surgery done to relive portal hypertension is done by connecting two veins Which of the following veins would be suitable for connection

a Inferior vena cava and portal vein b Superior vena cava and portal vein c Splenic vein and right renal vein d Splenic vein and left renal vein e Superior mesenteric vein and Inferior vena cava

A mother brings her 3-week-old infant to the pediatric clinic reporting a new scrotal bulge that she found -~-

while changing a diaper yesterday The infant is afebrile Physical examination reveals a palpable mass in

the scrotum while in the standing position resolution of the mass in the supine position and no

transillumination of the scrotal sac What is the most likely diagnOSiS

a Cryptorchidism b Direct inguinal hernia c Hydrocele d Indirect inguinal hernia ~ e varicocele

The Vagal trunks enter the abdomen by passing through which of the following openings in the

diaphragm

a Right crus b Esophageal hiatus ~ c Vena caval hiatus d Aortic hiatus e Left crus

2 The anterior boundary of the epiploic foramen of Winslow is bounded by

a) First part of duodenum b) Lesser curvature of stomach c) Liver d) Hepato-duodenalligament v ~

3 The ilio-inguinal nerve is derived from

a TI2 ry b LI c L2 d L3 e L23

15 Surgically the structure used to suspend the kidney to the diaphragm is

a) Renal fascia b) True capsule c) Perinephric fat d) Paranephric fat

6 If there is portal obstruction because of carcinoma affecting the pancreas which of these of the

following signs would be present

a Caput medusae b Esophageal varices c Rectal varices c

d Pulmonary edema

7 In a sliding hernia the gastro-esophageal junction lies

a) At its normal position b) Below the normal position c) Above the normal position V d) None of the above

8 Which of the following structures is retroperi toneal

A transverse colon B spleen IJ2f6 C ileum D descending colon v r 1pound1111111

9 The renal angle is fonned lgtetween the 12th rib and ______ muscle

a Psoas major -middotshyb Erector spinae c Quadratus Iumborum d Diaphragm

10 The anterior structure at the hilum of the kidney is

a) Renal vein ~

b) Renal artery I middot~ I

c) Ureter d) Accessory renal artery

11 Because of origin of the muscle from the lateral one third of the inguinal ligament it

could not fonn the anterior wall of the inguinal ligament

a) External oblique b) Internal oblique c) Transversus abdominis_ d) Rectus abdominis

12 A large tumor mass impinges on the splenic artery and its branches as the artery pass out from below

the greater curvature of the stomach Branches o(which of the following arteries would most likely to

effected by the pressure on the splenic artery

a Left gastric b Left gastro-epipJoic c Right gastric d Right gastro-epipoloic e Short gastric_

13 A new born baby has projectile vomiting after each feeding It is determined that there is obstruction

of the digestive tract as a result of annular pancreas Annular pancreas is as a result of an abnormality in which of the following process

a Rotation of the dorsal pancreatic bud around the first part of duodenum b Rotation of the dorsal pancreatic bud around the second part of duodenum c Rotation of the dorsal pancreatic bud around the third part of duodenum d Rotation of the ventral pancreatic bud around the first part of duodenum y Rotation of the ventral pancreatic bud around the second part of duodenum

14 As the liver bud enters the ventral mesogastrium the region of the mesogastrium stretching from the

liver to the anterior abdominal wall is called

a Lesser Omentum b Greater Omentum ~ Falcifrom ligament d Lacunar ligament e Ligamentum teres of liver

16 A patient has absence of his 12th rib In such a patient if the doctor makes an incision to approach his

kidney mistaking the 11 th rib for the 12t he would end up injuring

Which of the following arteries is a direct branch of the gastroduodenal artery The

A right gastric artery

B left gastric artery

C inferior pancreaticoduodenal artery D left gastroepiploic artery

i E)right gastroepiploic artery --

E x pI a nation The right gastric artery is typically a branch of the proper hepatic artery The left gastric artery is a direct

branch of the celiac trunk The right and left gastric arteries anastomose along the lesser curvature of the

stomach The inferior pancreaticoduodenal artery is a branch of the superior mesenteric artery it

anastomoses with the superior pancreaticoduodenal in the head of the pancreas The left gastroepiploic

artery is a branch of the splenic artery it anastomoses with the right gastroepiploic artery along the greater

curvature of the stomach The right gastroepiploic artery is a branch of the gastroduodenal artery The

other branch of the gastroduodenal artery is the superior pancreaticoduodenal artery

Which of the following pairs of veins join together to form the portal vein The

A superior mesenteric vein and inferior mesenteric vein

B inferior mesenteric vein and splenic vein

C superior mesenteric vein and splenic vein

Ip)splenic vein and left gastric vein E superior mesenteric vein and left gastric vein

Explanation

The portal vein is formed behind the neck of the pancreas by the union of the superior mesenteric vein

and the splenic vein The inferior mesenteric vein drains into the splenic vein The left gastric vein drains

directly into the portal vein After the portal vein forms it enters the hepatoduodenalligament of the

lesser omentum to reach the liver The portal vein is the most posterior structure in the hepatoduodenal

ligament

At which of the following vertebral levels does the duodenum pass anterior to the aorta - _- shy

All ~

B L2 7~

CL3 I

~DL4

E L5

Explanation

The duodenum begins at the pyloric sphincter at the level of Ll The second (or descending) portion of

the duodenum is to the right of the aorta and extends inferiorly from the level of Ll to the level of L3 The third part of the duodenum crosses the aorta from the right side to the left side at the level of L3 The

fourth (ascending) portion of the duodenum extends from the level of LJ to the level of L2 The

duodenum ends at the duodenojejunal flexure The superior mesenteric artery passes anterior to the

duodenum as the duodenum passes anterior to the aorta The duodenum can be constricted at this level

In which of the following locations will perforation of the digestive tract result in the spilling of luminal

contents into the - lesser peritoneal sac

A Anterior wall of the second portion of the duodenum B Posterior wall of the second portion of the duodenum

C Anterior wall of the stomach

~Posterior wall of the stomach E Posterior wall of the transverse colon

Explanation

The posterior wall of the stomach is related to the lesser peritoneal sac The anterior wall of the stomach is related to the greater peritoneal sac The anterior wall of the second portion of the duodenum is related to the greater peritoneal sac The posterior wall of the second portion of the duodenum is related to the retroperitoneal space The posterior wall of the transverse colon is related to the greater peritoneal sac

The ureter lies against the anterior surface of which of the following muscles shyA Crus oftne diaphragm B Quadratus lumborum

0 Psoas major D Transversus abdominis

E Iliacus

Explanation The ureter exits the renal pelvis at about the level of vertebra L2 As it descends along the posterior abdominal wall it lies on the anterior surface of the psoas major The psoas major muscle arises from the bodies of the lower lumbar vertebrae The psoas major muscle is joined by the iliacus to fonn the

iliopsoas muscle The iliopsoas muscle then attaches to the lesser trochanter of the femur and is the major

flexor of the hip

As the right ureter passes the pelvic brim it lies against the anterior surface of which of the following

blood vessels

A Gonadal artery B Inferiorvena cava C Internal iliac artery

rJ- External Iliac artery

E Inferior mesenteric artery

Explanation

The ureter lies in the extraperitoneal space in the posterior abdominal wall Alter leaving the kidney it

passes inferiorly on the anterior surface of the psoas major muscle At the pelvic brim the ureter passes

into the pelvis At this point the common iliac artery is dividing into the external and iliac arteries The

ureter lies on the anterior surface of the external iliac artery immediately distal to the bifurcation This is a useful landmark for a surgeon to locate the ureter

When extravasated urine passes from the superficial perineal space into the anterior abdominal wall it is

found immediately deep to which of the following layers of the anterior abdominal wall

-ltScarpas fascia

B External oblique muscle

C Internal oblique muscle D Transversus abdominis muscle

E Transversalis fascia

Explanation

The superficial perineal space is bound by Colles fascia the fibrous portion of the superficial fascia This

layer of fascia is continuous with Scarpas fascia the fibrous portion of the superficial fascia of the anterior abdominal wall Therefore urine that is deep to Colles fascia will remain deep to Scarpa s fascia The urine will spread in the plane between Scarpas fascia and the external oblique layer

When a horseshoe kidney develops the ascent of the kidney is restricted by the A internal iliac artery B external Iliac artery

C common iliac artery

inferior mesenteric artery

E superior mesenteric artery

Explanation

A horseshoe kidney develops when the inferior poles of the to kidneys fuse together as they ascend into

the abdomen from the pelvis The first anterior midline vessel that is encountered by the horseshoe kidney

is the inferior mesenteric artery This artery prevents the kidney from continuing its ascent

The left testicular vein drains into which of the following veins

A Left internal iliac vein B Left common iliac vein

bflnferior vena cava D Left renal vein I

E Left internal pudendal vein

Explanation

The left testicular vein drains into the left renal vein The right testicular ~i~[~nsltjectlY into the

inferior vena cava This difference in venous drainage is believed to explain the greater incidence of

varicocele on the left side than on the right The venous drainage from the penis is to the internal vein

which then drains into the internal Iliac vein

The spinal nerve that provides cutaneous branches to the skin around the umbilicus is

A TS B TW-shy

C TI2

DL2 EtA

Explanation

The tenth intercostal nerve is the anterior ramus of the TIO spinal nerve After passing through the tenth

intercostal space the nerve continues forward in the anterolateral abdominal wall in the plane between

the internal oblique muscle and the transversus abdominis muscle In the abdominal wall the nerve innervates to the abdominal wall muscles as well as the skin and the parietal peritoneum The umbilicus is

a useful landmark for the region of distribution of the tenth thoracic nerve

The ligament of the vertebral column that resists its extension is the Aligamentum flavum

B supraspinous ligament

C posterior longitudinal ligament

D anterior longitudinal ligament

E interspinous ligament

Explanation

The ligaments of the vertebral column that resist flexion of the column include the supraspinous ligament

interspinous ligament ligamentum fiavum and posterior longitudinal ligament The ligament that resists

extension is the anterior longitudinal ligament This longitudinal ligament is very broad and strong It

covers the anterior and anterolateral surfaces of the vertebral bodies and the intervertebral disks In

addition to resisting extension the anterior longitudinal ligament provides reinforcement to the anterior

and anterolateral surfaces of the intervertebral disk The posterior longitudinal ligament is relatively

narrow and covers the posterior surface of the vertebral bodies and the intervertebral disks This ligament

reinforces the posterior surface of the disk The posterolateral surface of the disk is not reinforced and it

is through this region that herniation of the nucleus pulposus usually occurs

A patient presents with epigastric and right upper quadrant pain The pain is most intense 2-4 hours after

eating and is reduced by the ingestion of antacids The patient states that he has passed black tarry stools

(melena) within the last week Fiberoptic endoscopy reveals a yellowish crater surrounded by a rim of

erythema that is 3 cm distal to the pylorus Accordingly an ulcer has been identified in the patients

A fundus

B antrum

C duodenum

D jejunum

E ileum

A number of physiologic genetic and other factors increase the risk of gastric (and duodenal) peptic

ulcers The evidence that H pylori plays a principle role is compelling Smoking and caffeine are known to adversely affect the morbidity mortality and healing rates of peptic ulcers In general first-degree

relatives of peptic ulcer patients as well as males have a threefold to fourfold increased risk of developing this disorder Paradoxically in gastric ulcer disease acid secretion is not elevated It is possible that

excess secreted hydrogen ion is reabsorbed across the injured gastric mucosa In general a defect in gastric mucosal defense is the more important local physiologic

A patient presents with symptoms of duodenal obstruction caused by an annular pancreas Annular pancreas is caused by

A rotation of the dorsal pancreatic bud into the ventral mesentery B rotation of the ventral pancreatic bud into the dorsal mesentery

fJ failure of the major and minor pancreatic ducts to fuse ~ ~ cleavage of the ventral pancreatic bud and rotation of the two portions in opposite directions around -the duodenum E formation of one pancreatic bud instead of two

Explanation Normally the ventral pancreatic bud rotates around the gut tube to reach the dorsal pancreatic bud The two buds fuse to form a single pancreas and the distal portions of the two ducts fuse The ventral pancreatic bud forms the inferior portion of the head of the pancreas the uncinate process and the major pancreatic duct (of Wirsung) The dorsal pancreatic bud forms the superior part of the head the neck body and tail and the minor pancreatic duct (of Santorini) Annular pancreas is the result of the ventral pancreatic bud dividing into two portions before it rotates into the dorsal mesentery Each portion rotates in opposite directions to get to the dorsal mesentery thus encircling the duodenum The presence of annular pancreas can constrict the duodenum thus obstructing its lumen

In n _ phranlc----

Gon ~l ----_1 Lum bltano

~~--- CornmQ1t bull ac

+-~4--- lnlllirnaJ ilic

xtem iliac

OBJECTIVE - Identify the blood supply to each of the structures listed in the table on the previous page

Ill give you a head start

FOREGUT - Supplied bV Celiac Tru nk (T12)

Proper hepatic

GastiooUod 13Jafter

1nferlor pancreaticoduodenal artery

Common epatlc

Lett gas ric iiirtery

Spfen artery

shy Gastroepiphgtic artery

~ Superior mesenteric 8rtfry

~

1 Esophagus is a derivative of the foregut so its blood supply originates from the celiac trunk

(T12) The predominant blood supply to abdominal portion of the esophagus is the Esophageal

A (Branch of L Gastric) The venous drainage of the esophagus is particularly important because

it is 1 of 3 clinically relevant sites of Portal Caval anastamoses The Portal Esophageal Vein

meets the Caval Azygos System Persistent bleeding manifests as Esophageal Varices - a fata I

condition

2 The Stomach is also a derivative of the foregut has EXTENSIVE blood supply and is very high

yield on anatomy exams The lesser curvature is supplied superiorly by the L Gastric A (1 of 3

major branches ofthe Celiac trunk) and inferiorly by the R Gastric A ( a branch ofthe proper

Hepatic A) The greater curvature is supplied superiorly by the L Gastroepiploic A (a major

branch of the splenic A) and inferiorly by the R Gastroepiploic A

The Short Gastric arteries (branches of Splenic Artery) supply the fundus of the stomach and

are referred to as EIID ARTERIES because they have no collateral blood supply Therefore if the

splenic artery were occluded (ex - increased pressure in the ommental bursa) - there would be

ischemia to the fundus of the stomach Venous drainage of the stomach is extensive via various

veins lead ing to the portal system Posterior to the stomach the IMV joins the splenic V which

joins the SMV to form the PORTAL VEIN ADAMS

3 Duodenum blood supply has high clinical relevance because it is the junction of the foregut and

midgut and therefore is the site of anastamoses between branches ofthe Celiac Trunk (main

foregut artery) and the Superior Messenteric Artery (main midgut artery) The Proper hepatic

artery gives off the gastroduodenal artery which travels behind the 1st part of the duodenum

This point has high clin ical relevance because duodenal ulcers are very common and a posterior

rupture of the 1st part of the duodenum could rupture the gastroduodenal artery causing

traumatic abdominal bleeding The Gastroduodenal artery first gives off the R Gastroepiploic A

(mentioned above) and proceeds as the Superior pancreatico duodenal artery (supplies the

pancreas and duodenum) which anastamoses with the inferior pancreatico duodenal A (branch

of the SMA) This is the junction of foregut and midgut and occurs near the opening of the

bil iary system into the duodenum (ampula of vater) Portal venous drainage here is responsible

for delivering nutrients from digestion to the liver for metabolism Appreciate that the Superior

mesenteric artery (artery of the midgut) branches from the aorta at Ll travels posterior to the

pancreas than moves anteriorly (at the jxn of the pancreatic headbody) and comes over the

3rd4th part of the duodenum Tumor of the head of the pancreas can compress the SMA

4 Jiver blood supply is via the common hepatic artery (major branch of the cel iac trunk) The

common hepatiC becomes the proper hepatic gives off the R gastric A and the Gastroduodenal

A and then joins the common bile duct and the portal vein in the portal triad Clinical- if a

patient were bleeding from the hepatic A a surgeon can stick his fingers in the epiplOic foramen

and squeeze the free edge of the hepatoduodenalligament in order to stop bleeding to the

area Please note that the hepatic a branches into Rand L hepatic A The Right hepatic artery

gives off the cystic artery which supplies the gallbladder Afferent venous supply is via the

Portal vein which is bringing nutrient rich blood to the liver After metabolism takes place

venous blood leaves the liver through the hepatic veins into the IVC PLEASE UNDERSTAND THE

RELATIONSHIP OF THESE STRUCTURES - ADAMSNETIERSNH Etc

5 Pancreas - Head is supplied via the superior and inferior pancreaticoduodenal arteries

(mentioned above) The tail (situated towards the hilum of the spleen) is supplied via the

pancreatic branches of the splenic artery (END ARTERIES) This blood supply is very important

because the endocrine Alpha and Beta Cells from the pancreatic islets of lagerhans are located

towards the tail This is where Insulin and Glucagon is released to the blood

Now complete this for mid and hindgut structures Make sure to note clinically relevant arterial

anastomoses as well as portal caval anastomoses FYI Appendix blood supply SMA + IMA

anastamoses marginal artery Portalcaval rectal veins fhemmorhoids) and periumbilical caput

medusa are high yield THE BUTT THE GUT and THE CAPUT

Abdominal Development

Liver

Ij1f

II wall b

oh liN ~ VltJrti n be- bull

Pancreas

Secondary Retroperitonealization e I~tl r 1 a v-mtrai m ellter

Rotations of the Gut I i Ij (lIl1UtIJ f~ l r tilt

()l td 10 me l-ft and he v

--~--- -~ -~-~

i

I AolaijonjoI~guf I

STOMACH BED (IDENTIFY IN ADAMS)- the structures posterior to the ommental bursa which

support the stomach in the supine position

Abdomnal JQrUI

Splnic vein

OmQ-oul tv~ ) O(s(Jroa)

Lojt(r o m nturrt (hpJtodu o d~n31 Hid

Gadrl)SplerH (g3stroll~nal) IIgam~nt

hiad h~~atogrtricent IIQdmiddotcrt~)

Lt Dome of Diaphragm (why left Look this up in Adams)

Spleen (What is the blood supply)

Left Kidney (What is the blood supply - AND how is it different from the R kidney)

Suprarenal Gland (What is the Arterial AND Venous Blood supply - how are they different)

Pancreas (How does supply differ from Head to Tail What is the SMA Relationship)

Transverse Mesocolon

liver - ADAMSWET - Make sure you look at the liver in wet lab

Left triangular nl1am~nt

ComoaDj ligamnt

Erophg~1 impre$ioo

Hepatio veins

In1erior -ifena middotr3)Ia

Fibrous appendix o-t

live

impr~j on

Heprorendl p~rtion of Q)(Qllary ligament

Righllri~n9ul r 1I~met

(Common) bile quol

Gr)mmCtr~ hepatic dlJct

Ccentic duct

Duodenal impression

GaJdate p-fr)~S

Hepatic artgtrl prop-f iiiiila - Faloiform ligament

_ - shy Round ligamen liver

~--F-- CoJio imprgt-ssi-on

Prta heptis

Identify the lobes impressions and embryonic remnants associated with the liver

Caudate Lobe Quadrate Lobe Right Lobe Left Lobe Round ligament Falciform Ligament

Ligamentum Venosum (what is its fxn in embryonic life) Hepatic Veins (NOT PART OF THE

PORTAL TRIAD) IVC PORTAL TRIAD - Contents relationship cross section etc Know the

Galbladder relationship to the lobes of the liver

Biliary Duct System - Make sure you understand the sequence of these structures - BE ABLE TO

DRAW A FLOW CHART

TPVd i

t

I t

1 __ Cm-(r

patk GlJet

I

J

Clinical = JAUNDICE is caused by anything that prevents delivery of bile to intestine Tumor of the

head of the pancreas Stones etc Patient will have pale stools and yellowish colored mucus

membranes

Clinical- Any scenario that tells you the patient has BILLOUS VOMIT means that the obstruction to

the flow of digestive contents is after the Ampulla of Vater (Site of Entry of Billiary system to the

duodenum) - ie Duodenal Atresia

Spleen -located posterior to the mid axillary line between ribs 9 and 11 Make sure you know that

the 10th rib is the main axis of the spleen and this organ is susceptible to injury (stab wound errant

thoracoce ntesis etc)

The spleen is derived from mesodermal cells - NOT THE GUT TUBE

The spleen rests on the left colic flexure associates with the tail of the pancreas Know the

structures entering the Hilum of the spleen

Sh rt O~-t~ic 1 0(0 10 rtiltSPIric Iloa nt

(cut)

Peritoneum - similar concept to Pleura - think of a fist in a balloon

Visceral Peritoneum - Layer of balloon touching your fist

Parietal Peritoneum - Layer of balloon not touching your fist

Your fist represents the organ your wrist is the hilum and your arm contains the blood supply

entering the organ

Appreciate that there will never be organs in the peritoneal cavity - rather these organs invaginate

the cavity Kaplan videos

RULES OF NOMENCLATUREshy

1 Organ completely surrounded by peritoneum - peritoneal organ

2 Organ partially surrounded by peritoneum- Retroperitoneal

3 Peritoneum surrounding peritoneal organ is VISCERAL peritoneum

4 Peritoneum surrounding retroperitoneal organ is PARIETAL peritoneum

5 Peritoneum connecting visceral to parietal is called messentary 2 messentaries in the

gut Dorsal (to the gut tube) and ventral (to the gut tube) messentary

Aorta is in Retro peritoneal position - but blood must reach peritoneal position - vessels travel through

messentary All peritoneal organs will have blood supply reaching through messentary

-Mesentery is a 2 layer peritoneum with a neurovascular communication between body wall and organ

- Ligament connects one organ with another or to the abdominal wall (Ommentum = ligament)

lesser Ommentum (attach lesser curvature of stomach and duodenum to liver) =Hepatoduodenal

Ligament and Hepatogastric Ligament

Has a Superior and Inferior Recess (Accumulation of Fluid in Ascites)

Communicates with the greater sac through the epiplic foramen (what structures pass through

this foramen)

Boundaries - you must be able to visualize this

o Anterior - stomach

o Posterior - parietal peritoneum pancreas

o Superior - superior recess (bw diaphragm and coronary ligament)

o Inferior -Inferior recess (bw layers or greater momentum

Greater Ommentum (attach greater curvature of stomach) Gastrophrenic ligament Gastrosplenic

ligament gastrocolic ligament

The greater omentum is the largest peritoneal fold It consists of a double sheet of peritoneum folded on itself so that it is made up of four layers The two layers which descend from the greater curvature of the stomach and commencement of the duodenum pass in front of the small intestines sometimes as low down as the pelvis they then turn upon themselves and ascend again as far as the transverse colon where they separate and enclose that part of the intestine

ABDOMINAL PAIN

Parietal Peritoneum - supplied by same vasculature lymphatics and nerves supplying body wall it

lines and diaphragm Sensitive to pain pressure heat cold well localized

Visceral Peritoneum - supplied by same vasculature lymphatics and somatic nerve of organ it covers

Insensitive to touch heat cold and laceration - referred to dermatome of spinal ganglia providing

sensory fibers Where does appendicitis refer to

Foregut pain - epigastric area (ie - cholycystitis)

Midgut pain - periumbilical area (ie - appendicitis)

Hindgut Pain - suprapubic area (ie - diverticulitis)

Extra ImagesConcepts

ll~_____-

FalifCtrm ligament oind r~ud ligamet f Ilver

Blood from splenio gastriC and inferiof rne$e-rteri v~ins

Ca-I tributaries

Lett gastrio Ifein

Posterior superior pan~reatioodul)denal vaihS

Lott gamo-om~nlal (9aropip lomiddotic) -in

Poq_~ tjol imerl-9-r panCJertlcorllJod-nal veiopound --amp----I- - ~J Right grtr~-omntal

Anwrior interi (gartroepiploic) Jjn

pan euaii cod vl)denal veins middot Inf~Ji (t r mesentric vein

Miqdle (olic vein

Right cl)licvein Sigmoid and rectosigml)id (ei ns

IhH)Collc(~io

--- Mi~dl laquooLJl gtjrltgt

PoM ca vl1 illasto)moses -----shyampoptoageal 2 Paraumbilie-lt11 Inferi or Fectal vei ns

3 Recial 4 REuoperHonea1

Know how the Portal vein is formed I 4 sites of portal caval anastamoses and 1 clinical shunt

Col li t ltt-~ otTl~tI ~nj pc~ 1lt1 turJoG

Ltf 14i1 tImiddot~ artoftl9 on tj phtAt$

L-oftqf 4t t~r 1=laquoIran d 1 bull shy~p l ci rj o fOOOts

Nerves follow the arteries - appreciate the splanchnic nervous system I

Uet~ric branch of left ~nal art

Ureterie branch of righi renal artelY

Left Zld lumbar in and co mlTlunication to as)erdin9 lumbar l(~in Hi ~ht tEZ1~~t~ t3r j t itn ~ nJ l1t- rlnd lfe i r1

Inferior me5nteri~ artery

Notice that the right testicular vein drains directly into the IVC and the right testicular artery drains

directly into the aorta However the left testicular vein drains into the L renal vein at a right angleshy

reason left testicle is lower and more susceptible to varicocele (bag of worms)

Also notice that the left renal vein has a longer course because the IVC is on the right side whereas

the right renal artery has a longer course because the aorta is on the left side

Appreciate the anterior to posterior relationship of structures in the hilum of the kidney - VAP - Vein

Artery Renal Pelvis (Ureter)

11____ __ L_ L_ n VJ __ _ _ t_L I I_ _ L __ L_ I -pound1 bull LI_~-I ____

Posterior View of Head of Pancreas in ( of Duodenum

Celiao hunk

Co mmon ~L~jJth art~ry

GastNduQdonal artrf (partilly in phantn)

P1)Sterior $Up~Jior panCflaticuduodfmal art~r~t

(Co mm on) bile duct

middot~1t~~t-1l---~-~- Right gshomiddotomental (gastoe plp lolc) 3rte (phantomost)

Grener paocre atic art-ry

1n1~rjor pancr-iatlc artery

Jtrifll supejo r pal)oreailcento)dJodenal artr1 (phantom)

Anastomotlo branch

POostetlor bJanch of jo f~ri of pan-reatir(lduodensl drttnj

Anterio r branch of i flferior palcreati~)duodenal art~(phan1om)

Notice the extensive blood supply to the pancreas and duodenum via the branches of the celiac trunk

Notice collateral supply from SMA branches - makes sense bc this is the jxn of foregutmidgut

Identify the vessels in this arteriogram

Hiltid i)f N~ck oi B)dvof Tail 01 pa nereas pan cent~as P-nmiddot-reas panCtCas

I nferie v~na cava

jHept1iic p(lrlai v~in

Port1 tnd H~pti lt a ftH prol

Comm on) bll duct

Ouodtnum

~ft colic (sio)Atta~ hmtrlt jt~xJr-ofha~elSe

muo(IIQn

Right ~lIc (h~j)tic)

il~gtture

In1triol m~oten lIein (rttr op~ritoMdO

SlJp efl or mes~n~fiC amrV and lipln

KNOW YOUR NEIGHBORHOOD

Questions

vVhiJh structure supplied by a bnmdlof the cclia( artery is not derivcd from foregut LemCJUCrITI

(A) Head of the pancte-a5

CD) Pyloric duolenum

Cystkduct

( Liver hepatocyt~~

~F) Body of the spleen

An infant presents with an omrhaJucele at birth -hi oJ the [oHm illg applies to his cM1-dition

(A) It is 31so seen ill p4titnts with aganghonic megacolon

(11) ft reuirs from a fal1ure of resorption of theviteUine d let

(C) It results from herniation at the-site of regression of the right umbilk vein

DJ It is caustd by faihtrc of recanalization of the midgut part of the duodenum

~ It ill camioo by a failuIt vf the midgul to return to the abGQminal uity after herniashytion in-n the urnbilk s l stalk

Ot er than the spleen occlusion Cif the spit-Ilk artery at its odgin wm most likely affect die blood supply to jllch st cnud

(A) Jejunum

(B) Body of th pal1~lltas

(C) LeSStT Cllmiddotlaturc of tl )toma-ch

(D Duodenum dista to the entrance of the Ornmou bile duct

E Fundus of the stomach

A 38-yeu-old batL~er with a history of heartburn suddenly experiences excluciating pain in the (plgastric region of th~ abdomeu SurgCry is perf~rme immediard y upon admisshysion to the 1IlcrgCJliy tuomh~re i~ evidence uf a ruptured ulcer in the posterior waU of the stomach Vhere will a surgeon first fi nd the stomach contenlSf

A) Greater p4ritoneal sac

rB) Cul~de-s~c of Douglas (--

C Omental bursa ~

--D) Paracolic gutter

rEj Between -he panttal perimltum and the posterior body wal1

At birth an infant presents with a st()ma~ rb~tbas~njJled jfltotb~diaplfagru 1A1ltre is the defect thatresulied iiitJle heini~t()n shy~tsophagealbiatus

7 - rH-- Hiatus for the inferior vena cava

( Pleuroperitoneal membrane -(0) Septum transvcrsum

(E) Right Crlt~

An infant born with DOVv7l syndrome presents with bili()u~ vomiting Ahat congenital defect does the infant have

(A) Pyloric stenosis

(B) Meckel diverticulum C) Ornphaloce1e

(D) Gastroschisis

( ~ ) Duodenal atresia y A patient with cirrhosis of the liver presents with ~ bacalvaricestnlreased retrograde pressure in which veins caused the varices

(A) Paraumuilical

(B) Splenic

(ct AzygltJus

(15))G~trk ( (-F) Superior mesemeric

A htaltby 3-year~old male patient experiences a hernial sa protruding from the anterior abdominal wall about halfway between me anterior superior ilia spine and the pubk tuberde Pulsations of al1 artery are palpated medial to the protrusion site through the abdominal walL Which layer of the anterior abdominal wall will first be traversed by the

1hctma

fA) Rectus sheath (B) External oblique aponeurosis

(C) Inguinal ligament

lD) Transversalis fusda

(E) Cremasteric fa~cia

After 5urgi(aj ffpair of a hernia the patient tXperienccs mtmlgtness in the skin on the anteshyrior aspect of the S(Totum_ Vhaf nerve may have been lesioned during thehemiorrhaphy

(A) Femoral

(B) Obturator

(C) Ilioinguinal

(D) lliohypogastrk

(E) Pudendal

A 23~year-LJld female secretary il1 good health ~-uddcn1) doubles over with pain in the a ea of the 1JmbRicu$ Sbe feels vartn and ltneasy and has no appetite That night the pain seems to have mQved to the tower right abdominal regjol1 and she calls her family doctor who then arranges for an ambulance to pk-k her up and take her to the hospitaL Wh ell ntn~ perceived in the area of the urnbilirus most Hkely carried lhe pairfu I sensations into the eNS

tA) Vagus nerves I~

V B)

) Lessersplanchnk nerves

tC) Pudendal nerves

(D) lIiohpogastrk nerves

(E) Greater splam ic l erves

A CT reveals carcinoma in the bOod of the ancreas Vhich blood vessel trut ourses ----~- - -bull ------ --shy

immediately poftterior to the body ofthe pancreas is the m~t likely to be oompressed

(A) Splenk artery

(B) Abdominal aorta (C) Portal vein

(1) Splenic vein

(E) Renal vein

A patient has a penrln1l1ng uker of the posterior wall ot the br~l part ot the (lUooenmn llkh blood vessel is subject to erosion

(A) Common hepatic artery

(B) Gastroouodenal artery

(C) Proper hevatic artery

(D) Celiac artery

(E) Anterior inferior 11amrelltlcoduodcnal attery

Your patient has been diagnosed -ith a carcinoma locallted to the head and l~e(k of the pancreas Another clinical sign would be

A esophageal varices

(8) hemorrhoids

C) a caput medusa

(D) increased pra Teuro n th~ hepatic veins

(E) enlarged right supra lavkular lymph nodes

Wltkh of the foUowing structures develops in the ventral mesentery

(A) Spleen

(B) Jeiunum (C) Head of1ht pancreas (D) Transverse colon (E) Stomach

ti l Uw ~ littwin~ f( S-t lil oai Imdge ~ hi(h or tbt la~)d J truetur tgt liJ llntn nl) he hl p UC iJd [IIi ell

c o

A) drains Ie tht infCrior a La aI

R t middot~nfl0 ~ill to th~ lunlgtn of h i dtlndCrlllfH

(e) m t bull JiJattd on tl l J n T ~H

D ) sup Lc O VSlt I Hlid bhtu l 1 li - -I un oid

( ) U~tpli(t tr j middottUh~ 1 v(( b~nt rfK n1ilc~Zm

ANSWERS AND EXPLANATIONS

Answer E The spleen is t hlttnopodicand lymph organ demlted from mesoderm

Answ~ R Al1 tlmphalocele is caused by it failure of the nlidgut to return to the ahdomir nat cavity after herniation into the umbiliau Stalk Choices Aand D maybe seen in infants with Down syndrome choice D ~s the specific CBuse ofduudcnal JtiCSitt Choice C is (ile cause of gclstrosbisis and Choice B nsults iu a Meurolktldivertku1-tlB

Answer B The fundus ofthe stomach is suppHed by soort gastric brunches of the splenic altery The splenic artery supplies the body and tail of the pancreas part of the greater curvature of the sttmla(h and the spleen Te jejunum part of the head of the pancreas and tht~ duodenum distal to the entrance of the commOll bile duct are supplied by the superior mesenterk artery clll~l ~be less r ctlt1ature cmd the pylQric antrum are supplied by the right and lei gastric art(ries

AnSWftt C Tbeomental bursa or lesser ~ritoneaj sac lies direcdy posterior to the proxshyimal part of the duodeTtlm and the stomach and would be the first site where stomach contents ~Ott1d be fpoundluncL

Answer C A defect in a llleuropcritoneal membrane (uswlly the left) is the typical site of i1 cc-ngenitlI diilphragluatic hemia llere the membr4ne fails to dose ()pound( of the perishycCirdiopcritulleal canals

Answer E DuoJenal atresia and aganglionic megacoion are congwitaI defects S~Il in patients with Dowmiddotnsyndrome

Answer D RulaTgemt~llt of and retrograde flow in g~lstrk vel_ns in particlJl~r the kft gas~ tricveins dilates the capillary bed in rhe wall of the esophagus in (ases of porta yper~

tension Blood flow would increase in and dilampte tribntarkgts of the (lZygOUS vein on the other side of the capiUary bed but flow in this vein is in the typical direction t()ward the superior vena cava Paraumbiii(ltU vein eilgorgement contributes to a caput medusH Splenic ~nlargement might prc~nt with 5plcnonlegaly and balt-kflow in to tlu superior m~~ntclic vein occurs but is asymptomatic

Answer D The patient hagt an indirect inguinal hernia whi~h emerges from the antt-rior abdominal wall through the deep inguinltilling Theeep ring is a fault in the transv~rshysaUs fascia this I~yer wiIJ be penetrated first by the hernia

An~Wer C The ilioinguinal nenc which provides sens~llion to the lnedlal thigh ltmclanteshytior SClotunl pass~lt th rough the 5uperfh_ial inguinal ring ind $subject to inj i1T) becaus-e

it is in the operatitm Held of the erniorrhapny

Auswer B The leMHr splanchnic nerves are sympathdic nerVlts that carry viscera l sensashytlltgtrogt ftom illtllt1m~d ()J stietched gust (itinteitinal ~tructures (in this case the pprndix) into tnt eNS Lesser splanchnic ntTYcsarisc from thmiddot T9--T12 spinal cord segments lt1nd provide sympathetic innenation tD rnidgut siruc1ures whiCh include CLe app~JldD Viscera] Pain arising from affecLed Inidgut ampt 1C1ure is referred over the same dl- matorne~ of spinal segrnertts v-hich provide the sympathetic Innervation n this G1SC of appendicitis the invohen~n t of the ltire) of t e unlhHku indud s the T 10 dermatome

Answer B Of the five choices onty the dscending olon is retroperiton~al aldwould be a lik ~ ( choice to be seen immediately a(~jilcent to t11e posterior abdominal middotn~L

Amwen D The SpltftlC ~-ein ourses posterior to the body of the panneas m its way tt drain into the superior mCSfttltlri( vein

Answcr B TILt glstrodllolticnal artery 1 direct hIamh of the comrootl hepatic artery courses immediately pt))iwri() to the duodenum and is slbject to erosion

Answer B Carcinoma of th pan middott3S in the 1tilt1 may compreampgt the portltil vein at irs orishygill The poTtai vcin is fomled when the splenic vein jQiaswith tfie superior meStllt eric vein The inferiot mesenteric vein joins the ~plenjc vein just priOT to tlli~ point at which the splenic joins the superior Jlleit1ltcri( vein Increescd venous presslu in the inferior mesenteric vein is a cause of emo hoid~

Answer C The- velltral pancreas wilich forms most of the head of the p ~ncr as develops in the ventral mes(ntery as antutgrowth of the hepatic diverticulum Th~ hepatic divershyticulull induding the biIJary appa~atus develops in tbe ventral mesentery of the foregut

Answer~ A The superior mesenteric ~in joins with the spienkvein to form the hepatic portal vciu

Answer D The structure at gttlK is the proper hepatic artery~ whkh suppUesoxygenated b middotood to the liver

MAKE SURE YOU KNOW the diff bw Rectus Sheath above and below the arcuate line

ABOVE

Aponeurosis of xiiltmal obllque musclo

Extemll f)biquw musde

Reotln ilbdomlnls musole S~in

Internal 9bliquQ mY~QI

AponeUfOsi$ of hJH$V~~S Lir9a a lb lbdolTlin~ musolo Tri OJV6 rUi

atldomlnis mUS(loe

Sub cutanlilous tiue (tatty ye r)

BElOW

A POrl lJfosis 01 etemal oblique muscle

Aponeul~)sis 01 Internal oblique mU$cl~

Anteriol lay~ of r~ltdus st~ath EXttom1 oblique rnu$cll

Rectus Jbdominis muscle Intoernal Aponeurc-sis of tra~fersU$ oblique muscle-

at-domlnis muscentl ~ Skio

Tra nsvitSus abdomioLs ml)ZClt

TralSVersaHs fascia Medial umQil iegtt1 1i9Jment -and folj

Uldchus Peritoneum (ir median Umbilj~al Suboutane ous

Extraprftone 11ascia

Ymbilimiddot~1 fold)

preu9poundiea1 fascia

tissue (fatty 4nd m~mbr3n(iUS layers)

o Above the arcuate line (A horizontal line 13 of the distance bw the umbilicus and the

pubic symphysis) -10 Aponeurosis divides into an AntPost Laminae

o The Ant Laminae joins EO and Post Laminae joins Trans Abdominis = Ant and Post

RECTUS SHEATH respectively

o BElOW the arcuate line - all 3 aponeurosis join ANTERIOR to rectus muscle to meet its

counterpart in the midline (linea Alba)

o Take away Msg - The abdomen is devoid of a posterior rectus sheath below the

arcuate line and is therefore more vulnerable to herniasinjuries

Question - A physician makes a deep incision in the patients midline immediately superior to

the pubic symphysis which of the following layers is his knife least likely to pass

Rectus Abdominis External Oblique Ant Rectus Sheath Posterior Rectus Sheath All of the

Above

Answer - All of the above None of the other answer choices are midline structures -LINEA

ALBA

Linea Alba has very poor blood supply - doesnt heal well after surgery Therefore this is a

common site for incisional hernias

a Spleen b Transverse colon c Descending colon d Stomach e Pleura

17 Meckels diverticulum is normally found 2 feet proximal from the

a Pyloric sphincter b Lower esophageal sphincter c Ileo-cecal valve d Middle valve of Huston e Anal valve

18 Ulcer in the posterior wall of the first part of the duodenum would erode ___ artery and would cause bleeding

a Left gastric b Right gastric c Hepatic artery proper d Gastroduodenal artery e Middle colic artery

19 An inflamed appendix is identified by a surgeon on the operation table by noting

a The appendicies epiploicae b The convergence of tenia c The artery of Drummond d The mesocolon e The mesosalphinx

20 The nerve which emerges through the psoas major is

a Femoral b Ilio-inguinal c Ilio-hypogastric d Pudendal e Subcostal

21 The right gonadal vein drains into the

a Azygos b Hemiazygos c Inferior Vena Cava d Right renal vein e Left renal vein

22 The hepatocytes in the liver is derived from

a Ectoderm b Endoderm c Mesoderm

d Neural ectoderm

23 Abscess in the lumbar vertebrae due to tuberculosis would spread to the adjacent muscle which is

a Psoas Major b Iliacus c Quadratus lumborum d Tranversus Abdominis

24 The anterior wall of the inguinal canal is formed by

a External oblique and transverses abdominis b External oblique and fascia transversalis c Internal oblique and external oblique d Internal oblique and transverses abdominis e Fascia transversalis and peritoneum

Meckels diverticulum is a result of which of the following developmental abnormalities shy

A Failure of the vitelline duct to close

B Failure of the herniated intestinal loop to retract into the abdomen

C Failure of the urachus to close

D Failure of the midgut to rotate

E Failure of the hepatic duct to close

Explanation

Meckels diverticulum is a result of the persistence of the proximal part of the vitelline duct This

diverticulum is usually found about 2 feet proximal to the ileocecal junction and is usually about 2 inches

long It is present in about 2 of the popUlation It may be the site of ectopic pancreatic tissue or gastric

mucosa and may develop inflammatory processes and ulcerations Acute Meckels diverticulitis

simulates appendicitis

Which of the following veins carries blood from the esophagus to the portal vein The

A right gastric vein

B left gastric vein c splenic vein D azygos vein

E left gastroepiploic vein

Explanation

The left gastric vein a direct branch of the portal vein drains blood from the lesser curvature of the

stomach and the inferior portion of the esophagus Because branches of the portal vein do not have

valves blood can flow in a retrograde path when there is an obstruction to flow through the portal system or liveL Rlooci Cln then flow from the nortl] vein thr()1Ph the left PRstric vein to the esonhlPlIS lno

through venous communications within the submucosa of the esophagus to esophageal veins that drain

into the azygos vein The increase in blood flow through the esophageal submucosal veins results in esophageal varices

On the posterior wall of the abdomen the celiac ganglion A contains cell bodies of postganglionic parasympathetic neurons B is synapsed upon by neurons in the posterior vagal trunk C is synapsed upon by neurons in the greater splanchnic nerve D contains sensory cell bodies of lumbar spinal nerves E contains cell bodies of neurons that cause an increase in the rate of peristasis

Explanation The celiac ganglion is one of the preaortic ganglia of the sympathetic nervous system It contains cell bodies of postganglionic sympathetic neurons The sympathetic splanchnic nerves contain preganglionic sympathetic neurons that pass through the sympathetic chain without synapsing These splanchnic nerves go to the preaortic ganglia to synapse The greater splanchnic nerve contains preganglionic neurons from spinal cord segments T5-T9 This nerve synapses in the celiac ganglion The nerve fibers in the vagal trunks are preganglionic parasympathetic fibers that go to the walls of the organs that they will innervate and synapse on postganglionic parasympathetic neurons in the walls of those organs Cell bodies of sensory neurons in the abdomen are found in the dorsal root ganglia or the sensory ganglia of the vagus nerve Sympathetic innervation decreases the rate of peristalsis parasympathetic innervation increases the rate of peristalsis

Which of the following pairs of arteries will allow blood to bypass an occlusion of the celiac trunk

A Left gastric artery-right gastric artery

B Left gastroepiploic artery-right gastroepiploic artery

C Superior pancreaticoduodenal artery-inferior pancreaticoduodenal artery

D Splenic artery-common hepatic artery

E Left gastric artery - proper hepatic artery

Explanation The anastoOlosis of a branch of the celiac trunk and a branch of the superior mesenteric artery will

provide collateral circulation around an occlusion of the celiac trunk Each of the other choices pair

branches of the celiac trunk therefore these will not provide collateral flow around the obstruction of the

celiac trunk The left gastric splenic and common hepatic arteries are direct branches of the celiac trunk

The right gastric artery is a branch of the proper hepatic artery which is a branch of the common hepatic artery The left gastroepiploic artery is a branch of the splenic artery The right gastroepiploic artery is a

branch of the gastroduodenal artery whlch is a branch of the common hepatic artery

Which of the following organs has appendices epiploica The

A sigmoid colon

Bjejunum

C duodenum

D stomach E esophagus

Explanation Appendices epiploica are characteristic of the colon Appendices epiploica are subserosal accumulations

of fat None of the organs of the gastrointestinal tract has appendices epiploica except the colon

Page 3: Chirag's Abdomen Review

portosystemiC anastomoses AV-azygollS vein

EV--esophageal vein lEV-inferior epigastric vein 1M V--int-erior mesenteric vein IRV~~nferior rectal vein IVe-interiQr vena cava LGV--Ieurolt gastric vein PlN--praumhiliCal vein PV--portal vein RV- renal vein SEV- -sliperior epi9astric vein SMV-superior mesentaficveln SRV-SlJperior rectal vein SV--splenic vein

bull Portal venous o SysternicvJnOus

Site- ()f tnlstolTIogtis Clinical sign Portal H systemic 1 Eophagus EsophHgto31 middot lti(e~ Lft gaslric H (-opblgt~ J 1 Umbilicus C1Pllt medmae PlnHUllbilicol H mperlicilI

1 11d inferior cpiglstricshyIkmormoios Snp( ri of re( tal H middle

and inferior redal

-ilfi cf~ pf8ut butt and caput M( commonly $(efl with portll hyperknsiun fli$trtin~] porlo(Iv11 sb unt d i llve(middotn the plenil anJ le ft renal v(ins relieves portJJ

hypertcnlti(n bygtlllH1ting blood to thtgt ~stemic cire-ulation

Make sure you know

How the Portal Vein is formed

Based on the site of occlusion - which Portal Hypertension clinical sign will present

Portocaval Shunt - Clinically Portal vein directly to IVC is not likely - Splenic to left Renal V

3

INGUINAL LigamentCanalHernias Inguinal canal

In fetal life the gonads descend from the posterior abdominal wall In the case of males the

testes must descend all the way outside of the body in order to maintain an optimal

temperature for spermatogenesis to be maintained

What you must know about the inguinal region

Boundaries

o Ant - Ext Oblique Aponeurosis

o Post - Transversalis Fascia amp Int Obliq (Form a conjoint tendon - attached Pub Symph)

o Roof - Aponeurosis of Int Obliq amp Trans Abd

o Floor -Inguinallig amp lacunar lig

o External Ring - Formed by Ext Obliq Aponeurosis

o Internal Ring - formed by conjoined tendon

layers

o Ext Spermatic Fascia (EOmuscle)

o Cremasteric Fascia (IOmuscle)

o Internal Spermatic Fascia (Transversalis Fascia)

Contents Dunkin Donuts Tastes Awesome Please Sip a Caffeinated Grande Vanilla latte

o Ductus Deferens (snip snip =vasectomy)

o Testicular Artery (Called Testicular Torsion if this gets wound up)

o Pampiform Venous Plexus (Bag of Worms -left Hangs lower - why)

o Sympathetic Fibers (Point and Shoot)

o Cremasteric Muscle (Tickle Tickle)

o Genital Branch of genitofemoral N

o Vestige Process Vagina lis

o lymphatic Vessels

Ox of Indirect vs Direct Inguinal Hernias (Protrusions of peritoneum through an opening)

Indired inguinal CQes through the INternal (deep) ingllil1111 FeHowS Ih( path of Ule descent hernil dng external (stlperGeial inguin~l rillg and of the testes Cocred by ~ll1

INto the scrotum Enters internl hlguin31 ring 3 layets )f spermatic filscilt) lateral to inferiof ejgtigastrh artery OC-(gtl1f in INfants owing to failure ofpfo(e~~us vaginalts to dose rfu c h more (Qmmon in males

Direct inguinal Protrudes through the inguilllt11 (Hes~eJhaehs l MDs dont tIc hemi~l triangle Bu1ges directly through abdominal 1ledial to inferior epigastric

wall medial to inferior epi~~1Strilt artcrr alte-ry =Dired hClTlkl

GOtgtS through the external (Hlperfidal ~ Lateral to inferior epigastriC ingnilltl1 ling only Covered by transversalis MtelT =Indired he micentl

fascia Usually in older men

Lymphatics - Please dont get Testes and Scrotum mixed up

o Testes Lymph follows blood supply through inguinal canal back to abdominal aorta to

lumbar and preaortic lymph nodes Metastasis of Testicular Cancer - Livestrong

o Scrotum Lymph (As do lymphatics from all ectodermally derived structures in this

region) drain to superficial inguinal lymph nodes

GI blood supply and innervation Emhryonic Artery Pltlrasytnpathdilt VCTt-cbral Structnr(s suVP li-lt gut region inIlervation level

For(ut Cdi ~1 C Sgl5 T12Ll St)(fIl(b to pmxtll1li dll(llteJlllJl1middot lin-f gallbladder IMlliHa

Ll Dist1I dwXlentll11 Jo proximll -f of lra n ~Hrc colo n

I~lA Di~t~lll lgt of tmmt rsl cokm to upp-r portion [f fthllll _pknic Ilexuf is a wtlkrshtd ngiol1

Heart - Gastric and

duodenal regions

Cenae artery

~IY-~ Primordiuln of liver

Superior llli3Senteric artery 10 mido~n

I Interior mesenteric Hhidgut artecrj

Celia( trunk Bran ches of cd ial( [rnnK (ornm011 ]lmiddotpltic splc- nicleft gastric lhclt -ornpri~t tile main blood supply of the toma(h

Short Ilstric han pC gtOf

1lIltlSlo nlOc if ~pl ~ llic nrkry

is bloch-d Strollt anasbmOHS ltxist

betwe tn

-Left and right ga gtir(jt~plpJoj (s

-Left Lind right gastriCs

KNOW YOUR ANASTAMOSES

IIhlieunci

~plecn to JID$tEfipr i pleric artc~

Biliary structures

BE ABLE TO DRAW A FLOW CHART OF BILE FLOW

Important GI Iigaments

I

COnnect t

Ucr lcl n l)le rid~ abdominal i LlnmeJl1mh teresshy wall 1

I

if o(tal i ri~l c) he~atlc ~l rhry_ l1ay be olnptessed -1 Prin1 vein emiddotolllmn be~ween tlmmb 111d

t blle dud indd fin~d Pticed in cpk ploic farunen nfWinslow hJ ~ntt 1l1Jeecling

t

6n~e tHtre~te(Urid I crIt

middotbullbullmiddot -os

trHr to 1 liSef clltUliue middot1 Gastric m~eri~ 1 n ter and~epIt1tes ghtgrel

of stom~cll I les~rsac _~b be qt 9Brtng urger 1

~ i to ac(~3S le~(r sa

Grcater luC ltinind Ifa1t ofgreater 1)1I1tmiddotntllln

[ran fr colon 1

Lcreater curv ep~r1tCs left gre~br and t $pI eil middot I lesscr bllt$

qI

Splenorenal linil Vd~l 1 bull bull abdaininal JltaU

___o-J-___ ~~~~

~~----- ~-- ~~- ---~

Be able to visualize these ligaments know their embryonic origins and contents

Recognize relationship of structures in epiploic foramen (Portal Triad)

Splenorenal is high yield

Know which ligaments contribute to the greater and lesser momentum

Clinical- If the stomach ruptures posteriorly - contents are confined to the lesser sac but if

the stomach ruptures anteriorly - contents in the greater sac can spread as far down as the

pelvis

Know the relationship of structures within the portal triad - Hepatic Vein will be the trick

answer choice

L sgt-J[ offiEntumHpatic artY proPll

8iloduct bull

Stomach

Glstrapllmic ligament

Ornootnl-+----+r--___-----cT-- ~Jrarn n

IH---V~tal

petitOllllU-m

Splllllofffial figamgn

TXH

LIVER - ClinicalDryWet

Coronary Iigameot

Ewph3g~al

Impression

Lett tria09ular Hepatic veins

ligament

Fibrougt Supralerfal impreurossionappendix of

tiOer Hoparorenal portion ot coronary ligament

Garuic RighitrianQularl igameot impresgtion

(Common) bile duct

Common hepalic duel

Clstic duel

Renal Impression Caudate lobe

Papiilary proc~ss Quadrat~ lobe

allbladder

___ FalCltltorm ligament

Hepatic artery PlOP - Round Jigam~nt 01 liver

Hepatic portal veio ~-- Coli c impression

FisUH for ligamen1vm teres

Porta hepatis

Make sure know

Embryonic origin of the iigaments

Anatomical vs Functional lobes and relationship to the Gallbladder

Portal Triad Structures (VH DISSECTOR)

Impressions

CLINICALS -

Trac heoesophageal Almocml COll 11ction htteell esophalIs ll nd tmlthe1 fistula Jost lI)OUllul1 StLblypt is hlind up tr () hngtls with lOler esophagus tO)HHcted to

lrac hea RcsulJ in (3 11 $($ (ho tng anc vo mitin with ftt(~~lS lir bllblllpoundQll CXR and )Qlvl V -~-

I ~ i I Esophageal y i J atresia - - Trachea

----shy

Z

Congenital pyloric l1l perlrnphy (J f le pylurus camt~ obstruti I) Pa Ipablc uhve mass in epigl~tric n~ion stenosis and nOl1biliolts PfOiL mnit+rrg- - - wetk of ltl4C Tn)tment is suriltai ineisi n

Ottmi in 160(llit births ofteIl in I ~ t-bOfl malts --

Peptk ulcer djsease G~ltri( ulcer Pain can he greater vdth mellls--Welghtl - ft(H OCClIlS in older putient

a Mlori tnfedion in 70 chronic NSAID use also implicated Dm to llHue-oml protection a~lllns t gfltrh l id

Duodella) ulcer Pain Deu(QSe5 wilh menh--weighl ~aln_ Almost IOO have H priori infection Dut to 1~ gastfk acid cerct 11 f( 1 2011 inpmiddotr-Ell lson )~md rol11t i ( r L m(lcos~ ll

protction Hyprttophy of Bntnllcr glands Tend to ha ve ltIeall pullcheJ-ouf margim unlike the tais(dJirre~lllar margins of

cnCinOl1lH P-te1itiai eomplications include bl eedin~ penetrnu(n into pnnerelS perfomtion and obs tmctiQH (not intrinsltllly pr((JIlcerom) (see Image 11-+

Question - A 57 year old obese chronic alcoholic presents with an ulcer which has ruptured the cI~wnpastelgrly Surgical investigation reveals blood in the peritoneum Which of shy

the following arteries is most likely responsible for the bleeding

a Splenic Artery

b Gastroduodenal Artery

c R Gastric Artery

d L Gastric Artery

e L Gastroepiploic Artery

Appendicitis All age ~rol1ps most common indication for em er(nl ilxkminai)lJrger) in ehildrefL huLial diffmc periumbilical pain --7 localized pain JtMcBmm~ point N~ltlSC~l kvef

r)13)- perfor~te - pcntouitis Difftr~ntiI1 divertieuli[i (elderly ectopic pr(~gnanl to [3-hCG 10 rule ont)o

-----

PcmiddotrhtltllCf of the ) lcllille dud Of ~Olk sh31k ~br lontli n e( topic ~l cid-seefehllg gatric m 1(0]

andor pmcrcatic ii~~lle Lllost common c(lllgenital anomliy aftlle CT tract Cm CllHt hkfciing illtusm~(~pjjon Dlnllus or nbstrudion nelr the tcrrnin)l ileum Contraslwitll QIB~efic nmiddot = cvtk dilalaUon of ittllilC dJet

------~-

The ile 2$

2Jpound~11 11~ 2 feet frolll the iie( middoteCill vke l~ QfiJ~ at 1~

CIllIn nly prcsenfltgt III rll~2

llf~ of lifc- by ilwe 2 ty )ts of

epilheH8 19ls-trie- pal1elli(i

Hirschsplungs disease

Congenital tnt91(middotolon characterized by lack of Think of a gian ~pring that ganliml ttlLJcnJ(rk ~~gJeXllS~( (~lihs and lei ~sner plCxpstTlrlsgIllent on inbstinal biop y ()It to-iIure of U(middotural t~restpoundtU migration =

has s nl

Presents as ronic comtipnHoll tHly in life Dilllted pOltioll of the colon proximal 10 the aganiionic

segment resulting in a middot trmsjtillll ZQl1t rnvolt~

rectum [huany farlur( to P~$ meconium

High Yield WetDrylTheory List-

Suggestion - in your study group try to write a question for each of these points and then

exchange with a friend and try to answer each others questions

Abdomen Blood Supply - Reference viks picture posted on my google group - this is THE MOST HIGH

YIELD TOPIC IN ALL OF ANATOMY - expect 5 questions on your mini and 5-10 questions on your shelf

Make sure you can draw the blood supply and answer tertiary questions

Example - If the patient had an occlusion of the celiac trunk - which of the following areas would

experience ischemia

Portal hypertension - Know the 3 clinically relevant sites of portal caval anastamoses

Testes vs Scrotum lymph drainage

Where are paraumbillicai veins located

Omalophcele - failure of the gutto come back in (if in yolk sac - fatal)

Marginal Arteries

Superior Messenteric Artery is in front of 3d part of duodenum

Caput medusa (Distended paraumbillical veins secondary to portal hypertension)

Kidney Constrictors -1 Renal Pelvis 2 Crossing Pelvic Brim 3 Entering Urinary Bladder

Ureter - wet lab

Vagus is PIIJS supply up to 23 trans colon than pelvic splanch N up to the ass

Hirshsprungs disease - baby cant poop - dilated colon

Meckels Diverticulum - rule of 2s - 2 feet prox from ileocecal ju nc

Urachal Fistula - weeping belly

Gall stones - common bile duct

Jaundice relation to tumor of the head of the pancreas

Hepatopancreatic ampulla

Blood supply of renal gland - s superrenal art m s suprarenal a abd aorta inf suprarenal art

R Kidney - Tl2-L3 L Kidney - Tl1-L2

Renal Artery - L2

Epiploic foramen - know the borders and contents

Alantois diverticulum - urachus - medial umbilical

Lateral Medial and Median umbillical Folds (know the contents)

Directindirect hernia - know how to diagnose where they enter and exit the inguinal region and which

one is congenital

Anular pancreas - projectile vomiting

Pyloric Stenosis - projectile vomiting (non bilous)

Duodenal Atresia - projectile vomiting (bilous)

Dry Lab - know x rays

Vitteline Fistula - food out of umbilicus

Major duodenal papilla - junc of foregutmidgut

Arcuate line - relationship to rectus sheath

Mcburneys point -13 from ASIS bw umbilicus

Parietal pain - what is the nerve supply

Internal Oblique - cremasteric relationship

Know spermatic fasia

Processes Vaginalis - connection bw peritoneum and gubernaculums

Umbillicus - TlO dermatome

Deep Inguinal-l25 cm above mid inguinal ligament

Superficial Inguinal Ligament- superolateral to pubic symphysis

Variocele - veins engorged in scrotum (bag of worms)

bull

bull External spermatic fascia derived from external obliques EO II Cremasteric fascia ~ from internal obliques fO bull Internal spermatic fascia derived from fascia transversalis bull Tunica vaginalis derived from processes vaginalis directly rests on testes bull know order from testes out to skin

note reflex o ilioinguinal nerve o Efferent =genital branch of the genitofemoral nerve

--lt gt-- info important anastamoses which connects thorax to abdomen

bull Sup

o Sup epigastric branch of internal thoracic o Inf branch external iliac

Venous drainage o Above umbilicus aXillary v o Below umbilicus veins in triangle o At level of umbilicus Paraumbilical veins -gt drain into the portal V

II Important in Portal Caval Venous system Venous drainage of testes

o Clinical correlation Varicocele 11 vein drains into IVC 11 Left testicular vein ~ drains into left renal v

bag of rmlt

for lymph drainage T10 axillary lymph nodes

ill Below T10 superficial inguinal lymph nodes (lateral

Umbilical Folds

Lateral umbilical folds inferior vessels

Medial umbillcial folds umbilical (fetal remnant)

Median umbilical fold urachus (fetal remnant)

Between these folds fossas o Supervesical fossa between median and medial folds

11 bladder o hesselbachs between medial and I folds

II DIRECT HERNIAS HERE Borders

Medial semilunar line

Lateral info Epigastric

Inferior inguinallig o Lateral Inguinal Fossa beyond lateral fold

INDIRECT HERNIAS HERE II Deep inguinal ring (lateral to inferior epigastric a)

Indirect inguinal hernia o Lateral to inferior epigastric a o more common o When inserting finger in superficial inguinal ring will feel on tip of finger (since it goes

throueh ineuinal canall

----

Dry Lab - Label subcostal iliohypogastric Ll Ilioinguinal (Ll)

Horesshoe Kidney - stuck under IMA

Renal Agenesis -failure of the ureter bud to develop

Double Ureter

Unilateral Agenesis -1 kidney

Kidneys - Metanephros

Fetal kidneys are at sacral level

Look at 3rd part of duodenum

Some of this stuff is repeated I know just copied and pasted a bunch of stuff I had copy

Dermatomes

bull T4 nipples

bull no umbilicus v o Pain referred to no in appendicitis o Pain referred to T7ITS in gastritis ~

Inguinal ligament = external abdominal oblique aponeurosis

bull Inserts at anterior superior iliac spine to the pubic tubercle o Why important to know -7 visualizing this line allows us to properly diagnose a hernia

Below the inguinallig femoral hernia Above the inguinallig =inguinal hernia

Also to palpate the deep inguinal ring you go about 12Scm above the mid-inguinal

point

bull Modifications to ligament o Pectinate ligament o Lacunar ligament -7 cut this ligament to relieve strain i~ stran ul~tEd hernia

Inguinal canal

bull in males -7 transmits spermatic cord o important structures of spermatic cord ductus deferens testicular a genital branch of

the genitofemoral n pampiniform plexus of veins bull in females -7 transmits round ligament

Borders

bull Floor -7 inguinal ligament + lacunar ligament bull Anterior -7 aponeurosis of external oblique + internal oblique bull Roof -7 internal oblique and traverse abdominal bull Posterior -7 transverse abdominal + transversalis fascia

o Reinforced by conjoint tendon bull Aponeurosis of internal abdominal obliques and transverse abdominus bull Lies immediately behind the superficial inguinal ring in what would otherwise be

a weak point in the abdominal wall bull Innervated by ilioinguinal nerve (Ll) ~why important

bull In appendicitis Ll can be injured which will injure this nerve and in turn

the conjoint tendon With loss of innervation to this supportive structure the patient is now predisposed to a direct inguinal hernia

o Only hernia that can transverse the inguinal canal o Associated with congenital condition persistent tunica vaginalis

bull Direct inguinal hernia o Medial to inferior epigastric a o When inserting finger in superficial inguinal ring will feel on back of finger o Associated w old age or recent surgery

Muscles (Abdomen RECTUS SHEATH)

bull Arcuate line at level of ASISor 13rd distance between pubis and umbilicus bull Above arcuate line rectus abdominus is surrounded by a rectus sheath anteriorly and

posteriorly

o EO and 10 lie over rectus abdominus o 10 and TA lie behind rectus abdominus

bull Below arcuate line rectus abdominus has no rectus sheath posteriorly o EO 10 and TA lie over rectus abdominus o Transversalis fascia lies behind rectus abdominus o Inf EpIgastric vessels pierces the rectus sheath here

Peritoneum serous sac which encloses most of the abdominal structures

bull Ovary =only intraperitoneal organ o Oocyte ejected from ovary then captured by fallopian tubes o Why impt Women more prone to infection that can enter peritoneum

Peritoneum forms

bull Mesentery double layered fold of peritoneum formed as the organ was pulled in

bull Ligament between 2 organs in general bull Omentum between stomach and another organ bull Bare area area of no peritoneum bull

Viscera innervation

bull Pa rasympathetics 11 o Afferents sense hunger o Efferents l peristalsis relaxes sphincters gland secretion

bull Sympathetics o Efferents do opposite o Afferents CARRY PAIN SENSATION OF THE VISCERA (dull stretching pain)

bull PARASYMPATHETIC INNERVATION o Vagus nerve 7 _1l to 23rd unct ion of la rgej nte~tine oJ)elVrcspla~~~)~~ic~rYe~ IJiU- ~rd aJ~lpoteotiD~ IMPT

Gut Embryology

Gut ~ We say that the gut is derived from endoderm We often forget that when we say so we mean

that only the mucosa is derived from the endoderm The submucosa and the muscle layer is actually derived from the splanchnopleuric mesoderm and the serosa is derived from the visceral peritoneum

~ The main function of the gut is to digest the food which is done by the glands derived (and are) in the mucosa (endoderm) The only two exceptions in the Gut where glands though derived from the endoderm do not stay there but migrate down into the submucosa are esophagus and duodenum These glands however have their ducts opening to the swface of the mucosa

bull

~ Lungs liver amp gall bladder and pancreas are off-shoots from the foregut Esophagusshy~ The region of the tube from the laryngeal diverticulum to the beginning of the stomach elongates

to form the esophagus ~ The glands which form in the endoderm (mucosa) migrate down into the submucosa The path

whlch it took migrate becomes the duct of the glands which open to the mucosa ~ Achalasia Cardia - Failure of relaxation of the lower esophageal sphincter because of congenital

absence of ganglia at the sphincter (The ganglia when present releases VIP (Vaso-IntestinalshyPeptide) which relaxes the sphincter)

Mid-Gut Rotation ~ Because of the 90 degree rotation of the primitive stomach all of the following events occur ~ Lesser curvature comes to the right Therefore lesser omentum also comes to the right ~ Greater curvature comes to the left Therefore greater omentum also comes to the left ~ Right side vagal trunk becomes posterior vagal trunk ~ Left side vagal trunk becomes anterior vagal trunk ~ The left side peritoneal cavity comes to the anterior aspect of the stomach and will later be called

as the greater sac ~ The right side peritoneal cavity comes to the posterior aspect of the stomach and is (relatively a

small sac because the liver is on the right) called the lesser sacomental bursaepiploic sac ~ Epiploic foramen of Winslow (the lower free margin of the ventral mesentry) wiII be the

communication between the greater and lesser sac ~ The Liver moves to the right and therefore actually causes the 90 degree rotation of the stomach

The spleen comes to lie on the left side ~ Axis Antero-posterior axis around the superior mesenteric artery

bull Counterclockwise bull Approximately 270deg bull During herniation (about 90deg) bull During return (remaining 180deg)

Duodenum ~ Becomes retroperitoneal (except the first part which is still suspended by the hepato-duodenal

part of lesser omentum) ~ Glands (of Brunner) go submucosal ~ An imaginary line drawn below the opening of the major duodenal papilla represents the junction

between the foregut and midgut ~ Duodenal atresia in Downs syndrome Liver ~ 3rd week

bull liver bud grow bull into the septum bull transversum

~ 10th week bull hematopoietic bull function

bull 10 of the total bull body weight

~ 12th week bull bile is formed

Pancreas ~ In about 10 of cases the duct system fails to fuse and the original double system persists ~ 3rd month

bull pancreatic islets (Langerhans) ~ 5th month

bull Insulin secretion ~ Annular pancreas

bull The right portiCn of the ventralbud migrates along its normal route but the left migrates in the opposite direction

~ Complete obstruction of duodenum ~ Accessory pancreatic tissue Polyhydramnios (Amniotic fluidgt 1500-2000 ml)

~ Congenital defects including central nervous system disorders (eg Anencephaly) and gastrointestinal defects (atresias ego Duodenal esophageal) prevent the infant from swallowing the amniotic fluid (failure of recanalization)

Oligohydramnios (Amniotic fluid lt 400 mt) ~ Cl~ldberenal-agenesis

bull Midgut _-_

~ Primary Midgut intestinal loop gives rise to bull Distal duodenum bull Jejunum bull Ileum bull Ascending colon bull Transverse colon - proximal two-thirds of the bull Transverse colon with the distal third

~ Primary intestinaltoop bull ncephalic limb distal part of the duodenum the jejunum and part of the ileum bull ncaudal limb lower portion of the ileum the cecum the appendix the ascending colon and

the proximal two-thirds of the transverse colon bull 6th week

bull Rapid elongation of the cephalic limb bull Rapid growth of the liver bull Intestinal loops enter the extraembryonic cavity in the umbilical cord

bull 10th week bull loops begin to return bull regression of the mesonephric kidney reduced growth of the liver expansion of the

abdominal cavity bull Jejunum -left bull Loops - more to the right

bull Cecal bud -last part (temporarily below the right lobe of the liver) ~ qIDlthaloseJe (Structures COlHLoArts9V~1tion)

bull Through umbilical ring bull 6th to 10th weeks

bull Associated with a high rate of mortality (25) and severe malformations bull Associated with chromosome abnormalities

~ Gastroschisis (Structures coming out are not covered by Amnion) bull herniation through the body wall ----=---=-shybull Into the amniotic cavity bull Lateral right of the umbilicus bull Sometimes the inferior wall fails to develop as a result lower abdominal structures like the

bladder would be exposed to the exterior not associated with chromosome abnormalities ~ Abnormalities of the Mesenteries

bull Mobile cecum persistence of mesocolon bull Extreme form - long mesentery bull Volvulus

~ Distal third of the transverse colon ~ Descending colon ~ Sigmoid colon ~ Rectum ~ Upper part of the anal canal ~ Primitive anorectal canal

bull 7th week cloacal membrane ruptures bull Tip of the urorectal septum perineal body bull Pectinate line

~ Hindgut anamolies bull Rectoanal atresias and fistulas bull Imperforate anus bull Congenital megacolon (aganglionic megacolon Hirschsprung disease)

bull

bull Hindgut

Chirags Abdomen Review - Part 2

Understanding Embryo makes learning blood supply EASY

I I

I t

~ -

)

Table l1r-~ L Adult SUmiddotuctu~SDrj~l Froln Each of he Three Dhisions of be Pringttive GUl Tube t-middot-----middotmiddotmiddotmiddot-

Foregu(

I_ (Celiac Trunk)

Ir-slt-gtphgus

S101na(b

I h -= LiJ~r

Pancre=l S

bull 1 i Biliary apparntu5

Gall bladdshy

i Pha11~Cal pltgtuchcs

LullSS-I

Mjig ---- bull __ _- ----n--duct----~---middot-------l--n

(Superior Jldesen1eric Artery)_-1I-(I_~__ middot __ O-=-)_in_middoto_r_M_e_se_n_t_e_r_i_c_An__

Uuodenu rn 2nd_ 3 lt141h V4Tt

Jejunun-~

nc-un]

tCCUJ11

AppltgtndLX

Transver5e -o1on (p~oxiln1l1 ~O Tbird)

bull__hytgtid~ _ _ ______ L _ __

Tr-dn~llt~se colon (diStul h lTd) I

)

i

Aa ca-nal -( uppeT patt) i

I I

_____ __ _ _ _ ___ - - rhe~ a(t clcriVOkt iV(5 opound~lt prbn1rC ~ nlQC blft TlI)( 134tof r~ tIonoinf~ i 1 ~l l1rd c- P Cle

Now Lets see how much youve learned

Questions

1) A pt receives a general anesthetic in preparation for a c~t~~my A right subcostal incision is made which begins near the xyphoid process runs along and immediately beneath the costal margin to an anterior axillary line and transects the rectus abdominus muscle and rectus sheath At the level of the transpyloric plane the anterior wall of the

-~~-~=--- _eco---shysheath of the rectus abdominus muscle receives contributions from which of the following

a Aponeuroses of the in~ande~tef-Ilal o~ues

b Aponeuroses of the transversus abdominis and internal oblique muscles c Aponeuroses of the transversus abdominis and internal and external oblique

muscles d Transversalis fascia e Transversalis fascia and aponeurosis of the transversus abdominus muscle

A

2) The lat~raJJJ11QjJt~gLfgJlLoneach side of the inner surface of the anterior abdominal wall is created by which of the following structures

K Falx inguinalis (~) Inferior epigastric a

c Lateral border of the rectus sheath d Obliterated umbilical a e Urachus

B

3) A man the victim of several knife wounds to the abdomen during a brawl at the Lobster Shack subsequently developed a direct inguinal hernY Damage to which of the following nerves is most likely responsible for the predisposing weakness of the abdominal wall

~ Genitofemoral nerve ( b) Ilioinguinal nerve ~-t Tenth intercostal nerve

d Subcostal nerve e Pelvic splanchnic nerve

B

4) Which of the following statements concerning a direct inguinal hernia is correct a It is the most common type of abdominal hernia b It transverses the entire length of the inguinal canal c It contains all3 fascia layers of the spermatic cord d It exits the inguinal canal via the superficial ingeJinal ring e It protrudes through H~acb strJg e

~(

1fltbS w E

tl

5) The conjoint tendon is

a Important in preventing indirect inguinal hernias b The fused aponeurotic layers of internal abdominal oblique and transversus

abdominus muscles c Posterior to the deep inguinal ring

d Medial fibers of the inguinal ligament

B

6) A 25 year old male is brought in to the ER after being involved in a car accident in which he received a crushed internal injury in his abdomen Examination reveals a lesion of parasympathetic fibers in the vagJsnerve which interferes with glandular secretory or

smooth muscle functions in which of the foliowingorgans a Bladder b Transverse coloiW c Descending colOO d Prostrate gland e Rectum

B

7) The spermatic cord includes all of the following contents except a Il ioinguinal nerve b Pampin iform plexus of veins c Vas deferens d Genitofemoral nerve

A

8 Which abdominal structure gives rise to the internal spermatic fascia (muscle) following the descent of testes in development

a External abdominal oblique aponeurosis b Transversalis fascia c Transversus abdominis muscle d Peritoneum e Internal abdominal oblique

B

9 Which abdominal structure gives rise to the tunica vaginalis fotlowing the descent of testes during development shy

a External abdominal oblique aponeurosis b Transversalis fascia c Transversus abdominis muscle d Peritoneum e Internal abdominal oblique

D

10) The lesser omentum is a peritoneal fold which is su bdivided into the a Hepatogastric and gastrosplenic ligaments b Hepatoduodenal and gastroomentalligaments c Hepatoduodenal and gastrosplenic ligaments d Hepatogastric and hepatoduoden9-jrj igaments

D

11) A posteriorly perforating ulcer in the pyloric antrum of the stomach is most likely to produce initiallocalized peritonitis or abcess formation in which ofthS fQllowing

a Great-sac - -- -

b Paracolic recess

c Omental bursa

d Right subphrenic space

c

The inferior mesenteric artery arises from the abdominal aorta ilm_ediill~y_J-Qs1eriQLto which of the foowing org~ns A-F~t~filie duodenum B Head of the pan~eis C Neck of the pandeas

D Second part of the duodenum

E Third part of the duooenum_shylaquoshy

shy

The correct answer is E The inferior mesenteric artery arises from the anterior surface of the aorta at the level of the third lumbar vertebra The third part of the duodenum crosses the midline at the level of the third lumbar vertebra and passes anterior to the aorta at the origin of the inferior mesenteric artery The

first part of the duodenum (choice A) lies horizontally to the right of the midline at the level of the first

lumbar vertebra The head of the pancreas (choice B) is to the right of the midline and extends from the

level of the first lumbar vertebra to the third lumbar vertebra It lies within the concavity of the

duodenum The neck of the pancreas (choice C) lies in the midline at the level of the first lumbar

vertebra It lies on the anterior surface of the aorta at the origin of the superior mesenteric artery The second part of the duodenum (choice D) lies vertically to the right of the midline and extends from the

level of the first lumbar vertebra to the level of the third lumbar vertebra

The left adrenaLvein drains directly into which of the following veins A Hemiazygos vein

B Inferior vena cavaee C Left renal veiri -

D Splenic vein

E Superior mesenteric vein

a

The correct answer is C The left adrenal vein and the left gonadal vein (either testicular or ovarian) drain into the left renal vein TheTeft renal vein t~ains intothe- inferior vena cava In contrast the right

adrenal ~~inandnght gonadal veindrai~ gLr~ctJy iQtoJhe iilferiQ[ Vencava -- -

ThehemTazygoS7ein- (~h-~i-~ A)~~c~i~es the venous drainage from the body wall on the left side of the

thorax and abdomen No visceral organs drain directly to the azygos or hemiazygos veins The inferior vena cava (choice B) receives the direct venous drainage from the right adrenal vein but not

the left adrenal vein Remember the inferior vena cava is on the right side of the abdomen The splenic

vein (choice D) receives the venous drainage from the spleen and part of the pancreas and stomach The splenic vein is part of the portal venous system

The superior mesenteric vein (choice E) receives venous drainage from much of the intestinal tract It is part of the portal venous system and joins with the splenic vein to form the portal vein

A 43-year-old man presents complaining of pain in the groin On examination his physician palpates a

bulge in the region of the superficial inguinal ring which he diagnoses as a direct inguinal hernia The hernial sac most likely

A is covered by all three layers of the spennatic fascia B passes medial to the inferior epi gastric artery

C passes medial to the lateral border of the rectus abdominis muscle

D passes posterior to the inguinal ligament E passes through the deep inguinal ring

The correct answer is B Direct inguinal hernias enter the inguinal canal by tearing through the posterior

wall of that structure The typical location for this type of hernia is through the inguinal triangle bounded

laterally by the inferior epigastric artery medially by the lateral border of the rectus abdominis and

inferiorly by the inguinal ligament Direct inguinal hernias pass medial to the inferior epigastric artery

whereas indirect inguinal hernias pass lateral to the inferior epigastric artery because the deep inguinal

ring is lateral to the artery Indirect inguinal hernias are covered by all three layers of the spermatic fascia (choice A) Direct inguinal hernias are covered by fewer than all three layers because the direct inguinal

hernia tears through one or more layers of fascia as it emerges though the abdominal wall The lateral

border of the rectus abdominis muscle (choice C) forms the medial border of the inguinal triangle All

inguinal hernias pass lateral to the rectus abdominis Femoral hernias pass posterior to the inguinal ligament (choice D) Inguinal hernias emerge through the superficial inguinal ring which is superior to the inguinal ligament Inguinal hernias that descend below the inguinal ligament pass anterior to the

ligament Indirect inguinal hernias pass through the deep inguinal ring (choice H) direct inguinal hernias

do not Both types of inguinal hernias pass through the superficial inguinal ring

During a gastric resection in a patient with stomach cancer a surgeon wants to remove the lesser

omentum because of tumor extension into it Which of the following structures lie in the free edge of the

l~~g omentum and consequently must be dissected out in order to be preserved

A Common bile duct cystic duct and hepatic artery 6

B Cystic duct hepatic artery and hepatic vein

e Hepatic vein and cystic duct

Portal vein common bile duct and hepatic artery

E Portal vein hepatic artery and hepatic vein

The correct answer is D The free edge of the lesser omentum contains three important structures the

common bile duct the hepatic artery and the portal vein Nei ther the cystic duct (choices A B and C) nor the hepatic vein (choices B C and E) lies in the free

edge of the lesser omentum

A 55-year-old male patient with chronic liver disease has portal hypertension To relieve the pressure in the portal system a porto-caval shunt is performed Which of the following veins may by anastomosed to

accomplish this porto-caval shunt A Left renal vein-left testicular veingt

B Right renal vein-right suprarenal vein I shy

e Splenic vein -left renal vein J

D Superior mesenteric vein-inferior mesenteric vein E Superior mesenteric vein-splenic vein

The correct answer is C The splenic vein drains directly into the portal vein The left renal vein drains

directly into the inferior vena cava Anastomosis of these veins would allow blood from the portal vein to

drain retrograde though the splenic vein into the renal vein and then into the inferior vena cava The left

renal vein (choice A) drains directly into the inferior vena cava The left testicular vein drains directly into

the left renal vein Thus these veins are already in communication and neither vein is part of the portal venous system The right renal vein (choice B) drains directly into the inferior vena cava The right

suprarenal vein also drains directly into the inferior vena cava Thus neither vein is part of the portal

venous system The superior mesenteric vein (choice D) drains directly into the portal vein The inferior

mesenteric vein drains into the splenic vein which then drains into the portal vein Thus neither vein is

part of the caval venous system The superior mesenteric vein (choice E) drains directly into the portal

vein The splenic vein also drains directly into the portal vein Thus neither vein is part of the caval

venous system

A 12 year old boy has fever vomiting and para-umbilical pain After examining the patient the doctor

makes an initial diagnosis of appendicitis Appendicular pain which is initially referred to the umbilicus goes to the dorsal root ganglion of

a TI b TI2 c L1 d T7

(e I TIO

A 59-year-old male undergoes a neurological examination which reveals that when the abdominal wall is

stroked the muscles of the abdominal wall of the side of the body stimulated failed to contract Other

neurological tests appeared normal The likely region affected includes

a CI - C5 spinal segments b C6 - TI c T2-TI ~T8-T12

e Ll- L5

The surgery done to relive portal hypertension is done by connecting two veins Which of the following veins would be suitable for connection

a Inferior vena cava and portal vein b Superior vena cava and portal vein c Splenic vein and right renal vein d Splenic vein and left renal vein e Superior mesenteric vein and Inferior vena cava

A mother brings her 3-week-old infant to the pediatric clinic reporting a new scrotal bulge that she found -~-

while changing a diaper yesterday The infant is afebrile Physical examination reveals a palpable mass in

the scrotum while in the standing position resolution of the mass in the supine position and no

transillumination of the scrotal sac What is the most likely diagnOSiS

a Cryptorchidism b Direct inguinal hernia c Hydrocele d Indirect inguinal hernia ~ e varicocele

The Vagal trunks enter the abdomen by passing through which of the following openings in the

diaphragm

a Right crus b Esophageal hiatus ~ c Vena caval hiatus d Aortic hiatus e Left crus

2 The anterior boundary of the epiploic foramen of Winslow is bounded by

a) First part of duodenum b) Lesser curvature of stomach c) Liver d) Hepato-duodenalligament v ~

3 The ilio-inguinal nerve is derived from

a TI2 ry b LI c L2 d L3 e L23

15 Surgically the structure used to suspend the kidney to the diaphragm is

a) Renal fascia b) True capsule c) Perinephric fat d) Paranephric fat

6 If there is portal obstruction because of carcinoma affecting the pancreas which of these of the

following signs would be present

a Caput medusae b Esophageal varices c Rectal varices c

d Pulmonary edema

7 In a sliding hernia the gastro-esophageal junction lies

a) At its normal position b) Below the normal position c) Above the normal position V d) None of the above

8 Which of the following structures is retroperi toneal

A transverse colon B spleen IJ2f6 C ileum D descending colon v r 1pound1111111

9 The renal angle is fonned lgtetween the 12th rib and ______ muscle

a Psoas major -middotshyb Erector spinae c Quadratus Iumborum d Diaphragm

10 The anterior structure at the hilum of the kidney is

a) Renal vein ~

b) Renal artery I middot~ I

c) Ureter d) Accessory renal artery

11 Because of origin of the muscle from the lateral one third of the inguinal ligament it

could not fonn the anterior wall of the inguinal ligament

a) External oblique b) Internal oblique c) Transversus abdominis_ d) Rectus abdominis

12 A large tumor mass impinges on the splenic artery and its branches as the artery pass out from below

the greater curvature of the stomach Branches o(which of the following arteries would most likely to

effected by the pressure on the splenic artery

a Left gastric b Left gastro-epipJoic c Right gastric d Right gastro-epipoloic e Short gastric_

13 A new born baby has projectile vomiting after each feeding It is determined that there is obstruction

of the digestive tract as a result of annular pancreas Annular pancreas is as a result of an abnormality in which of the following process

a Rotation of the dorsal pancreatic bud around the first part of duodenum b Rotation of the dorsal pancreatic bud around the second part of duodenum c Rotation of the dorsal pancreatic bud around the third part of duodenum d Rotation of the ventral pancreatic bud around the first part of duodenum y Rotation of the ventral pancreatic bud around the second part of duodenum

14 As the liver bud enters the ventral mesogastrium the region of the mesogastrium stretching from the

liver to the anterior abdominal wall is called

a Lesser Omentum b Greater Omentum ~ Falcifrom ligament d Lacunar ligament e Ligamentum teres of liver

16 A patient has absence of his 12th rib In such a patient if the doctor makes an incision to approach his

kidney mistaking the 11 th rib for the 12t he would end up injuring

Which of the following arteries is a direct branch of the gastroduodenal artery The

A right gastric artery

B left gastric artery

C inferior pancreaticoduodenal artery D left gastroepiploic artery

i E)right gastroepiploic artery --

E x pI a nation The right gastric artery is typically a branch of the proper hepatic artery The left gastric artery is a direct

branch of the celiac trunk The right and left gastric arteries anastomose along the lesser curvature of the

stomach The inferior pancreaticoduodenal artery is a branch of the superior mesenteric artery it

anastomoses with the superior pancreaticoduodenal in the head of the pancreas The left gastroepiploic

artery is a branch of the splenic artery it anastomoses with the right gastroepiploic artery along the greater

curvature of the stomach The right gastroepiploic artery is a branch of the gastroduodenal artery The

other branch of the gastroduodenal artery is the superior pancreaticoduodenal artery

Which of the following pairs of veins join together to form the portal vein The

A superior mesenteric vein and inferior mesenteric vein

B inferior mesenteric vein and splenic vein

C superior mesenteric vein and splenic vein

Ip)splenic vein and left gastric vein E superior mesenteric vein and left gastric vein

Explanation

The portal vein is formed behind the neck of the pancreas by the union of the superior mesenteric vein

and the splenic vein The inferior mesenteric vein drains into the splenic vein The left gastric vein drains

directly into the portal vein After the portal vein forms it enters the hepatoduodenalligament of the

lesser omentum to reach the liver The portal vein is the most posterior structure in the hepatoduodenal

ligament

At which of the following vertebral levels does the duodenum pass anterior to the aorta - _- shy

All ~

B L2 7~

CL3 I

~DL4

E L5

Explanation

The duodenum begins at the pyloric sphincter at the level of Ll The second (or descending) portion of

the duodenum is to the right of the aorta and extends inferiorly from the level of Ll to the level of L3 The third part of the duodenum crosses the aorta from the right side to the left side at the level of L3 The

fourth (ascending) portion of the duodenum extends from the level of LJ to the level of L2 The

duodenum ends at the duodenojejunal flexure The superior mesenteric artery passes anterior to the

duodenum as the duodenum passes anterior to the aorta The duodenum can be constricted at this level

In which of the following locations will perforation of the digestive tract result in the spilling of luminal

contents into the - lesser peritoneal sac

A Anterior wall of the second portion of the duodenum B Posterior wall of the second portion of the duodenum

C Anterior wall of the stomach

~Posterior wall of the stomach E Posterior wall of the transverse colon

Explanation

The posterior wall of the stomach is related to the lesser peritoneal sac The anterior wall of the stomach is related to the greater peritoneal sac The anterior wall of the second portion of the duodenum is related to the greater peritoneal sac The posterior wall of the second portion of the duodenum is related to the retroperitoneal space The posterior wall of the transverse colon is related to the greater peritoneal sac

The ureter lies against the anterior surface of which of the following muscles shyA Crus oftne diaphragm B Quadratus lumborum

0 Psoas major D Transversus abdominis

E Iliacus

Explanation The ureter exits the renal pelvis at about the level of vertebra L2 As it descends along the posterior abdominal wall it lies on the anterior surface of the psoas major The psoas major muscle arises from the bodies of the lower lumbar vertebrae The psoas major muscle is joined by the iliacus to fonn the

iliopsoas muscle The iliopsoas muscle then attaches to the lesser trochanter of the femur and is the major

flexor of the hip

As the right ureter passes the pelvic brim it lies against the anterior surface of which of the following

blood vessels

A Gonadal artery B Inferiorvena cava C Internal iliac artery

rJ- External Iliac artery

E Inferior mesenteric artery

Explanation

The ureter lies in the extraperitoneal space in the posterior abdominal wall Alter leaving the kidney it

passes inferiorly on the anterior surface of the psoas major muscle At the pelvic brim the ureter passes

into the pelvis At this point the common iliac artery is dividing into the external and iliac arteries The

ureter lies on the anterior surface of the external iliac artery immediately distal to the bifurcation This is a useful landmark for a surgeon to locate the ureter

When extravasated urine passes from the superficial perineal space into the anterior abdominal wall it is

found immediately deep to which of the following layers of the anterior abdominal wall

-ltScarpas fascia

B External oblique muscle

C Internal oblique muscle D Transversus abdominis muscle

E Transversalis fascia

Explanation

The superficial perineal space is bound by Colles fascia the fibrous portion of the superficial fascia This

layer of fascia is continuous with Scarpas fascia the fibrous portion of the superficial fascia of the anterior abdominal wall Therefore urine that is deep to Colles fascia will remain deep to Scarpa s fascia The urine will spread in the plane between Scarpas fascia and the external oblique layer

When a horseshoe kidney develops the ascent of the kidney is restricted by the A internal iliac artery B external Iliac artery

C common iliac artery

inferior mesenteric artery

E superior mesenteric artery

Explanation

A horseshoe kidney develops when the inferior poles of the to kidneys fuse together as they ascend into

the abdomen from the pelvis The first anterior midline vessel that is encountered by the horseshoe kidney

is the inferior mesenteric artery This artery prevents the kidney from continuing its ascent

The left testicular vein drains into which of the following veins

A Left internal iliac vein B Left common iliac vein

bflnferior vena cava D Left renal vein I

E Left internal pudendal vein

Explanation

The left testicular vein drains into the left renal vein The right testicular ~i~[~nsltjectlY into the

inferior vena cava This difference in venous drainage is believed to explain the greater incidence of

varicocele on the left side than on the right The venous drainage from the penis is to the internal vein

which then drains into the internal Iliac vein

The spinal nerve that provides cutaneous branches to the skin around the umbilicus is

A TS B TW-shy

C TI2

DL2 EtA

Explanation

The tenth intercostal nerve is the anterior ramus of the TIO spinal nerve After passing through the tenth

intercostal space the nerve continues forward in the anterolateral abdominal wall in the plane between

the internal oblique muscle and the transversus abdominis muscle In the abdominal wall the nerve innervates to the abdominal wall muscles as well as the skin and the parietal peritoneum The umbilicus is

a useful landmark for the region of distribution of the tenth thoracic nerve

The ligament of the vertebral column that resists its extension is the Aligamentum flavum

B supraspinous ligament

C posterior longitudinal ligament

D anterior longitudinal ligament

E interspinous ligament

Explanation

The ligaments of the vertebral column that resist flexion of the column include the supraspinous ligament

interspinous ligament ligamentum fiavum and posterior longitudinal ligament The ligament that resists

extension is the anterior longitudinal ligament This longitudinal ligament is very broad and strong It

covers the anterior and anterolateral surfaces of the vertebral bodies and the intervertebral disks In

addition to resisting extension the anterior longitudinal ligament provides reinforcement to the anterior

and anterolateral surfaces of the intervertebral disk The posterior longitudinal ligament is relatively

narrow and covers the posterior surface of the vertebral bodies and the intervertebral disks This ligament

reinforces the posterior surface of the disk The posterolateral surface of the disk is not reinforced and it

is through this region that herniation of the nucleus pulposus usually occurs

A patient presents with epigastric and right upper quadrant pain The pain is most intense 2-4 hours after

eating and is reduced by the ingestion of antacids The patient states that he has passed black tarry stools

(melena) within the last week Fiberoptic endoscopy reveals a yellowish crater surrounded by a rim of

erythema that is 3 cm distal to the pylorus Accordingly an ulcer has been identified in the patients

A fundus

B antrum

C duodenum

D jejunum

E ileum

A number of physiologic genetic and other factors increase the risk of gastric (and duodenal) peptic

ulcers The evidence that H pylori plays a principle role is compelling Smoking and caffeine are known to adversely affect the morbidity mortality and healing rates of peptic ulcers In general first-degree

relatives of peptic ulcer patients as well as males have a threefold to fourfold increased risk of developing this disorder Paradoxically in gastric ulcer disease acid secretion is not elevated It is possible that

excess secreted hydrogen ion is reabsorbed across the injured gastric mucosa In general a defect in gastric mucosal defense is the more important local physiologic

A patient presents with symptoms of duodenal obstruction caused by an annular pancreas Annular pancreas is caused by

A rotation of the dorsal pancreatic bud into the ventral mesentery B rotation of the ventral pancreatic bud into the dorsal mesentery

fJ failure of the major and minor pancreatic ducts to fuse ~ ~ cleavage of the ventral pancreatic bud and rotation of the two portions in opposite directions around -the duodenum E formation of one pancreatic bud instead of two

Explanation Normally the ventral pancreatic bud rotates around the gut tube to reach the dorsal pancreatic bud The two buds fuse to form a single pancreas and the distal portions of the two ducts fuse The ventral pancreatic bud forms the inferior portion of the head of the pancreas the uncinate process and the major pancreatic duct (of Wirsung) The dorsal pancreatic bud forms the superior part of the head the neck body and tail and the minor pancreatic duct (of Santorini) Annular pancreas is the result of the ventral pancreatic bud dividing into two portions before it rotates into the dorsal mesentery Each portion rotates in opposite directions to get to the dorsal mesentery thus encircling the duodenum The presence of annular pancreas can constrict the duodenum thus obstructing its lumen

In n _ phranlc----

Gon ~l ----_1 Lum bltano

~~--- CornmQ1t bull ac

+-~4--- lnlllirnaJ ilic

xtem iliac

OBJECTIVE - Identify the blood supply to each of the structures listed in the table on the previous page

Ill give you a head start

FOREGUT - Supplied bV Celiac Tru nk (T12)

Proper hepatic

GastiooUod 13Jafter

1nferlor pancreaticoduodenal artery

Common epatlc

Lett gas ric iiirtery

Spfen artery

shy Gastroepiphgtic artery

~ Superior mesenteric 8rtfry

~

1 Esophagus is a derivative of the foregut so its blood supply originates from the celiac trunk

(T12) The predominant blood supply to abdominal portion of the esophagus is the Esophageal

A (Branch of L Gastric) The venous drainage of the esophagus is particularly important because

it is 1 of 3 clinically relevant sites of Portal Caval anastamoses The Portal Esophageal Vein

meets the Caval Azygos System Persistent bleeding manifests as Esophageal Varices - a fata I

condition

2 The Stomach is also a derivative of the foregut has EXTENSIVE blood supply and is very high

yield on anatomy exams The lesser curvature is supplied superiorly by the L Gastric A (1 of 3

major branches ofthe Celiac trunk) and inferiorly by the R Gastric A ( a branch ofthe proper

Hepatic A) The greater curvature is supplied superiorly by the L Gastroepiploic A (a major

branch of the splenic A) and inferiorly by the R Gastroepiploic A

The Short Gastric arteries (branches of Splenic Artery) supply the fundus of the stomach and

are referred to as EIID ARTERIES because they have no collateral blood supply Therefore if the

splenic artery were occluded (ex - increased pressure in the ommental bursa) - there would be

ischemia to the fundus of the stomach Venous drainage of the stomach is extensive via various

veins lead ing to the portal system Posterior to the stomach the IMV joins the splenic V which

joins the SMV to form the PORTAL VEIN ADAMS

3 Duodenum blood supply has high clinical relevance because it is the junction of the foregut and

midgut and therefore is the site of anastamoses between branches ofthe Celiac Trunk (main

foregut artery) and the Superior Messenteric Artery (main midgut artery) The Proper hepatic

artery gives off the gastroduodenal artery which travels behind the 1st part of the duodenum

This point has high clin ical relevance because duodenal ulcers are very common and a posterior

rupture of the 1st part of the duodenum could rupture the gastroduodenal artery causing

traumatic abdominal bleeding The Gastroduodenal artery first gives off the R Gastroepiploic A

(mentioned above) and proceeds as the Superior pancreatico duodenal artery (supplies the

pancreas and duodenum) which anastamoses with the inferior pancreatico duodenal A (branch

of the SMA) This is the junction of foregut and midgut and occurs near the opening of the

bil iary system into the duodenum (ampula of vater) Portal venous drainage here is responsible

for delivering nutrients from digestion to the liver for metabolism Appreciate that the Superior

mesenteric artery (artery of the midgut) branches from the aorta at Ll travels posterior to the

pancreas than moves anteriorly (at the jxn of the pancreatic headbody) and comes over the

3rd4th part of the duodenum Tumor of the head of the pancreas can compress the SMA

4 Jiver blood supply is via the common hepatic artery (major branch of the cel iac trunk) The

common hepatiC becomes the proper hepatic gives off the R gastric A and the Gastroduodenal

A and then joins the common bile duct and the portal vein in the portal triad Clinical- if a

patient were bleeding from the hepatic A a surgeon can stick his fingers in the epiplOic foramen

and squeeze the free edge of the hepatoduodenalligament in order to stop bleeding to the

area Please note that the hepatic a branches into Rand L hepatic A The Right hepatic artery

gives off the cystic artery which supplies the gallbladder Afferent venous supply is via the

Portal vein which is bringing nutrient rich blood to the liver After metabolism takes place

venous blood leaves the liver through the hepatic veins into the IVC PLEASE UNDERSTAND THE

RELATIONSHIP OF THESE STRUCTURES - ADAMSNETIERSNH Etc

5 Pancreas - Head is supplied via the superior and inferior pancreaticoduodenal arteries

(mentioned above) The tail (situated towards the hilum of the spleen) is supplied via the

pancreatic branches of the splenic artery (END ARTERIES) This blood supply is very important

because the endocrine Alpha and Beta Cells from the pancreatic islets of lagerhans are located

towards the tail This is where Insulin and Glucagon is released to the blood

Now complete this for mid and hindgut structures Make sure to note clinically relevant arterial

anastomoses as well as portal caval anastomoses FYI Appendix blood supply SMA + IMA

anastamoses marginal artery Portalcaval rectal veins fhemmorhoids) and periumbilical caput

medusa are high yield THE BUTT THE GUT and THE CAPUT

Abdominal Development

Liver

Ij1f

II wall b

oh liN ~ VltJrti n be- bull

Pancreas

Secondary Retroperitonealization e I~tl r 1 a v-mtrai m ellter

Rotations of the Gut I i Ij (lIl1UtIJ f~ l r tilt

()l td 10 me l-ft and he v

--~--- -~ -~-~

i

I AolaijonjoI~guf I

STOMACH BED (IDENTIFY IN ADAMS)- the structures posterior to the ommental bursa which

support the stomach in the supine position

Abdomnal JQrUI

Splnic vein

OmQ-oul tv~ ) O(s(Jroa)

Lojt(r o m nturrt (hpJtodu o d~n31 Hid

Gadrl)SplerH (g3stroll~nal) IIgam~nt

hiad h~~atogrtricent IIQdmiddotcrt~)

Lt Dome of Diaphragm (why left Look this up in Adams)

Spleen (What is the blood supply)

Left Kidney (What is the blood supply - AND how is it different from the R kidney)

Suprarenal Gland (What is the Arterial AND Venous Blood supply - how are they different)

Pancreas (How does supply differ from Head to Tail What is the SMA Relationship)

Transverse Mesocolon

liver - ADAMSWET - Make sure you look at the liver in wet lab

Left triangular nl1am~nt

ComoaDj ligamnt

Erophg~1 impre$ioo

Hepatio veins

In1erior -ifena middotr3)Ia

Fibrous appendix o-t

live

impr~j on

Heprorendl p~rtion of Q)(Qllary ligament

Righllri~n9ul r 1I~met

(Common) bile quol

Gr)mmCtr~ hepatic dlJct

Ccentic duct

Duodenal impression

GaJdate p-fr)~S

Hepatic artgtrl prop-f iiiiila - Faloiform ligament

_ - shy Round ligamen liver

~--F-- CoJio imprgt-ssi-on

Prta heptis

Identify the lobes impressions and embryonic remnants associated with the liver

Caudate Lobe Quadrate Lobe Right Lobe Left Lobe Round ligament Falciform Ligament

Ligamentum Venosum (what is its fxn in embryonic life) Hepatic Veins (NOT PART OF THE

PORTAL TRIAD) IVC PORTAL TRIAD - Contents relationship cross section etc Know the

Galbladder relationship to the lobes of the liver

Biliary Duct System - Make sure you understand the sequence of these structures - BE ABLE TO

DRAW A FLOW CHART

TPVd i

t

I t

1 __ Cm-(r

patk GlJet

I

J

Clinical = JAUNDICE is caused by anything that prevents delivery of bile to intestine Tumor of the

head of the pancreas Stones etc Patient will have pale stools and yellowish colored mucus

membranes

Clinical- Any scenario that tells you the patient has BILLOUS VOMIT means that the obstruction to

the flow of digestive contents is after the Ampulla of Vater (Site of Entry of Billiary system to the

duodenum) - ie Duodenal Atresia

Spleen -located posterior to the mid axillary line between ribs 9 and 11 Make sure you know that

the 10th rib is the main axis of the spleen and this organ is susceptible to injury (stab wound errant

thoracoce ntesis etc)

The spleen is derived from mesodermal cells - NOT THE GUT TUBE

The spleen rests on the left colic flexure associates with the tail of the pancreas Know the

structures entering the Hilum of the spleen

Sh rt O~-t~ic 1 0(0 10 rtiltSPIric Iloa nt

(cut)

Peritoneum - similar concept to Pleura - think of a fist in a balloon

Visceral Peritoneum - Layer of balloon touching your fist

Parietal Peritoneum - Layer of balloon not touching your fist

Your fist represents the organ your wrist is the hilum and your arm contains the blood supply

entering the organ

Appreciate that there will never be organs in the peritoneal cavity - rather these organs invaginate

the cavity Kaplan videos

RULES OF NOMENCLATUREshy

1 Organ completely surrounded by peritoneum - peritoneal organ

2 Organ partially surrounded by peritoneum- Retroperitoneal

3 Peritoneum surrounding peritoneal organ is VISCERAL peritoneum

4 Peritoneum surrounding retroperitoneal organ is PARIETAL peritoneum

5 Peritoneum connecting visceral to parietal is called messentary 2 messentaries in the

gut Dorsal (to the gut tube) and ventral (to the gut tube) messentary

Aorta is in Retro peritoneal position - but blood must reach peritoneal position - vessels travel through

messentary All peritoneal organs will have blood supply reaching through messentary

-Mesentery is a 2 layer peritoneum with a neurovascular communication between body wall and organ

- Ligament connects one organ with another or to the abdominal wall (Ommentum = ligament)

lesser Ommentum (attach lesser curvature of stomach and duodenum to liver) =Hepatoduodenal

Ligament and Hepatogastric Ligament

Has a Superior and Inferior Recess (Accumulation of Fluid in Ascites)

Communicates with the greater sac through the epiplic foramen (what structures pass through

this foramen)

Boundaries - you must be able to visualize this

o Anterior - stomach

o Posterior - parietal peritoneum pancreas

o Superior - superior recess (bw diaphragm and coronary ligament)

o Inferior -Inferior recess (bw layers or greater momentum

Greater Ommentum (attach greater curvature of stomach) Gastrophrenic ligament Gastrosplenic

ligament gastrocolic ligament

The greater omentum is the largest peritoneal fold It consists of a double sheet of peritoneum folded on itself so that it is made up of four layers The two layers which descend from the greater curvature of the stomach and commencement of the duodenum pass in front of the small intestines sometimes as low down as the pelvis they then turn upon themselves and ascend again as far as the transverse colon where they separate and enclose that part of the intestine

ABDOMINAL PAIN

Parietal Peritoneum - supplied by same vasculature lymphatics and nerves supplying body wall it

lines and diaphragm Sensitive to pain pressure heat cold well localized

Visceral Peritoneum - supplied by same vasculature lymphatics and somatic nerve of organ it covers

Insensitive to touch heat cold and laceration - referred to dermatome of spinal ganglia providing

sensory fibers Where does appendicitis refer to

Foregut pain - epigastric area (ie - cholycystitis)

Midgut pain - periumbilical area (ie - appendicitis)

Hindgut Pain - suprapubic area (ie - diverticulitis)

Extra ImagesConcepts

ll~_____-

FalifCtrm ligament oind r~ud ligamet f Ilver

Blood from splenio gastriC and inferiof rne$e-rteri v~ins

Ca-I tributaries

Lett gastrio Ifein

Posterior superior pan~reatioodul)denal vaihS

Lott gamo-om~nlal (9aropip lomiddotic) -in

Poq_~ tjol imerl-9-r panCJertlcorllJod-nal veiopound --amp----I- - ~J Right grtr~-omntal

Anwrior interi (gartroepiploic) Jjn

pan euaii cod vl)denal veins middot Inf~Ji (t r mesentric vein

Miqdle (olic vein

Right cl)licvein Sigmoid and rectosigml)id (ei ns

IhH)Collc(~io

--- Mi~dl laquooLJl gtjrltgt

PoM ca vl1 illasto)moses -----shyampoptoageal 2 Paraumbilie-lt11 Inferi or Fectal vei ns

3 Recial 4 REuoperHonea1

Know how the Portal vein is formed I 4 sites of portal caval anastamoses and 1 clinical shunt

Col li t ltt-~ otTl~tI ~nj pc~ 1lt1 turJoG

Ltf 14i1 tImiddot~ artoftl9 on tj phtAt$

L-oftqf 4t t~r 1=laquoIran d 1 bull shy~p l ci rj o fOOOts

Nerves follow the arteries - appreciate the splanchnic nervous system I

Uet~ric branch of left ~nal art

Ureterie branch of righi renal artelY

Left Zld lumbar in and co mlTlunication to as)erdin9 lumbar l(~in Hi ~ht tEZ1~~t~ t3r j t itn ~ nJ l1t- rlnd lfe i r1

Inferior me5nteri~ artery

Notice that the right testicular vein drains directly into the IVC and the right testicular artery drains

directly into the aorta However the left testicular vein drains into the L renal vein at a right angleshy

reason left testicle is lower and more susceptible to varicocele (bag of worms)

Also notice that the left renal vein has a longer course because the IVC is on the right side whereas

the right renal artery has a longer course because the aorta is on the left side

Appreciate the anterior to posterior relationship of structures in the hilum of the kidney - VAP - Vein

Artery Renal Pelvis (Ureter)

11____ __ L_ L_ n VJ __ _ _ t_L I I_ _ L __ L_ I -pound1 bull LI_~-I ____

Posterior View of Head of Pancreas in ( of Duodenum

Celiao hunk

Co mmon ~L~jJth art~ry

GastNduQdonal artrf (partilly in phantn)

P1)Sterior $Up~Jior panCflaticuduodfmal art~r~t

(Co mm on) bile duct

middot~1t~~t-1l---~-~- Right gshomiddotomental (gastoe plp lolc) 3rte (phantomost)

Grener paocre atic art-ry

1n1~rjor pancr-iatlc artery

Jtrifll supejo r pal)oreailcento)dJodenal artr1 (phantom)

Anastomotlo branch

POostetlor bJanch of jo f~ri of pan-reatir(lduodensl drttnj

Anterio r branch of i flferior palcreati~)duodenal art~(phan1om)

Notice the extensive blood supply to the pancreas and duodenum via the branches of the celiac trunk

Notice collateral supply from SMA branches - makes sense bc this is the jxn of foregutmidgut

Identify the vessels in this arteriogram

Hiltid i)f N~ck oi B)dvof Tail 01 pa nereas pan cent~as P-nmiddot-reas panCtCas

I nferie v~na cava

jHept1iic p(lrlai v~in

Port1 tnd H~pti lt a ftH prol

Comm on) bll duct

Ouodtnum

~ft colic (sio)Atta~ hmtrlt jt~xJr-ofha~elSe

muo(IIQn

Right ~lIc (h~j)tic)

il~gtture

In1triol m~oten lIein (rttr op~ritoMdO

SlJp efl or mes~n~fiC amrV and lipln

KNOW YOUR NEIGHBORHOOD

Questions

vVhiJh structure supplied by a bnmdlof the cclia( artery is not derivcd from foregut LemCJUCrITI

(A) Head of the pancte-a5

CD) Pyloric duolenum

Cystkduct

( Liver hepatocyt~~

~F) Body of the spleen

An infant presents with an omrhaJucele at birth -hi oJ the [oHm illg applies to his cM1-dition

(A) It is 31so seen ill p4titnts with aganghonic megacolon

(11) ft reuirs from a fal1ure of resorption of theviteUine d let

(C) It results from herniation at the-site of regression of the right umbilk vein

DJ It is caustd by faihtrc of recanalization of the midgut part of the duodenum

~ It ill camioo by a failuIt vf the midgul to return to the abGQminal uity after herniashytion in-n the urnbilk s l stalk

Ot er than the spleen occlusion Cif the spit-Ilk artery at its odgin wm most likely affect die blood supply to jllch st cnud

(A) Jejunum

(B) Body of th pal1~lltas

(C) LeSStT Cllmiddotlaturc of tl )toma-ch

(D Duodenum dista to the entrance of the Ornmou bile duct

E Fundus of the stomach

A 38-yeu-old batL~er with a history of heartburn suddenly experiences excluciating pain in the (plgastric region of th~ abdomeu SurgCry is perf~rme immediard y upon admisshysion to the 1IlcrgCJliy tuomh~re i~ evidence uf a ruptured ulcer in the posterior waU of the stomach Vhere will a surgeon first fi nd the stomach contenlSf

A) Greater p4ritoneal sac

rB) Cul~de-s~c of Douglas (--

C Omental bursa ~

--D) Paracolic gutter

rEj Between -he panttal perimltum and the posterior body wal1

At birth an infant presents with a st()ma~ rb~tbas~njJled jfltotb~diaplfagru 1A1ltre is the defect thatresulied iiitJle heini~t()n shy~tsophagealbiatus

7 - rH-- Hiatus for the inferior vena cava

( Pleuroperitoneal membrane -(0) Septum transvcrsum

(E) Right Crlt~

An infant born with DOVv7l syndrome presents with bili()u~ vomiting Ahat congenital defect does the infant have

(A) Pyloric stenosis

(B) Meckel diverticulum C) Ornphaloce1e

(D) Gastroschisis

( ~ ) Duodenal atresia y A patient with cirrhosis of the liver presents with ~ bacalvaricestnlreased retrograde pressure in which veins caused the varices

(A) Paraumuilical

(B) Splenic

(ct AzygltJus

(15))G~trk ( (-F) Superior mesemeric

A htaltby 3-year~old male patient experiences a hernial sa protruding from the anterior abdominal wall about halfway between me anterior superior ilia spine and the pubk tuberde Pulsations of al1 artery are palpated medial to the protrusion site through the abdominal walL Which layer of the anterior abdominal wall will first be traversed by the

1hctma

fA) Rectus sheath (B) External oblique aponeurosis

(C) Inguinal ligament

lD) Transversalis fusda

(E) Cremasteric fa~cia

After 5urgi(aj ffpair of a hernia the patient tXperienccs mtmlgtness in the skin on the anteshyrior aspect of the S(Totum_ Vhaf nerve may have been lesioned during thehemiorrhaphy

(A) Femoral

(B) Obturator

(C) Ilioinguinal

(D) lliohypogastrk

(E) Pudendal

A 23~year-LJld female secretary il1 good health ~-uddcn1) doubles over with pain in the a ea of the 1JmbRicu$ Sbe feels vartn and ltneasy and has no appetite That night the pain seems to have mQved to the tower right abdominal regjol1 and she calls her family doctor who then arranges for an ambulance to pk-k her up and take her to the hospitaL Wh ell ntn~ perceived in the area of the urnbilirus most Hkely carried lhe pairfu I sensations into the eNS

tA) Vagus nerves I~

V B)

) Lessersplanchnk nerves

tC) Pudendal nerves

(D) lIiohpogastrk nerves

(E) Greater splam ic l erves

A CT reveals carcinoma in the bOod of the ancreas Vhich blood vessel trut ourses ----~- - -bull ------ --shy

immediately poftterior to the body ofthe pancreas is the m~t likely to be oompressed

(A) Splenk artery

(B) Abdominal aorta (C) Portal vein

(1) Splenic vein

(E) Renal vein

A patient has a penrln1l1ng uker of the posterior wall ot the br~l part ot the (lUooenmn llkh blood vessel is subject to erosion

(A) Common hepatic artery

(B) Gastroouodenal artery

(C) Proper hevatic artery

(D) Celiac artery

(E) Anterior inferior 11amrelltlcoduodcnal attery

Your patient has been diagnosed -ith a carcinoma locallted to the head and l~e(k of the pancreas Another clinical sign would be

A esophageal varices

(8) hemorrhoids

C) a caput medusa

(D) increased pra Teuro n th~ hepatic veins

(E) enlarged right supra lavkular lymph nodes

Wltkh of the foUowing structures develops in the ventral mesentery

(A) Spleen

(B) Jeiunum (C) Head of1ht pancreas (D) Transverse colon (E) Stomach

ti l Uw ~ littwin~ f( S-t lil oai Imdge ~ hi(h or tbt la~)d J truetur tgt liJ llntn nl) he hl p UC iJd [IIi ell

c o

A) drains Ie tht infCrior a La aI

R t middot~nfl0 ~ill to th~ lunlgtn of h i dtlndCrlllfH

(e) m t bull JiJattd on tl l J n T ~H

D ) sup Lc O VSlt I Hlid bhtu l 1 li - -I un oid

( ) U~tpli(t tr j middottUh~ 1 v(( b~nt rfK n1ilc~Zm

ANSWERS AND EXPLANATIONS

Answer E The spleen is t hlttnopodicand lymph organ demlted from mesoderm

Answ~ R Al1 tlmphalocele is caused by it failure of the nlidgut to return to the ahdomir nat cavity after herniation into the umbiliau Stalk Choices Aand D maybe seen in infants with Down syndrome choice D ~s the specific CBuse ofduudcnal JtiCSitt Choice C is (ile cause of gclstrosbisis and Choice B nsults iu a Meurolktldivertku1-tlB

Answer B The fundus ofthe stomach is suppHed by soort gastric brunches of the splenic altery The splenic artery supplies the body and tail of the pancreas part of the greater curvature of the sttmla(h and the spleen Te jejunum part of the head of the pancreas and tht~ duodenum distal to the entrance of the commOll bile duct are supplied by the superior mesenterk artery clll~l ~be less r ctlt1ature cmd the pylQric antrum are supplied by the right and lei gastric art(ries

AnSWftt C Tbeomental bursa or lesser ~ritoneaj sac lies direcdy posterior to the proxshyimal part of the duodeTtlm and the stomach and would be the first site where stomach contents ~Ott1d be fpoundluncL

Answer C A defect in a llleuropcritoneal membrane (uswlly the left) is the typical site of i1 cc-ngenitlI diilphragluatic hemia llere the membr4ne fails to dose ()pound( of the perishycCirdiopcritulleal canals

Answer E DuoJenal atresia and aganglionic megacoion are congwitaI defects S~Il in patients with Dowmiddotnsyndrome

Answer D RulaTgemt~llt of and retrograde flow in g~lstrk vel_ns in particlJl~r the kft gas~ tricveins dilates the capillary bed in rhe wall of the esophagus in (ases of porta yper~

tension Blood flow would increase in and dilampte tribntarkgts of the (lZygOUS vein on the other side of the capiUary bed but flow in this vein is in the typical direction t()ward the superior vena cava Paraumbiii(ltU vein eilgorgement contributes to a caput medusH Splenic ~nlargement might prc~nt with 5plcnonlegaly and balt-kflow in to tlu superior m~~ntclic vein occurs but is asymptomatic

Answer D The patient hagt an indirect inguinal hernia whi~h emerges from the antt-rior abdominal wall through the deep inguinltilling Theeep ring is a fault in the transv~rshysaUs fascia this I~yer wiIJ be penetrated first by the hernia

An~Wer C The ilioinguinal nenc which provides sens~llion to the lnedlal thigh ltmclanteshytior SClotunl pass~lt th rough the 5uperfh_ial inguinal ring ind $subject to inj i1T) becaus-e

it is in the operatitm Held of the erniorrhapny

Auswer B The leMHr splanchnic nerves are sympathdic nerVlts that carry viscera l sensashytlltgtrogt ftom illtllt1m~d ()J stietched gust (itinteitinal ~tructures (in this case the pprndix) into tnt eNS Lesser splanchnic ntTYcsarisc from thmiddot T9--T12 spinal cord segments lt1nd provide sympathetic innenation tD rnidgut siruc1ures whiCh include CLe app~JldD Viscera] Pain arising from affecLed Inidgut ampt 1C1ure is referred over the same dl- matorne~ of spinal segrnertts v-hich provide the sympathetic Innervation n this G1SC of appendicitis the invohen~n t of the ltire) of t e unlhHku indud s the T 10 dermatome

Answer B Of the five choices onty the dscending olon is retroperiton~al aldwould be a lik ~ ( choice to be seen immediately a(~jilcent to t11e posterior abdominal middotn~L

Amwen D The SpltftlC ~-ein ourses posterior to the body of the panneas m its way tt drain into the superior mCSfttltlri( vein

Answcr B TILt glstrodllolticnal artery 1 direct hIamh of the comrootl hepatic artery courses immediately pt))iwri() to the duodenum and is slbject to erosion

Answer B Carcinoma of th pan middott3S in the 1tilt1 may compreampgt the portltil vein at irs orishygill The poTtai vcin is fomled when the splenic vein jQiaswith tfie superior meStllt eric vein The inferiot mesenteric vein joins the ~plenjc vein just priOT to tlli~ point at which the splenic joins the superior Jlleit1ltcri( vein Increescd venous presslu in the inferior mesenteric vein is a cause of emo hoid~

Answer C The- velltral pancreas wilich forms most of the head of the p ~ncr as develops in the ventral mes(ntery as antutgrowth of the hepatic diverticulum Th~ hepatic divershyticulull induding the biIJary appa~atus develops in tbe ventral mesentery of the foregut

Answer~ A The superior mesenteric ~in joins with the spienkvein to form the hepatic portal vciu

Answer D The structure at gttlK is the proper hepatic artery~ whkh suppUesoxygenated b middotood to the liver

MAKE SURE YOU KNOW the diff bw Rectus Sheath above and below the arcuate line

ABOVE

Aponeurosis of xiiltmal obllque musclo

Extemll f)biquw musde

Reotln ilbdomlnls musole S~in

Internal 9bliquQ mY~QI

AponeUfOsi$ of hJH$V~~S Lir9a a lb lbdolTlin~ musolo Tri OJV6 rUi

atldomlnis mUS(loe

Sub cutanlilous tiue (tatty ye r)

BElOW

A POrl lJfosis 01 etemal oblique muscle

Aponeul~)sis 01 Internal oblique mU$cl~

Anteriol lay~ of r~ltdus st~ath EXttom1 oblique rnu$cll

Rectus Jbdominis muscle Intoernal Aponeurc-sis of tra~fersU$ oblique muscle-

at-domlnis muscentl ~ Skio

Tra nsvitSus abdomioLs ml)ZClt

TralSVersaHs fascia Medial umQil iegtt1 1i9Jment -and folj

Uldchus Peritoneum (ir median Umbilj~al Suboutane ous

Extraprftone 11ascia

Ymbilimiddot~1 fold)

preu9poundiea1 fascia

tissue (fatty 4nd m~mbr3n(iUS layers)

o Above the arcuate line (A horizontal line 13 of the distance bw the umbilicus and the

pubic symphysis) -10 Aponeurosis divides into an AntPost Laminae

o The Ant Laminae joins EO and Post Laminae joins Trans Abdominis = Ant and Post

RECTUS SHEATH respectively

o BElOW the arcuate line - all 3 aponeurosis join ANTERIOR to rectus muscle to meet its

counterpart in the midline (linea Alba)

o Take away Msg - The abdomen is devoid of a posterior rectus sheath below the

arcuate line and is therefore more vulnerable to herniasinjuries

Question - A physician makes a deep incision in the patients midline immediately superior to

the pubic symphysis which of the following layers is his knife least likely to pass

Rectus Abdominis External Oblique Ant Rectus Sheath Posterior Rectus Sheath All of the

Above

Answer - All of the above None of the other answer choices are midline structures -LINEA

ALBA

Linea Alba has very poor blood supply - doesnt heal well after surgery Therefore this is a

common site for incisional hernias

a Spleen b Transverse colon c Descending colon d Stomach e Pleura

17 Meckels diverticulum is normally found 2 feet proximal from the

a Pyloric sphincter b Lower esophageal sphincter c Ileo-cecal valve d Middle valve of Huston e Anal valve

18 Ulcer in the posterior wall of the first part of the duodenum would erode ___ artery and would cause bleeding

a Left gastric b Right gastric c Hepatic artery proper d Gastroduodenal artery e Middle colic artery

19 An inflamed appendix is identified by a surgeon on the operation table by noting

a The appendicies epiploicae b The convergence of tenia c The artery of Drummond d The mesocolon e The mesosalphinx

20 The nerve which emerges through the psoas major is

a Femoral b Ilio-inguinal c Ilio-hypogastric d Pudendal e Subcostal

21 The right gonadal vein drains into the

a Azygos b Hemiazygos c Inferior Vena Cava d Right renal vein e Left renal vein

22 The hepatocytes in the liver is derived from

a Ectoderm b Endoderm c Mesoderm

d Neural ectoderm

23 Abscess in the lumbar vertebrae due to tuberculosis would spread to the adjacent muscle which is

a Psoas Major b Iliacus c Quadratus lumborum d Tranversus Abdominis

24 The anterior wall of the inguinal canal is formed by

a External oblique and transverses abdominis b External oblique and fascia transversalis c Internal oblique and external oblique d Internal oblique and transverses abdominis e Fascia transversalis and peritoneum

Meckels diverticulum is a result of which of the following developmental abnormalities shy

A Failure of the vitelline duct to close

B Failure of the herniated intestinal loop to retract into the abdomen

C Failure of the urachus to close

D Failure of the midgut to rotate

E Failure of the hepatic duct to close

Explanation

Meckels diverticulum is a result of the persistence of the proximal part of the vitelline duct This

diverticulum is usually found about 2 feet proximal to the ileocecal junction and is usually about 2 inches

long It is present in about 2 of the popUlation It may be the site of ectopic pancreatic tissue or gastric

mucosa and may develop inflammatory processes and ulcerations Acute Meckels diverticulitis

simulates appendicitis

Which of the following veins carries blood from the esophagus to the portal vein The

A right gastric vein

B left gastric vein c splenic vein D azygos vein

E left gastroepiploic vein

Explanation

The left gastric vein a direct branch of the portal vein drains blood from the lesser curvature of the

stomach and the inferior portion of the esophagus Because branches of the portal vein do not have

valves blood can flow in a retrograde path when there is an obstruction to flow through the portal system or liveL Rlooci Cln then flow from the nortl] vein thr()1Ph the left PRstric vein to the esonhlPlIS lno

through venous communications within the submucosa of the esophagus to esophageal veins that drain

into the azygos vein The increase in blood flow through the esophageal submucosal veins results in esophageal varices

On the posterior wall of the abdomen the celiac ganglion A contains cell bodies of postganglionic parasympathetic neurons B is synapsed upon by neurons in the posterior vagal trunk C is synapsed upon by neurons in the greater splanchnic nerve D contains sensory cell bodies of lumbar spinal nerves E contains cell bodies of neurons that cause an increase in the rate of peristasis

Explanation The celiac ganglion is one of the preaortic ganglia of the sympathetic nervous system It contains cell bodies of postganglionic sympathetic neurons The sympathetic splanchnic nerves contain preganglionic sympathetic neurons that pass through the sympathetic chain without synapsing These splanchnic nerves go to the preaortic ganglia to synapse The greater splanchnic nerve contains preganglionic neurons from spinal cord segments T5-T9 This nerve synapses in the celiac ganglion The nerve fibers in the vagal trunks are preganglionic parasympathetic fibers that go to the walls of the organs that they will innervate and synapse on postganglionic parasympathetic neurons in the walls of those organs Cell bodies of sensory neurons in the abdomen are found in the dorsal root ganglia or the sensory ganglia of the vagus nerve Sympathetic innervation decreases the rate of peristalsis parasympathetic innervation increases the rate of peristalsis

Which of the following pairs of arteries will allow blood to bypass an occlusion of the celiac trunk

A Left gastric artery-right gastric artery

B Left gastroepiploic artery-right gastroepiploic artery

C Superior pancreaticoduodenal artery-inferior pancreaticoduodenal artery

D Splenic artery-common hepatic artery

E Left gastric artery - proper hepatic artery

Explanation The anastoOlosis of a branch of the celiac trunk and a branch of the superior mesenteric artery will

provide collateral circulation around an occlusion of the celiac trunk Each of the other choices pair

branches of the celiac trunk therefore these will not provide collateral flow around the obstruction of the

celiac trunk The left gastric splenic and common hepatic arteries are direct branches of the celiac trunk

The right gastric artery is a branch of the proper hepatic artery which is a branch of the common hepatic artery The left gastroepiploic artery is a branch of the splenic artery The right gastroepiploic artery is a

branch of the gastroduodenal artery whlch is a branch of the common hepatic artery

Which of the following organs has appendices epiploica The

A sigmoid colon

Bjejunum

C duodenum

D stomach E esophagus

Explanation Appendices epiploica are characteristic of the colon Appendices epiploica are subserosal accumulations

of fat None of the organs of the gastrointestinal tract has appendices epiploica except the colon

Page 4: Chirag's Abdomen Review

INGUINAL LigamentCanalHernias Inguinal canal

In fetal life the gonads descend from the posterior abdominal wall In the case of males the

testes must descend all the way outside of the body in order to maintain an optimal

temperature for spermatogenesis to be maintained

What you must know about the inguinal region

Boundaries

o Ant - Ext Oblique Aponeurosis

o Post - Transversalis Fascia amp Int Obliq (Form a conjoint tendon - attached Pub Symph)

o Roof - Aponeurosis of Int Obliq amp Trans Abd

o Floor -Inguinallig amp lacunar lig

o External Ring - Formed by Ext Obliq Aponeurosis

o Internal Ring - formed by conjoined tendon

layers

o Ext Spermatic Fascia (EOmuscle)

o Cremasteric Fascia (IOmuscle)

o Internal Spermatic Fascia (Transversalis Fascia)

Contents Dunkin Donuts Tastes Awesome Please Sip a Caffeinated Grande Vanilla latte

o Ductus Deferens (snip snip =vasectomy)

o Testicular Artery (Called Testicular Torsion if this gets wound up)

o Pampiform Venous Plexus (Bag of Worms -left Hangs lower - why)

o Sympathetic Fibers (Point and Shoot)

o Cremasteric Muscle (Tickle Tickle)

o Genital Branch of genitofemoral N

o Vestige Process Vagina lis

o lymphatic Vessels

Ox of Indirect vs Direct Inguinal Hernias (Protrusions of peritoneum through an opening)

Indired inguinal CQes through the INternal (deep) ingllil1111 FeHowS Ih( path of Ule descent hernil dng external (stlperGeial inguin~l rillg and of the testes Cocred by ~ll1

INto the scrotum Enters internl hlguin31 ring 3 layets )f spermatic filscilt) lateral to inferiof ejgtigastrh artery OC-(gtl1f in INfants owing to failure ofpfo(e~~us vaginalts to dose rfu c h more (Qmmon in males

Direct inguinal Protrudes through the inguilllt11 (Hes~eJhaehs l MDs dont tIc hemi~l triangle Bu1ges directly through abdominal 1ledial to inferior epigastric

wall medial to inferior epi~~1Strilt artcrr alte-ry =Dired hClTlkl

GOtgtS through the external (Hlperfidal ~ Lateral to inferior epigastriC ingnilltl1 ling only Covered by transversalis MtelT =Indired he micentl

fascia Usually in older men

Lymphatics - Please dont get Testes and Scrotum mixed up

o Testes Lymph follows blood supply through inguinal canal back to abdominal aorta to

lumbar and preaortic lymph nodes Metastasis of Testicular Cancer - Livestrong

o Scrotum Lymph (As do lymphatics from all ectodermally derived structures in this

region) drain to superficial inguinal lymph nodes

GI blood supply and innervation Emhryonic Artery Pltlrasytnpathdilt VCTt-cbral Structnr(s suVP li-lt gut region inIlervation level

For(ut Cdi ~1 C Sgl5 T12Ll St)(fIl(b to pmxtll1li dll(llteJlllJl1middot lin-f gallbladder IMlliHa

Ll Dist1I dwXlentll11 Jo proximll -f of lra n ~Hrc colo n

I~lA Di~t~lll lgt of tmmt rsl cokm to upp-r portion [f fthllll _pknic Ilexuf is a wtlkrshtd ngiol1

Heart - Gastric and

duodenal regions

Cenae artery

~IY-~ Primordiuln of liver

Superior llli3Senteric artery 10 mido~n

I Interior mesenteric Hhidgut artecrj

Celia( trunk Bran ches of cd ial( [rnnK (ornm011 ]lmiddotpltic splc- nicleft gastric lhclt -ornpri~t tile main blood supply of the toma(h

Short Ilstric han pC gtOf

1lIltlSlo nlOc if ~pl ~ llic nrkry

is bloch-d Strollt anasbmOHS ltxist

betwe tn

-Left and right ga gtir(jt~plpJoj (s

-Left Lind right gastriCs

KNOW YOUR ANASTAMOSES

IIhlieunci

~plecn to JID$tEfipr i pleric artc~

Biliary structures

BE ABLE TO DRAW A FLOW CHART OF BILE FLOW

Important GI Iigaments

I

COnnect t

Ucr lcl n l)le rid~ abdominal i LlnmeJl1mh teresshy wall 1

I

if o(tal i ri~l c) he~atlc ~l rhry_ l1ay be olnptessed -1 Prin1 vein emiddotolllmn be~ween tlmmb 111d

t blle dud indd fin~d Pticed in cpk ploic farunen nfWinslow hJ ~ntt 1l1Jeecling

t

6n~e tHtre~te(Urid I crIt

middotbullbullmiddot -os

trHr to 1 liSef clltUliue middot1 Gastric m~eri~ 1 n ter and~epIt1tes ghtgrel

of stom~cll I les~rsac _~b be qt 9Brtng urger 1

~ i to ac(~3S le~(r sa

Grcater luC ltinind Ifa1t ofgreater 1)1I1tmiddotntllln

[ran fr colon 1

Lcreater curv ep~r1tCs left gre~br and t $pI eil middot I lesscr bllt$

qI

Splenorenal linil Vd~l 1 bull bull abdaininal JltaU

___o-J-___ ~~~~

~~----- ~-- ~~- ---~

Be able to visualize these ligaments know their embryonic origins and contents

Recognize relationship of structures in epiploic foramen (Portal Triad)

Splenorenal is high yield

Know which ligaments contribute to the greater and lesser momentum

Clinical- If the stomach ruptures posteriorly - contents are confined to the lesser sac but if

the stomach ruptures anteriorly - contents in the greater sac can spread as far down as the

pelvis

Know the relationship of structures within the portal triad - Hepatic Vein will be the trick

answer choice

L sgt-J[ offiEntumHpatic artY proPll

8iloduct bull

Stomach

Glstrapllmic ligament

Ornootnl-+----+r--___-----cT-- ~Jrarn n

IH---V~tal

petitOllllU-m

Splllllofffial figamgn

TXH

LIVER - ClinicalDryWet

Coronary Iigameot

Ewph3g~al

Impression

Lett tria09ular Hepatic veins

ligament

Fibrougt Supralerfal impreurossionappendix of

tiOer Hoparorenal portion ot coronary ligament

Garuic RighitrianQularl igameot impresgtion

(Common) bile duct

Common hepalic duel

Clstic duel

Renal Impression Caudate lobe

Papiilary proc~ss Quadrat~ lobe

allbladder

___ FalCltltorm ligament

Hepatic artery PlOP - Round Jigam~nt 01 liver

Hepatic portal veio ~-- Coli c impression

FisUH for ligamen1vm teres

Porta hepatis

Make sure know

Embryonic origin of the iigaments

Anatomical vs Functional lobes and relationship to the Gallbladder

Portal Triad Structures (VH DISSECTOR)

Impressions

CLINICALS -

Trac heoesophageal Almocml COll 11ction htteell esophalIs ll nd tmlthe1 fistula Jost lI)OUllul1 StLblypt is hlind up tr () hngtls with lOler esophagus tO)HHcted to

lrac hea RcsulJ in (3 11 $($ (ho tng anc vo mitin with ftt(~~lS lir bllblllpoundQll CXR and )Qlvl V -~-

I ~ i I Esophageal y i J atresia - - Trachea

----shy

Z

Congenital pyloric l1l perlrnphy (J f le pylurus camt~ obstruti I) Pa Ipablc uhve mass in epigl~tric n~ion stenosis and nOl1biliolts PfOiL mnit+rrg- - - wetk of ltl4C Tn)tment is suriltai ineisi n

Ottmi in 160(llit births ofteIl in I ~ t-bOfl malts --

Peptk ulcer djsease G~ltri( ulcer Pain can he greater vdth mellls--Welghtl - ft(H OCClIlS in older putient

a Mlori tnfedion in 70 chronic NSAID use also implicated Dm to llHue-oml protection a~lllns t gfltrh l id

Duodella) ulcer Pain Deu(QSe5 wilh menh--weighl ~aln_ Almost IOO have H priori infection Dut to 1~ gastfk acid cerct 11 f( 1 2011 inpmiddotr-Ell lson )~md rol11t i ( r L m(lcos~ ll

protction Hyprttophy of Bntnllcr glands Tend to ha ve ltIeall pullcheJ-ouf margim unlike the tais(dJirre~lllar margins of

cnCinOl1lH P-te1itiai eomplications include bl eedin~ penetrnu(n into pnnerelS perfomtion and obs tmctiQH (not intrinsltllly pr((JIlcerom) (see Image 11-+

Question - A 57 year old obese chronic alcoholic presents with an ulcer which has ruptured the cI~wnpastelgrly Surgical investigation reveals blood in the peritoneum Which of shy

the following arteries is most likely responsible for the bleeding

a Splenic Artery

b Gastroduodenal Artery

c R Gastric Artery

d L Gastric Artery

e L Gastroepiploic Artery

Appendicitis All age ~rol1ps most common indication for em er(nl ilxkminai)lJrger) in ehildrefL huLial diffmc periumbilical pain --7 localized pain JtMcBmm~ point N~ltlSC~l kvef

r)13)- perfor~te - pcntouitis Difftr~ntiI1 divertieuli[i (elderly ectopic pr(~gnanl to [3-hCG 10 rule ont)o

-----

PcmiddotrhtltllCf of the ) lcllille dud Of ~Olk sh31k ~br lontli n e( topic ~l cid-seefehllg gatric m 1(0]

andor pmcrcatic ii~~lle Lllost common c(lllgenital anomliy aftlle CT tract Cm CllHt hkfciing illtusm~(~pjjon Dlnllus or nbstrudion nelr the tcrrnin)l ileum Contraslwitll QIB~efic nmiddot = cvtk dilalaUon of ittllilC dJet

------~-

The ile 2$

2Jpound~11 11~ 2 feet frolll the iie( middoteCill vke l~ QfiJ~ at 1~

CIllIn nly prcsenfltgt III rll~2

llf~ of lifc- by ilwe 2 ty )ts of

epilheH8 19ls-trie- pal1elli(i

Hirschsplungs disease

Congenital tnt91(middotolon characterized by lack of Think of a gian ~pring that ganliml ttlLJcnJ(rk ~~gJeXllS~( (~lihs and lei ~sner plCxpstTlrlsgIllent on inbstinal biop y ()It to-iIure of U(middotural t~restpoundtU migration =

has s nl

Presents as ronic comtipnHoll tHly in life Dilllted pOltioll of the colon proximal 10 the aganiionic

segment resulting in a middot trmsjtillll ZQl1t rnvolt~

rectum [huany farlur( to P~$ meconium

High Yield WetDrylTheory List-

Suggestion - in your study group try to write a question for each of these points and then

exchange with a friend and try to answer each others questions

Abdomen Blood Supply - Reference viks picture posted on my google group - this is THE MOST HIGH

YIELD TOPIC IN ALL OF ANATOMY - expect 5 questions on your mini and 5-10 questions on your shelf

Make sure you can draw the blood supply and answer tertiary questions

Example - If the patient had an occlusion of the celiac trunk - which of the following areas would

experience ischemia

Portal hypertension - Know the 3 clinically relevant sites of portal caval anastamoses

Testes vs Scrotum lymph drainage

Where are paraumbillicai veins located

Omalophcele - failure of the gutto come back in (if in yolk sac - fatal)

Marginal Arteries

Superior Messenteric Artery is in front of 3d part of duodenum

Caput medusa (Distended paraumbillical veins secondary to portal hypertension)

Kidney Constrictors -1 Renal Pelvis 2 Crossing Pelvic Brim 3 Entering Urinary Bladder

Ureter - wet lab

Vagus is PIIJS supply up to 23 trans colon than pelvic splanch N up to the ass

Hirshsprungs disease - baby cant poop - dilated colon

Meckels Diverticulum - rule of 2s - 2 feet prox from ileocecal ju nc

Urachal Fistula - weeping belly

Gall stones - common bile duct

Jaundice relation to tumor of the head of the pancreas

Hepatopancreatic ampulla

Blood supply of renal gland - s superrenal art m s suprarenal a abd aorta inf suprarenal art

R Kidney - Tl2-L3 L Kidney - Tl1-L2

Renal Artery - L2

Epiploic foramen - know the borders and contents

Alantois diverticulum - urachus - medial umbilical

Lateral Medial and Median umbillical Folds (know the contents)

Directindirect hernia - know how to diagnose where they enter and exit the inguinal region and which

one is congenital

Anular pancreas - projectile vomiting

Pyloric Stenosis - projectile vomiting (non bilous)

Duodenal Atresia - projectile vomiting (bilous)

Dry Lab - know x rays

Vitteline Fistula - food out of umbilicus

Major duodenal papilla - junc of foregutmidgut

Arcuate line - relationship to rectus sheath

Mcburneys point -13 from ASIS bw umbilicus

Parietal pain - what is the nerve supply

Internal Oblique - cremasteric relationship

Know spermatic fasia

Processes Vaginalis - connection bw peritoneum and gubernaculums

Umbillicus - TlO dermatome

Deep Inguinal-l25 cm above mid inguinal ligament

Superficial Inguinal Ligament- superolateral to pubic symphysis

Variocele - veins engorged in scrotum (bag of worms)

bull

bull External spermatic fascia derived from external obliques EO II Cremasteric fascia ~ from internal obliques fO bull Internal spermatic fascia derived from fascia transversalis bull Tunica vaginalis derived from processes vaginalis directly rests on testes bull know order from testes out to skin

note reflex o ilioinguinal nerve o Efferent =genital branch of the genitofemoral nerve

--lt gt-- info important anastamoses which connects thorax to abdomen

bull Sup

o Sup epigastric branch of internal thoracic o Inf branch external iliac

Venous drainage o Above umbilicus aXillary v o Below umbilicus veins in triangle o At level of umbilicus Paraumbilical veins -gt drain into the portal V

II Important in Portal Caval Venous system Venous drainage of testes

o Clinical correlation Varicocele 11 vein drains into IVC 11 Left testicular vein ~ drains into left renal v

bag of rmlt

for lymph drainage T10 axillary lymph nodes

ill Below T10 superficial inguinal lymph nodes (lateral

Umbilical Folds

Lateral umbilical folds inferior vessels

Medial umbillcial folds umbilical (fetal remnant)

Median umbilical fold urachus (fetal remnant)

Between these folds fossas o Supervesical fossa between median and medial folds

11 bladder o hesselbachs between medial and I folds

II DIRECT HERNIAS HERE Borders

Medial semilunar line

Lateral info Epigastric

Inferior inguinallig o Lateral Inguinal Fossa beyond lateral fold

INDIRECT HERNIAS HERE II Deep inguinal ring (lateral to inferior epigastric a)

Indirect inguinal hernia o Lateral to inferior epigastric a o more common o When inserting finger in superficial inguinal ring will feel on tip of finger (since it goes

throueh ineuinal canall

----

Dry Lab - Label subcostal iliohypogastric Ll Ilioinguinal (Ll)

Horesshoe Kidney - stuck under IMA

Renal Agenesis -failure of the ureter bud to develop

Double Ureter

Unilateral Agenesis -1 kidney

Kidneys - Metanephros

Fetal kidneys are at sacral level

Look at 3rd part of duodenum

Some of this stuff is repeated I know just copied and pasted a bunch of stuff I had copy

Dermatomes

bull T4 nipples

bull no umbilicus v o Pain referred to no in appendicitis o Pain referred to T7ITS in gastritis ~

Inguinal ligament = external abdominal oblique aponeurosis

bull Inserts at anterior superior iliac spine to the pubic tubercle o Why important to know -7 visualizing this line allows us to properly diagnose a hernia

Below the inguinallig femoral hernia Above the inguinallig =inguinal hernia

Also to palpate the deep inguinal ring you go about 12Scm above the mid-inguinal

point

bull Modifications to ligament o Pectinate ligament o Lacunar ligament -7 cut this ligament to relieve strain i~ stran ul~tEd hernia

Inguinal canal

bull in males -7 transmits spermatic cord o important structures of spermatic cord ductus deferens testicular a genital branch of

the genitofemoral n pampiniform plexus of veins bull in females -7 transmits round ligament

Borders

bull Floor -7 inguinal ligament + lacunar ligament bull Anterior -7 aponeurosis of external oblique + internal oblique bull Roof -7 internal oblique and traverse abdominal bull Posterior -7 transverse abdominal + transversalis fascia

o Reinforced by conjoint tendon bull Aponeurosis of internal abdominal obliques and transverse abdominus bull Lies immediately behind the superficial inguinal ring in what would otherwise be

a weak point in the abdominal wall bull Innervated by ilioinguinal nerve (Ll) ~why important

bull In appendicitis Ll can be injured which will injure this nerve and in turn

the conjoint tendon With loss of innervation to this supportive structure the patient is now predisposed to a direct inguinal hernia

o Only hernia that can transverse the inguinal canal o Associated with congenital condition persistent tunica vaginalis

bull Direct inguinal hernia o Medial to inferior epigastric a o When inserting finger in superficial inguinal ring will feel on back of finger o Associated w old age or recent surgery

Muscles (Abdomen RECTUS SHEATH)

bull Arcuate line at level of ASISor 13rd distance between pubis and umbilicus bull Above arcuate line rectus abdominus is surrounded by a rectus sheath anteriorly and

posteriorly

o EO and 10 lie over rectus abdominus o 10 and TA lie behind rectus abdominus

bull Below arcuate line rectus abdominus has no rectus sheath posteriorly o EO 10 and TA lie over rectus abdominus o Transversalis fascia lies behind rectus abdominus o Inf EpIgastric vessels pierces the rectus sheath here

Peritoneum serous sac which encloses most of the abdominal structures

bull Ovary =only intraperitoneal organ o Oocyte ejected from ovary then captured by fallopian tubes o Why impt Women more prone to infection that can enter peritoneum

Peritoneum forms

bull Mesentery double layered fold of peritoneum formed as the organ was pulled in

bull Ligament between 2 organs in general bull Omentum between stomach and another organ bull Bare area area of no peritoneum bull

Viscera innervation

bull Pa rasympathetics 11 o Afferents sense hunger o Efferents l peristalsis relaxes sphincters gland secretion

bull Sympathetics o Efferents do opposite o Afferents CARRY PAIN SENSATION OF THE VISCERA (dull stretching pain)

bull PARASYMPATHETIC INNERVATION o Vagus nerve 7 _1l to 23rd unct ion of la rgej nte~tine oJ)elVrcspla~~~)~~ic~rYe~ IJiU- ~rd aJ~lpoteotiD~ IMPT

Gut Embryology

Gut ~ We say that the gut is derived from endoderm We often forget that when we say so we mean

that only the mucosa is derived from the endoderm The submucosa and the muscle layer is actually derived from the splanchnopleuric mesoderm and the serosa is derived from the visceral peritoneum

~ The main function of the gut is to digest the food which is done by the glands derived (and are) in the mucosa (endoderm) The only two exceptions in the Gut where glands though derived from the endoderm do not stay there but migrate down into the submucosa are esophagus and duodenum These glands however have their ducts opening to the swface of the mucosa

bull

~ Lungs liver amp gall bladder and pancreas are off-shoots from the foregut Esophagusshy~ The region of the tube from the laryngeal diverticulum to the beginning of the stomach elongates

to form the esophagus ~ The glands which form in the endoderm (mucosa) migrate down into the submucosa The path

whlch it took migrate becomes the duct of the glands which open to the mucosa ~ Achalasia Cardia - Failure of relaxation of the lower esophageal sphincter because of congenital

absence of ganglia at the sphincter (The ganglia when present releases VIP (Vaso-IntestinalshyPeptide) which relaxes the sphincter)

Mid-Gut Rotation ~ Because of the 90 degree rotation of the primitive stomach all of the following events occur ~ Lesser curvature comes to the right Therefore lesser omentum also comes to the right ~ Greater curvature comes to the left Therefore greater omentum also comes to the left ~ Right side vagal trunk becomes posterior vagal trunk ~ Left side vagal trunk becomes anterior vagal trunk ~ The left side peritoneal cavity comes to the anterior aspect of the stomach and will later be called

as the greater sac ~ The right side peritoneal cavity comes to the posterior aspect of the stomach and is (relatively a

small sac because the liver is on the right) called the lesser sacomental bursaepiploic sac ~ Epiploic foramen of Winslow (the lower free margin of the ventral mesentry) wiII be the

communication between the greater and lesser sac ~ The Liver moves to the right and therefore actually causes the 90 degree rotation of the stomach

The spleen comes to lie on the left side ~ Axis Antero-posterior axis around the superior mesenteric artery

bull Counterclockwise bull Approximately 270deg bull During herniation (about 90deg) bull During return (remaining 180deg)

Duodenum ~ Becomes retroperitoneal (except the first part which is still suspended by the hepato-duodenal

part of lesser omentum) ~ Glands (of Brunner) go submucosal ~ An imaginary line drawn below the opening of the major duodenal papilla represents the junction

between the foregut and midgut ~ Duodenal atresia in Downs syndrome Liver ~ 3rd week

bull liver bud grow bull into the septum bull transversum

~ 10th week bull hematopoietic bull function

bull 10 of the total bull body weight

~ 12th week bull bile is formed

Pancreas ~ In about 10 of cases the duct system fails to fuse and the original double system persists ~ 3rd month

bull pancreatic islets (Langerhans) ~ 5th month

bull Insulin secretion ~ Annular pancreas

bull The right portiCn of the ventralbud migrates along its normal route but the left migrates in the opposite direction

~ Complete obstruction of duodenum ~ Accessory pancreatic tissue Polyhydramnios (Amniotic fluidgt 1500-2000 ml)

~ Congenital defects including central nervous system disorders (eg Anencephaly) and gastrointestinal defects (atresias ego Duodenal esophageal) prevent the infant from swallowing the amniotic fluid (failure of recanalization)

Oligohydramnios (Amniotic fluid lt 400 mt) ~ Cl~ldberenal-agenesis

bull Midgut _-_

~ Primary Midgut intestinal loop gives rise to bull Distal duodenum bull Jejunum bull Ileum bull Ascending colon bull Transverse colon - proximal two-thirds of the bull Transverse colon with the distal third

~ Primary intestinaltoop bull ncephalic limb distal part of the duodenum the jejunum and part of the ileum bull ncaudal limb lower portion of the ileum the cecum the appendix the ascending colon and

the proximal two-thirds of the transverse colon bull 6th week

bull Rapid elongation of the cephalic limb bull Rapid growth of the liver bull Intestinal loops enter the extraembryonic cavity in the umbilical cord

bull 10th week bull loops begin to return bull regression of the mesonephric kidney reduced growth of the liver expansion of the

abdominal cavity bull Jejunum -left bull Loops - more to the right

bull Cecal bud -last part (temporarily below the right lobe of the liver) ~ qIDlthaloseJe (Structures COlHLoArts9V~1tion)

bull Through umbilical ring bull 6th to 10th weeks

bull Associated with a high rate of mortality (25) and severe malformations bull Associated with chromosome abnormalities

~ Gastroschisis (Structures coming out are not covered by Amnion) bull herniation through the body wall ----=---=-shybull Into the amniotic cavity bull Lateral right of the umbilicus bull Sometimes the inferior wall fails to develop as a result lower abdominal structures like the

bladder would be exposed to the exterior not associated with chromosome abnormalities ~ Abnormalities of the Mesenteries

bull Mobile cecum persistence of mesocolon bull Extreme form - long mesentery bull Volvulus

~ Distal third of the transverse colon ~ Descending colon ~ Sigmoid colon ~ Rectum ~ Upper part of the anal canal ~ Primitive anorectal canal

bull 7th week cloacal membrane ruptures bull Tip of the urorectal septum perineal body bull Pectinate line

~ Hindgut anamolies bull Rectoanal atresias and fistulas bull Imperforate anus bull Congenital megacolon (aganglionic megacolon Hirschsprung disease)

bull

bull Hindgut

Chirags Abdomen Review - Part 2

Understanding Embryo makes learning blood supply EASY

I I

I t

~ -

)

Table l1r-~ L Adult SUmiddotuctu~SDrj~l Froln Each of he Three Dhisions of be Pringttive GUl Tube t-middot-----middotmiddotmiddotmiddot-

Foregu(

I_ (Celiac Trunk)

Ir-slt-gtphgus

S101na(b

I h -= LiJ~r

Pancre=l S

bull 1 i Biliary apparntu5

Gall bladdshy

i Pha11~Cal pltgtuchcs

LullSS-I

Mjig ---- bull __ _- ----n--duct----~---middot-------l--n

(Superior Jldesen1eric Artery)_-1I-(I_~__ middot __ O-=-)_in_middoto_r_M_e_se_n_t_e_r_i_c_An__

Uuodenu rn 2nd_ 3 lt141h V4Tt

Jejunun-~

nc-un]

tCCUJ11

AppltgtndLX

Transver5e -o1on (p~oxiln1l1 ~O Tbird)

bull__hytgtid~ _ _ ______ L _ __

Tr-dn~llt~se colon (diStul h lTd) I

)

i

Aa ca-nal -( uppeT patt) i

I I

_____ __ _ _ _ ___ - - rhe~ a(t clcriVOkt iV(5 opound~lt prbn1rC ~ nlQC blft TlI)( 134tof r~ tIonoinf~ i 1 ~l l1rd c- P Cle

Now Lets see how much youve learned

Questions

1) A pt receives a general anesthetic in preparation for a c~t~~my A right subcostal incision is made which begins near the xyphoid process runs along and immediately beneath the costal margin to an anterior axillary line and transects the rectus abdominus muscle and rectus sheath At the level of the transpyloric plane the anterior wall of the

-~~-~=--- _eco---shysheath of the rectus abdominus muscle receives contributions from which of the following

a Aponeuroses of the in~ande~tef-Ilal o~ues

b Aponeuroses of the transversus abdominis and internal oblique muscles c Aponeuroses of the transversus abdominis and internal and external oblique

muscles d Transversalis fascia e Transversalis fascia and aponeurosis of the transversus abdominus muscle

A

2) The lat~raJJJ11QjJt~gLfgJlLoneach side of the inner surface of the anterior abdominal wall is created by which of the following structures

K Falx inguinalis (~) Inferior epigastric a

c Lateral border of the rectus sheath d Obliterated umbilical a e Urachus

B

3) A man the victim of several knife wounds to the abdomen during a brawl at the Lobster Shack subsequently developed a direct inguinal hernY Damage to which of the following nerves is most likely responsible for the predisposing weakness of the abdominal wall

~ Genitofemoral nerve ( b) Ilioinguinal nerve ~-t Tenth intercostal nerve

d Subcostal nerve e Pelvic splanchnic nerve

B

4) Which of the following statements concerning a direct inguinal hernia is correct a It is the most common type of abdominal hernia b It transverses the entire length of the inguinal canal c It contains all3 fascia layers of the spermatic cord d It exits the inguinal canal via the superficial ingeJinal ring e It protrudes through H~acb strJg e

~(

1fltbS w E

tl

5) The conjoint tendon is

a Important in preventing indirect inguinal hernias b The fused aponeurotic layers of internal abdominal oblique and transversus

abdominus muscles c Posterior to the deep inguinal ring

d Medial fibers of the inguinal ligament

B

6) A 25 year old male is brought in to the ER after being involved in a car accident in which he received a crushed internal injury in his abdomen Examination reveals a lesion of parasympathetic fibers in the vagJsnerve which interferes with glandular secretory or

smooth muscle functions in which of the foliowingorgans a Bladder b Transverse coloiW c Descending colOO d Prostrate gland e Rectum

B

7) The spermatic cord includes all of the following contents except a Il ioinguinal nerve b Pampin iform plexus of veins c Vas deferens d Genitofemoral nerve

A

8 Which abdominal structure gives rise to the internal spermatic fascia (muscle) following the descent of testes in development

a External abdominal oblique aponeurosis b Transversalis fascia c Transversus abdominis muscle d Peritoneum e Internal abdominal oblique

B

9 Which abdominal structure gives rise to the tunica vaginalis fotlowing the descent of testes during development shy

a External abdominal oblique aponeurosis b Transversalis fascia c Transversus abdominis muscle d Peritoneum e Internal abdominal oblique

D

10) The lesser omentum is a peritoneal fold which is su bdivided into the a Hepatogastric and gastrosplenic ligaments b Hepatoduodenal and gastroomentalligaments c Hepatoduodenal and gastrosplenic ligaments d Hepatogastric and hepatoduoden9-jrj igaments

D

11) A posteriorly perforating ulcer in the pyloric antrum of the stomach is most likely to produce initiallocalized peritonitis or abcess formation in which ofthS fQllowing

a Great-sac - -- -

b Paracolic recess

c Omental bursa

d Right subphrenic space

c

The inferior mesenteric artery arises from the abdominal aorta ilm_ediill~y_J-Qs1eriQLto which of the foowing org~ns A-F~t~filie duodenum B Head of the pan~eis C Neck of the pandeas

D Second part of the duodenum

E Third part of the duooenum_shylaquoshy

shy

The correct answer is E The inferior mesenteric artery arises from the anterior surface of the aorta at the level of the third lumbar vertebra The third part of the duodenum crosses the midline at the level of the third lumbar vertebra and passes anterior to the aorta at the origin of the inferior mesenteric artery The

first part of the duodenum (choice A) lies horizontally to the right of the midline at the level of the first

lumbar vertebra The head of the pancreas (choice B) is to the right of the midline and extends from the

level of the first lumbar vertebra to the third lumbar vertebra It lies within the concavity of the

duodenum The neck of the pancreas (choice C) lies in the midline at the level of the first lumbar

vertebra It lies on the anterior surface of the aorta at the origin of the superior mesenteric artery The second part of the duodenum (choice D) lies vertically to the right of the midline and extends from the

level of the first lumbar vertebra to the level of the third lumbar vertebra

The left adrenaLvein drains directly into which of the following veins A Hemiazygos vein

B Inferior vena cavaee C Left renal veiri -

D Splenic vein

E Superior mesenteric vein

a

The correct answer is C The left adrenal vein and the left gonadal vein (either testicular or ovarian) drain into the left renal vein TheTeft renal vein t~ains intothe- inferior vena cava In contrast the right

adrenal ~~inandnght gonadal veindrai~ gLr~ctJy iQtoJhe iilferiQ[ Vencava -- -

ThehemTazygoS7ein- (~h-~i-~ A)~~c~i~es the venous drainage from the body wall on the left side of the

thorax and abdomen No visceral organs drain directly to the azygos or hemiazygos veins The inferior vena cava (choice B) receives the direct venous drainage from the right adrenal vein but not

the left adrenal vein Remember the inferior vena cava is on the right side of the abdomen The splenic

vein (choice D) receives the venous drainage from the spleen and part of the pancreas and stomach The splenic vein is part of the portal venous system

The superior mesenteric vein (choice E) receives venous drainage from much of the intestinal tract It is part of the portal venous system and joins with the splenic vein to form the portal vein

A 43-year-old man presents complaining of pain in the groin On examination his physician palpates a

bulge in the region of the superficial inguinal ring which he diagnoses as a direct inguinal hernia The hernial sac most likely

A is covered by all three layers of the spennatic fascia B passes medial to the inferior epi gastric artery

C passes medial to the lateral border of the rectus abdominis muscle

D passes posterior to the inguinal ligament E passes through the deep inguinal ring

The correct answer is B Direct inguinal hernias enter the inguinal canal by tearing through the posterior

wall of that structure The typical location for this type of hernia is through the inguinal triangle bounded

laterally by the inferior epigastric artery medially by the lateral border of the rectus abdominis and

inferiorly by the inguinal ligament Direct inguinal hernias pass medial to the inferior epigastric artery

whereas indirect inguinal hernias pass lateral to the inferior epigastric artery because the deep inguinal

ring is lateral to the artery Indirect inguinal hernias are covered by all three layers of the spermatic fascia (choice A) Direct inguinal hernias are covered by fewer than all three layers because the direct inguinal

hernia tears through one or more layers of fascia as it emerges though the abdominal wall The lateral

border of the rectus abdominis muscle (choice C) forms the medial border of the inguinal triangle All

inguinal hernias pass lateral to the rectus abdominis Femoral hernias pass posterior to the inguinal ligament (choice D) Inguinal hernias emerge through the superficial inguinal ring which is superior to the inguinal ligament Inguinal hernias that descend below the inguinal ligament pass anterior to the

ligament Indirect inguinal hernias pass through the deep inguinal ring (choice H) direct inguinal hernias

do not Both types of inguinal hernias pass through the superficial inguinal ring

During a gastric resection in a patient with stomach cancer a surgeon wants to remove the lesser

omentum because of tumor extension into it Which of the following structures lie in the free edge of the

l~~g omentum and consequently must be dissected out in order to be preserved

A Common bile duct cystic duct and hepatic artery 6

B Cystic duct hepatic artery and hepatic vein

e Hepatic vein and cystic duct

Portal vein common bile duct and hepatic artery

E Portal vein hepatic artery and hepatic vein

The correct answer is D The free edge of the lesser omentum contains three important structures the

common bile duct the hepatic artery and the portal vein Nei ther the cystic duct (choices A B and C) nor the hepatic vein (choices B C and E) lies in the free

edge of the lesser omentum

A 55-year-old male patient with chronic liver disease has portal hypertension To relieve the pressure in the portal system a porto-caval shunt is performed Which of the following veins may by anastomosed to

accomplish this porto-caval shunt A Left renal vein-left testicular veingt

B Right renal vein-right suprarenal vein I shy

e Splenic vein -left renal vein J

D Superior mesenteric vein-inferior mesenteric vein E Superior mesenteric vein-splenic vein

The correct answer is C The splenic vein drains directly into the portal vein The left renal vein drains

directly into the inferior vena cava Anastomosis of these veins would allow blood from the portal vein to

drain retrograde though the splenic vein into the renal vein and then into the inferior vena cava The left

renal vein (choice A) drains directly into the inferior vena cava The left testicular vein drains directly into

the left renal vein Thus these veins are already in communication and neither vein is part of the portal venous system The right renal vein (choice B) drains directly into the inferior vena cava The right

suprarenal vein also drains directly into the inferior vena cava Thus neither vein is part of the portal

venous system The superior mesenteric vein (choice D) drains directly into the portal vein The inferior

mesenteric vein drains into the splenic vein which then drains into the portal vein Thus neither vein is

part of the caval venous system The superior mesenteric vein (choice E) drains directly into the portal

vein The splenic vein also drains directly into the portal vein Thus neither vein is part of the caval

venous system

A 12 year old boy has fever vomiting and para-umbilical pain After examining the patient the doctor

makes an initial diagnosis of appendicitis Appendicular pain which is initially referred to the umbilicus goes to the dorsal root ganglion of

a TI b TI2 c L1 d T7

(e I TIO

A 59-year-old male undergoes a neurological examination which reveals that when the abdominal wall is

stroked the muscles of the abdominal wall of the side of the body stimulated failed to contract Other

neurological tests appeared normal The likely region affected includes

a CI - C5 spinal segments b C6 - TI c T2-TI ~T8-T12

e Ll- L5

The surgery done to relive portal hypertension is done by connecting two veins Which of the following veins would be suitable for connection

a Inferior vena cava and portal vein b Superior vena cava and portal vein c Splenic vein and right renal vein d Splenic vein and left renal vein e Superior mesenteric vein and Inferior vena cava

A mother brings her 3-week-old infant to the pediatric clinic reporting a new scrotal bulge that she found -~-

while changing a diaper yesterday The infant is afebrile Physical examination reveals a palpable mass in

the scrotum while in the standing position resolution of the mass in the supine position and no

transillumination of the scrotal sac What is the most likely diagnOSiS

a Cryptorchidism b Direct inguinal hernia c Hydrocele d Indirect inguinal hernia ~ e varicocele

The Vagal trunks enter the abdomen by passing through which of the following openings in the

diaphragm

a Right crus b Esophageal hiatus ~ c Vena caval hiatus d Aortic hiatus e Left crus

2 The anterior boundary of the epiploic foramen of Winslow is bounded by

a) First part of duodenum b) Lesser curvature of stomach c) Liver d) Hepato-duodenalligament v ~

3 The ilio-inguinal nerve is derived from

a TI2 ry b LI c L2 d L3 e L23

15 Surgically the structure used to suspend the kidney to the diaphragm is

a) Renal fascia b) True capsule c) Perinephric fat d) Paranephric fat

6 If there is portal obstruction because of carcinoma affecting the pancreas which of these of the

following signs would be present

a Caput medusae b Esophageal varices c Rectal varices c

d Pulmonary edema

7 In a sliding hernia the gastro-esophageal junction lies

a) At its normal position b) Below the normal position c) Above the normal position V d) None of the above

8 Which of the following structures is retroperi toneal

A transverse colon B spleen IJ2f6 C ileum D descending colon v r 1pound1111111

9 The renal angle is fonned lgtetween the 12th rib and ______ muscle

a Psoas major -middotshyb Erector spinae c Quadratus Iumborum d Diaphragm

10 The anterior structure at the hilum of the kidney is

a) Renal vein ~

b) Renal artery I middot~ I

c) Ureter d) Accessory renal artery

11 Because of origin of the muscle from the lateral one third of the inguinal ligament it

could not fonn the anterior wall of the inguinal ligament

a) External oblique b) Internal oblique c) Transversus abdominis_ d) Rectus abdominis

12 A large tumor mass impinges on the splenic artery and its branches as the artery pass out from below

the greater curvature of the stomach Branches o(which of the following arteries would most likely to

effected by the pressure on the splenic artery

a Left gastric b Left gastro-epipJoic c Right gastric d Right gastro-epipoloic e Short gastric_

13 A new born baby has projectile vomiting after each feeding It is determined that there is obstruction

of the digestive tract as a result of annular pancreas Annular pancreas is as a result of an abnormality in which of the following process

a Rotation of the dorsal pancreatic bud around the first part of duodenum b Rotation of the dorsal pancreatic bud around the second part of duodenum c Rotation of the dorsal pancreatic bud around the third part of duodenum d Rotation of the ventral pancreatic bud around the first part of duodenum y Rotation of the ventral pancreatic bud around the second part of duodenum

14 As the liver bud enters the ventral mesogastrium the region of the mesogastrium stretching from the

liver to the anterior abdominal wall is called

a Lesser Omentum b Greater Omentum ~ Falcifrom ligament d Lacunar ligament e Ligamentum teres of liver

16 A patient has absence of his 12th rib In such a patient if the doctor makes an incision to approach his

kidney mistaking the 11 th rib for the 12t he would end up injuring

Which of the following arteries is a direct branch of the gastroduodenal artery The

A right gastric artery

B left gastric artery

C inferior pancreaticoduodenal artery D left gastroepiploic artery

i E)right gastroepiploic artery --

E x pI a nation The right gastric artery is typically a branch of the proper hepatic artery The left gastric artery is a direct

branch of the celiac trunk The right and left gastric arteries anastomose along the lesser curvature of the

stomach The inferior pancreaticoduodenal artery is a branch of the superior mesenteric artery it

anastomoses with the superior pancreaticoduodenal in the head of the pancreas The left gastroepiploic

artery is a branch of the splenic artery it anastomoses with the right gastroepiploic artery along the greater

curvature of the stomach The right gastroepiploic artery is a branch of the gastroduodenal artery The

other branch of the gastroduodenal artery is the superior pancreaticoduodenal artery

Which of the following pairs of veins join together to form the portal vein The

A superior mesenteric vein and inferior mesenteric vein

B inferior mesenteric vein and splenic vein

C superior mesenteric vein and splenic vein

Ip)splenic vein and left gastric vein E superior mesenteric vein and left gastric vein

Explanation

The portal vein is formed behind the neck of the pancreas by the union of the superior mesenteric vein

and the splenic vein The inferior mesenteric vein drains into the splenic vein The left gastric vein drains

directly into the portal vein After the portal vein forms it enters the hepatoduodenalligament of the

lesser omentum to reach the liver The portal vein is the most posterior structure in the hepatoduodenal

ligament

At which of the following vertebral levels does the duodenum pass anterior to the aorta - _- shy

All ~

B L2 7~

CL3 I

~DL4

E L5

Explanation

The duodenum begins at the pyloric sphincter at the level of Ll The second (or descending) portion of

the duodenum is to the right of the aorta and extends inferiorly from the level of Ll to the level of L3 The third part of the duodenum crosses the aorta from the right side to the left side at the level of L3 The

fourth (ascending) portion of the duodenum extends from the level of LJ to the level of L2 The

duodenum ends at the duodenojejunal flexure The superior mesenteric artery passes anterior to the

duodenum as the duodenum passes anterior to the aorta The duodenum can be constricted at this level

In which of the following locations will perforation of the digestive tract result in the spilling of luminal

contents into the - lesser peritoneal sac

A Anterior wall of the second portion of the duodenum B Posterior wall of the second portion of the duodenum

C Anterior wall of the stomach

~Posterior wall of the stomach E Posterior wall of the transverse colon

Explanation

The posterior wall of the stomach is related to the lesser peritoneal sac The anterior wall of the stomach is related to the greater peritoneal sac The anterior wall of the second portion of the duodenum is related to the greater peritoneal sac The posterior wall of the second portion of the duodenum is related to the retroperitoneal space The posterior wall of the transverse colon is related to the greater peritoneal sac

The ureter lies against the anterior surface of which of the following muscles shyA Crus oftne diaphragm B Quadratus lumborum

0 Psoas major D Transversus abdominis

E Iliacus

Explanation The ureter exits the renal pelvis at about the level of vertebra L2 As it descends along the posterior abdominal wall it lies on the anterior surface of the psoas major The psoas major muscle arises from the bodies of the lower lumbar vertebrae The psoas major muscle is joined by the iliacus to fonn the

iliopsoas muscle The iliopsoas muscle then attaches to the lesser trochanter of the femur and is the major

flexor of the hip

As the right ureter passes the pelvic brim it lies against the anterior surface of which of the following

blood vessels

A Gonadal artery B Inferiorvena cava C Internal iliac artery

rJ- External Iliac artery

E Inferior mesenteric artery

Explanation

The ureter lies in the extraperitoneal space in the posterior abdominal wall Alter leaving the kidney it

passes inferiorly on the anterior surface of the psoas major muscle At the pelvic brim the ureter passes

into the pelvis At this point the common iliac artery is dividing into the external and iliac arteries The

ureter lies on the anterior surface of the external iliac artery immediately distal to the bifurcation This is a useful landmark for a surgeon to locate the ureter

When extravasated urine passes from the superficial perineal space into the anterior abdominal wall it is

found immediately deep to which of the following layers of the anterior abdominal wall

-ltScarpas fascia

B External oblique muscle

C Internal oblique muscle D Transversus abdominis muscle

E Transversalis fascia

Explanation

The superficial perineal space is bound by Colles fascia the fibrous portion of the superficial fascia This

layer of fascia is continuous with Scarpas fascia the fibrous portion of the superficial fascia of the anterior abdominal wall Therefore urine that is deep to Colles fascia will remain deep to Scarpa s fascia The urine will spread in the plane between Scarpas fascia and the external oblique layer

When a horseshoe kidney develops the ascent of the kidney is restricted by the A internal iliac artery B external Iliac artery

C common iliac artery

inferior mesenteric artery

E superior mesenteric artery

Explanation

A horseshoe kidney develops when the inferior poles of the to kidneys fuse together as they ascend into

the abdomen from the pelvis The first anterior midline vessel that is encountered by the horseshoe kidney

is the inferior mesenteric artery This artery prevents the kidney from continuing its ascent

The left testicular vein drains into which of the following veins

A Left internal iliac vein B Left common iliac vein

bflnferior vena cava D Left renal vein I

E Left internal pudendal vein

Explanation

The left testicular vein drains into the left renal vein The right testicular ~i~[~nsltjectlY into the

inferior vena cava This difference in venous drainage is believed to explain the greater incidence of

varicocele on the left side than on the right The venous drainage from the penis is to the internal vein

which then drains into the internal Iliac vein

The spinal nerve that provides cutaneous branches to the skin around the umbilicus is

A TS B TW-shy

C TI2

DL2 EtA

Explanation

The tenth intercostal nerve is the anterior ramus of the TIO spinal nerve After passing through the tenth

intercostal space the nerve continues forward in the anterolateral abdominal wall in the plane between

the internal oblique muscle and the transversus abdominis muscle In the abdominal wall the nerve innervates to the abdominal wall muscles as well as the skin and the parietal peritoneum The umbilicus is

a useful landmark for the region of distribution of the tenth thoracic nerve

The ligament of the vertebral column that resists its extension is the Aligamentum flavum

B supraspinous ligament

C posterior longitudinal ligament

D anterior longitudinal ligament

E interspinous ligament

Explanation

The ligaments of the vertebral column that resist flexion of the column include the supraspinous ligament

interspinous ligament ligamentum fiavum and posterior longitudinal ligament The ligament that resists

extension is the anterior longitudinal ligament This longitudinal ligament is very broad and strong It

covers the anterior and anterolateral surfaces of the vertebral bodies and the intervertebral disks In

addition to resisting extension the anterior longitudinal ligament provides reinforcement to the anterior

and anterolateral surfaces of the intervertebral disk The posterior longitudinal ligament is relatively

narrow and covers the posterior surface of the vertebral bodies and the intervertebral disks This ligament

reinforces the posterior surface of the disk The posterolateral surface of the disk is not reinforced and it

is through this region that herniation of the nucleus pulposus usually occurs

A patient presents with epigastric and right upper quadrant pain The pain is most intense 2-4 hours after

eating and is reduced by the ingestion of antacids The patient states that he has passed black tarry stools

(melena) within the last week Fiberoptic endoscopy reveals a yellowish crater surrounded by a rim of

erythema that is 3 cm distal to the pylorus Accordingly an ulcer has been identified in the patients

A fundus

B antrum

C duodenum

D jejunum

E ileum

A number of physiologic genetic and other factors increase the risk of gastric (and duodenal) peptic

ulcers The evidence that H pylori plays a principle role is compelling Smoking and caffeine are known to adversely affect the morbidity mortality and healing rates of peptic ulcers In general first-degree

relatives of peptic ulcer patients as well as males have a threefold to fourfold increased risk of developing this disorder Paradoxically in gastric ulcer disease acid secretion is not elevated It is possible that

excess secreted hydrogen ion is reabsorbed across the injured gastric mucosa In general a defect in gastric mucosal defense is the more important local physiologic

A patient presents with symptoms of duodenal obstruction caused by an annular pancreas Annular pancreas is caused by

A rotation of the dorsal pancreatic bud into the ventral mesentery B rotation of the ventral pancreatic bud into the dorsal mesentery

fJ failure of the major and minor pancreatic ducts to fuse ~ ~ cleavage of the ventral pancreatic bud and rotation of the two portions in opposite directions around -the duodenum E formation of one pancreatic bud instead of two

Explanation Normally the ventral pancreatic bud rotates around the gut tube to reach the dorsal pancreatic bud The two buds fuse to form a single pancreas and the distal portions of the two ducts fuse The ventral pancreatic bud forms the inferior portion of the head of the pancreas the uncinate process and the major pancreatic duct (of Wirsung) The dorsal pancreatic bud forms the superior part of the head the neck body and tail and the minor pancreatic duct (of Santorini) Annular pancreas is the result of the ventral pancreatic bud dividing into two portions before it rotates into the dorsal mesentery Each portion rotates in opposite directions to get to the dorsal mesentery thus encircling the duodenum The presence of annular pancreas can constrict the duodenum thus obstructing its lumen

In n _ phranlc----

Gon ~l ----_1 Lum bltano

~~--- CornmQ1t bull ac

+-~4--- lnlllirnaJ ilic

xtem iliac

OBJECTIVE - Identify the blood supply to each of the structures listed in the table on the previous page

Ill give you a head start

FOREGUT - Supplied bV Celiac Tru nk (T12)

Proper hepatic

GastiooUod 13Jafter

1nferlor pancreaticoduodenal artery

Common epatlc

Lett gas ric iiirtery

Spfen artery

shy Gastroepiphgtic artery

~ Superior mesenteric 8rtfry

~

1 Esophagus is a derivative of the foregut so its blood supply originates from the celiac trunk

(T12) The predominant blood supply to abdominal portion of the esophagus is the Esophageal

A (Branch of L Gastric) The venous drainage of the esophagus is particularly important because

it is 1 of 3 clinically relevant sites of Portal Caval anastamoses The Portal Esophageal Vein

meets the Caval Azygos System Persistent bleeding manifests as Esophageal Varices - a fata I

condition

2 The Stomach is also a derivative of the foregut has EXTENSIVE blood supply and is very high

yield on anatomy exams The lesser curvature is supplied superiorly by the L Gastric A (1 of 3

major branches ofthe Celiac trunk) and inferiorly by the R Gastric A ( a branch ofthe proper

Hepatic A) The greater curvature is supplied superiorly by the L Gastroepiploic A (a major

branch of the splenic A) and inferiorly by the R Gastroepiploic A

The Short Gastric arteries (branches of Splenic Artery) supply the fundus of the stomach and

are referred to as EIID ARTERIES because they have no collateral blood supply Therefore if the

splenic artery were occluded (ex - increased pressure in the ommental bursa) - there would be

ischemia to the fundus of the stomach Venous drainage of the stomach is extensive via various

veins lead ing to the portal system Posterior to the stomach the IMV joins the splenic V which

joins the SMV to form the PORTAL VEIN ADAMS

3 Duodenum blood supply has high clinical relevance because it is the junction of the foregut and

midgut and therefore is the site of anastamoses between branches ofthe Celiac Trunk (main

foregut artery) and the Superior Messenteric Artery (main midgut artery) The Proper hepatic

artery gives off the gastroduodenal artery which travels behind the 1st part of the duodenum

This point has high clin ical relevance because duodenal ulcers are very common and a posterior

rupture of the 1st part of the duodenum could rupture the gastroduodenal artery causing

traumatic abdominal bleeding The Gastroduodenal artery first gives off the R Gastroepiploic A

(mentioned above) and proceeds as the Superior pancreatico duodenal artery (supplies the

pancreas and duodenum) which anastamoses with the inferior pancreatico duodenal A (branch

of the SMA) This is the junction of foregut and midgut and occurs near the opening of the

bil iary system into the duodenum (ampula of vater) Portal venous drainage here is responsible

for delivering nutrients from digestion to the liver for metabolism Appreciate that the Superior

mesenteric artery (artery of the midgut) branches from the aorta at Ll travels posterior to the

pancreas than moves anteriorly (at the jxn of the pancreatic headbody) and comes over the

3rd4th part of the duodenum Tumor of the head of the pancreas can compress the SMA

4 Jiver blood supply is via the common hepatic artery (major branch of the cel iac trunk) The

common hepatiC becomes the proper hepatic gives off the R gastric A and the Gastroduodenal

A and then joins the common bile duct and the portal vein in the portal triad Clinical- if a

patient were bleeding from the hepatic A a surgeon can stick his fingers in the epiplOic foramen

and squeeze the free edge of the hepatoduodenalligament in order to stop bleeding to the

area Please note that the hepatic a branches into Rand L hepatic A The Right hepatic artery

gives off the cystic artery which supplies the gallbladder Afferent venous supply is via the

Portal vein which is bringing nutrient rich blood to the liver After metabolism takes place

venous blood leaves the liver through the hepatic veins into the IVC PLEASE UNDERSTAND THE

RELATIONSHIP OF THESE STRUCTURES - ADAMSNETIERSNH Etc

5 Pancreas - Head is supplied via the superior and inferior pancreaticoduodenal arteries

(mentioned above) The tail (situated towards the hilum of the spleen) is supplied via the

pancreatic branches of the splenic artery (END ARTERIES) This blood supply is very important

because the endocrine Alpha and Beta Cells from the pancreatic islets of lagerhans are located

towards the tail This is where Insulin and Glucagon is released to the blood

Now complete this for mid and hindgut structures Make sure to note clinically relevant arterial

anastomoses as well as portal caval anastomoses FYI Appendix blood supply SMA + IMA

anastamoses marginal artery Portalcaval rectal veins fhemmorhoids) and periumbilical caput

medusa are high yield THE BUTT THE GUT and THE CAPUT

Abdominal Development

Liver

Ij1f

II wall b

oh liN ~ VltJrti n be- bull

Pancreas

Secondary Retroperitonealization e I~tl r 1 a v-mtrai m ellter

Rotations of the Gut I i Ij (lIl1UtIJ f~ l r tilt

()l td 10 me l-ft and he v

--~--- -~ -~-~

i

I AolaijonjoI~guf I

STOMACH BED (IDENTIFY IN ADAMS)- the structures posterior to the ommental bursa which

support the stomach in the supine position

Abdomnal JQrUI

Splnic vein

OmQ-oul tv~ ) O(s(Jroa)

Lojt(r o m nturrt (hpJtodu o d~n31 Hid

Gadrl)SplerH (g3stroll~nal) IIgam~nt

hiad h~~atogrtricent IIQdmiddotcrt~)

Lt Dome of Diaphragm (why left Look this up in Adams)

Spleen (What is the blood supply)

Left Kidney (What is the blood supply - AND how is it different from the R kidney)

Suprarenal Gland (What is the Arterial AND Venous Blood supply - how are they different)

Pancreas (How does supply differ from Head to Tail What is the SMA Relationship)

Transverse Mesocolon

liver - ADAMSWET - Make sure you look at the liver in wet lab

Left triangular nl1am~nt

ComoaDj ligamnt

Erophg~1 impre$ioo

Hepatio veins

In1erior -ifena middotr3)Ia

Fibrous appendix o-t

live

impr~j on

Heprorendl p~rtion of Q)(Qllary ligament

Righllri~n9ul r 1I~met

(Common) bile quol

Gr)mmCtr~ hepatic dlJct

Ccentic duct

Duodenal impression

GaJdate p-fr)~S

Hepatic artgtrl prop-f iiiiila - Faloiform ligament

_ - shy Round ligamen liver

~--F-- CoJio imprgt-ssi-on

Prta heptis

Identify the lobes impressions and embryonic remnants associated with the liver

Caudate Lobe Quadrate Lobe Right Lobe Left Lobe Round ligament Falciform Ligament

Ligamentum Venosum (what is its fxn in embryonic life) Hepatic Veins (NOT PART OF THE

PORTAL TRIAD) IVC PORTAL TRIAD - Contents relationship cross section etc Know the

Galbladder relationship to the lobes of the liver

Biliary Duct System - Make sure you understand the sequence of these structures - BE ABLE TO

DRAW A FLOW CHART

TPVd i

t

I t

1 __ Cm-(r

patk GlJet

I

J

Clinical = JAUNDICE is caused by anything that prevents delivery of bile to intestine Tumor of the

head of the pancreas Stones etc Patient will have pale stools and yellowish colored mucus

membranes

Clinical- Any scenario that tells you the patient has BILLOUS VOMIT means that the obstruction to

the flow of digestive contents is after the Ampulla of Vater (Site of Entry of Billiary system to the

duodenum) - ie Duodenal Atresia

Spleen -located posterior to the mid axillary line between ribs 9 and 11 Make sure you know that

the 10th rib is the main axis of the spleen and this organ is susceptible to injury (stab wound errant

thoracoce ntesis etc)

The spleen is derived from mesodermal cells - NOT THE GUT TUBE

The spleen rests on the left colic flexure associates with the tail of the pancreas Know the

structures entering the Hilum of the spleen

Sh rt O~-t~ic 1 0(0 10 rtiltSPIric Iloa nt

(cut)

Peritoneum - similar concept to Pleura - think of a fist in a balloon

Visceral Peritoneum - Layer of balloon touching your fist

Parietal Peritoneum - Layer of balloon not touching your fist

Your fist represents the organ your wrist is the hilum and your arm contains the blood supply

entering the organ

Appreciate that there will never be organs in the peritoneal cavity - rather these organs invaginate

the cavity Kaplan videos

RULES OF NOMENCLATUREshy

1 Organ completely surrounded by peritoneum - peritoneal organ

2 Organ partially surrounded by peritoneum- Retroperitoneal

3 Peritoneum surrounding peritoneal organ is VISCERAL peritoneum

4 Peritoneum surrounding retroperitoneal organ is PARIETAL peritoneum

5 Peritoneum connecting visceral to parietal is called messentary 2 messentaries in the

gut Dorsal (to the gut tube) and ventral (to the gut tube) messentary

Aorta is in Retro peritoneal position - but blood must reach peritoneal position - vessels travel through

messentary All peritoneal organs will have blood supply reaching through messentary

-Mesentery is a 2 layer peritoneum with a neurovascular communication between body wall and organ

- Ligament connects one organ with another or to the abdominal wall (Ommentum = ligament)

lesser Ommentum (attach lesser curvature of stomach and duodenum to liver) =Hepatoduodenal

Ligament and Hepatogastric Ligament

Has a Superior and Inferior Recess (Accumulation of Fluid in Ascites)

Communicates with the greater sac through the epiplic foramen (what structures pass through

this foramen)

Boundaries - you must be able to visualize this

o Anterior - stomach

o Posterior - parietal peritoneum pancreas

o Superior - superior recess (bw diaphragm and coronary ligament)

o Inferior -Inferior recess (bw layers or greater momentum

Greater Ommentum (attach greater curvature of stomach) Gastrophrenic ligament Gastrosplenic

ligament gastrocolic ligament

The greater omentum is the largest peritoneal fold It consists of a double sheet of peritoneum folded on itself so that it is made up of four layers The two layers which descend from the greater curvature of the stomach and commencement of the duodenum pass in front of the small intestines sometimes as low down as the pelvis they then turn upon themselves and ascend again as far as the transverse colon where they separate and enclose that part of the intestine

ABDOMINAL PAIN

Parietal Peritoneum - supplied by same vasculature lymphatics and nerves supplying body wall it

lines and diaphragm Sensitive to pain pressure heat cold well localized

Visceral Peritoneum - supplied by same vasculature lymphatics and somatic nerve of organ it covers

Insensitive to touch heat cold and laceration - referred to dermatome of spinal ganglia providing

sensory fibers Where does appendicitis refer to

Foregut pain - epigastric area (ie - cholycystitis)

Midgut pain - periumbilical area (ie - appendicitis)

Hindgut Pain - suprapubic area (ie - diverticulitis)

Extra ImagesConcepts

ll~_____-

FalifCtrm ligament oind r~ud ligamet f Ilver

Blood from splenio gastriC and inferiof rne$e-rteri v~ins

Ca-I tributaries

Lett gastrio Ifein

Posterior superior pan~reatioodul)denal vaihS

Lott gamo-om~nlal (9aropip lomiddotic) -in

Poq_~ tjol imerl-9-r panCJertlcorllJod-nal veiopound --amp----I- - ~J Right grtr~-omntal

Anwrior interi (gartroepiploic) Jjn

pan euaii cod vl)denal veins middot Inf~Ji (t r mesentric vein

Miqdle (olic vein

Right cl)licvein Sigmoid and rectosigml)id (ei ns

IhH)Collc(~io

--- Mi~dl laquooLJl gtjrltgt

PoM ca vl1 illasto)moses -----shyampoptoageal 2 Paraumbilie-lt11 Inferi or Fectal vei ns

3 Recial 4 REuoperHonea1

Know how the Portal vein is formed I 4 sites of portal caval anastamoses and 1 clinical shunt

Col li t ltt-~ otTl~tI ~nj pc~ 1lt1 turJoG

Ltf 14i1 tImiddot~ artoftl9 on tj phtAt$

L-oftqf 4t t~r 1=laquoIran d 1 bull shy~p l ci rj o fOOOts

Nerves follow the arteries - appreciate the splanchnic nervous system I

Uet~ric branch of left ~nal art

Ureterie branch of righi renal artelY

Left Zld lumbar in and co mlTlunication to as)erdin9 lumbar l(~in Hi ~ht tEZ1~~t~ t3r j t itn ~ nJ l1t- rlnd lfe i r1

Inferior me5nteri~ artery

Notice that the right testicular vein drains directly into the IVC and the right testicular artery drains

directly into the aorta However the left testicular vein drains into the L renal vein at a right angleshy

reason left testicle is lower and more susceptible to varicocele (bag of worms)

Also notice that the left renal vein has a longer course because the IVC is on the right side whereas

the right renal artery has a longer course because the aorta is on the left side

Appreciate the anterior to posterior relationship of structures in the hilum of the kidney - VAP - Vein

Artery Renal Pelvis (Ureter)

11____ __ L_ L_ n VJ __ _ _ t_L I I_ _ L __ L_ I -pound1 bull LI_~-I ____

Posterior View of Head of Pancreas in ( of Duodenum

Celiao hunk

Co mmon ~L~jJth art~ry

GastNduQdonal artrf (partilly in phantn)

P1)Sterior $Up~Jior panCflaticuduodfmal art~r~t

(Co mm on) bile duct

middot~1t~~t-1l---~-~- Right gshomiddotomental (gastoe plp lolc) 3rte (phantomost)

Grener paocre atic art-ry

1n1~rjor pancr-iatlc artery

Jtrifll supejo r pal)oreailcento)dJodenal artr1 (phantom)

Anastomotlo branch

POostetlor bJanch of jo f~ri of pan-reatir(lduodensl drttnj

Anterio r branch of i flferior palcreati~)duodenal art~(phan1om)

Notice the extensive blood supply to the pancreas and duodenum via the branches of the celiac trunk

Notice collateral supply from SMA branches - makes sense bc this is the jxn of foregutmidgut

Identify the vessels in this arteriogram

Hiltid i)f N~ck oi B)dvof Tail 01 pa nereas pan cent~as P-nmiddot-reas panCtCas

I nferie v~na cava

jHept1iic p(lrlai v~in

Port1 tnd H~pti lt a ftH prol

Comm on) bll duct

Ouodtnum

~ft colic (sio)Atta~ hmtrlt jt~xJr-ofha~elSe

muo(IIQn

Right ~lIc (h~j)tic)

il~gtture

In1triol m~oten lIein (rttr op~ritoMdO

SlJp efl or mes~n~fiC amrV and lipln

KNOW YOUR NEIGHBORHOOD

Questions

vVhiJh structure supplied by a bnmdlof the cclia( artery is not derivcd from foregut LemCJUCrITI

(A) Head of the pancte-a5

CD) Pyloric duolenum

Cystkduct

( Liver hepatocyt~~

~F) Body of the spleen

An infant presents with an omrhaJucele at birth -hi oJ the [oHm illg applies to his cM1-dition

(A) It is 31so seen ill p4titnts with aganghonic megacolon

(11) ft reuirs from a fal1ure of resorption of theviteUine d let

(C) It results from herniation at the-site of regression of the right umbilk vein

DJ It is caustd by faihtrc of recanalization of the midgut part of the duodenum

~ It ill camioo by a failuIt vf the midgul to return to the abGQminal uity after herniashytion in-n the urnbilk s l stalk

Ot er than the spleen occlusion Cif the spit-Ilk artery at its odgin wm most likely affect die blood supply to jllch st cnud

(A) Jejunum

(B) Body of th pal1~lltas

(C) LeSStT Cllmiddotlaturc of tl )toma-ch

(D Duodenum dista to the entrance of the Ornmou bile duct

E Fundus of the stomach

A 38-yeu-old batL~er with a history of heartburn suddenly experiences excluciating pain in the (plgastric region of th~ abdomeu SurgCry is perf~rme immediard y upon admisshysion to the 1IlcrgCJliy tuomh~re i~ evidence uf a ruptured ulcer in the posterior waU of the stomach Vhere will a surgeon first fi nd the stomach contenlSf

A) Greater p4ritoneal sac

rB) Cul~de-s~c of Douglas (--

C Omental bursa ~

--D) Paracolic gutter

rEj Between -he panttal perimltum and the posterior body wal1

At birth an infant presents with a st()ma~ rb~tbas~njJled jfltotb~diaplfagru 1A1ltre is the defect thatresulied iiitJle heini~t()n shy~tsophagealbiatus

7 - rH-- Hiatus for the inferior vena cava

( Pleuroperitoneal membrane -(0) Septum transvcrsum

(E) Right Crlt~

An infant born with DOVv7l syndrome presents with bili()u~ vomiting Ahat congenital defect does the infant have

(A) Pyloric stenosis

(B) Meckel diverticulum C) Ornphaloce1e

(D) Gastroschisis

( ~ ) Duodenal atresia y A patient with cirrhosis of the liver presents with ~ bacalvaricestnlreased retrograde pressure in which veins caused the varices

(A) Paraumuilical

(B) Splenic

(ct AzygltJus

(15))G~trk ( (-F) Superior mesemeric

A htaltby 3-year~old male patient experiences a hernial sa protruding from the anterior abdominal wall about halfway between me anterior superior ilia spine and the pubk tuberde Pulsations of al1 artery are palpated medial to the protrusion site through the abdominal walL Which layer of the anterior abdominal wall will first be traversed by the

1hctma

fA) Rectus sheath (B) External oblique aponeurosis

(C) Inguinal ligament

lD) Transversalis fusda

(E) Cremasteric fa~cia

After 5urgi(aj ffpair of a hernia the patient tXperienccs mtmlgtness in the skin on the anteshyrior aspect of the S(Totum_ Vhaf nerve may have been lesioned during thehemiorrhaphy

(A) Femoral

(B) Obturator

(C) Ilioinguinal

(D) lliohypogastrk

(E) Pudendal

A 23~year-LJld female secretary il1 good health ~-uddcn1) doubles over with pain in the a ea of the 1JmbRicu$ Sbe feels vartn and ltneasy and has no appetite That night the pain seems to have mQved to the tower right abdominal regjol1 and she calls her family doctor who then arranges for an ambulance to pk-k her up and take her to the hospitaL Wh ell ntn~ perceived in the area of the urnbilirus most Hkely carried lhe pairfu I sensations into the eNS

tA) Vagus nerves I~

V B)

) Lessersplanchnk nerves

tC) Pudendal nerves

(D) lIiohpogastrk nerves

(E) Greater splam ic l erves

A CT reveals carcinoma in the bOod of the ancreas Vhich blood vessel trut ourses ----~- - -bull ------ --shy

immediately poftterior to the body ofthe pancreas is the m~t likely to be oompressed

(A) Splenk artery

(B) Abdominal aorta (C) Portal vein

(1) Splenic vein

(E) Renal vein

A patient has a penrln1l1ng uker of the posterior wall ot the br~l part ot the (lUooenmn llkh blood vessel is subject to erosion

(A) Common hepatic artery

(B) Gastroouodenal artery

(C) Proper hevatic artery

(D) Celiac artery

(E) Anterior inferior 11amrelltlcoduodcnal attery

Your patient has been diagnosed -ith a carcinoma locallted to the head and l~e(k of the pancreas Another clinical sign would be

A esophageal varices

(8) hemorrhoids

C) a caput medusa

(D) increased pra Teuro n th~ hepatic veins

(E) enlarged right supra lavkular lymph nodes

Wltkh of the foUowing structures develops in the ventral mesentery

(A) Spleen

(B) Jeiunum (C) Head of1ht pancreas (D) Transverse colon (E) Stomach

ti l Uw ~ littwin~ f( S-t lil oai Imdge ~ hi(h or tbt la~)d J truetur tgt liJ llntn nl) he hl p UC iJd [IIi ell

c o

A) drains Ie tht infCrior a La aI

R t middot~nfl0 ~ill to th~ lunlgtn of h i dtlndCrlllfH

(e) m t bull JiJattd on tl l J n T ~H

D ) sup Lc O VSlt I Hlid bhtu l 1 li - -I un oid

( ) U~tpli(t tr j middottUh~ 1 v(( b~nt rfK n1ilc~Zm

ANSWERS AND EXPLANATIONS

Answer E The spleen is t hlttnopodicand lymph organ demlted from mesoderm

Answ~ R Al1 tlmphalocele is caused by it failure of the nlidgut to return to the ahdomir nat cavity after herniation into the umbiliau Stalk Choices Aand D maybe seen in infants with Down syndrome choice D ~s the specific CBuse ofduudcnal JtiCSitt Choice C is (ile cause of gclstrosbisis and Choice B nsults iu a Meurolktldivertku1-tlB

Answer B The fundus ofthe stomach is suppHed by soort gastric brunches of the splenic altery The splenic artery supplies the body and tail of the pancreas part of the greater curvature of the sttmla(h and the spleen Te jejunum part of the head of the pancreas and tht~ duodenum distal to the entrance of the commOll bile duct are supplied by the superior mesenterk artery clll~l ~be less r ctlt1ature cmd the pylQric antrum are supplied by the right and lei gastric art(ries

AnSWftt C Tbeomental bursa or lesser ~ritoneaj sac lies direcdy posterior to the proxshyimal part of the duodeTtlm and the stomach and would be the first site where stomach contents ~Ott1d be fpoundluncL

Answer C A defect in a llleuropcritoneal membrane (uswlly the left) is the typical site of i1 cc-ngenitlI diilphragluatic hemia llere the membr4ne fails to dose ()pound( of the perishycCirdiopcritulleal canals

Answer E DuoJenal atresia and aganglionic megacoion are congwitaI defects S~Il in patients with Dowmiddotnsyndrome

Answer D RulaTgemt~llt of and retrograde flow in g~lstrk vel_ns in particlJl~r the kft gas~ tricveins dilates the capillary bed in rhe wall of the esophagus in (ases of porta yper~

tension Blood flow would increase in and dilampte tribntarkgts of the (lZygOUS vein on the other side of the capiUary bed but flow in this vein is in the typical direction t()ward the superior vena cava Paraumbiii(ltU vein eilgorgement contributes to a caput medusH Splenic ~nlargement might prc~nt with 5plcnonlegaly and balt-kflow in to tlu superior m~~ntclic vein occurs but is asymptomatic

Answer D The patient hagt an indirect inguinal hernia whi~h emerges from the antt-rior abdominal wall through the deep inguinltilling Theeep ring is a fault in the transv~rshysaUs fascia this I~yer wiIJ be penetrated first by the hernia

An~Wer C The ilioinguinal nenc which provides sens~llion to the lnedlal thigh ltmclanteshytior SClotunl pass~lt th rough the 5uperfh_ial inguinal ring ind $subject to inj i1T) becaus-e

it is in the operatitm Held of the erniorrhapny

Auswer B The leMHr splanchnic nerves are sympathdic nerVlts that carry viscera l sensashytlltgtrogt ftom illtllt1m~d ()J stietched gust (itinteitinal ~tructures (in this case the pprndix) into tnt eNS Lesser splanchnic ntTYcsarisc from thmiddot T9--T12 spinal cord segments lt1nd provide sympathetic innenation tD rnidgut siruc1ures whiCh include CLe app~JldD Viscera] Pain arising from affecLed Inidgut ampt 1C1ure is referred over the same dl- matorne~ of spinal segrnertts v-hich provide the sympathetic Innervation n this G1SC of appendicitis the invohen~n t of the ltire) of t e unlhHku indud s the T 10 dermatome

Answer B Of the five choices onty the dscending olon is retroperiton~al aldwould be a lik ~ ( choice to be seen immediately a(~jilcent to t11e posterior abdominal middotn~L

Amwen D The SpltftlC ~-ein ourses posterior to the body of the panneas m its way tt drain into the superior mCSfttltlri( vein

Answcr B TILt glstrodllolticnal artery 1 direct hIamh of the comrootl hepatic artery courses immediately pt))iwri() to the duodenum and is slbject to erosion

Answer B Carcinoma of th pan middott3S in the 1tilt1 may compreampgt the portltil vein at irs orishygill The poTtai vcin is fomled when the splenic vein jQiaswith tfie superior meStllt eric vein The inferiot mesenteric vein joins the ~plenjc vein just priOT to tlli~ point at which the splenic joins the superior Jlleit1ltcri( vein Increescd venous presslu in the inferior mesenteric vein is a cause of emo hoid~

Answer C The- velltral pancreas wilich forms most of the head of the p ~ncr as develops in the ventral mes(ntery as antutgrowth of the hepatic diverticulum Th~ hepatic divershyticulull induding the biIJary appa~atus develops in tbe ventral mesentery of the foregut

Answer~ A The superior mesenteric ~in joins with the spienkvein to form the hepatic portal vciu

Answer D The structure at gttlK is the proper hepatic artery~ whkh suppUesoxygenated b middotood to the liver

MAKE SURE YOU KNOW the diff bw Rectus Sheath above and below the arcuate line

ABOVE

Aponeurosis of xiiltmal obllque musclo

Extemll f)biquw musde

Reotln ilbdomlnls musole S~in

Internal 9bliquQ mY~QI

AponeUfOsi$ of hJH$V~~S Lir9a a lb lbdolTlin~ musolo Tri OJV6 rUi

atldomlnis mUS(loe

Sub cutanlilous tiue (tatty ye r)

BElOW

A POrl lJfosis 01 etemal oblique muscle

Aponeul~)sis 01 Internal oblique mU$cl~

Anteriol lay~ of r~ltdus st~ath EXttom1 oblique rnu$cll

Rectus Jbdominis muscle Intoernal Aponeurc-sis of tra~fersU$ oblique muscle-

at-domlnis muscentl ~ Skio

Tra nsvitSus abdomioLs ml)ZClt

TralSVersaHs fascia Medial umQil iegtt1 1i9Jment -and folj

Uldchus Peritoneum (ir median Umbilj~al Suboutane ous

Extraprftone 11ascia

Ymbilimiddot~1 fold)

preu9poundiea1 fascia

tissue (fatty 4nd m~mbr3n(iUS layers)

o Above the arcuate line (A horizontal line 13 of the distance bw the umbilicus and the

pubic symphysis) -10 Aponeurosis divides into an AntPost Laminae

o The Ant Laminae joins EO and Post Laminae joins Trans Abdominis = Ant and Post

RECTUS SHEATH respectively

o BElOW the arcuate line - all 3 aponeurosis join ANTERIOR to rectus muscle to meet its

counterpart in the midline (linea Alba)

o Take away Msg - The abdomen is devoid of a posterior rectus sheath below the

arcuate line and is therefore more vulnerable to herniasinjuries

Question - A physician makes a deep incision in the patients midline immediately superior to

the pubic symphysis which of the following layers is his knife least likely to pass

Rectus Abdominis External Oblique Ant Rectus Sheath Posterior Rectus Sheath All of the

Above

Answer - All of the above None of the other answer choices are midline structures -LINEA

ALBA

Linea Alba has very poor blood supply - doesnt heal well after surgery Therefore this is a

common site for incisional hernias

a Spleen b Transverse colon c Descending colon d Stomach e Pleura

17 Meckels diverticulum is normally found 2 feet proximal from the

a Pyloric sphincter b Lower esophageal sphincter c Ileo-cecal valve d Middle valve of Huston e Anal valve

18 Ulcer in the posterior wall of the first part of the duodenum would erode ___ artery and would cause bleeding

a Left gastric b Right gastric c Hepatic artery proper d Gastroduodenal artery e Middle colic artery

19 An inflamed appendix is identified by a surgeon on the operation table by noting

a The appendicies epiploicae b The convergence of tenia c The artery of Drummond d The mesocolon e The mesosalphinx

20 The nerve which emerges through the psoas major is

a Femoral b Ilio-inguinal c Ilio-hypogastric d Pudendal e Subcostal

21 The right gonadal vein drains into the

a Azygos b Hemiazygos c Inferior Vena Cava d Right renal vein e Left renal vein

22 The hepatocytes in the liver is derived from

a Ectoderm b Endoderm c Mesoderm

d Neural ectoderm

23 Abscess in the lumbar vertebrae due to tuberculosis would spread to the adjacent muscle which is

a Psoas Major b Iliacus c Quadratus lumborum d Tranversus Abdominis

24 The anterior wall of the inguinal canal is formed by

a External oblique and transverses abdominis b External oblique and fascia transversalis c Internal oblique and external oblique d Internal oblique and transverses abdominis e Fascia transversalis and peritoneum

Meckels diverticulum is a result of which of the following developmental abnormalities shy

A Failure of the vitelline duct to close

B Failure of the herniated intestinal loop to retract into the abdomen

C Failure of the urachus to close

D Failure of the midgut to rotate

E Failure of the hepatic duct to close

Explanation

Meckels diverticulum is a result of the persistence of the proximal part of the vitelline duct This

diverticulum is usually found about 2 feet proximal to the ileocecal junction and is usually about 2 inches

long It is present in about 2 of the popUlation It may be the site of ectopic pancreatic tissue or gastric

mucosa and may develop inflammatory processes and ulcerations Acute Meckels diverticulitis

simulates appendicitis

Which of the following veins carries blood from the esophagus to the portal vein The

A right gastric vein

B left gastric vein c splenic vein D azygos vein

E left gastroepiploic vein

Explanation

The left gastric vein a direct branch of the portal vein drains blood from the lesser curvature of the

stomach and the inferior portion of the esophagus Because branches of the portal vein do not have

valves blood can flow in a retrograde path when there is an obstruction to flow through the portal system or liveL Rlooci Cln then flow from the nortl] vein thr()1Ph the left PRstric vein to the esonhlPlIS lno

through venous communications within the submucosa of the esophagus to esophageal veins that drain

into the azygos vein The increase in blood flow through the esophageal submucosal veins results in esophageal varices

On the posterior wall of the abdomen the celiac ganglion A contains cell bodies of postganglionic parasympathetic neurons B is synapsed upon by neurons in the posterior vagal trunk C is synapsed upon by neurons in the greater splanchnic nerve D contains sensory cell bodies of lumbar spinal nerves E contains cell bodies of neurons that cause an increase in the rate of peristasis

Explanation The celiac ganglion is one of the preaortic ganglia of the sympathetic nervous system It contains cell bodies of postganglionic sympathetic neurons The sympathetic splanchnic nerves contain preganglionic sympathetic neurons that pass through the sympathetic chain without synapsing These splanchnic nerves go to the preaortic ganglia to synapse The greater splanchnic nerve contains preganglionic neurons from spinal cord segments T5-T9 This nerve synapses in the celiac ganglion The nerve fibers in the vagal trunks are preganglionic parasympathetic fibers that go to the walls of the organs that they will innervate and synapse on postganglionic parasympathetic neurons in the walls of those organs Cell bodies of sensory neurons in the abdomen are found in the dorsal root ganglia or the sensory ganglia of the vagus nerve Sympathetic innervation decreases the rate of peristalsis parasympathetic innervation increases the rate of peristalsis

Which of the following pairs of arteries will allow blood to bypass an occlusion of the celiac trunk

A Left gastric artery-right gastric artery

B Left gastroepiploic artery-right gastroepiploic artery

C Superior pancreaticoduodenal artery-inferior pancreaticoduodenal artery

D Splenic artery-common hepatic artery

E Left gastric artery - proper hepatic artery

Explanation The anastoOlosis of a branch of the celiac trunk and a branch of the superior mesenteric artery will

provide collateral circulation around an occlusion of the celiac trunk Each of the other choices pair

branches of the celiac trunk therefore these will not provide collateral flow around the obstruction of the

celiac trunk The left gastric splenic and common hepatic arteries are direct branches of the celiac trunk

The right gastric artery is a branch of the proper hepatic artery which is a branch of the common hepatic artery The left gastroepiploic artery is a branch of the splenic artery The right gastroepiploic artery is a

branch of the gastroduodenal artery whlch is a branch of the common hepatic artery

Which of the following organs has appendices epiploica The

A sigmoid colon

Bjejunum

C duodenum

D stomach E esophagus

Explanation Appendices epiploica are characteristic of the colon Appendices epiploica are subserosal accumulations

of fat None of the organs of the gastrointestinal tract has appendices epiploica except the colon

Page 5: Chirag's Abdomen Review

Ox of Indirect vs Direct Inguinal Hernias (Protrusions of peritoneum through an opening)

Indired inguinal CQes through the INternal (deep) ingllil1111 FeHowS Ih( path of Ule descent hernil dng external (stlperGeial inguin~l rillg and of the testes Cocred by ~ll1

INto the scrotum Enters internl hlguin31 ring 3 layets )f spermatic filscilt) lateral to inferiof ejgtigastrh artery OC-(gtl1f in INfants owing to failure ofpfo(e~~us vaginalts to dose rfu c h more (Qmmon in males

Direct inguinal Protrudes through the inguilllt11 (Hes~eJhaehs l MDs dont tIc hemi~l triangle Bu1ges directly through abdominal 1ledial to inferior epigastric

wall medial to inferior epi~~1Strilt artcrr alte-ry =Dired hClTlkl

GOtgtS through the external (Hlperfidal ~ Lateral to inferior epigastriC ingnilltl1 ling only Covered by transversalis MtelT =Indired he micentl

fascia Usually in older men

Lymphatics - Please dont get Testes and Scrotum mixed up

o Testes Lymph follows blood supply through inguinal canal back to abdominal aorta to

lumbar and preaortic lymph nodes Metastasis of Testicular Cancer - Livestrong

o Scrotum Lymph (As do lymphatics from all ectodermally derived structures in this

region) drain to superficial inguinal lymph nodes

GI blood supply and innervation Emhryonic Artery Pltlrasytnpathdilt VCTt-cbral Structnr(s suVP li-lt gut region inIlervation level

For(ut Cdi ~1 C Sgl5 T12Ll St)(fIl(b to pmxtll1li dll(llteJlllJl1middot lin-f gallbladder IMlliHa

Ll Dist1I dwXlentll11 Jo proximll -f of lra n ~Hrc colo n

I~lA Di~t~lll lgt of tmmt rsl cokm to upp-r portion [f fthllll _pknic Ilexuf is a wtlkrshtd ngiol1

Heart - Gastric and

duodenal regions

Cenae artery

~IY-~ Primordiuln of liver

Superior llli3Senteric artery 10 mido~n

I Interior mesenteric Hhidgut artecrj

Celia( trunk Bran ches of cd ial( [rnnK (ornm011 ]lmiddotpltic splc- nicleft gastric lhclt -ornpri~t tile main blood supply of the toma(h

Short Ilstric han pC gtOf

1lIltlSlo nlOc if ~pl ~ llic nrkry

is bloch-d Strollt anasbmOHS ltxist

betwe tn

-Left and right ga gtir(jt~plpJoj (s

-Left Lind right gastriCs

KNOW YOUR ANASTAMOSES

IIhlieunci

~plecn to JID$tEfipr i pleric artc~

Biliary structures

BE ABLE TO DRAW A FLOW CHART OF BILE FLOW

Important GI Iigaments

I

COnnect t

Ucr lcl n l)le rid~ abdominal i LlnmeJl1mh teresshy wall 1

I

if o(tal i ri~l c) he~atlc ~l rhry_ l1ay be olnptessed -1 Prin1 vein emiddotolllmn be~ween tlmmb 111d

t blle dud indd fin~d Pticed in cpk ploic farunen nfWinslow hJ ~ntt 1l1Jeecling

t

6n~e tHtre~te(Urid I crIt

middotbullbullmiddot -os

trHr to 1 liSef clltUliue middot1 Gastric m~eri~ 1 n ter and~epIt1tes ghtgrel

of stom~cll I les~rsac _~b be qt 9Brtng urger 1

~ i to ac(~3S le~(r sa

Grcater luC ltinind Ifa1t ofgreater 1)1I1tmiddotntllln

[ran fr colon 1

Lcreater curv ep~r1tCs left gre~br and t $pI eil middot I lesscr bllt$

qI

Splenorenal linil Vd~l 1 bull bull abdaininal JltaU

___o-J-___ ~~~~

~~----- ~-- ~~- ---~

Be able to visualize these ligaments know their embryonic origins and contents

Recognize relationship of structures in epiploic foramen (Portal Triad)

Splenorenal is high yield

Know which ligaments contribute to the greater and lesser momentum

Clinical- If the stomach ruptures posteriorly - contents are confined to the lesser sac but if

the stomach ruptures anteriorly - contents in the greater sac can spread as far down as the

pelvis

Know the relationship of structures within the portal triad - Hepatic Vein will be the trick

answer choice

L sgt-J[ offiEntumHpatic artY proPll

8iloduct bull

Stomach

Glstrapllmic ligament

Ornootnl-+----+r--___-----cT-- ~Jrarn n

IH---V~tal

petitOllllU-m

Splllllofffial figamgn

TXH

LIVER - ClinicalDryWet

Coronary Iigameot

Ewph3g~al

Impression

Lett tria09ular Hepatic veins

ligament

Fibrougt Supralerfal impreurossionappendix of

tiOer Hoparorenal portion ot coronary ligament

Garuic RighitrianQularl igameot impresgtion

(Common) bile duct

Common hepalic duel

Clstic duel

Renal Impression Caudate lobe

Papiilary proc~ss Quadrat~ lobe

allbladder

___ FalCltltorm ligament

Hepatic artery PlOP - Round Jigam~nt 01 liver

Hepatic portal veio ~-- Coli c impression

FisUH for ligamen1vm teres

Porta hepatis

Make sure know

Embryonic origin of the iigaments

Anatomical vs Functional lobes and relationship to the Gallbladder

Portal Triad Structures (VH DISSECTOR)

Impressions

CLINICALS -

Trac heoesophageal Almocml COll 11ction htteell esophalIs ll nd tmlthe1 fistula Jost lI)OUllul1 StLblypt is hlind up tr () hngtls with lOler esophagus tO)HHcted to

lrac hea RcsulJ in (3 11 $($ (ho tng anc vo mitin with ftt(~~lS lir bllblllpoundQll CXR and )Qlvl V -~-

I ~ i I Esophageal y i J atresia - - Trachea

----shy

Z

Congenital pyloric l1l perlrnphy (J f le pylurus camt~ obstruti I) Pa Ipablc uhve mass in epigl~tric n~ion stenosis and nOl1biliolts PfOiL mnit+rrg- - - wetk of ltl4C Tn)tment is suriltai ineisi n

Ottmi in 160(llit births ofteIl in I ~ t-bOfl malts --

Peptk ulcer djsease G~ltri( ulcer Pain can he greater vdth mellls--Welghtl - ft(H OCClIlS in older putient

a Mlori tnfedion in 70 chronic NSAID use also implicated Dm to llHue-oml protection a~lllns t gfltrh l id

Duodella) ulcer Pain Deu(QSe5 wilh menh--weighl ~aln_ Almost IOO have H priori infection Dut to 1~ gastfk acid cerct 11 f( 1 2011 inpmiddotr-Ell lson )~md rol11t i ( r L m(lcos~ ll

protction Hyprttophy of Bntnllcr glands Tend to ha ve ltIeall pullcheJ-ouf margim unlike the tais(dJirre~lllar margins of

cnCinOl1lH P-te1itiai eomplications include bl eedin~ penetrnu(n into pnnerelS perfomtion and obs tmctiQH (not intrinsltllly pr((JIlcerom) (see Image 11-+

Question - A 57 year old obese chronic alcoholic presents with an ulcer which has ruptured the cI~wnpastelgrly Surgical investigation reveals blood in the peritoneum Which of shy

the following arteries is most likely responsible for the bleeding

a Splenic Artery

b Gastroduodenal Artery

c R Gastric Artery

d L Gastric Artery

e L Gastroepiploic Artery

Appendicitis All age ~rol1ps most common indication for em er(nl ilxkminai)lJrger) in ehildrefL huLial diffmc periumbilical pain --7 localized pain JtMcBmm~ point N~ltlSC~l kvef

r)13)- perfor~te - pcntouitis Difftr~ntiI1 divertieuli[i (elderly ectopic pr(~gnanl to [3-hCG 10 rule ont)o

-----

PcmiddotrhtltllCf of the ) lcllille dud Of ~Olk sh31k ~br lontli n e( topic ~l cid-seefehllg gatric m 1(0]

andor pmcrcatic ii~~lle Lllost common c(lllgenital anomliy aftlle CT tract Cm CllHt hkfciing illtusm~(~pjjon Dlnllus or nbstrudion nelr the tcrrnin)l ileum Contraslwitll QIB~efic nmiddot = cvtk dilalaUon of ittllilC dJet

------~-

The ile 2$

2Jpound~11 11~ 2 feet frolll the iie( middoteCill vke l~ QfiJ~ at 1~

CIllIn nly prcsenfltgt III rll~2

llf~ of lifc- by ilwe 2 ty )ts of

epilheH8 19ls-trie- pal1elli(i

Hirschsplungs disease

Congenital tnt91(middotolon characterized by lack of Think of a gian ~pring that ganliml ttlLJcnJ(rk ~~gJeXllS~( (~lihs and lei ~sner plCxpstTlrlsgIllent on inbstinal biop y ()It to-iIure of U(middotural t~restpoundtU migration =

has s nl

Presents as ronic comtipnHoll tHly in life Dilllted pOltioll of the colon proximal 10 the aganiionic

segment resulting in a middot trmsjtillll ZQl1t rnvolt~

rectum [huany farlur( to P~$ meconium

High Yield WetDrylTheory List-

Suggestion - in your study group try to write a question for each of these points and then

exchange with a friend and try to answer each others questions

Abdomen Blood Supply - Reference viks picture posted on my google group - this is THE MOST HIGH

YIELD TOPIC IN ALL OF ANATOMY - expect 5 questions on your mini and 5-10 questions on your shelf

Make sure you can draw the blood supply and answer tertiary questions

Example - If the patient had an occlusion of the celiac trunk - which of the following areas would

experience ischemia

Portal hypertension - Know the 3 clinically relevant sites of portal caval anastamoses

Testes vs Scrotum lymph drainage

Where are paraumbillicai veins located

Omalophcele - failure of the gutto come back in (if in yolk sac - fatal)

Marginal Arteries

Superior Messenteric Artery is in front of 3d part of duodenum

Caput medusa (Distended paraumbillical veins secondary to portal hypertension)

Kidney Constrictors -1 Renal Pelvis 2 Crossing Pelvic Brim 3 Entering Urinary Bladder

Ureter - wet lab

Vagus is PIIJS supply up to 23 trans colon than pelvic splanch N up to the ass

Hirshsprungs disease - baby cant poop - dilated colon

Meckels Diverticulum - rule of 2s - 2 feet prox from ileocecal ju nc

Urachal Fistula - weeping belly

Gall stones - common bile duct

Jaundice relation to tumor of the head of the pancreas

Hepatopancreatic ampulla

Blood supply of renal gland - s superrenal art m s suprarenal a abd aorta inf suprarenal art

R Kidney - Tl2-L3 L Kidney - Tl1-L2

Renal Artery - L2

Epiploic foramen - know the borders and contents

Alantois diverticulum - urachus - medial umbilical

Lateral Medial and Median umbillical Folds (know the contents)

Directindirect hernia - know how to diagnose where they enter and exit the inguinal region and which

one is congenital

Anular pancreas - projectile vomiting

Pyloric Stenosis - projectile vomiting (non bilous)

Duodenal Atresia - projectile vomiting (bilous)

Dry Lab - know x rays

Vitteline Fistula - food out of umbilicus

Major duodenal papilla - junc of foregutmidgut

Arcuate line - relationship to rectus sheath

Mcburneys point -13 from ASIS bw umbilicus

Parietal pain - what is the nerve supply

Internal Oblique - cremasteric relationship

Know spermatic fasia

Processes Vaginalis - connection bw peritoneum and gubernaculums

Umbillicus - TlO dermatome

Deep Inguinal-l25 cm above mid inguinal ligament

Superficial Inguinal Ligament- superolateral to pubic symphysis

Variocele - veins engorged in scrotum (bag of worms)

bull

bull External spermatic fascia derived from external obliques EO II Cremasteric fascia ~ from internal obliques fO bull Internal spermatic fascia derived from fascia transversalis bull Tunica vaginalis derived from processes vaginalis directly rests on testes bull know order from testes out to skin

note reflex o ilioinguinal nerve o Efferent =genital branch of the genitofemoral nerve

--lt gt-- info important anastamoses which connects thorax to abdomen

bull Sup

o Sup epigastric branch of internal thoracic o Inf branch external iliac

Venous drainage o Above umbilicus aXillary v o Below umbilicus veins in triangle o At level of umbilicus Paraumbilical veins -gt drain into the portal V

II Important in Portal Caval Venous system Venous drainage of testes

o Clinical correlation Varicocele 11 vein drains into IVC 11 Left testicular vein ~ drains into left renal v

bag of rmlt

for lymph drainage T10 axillary lymph nodes

ill Below T10 superficial inguinal lymph nodes (lateral

Umbilical Folds

Lateral umbilical folds inferior vessels

Medial umbillcial folds umbilical (fetal remnant)

Median umbilical fold urachus (fetal remnant)

Between these folds fossas o Supervesical fossa between median and medial folds

11 bladder o hesselbachs between medial and I folds

II DIRECT HERNIAS HERE Borders

Medial semilunar line

Lateral info Epigastric

Inferior inguinallig o Lateral Inguinal Fossa beyond lateral fold

INDIRECT HERNIAS HERE II Deep inguinal ring (lateral to inferior epigastric a)

Indirect inguinal hernia o Lateral to inferior epigastric a o more common o When inserting finger in superficial inguinal ring will feel on tip of finger (since it goes

throueh ineuinal canall

----

Dry Lab - Label subcostal iliohypogastric Ll Ilioinguinal (Ll)

Horesshoe Kidney - stuck under IMA

Renal Agenesis -failure of the ureter bud to develop

Double Ureter

Unilateral Agenesis -1 kidney

Kidneys - Metanephros

Fetal kidneys are at sacral level

Look at 3rd part of duodenum

Some of this stuff is repeated I know just copied and pasted a bunch of stuff I had copy

Dermatomes

bull T4 nipples

bull no umbilicus v o Pain referred to no in appendicitis o Pain referred to T7ITS in gastritis ~

Inguinal ligament = external abdominal oblique aponeurosis

bull Inserts at anterior superior iliac spine to the pubic tubercle o Why important to know -7 visualizing this line allows us to properly diagnose a hernia

Below the inguinallig femoral hernia Above the inguinallig =inguinal hernia

Also to palpate the deep inguinal ring you go about 12Scm above the mid-inguinal

point

bull Modifications to ligament o Pectinate ligament o Lacunar ligament -7 cut this ligament to relieve strain i~ stran ul~tEd hernia

Inguinal canal

bull in males -7 transmits spermatic cord o important structures of spermatic cord ductus deferens testicular a genital branch of

the genitofemoral n pampiniform plexus of veins bull in females -7 transmits round ligament

Borders

bull Floor -7 inguinal ligament + lacunar ligament bull Anterior -7 aponeurosis of external oblique + internal oblique bull Roof -7 internal oblique and traverse abdominal bull Posterior -7 transverse abdominal + transversalis fascia

o Reinforced by conjoint tendon bull Aponeurosis of internal abdominal obliques and transverse abdominus bull Lies immediately behind the superficial inguinal ring in what would otherwise be

a weak point in the abdominal wall bull Innervated by ilioinguinal nerve (Ll) ~why important

bull In appendicitis Ll can be injured which will injure this nerve and in turn

the conjoint tendon With loss of innervation to this supportive structure the patient is now predisposed to a direct inguinal hernia

o Only hernia that can transverse the inguinal canal o Associated with congenital condition persistent tunica vaginalis

bull Direct inguinal hernia o Medial to inferior epigastric a o When inserting finger in superficial inguinal ring will feel on back of finger o Associated w old age or recent surgery

Muscles (Abdomen RECTUS SHEATH)

bull Arcuate line at level of ASISor 13rd distance between pubis and umbilicus bull Above arcuate line rectus abdominus is surrounded by a rectus sheath anteriorly and

posteriorly

o EO and 10 lie over rectus abdominus o 10 and TA lie behind rectus abdominus

bull Below arcuate line rectus abdominus has no rectus sheath posteriorly o EO 10 and TA lie over rectus abdominus o Transversalis fascia lies behind rectus abdominus o Inf EpIgastric vessels pierces the rectus sheath here

Peritoneum serous sac which encloses most of the abdominal structures

bull Ovary =only intraperitoneal organ o Oocyte ejected from ovary then captured by fallopian tubes o Why impt Women more prone to infection that can enter peritoneum

Peritoneum forms

bull Mesentery double layered fold of peritoneum formed as the organ was pulled in

bull Ligament between 2 organs in general bull Omentum between stomach and another organ bull Bare area area of no peritoneum bull

Viscera innervation

bull Pa rasympathetics 11 o Afferents sense hunger o Efferents l peristalsis relaxes sphincters gland secretion

bull Sympathetics o Efferents do opposite o Afferents CARRY PAIN SENSATION OF THE VISCERA (dull stretching pain)

bull PARASYMPATHETIC INNERVATION o Vagus nerve 7 _1l to 23rd unct ion of la rgej nte~tine oJ)elVrcspla~~~)~~ic~rYe~ IJiU- ~rd aJ~lpoteotiD~ IMPT

Gut Embryology

Gut ~ We say that the gut is derived from endoderm We often forget that when we say so we mean

that only the mucosa is derived from the endoderm The submucosa and the muscle layer is actually derived from the splanchnopleuric mesoderm and the serosa is derived from the visceral peritoneum

~ The main function of the gut is to digest the food which is done by the glands derived (and are) in the mucosa (endoderm) The only two exceptions in the Gut where glands though derived from the endoderm do not stay there but migrate down into the submucosa are esophagus and duodenum These glands however have their ducts opening to the swface of the mucosa

bull

~ Lungs liver amp gall bladder and pancreas are off-shoots from the foregut Esophagusshy~ The region of the tube from the laryngeal diverticulum to the beginning of the stomach elongates

to form the esophagus ~ The glands which form in the endoderm (mucosa) migrate down into the submucosa The path

whlch it took migrate becomes the duct of the glands which open to the mucosa ~ Achalasia Cardia - Failure of relaxation of the lower esophageal sphincter because of congenital

absence of ganglia at the sphincter (The ganglia when present releases VIP (Vaso-IntestinalshyPeptide) which relaxes the sphincter)

Mid-Gut Rotation ~ Because of the 90 degree rotation of the primitive stomach all of the following events occur ~ Lesser curvature comes to the right Therefore lesser omentum also comes to the right ~ Greater curvature comes to the left Therefore greater omentum also comes to the left ~ Right side vagal trunk becomes posterior vagal trunk ~ Left side vagal trunk becomes anterior vagal trunk ~ The left side peritoneal cavity comes to the anterior aspect of the stomach and will later be called

as the greater sac ~ The right side peritoneal cavity comes to the posterior aspect of the stomach and is (relatively a

small sac because the liver is on the right) called the lesser sacomental bursaepiploic sac ~ Epiploic foramen of Winslow (the lower free margin of the ventral mesentry) wiII be the

communication between the greater and lesser sac ~ The Liver moves to the right and therefore actually causes the 90 degree rotation of the stomach

The spleen comes to lie on the left side ~ Axis Antero-posterior axis around the superior mesenteric artery

bull Counterclockwise bull Approximately 270deg bull During herniation (about 90deg) bull During return (remaining 180deg)

Duodenum ~ Becomes retroperitoneal (except the first part which is still suspended by the hepato-duodenal

part of lesser omentum) ~ Glands (of Brunner) go submucosal ~ An imaginary line drawn below the opening of the major duodenal papilla represents the junction

between the foregut and midgut ~ Duodenal atresia in Downs syndrome Liver ~ 3rd week

bull liver bud grow bull into the septum bull transversum

~ 10th week bull hematopoietic bull function

bull 10 of the total bull body weight

~ 12th week bull bile is formed

Pancreas ~ In about 10 of cases the duct system fails to fuse and the original double system persists ~ 3rd month

bull pancreatic islets (Langerhans) ~ 5th month

bull Insulin secretion ~ Annular pancreas

bull The right portiCn of the ventralbud migrates along its normal route but the left migrates in the opposite direction

~ Complete obstruction of duodenum ~ Accessory pancreatic tissue Polyhydramnios (Amniotic fluidgt 1500-2000 ml)

~ Congenital defects including central nervous system disorders (eg Anencephaly) and gastrointestinal defects (atresias ego Duodenal esophageal) prevent the infant from swallowing the amniotic fluid (failure of recanalization)

Oligohydramnios (Amniotic fluid lt 400 mt) ~ Cl~ldberenal-agenesis

bull Midgut _-_

~ Primary Midgut intestinal loop gives rise to bull Distal duodenum bull Jejunum bull Ileum bull Ascending colon bull Transverse colon - proximal two-thirds of the bull Transverse colon with the distal third

~ Primary intestinaltoop bull ncephalic limb distal part of the duodenum the jejunum and part of the ileum bull ncaudal limb lower portion of the ileum the cecum the appendix the ascending colon and

the proximal two-thirds of the transverse colon bull 6th week

bull Rapid elongation of the cephalic limb bull Rapid growth of the liver bull Intestinal loops enter the extraembryonic cavity in the umbilical cord

bull 10th week bull loops begin to return bull regression of the mesonephric kidney reduced growth of the liver expansion of the

abdominal cavity bull Jejunum -left bull Loops - more to the right

bull Cecal bud -last part (temporarily below the right lobe of the liver) ~ qIDlthaloseJe (Structures COlHLoArts9V~1tion)

bull Through umbilical ring bull 6th to 10th weeks

bull Associated with a high rate of mortality (25) and severe malformations bull Associated with chromosome abnormalities

~ Gastroschisis (Structures coming out are not covered by Amnion) bull herniation through the body wall ----=---=-shybull Into the amniotic cavity bull Lateral right of the umbilicus bull Sometimes the inferior wall fails to develop as a result lower abdominal structures like the

bladder would be exposed to the exterior not associated with chromosome abnormalities ~ Abnormalities of the Mesenteries

bull Mobile cecum persistence of mesocolon bull Extreme form - long mesentery bull Volvulus

~ Distal third of the transverse colon ~ Descending colon ~ Sigmoid colon ~ Rectum ~ Upper part of the anal canal ~ Primitive anorectal canal

bull 7th week cloacal membrane ruptures bull Tip of the urorectal septum perineal body bull Pectinate line

~ Hindgut anamolies bull Rectoanal atresias and fistulas bull Imperforate anus bull Congenital megacolon (aganglionic megacolon Hirschsprung disease)

bull

bull Hindgut

Chirags Abdomen Review - Part 2

Understanding Embryo makes learning blood supply EASY

I I

I t

~ -

)

Table l1r-~ L Adult SUmiddotuctu~SDrj~l Froln Each of he Three Dhisions of be Pringttive GUl Tube t-middot-----middotmiddotmiddotmiddot-

Foregu(

I_ (Celiac Trunk)

Ir-slt-gtphgus

S101na(b

I h -= LiJ~r

Pancre=l S

bull 1 i Biliary apparntu5

Gall bladdshy

i Pha11~Cal pltgtuchcs

LullSS-I

Mjig ---- bull __ _- ----n--duct----~---middot-------l--n

(Superior Jldesen1eric Artery)_-1I-(I_~__ middot __ O-=-)_in_middoto_r_M_e_se_n_t_e_r_i_c_An__

Uuodenu rn 2nd_ 3 lt141h V4Tt

Jejunun-~

nc-un]

tCCUJ11

AppltgtndLX

Transver5e -o1on (p~oxiln1l1 ~O Tbird)

bull__hytgtid~ _ _ ______ L _ __

Tr-dn~llt~se colon (diStul h lTd) I

)

i

Aa ca-nal -( uppeT patt) i

I I

_____ __ _ _ _ ___ - - rhe~ a(t clcriVOkt iV(5 opound~lt prbn1rC ~ nlQC blft TlI)( 134tof r~ tIonoinf~ i 1 ~l l1rd c- P Cle

Now Lets see how much youve learned

Questions

1) A pt receives a general anesthetic in preparation for a c~t~~my A right subcostal incision is made which begins near the xyphoid process runs along and immediately beneath the costal margin to an anterior axillary line and transects the rectus abdominus muscle and rectus sheath At the level of the transpyloric plane the anterior wall of the

-~~-~=--- _eco---shysheath of the rectus abdominus muscle receives contributions from which of the following

a Aponeuroses of the in~ande~tef-Ilal o~ues

b Aponeuroses of the transversus abdominis and internal oblique muscles c Aponeuroses of the transversus abdominis and internal and external oblique

muscles d Transversalis fascia e Transversalis fascia and aponeurosis of the transversus abdominus muscle

A

2) The lat~raJJJ11QjJt~gLfgJlLoneach side of the inner surface of the anterior abdominal wall is created by which of the following structures

K Falx inguinalis (~) Inferior epigastric a

c Lateral border of the rectus sheath d Obliterated umbilical a e Urachus

B

3) A man the victim of several knife wounds to the abdomen during a brawl at the Lobster Shack subsequently developed a direct inguinal hernY Damage to which of the following nerves is most likely responsible for the predisposing weakness of the abdominal wall

~ Genitofemoral nerve ( b) Ilioinguinal nerve ~-t Tenth intercostal nerve

d Subcostal nerve e Pelvic splanchnic nerve

B

4) Which of the following statements concerning a direct inguinal hernia is correct a It is the most common type of abdominal hernia b It transverses the entire length of the inguinal canal c It contains all3 fascia layers of the spermatic cord d It exits the inguinal canal via the superficial ingeJinal ring e It protrudes through H~acb strJg e

~(

1fltbS w E

tl

5) The conjoint tendon is

a Important in preventing indirect inguinal hernias b The fused aponeurotic layers of internal abdominal oblique and transversus

abdominus muscles c Posterior to the deep inguinal ring

d Medial fibers of the inguinal ligament

B

6) A 25 year old male is brought in to the ER after being involved in a car accident in which he received a crushed internal injury in his abdomen Examination reveals a lesion of parasympathetic fibers in the vagJsnerve which interferes with glandular secretory or

smooth muscle functions in which of the foliowingorgans a Bladder b Transverse coloiW c Descending colOO d Prostrate gland e Rectum

B

7) The spermatic cord includes all of the following contents except a Il ioinguinal nerve b Pampin iform plexus of veins c Vas deferens d Genitofemoral nerve

A

8 Which abdominal structure gives rise to the internal spermatic fascia (muscle) following the descent of testes in development

a External abdominal oblique aponeurosis b Transversalis fascia c Transversus abdominis muscle d Peritoneum e Internal abdominal oblique

B

9 Which abdominal structure gives rise to the tunica vaginalis fotlowing the descent of testes during development shy

a External abdominal oblique aponeurosis b Transversalis fascia c Transversus abdominis muscle d Peritoneum e Internal abdominal oblique

D

10) The lesser omentum is a peritoneal fold which is su bdivided into the a Hepatogastric and gastrosplenic ligaments b Hepatoduodenal and gastroomentalligaments c Hepatoduodenal and gastrosplenic ligaments d Hepatogastric and hepatoduoden9-jrj igaments

D

11) A posteriorly perforating ulcer in the pyloric antrum of the stomach is most likely to produce initiallocalized peritonitis or abcess formation in which ofthS fQllowing

a Great-sac - -- -

b Paracolic recess

c Omental bursa

d Right subphrenic space

c

The inferior mesenteric artery arises from the abdominal aorta ilm_ediill~y_J-Qs1eriQLto which of the foowing org~ns A-F~t~filie duodenum B Head of the pan~eis C Neck of the pandeas

D Second part of the duodenum

E Third part of the duooenum_shylaquoshy

shy

The correct answer is E The inferior mesenteric artery arises from the anterior surface of the aorta at the level of the third lumbar vertebra The third part of the duodenum crosses the midline at the level of the third lumbar vertebra and passes anterior to the aorta at the origin of the inferior mesenteric artery The

first part of the duodenum (choice A) lies horizontally to the right of the midline at the level of the first

lumbar vertebra The head of the pancreas (choice B) is to the right of the midline and extends from the

level of the first lumbar vertebra to the third lumbar vertebra It lies within the concavity of the

duodenum The neck of the pancreas (choice C) lies in the midline at the level of the first lumbar

vertebra It lies on the anterior surface of the aorta at the origin of the superior mesenteric artery The second part of the duodenum (choice D) lies vertically to the right of the midline and extends from the

level of the first lumbar vertebra to the level of the third lumbar vertebra

The left adrenaLvein drains directly into which of the following veins A Hemiazygos vein

B Inferior vena cavaee C Left renal veiri -

D Splenic vein

E Superior mesenteric vein

a

The correct answer is C The left adrenal vein and the left gonadal vein (either testicular or ovarian) drain into the left renal vein TheTeft renal vein t~ains intothe- inferior vena cava In contrast the right

adrenal ~~inandnght gonadal veindrai~ gLr~ctJy iQtoJhe iilferiQ[ Vencava -- -

ThehemTazygoS7ein- (~h-~i-~ A)~~c~i~es the venous drainage from the body wall on the left side of the

thorax and abdomen No visceral organs drain directly to the azygos or hemiazygos veins The inferior vena cava (choice B) receives the direct venous drainage from the right adrenal vein but not

the left adrenal vein Remember the inferior vena cava is on the right side of the abdomen The splenic

vein (choice D) receives the venous drainage from the spleen and part of the pancreas and stomach The splenic vein is part of the portal venous system

The superior mesenteric vein (choice E) receives venous drainage from much of the intestinal tract It is part of the portal venous system and joins with the splenic vein to form the portal vein

A 43-year-old man presents complaining of pain in the groin On examination his physician palpates a

bulge in the region of the superficial inguinal ring which he diagnoses as a direct inguinal hernia The hernial sac most likely

A is covered by all three layers of the spennatic fascia B passes medial to the inferior epi gastric artery

C passes medial to the lateral border of the rectus abdominis muscle

D passes posterior to the inguinal ligament E passes through the deep inguinal ring

The correct answer is B Direct inguinal hernias enter the inguinal canal by tearing through the posterior

wall of that structure The typical location for this type of hernia is through the inguinal triangle bounded

laterally by the inferior epigastric artery medially by the lateral border of the rectus abdominis and

inferiorly by the inguinal ligament Direct inguinal hernias pass medial to the inferior epigastric artery

whereas indirect inguinal hernias pass lateral to the inferior epigastric artery because the deep inguinal

ring is lateral to the artery Indirect inguinal hernias are covered by all three layers of the spermatic fascia (choice A) Direct inguinal hernias are covered by fewer than all three layers because the direct inguinal

hernia tears through one or more layers of fascia as it emerges though the abdominal wall The lateral

border of the rectus abdominis muscle (choice C) forms the medial border of the inguinal triangle All

inguinal hernias pass lateral to the rectus abdominis Femoral hernias pass posterior to the inguinal ligament (choice D) Inguinal hernias emerge through the superficial inguinal ring which is superior to the inguinal ligament Inguinal hernias that descend below the inguinal ligament pass anterior to the

ligament Indirect inguinal hernias pass through the deep inguinal ring (choice H) direct inguinal hernias

do not Both types of inguinal hernias pass through the superficial inguinal ring

During a gastric resection in a patient with stomach cancer a surgeon wants to remove the lesser

omentum because of tumor extension into it Which of the following structures lie in the free edge of the

l~~g omentum and consequently must be dissected out in order to be preserved

A Common bile duct cystic duct and hepatic artery 6

B Cystic duct hepatic artery and hepatic vein

e Hepatic vein and cystic duct

Portal vein common bile duct and hepatic artery

E Portal vein hepatic artery and hepatic vein

The correct answer is D The free edge of the lesser omentum contains three important structures the

common bile duct the hepatic artery and the portal vein Nei ther the cystic duct (choices A B and C) nor the hepatic vein (choices B C and E) lies in the free

edge of the lesser omentum

A 55-year-old male patient with chronic liver disease has portal hypertension To relieve the pressure in the portal system a porto-caval shunt is performed Which of the following veins may by anastomosed to

accomplish this porto-caval shunt A Left renal vein-left testicular veingt

B Right renal vein-right suprarenal vein I shy

e Splenic vein -left renal vein J

D Superior mesenteric vein-inferior mesenteric vein E Superior mesenteric vein-splenic vein

The correct answer is C The splenic vein drains directly into the portal vein The left renal vein drains

directly into the inferior vena cava Anastomosis of these veins would allow blood from the portal vein to

drain retrograde though the splenic vein into the renal vein and then into the inferior vena cava The left

renal vein (choice A) drains directly into the inferior vena cava The left testicular vein drains directly into

the left renal vein Thus these veins are already in communication and neither vein is part of the portal venous system The right renal vein (choice B) drains directly into the inferior vena cava The right

suprarenal vein also drains directly into the inferior vena cava Thus neither vein is part of the portal

venous system The superior mesenteric vein (choice D) drains directly into the portal vein The inferior

mesenteric vein drains into the splenic vein which then drains into the portal vein Thus neither vein is

part of the caval venous system The superior mesenteric vein (choice E) drains directly into the portal

vein The splenic vein also drains directly into the portal vein Thus neither vein is part of the caval

venous system

A 12 year old boy has fever vomiting and para-umbilical pain After examining the patient the doctor

makes an initial diagnosis of appendicitis Appendicular pain which is initially referred to the umbilicus goes to the dorsal root ganglion of

a TI b TI2 c L1 d T7

(e I TIO

A 59-year-old male undergoes a neurological examination which reveals that when the abdominal wall is

stroked the muscles of the abdominal wall of the side of the body stimulated failed to contract Other

neurological tests appeared normal The likely region affected includes

a CI - C5 spinal segments b C6 - TI c T2-TI ~T8-T12

e Ll- L5

The surgery done to relive portal hypertension is done by connecting two veins Which of the following veins would be suitable for connection

a Inferior vena cava and portal vein b Superior vena cava and portal vein c Splenic vein and right renal vein d Splenic vein and left renal vein e Superior mesenteric vein and Inferior vena cava

A mother brings her 3-week-old infant to the pediatric clinic reporting a new scrotal bulge that she found -~-

while changing a diaper yesterday The infant is afebrile Physical examination reveals a palpable mass in

the scrotum while in the standing position resolution of the mass in the supine position and no

transillumination of the scrotal sac What is the most likely diagnOSiS

a Cryptorchidism b Direct inguinal hernia c Hydrocele d Indirect inguinal hernia ~ e varicocele

The Vagal trunks enter the abdomen by passing through which of the following openings in the

diaphragm

a Right crus b Esophageal hiatus ~ c Vena caval hiatus d Aortic hiatus e Left crus

2 The anterior boundary of the epiploic foramen of Winslow is bounded by

a) First part of duodenum b) Lesser curvature of stomach c) Liver d) Hepato-duodenalligament v ~

3 The ilio-inguinal nerve is derived from

a TI2 ry b LI c L2 d L3 e L23

15 Surgically the structure used to suspend the kidney to the diaphragm is

a) Renal fascia b) True capsule c) Perinephric fat d) Paranephric fat

6 If there is portal obstruction because of carcinoma affecting the pancreas which of these of the

following signs would be present

a Caput medusae b Esophageal varices c Rectal varices c

d Pulmonary edema

7 In a sliding hernia the gastro-esophageal junction lies

a) At its normal position b) Below the normal position c) Above the normal position V d) None of the above

8 Which of the following structures is retroperi toneal

A transverse colon B spleen IJ2f6 C ileum D descending colon v r 1pound1111111

9 The renal angle is fonned lgtetween the 12th rib and ______ muscle

a Psoas major -middotshyb Erector spinae c Quadratus Iumborum d Diaphragm

10 The anterior structure at the hilum of the kidney is

a) Renal vein ~

b) Renal artery I middot~ I

c) Ureter d) Accessory renal artery

11 Because of origin of the muscle from the lateral one third of the inguinal ligament it

could not fonn the anterior wall of the inguinal ligament

a) External oblique b) Internal oblique c) Transversus abdominis_ d) Rectus abdominis

12 A large tumor mass impinges on the splenic artery and its branches as the artery pass out from below

the greater curvature of the stomach Branches o(which of the following arteries would most likely to

effected by the pressure on the splenic artery

a Left gastric b Left gastro-epipJoic c Right gastric d Right gastro-epipoloic e Short gastric_

13 A new born baby has projectile vomiting after each feeding It is determined that there is obstruction

of the digestive tract as a result of annular pancreas Annular pancreas is as a result of an abnormality in which of the following process

a Rotation of the dorsal pancreatic bud around the first part of duodenum b Rotation of the dorsal pancreatic bud around the second part of duodenum c Rotation of the dorsal pancreatic bud around the third part of duodenum d Rotation of the ventral pancreatic bud around the first part of duodenum y Rotation of the ventral pancreatic bud around the second part of duodenum

14 As the liver bud enters the ventral mesogastrium the region of the mesogastrium stretching from the

liver to the anterior abdominal wall is called

a Lesser Omentum b Greater Omentum ~ Falcifrom ligament d Lacunar ligament e Ligamentum teres of liver

16 A patient has absence of his 12th rib In such a patient if the doctor makes an incision to approach his

kidney mistaking the 11 th rib for the 12t he would end up injuring

Which of the following arteries is a direct branch of the gastroduodenal artery The

A right gastric artery

B left gastric artery

C inferior pancreaticoduodenal artery D left gastroepiploic artery

i E)right gastroepiploic artery --

E x pI a nation The right gastric artery is typically a branch of the proper hepatic artery The left gastric artery is a direct

branch of the celiac trunk The right and left gastric arteries anastomose along the lesser curvature of the

stomach The inferior pancreaticoduodenal artery is a branch of the superior mesenteric artery it

anastomoses with the superior pancreaticoduodenal in the head of the pancreas The left gastroepiploic

artery is a branch of the splenic artery it anastomoses with the right gastroepiploic artery along the greater

curvature of the stomach The right gastroepiploic artery is a branch of the gastroduodenal artery The

other branch of the gastroduodenal artery is the superior pancreaticoduodenal artery

Which of the following pairs of veins join together to form the portal vein The

A superior mesenteric vein and inferior mesenteric vein

B inferior mesenteric vein and splenic vein

C superior mesenteric vein and splenic vein

Ip)splenic vein and left gastric vein E superior mesenteric vein and left gastric vein

Explanation

The portal vein is formed behind the neck of the pancreas by the union of the superior mesenteric vein

and the splenic vein The inferior mesenteric vein drains into the splenic vein The left gastric vein drains

directly into the portal vein After the portal vein forms it enters the hepatoduodenalligament of the

lesser omentum to reach the liver The portal vein is the most posterior structure in the hepatoduodenal

ligament

At which of the following vertebral levels does the duodenum pass anterior to the aorta - _- shy

All ~

B L2 7~

CL3 I

~DL4

E L5

Explanation

The duodenum begins at the pyloric sphincter at the level of Ll The second (or descending) portion of

the duodenum is to the right of the aorta and extends inferiorly from the level of Ll to the level of L3 The third part of the duodenum crosses the aorta from the right side to the left side at the level of L3 The

fourth (ascending) portion of the duodenum extends from the level of LJ to the level of L2 The

duodenum ends at the duodenojejunal flexure The superior mesenteric artery passes anterior to the

duodenum as the duodenum passes anterior to the aorta The duodenum can be constricted at this level

In which of the following locations will perforation of the digestive tract result in the spilling of luminal

contents into the - lesser peritoneal sac

A Anterior wall of the second portion of the duodenum B Posterior wall of the second portion of the duodenum

C Anterior wall of the stomach

~Posterior wall of the stomach E Posterior wall of the transverse colon

Explanation

The posterior wall of the stomach is related to the lesser peritoneal sac The anterior wall of the stomach is related to the greater peritoneal sac The anterior wall of the second portion of the duodenum is related to the greater peritoneal sac The posterior wall of the second portion of the duodenum is related to the retroperitoneal space The posterior wall of the transverse colon is related to the greater peritoneal sac

The ureter lies against the anterior surface of which of the following muscles shyA Crus oftne diaphragm B Quadratus lumborum

0 Psoas major D Transversus abdominis

E Iliacus

Explanation The ureter exits the renal pelvis at about the level of vertebra L2 As it descends along the posterior abdominal wall it lies on the anterior surface of the psoas major The psoas major muscle arises from the bodies of the lower lumbar vertebrae The psoas major muscle is joined by the iliacus to fonn the

iliopsoas muscle The iliopsoas muscle then attaches to the lesser trochanter of the femur and is the major

flexor of the hip

As the right ureter passes the pelvic brim it lies against the anterior surface of which of the following

blood vessels

A Gonadal artery B Inferiorvena cava C Internal iliac artery

rJ- External Iliac artery

E Inferior mesenteric artery

Explanation

The ureter lies in the extraperitoneal space in the posterior abdominal wall Alter leaving the kidney it

passes inferiorly on the anterior surface of the psoas major muscle At the pelvic brim the ureter passes

into the pelvis At this point the common iliac artery is dividing into the external and iliac arteries The

ureter lies on the anterior surface of the external iliac artery immediately distal to the bifurcation This is a useful landmark for a surgeon to locate the ureter

When extravasated urine passes from the superficial perineal space into the anterior abdominal wall it is

found immediately deep to which of the following layers of the anterior abdominal wall

-ltScarpas fascia

B External oblique muscle

C Internal oblique muscle D Transversus abdominis muscle

E Transversalis fascia

Explanation

The superficial perineal space is bound by Colles fascia the fibrous portion of the superficial fascia This

layer of fascia is continuous with Scarpas fascia the fibrous portion of the superficial fascia of the anterior abdominal wall Therefore urine that is deep to Colles fascia will remain deep to Scarpa s fascia The urine will spread in the plane between Scarpas fascia and the external oblique layer

When a horseshoe kidney develops the ascent of the kidney is restricted by the A internal iliac artery B external Iliac artery

C common iliac artery

inferior mesenteric artery

E superior mesenteric artery

Explanation

A horseshoe kidney develops when the inferior poles of the to kidneys fuse together as they ascend into

the abdomen from the pelvis The first anterior midline vessel that is encountered by the horseshoe kidney

is the inferior mesenteric artery This artery prevents the kidney from continuing its ascent

The left testicular vein drains into which of the following veins

A Left internal iliac vein B Left common iliac vein

bflnferior vena cava D Left renal vein I

E Left internal pudendal vein

Explanation

The left testicular vein drains into the left renal vein The right testicular ~i~[~nsltjectlY into the

inferior vena cava This difference in venous drainage is believed to explain the greater incidence of

varicocele on the left side than on the right The venous drainage from the penis is to the internal vein

which then drains into the internal Iliac vein

The spinal nerve that provides cutaneous branches to the skin around the umbilicus is

A TS B TW-shy

C TI2

DL2 EtA

Explanation

The tenth intercostal nerve is the anterior ramus of the TIO spinal nerve After passing through the tenth

intercostal space the nerve continues forward in the anterolateral abdominal wall in the plane between

the internal oblique muscle and the transversus abdominis muscle In the abdominal wall the nerve innervates to the abdominal wall muscles as well as the skin and the parietal peritoneum The umbilicus is

a useful landmark for the region of distribution of the tenth thoracic nerve

The ligament of the vertebral column that resists its extension is the Aligamentum flavum

B supraspinous ligament

C posterior longitudinal ligament

D anterior longitudinal ligament

E interspinous ligament

Explanation

The ligaments of the vertebral column that resist flexion of the column include the supraspinous ligament

interspinous ligament ligamentum fiavum and posterior longitudinal ligament The ligament that resists

extension is the anterior longitudinal ligament This longitudinal ligament is very broad and strong It

covers the anterior and anterolateral surfaces of the vertebral bodies and the intervertebral disks In

addition to resisting extension the anterior longitudinal ligament provides reinforcement to the anterior

and anterolateral surfaces of the intervertebral disk The posterior longitudinal ligament is relatively

narrow and covers the posterior surface of the vertebral bodies and the intervertebral disks This ligament

reinforces the posterior surface of the disk The posterolateral surface of the disk is not reinforced and it

is through this region that herniation of the nucleus pulposus usually occurs

A patient presents with epigastric and right upper quadrant pain The pain is most intense 2-4 hours after

eating and is reduced by the ingestion of antacids The patient states that he has passed black tarry stools

(melena) within the last week Fiberoptic endoscopy reveals a yellowish crater surrounded by a rim of

erythema that is 3 cm distal to the pylorus Accordingly an ulcer has been identified in the patients

A fundus

B antrum

C duodenum

D jejunum

E ileum

A number of physiologic genetic and other factors increase the risk of gastric (and duodenal) peptic

ulcers The evidence that H pylori plays a principle role is compelling Smoking and caffeine are known to adversely affect the morbidity mortality and healing rates of peptic ulcers In general first-degree

relatives of peptic ulcer patients as well as males have a threefold to fourfold increased risk of developing this disorder Paradoxically in gastric ulcer disease acid secretion is not elevated It is possible that

excess secreted hydrogen ion is reabsorbed across the injured gastric mucosa In general a defect in gastric mucosal defense is the more important local physiologic

A patient presents with symptoms of duodenal obstruction caused by an annular pancreas Annular pancreas is caused by

A rotation of the dorsal pancreatic bud into the ventral mesentery B rotation of the ventral pancreatic bud into the dorsal mesentery

fJ failure of the major and minor pancreatic ducts to fuse ~ ~ cleavage of the ventral pancreatic bud and rotation of the two portions in opposite directions around -the duodenum E formation of one pancreatic bud instead of two

Explanation Normally the ventral pancreatic bud rotates around the gut tube to reach the dorsal pancreatic bud The two buds fuse to form a single pancreas and the distal portions of the two ducts fuse The ventral pancreatic bud forms the inferior portion of the head of the pancreas the uncinate process and the major pancreatic duct (of Wirsung) The dorsal pancreatic bud forms the superior part of the head the neck body and tail and the minor pancreatic duct (of Santorini) Annular pancreas is the result of the ventral pancreatic bud dividing into two portions before it rotates into the dorsal mesentery Each portion rotates in opposite directions to get to the dorsal mesentery thus encircling the duodenum The presence of annular pancreas can constrict the duodenum thus obstructing its lumen

In n _ phranlc----

Gon ~l ----_1 Lum bltano

~~--- CornmQ1t bull ac

+-~4--- lnlllirnaJ ilic

xtem iliac

OBJECTIVE - Identify the blood supply to each of the structures listed in the table on the previous page

Ill give you a head start

FOREGUT - Supplied bV Celiac Tru nk (T12)

Proper hepatic

GastiooUod 13Jafter

1nferlor pancreaticoduodenal artery

Common epatlc

Lett gas ric iiirtery

Spfen artery

shy Gastroepiphgtic artery

~ Superior mesenteric 8rtfry

~

1 Esophagus is a derivative of the foregut so its blood supply originates from the celiac trunk

(T12) The predominant blood supply to abdominal portion of the esophagus is the Esophageal

A (Branch of L Gastric) The venous drainage of the esophagus is particularly important because

it is 1 of 3 clinically relevant sites of Portal Caval anastamoses The Portal Esophageal Vein

meets the Caval Azygos System Persistent bleeding manifests as Esophageal Varices - a fata I

condition

2 The Stomach is also a derivative of the foregut has EXTENSIVE blood supply and is very high

yield on anatomy exams The lesser curvature is supplied superiorly by the L Gastric A (1 of 3

major branches ofthe Celiac trunk) and inferiorly by the R Gastric A ( a branch ofthe proper

Hepatic A) The greater curvature is supplied superiorly by the L Gastroepiploic A (a major

branch of the splenic A) and inferiorly by the R Gastroepiploic A

The Short Gastric arteries (branches of Splenic Artery) supply the fundus of the stomach and

are referred to as EIID ARTERIES because they have no collateral blood supply Therefore if the

splenic artery were occluded (ex - increased pressure in the ommental bursa) - there would be

ischemia to the fundus of the stomach Venous drainage of the stomach is extensive via various

veins lead ing to the portal system Posterior to the stomach the IMV joins the splenic V which

joins the SMV to form the PORTAL VEIN ADAMS

3 Duodenum blood supply has high clinical relevance because it is the junction of the foregut and

midgut and therefore is the site of anastamoses between branches ofthe Celiac Trunk (main

foregut artery) and the Superior Messenteric Artery (main midgut artery) The Proper hepatic

artery gives off the gastroduodenal artery which travels behind the 1st part of the duodenum

This point has high clin ical relevance because duodenal ulcers are very common and a posterior

rupture of the 1st part of the duodenum could rupture the gastroduodenal artery causing

traumatic abdominal bleeding The Gastroduodenal artery first gives off the R Gastroepiploic A

(mentioned above) and proceeds as the Superior pancreatico duodenal artery (supplies the

pancreas and duodenum) which anastamoses with the inferior pancreatico duodenal A (branch

of the SMA) This is the junction of foregut and midgut and occurs near the opening of the

bil iary system into the duodenum (ampula of vater) Portal venous drainage here is responsible

for delivering nutrients from digestion to the liver for metabolism Appreciate that the Superior

mesenteric artery (artery of the midgut) branches from the aorta at Ll travels posterior to the

pancreas than moves anteriorly (at the jxn of the pancreatic headbody) and comes over the

3rd4th part of the duodenum Tumor of the head of the pancreas can compress the SMA

4 Jiver blood supply is via the common hepatic artery (major branch of the cel iac trunk) The

common hepatiC becomes the proper hepatic gives off the R gastric A and the Gastroduodenal

A and then joins the common bile duct and the portal vein in the portal triad Clinical- if a

patient were bleeding from the hepatic A a surgeon can stick his fingers in the epiplOic foramen

and squeeze the free edge of the hepatoduodenalligament in order to stop bleeding to the

area Please note that the hepatic a branches into Rand L hepatic A The Right hepatic artery

gives off the cystic artery which supplies the gallbladder Afferent venous supply is via the

Portal vein which is bringing nutrient rich blood to the liver After metabolism takes place

venous blood leaves the liver through the hepatic veins into the IVC PLEASE UNDERSTAND THE

RELATIONSHIP OF THESE STRUCTURES - ADAMSNETIERSNH Etc

5 Pancreas - Head is supplied via the superior and inferior pancreaticoduodenal arteries

(mentioned above) The tail (situated towards the hilum of the spleen) is supplied via the

pancreatic branches of the splenic artery (END ARTERIES) This blood supply is very important

because the endocrine Alpha and Beta Cells from the pancreatic islets of lagerhans are located

towards the tail This is where Insulin and Glucagon is released to the blood

Now complete this for mid and hindgut structures Make sure to note clinically relevant arterial

anastomoses as well as portal caval anastomoses FYI Appendix blood supply SMA + IMA

anastamoses marginal artery Portalcaval rectal veins fhemmorhoids) and periumbilical caput

medusa are high yield THE BUTT THE GUT and THE CAPUT

Abdominal Development

Liver

Ij1f

II wall b

oh liN ~ VltJrti n be- bull

Pancreas

Secondary Retroperitonealization e I~tl r 1 a v-mtrai m ellter

Rotations of the Gut I i Ij (lIl1UtIJ f~ l r tilt

()l td 10 me l-ft and he v

--~--- -~ -~-~

i

I AolaijonjoI~guf I

STOMACH BED (IDENTIFY IN ADAMS)- the structures posterior to the ommental bursa which

support the stomach in the supine position

Abdomnal JQrUI

Splnic vein

OmQ-oul tv~ ) O(s(Jroa)

Lojt(r o m nturrt (hpJtodu o d~n31 Hid

Gadrl)SplerH (g3stroll~nal) IIgam~nt

hiad h~~atogrtricent IIQdmiddotcrt~)

Lt Dome of Diaphragm (why left Look this up in Adams)

Spleen (What is the blood supply)

Left Kidney (What is the blood supply - AND how is it different from the R kidney)

Suprarenal Gland (What is the Arterial AND Venous Blood supply - how are they different)

Pancreas (How does supply differ from Head to Tail What is the SMA Relationship)

Transverse Mesocolon

liver - ADAMSWET - Make sure you look at the liver in wet lab

Left triangular nl1am~nt

ComoaDj ligamnt

Erophg~1 impre$ioo

Hepatio veins

In1erior -ifena middotr3)Ia

Fibrous appendix o-t

live

impr~j on

Heprorendl p~rtion of Q)(Qllary ligament

Righllri~n9ul r 1I~met

(Common) bile quol

Gr)mmCtr~ hepatic dlJct

Ccentic duct

Duodenal impression

GaJdate p-fr)~S

Hepatic artgtrl prop-f iiiiila - Faloiform ligament

_ - shy Round ligamen liver

~--F-- CoJio imprgt-ssi-on

Prta heptis

Identify the lobes impressions and embryonic remnants associated with the liver

Caudate Lobe Quadrate Lobe Right Lobe Left Lobe Round ligament Falciform Ligament

Ligamentum Venosum (what is its fxn in embryonic life) Hepatic Veins (NOT PART OF THE

PORTAL TRIAD) IVC PORTAL TRIAD - Contents relationship cross section etc Know the

Galbladder relationship to the lobes of the liver

Biliary Duct System - Make sure you understand the sequence of these structures - BE ABLE TO

DRAW A FLOW CHART

TPVd i

t

I t

1 __ Cm-(r

patk GlJet

I

J

Clinical = JAUNDICE is caused by anything that prevents delivery of bile to intestine Tumor of the

head of the pancreas Stones etc Patient will have pale stools and yellowish colored mucus

membranes

Clinical- Any scenario that tells you the patient has BILLOUS VOMIT means that the obstruction to

the flow of digestive contents is after the Ampulla of Vater (Site of Entry of Billiary system to the

duodenum) - ie Duodenal Atresia

Spleen -located posterior to the mid axillary line between ribs 9 and 11 Make sure you know that

the 10th rib is the main axis of the spleen and this organ is susceptible to injury (stab wound errant

thoracoce ntesis etc)

The spleen is derived from mesodermal cells - NOT THE GUT TUBE

The spleen rests on the left colic flexure associates with the tail of the pancreas Know the

structures entering the Hilum of the spleen

Sh rt O~-t~ic 1 0(0 10 rtiltSPIric Iloa nt

(cut)

Peritoneum - similar concept to Pleura - think of a fist in a balloon

Visceral Peritoneum - Layer of balloon touching your fist

Parietal Peritoneum - Layer of balloon not touching your fist

Your fist represents the organ your wrist is the hilum and your arm contains the blood supply

entering the organ

Appreciate that there will never be organs in the peritoneal cavity - rather these organs invaginate

the cavity Kaplan videos

RULES OF NOMENCLATUREshy

1 Organ completely surrounded by peritoneum - peritoneal organ

2 Organ partially surrounded by peritoneum- Retroperitoneal

3 Peritoneum surrounding peritoneal organ is VISCERAL peritoneum

4 Peritoneum surrounding retroperitoneal organ is PARIETAL peritoneum

5 Peritoneum connecting visceral to parietal is called messentary 2 messentaries in the

gut Dorsal (to the gut tube) and ventral (to the gut tube) messentary

Aorta is in Retro peritoneal position - but blood must reach peritoneal position - vessels travel through

messentary All peritoneal organs will have blood supply reaching through messentary

-Mesentery is a 2 layer peritoneum with a neurovascular communication between body wall and organ

- Ligament connects one organ with another or to the abdominal wall (Ommentum = ligament)

lesser Ommentum (attach lesser curvature of stomach and duodenum to liver) =Hepatoduodenal

Ligament and Hepatogastric Ligament

Has a Superior and Inferior Recess (Accumulation of Fluid in Ascites)

Communicates with the greater sac through the epiplic foramen (what structures pass through

this foramen)

Boundaries - you must be able to visualize this

o Anterior - stomach

o Posterior - parietal peritoneum pancreas

o Superior - superior recess (bw diaphragm and coronary ligament)

o Inferior -Inferior recess (bw layers or greater momentum

Greater Ommentum (attach greater curvature of stomach) Gastrophrenic ligament Gastrosplenic

ligament gastrocolic ligament

The greater omentum is the largest peritoneal fold It consists of a double sheet of peritoneum folded on itself so that it is made up of four layers The two layers which descend from the greater curvature of the stomach and commencement of the duodenum pass in front of the small intestines sometimes as low down as the pelvis they then turn upon themselves and ascend again as far as the transverse colon where they separate and enclose that part of the intestine

ABDOMINAL PAIN

Parietal Peritoneum - supplied by same vasculature lymphatics and nerves supplying body wall it

lines and diaphragm Sensitive to pain pressure heat cold well localized

Visceral Peritoneum - supplied by same vasculature lymphatics and somatic nerve of organ it covers

Insensitive to touch heat cold and laceration - referred to dermatome of spinal ganglia providing

sensory fibers Where does appendicitis refer to

Foregut pain - epigastric area (ie - cholycystitis)

Midgut pain - periumbilical area (ie - appendicitis)

Hindgut Pain - suprapubic area (ie - diverticulitis)

Extra ImagesConcepts

ll~_____-

FalifCtrm ligament oind r~ud ligamet f Ilver

Blood from splenio gastriC and inferiof rne$e-rteri v~ins

Ca-I tributaries

Lett gastrio Ifein

Posterior superior pan~reatioodul)denal vaihS

Lott gamo-om~nlal (9aropip lomiddotic) -in

Poq_~ tjol imerl-9-r panCJertlcorllJod-nal veiopound --amp----I- - ~J Right grtr~-omntal

Anwrior interi (gartroepiploic) Jjn

pan euaii cod vl)denal veins middot Inf~Ji (t r mesentric vein

Miqdle (olic vein

Right cl)licvein Sigmoid and rectosigml)id (ei ns

IhH)Collc(~io

--- Mi~dl laquooLJl gtjrltgt

PoM ca vl1 illasto)moses -----shyampoptoageal 2 Paraumbilie-lt11 Inferi or Fectal vei ns

3 Recial 4 REuoperHonea1

Know how the Portal vein is formed I 4 sites of portal caval anastamoses and 1 clinical shunt

Col li t ltt-~ otTl~tI ~nj pc~ 1lt1 turJoG

Ltf 14i1 tImiddot~ artoftl9 on tj phtAt$

L-oftqf 4t t~r 1=laquoIran d 1 bull shy~p l ci rj o fOOOts

Nerves follow the arteries - appreciate the splanchnic nervous system I

Uet~ric branch of left ~nal art

Ureterie branch of righi renal artelY

Left Zld lumbar in and co mlTlunication to as)erdin9 lumbar l(~in Hi ~ht tEZ1~~t~ t3r j t itn ~ nJ l1t- rlnd lfe i r1

Inferior me5nteri~ artery

Notice that the right testicular vein drains directly into the IVC and the right testicular artery drains

directly into the aorta However the left testicular vein drains into the L renal vein at a right angleshy

reason left testicle is lower and more susceptible to varicocele (bag of worms)

Also notice that the left renal vein has a longer course because the IVC is on the right side whereas

the right renal artery has a longer course because the aorta is on the left side

Appreciate the anterior to posterior relationship of structures in the hilum of the kidney - VAP - Vein

Artery Renal Pelvis (Ureter)

11____ __ L_ L_ n VJ __ _ _ t_L I I_ _ L __ L_ I -pound1 bull LI_~-I ____

Posterior View of Head of Pancreas in ( of Duodenum

Celiao hunk

Co mmon ~L~jJth art~ry

GastNduQdonal artrf (partilly in phantn)

P1)Sterior $Up~Jior panCflaticuduodfmal art~r~t

(Co mm on) bile duct

middot~1t~~t-1l---~-~- Right gshomiddotomental (gastoe plp lolc) 3rte (phantomost)

Grener paocre atic art-ry

1n1~rjor pancr-iatlc artery

Jtrifll supejo r pal)oreailcento)dJodenal artr1 (phantom)

Anastomotlo branch

POostetlor bJanch of jo f~ri of pan-reatir(lduodensl drttnj

Anterio r branch of i flferior palcreati~)duodenal art~(phan1om)

Notice the extensive blood supply to the pancreas and duodenum via the branches of the celiac trunk

Notice collateral supply from SMA branches - makes sense bc this is the jxn of foregutmidgut

Identify the vessels in this arteriogram

Hiltid i)f N~ck oi B)dvof Tail 01 pa nereas pan cent~as P-nmiddot-reas panCtCas

I nferie v~na cava

jHept1iic p(lrlai v~in

Port1 tnd H~pti lt a ftH prol

Comm on) bll duct

Ouodtnum

~ft colic (sio)Atta~ hmtrlt jt~xJr-ofha~elSe

muo(IIQn

Right ~lIc (h~j)tic)

il~gtture

In1triol m~oten lIein (rttr op~ritoMdO

SlJp efl or mes~n~fiC amrV and lipln

KNOW YOUR NEIGHBORHOOD

Questions

vVhiJh structure supplied by a bnmdlof the cclia( artery is not derivcd from foregut LemCJUCrITI

(A) Head of the pancte-a5

CD) Pyloric duolenum

Cystkduct

( Liver hepatocyt~~

~F) Body of the spleen

An infant presents with an omrhaJucele at birth -hi oJ the [oHm illg applies to his cM1-dition

(A) It is 31so seen ill p4titnts with aganghonic megacolon

(11) ft reuirs from a fal1ure of resorption of theviteUine d let

(C) It results from herniation at the-site of regression of the right umbilk vein

DJ It is caustd by faihtrc of recanalization of the midgut part of the duodenum

~ It ill camioo by a failuIt vf the midgul to return to the abGQminal uity after herniashytion in-n the urnbilk s l stalk

Ot er than the spleen occlusion Cif the spit-Ilk artery at its odgin wm most likely affect die blood supply to jllch st cnud

(A) Jejunum

(B) Body of th pal1~lltas

(C) LeSStT Cllmiddotlaturc of tl )toma-ch

(D Duodenum dista to the entrance of the Ornmou bile duct

E Fundus of the stomach

A 38-yeu-old batL~er with a history of heartburn suddenly experiences excluciating pain in the (plgastric region of th~ abdomeu SurgCry is perf~rme immediard y upon admisshysion to the 1IlcrgCJliy tuomh~re i~ evidence uf a ruptured ulcer in the posterior waU of the stomach Vhere will a surgeon first fi nd the stomach contenlSf

A) Greater p4ritoneal sac

rB) Cul~de-s~c of Douglas (--

C Omental bursa ~

--D) Paracolic gutter

rEj Between -he panttal perimltum and the posterior body wal1

At birth an infant presents with a st()ma~ rb~tbas~njJled jfltotb~diaplfagru 1A1ltre is the defect thatresulied iiitJle heini~t()n shy~tsophagealbiatus

7 - rH-- Hiatus for the inferior vena cava

( Pleuroperitoneal membrane -(0) Septum transvcrsum

(E) Right Crlt~

An infant born with DOVv7l syndrome presents with bili()u~ vomiting Ahat congenital defect does the infant have

(A) Pyloric stenosis

(B) Meckel diverticulum C) Ornphaloce1e

(D) Gastroschisis

( ~ ) Duodenal atresia y A patient with cirrhosis of the liver presents with ~ bacalvaricestnlreased retrograde pressure in which veins caused the varices

(A) Paraumuilical

(B) Splenic

(ct AzygltJus

(15))G~trk ( (-F) Superior mesemeric

A htaltby 3-year~old male patient experiences a hernial sa protruding from the anterior abdominal wall about halfway between me anterior superior ilia spine and the pubk tuberde Pulsations of al1 artery are palpated medial to the protrusion site through the abdominal walL Which layer of the anterior abdominal wall will first be traversed by the

1hctma

fA) Rectus sheath (B) External oblique aponeurosis

(C) Inguinal ligament

lD) Transversalis fusda

(E) Cremasteric fa~cia

After 5urgi(aj ffpair of a hernia the patient tXperienccs mtmlgtness in the skin on the anteshyrior aspect of the S(Totum_ Vhaf nerve may have been lesioned during thehemiorrhaphy

(A) Femoral

(B) Obturator

(C) Ilioinguinal

(D) lliohypogastrk

(E) Pudendal

A 23~year-LJld female secretary il1 good health ~-uddcn1) doubles over with pain in the a ea of the 1JmbRicu$ Sbe feels vartn and ltneasy and has no appetite That night the pain seems to have mQved to the tower right abdominal regjol1 and she calls her family doctor who then arranges for an ambulance to pk-k her up and take her to the hospitaL Wh ell ntn~ perceived in the area of the urnbilirus most Hkely carried lhe pairfu I sensations into the eNS

tA) Vagus nerves I~

V B)

) Lessersplanchnk nerves

tC) Pudendal nerves

(D) lIiohpogastrk nerves

(E) Greater splam ic l erves

A CT reveals carcinoma in the bOod of the ancreas Vhich blood vessel trut ourses ----~- - -bull ------ --shy

immediately poftterior to the body ofthe pancreas is the m~t likely to be oompressed

(A) Splenk artery

(B) Abdominal aorta (C) Portal vein

(1) Splenic vein

(E) Renal vein

A patient has a penrln1l1ng uker of the posterior wall ot the br~l part ot the (lUooenmn llkh blood vessel is subject to erosion

(A) Common hepatic artery

(B) Gastroouodenal artery

(C) Proper hevatic artery

(D) Celiac artery

(E) Anterior inferior 11amrelltlcoduodcnal attery

Your patient has been diagnosed -ith a carcinoma locallted to the head and l~e(k of the pancreas Another clinical sign would be

A esophageal varices

(8) hemorrhoids

C) a caput medusa

(D) increased pra Teuro n th~ hepatic veins

(E) enlarged right supra lavkular lymph nodes

Wltkh of the foUowing structures develops in the ventral mesentery

(A) Spleen

(B) Jeiunum (C) Head of1ht pancreas (D) Transverse colon (E) Stomach

ti l Uw ~ littwin~ f( S-t lil oai Imdge ~ hi(h or tbt la~)d J truetur tgt liJ llntn nl) he hl p UC iJd [IIi ell

c o

A) drains Ie tht infCrior a La aI

R t middot~nfl0 ~ill to th~ lunlgtn of h i dtlndCrlllfH

(e) m t bull JiJattd on tl l J n T ~H

D ) sup Lc O VSlt I Hlid bhtu l 1 li - -I un oid

( ) U~tpli(t tr j middottUh~ 1 v(( b~nt rfK n1ilc~Zm

ANSWERS AND EXPLANATIONS

Answer E The spleen is t hlttnopodicand lymph organ demlted from mesoderm

Answ~ R Al1 tlmphalocele is caused by it failure of the nlidgut to return to the ahdomir nat cavity after herniation into the umbiliau Stalk Choices Aand D maybe seen in infants with Down syndrome choice D ~s the specific CBuse ofduudcnal JtiCSitt Choice C is (ile cause of gclstrosbisis and Choice B nsults iu a Meurolktldivertku1-tlB

Answer B The fundus ofthe stomach is suppHed by soort gastric brunches of the splenic altery The splenic artery supplies the body and tail of the pancreas part of the greater curvature of the sttmla(h and the spleen Te jejunum part of the head of the pancreas and tht~ duodenum distal to the entrance of the commOll bile duct are supplied by the superior mesenterk artery clll~l ~be less r ctlt1ature cmd the pylQric antrum are supplied by the right and lei gastric art(ries

AnSWftt C Tbeomental bursa or lesser ~ritoneaj sac lies direcdy posterior to the proxshyimal part of the duodeTtlm and the stomach and would be the first site where stomach contents ~Ott1d be fpoundluncL

Answer C A defect in a llleuropcritoneal membrane (uswlly the left) is the typical site of i1 cc-ngenitlI diilphragluatic hemia llere the membr4ne fails to dose ()pound( of the perishycCirdiopcritulleal canals

Answer E DuoJenal atresia and aganglionic megacoion are congwitaI defects S~Il in patients with Dowmiddotnsyndrome

Answer D RulaTgemt~llt of and retrograde flow in g~lstrk vel_ns in particlJl~r the kft gas~ tricveins dilates the capillary bed in rhe wall of the esophagus in (ases of porta yper~

tension Blood flow would increase in and dilampte tribntarkgts of the (lZygOUS vein on the other side of the capiUary bed but flow in this vein is in the typical direction t()ward the superior vena cava Paraumbiii(ltU vein eilgorgement contributes to a caput medusH Splenic ~nlargement might prc~nt with 5plcnonlegaly and balt-kflow in to tlu superior m~~ntclic vein occurs but is asymptomatic

Answer D The patient hagt an indirect inguinal hernia whi~h emerges from the antt-rior abdominal wall through the deep inguinltilling Theeep ring is a fault in the transv~rshysaUs fascia this I~yer wiIJ be penetrated first by the hernia

An~Wer C The ilioinguinal nenc which provides sens~llion to the lnedlal thigh ltmclanteshytior SClotunl pass~lt th rough the 5uperfh_ial inguinal ring ind $subject to inj i1T) becaus-e

it is in the operatitm Held of the erniorrhapny

Auswer B The leMHr splanchnic nerves are sympathdic nerVlts that carry viscera l sensashytlltgtrogt ftom illtllt1m~d ()J stietched gust (itinteitinal ~tructures (in this case the pprndix) into tnt eNS Lesser splanchnic ntTYcsarisc from thmiddot T9--T12 spinal cord segments lt1nd provide sympathetic innenation tD rnidgut siruc1ures whiCh include CLe app~JldD Viscera] Pain arising from affecLed Inidgut ampt 1C1ure is referred over the same dl- matorne~ of spinal segrnertts v-hich provide the sympathetic Innervation n this G1SC of appendicitis the invohen~n t of the ltire) of t e unlhHku indud s the T 10 dermatome

Answer B Of the five choices onty the dscending olon is retroperiton~al aldwould be a lik ~ ( choice to be seen immediately a(~jilcent to t11e posterior abdominal middotn~L

Amwen D The SpltftlC ~-ein ourses posterior to the body of the panneas m its way tt drain into the superior mCSfttltlri( vein

Answcr B TILt glstrodllolticnal artery 1 direct hIamh of the comrootl hepatic artery courses immediately pt))iwri() to the duodenum and is slbject to erosion

Answer B Carcinoma of th pan middott3S in the 1tilt1 may compreampgt the portltil vein at irs orishygill The poTtai vcin is fomled when the splenic vein jQiaswith tfie superior meStllt eric vein The inferiot mesenteric vein joins the ~plenjc vein just priOT to tlli~ point at which the splenic joins the superior Jlleit1ltcri( vein Increescd venous presslu in the inferior mesenteric vein is a cause of emo hoid~

Answer C The- velltral pancreas wilich forms most of the head of the p ~ncr as develops in the ventral mes(ntery as antutgrowth of the hepatic diverticulum Th~ hepatic divershyticulull induding the biIJary appa~atus develops in tbe ventral mesentery of the foregut

Answer~ A The superior mesenteric ~in joins with the spienkvein to form the hepatic portal vciu

Answer D The structure at gttlK is the proper hepatic artery~ whkh suppUesoxygenated b middotood to the liver

MAKE SURE YOU KNOW the diff bw Rectus Sheath above and below the arcuate line

ABOVE

Aponeurosis of xiiltmal obllque musclo

Extemll f)biquw musde

Reotln ilbdomlnls musole S~in

Internal 9bliquQ mY~QI

AponeUfOsi$ of hJH$V~~S Lir9a a lb lbdolTlin~ musolo Tri OJV6 rUi

atldomlnis mUS(loe

Sub cutanlilous tiue (tatty ye r)

BElOW

A POrl lJfosis 01 etemal oblique muscle

Aponeul~)sis 01 Internal oblique mU$cl~

Anteriol lay~ of r~ltdus st~ath EXttom1 oblique rnu$cll

Rectus Jbdominis muscle Intoernal Aponeurc-sis of tra~fersU$ oblique muscle-

at-domlnis muscentl ~ Skio

Tra nsvitSus abdomioLs ml)ZClt

TralSVersaHs fascia Medial umQil iegtt1 1i9Jment -and folj

Uldchus Peritoneum (ir median Umbilj~al Suboutane ous

Extraprftone 11ascia

Ymbilimiddot~1 fold)

preu9poundiea1 fascia

tissue (fatty 4nd m~mbr3n(iUS layers)

o Above the arcuate line (A horizontal line 13 of the distance bw the umbilicus and the

pubic symphysis) -10 Aponeurosis divides into an AntPost Laminae

o The Ant Laminae joins EO and Post Laminae joins Trans Abdominis = Ant and Post

RECTUS SHEATH respectively

o BElOW the arcuate line - all 3 aponeurosis join ANTERIOR to rectus muscle to meet its

counterpart in the midline (linea Alba)

o Take away Msg - The abdomen is devoid of a posterior rectus sheath below the

arcuate line and is therefore more vulnerable to herniasinjuries

Question - A physician makes a deep incision in the patients midline immediately superior to

the pubic symphysis which of the following layers is his knife least likely to pass

Rectus Abdominis External Oblique Ant Rectus Sheath Posterior Rectus Sheath All of the

Above

Answer - All of the above None of the other answer choices are midline structures -LINEA

ALBA

Linea Alba has very poor blood supply - doesnt heal well after surgery Therefore this is a

common site for incisional hernias

a Spleen b Transverse colon c Descending colon d Stomach e Pleura

17 Meckels diverticulum is normally found 2 feet proximal from the

a Pyloric sphincter b Lower esophageal sphincter c Ileo-cecal valve d Middle valve of Huston e Anal valve

18 Ulcer in the posterior wall of the first part of the duodenum would erode ___ artery and would cause bleeding

a Left gastric b Right gastric c Hepatic artery proper d Gastroduodenal artery e Middle colic artery

19 An inflamed appendix is identified by a surgeon on the operation table by noting

a The appendicies epiploicae b The convergence of tenia c The artery of Drummond d The mesocolon e The mesosalphinx

20 The nerve which emerges through the psoas major is

a Femoral b Ilio-inguinal c Ilio-hypogastric d Pudendal e Subcostal

21 The right gonadal vein drains into the

a Azygos b Hemiazygos c Inferior Vena Cava d Right renal vein e Left renal vein

22 The hepatocytes in the liver is derived from

a Ectoderm b Endoderm c Mesoderm

d Neural ectoderm

23 Abscess in the lumbar vertebrae due to tuberculosis would spread to the adjacent muscle which is

a Psoas Major b Iliacus c Quadratus lumborum d Tranversus Abdominis

24 The anterior wall of the inguinal canal is formed by

a External oblique and transverses abdominis b External oblique and fascia transversalis c Internal oblique and external oblique d Internal oblique and transverses abdominis e Fascia transversalis and peritoneum

Meckels diverticulum is a result of which of the following developmental abnormalities shy

A Failure of the vitelline duct to close

B Failure of the herniated intestinal loop to retract into the abdomen

C Failure of the urachus to close

D Failure of the midgut to rotate

E Failure of the hepatic duct to close

Explanation

Meckels diverticulum is a result of the persistence of the proximal part of the vitelline duct This

diverticulum is usually found about 2 feet proximal to the ileocecal junction and is usually about 2 inches

long It is present in about 2 of the popUlation It may be the site of ectopic pancreatic tissue or gastric

mucosa and may develop inflammatory processes and ulcerations Acute Meckels diverticulitis

simulates appendicitis

Which of the following veins carries blood from the esophagus to the portal vein The

A right gastric vein

B left gastric vein c splenic vein D azygos vein

E left gastroepiploic vein

Explanation

The left gastric vein a direct branch of the portal vein drains blood from the lesser curvature of the

stomach and the inferior portion of the esophagus Because branches of the portal vein do not have

valves blood can flow in a retrograde path when there is an obstruction to flow through the portal system or liveL Rlooci Cln then flow from the nortl] vein thr()1Ph the left PRstric vein to the esonhlPlIS lno

through venous communications within the submucosa of the esophagus to esophageal veins that drain

into the azygos vein The increase in blood flow through the esophageal submucosal veins results in esophageal varices

On the posterior wall of the abdomen the celiac ganglion A contains cell bodies of postganglionic parasympathetic neurons B is synapsed upon by neurons in the posterior vagal trunk C is synapsed upon by neurons in the greater splanchnic nerve D contains sensory cell bodies of lumbar spinal nerves E contains cell bodies of neurons that cause an increase in the rate of peristasis

Explanation The celiac ganglion is one of the preaortic ganglia of the sympathetic nervous system It contains cell bodies of postganglionic sympathetic neurons The sympathetic splanchnic nerves contain preganglionic sympathetic neurons that pass through the sympathetic chain without synapsing These splanchnic nerves go to the preaortic ganglia to synapse The greater splanchnic nerve contains preganglionic neurons from spinal cord segments T5-T9 This nerve synapses in the celiac ganglion The nerve fibers in the vagal trunks are preganglionic parasympathetic fibers that go to the walls of the organs that they will innervate and synapse on postganglionic parasympathetic neurons in the walls of those organs Cell bodies of sensory neurons in the abdomen are found in the dorsal root ganglia or the sensory ganglia of the vagus nerve Sympathetic innervation decreases the rate of peristalsis parasympathetic innervation increases the rate of peristalsis

Which of the following pairs of arteries will allow blood to bypass an occlusion of the celiac trunk

A Left gastric artery-right gastric artery

B Left gastroepiploic artery-right gastroepiploic artery

C Superior pancreaticoduodenal artery-inferior pancreaticoduodenal artery

D Splenic artery-common hepatic artery

E Left gastric artery - proper hepatic artery

Explanation The anastoOlosis of a branch of the celiac trunk and a branch of the superior mesenteric artery will

provide collateral circulation around an occlusion of the celiac trunk Each of the other choices pair

branches of the celiac trunk therefore these will not provide collateral flow around the obstruction of the

celiac trunk The left gastric splenic and common hepatic arteries are direct branches of the celiac trunk

The right gastric artery is a branch of the proper hepatic artery which is a branch of the common hepatic artery The left gastroepiploic artery is a branch of the splenic artery The right gastroepiploic artery is a

branch of the gastroduodenal artery whlch is a branch of the common hepatic artery

Which of the following organs has appendices epiploica The

A sigmoid colon

Bjejunum

C duodenum

D stomach E esophagus

Explanation Appendices epiploica are characteristic of the colon Appendices epiploica are subserosal accumulations

of fat None of the organs of the gastrointestinal tract has appendices epiploica except the colon

Page 6: Chirag's Abdomen Review

IIhlieunci

~plecn to JID$tEfipr i pleric artc~

Biliary structures

BE ABLE TO DRAW A FLOW CHART OF BILE FLOW

Important GI Iigaments

I

COnnect t

Ucr lcl n l)le rid~ abdominal i LlnmeJl1mh teresshy wall 1

I

if o(tal i ri~l c) he~atlc ~l rhry_ l1ay be olnptessed -1 Prin1 vein emiddotolllmn be~ween tlmmb 111d

t blle dud indd fin~d Pticed in cpk ploic farunen nfWinslow hJ ~ntt 1l1Jeecling

t

6n~e tHtre~te(Urid I crIt

middotbullbullmiddot -os

trHr to 1 liSef clltUliue middot1 Gastric m~eri~ 1 n ter and~epIt1tes ghtgrel

of stom~cll I les~rsac _~b be qt 9Brtng urger 1

~ i to ac(~3S le~(r sa

Grcater luC ltinind Ifa1t ofgreater 1)1I1tmiddotntllln

[ran fr colon 1

Lcreater curv ep~r1tCs left gre~br and t $pI eil middot I lesscr bllt$

qI

Splenorenal linil Vd~l 1 bull bull abdaininal JltaU

___o-J-___ ~~~~

~~----- ~-- ~~- ---~

Be able to visualize these ligaments know their embryonic origins and contents

Recognize relationship of structures in epiploic foramen (Portal Triad)

Splenorenal is high yield

Know which ligaments contribute to the greater and lesser momentum

Clinical- If the stomach ruptures posteriorly - contents are confined to the lesser sac but if

the stomach ruptures anteriorly - contents in the greater sac can spread as far down as the

pelvis

Know the relationship of structures within the portal triad - Hepatic Vein will be the trick

answer choice

L sgt-J[ offiEntumHpatic artY proPll

8iloduct bull

Stomach

Glstrapllmic ligament

Ornootnl-+----+r--___-----cT-- ~Jrarn n

IH---V~tal

petitOllllU-m

Splllllofffial figamgn

TXH

LIVER - ClinicalDryWet

Coronary Iigameot

Ewph3g~al

Impression

Lett tria09ular Hepatic veins

ligament

Fibrougt Supralerfal impreurossionappendix of

tiOer Hoparorenal portion ot coronary ligament

Garuic RighitrianQularl igameot impresgtion

(Common) bile duct

Common hepalic duel

Clstic duel

Renal Impression Caudate lobe

Papiilary proc~ss Quadrat~ lobe

allbladder

___ FalCltltorm ligament

Hepatic artery PlOP - Round Jigam~nt 01 liver

Hepatic portal veio ~-- Coli c impression

FisUH for ligamen1vm teres

Porta hepatis

Make sure know

Embryonic origin of the iigaments

Anatomical vs Functional lobes and relationship to the Gallbladder

Portal Triad Structures (VH DISSECTOR)

Impressions

CLINICALS -

Trac heoesophageal Almocml COll 11ction htteell esophalIs ll nd tmlthe1 fistula Jost lI)OUllul1 StLblypt is hlind up tr () hngtls with lOler esophagus tO)HHcted to

lrac hea RcsulJ in (3 11 $($ (ho tng anc vo mitin with ftt(~~lS lir bllblllpoundQll CXR and )Qlvl V -~-

I ~ i I Esophageal y i J atresia - - Trachea

----shy

Z

Congenital pyloric l1l perlrnphy (J f le pylurus camt~ obstruti I) Pa Ipablc uhve mass in epigl~tric n~ion stenosis and nOl1biliolts PfOiL mnit+rrg- - - wetk of ltl4C Tn)tment is suriltai ineisi n

Ottmi in 160(llit births ofteIl in I ~ t-bOfl malts --

Peptk ulcer djsease G~ltri( ulcer Pain can he greater vdth mellls--Welghtl - ft(H OCClIlS in older putient

a Mlori tnfedion in 70 chronic NSAID use also implicated Dm to llHue-oml protection a~lllns t gfltrh l id

Duodella) ulcer Pain Deu(QSe5 wilh menh--weighl ~aln_ Almost IOO have H priori infection Dut to 1~ gastfk acid cerct 11 f( 1 2011 inpmiddotr-Ell lson )~md rol11t i ( r L m(lcos~ ll

protction Hyprttophy of Bntnllcr glands Tend to ha ve ltIeall pullcheJ-ouf margim unlike the tais(dJirre~lllar margins of

cnCinOl1lH P-te1itiai eomplications include bl eedin~ penetrnu(n into pnnerelS perfomtion and obs tmctiQH (not intrinsltllly pr((JIlcerom) (see Image 11-+

Question - A 57 year old obese chronic alcoholic presents with an ulcer which has ruptured the cI~wnpastelgrly Surgical investigation reveals blood in the peritoneum Which of shy

the following arteries is most likely responsible for the bleeding

a Splenic Artery

b Gastroduodenal Artery

c R Gastric Artery

d L Gastric Artery

e L Gastroepiploic Artery

Appendicitis All age ~rol1ps most common indication for em er(nl ilxkminai)lJrger) in ehildrefL huLial diffmc periumbilical pain --7 localized pain JtMcBmm~ point N~ltlSC~l kvef

r)13)- perfor~te - pcntouitis Difftr~ntiI1 divertieuli[i (elderly ectopic pr(~gnanl to [3-hCG 10 rule ont)o

-----

PcmiddotrhtltllCf of the ) lcllille dud Of ~Olk sh31k ~br lontli n e( topic ~l cid-seefehllg gatric m 1(0]

andor pmcrcatic ii~~lle Lllost common c(lllgenital anomliy aftlle CT tract Cm CllHt hkfciing illtusm~(~pjjon Dlnllus or nbstrudion nelr the tcrrnin)l ileum Contraslwitll QIB~efic nmiddot = cvtk dilalaUon of ittllilC dJet

------~-

The ile 2$

2Jpound~11 11~ 2 feet frolll the iie( middoteCill vke l~ QfiJ~ at 1~

CIllIn nly prcsenfltgt III rll~2

llf~ of lifc- by ilwe 2 ty )ts of

epilheH8 19ls-trie- pal1elli(i

Hirschsplungs disease

Congenital tnt91(middotolon characterized by lack of Think of a gian ~pring that ganliml ttlLJcnJ(rk ~~gJeXllS~( (~lihs and lei ~sner plCxpstTlrlsgIllent on inbstinal biop y ()It to-iIure of U(middotural t~restpoundtU migration =

has s nl

Presents as ronic comtipnHoll tHly in life Dilllted pOltioll of the colon proximal 10 the aganiionic

segment resulting in a middot trmsjtillll ZQl1t rnvolt~

rectum [huany farlur( to P~$ meconium

High Yield WetDrylTheory List-

Suggestion - in your study group try to write a question for each of these points and then

exchange with a friend and try to answer each others questions

Abdomen Blood Supply - Reference viks picture posted on my google group - this is THE MOST HIGH

YIELD TOPIC IN ALL OF ANATOMY - expect 5 questions on your mini and 5-10 questions on your shelf

Make sure you can draw the blood supply and answer tertiary questions

Example - If the patient had an occlusion of the celiac trunk - which of the following areas would

experience ischemia

Portal hypertension - Know the 3 clinically relevant sites of portal caval anastamoses

Testes vs Scrotum lymph drainage

Where are paraumbillicai veins located

Omalophcele - failure of the gutto come back in (if in yolk sac - fatal)

Marginal Arteries

Superior Messenteric Artery is in front of 3d part of duodenum

Caput medusa (Distended paraumbillical veins secondary to portal hypertension)

Kidney Constrictors -1 Renal Pelvis 2 Crossing Pelvic Brim 3 Entering Urinary Bladder

Ureter - wet lab

Vagus is PIIJS supply up to 23 trans colon than pelvic splanch N up to the ass

Hirshsprungs disease - baby cant poop - dilated colon

Meckels Diverticulum - rule of 2s - 2 feet prox from ileocecal ju nc

Urachal Fistula - weeping belly

Gall stones - common bile duct

Jaundice relation to tumor of the head of the pancreas

Hepatopancreatic ampulla

Blood supply of renal gland - s superrenal art m s suprarenal a abd aorta inf suprarenal art

R Kidney - Tl2-L3 L Kidney - Tl1-L2

Renal Artery - L2

Epiploic foramen - know the borders and contents

Alantois diverticulum - urachus - medial umbilical

Lateral Medial and Median umbillical Folds (know the contents)

Directindirect hernia - know how to diagnose where they enter and exit the inguinal region and which

one is congenital

Anular pancreas - projectile vomiting

Pyloric Stenosis - projectile vomiting (non bilous)

Duodenal Atresia - projectile vomiting (bilous)

Dry Lab - know x rays

Vitteline Fistula - food out of umbilicus

Major duodenal papilla - junc of foregutmidgut

Arcuate line - relationship to rectus sheath

Mcburneys point -13 from ASIS bw umbilicus

Parietal pain - what is the nerve supply

Internal Oblique - cremasteric relationship

Know spermatic fasia

Processes Vaginalis - connection bw peritoneum and gubernaculums

Umbillicus - TlO dermatome

Deep Inguinal-l25 cm above mid inguinal ligament

Superficial Inguinal Ligament- superolateral to pubic symphysis

Variocele - veins engorged in scrotum (bag of worms)

bull

bull External spermatic fascia derived from external obliques EO II Cremasteric fascia ~ from internal obliques fO bull Internal spermatic fascia derived from fascia transversalis bull Tunica vaginalis derived from processes vaginalis directly rests on testes bull know order from testes out to skin

note reflex o ilioinguinal nerve o Efferent =genital branch of the genitofemoral nerve

--lt gt-- info important anastamoses which connects thorax to abdomen

bull Sup

o Sup epigastric branch of internal thoracic o Inf branch external iliac

Venous drainage o Above umbilicus aXillary v o Below umbilicus veins in triangle o At level of umbilicus Paraumbilical veins -gt drain into the portal V

II Important in Portal Caval Venous system Venous drainage of testes

o Clinical correlation Varicocele 11 vein drains into IVC 11 Left testicular vein ~ drains into left renal v

bag of rmlt

for lymph drainage T10 axillary lymph nodes

ill Below T10 superficial inguinal lymph nodes (lateral

Umbilical Folds

Lateral umbilical folds inferior vessels

Medial umbillcial folds umbilical (fetal remnant)

Median umbilical fold urachus (fetal remnant)

Between these folds fossas o Supervesical fossa between median and medial folds

11 bladder o hesselbachs between medial and I folds

II DIRECT HERNIAS HERE Borders

Medial semilunar line

Lateral info Epigastric

Inferior inguinallig o Lateral Inguinal Fossa beyond lateral fold

INDIRECT HERNIAS HERE II Deep inguinal ring (lateral to inferior epigastric a)

Indirect inguinal hernia o Lateral to inferior epigastric a o more common o When inserting finger in superficial inguinal ring will feel on tip of finger (since it goes

throueh ineuinal canall

----

Dry Lab - Label subcostal iliohypogastric Ll Ilioinguinal (Ll)

Horesshoe Kidney - stuck under IMA

Renal Agenesis -failure of the ureter bud to develop

Double Ureter

Unilateral Agenesis -1 kidney

Kidneys - Metanephros

Fetal kidneys are at sacral level

Look at 3rd part of duodenum

Some of this stuff is repeated I know just copied and pasted a bunch of stuff I had copy

Dermatomes

bull T4 nipples

bull no umbilicus v o Pain referred to no in appendicitis o Pain referred to T7ITS in gastritis ~

Inguinal ligament = external abdominal oblique aponeurosis

bull Inserts at anterior superior iliac spine to the pubic tubercle o Why important to know -7 visualizing this line allows us to properly diagnose a hernia

Below the inguinallig femoral hernia Above the inguinallig =inguinal hernia

Also to palpate the deep inguinal ring you go about 12Scm above the mid-inguinal

point

bull Modifications to ligament o Pectinate ligament o Lacunar ligament -7 cut this ligament to relieve strain i~ stran ul~tEd hernia

Inguinal canal

bull in males -7 transmits spermatic cord o important structures of spermatic cord ductus deferens testicular a genital branch of

the genitofemoral n pampiniform plexus of veins bull in females -7 transmits round ligament

Borders

bull Floor -7 inguinal ligament + lacunar ligament bull Anterior -7 aponeurosis of external oblique + internal oblique bull Roof -7 internal oblique and traverse abdominal bull Posterior -7 transverse abdominal + transversalis fascia

o Reinforced by conjoint tendon bull Aponeurosis of internal abdominal obliques and transverse abdominus bull Lies immediately behind the superficial inguinal ring in what would otherwise be

a weak point in the abdominal wall bull Innervated by ilioinguinal nerve (Ll) ~why important

bull In appendicitis Ll can be injured which will injure this nerve and in turn

the conjoint tendon With loss of innervation to this supportive structure the patient is now predisposed to a direct inguinal hernia

o Only hernia that can transverse the inguinal canal o Associated with congenital condition persistent tunica vaginalis

bull Direct inguinal hernia o Medial to inferior epigastric a o When inserting finger in superficial inguinal ring will feel on back of finger o Associated w old age or recent surgery

Muscles (Abdomen RECTUS SHEATH)

bull Arcuate line at level of ASISor 13rd distance between pubis and umbilicus bull Above arcuate line rectus abdominus is surrounded by a rectus sheath anteriorly and

posteriorly

o EO and 10 lie over rectus abdominus o 10 and TA lie behind rectus abdominus

bull Below arcuate line rectus abdominus has no rectus sheath posteriorly o EO 10 and TA lie over rectus abdominus o Transversalis fascia lies behind rectus abdominus o Inf EpIgastric vessels pierces the rectus sheath here

Peritoneum serous sac which encloses most of the abdominal structures

bull Ovary =only intraperitoneal organ o Oocyte ejected from ovary then captured by fallopian tubes o Why impt Women more prone to infection that can enter peritoneum

Peritoneum forms

bull Mesentery double layered fold of peritoneum formed as the organ was pulled in

bull Ligament between 2 organs in general bull Omentum between stomach and another organ bull Bare area area of no peritoneum bull

Viscera innervation

bull Pa rasympathetics 11 o Afferents sense hunger o Efferents l peristalsis relaxes sphincters gland secretion

bull Sympathetics o Efferents do opposite o Afferents CARRY PAIN SENSATION OF THE VISCERA (dull stretching pain)

bull PARASYMPATHETIC INNERVATION o Vagus nerve 7 _1l to 23rd unct ion of la rgej nte~tine oJ)elVrcspla~~~)~~ic~rYe~ IJiU- ~rd aJ~lpoteotiD~ IMPT

Gut Embryology

Gut ~ We say that the gut is derived from endoderm We often forget that when we say so we mean

that only the mucosa is derived from the endoderm The submucosa and the muscle layer is actually derived from the splanchnopleuric mesoderm and the serosa is derived from the visceral peritoneum

~ The main function of the gut is to digest the food which is done by the glands derived (and are) in the mucosa (endoderm) The only two exceptions in the Gut where glands though derived from the endoderm do not stay there but migrate down into the submucosa are esophagus and duodenum These glands however have their ducts opening to the swface of the mucosa

bull

~ Lungs liver amp gall bladder and pancreas are off-shoots from the foregut Esophagusshy~ The region of the tube from the laryngeal diverticulum to the beginning of the stomach elongates

to form the esophagus ~ The glands which form in the endoderm (mucosa) migrate down into the submucosa The path

whlch it took migrate becomes the duct of the glands which open to the mucosa ~ Achalasia Cardia - Failure of relaxation of the lower esophageal sphincter because of congenital

absence of ganglia at the sphincter (The ganglia when present releases VIP (Vaso-IntestinalshyPeptide) which relaxes the sphincter)

Mid-Gut Rotation ~ Because of the 90 degree rotation of the primitive stomach all of the following events occur ~ Lesser curvature comes to the right Therefore lesser omentum also comes to the right ~ Greater curvature comes to the left Therefore greater omentum also comes to the left ~ Right side vagal trunk becomes posterior vagal trunk ~ Left side vagal trunk becomes anterior vagal trunk ~ The left side peritoneal cavity comes to the anterior aspect of the stomach and will later be called

as the greater sac ~ The right side peritoneal cavity comes to the posterior aspect of the stomach and is (relatively a

small sac because the liver is on the right) called the lesser sacomental bursaepiploic sac ~ Epiploic foramen of Winslow (the lower free margin of the ventral mesentry) wiII be the

communication between the greater and lesser sac ~ The Liver moves to the right and therefore actually causes the 90 degree rotation of the stomach

The spleen comes to lie on the left side ~ Axis Antero-posterior axis around the superior mesenteric artery

bull Counterclockwise bull Approximately 270deg bull During herniation (about 90deg) bull During return (remaining 180deg)

Duodenum ~ Becomes retroperitoneal (except the first part which is still suspended by the hepato-duodenal

part of lesser omentum) ~ Glands (of Brunner) go submucosal ~ An imaginary line drawn below the opening of the major duodenal papilla represents the junction

between the foregut and midgut ~ Duodenal atresia in Downs syndrome Liver ~ 3rd week

bull liver bud grow bull into the septum bull transversum

~ 10th week bull hematopoietic bull function

bull 10 of the total bull body weight

~ 12th week bull bile is formed

Pancreas ~ In about 10 of cases the duct system fails to fuse and the original double system persists ~ 3rd month

bull pancreatic islets (Langerhans) ~ 5th month

bull Insulin secretion ~ Annular pancreas

bull The right portiCn of the ventralbud migrates along its normal route but the left migrates in the opposite direction

~ Complete obstruction of duodenum ~ Accessory pancreatic tissue Polyhydramnios (Amniotic fluidgt 1500-2000 ml)

~ Congenital defects including central nervous system disorders (eg Anencephaly) and gastrointestinal defects (atresias ego Duodenal esophageal) prevent the infant from swallowing the amniotic fluid (failure of recanalization)

Oligohydramnios (Amniotic fluid lt 400 mt) ~ Cl~ldberenal-agenesis

bull Midgut _-_

~ Primary Midgut intestinal loop gives rise to bull Distal duodenum bull Jejunum bull Ileum bull Ascending colon bull Transverse colon - proximal two-thirds of the bull Transverse colon with the distal third

~ Primary intestinaltoop bull ncephalic limb distal part of the duodenum the jejunum and part of the ileum bull ncaudal limb lower portion of the ileum the cecum the appendix the ascending colon and

the proximal two-thirds of the transverse colon bull 6th week

bull Rapid elongation of the cephalic limb bull Rapid growth of the liver bull Intestinal loops enter the extraembryonic cavity in the umbilical cord

bull 10th week bull loops begin to return bull regression of the mesonephric kidney reduced growth of the liver expansion of the

abdominal cavity bull Jejunum -left bull Loops - more to the right

bull Cecal bud -last part (temporarily below the right lobe of the liver) ~ qIDlthaloseJe (Structures COlHLoArts9V~1tion)

bull Through umbilical ring bull 6th to 10th weeks

bull Associated with a high rate of mortality (25) and severe malformations bull Associated with chromosome abnormalities

~ Gastroschisis (Structures coming out are not covered by Amnion) bull herniation through the body wall ----=---=-shybull Into the amniotic cavity bull Lateral right of the umbilicus bull Sometimes the inferior wall fails to develop as a result lower abdominal structures like the

bladder would be exposed to the exterior not associated with chromosome abnormalities ~ Abnormalities of the Mesenteries

bull Mobile cecum persistence of mesocolon bull Extreme form - long mesentery bull Volvulus

~ Distal third of the transverse colon ~ Descending colon ~ Sigmoid colon ~ Rectum ~ Upper part of the anal canal ~ Primitive anorectal canal

bull 7th week cloacal membrane ruptures bull Tip of the urorectal septum perineal body bull Pectinate line

~ Hindgut anamolies bull Rectoanal atresias and fistulas bull Imperforate anus bull Congenital megacolon (aganglionic megacolon Hirschsprung disease)

bull

bull Hindgut

Chirags Abdomen Review - Part 2

Understanding Embryo makes learning blood supply EASY

I I

I t

~ -

)

Table l1r-~ L Adult SUmiddotuctu~SDrj~l Froln Each of he Three Dhisions of be Pringttive GUl Tube t-middot-----middotmiddotmiddotmiddot-

Foregu(

I_ (Celiac Trunk)

Ir-slt-gtphgus

S101na(b

I h -= LiJ~r

Pancre=l S

bull 1 i Biliary apparntu5

Gall bladdshy

i Pha11~Cal pltgtuchcs

LullSS-I

Mjig ---- bull __ _- ----n--duct----~---middot-------l--n

(Superior Jldesen1eric Artery)_-1I-(I_~__ middot __ O-=-)_in_middoto_r_M_e_se_n_t_e_r_i_c_An__

Uuodenu rn 2nd_ 3 lt141h V4Tt

Jejunun-~

nc-un]

tCCUJ11

AppltgtndLX

Transver5e -o1on (p~oxiln1l1 ~O Tbird)

bull__hytgtid~ _ _ ______ L _ __

Tr-dn~llt~se colon (diStul h lTd) I

)

i

Aa ca-nal -( uppeT patt) i

I I

_____ __ _ _ _ ___ - - rhe~ a(t clcriVOkt iV(5 opound~lt prbn1rC ~ nlQC blft TlI)( 134tof r~ tIonoinf~ i 1 ~l l1rd c- P Cle

Now Lets see how much youve learned

Questions

1) A pt receives a general anesthetic in preparation for a c~t~~my A right subcostal incision is made which begins near the xyphoid process runs along and immediately beneath the costal margin to an anterior axillary line and transects the rectus abdominus muscle and rectus sheath At the level of the transpyloric plane the anterior wall of the

-~~-~=--- _eco---shysheath of the rectus abdominus muscle receives contributions from which of the following

a Aponeuroses of the in~ande~tef-Ilal o~ues

b Aponeuroses of the transversus abdominis and internal oblique muscles c Aponeuroses of the transversus abdominis and internal and external oblique

muscles d Transversalis fascia e Transversalis fascia and aponeurosis of the transversus abdominus muscle

A

2) The lat~raJJJ11QjJt~gLfgJlLoneach side of the inner surface of the anterior abdominal wall is created by which of the following structures

K Falx inguinalis (~) Inferior epigastric a

c Lateral border of the rectus sheath d Obliterated umbilical a e Urachus

B

3) A man the victim of several knife wounds to the abdomen during a brawl at the Lobster Shack subsequently developed a direct inguinal hernY Damage to which of the following nerves is most likely responsible for the predisposing weakness of the abdominal wall

~ Genitofemoral nerve ( b) Ilioinguinal nerve ~-t Tenth intercostal nerve

d Subcostal nerve e Pelvic splanchnic nerve

B

4) Which of the following statements concerning a direct inguinal hernia is correct a It is the most common type of abdominal hernia b It transverses the entire length of the inguinal canal c It contains all3 fascia layers of the spermatic cord d It exits the inguinal canal via the superficial ingeJinal ring e It protrudes through H~acb strJg e

~(

1fltbS w E

tl

5) The conjoint tendon is

a Important in preventing indirect inguinal hernias b The fused aponeurotic layers of internal abdominal oblique and transversus

abdominus muscles c Posterior to the deep inguinal ring

d Medial fibers of the inguinal ligament

B

6) A 25 year old male is brought in to the ER after being involved in a car accident in which he received a crushed internal injury in his abdomen Examination reveals a lesion of parasympathetic fibers in the vagJsnerve which interferes with glandular secretory or

smooth muscle functions in which of the foliowingorgans a Bladder b Transverse coloiW c Descending colOO d Prostrate gland e Rectum

B

7) The spermatic cord includes all of the following contents except a Il ioinguinal nerve b Pampin iform plexus of veins c Vas deferens d Genitofemoral nerve

A

8 Which abdominal structure gives rise to the internal spermatic fascia (muscle) following the descent of testes in development

a External abdominal oblique aponeurosis b Transversalis fascia c Transversus abdominis muscle d Peritoneum e Internal abdominal oblique

B

9 Which abdominal structure gives rise to the tunica vaginalis fotlowing the descent of testes during development shy

a External abdominal oblique aponeurosis b Transversalis fascia c Transversus abdominis muscle d Peritoneum e Internal abdominal oblique

D

10) The lesser omentum is a peritoneal fold which is su bdivided into the a Hepatogastric and gastrosplenic ligaments b Hepatoduodenal and gastroomentalligaments c Hepatoduodenal and gastrosplenic ligaments d Hepatogastric and hepatoduoden9-jrj igaments

D

11) A posteriorly perforating ulcer in the pyloric antrum of the stomach is most likely to produce initiallocalized peritonitis or abcess formation in which ofthS fQllowing

a Great-sac - -- -

b Paracolic recess

c Omental bursa

d Right subphrenic space

c

The inferior mesenteric artery arises from the abdominal aorta ilm_ediill~y_J-Qs1eriQLto which of the foowing org~ns A-F~t~filie duodenum B Head of the pan~eis C Neck of the pandeas

D Second part of the duodenum

E Third part of the duooenum_shylaquoshy

shy

The correct answer is E The inferior mesenteric artery arises from the anterior surface of the aorta at the level of the third lumbar vertebra The third part of the duodenum crosses the midline at the level of the third lumbar vertebra and passes anterior to the aorta at the origin of the inferior mesenteric artery The

first part of the duodenum (choice A) lies horizontally to the right of the midline at the level of the first

lumbar vertebra The head of the pancreas (choice B) is to the right of the midline and extends from the

level of the first lumbar vertebra to the third lumbar vertebra It lies within the concavity of the

duodenum The neck of the pancreas (choice C) lies in the midline at the level of the first lumbar

vertebra It lies on the anterior surface of the aorta at the origin of the superior mesenteric artery The second part of the duodenum (choice D) lies vertically to the right of the midline and extends from the

level of the first lumbar vertebra to the level of the third lumbar vertebra

The left adrenaLvein drains directly into which of the following veins A Hemiazygos vein

B Inferior vena cavaee C Left renal veiri -

D Splenic vein

E Superior mesenteric vein

a

The correct answer is C The left adrenal vein and the left gonadal vein (either testicular or ovarian) drain into the left renal vein TheTeft renal vein t~ains intothe- inferior vena cava In contrast the right

adrenal ~~inandnght gonadal veindrai~ gLr~ctJy iQtoJhe iilferiQ[ Vencava -- -

ThehemTazygoS7ein- (~h-~i-~ A)~~c~i~es the venous drainage from the body wall on the left side of the

thorax and abdomen No visceral organs drain directly to the azygos or hemiazygos veins The inferior vena cava (choice B) receives the direct venous drainage from the right adrenal vein but not

the left adrenal vein Remember the inferior vena cava is on the right side of the abdomen The splenic

vein (choice D) receives the venous drainage from the spleen and part of the pancreas and stomach The splenic vein is part of the portal venous system

The superior mesenteric vein (choice E) receives venous drainage from much of the intestinal tract It is part of the portal venous system and joins with the splenic vein to form the portal vein

A 43-year-old man presents complaining of pain in the groin On examination his physician palpates a

bulge in the region of the superficial inguinal ring which he diagnoses as a direct inguinal hernia The hernial sac most likely

A is covered by all three layers of the spennatic fascia B passes medial to the inferior epi gastric artery

C passes medial to the lateral border of the rectus abdominis muscle

D passes posterior to the inguinal ligament E passes through the deep inguinal ring

The correct answer is B Direct inguinal hernias enter the inguinal canal by tearing through the posterior

wall of that structure The typical location for this type of hernia is through the inguinal triangle bounded

laterally by the inferior epigastric artery medially by the lateral border of the rectus abdominis and

inferiorly by the inguinal ligament Direct inguinal hernias pass medial to the inferior epigastric artery

whereas indirect inguinal hernias pass lateral to the inferior epigastric artery because the deep inguinal

ring is lateral to the artery Indirect inguinal hernias are covered by all three layers of the spermatic fascia (choice A) Direct inguinal hernias are covered by fewer than all three layers because the direct inguinal

hernia tears through one or more layers of fascia as it emerges though the abdominal wall The lateral

border of the rectus abdominis muscle (choice C) forms the medial border of the inguinal triangle All

inguinal hernias pass lateral to the rectus abdominis Femoral hernias pass posterior to the inguinal ligament (choice D) Inguinal hernias emerge through the superficial inguinal ring which is superior to the inguinal ligament Inguinal hernias that descend below the inguinal ligament pass anterior to the

ligament Indirect inguinal hernias pass through the deep inguinal ring (choice H) direct inguinal hernias

do not Both types of inguinal hernias pass through the superficial inguinal ring

During a gastric resection in a patient with stomach cancer a surgeon wants to remove the lesser

omentum because of tumor extension into it Which of the following structures lie in the free edge of the

l~~g omentum and consequently must be dissected out in order to be preserved

A Common bile duct cystic duct and hepatic artery 6

B Cystic duct hepatic artery and hepatic vein

e Hepatic vein and cystic duct

Portal vein common bile duct and hepatic artery

E Portal vein hepatic artery and hepatic vein

The correct answer is D The free edge of the lesser omentum contains three important structures the

common bile duct the hepatic artery and the portal vein Nei ther the cystic duct (choices A B and C) nor the hepatic vein (choices B C and E) lies in the free

edge of the lesser omentum

A 55-year-old male patient with chronic liver disease has portal hypertension To relieve the pressure in the portal system a porto-caval shunt is performed Which of the following veins may by anastomosed to

accomplish this porto-caval shunt A Left renal vein-left testicular veingt

B Right renal vein-right suprarenal vein I shy

e Splenic vein -left renal vein J

D Superior mesenteric vein-inferior mesenteric vein E Superior mesenteric vein-splenic vein

The correct answer is C The splenic vein drains directly into the portal vein The left renal vein drains

directly into the inferior vena cava Anastomosis of these veins would allow blood from the portal vein to

drain retrograde though the splenic vein into the renal vein and then into the inferior vena cava The left

renal vein (choice A) drains directly into the inferior vena cava The left testicular vein drains directly into

the left renal vein Thus these veins are already in communication and neither vein is part of the portal venous system The right renal vein (choice B) drains directly into the inferior vena cava The right

suprarenal vein also drains directly into the inferior vena cava Thus neither vein is part of the portal

venous system The superior mesenteric vein (choice D) drains directly into the portal vein The inferior

mesenteric vein drains into the splenic vein which then drains into the portal vein Thus neither vein is

part of the caval venous system The superior mesenteric vein (choice E) drains directly into the portal

vein The splenic vein also drains directly into the portal vein Thus neither vein is part of the caval

venous system

A 12 year old boy has fever vomiting and para-umbilical pain After examining the patient the doctor

makes an initial diagnosis of appendicitis Appendicular pain which is initially referred to the umbilicus goes to the dorsal root ganglion of

a TI b TI2 c L1 d T7

(e I TIO

A 59-year-old male undergoes a neurological examination which reveals that when the abdominal wall is

stroked the muscles of the abdominal wall of the side of the body stimulated failed to contract Other

neurological tests appeared normal The likely region affected includes

a CI - C5 spinal segments b C6 - TI c T2-TI ~T8-T12

e Ll- L5

The surgery done to relive portal hypertension is done by connecting two veins Which of the following veins would be suitable for connection

a Inferior vena cava and portal vein b Superior vena cava and portal vein c Splenic vein and right renal vein d Splenic vein and left renal vein e Superior mesenteric vein and Inferior vena cava

A mother brings her 3-week-old infant to the pediatric clinic reporting a new scrotal bulge that she found -~-

while changing a diaper yesterday The infant is afebrile Physical examination reveals a palpable mass in

the scrotum while in the standing position resolution of the mass in the supine position and no

transillumination of the scrotal sac What is the most likely diagnOSiS

a Cryptorchidism b Direct inguinal hernia c Hydrocele d Indirect inguinal hernia ~ e varicocele

The Vagal trunks enter the abdomen by passing through which of the following openings in the

diaphragm

a Right crus b Esophageal hiatus ~ c Vena caval hiatus d Aortic hiatus e Left crus

2 The anterior boundary of the epiploic foramen of Winslow is bounded by

a) First part of duodenum b) Lesser curvature of stomach c) Liver d) Hepato-duodenalligament v ~

3 The ilio-inguinal nerve is derived from

a TI2 ry b LI c L2 d L3 e L23

15 Surgically the structure used to suspend the kidney to the diaphragm is

a) Renal fascia b) True capsule c) Perinephric fat d) Paranephric fat

6 If there is portal obstruction because of carcinoma affecting the pancreas which of these of the

following signs would be present

a Caput medusae b Esophageal varices c Rectal varices c

d Pulmonary edema

7 In a sliding hernia the gastro-esophageal junction lies

a) At its normal position b) Below the normal position c) Above the normal position V d) None of the above

8 Which of the following structures is retroperi toneal

A transverse colon B spleen IJ2f6 C ileum D descending colon v r 1pound1111111

9 The renal angle is fonned lgtetween the 12th rib and ______ muscle

a Psoas major -middotshyb Erector spinae c Quadratus Iumborum d Diaphragm

10 The anterior structure at the hilum of the kidney is

a) Renal vein ~

b) Renal artery I middot~ I

c) Ureter d) Accessory renal artery

11 Because of origin of the muscle from the lateral one third of the inguinal ligament it

could not fonn the anterior wall of the inguinal ligament

a) External oblique b) Internal oblique c) Transversus abdominis_ d) Rectus abdominis

12 A large tumor mass impinges on the splenic artery and its branches as the artery pass out from below

the greater curvature of the stomach Branches o(which of the following arteries would most likely to

effected by the pressure on the splenic artery

a Left gastric b Left gastro-epipJoic c Right gastric d Right gastro-epipoloic e Short gastric_

13 A new born baby has projectile vomiting after each feeding It is determined that there is obstruction

of the digestive tract as a result of annular pancreas Annular pancreas is as a result of an abnormality in which of the following process

a Rotation of the dorsal pancreatic bud around the first part of duodenum b Rotation of the dorsal pancreatic bud around the second part of duodenum c Rotation of the dorsal pancreatic bud around the third part of duodenum d Rotation of the ventral pancreatic bud around the first part of duodenum y Rotation of the ventral pancreatic bud around the second part of duodenum

14 As the liver bud enters the ventral mesogastrium the region of the mesogastrium stretching from the

liver to the anterior abdominal wall is called

a Lesser Omentum b Greater Omentum ~ Falcifrom ligament d Lacunar ligament e Ligamentum teres of liver

16 A patient has absence of his 12th rib In such a patient if the doctor makes an incision to approach his

kidney mistaking the 11 th rib for the 12t he would end up injuring

Which of the following arteries is a direct branch of the gastroduodenal artery The

A right gastric artery

B left gastric artery

C inferior pancreaticoduodenal artery D left gastroepiploic artery

i E)right gastroepiploic artery --

E x pI a nation The right gastric artery is typically a branch of the proper hepatic artery The left gastric artery is a direct

branch of the celiac trunk The right and left gastric arteries anastomose along the lesser curvature of the

stomach The inferior pancreaticoduodenal artery is a branch of the superior mesenteric artery it

anastomoses with the superior pancreaticoduodenal in the head of the pancreas The left gastroepiploic

artery is a branch of the splenic artery it anastomoses with the right gastroepiploic artery along the greater

curvature of the stomach The right gastroepiploic artery is a branch of the gastroduodenal artery The

other branch of the gastroduodenal artery is the superior pancreaticoduodenal artery

Which of the following pairs of veins join together to form the portal vein The

A superior mesenteric vein and inferior mesenteric vein

B inferior mesenteric vein and splenic vein

C superior mesenteric vein and splenic vein

Ip)splenic vein and left gastric vein E superior mesenteric vein and left gastric vein

Explanation

The portal vein is formed behind the neck of the pancreas by the union of the superior mesenteric vein

and the splenic vein The inferior mesenteric vein drains into the splenic vein The left gastric vein drains

directly into the portal vein After the portal vein forms it enters the hepatoduodenalligament of the

lesser omentum to reach the liver The portal vein is the most posterior structure in the hepatoduodenal

ligament

At which of the following vertebral levels does the duodenum pass anterior to the aorta - _- shy

All ~

B L2 7~

CL3 I

~DL4

E L5

Explanation

The duodenum begins at the pyloric sphincter at the level of Ll The second (or descending) portion of

the duodenum is to the right of the aorta and extends inferiorly from the level of Ll to the level of L3 The third part of the duodenum crosses the aorta from the right side to the left side at the level of L3 The

fourth (ascending) portion of the duodenum extends from the level of LJ to the level of L2 The

duodenum ends at the duodenojejunal flexure The superior mesenteric artery passes anterior to the

duodenum as the duodenum passes anterior to the aorta The duodenum can be constricted at this level

In which of the following locations will perforation of the digestive tract result in the spilling of luminal

contents into the - lesser peritoneal sac

A Anterior wall of the second portion of the duodenum B Posterior wall of the second portion of the duodenum

C Anterior wall of the stomach

~Posterior wall of the stomach E Posterior wall of the transverse colon

Explanation

The posterior wall of the stomach is related to the lesser peritoneal sac The anterior wall of the stomach is related to the greater peritoneal sac The anterior wall of the second portion of the duodenum is related to the greater peritoneal sac The posterior wall of the second portion of the duodenum is related to the retroperitoneal space The posterior wall of the transverse colon is related to the greater peritoneal sac

The ureter lies against the anterior surface of which of the following muscles shyA Crus oftne diaphragm B Quadratus lumborum

0 Psoas major D Transversus abdominis

E Iliacus

Explanation The ureter exits the renal pelvis at about the level of vertebra L2 As it descends along the posterior abdominal wall it lies on the anterior surface of the psoas major The psoas major muscle arises from the bodies of the lower lumbar vertebrae The psoas major muscle is joined by the iliacus to fonn the

iliopsoas muscle The iliopsoas muscle then attaches to the lesser trochanter of the femur and is the major

flexor of the hip

As the right ureter passes the pelvic brim it lies against the anterior surface of which of the following

blood vessels

A Gonadal artery B Inferiorvena cava C Internal iliac artery

rJ- External Iliac artery

E Inferior mesenteric artery

Explanation

The ureter lies in the extraperitoneal space in the posterior abdominal wall Alter leaving the kidney it

passes inferiorly on the anterior surface of the psoas major muscle At the pelvic brim the ureter passes

into the pelvis At this point the common iliac artery is dividing into the external and iliac arteries The

ureter lies on the anterior surface of the external iliac artery immediately distal to the bifurcation This is a useful landmark for a surgeon to locate the ureter

When extravasated urine passes from the superficial perineal space into the anterior abdominal wall it is

found immediately deep to which of the following layers of the anterior abdominal wall

-ltScarpas fascia

B External oblique muscle

C Internal oblique muscle D Transversus abdominis muscle

E Transversalis fascia

Explanation

The superficial perineal space is bound by Colles fascia the fibrous portion of the superficial fascia This

layer of fascia is continuous with Scarpas fascia the fibrous portion of the superficial fascia of the anterior abdominal wall Therefore urine that is deep to Colles fascia will remain deep to Scarpa s fascia The urine will spread in the plane between Scarpas fascia and the external oblique layer

When a horseshoe kidney develops the ascent of the kidney is restricted by the A internal iliac artery B external Iliac artery

C common iliac artery

inferior mesenteric artery

E superior mesenteric artery

Explanation

A horseshoe kidney develops when the inferior poles of the to kidneys fuse together as they ascend into

the abdomen from the pelvis The first anterior midline vessel that is encountered by the horseshoe kidney

is the inferior mesenteric artery This artery prevents the kidney from continuing its ascent

The left testicular vein drains into which of the following veins

A Left internal iliac vein B Left common iliac vein

bflnferior vena cava D Left renal vein I

E Left internal pudendal vein

Explanation

The left testicular vein drains into the left renal vein The right testicular ~i~[~nsltjectlY into the

inferior vena cava This difference in venous drainage is believed to explain the greater incidence of

varicocele on the left side than on the right The venous drainage from the penis is to the internal vein

which then drains into the internal Iliac vein

The spinal nerve that provides cutaneous branches to the skin around the umbilicus is

A TS B TW-shy

C TI2

DL2 EtA

Explanation

The tenth intercostal nerve is the anterior ramus of the TIO spinal nerve After passing through the tenth

intercostal space the nerve continues forward in the anterolateral abdominal wall in the plane between

the internal oblique muscle and the transversus abdominis muscle In the abdominal wall the nerve innervates to the abdominal wall muscles as well as the skin and the parietal peritoneum The umbilicus is

a useful landmark for the region of distribution of the tenth thoracic nerve

The ligament of the vertebral column that resists its extension is the Aligamentum flavum

B supraspinous ligament

C posterior longitudinal ligament

D anterior longitudinal ligament

E interspinous ligament

Explanation

The ligaments of the vertebral column that resist flexion of the column include the supraspinous ligament

interspinous ligament ligamentum fiavum and posterior longitudinal ligament The ligament that resists

extension is the anterior longitudinal ligament This longitudinal ligament is very broad and strong It

covers the anterior and anterolateral surfaces of the vertebral bodies and the intervertebral disks In

addition to resisting extension the anterior longitudinal ligament provides reinforcement to the anterior

and anterolateral surfaces of the intervertebral disk The posterior longitudinal ligament is relatively

narrow and covers the posterior surface of the vertebral bodies and the intervertebral disks This ligament

reinforces the posterior surface of the disk The posterolateral surface of the disk is not reinforced and it

is through this region that herniation of the nucleus pulposus usually occurs

A patient presents with epigastric and right upper quadrant pain The pain is most intense 2-4 hours after

eating and is reduced by the ingestion of antacids The patient states that he has passed black tarry stools

(melena) within the last week Fiberoptic endoscopy reveals a yellowish crater surrounded by a rim of

erythema that is 3 cm distal to the pylorus Accordingly an ulcer has been identified in the patients

A fundus

B antrum

C duodenum

D jejunum

E ileum

A number of physiologic genetic and other factors increase the risk of gastric (and duodenal) peptic

ulcers The evidence that H pylori plays a principle role is compelling Smoking and caffeine are known to adversely affect the morbidity mortality and healing rates of peptic ulcers In general first-degree

relatives of peptic ulcer patients as well as males have a threefold to fourfold increased risk of developing this disorder Paradoxically in gastric ulcer disease acid secretion is not elevated It is possible that

excess secreted hydrogen ion is reabsorbed across the injured gastric mucosa In general a defect in gastric mucosal defense is the more important local physiologic

A patient presents with symptoms of duodenal obstruction caused by an annular pancreas Annular pancreas is caused by

A rotation of the dorsal pancreatic bud into the ventral mesentery B rotation of the ventral pancreatic bud into the dorsal mesentery

fJ failure of the major and minor pancreatic ducts to fuse ~ ~ cleavage of the ventral pancreatic bud and rotation of the two portions in opposite directions around -the duodenum E formation of one pancreatic bud instead of two

Explanation Normally the ventral pancreatic bud rotates around the gut tube to reach the dorsal pancreatic bud The two buds fuse to form a single pancreas and the distal portions of the two ducts fuse The ventral pancreatic bud forms the inferior portion of the head of the pancreas the uncinate process and the major pancreatic duct (of Wirsung) The dorsal pancreatic bud forms the superior part of the head the neck body and tail and the minor pancreatic duct (of Santorini) Annular pancreas is the result of the ventral pancreatic bud dividing into two portions before it rotates into the dorsal mesentery Each portion rotates in opposite directions to get to the dorsal mesentery thus encircling the duodenum The presence of annular pancreas can constrict the duodenum thus obstructing its lumen

In n _ phranlc----

Gon ~l ----_1 Lum bltano

~~--- CornmQ1t bull ac

+-~4--- lnlllirnaJ ilic

xtem iliac

OBJECTIVE - Identify the blood supply to each of the structures listed in the table on the previous page

Ill give you a head start

FOREGUT - Supplied bV Celiac Tru nk (T12)

Proper hepatic

GastiooUod 13Jafter

1nferlor pancreaticoduodenal artery

Common epatlc

Lett gas ric iiirtery

Spfen artery

shy Gastroepiphgtic artery

~ Superior mesenteric 8rtfry

~

1 Esophagus is a derivative of the foregut so its blood supply originates from the celiac trunk

(T12) The predominant blood supply to abdominal portion of the esophagus is the Esophageal

A (Branch of L Gastric) The venous drainage of the esophagus is particularly important because

it is 1 of 3 clinically relevant sites of Portal Caval anastamoses The Portal Esophageal Vein

meets the Caval Azygos System Persistent bleeding manifests as Esophageal Varices - a fata I

condition

2 The Stomach is also a derivative of the foregut has EXTENSIVE blood supply and is very high

yield on anatomy exams The lesser curvature is supplied superiorly by the L Gastric A (1 of 3

major branches ofthe Celiac trunk) and inferiorly by the R Gastric A ( a branch ofthe proper

Hepatic A) The greater curvature is supplied superiorly by the L Gastroepiploic A (a major

branch of the splenic A) and inferiorly by the R Gastroepiploic A

The Short Gastric arteries (branches of Splenic Artery) supply the fundus of the stomach and

are referred to as EIID ARTERIES because they have no collateral blood supply Therefore if the

splenic artery were occluded (ex - increased pressure in the ommental bursa) - there would be

ischemia to the fundus of the stomach Venous drainage of the stomach is extensive via various

veins lead ing to the portal system Posterior to the stomach the IMV joins the splenic V which

joins the SMV to form the PORTAL VEIN ADAMS

3 Duodenum blood supply has high clinical relevance because it is the junction of the foregut and

midgut and therefore is the site of anastamoses between branches ofthe Celiac Trunk (main

foregut artery) and the Superior Messenteric Artery (main midgut artery) The Proper hepatic

artery gives off the gastroduodenal artery which travels behind the 1st part of the duodenum

This point has high clin ical relevance because duodenal ulcers are very common and a posterior

rupture of the 1st part of the duodenum could rupture the gastroduodenal artery causing

traumatic abdominal bleeding The Gastroduodenal artery first gives off the R Gastroepiploic A

(mentioned above) and proceeds as the Superior pancreatico duodenal artery (supplies the

pancreas and duodenum) which anastamoses with the inferior pancreatico duodenal A (branch

of the SMA) This is the junction of foregut and midgut and occurs near the opening of the

bil iary system into the duodenum (ampula of vater) Portal venous drainage here is responsible

for delivering nutrients from digestion to the liver for metabolism Appreciate that the Superior

mesenteric artery (artery of the midgut) branches from the aorta at Ll travels posterior to the

pancreas than moves anteriorly (at the jxn of the pancreatic headbody) and comes over the

3rd4th part of the duodenum Tumor of the head of the pancreas can compress the SMA

4 Jiver blood supply is via the common hepatic artery (major branch of the cel iac trunk) The

common hepatiC becomes the proper hepatic gives off the R gastric A and the Gastroduodenal

A and then joins the common bile duct and the portal vein in the portal triad Clinical- if a

patient were bleeding from the hepatic A a surgeon can stick his fingers in the epiplOic foramen

and squeeze the free edge of the hepatoduodenalligament in order to stop bleeding to the

area Please note that the hepatic a branches into Rand L hepatic A The Right hepatic artery

gives off the cystic artery which supplies the gallbladder Afferent venous supply is via the

Portal vein which is bringing nutrient rich blood to the liver After metabolism takes place

venous blood leaves the liver through the hepatic veins into the IVC PLEASE UNDERSTAND THE

RELATIONSHIP OF THESE STRUCTURES - ADAMSNETIERSNH Etc

5 Pancreas - Head is supplied via the superior and inferior pancreaticoduodenal arteries

(mentioned above) The tail (situated towards the hilum of the spleen) is supplied via the

pancreatic branches of the splenic artery (END ARTERIES) This blood supply is very important

because the endocrine Alpha and Beta Cells from the pancreatic islets of lagerhans are located

towards the tail This is where Insulin and Glucagon is released to the blood

Now complete this for mid and hindgut structures Make sure to note clinically relevant arterial

anastomoses as well as portal caval anastomoses FYI Appendix blood supply SMA + IMA

anastamoses marginal artery Portalcaval rectal veins fhemmorhoids) and periumbilical caput

medusa are high yield THE BUTT THE GUT and THE CAPUT

Abdominal Development

Liver

Ij1f

II wall b

oh liN ~ VltJrti n be- bull

Pancreas

Secondary Retroperitonealization e I~tl r 1 a v-mtrai m ellter

Rotations of the Gut I i Ij (lIl1UtIJ f~ l r tilt

()l td 10 me l-ft and he v

--~--- -~ -~-~

i

I AolaijonjoI~guf I

STOMACH BED (IDENTIFY IN ADAMS)- the structures posterior to the ommental bursa which

support the stomach in the supine position

Abdomnal JQrUI

Splnic vein

OmQ-oul tv~ ) O(s(Jroa)

Lojt(r o m nturrt (hpJtodu o d~n31 Hid

Gadrl)SplerH (g3stroll~nal) IIgam~nt

hiad h~~atogrtricent IIQdmiddotcrt~)

Lt Dome of Diaphragm (why left Look this up in Adams)

Spleen (What is the blood supply)

Left Kidney (What is the blood supply - AND how is it different from the R kidney)

Suprarenal Gland (What is the Arterial AND Venous Blood supply - how are they different)

Pancreas (How does supply differ from Head to Tail What is the SMA Relationship)

Transverse Mesocolon

liver - ADAMSWET - Make sure you look at the liver in wet lab

Left triangular nl1am~nt

ComoaDj ligamnt

Erophg~1 impre$ioo

Hepatio veins

In1erior -ifena middotr3)Ia

Fibrous appendix o-t

live

impr~j on

Heprorendl p~rtion of Q)(Qllary ligament

Righllri~n9ul r 1I~met

(Common) bile quol

Gr)mmCtr~ hepatic dlJct

Ccentic duct

Duodenal impression

GaJdate p-fr)~S

Hepatic artgtrl prop-f iiiiila - Faloiform ligament

_ - shy Round ligamen liver

~--F-- CoJio imprgt-ssi-on

Prta heptis

Identify the lobes impressions and embryonic remnants associated with the liver

Caudate Lobe Quadrate Lobe Right Lobe Left Lobe Round ligament Falciform Ligament

Ligamentum Venosum (what is its fxn in embryonic life) Hepatic Veins (NOT PART OF THE

PORTAL TRIAD) IVC PORTAL TRIAD - Contents relationship cross section etc Know the

Galbladder relationship to the lobes of the liver

Biliary Duct System - Make sure you understand the sequence of these structures - BE ABLE TO

DRAW A FLOW CHART

TPVd i

t

I t

1 __ Cm-(r

patk GlJet

I

J

Clinical = JAUNDICE is caused by anything that prevents delivery of bile to intestine Tumor of the

head of the pancreas Stones etc Patient will have pale stools and yellowish colored mucus

membranes

Clinical- Any scenario that tells you the patient has BILLOUS VOMIT means that the obstruction to

the flow of digestive contents is after the Ampulla of Vater (Site of Entry of Billiary system to the

duodenum) - ie Duodenal Atresia

Spleen -located posterior to the mid axillary line between ribs 9 and 11 Make sure you know that

the 10th rib is the main axis of the spleen and this organ is susceptible to injury (stab wound errant

thoracoce ntesis etc)

The spleen is derived from mesodermal cells - NOT THE GUT TUBE

The spleen rests on the left colic flexure associates with the tail of the pancreas Know the

structures entering the Hilum of the spleen

Sh rt O~-t~ic 1 0(0 10 rtiltSPIric Iloa nt

(cut)

Peritoneum - similar concept to Pleura - think of a fist in a balloon

Visceral Peritoneum - Layer of balloon touching your fist

Parietal Peritoneum - Layer of balloon not touching your fist

Your fist represents the organ your wrist is the hilum and your arm contains the blood supply

entering the organ

Appreciate that there will never be organs in the peritoneal cavity - rather these organs invaginate

the cavity Kaplan videos

RULES OF NOMENCLATUREshy

1 Organ completely surrounded by peritoneum - peritoneal organ

2 Organ partially surrounded by peritoneum- Retroperitoneal

3 Peritoneum surrounding peritoneal organ is VISCERAL peritoneum

4 Peritoneum surrounding retroperitoneal organ is PARIETAL peritoneum

5 Peritoneum connecting visceral to parietal is called messentary 2 messentaries in the

gut Dorsal (to the gut tube) and ventral (to the gut tube) messentary

Aorta is in Retro peritoneal position - but blood must reach peritoneal position - vessels travel through

messentary All peritoneal organs will have blood supply reaching through messentary

-Mesentery is a 2 layer peritoneum with a neurovascular communication between body wall and organ

- Ligament connects one organ with another or to the abdominal wall (Ommentum = ligament)

lesser Ommentum (attach lesser curvature of stomach and duodenum to liver) =Hepatoduodenal

Ligament and Hepatogastric Ligament

Has a Superior and Inferior Recess (Accumulation of Fluid in Ascites)

Communicates with the greater sac through the epiplic foramen (what structures pass through

this foramen)

Boundaries - you must be able to visualize this

o Anterior - stomach

o Posterior - parietal peritoneum pancreas

o Superior - superior recess (bw diaphragm and coronary ligament)

o Inferior -Inferior recess (bw layers or greater momentum

Greater Ommentum (attach greater curvature of stomach) Gastrophrenic ligament Gastrosplenic

ligament gastrocolic ligament

The greater omentum is the largest peritoneal fold It consists of a double sheet of peritoneum folded on itself so that it is made up of four layers The two layers which descend from the greater curvature of the stomach and commencement of the duodenum pass in front of the small intestines sometimes as low down as the pelvis they then turn upon themselves and ascend again as far as the transverse colon where they separate and enclose that part of the intestine

ABDOMINAL PAIN

Parietal Peritoneum - supplied by same vasculature lymphatics and nerves supplying body wall it

lines and diaphragm Sensitive to pain pressure heat cold well localized

Visceral Peritoneum - supplied by same vasculature lymphatics and somatic nerve of organ it covers

Insensitive to touch heat cold and laceration - referred to dermatome of spinal ganglia providing

sensory fibers Where does appendicitis refer to

Foregut pain - epigastric area (ie - cholycystitis)

Midgut pain - periumbilical area (ie - appendicitis)

Hindgut Pain - suprapubic area (ie - diverticulitis)

Extra ImagesConcepts

ll~_____-

FalifCtrm ligament oind r~ud ligamet f Ilver

Blood from splenio gastriC and inferiof rne$e-rteri v~ins

Ca-I tributaries

Lett gastrio Ifein

Posterior superior pan~reatioodul)denal vaihS

Lott gamo-om~nlal (9aropip lomiddotic) -in

Poq_~ tjol imerl-9-r panCJertlcorllJod-nal veiopound --amp----I- - ~J Right grtr~-omntal

Anwrior interi (gartroepiploic) Jjn

pan euaii cod vl)denal veins middot Inf~Ji (t r mesentric vein

Miqdle (olic vein

Right cl)licvein Sigmoid and rectosigml)id (ei ns

IhH)Collc(~io

--- Mi~dl laquooLJl gtjrltgt

PoM ca vl1 illasto)moses -----shyampoptoageal 2 Paraumbilie-lt11 Inferi or Fectal vei ns

3 Recial 4 REuoperHonea1

Know how the Portal vein is formed I 4 sites of portal caval anastamoses and 1 clinical shunt

Col li t ltt-~ otTl~tI ~nj pc~ 1lt1 turJoG

Ltf 14i1 tImiddot~ artoftl9 on tj phtAt$

L-oftqf 4t t~r 1=laquoIran d 1 bull shy~p l ci rj o fOOOts

Nerves follow the arteries - appreciate the splanchnic nervous system I

Uet~ric branch of left ~nal art

Ureterie branch of righi renal artelY

Left Zld lumbar in and co mlTlunication to as)erdin9 lumbar l(~in Hi ~ht tEZ1~~t~ t3r j t itn ~ nJ l1t- rlnd lfe i r1

Inferior me5nteri~ artery

Notice that the right testicular vein drains directly into the IVC and the right testicular artery drains

directly into the aorta However the left testicular vein drains into the L renal vein at a right angleshy

reason left testicle is lower and more susceptible to varicocele (bag of worms)

Also notice that the left renal vein has a longer course because the IVC is on the right side whereas

the right renal artery has a longer course because the aorta is on the left side

Appreciate the anterior to posterior relationship of structures in the hilum of the kidney - VAP - Vein

Artery Renal Pelvis (Ureter)

11____ __ L_ L_ n VJ __ _ _ t_L I I_ _ L __ L_ I -pound1 bull LI_~-I ____

Posterior View of Head of Pancreas in ( of Duodenum

Celiao hunk

Co mmon ~L~jJth art~ry

GastNduQdonal artrf (partilly in phantn)

P1)Sterior $Up~Jior panCflaticuduodfmal art~r~t

(Co mm on) bile duct

middot~1t~~t-1l---~-~- Right gshomiddotomental (gastoe plp lolc) 3rte (phantomost)

Grener paocre atic art-ry

1n1~rjor pancr-iatlc artery

Jtrifll supejo r pal)oreailcento)dJodenal artr1 (phantom)

Anastomotlo branch

POostetlor bJanch of jo f~ri of pan-reatir(lduodensl drttnj

Anterio r branch of i flferior palcreati~)duodenal art~(phan1om)

Notice the extensive blood supply to the pancreas and duodenum via the branches of the celiac trunk

Notice collateral supply from SMA branches - makes sense bc this is the jxn of foregutmidgut

Identify the vessels in this arteriogram

Hiltid i)f N~ck oi B)dvof Tail 01 pa nereas pan cent~as P-nmiddot-reas panCtCas

I nferie v~na cava

jHept1iic p(lrlai v~in

Port1 tnd H~pti lt a ftH prol

Comm on) bll duct

Ouodtnum

~ft colic (sio)Atta~ hmtrlt jt~xJr-ofha~elSe

muo(IIQn

Right ~lIc (h~j)tic)

il~gtture

In1triol m~oten lIein (rttr op~ritoMdO

SlJp efl or mes~n~fiC amrV and lipln

KNOW YOUR NEIGHBORHOOD

Questions

vVhiJh structure supplied by a bnmdlof the cclia( artery is not derivcd from foregut LemCJUCrITI

(A) Head of the pancte-a5

CD) Pyloric duolenum

Cystkduct

( Liver hepatocyt~~

~F) Body of the spleen

An infant presents with an omrhaJucele at birth -hi oJ the [oHm illg applies to his cM1-dition

(A) It is 31so seen ill p4titnts with aganghonic megacolon

(11) ft reuirs from a fal1ure of resorption of theviteUine d let

(C) It results from herniation at the-site of regression of the right umbilk vein

DJ It is caustd by faihtrc of recanalization of the midgut part of the duodenum

~ It ill camioo by a failuIt vf the midgul to return to the abGQminal uity after herniashytion in-n the urnbilk s l stalk

Ot er than the spleen occlusion Cif the spit-Ilk artery at its odgin wm most likely affect die blood supply to jllch st cnud

(A) Jejunum

(B) Body of th pal1~lltas

(C) LeSStT Cllmiddotlaturc of tl )toma-ch

(D Duodenum dista to the entrance of the Ornmou bile duct

E Fundus of the stomach

A 38-yeu-old batL~er with a history of heartburn suddenly experiences excluciating pain in the (plgastric region of th~ abdomeu SurgCry is perf~rme immediard y upon admisshysion to the 1IlcrgCJliy tuomh~re i~ evidence uf a ruptured ulcer in the posterior waU of the stomach Vhere will a surgeon first fi nd the stomach contenlSf

A) Greater p4ritoneal sac

rB) Cul~de-s~c of Douglas (--

C Omental bursa ~

--D) Paracolic gutter

rEj Between -he panttal perimltum and the posterior body wal1

At birth an infant presents with a st()ma~ rb~tbas~njJled jfltotb~diaplfagru 1A1ltre is the defect thatresulied iiitJle heini~t()n shy~tsophagealbiatus

7 - rH-- Hiatus for the inferior vena cava

( Pleuroperitoneal membrane -(0) Septum transvcrsum

(E) Right Crlt~

An infant born with DOVv7l syndrome presents with bili()u~ vomiting Ahat congenital defect does the infant have

(A) Pyloric stenosis

(B) Meckel diverticulum C) Ornphaloce1e

(D) Gastroschisis

( ~ ) Duodenal atresia y A patient with cirrhosis of the liver presents with ~ bacalvaricestnlreased retrograde pressure in which veins caused the varices

(A) Paraumuilical

(B) Splenic

(ct AzygltJus

(15))G~trk ( (-F) Superior mesemeric

A htaltby 3-year~old male patient experiences a hernial sa protruding from the anterior abdominal wall about halfway between me anterior superior ilia spine and the pubk tuberde Pulsations of al1 artery are palpated medial to the protrusion site through the abdominal walL Which layer of the anterior abdominal wall will first be traversed by the

1hctma

fA) Rectus sheath (B) External oblique aponeurosis

(C) Inguinal ligament

lD) Transversalis fusda

(E) Cremasteric fa~cia

After 5urgi(aj ffpair of a hernia the patient tXperienccs mtmlgtness in the skin on the anteshyrior aspect of the S(Totum_ Vhaf nerve may have been lesioned during thehemiorrhaphy

(A) Femoral

(B) Obturator

(C) Ilioinguinal

(D) lliohypogastrk

(E) Pudendal

A 23~year-LJld female secretary il1 good health ~-uddcn1) doubles over with pain in the a ea of the 1JmbRicu$ Sbe feels vartn and ltneasy and has no appetite That night the pain seems to have mQved to the tower right abdominal regjol1 and she calls her family doctor who then arranges for an ambulance to pk-k her up and take her to the hospitaL Wh ell ntn~ perceived in the area of the urnbilirus most Hkely carried lhe pairfu I sensations into the eNS

tA) Vagus nerves I~

V B)

) Lessersplanchnk nerves

tC) Pudendal nerves

(D) lIiohpogastrk nerves

(E) Greater splam ic l erves

A CT reveals carcinoma in the bOod of the ancreas Vhich blood vessel trut ourses ----~- - -bull ------ --shy

immediately poftterior to the body ofthe pancreas is the m~t likely to be oompressed

(A) Splenk artery

(B) Abdominal aorta (C) Portal vein

(1) Splenic vein

(E) Renal vein

A patient has a penrln1l1ng uker of the posterior wall ot the br~l part ot the (lUooenmn llkh blood vessel is subject to erosion

(A) Common hepatic artery

(B) Gastroouodenal artery

(C) Proper hevatic artery

(D) Celiac artery

(E) Anterior inferior 11amrelltlcoduodcnal attery

Your patient has been diagnosed -ith a carcinoma locallted to the head and l~e(k of the pancreas Another clinical sign would be

A esophageal varices

(8) hemorrhoids

C) a caput medusa

(D) increased pra Teuro n th~ hepatic veins

(E) enlarged right supra lavkular lymph nodes

Wltkh of the foUowing structures develops in the ventral mesentery

(A) Spleen

(B) Jeiunum (C) Head of1ht pancreas (D) Transverse colon (E) Stomach

ti l Uw ~ littwin~ f( S-t lil oai Imdge ~ hi(h or tbt la~)d J truetur tgt liJ llntn nl) he hl p UC iJd [IIi ell

c o

A) drains Ie tht infCrior a La aI

R t middot~nfl0 ~ill to th~ lunlgtn of h i dtlndCrlllfH

(e) m t bull JiJattd on tl l J n T ~H

D ) sup Lc O VSlt I Hlid bhtu l 1 li - -I un oid

( ) U~tpli(t tr j middottUh~ 1 v(( b~nt rfK n1ilc~Zm

ANSWERS AND EXPLANATIONS

Answer E The spleen is t hlttnopodicand lymph organ demlted from mesoderm

Answ~ R Al1 tlmphalocele is caused by it failure of the nlidgut to return to the ahdomir nat cavity after herniation into the umbiliau Stalk Choices Aand D maybe seen in infants with Down syndrome choice D ~s the specific CBuse ofduudcnal JtiCSitt Choice C is (ile cause of gclstrosbisis and Choice B nsults iu a Meurolktldivertku1-tlB

Answer B The fundus ofthe stomach is suppHed by soort gastric brunches of the splenic altery The splenic artery supplies the body and tail of the pancreas part of the greater curvature of the sttmla(h and the spleen Te jejunum part of the head of the pancreas and tht~ duodenum distal to the entrance of the commOll bile duct are supplied by the superior mesenterk artery clll~l ~be less r ctlt1ature cmd the pylQric antrum are supplied by the right and lei gastric art(ries

AnSWftt C Tbeomental bursa or lesser ~ritoneaj sac lies direcdy posterior to the proxshyimal part of the duodeTtlm and the stomach and would be the first site where stomach contents ~Ott1d be fpoundluncL

Answer C A defect in a llleuropcritoneal membrane (uswlly the left) is the typical site of i1 cc-ngenitlI diilphragluatic hemia llere the membr4ne fails to dose ()pound( of the perishycCirdiopcritulleal canals

Answer E DuoJenal atresia and aganglionic megacoion are congwitaI defects S~Il in patients with Dowmiddotnsyndrome

Answer D RulaTgemt~llt of and retrograde flow in g~lstrk vel_ns in particlJl~r the kft gas~ tricveins dilates the capillary bed in rhe wall of the esophagus in (ases of porta yper~

tension Blood flow would increase in and dilampte tribntarkgts of the (lZygOUS vein on the other side of the capiUary bed but flow in this vein is in the typical direction t()ward the superior vena cava Paraumbiii(ltU vein eilgorgement contributes to a caput medusH Splenic ~nlargement might prc~nt with 5plcnonlegaly and balt-kflow in to tlu superior m~~ntclic vein occurs but is asymptomatic

Answer D The patient hagt an indirect inguinal hernia whi~h emerges from the antt-rior abdominal wall through the deep inguinltilling Theeep ring is a fault in the transv~rshysaUs fascia this I~yer wiIJ be penetrated first by the hernia

An~Wer C The ilioinguinal nenc which provides sens~llion to the lnedlal thigh ltmclanteshytior SClotunl pass~lt th rough the 5uperfh_ial inguinal ring ind $subject to inj i1T) becaus-e

it is in the operatitm Held of the erniorrhapny

Auswer B The leMHr splanchnic nerves are sympathdic nerVlts that carry viscera l sensashytlltgtrogt ftom illtllt1m~d ()J stietched gust (itinteitinal ~tructures (in this case the pprndix) into tnt eNS Lesser splanchnic ntTYcsarisc from thmiddot T9--T12 spinal cord segments lt1nd provide sympathetic innenation tD rnidgut siruc1ures whiCh include CLe app~JldD Viscera] Pain arising from affecLed Inidgut ampt 1C1ure is referred over the same dl- matorne~ of spinal segrnertts v-hich provide the sympathetic Innervation n this G1SC of appendicitis the invohen~n t of the ltire) of t e unlhHku indud s the T 10 dermatome

Answer B Of the five choices onty the dscending olon is retroperiton~al aldwould be a lik ~ ( choice to be seen immediately a(~jilcent to t11e posterior abdominal middotn~L

Amwen D The SpltftlC ~-ein ourses posterior to the body of the panneas m its way tt drain into the superior mCSfttltlri( vein

Answcr B TILt glstrodllolticnal artery 1 direct hIamh of the comrootl hepatic artery courses immediately pt))iwri() to the duodenum and is slbject to erosion

Answer B Carcinoma of th pan middott3S in the 1tilt1 may compreampgt the portltil vein at irs orishygill The poTtai vcin is fomled when the splenic vein jQiaswith tfie superior meStllt eric vein The inferiot mesenteric vein joins the ~plenjc vein just priOT to tlli~ point at which the splenic joins the superior Jlleit1ltcri( vein Increescd venous presslu in the inferior mesenteric vein is a cause of emo hoid~

Answer C The- velltral pancreas wilich forms most of the head of the p ~ncr as develops in the ventral mes(ntery as antutgrowth of the hepatic diverticulum Th~ hepatic divershyticulull induding the biIJary appa~atus develops in tbe ventral mesentery of the foregut

Answer~ A The superior mesenteric ~in joins with the spienkvein to form the hepatic portal vciu

Answer D The structure at gttlK is the proper hepatic artery~ whkh suppUesoxygenated b middotood to the liver

MAKE SURE YOU KNOW the diff bw Rectus Sheath above and below the arcuate line

ABOVE

Aponeurosis of xiiltmal obllque musclo

Extemll f)biquw musde

Reotln ilbdomlnls musole S~in

Internal 9bliquQ mY~QI

AponeUfOsi$ of hJH$V~~S Lir9a a lb lbdolTlin~ musolo Tri OJV6 rUi

atldomlnis mUS(loe

Sub cutanlilous tiue (tatty ye r)

BElOW

A POrl lJfosis 01 etemal oblique muscle

Aponeul~)sis 01 Internal oblique mU$cl~

Anteriol lay~ of r~ltdus st~ath EXttom1 oblique rnu$cll

Rectus Jbdominis muscle Intoernal Aponeurc-sis of tra~fersU$ oblique muscle-

at-domlnis muscentl ~ Skio

Tra nsvitSus abdomioLs ml)ZClt

TralSVersaHs fascia Medial umQil iegtt1 1i9Jment -and folj

Uldchus Peritoneum (ir median Umbilj~al Suboutane ous

Extraprftone 11ascia

Ymbilimiddot~1 fold)

preu9poundiea1 fascia

tissue (fatty 4nd m~mbr3n(iUS layers)

o Above the arcuate line (A horizontal line 13 of the distance bw the umbilicus and the

pubic symphysis) -10 Aponeurosis divides into an AntPost Laminae

o The Ant Laminae joins EO and Post Laminae joins Trans Abdominis = Ant and Post

RECTUS SHEATH respectively

o BElOW the arcuate line - all 3 aponeurosis join ANTERIOR to rectus muscle to meet its

counterpart in the midline (linea Alba)

o Take away Msg - The abdomen is devoid of a posterior rectus sheath below the

arcuate line and is therefore more vulnerable to herniasinjuries

Question - A physician makes a deep incision in the patients midline immediately superior to

the pubic symphysis which of the following layers is his knife least likely to pass

Rectus Abdominis External Oblique Ant Rectus Sheath Posterior Rectus Sheath All of the

Above

Answer - All of the above None of the other answer choices are midline structures -LINEA

ALBA

Linea Alba has very poor blood supply - doesnt heal well after surgery Therefore this is a

common site for incisional hernias

a Spleen b Transverse colon c Descending colon d Stomach e Pleura

17 Meckels diverticulum is normally found 2 feet proximal from the

a Pyloric sphincter b Lower esophageal sphincter c Ileo-cecal valve d Middle valve of Huston e Anal valve

18 Ulcer in the posterior wall of the first part of the duodenum would erode ___ artery and would cause bleeding

a Left gastric b Right gastric c Hepatic artery proper d Gastroduodenal artery e Middle colic artery

19 An inflamed appendix is identified by a surgeon on the operation table by noting

a The appendicies epiploicae b The convergence of tenia c The artery of Drummond d The mesocolon e The mesosalphinx

20 The nerve which emerges through the psoas major is

a Femoral b Ilio-inguinal c Ilio-hypogastric d Pudendal e Subcostal

21 The right gonadal vein drains into the

a Azygos b Hemiazygos c Inferior Vena Cava d Right renal vein e Left renal vein

22 The hepatocytes in the liver is derived from

a Ectoderm b Endoderm c Mesoderm

d Neural ectoderm

23 Abscess in the lumbar vertebrae due to tuberculosis would spread to the adjacent muscle which is

a Psoas Major b Iliacus c Quadratus lumborum d Tranversus Abdominis

24 The anterior wall of the inguinal canal is formed by

a External oblique and transverses abdominis b External oblique and fascia transversalis c Internal oblique and external oblique d Internal oblique and transverses abdominis e Fascia transversalis and peritoneum

Meckels diverticulum is a result of which of the following developmental abnormalities shy

A Failure of the vitelline duct to close

B Failure of the herniated intestinal loop to retract into the abdomen

C Failure of the urachus to close

D Failure of the midgut to rotate

E Failure of the hepatic duct to close

Explanation

Meckels diverticulum is a result of the persistence of the proximal part of the vitelline duct This

diverticulum is usually found about 2 feet proximal to the ileocecal junction and is usually about 2 inches

long It is present in about 2 of the popUlation It may be the site of ectopic pancreatic tissue or gastric

mucosa and may develop inflammatory processes and ulcerations Acute Meckels diverticulitis

simulates appendicitis

Which of the following veins carries blood from the esophagus to the portal vein The

A right gastric vein

B left gastric vein c splenic vein D azygos vein

E left gastroepiploic vein

Explanation

The left gastric vein a direct branch of the portal vein drains blood from the lesser curvature of the

stomach and the inferior portion of the esophagus Because branches of the portal vein do not have

valves blood can flow in a retrograde path when there is an obstruction to flow through the portal system or liveL Rlooci Cln then flow from the nortl] vein thr()1Ph the left PRstric vein to the esonhlPlIS lno

through venous communications within the submucosa of the esophagus to esophageal veins that drain

into the azygos vein The increase in blood flow through the esophageal submucosal veins results in esophageal varices

On the posterior wall of the abdomen the celiac ganglion A contains cell bodies of postganglionic parasympathetic neurons B is synapsed upon by neurons in the posterior vagal trunk C is synapsed upon by neurons in the greater splanchnic nerve D contains sensory cell bodies of lumbar spinal nerves E contains cell bodies of neurons that cause an increase in the rate of peristasis

Explanation The celiac ganglion is one of the preaortic ganglia of the sympathetic nervous system It contains cell bodies of postganglionic sympathetic neurons The sympathetic splanchnic nerves contain preganglionic sympathetic neurons that pass through the sympathetic chain without synapsing These splanchnic nerves go to the preaortic ganglia to synapse The greater splanchnic nerve contains preganglionic neurons from spinal cord segments T5-T9 This nerve synapses in the celiac ganglion The nerve fibers in the vagal trunks are preganglionic parasympathetic fibers that go to the walls of the organs that they will innervate and synapse on postganglionic parasympathetic neurons in the walls of those organs Cell bodies of sensory neurons in the abdomen are found in the dorsal root ganglia or the sensory ganglia of the vagus nerve Sympathetic innervation decreases the rate of peristalsis parasympathetic innervation increases the rate of peristalsis

Which of the following pairs of arteries will allow blood to bypass an occlusion of the celiac trunk

A Left gastric artery-right gastric artery

B Left gastroepiploic artery-right gastroepiploic artery

C Superior pancreaticoduodenal artery-inferior pancreaticoduodenal artery

D Splenic artery-common hepatic artery

E Left gastric artery - proper hepatic artery

Explanation The anastoOlosis of a branch of the celiac trunk and a branch of the superior mesenteric artery will

provide collateral circulation around an occlusion of the celiac trunk Each of the other choices pair

branches of the celiac trunk therefore these will not provide collateral flow around the obstruction of the

celiac trunk The left gastric splenic and common hepatic arteries are direct branches of the celiac trunk

The right gastric artery is a branch of the proper hepatic artery which is a branch of the common hepatic artery The left gastroepiploic artery is a branch of the splenic artery The right gastroepiploic artery is a

branch of the gastroduodenal artery whlch is a branch of the common hepatic artery

Which of the following organs has appendices epiploica The

A sigmoid colon

Bjejunum

C duodenum

D stomach E esophagus

Explanation Appendices epiploica are characteristic of the colon Appendices epiploica are subserosal accumulations

of fat None of the organs of the gastrointestinal tract has appendices epiploica except the colon

Page 7: Chirag's Abdomen Review

L sgt-J[ offiEntumHpatic artY proPll

8iloduct bull

Stomach

Glstrapllmic ligament

Ornootnl-+----+r--___-----cT-- ~Jrarn n

IH---V~tal

petitOllllU-m

Splllllofffial figamgn

TXH

LIVER - ClinicalDryWet

Coronary Iigameot

Ewph3g~al

Impression

Lett tria09ular Hepatic veins

ligament

Fibrougt Supralerfal impreurossionappendix of

tiOer Hoparorenal portion ot coronary ligament

Garuic RighitrianQularl igameot impresgtion

(Common) bile duct

Common hepalic duel

Clstic duel

Renal Impression Caudate lobe

Papiilary proc~ss Quadrat~ lobe

allbladder

___ FalCltltorm ligament

Hepatic artery PlOP - Round Jigam~nt 01 liver

Hepatic portal veio ~-- Coli c impression

FisUH for ligamen1vm teres

Porta hepatis

Make sure know

Embryonic origin of the iigaments

Anatomical vs Functional lobes and relationship to the Gallbladder

Portal Triad Structures (VH DISSECTOR)

Impressions

CLINICALS -

Trac heoesophageal Almocml COll 11ction htteell esophalIs ll nd tmlthe1 fistula Jost lI)OUllul1 StLblypt is hlind up tr () hngtls with lOler esophagus tO)HHcted to

lrac hea RcsulJ in (3 11 $($ (ho tng anc vo mitin with ftt(~~lS lir bllblllpoundQll CXR and )Qlvl V -~-

I ~ i I Esophageal y i J atresia - - Trachea

----shy

Z

Congenital pyloric l1l perlrnphy (J f le pylurus camt~ obstruti I) Pa Ipablc uhve mass in epigl~tric n~ion stenosis and nOl1biliolts PfOiL mnit+rrg- - - wetk of ltl4C Tn)tment is suriltai ineisi n

Ottmi in 160(llit births ofteIl in I ~ t-bOfl malts --

Peptk ulcer djsease G~ltri( ulcer Pain can he greater vdth mellls--Welghtl - ft(H OCClIlS in older putient

a Mlori tnfedion in 70 chronic NSAID use also implicated Dm to llHue-oml protection a~lllns t gfltrh l id

Duodella) ulcer Pain Deu(QSe5 wilh menh--weighl ~aln_ Almost IOO have H priori infection Dut to 1~ gastfk acid cerct 11 f( 1 2011 inpmiddotr-Ell lson )~md rol11t i ( r L m(lcos~ ll

protction Hyprttophy of Bntnllcr glands Tend to ha ve ltIeall pullcheJ-ouf margim unlike the tais(dJirre~lllar margins of

cnCinOl1lH P-te1itiai eomplications include bl eedin~ penetrnu(n into pnnerelS perfomtion and obs tmctiQH (not intrinsltllly pr((JIlcerom) (see Image 11-+

Question - A 57 year old obese chronic alcoholic presents with an ulcer which has ruptured the cI~wnpastelgrly Surgical investigation reveals blood in the peritoneum Which of shy

the following arteries is most likely responsible for the bleeding

a Splenic Artery

b Gastroduodenal Artery

c R Gastric Artery

d L Gastric Artery

e L Gastroepiploic Artery

Appendicitis All age ~rol1ps most common indication for em er(nl ilxkminai)lJrger) in ehildrefL huLial diffmc periumbilical pain --7 localized pain JtMcBmm~ point N~ltlSC~l kvef

r)13)- perfor~te - pcntouitis Difftr~ntiI1 divertieuli[i (elderly ectopic pr(~gnanl to [3-hCG 10 rule ont)o

-----

PcmiddotrhtltllCf of the ) lcllille dud Of ~Olk sh31k ~br lontli n e( topic ~l cid-seefehllg gatric m 1(0]

andor pmcrcatic ii~~lle Lllost common c(lllgenital anomliy aftlle CT tract Cm CllHt hkfciing illtusm~(~pjjon Dlnllus or nbstrudion nelr the tcrrnin)l ileum Contraslwitll QIB~efic nmiddot = cvtk dilalaUon of ittllilC dJet

------~-

The ile 2$

2Jpound~11 11~ 2 feet frolll the iie( middoteCill vke l~ QfiJ~ at 1~

CIllIn nly prcsenfltgt III rll~2

llf~ of lifc- by ilwe 2 ty )ts of

epilheH8 19ls-trie- pal1elli(i

Hirschsplungs disease

Congenital tnt91(middotolon characterized by lack of Think of a gian ~pring that ganliml ttlLJcnJ(rk ~~gJeXllS~( (~lihs and lei ~sner plCxpstTlrlsgIllent on inbstinal biop y ()It to-iIure of U(middotural t~restpoundtU migration =

has s nl

Presents as ronic comtipnHoll tHly in life Dilllted pOltioll of the colon proximal 10 the aganiionic

segment resulting in a middot trmsjtillll ZQl1t rnvolt~

rectum [huany farlur( to P~$ meconium

High Yield WetDrylTheory List-

Suggestion - in your study group try to write a question for each of these points and then

exchange with a friend and try to answer each others questions

Abdomen Blood Supply - Reference viks picture posted on my google group - this is THE MOST HIGH

YIELD TOPIC IN ALL OF ANATOMY - expect 5 questions on your mini and 5-10 questions on your shelf

Make sure you can draw the blood supply and answer tertiary questions

Example - If the patient had an occlusion of the celiac trunk - which of the following areas would

experience ischemia

Portal hypertension - Know the 3 clinically relevant sites of portal caval anastamoses

Testes vs Scrotum lymph drainage

Where are paraumbillicai veins located

Omalophcele - failure of the gutto come back in (if in yolk sac - fatal)

Marginal Arteries

Superior Messenteric Artery is in front of 3d part of duodenum

Caput medusa (Distended paraumbillical veins secondary to portal hypertension)

Kidney Constrictors -1 Renal Pelvis 2 Crossing Pelvic Brim 3 Entering Urinary Bladder

Ureter - wet lab

Vagus is PIIJS supply up to 23 trans colon than pelvic splanch N up to the ass

Hirshsprungs disease - baby cant poop - dilated colon

Meckels Diverticulum - rule of 2s - 2 feet prox from ileocecal ju nc

Urachal Fistula - weeping belly

Gall stones - common bile duct

Jaundice relation to tumor of the head of the pancreas

Hepatopancreatic ampulla

Blood supply of renal gland - s superrenal art m s suprarenal a abd aorta inf suprarenal art

R Kidney - Tl2-L3 L Kidney - Tl1-L2

Renal Artery - L2

Epiploic foramen - know the borders and contents

Alantois diverticulum - urachus - medial umbilical

Lateral Medial and Median umbillical Folds (know the contents)

Directindirect hernia - know how to diagnose where they enter and exit the inguinal region and which

one is congenital

Anular pancreas - projectile vomiting

Pyloric Stenosis - projectile vomiting (non bilous)

Duodenal Atresia - projectile vomiting (bilous)

Dry Lab - know x rays

Vitteline Fistula - food out of umbilicus

Major duodenal papilla - junc of foregutmidgut

Arcuate line - relationship to rectus sheath

Mcburneys point -13 from ASIS bw umbilicus

Parietal pain - what is the nerve supply

Internal Oblique - cremasteric relationship

Know spermatic fasia

Processes Vaginalis - connection bw peritoneum and gubernaculums

Umbillicus - TlO dermatome

Deep Inguinal-l25 cm above mid inguinal ligament

Superficial Inguinal Ligament- superolateral to pubic symphysis

Variocele - veins engorged in scrotum (bag of worms)

bull

bull External spermatic fascia derived from external obliques EO II Cremasteric fascia ~ from internal obliques fO bull Internal spermatic fascia derived from fascia transversalis bull Tunica vaginalis derived from processes vaginalis directly rests on testes bull know order from testes out to skin

note reflex o ilioinguinal nerve o Efferent =genital branch of the genitofemoral nerve

--lt gt-- info important anastamoses which connects thorax to abdomen

bull Sup

o Sup epigastric branch of internal thoracic o Inf branch external iliac

Venous drainage o Above umbilicus aXillary v o Below umbilicus veins in triangle o At level of umbilicus Paraumbilical veins -gt drain into the portal V

II Important in Portal Caval Venous system Venous drainage of testes

o Clinical correlation Varicocele 11 vein drains into IVC 11 Left testicular vein ~ drains into left renal v

bag of rmlt

for lymph drainage T10 axillary lymph nodes

ill Below T10 superficial inguinal lymph nodes (lateral

Umbilical Folds

Lateral umbilical folds inferior vessels

Medial umbillcial folds umbilical (fetal remnant)

Median umbilical fold urachus (fetal remnant)

Between these folds fossas o Supervesical fossa between median and medial folds

11 bladder o hesselbachs between medial and I folds

II DIRECT HERNIAS HERE Borders

Medial semilunar line

Lateral info Epigastric

Inferior inguinallig o Lateral Inguinal Fossa beyond lateral fold

INDIRECT HERNIAS HERE II Deep inguinal ring (lateral to inferior epigastric a)

Indirect inguinal hernia o Lateral to inferior epigastric a o more common o When inserting finger in superficial inguinal ring will feel on tip of finger (since it goes

throueh ineuinal canall

----

Dry Lab - Label subcostal iliohypogastric Ll Ilioinguinal (Ll)

Horesshoe Kidney - stuck under IMA

Renal Agenesis -failure of the ureter bud to develop

Double Ureter

Unilateral Agenesis -1 kidney

Kidneys - Metanephros

Fetal kidneys are at sacral level

Look at 3rd part of duodenum

Some of this stuff is repeated I know just copied and pasted a bunch of stuff I had copy

Dermatomes

bull T4 nipples

bull no umbilicus v o Pain referred to no in appendicitis o Pain referred to T7ITS in gastritis ~

Inguinal ligament = external abdominal oblique aponeurosis

bull Inserts at anterior superior iliac spine to the pubic tubercle o Why important to know -7 visualizing this line allows us to properly diagnose a hernia

Below the inguinallig femoral hernia Above the inguinallig =inguinal hernia

Also to palpate the deep inguinal ring you go about 12Scm above the mid-inguinal

point

bull Modifications to ligament o Pectinate ligament o Lacunar ligament -7 cut this ligament to relieve strain i~ stran ul~tEd hernia

Inguinal canal

bull in males -7 transmits spermatic cord o important structures of spermatic cord ductus deferens testicular a genital branch of

the genitofemoral n pampiniform plexus of veins bull in females -7 transmits round ligament

Borders

bull Floor -7 inguinal ligament + lacunar ligament bull Anterior -7 aponeurosis of external oblique + internal oblique bull Roof -7 internal oblique and traverse abdominal bull Posterior -7 transverse abdominal + transversalis fascia

o Reinforced by conjoint tendon bull Aponeurosis of internal abdominal obliques and transverse abdominus bull Lies immediately behind the superficial inguinal ring in what would otherwise be

a weak point in the abdominal wall bull Innervated by ilioinguinal nerve (Ll) ~why important

bull In appendicitis Ll can be injured which will injure this nerve and in turn

the conjoint tendon With loss of innervation to this supportive structure the patient is now predisposed to a direct inguinal hernia

o Only hernia that can transverse the inguinal canal o Associated with congenital condition persistent tunica vaginalis

bull Direct inguinal hernia o Medial to inferior epigastric a o When inserting finger in superficial inguinal ring will feel on back of finger o Associated w old age or recent surgery

Muscles (Abdomen RECTUS SHEATH)

bull Arcuate line at level of ASISor 13rd distance between pubis and umbilicus bull Above arcuate line rectus abdominus is surrounded by a rectus sheath anteriorly and

posteriorly

o EO and 10 lie over rectus abdominus o 10 and TA lie behind rectus abdominus

bull Below arcuate line rectus abdominus has no rectus sheath posteriorly o EO 10 and TA lie over rectus abdominus o Transversalis fascia lies behind rectus abdominus o Inf EpIgastric vessels pierces the rectus sheath here

Peritoneum serous sac which encloses most of the abdominal structures

bull Ovary =only intraperitoneal organ o Oocyte ejected from ovary then captured by fallopian tubes o Why impt Women more prone to infection that can enter peritoneum

Peritoneum forms

bull Mesentery double layered fold of peritoneum formed as the organ was pulled in

bull Ligament between 2 organs in general bull Omentum between stomach and another organ bull Bare area area of no peritoneum bull

Viscera innervation

bull Pa rasympathetics 11 o Afferents sense hunger o Efferents l peristalsis relaxes sphincters gland secretion

bull Sympathetics o Efferents do opposite o Afferents CARRY PAIN SENSATION OF THE VISCERA (dull stretching pain)

bull PARASYMPATHETIC INNERVATION o Vagus nerve 7 _1l to 23rd unct ion of la rgej nte~tine oJ)elVrcspla~~~)~~ic~rYe~ IJiU- ~rd aJ~lpoteotiD~ IMPT

Gut Embryology

Gut ~ We say that the gut is derived from endoderm We often forget that when we say so we mean

that only the mucosa is derived from the endoderm The submucosa and the muscle layer is actually derived from the splanchnopleuric mesoderm and the serosa is derived from the visceral peritoneum

~ The main function of the gut is to digest the food which is done by the glands derived (and are) in the mucosa (endoderm) The only two exceptions in the Gut where glands though derived from the endoderm do not stay there but migrate down into the submucosa are esophagus and duodenum These glands however have their ducts opening to the swface of the mucosa

bull

~ Lungs liver amp gall bladder and pancreas are off-shoots from the foregut Esophagusshy~ The region of the tube from the laryngeal diverticulum to the beginning of the stomach elongates

to form the esophagus ~ The glands which form in the endoderm (mucosa) migrate down into the submucosa The path

whlch it took migrate becomes the duct of the glands which open to the mucosa ~ Achalasia Cardia - Failure of relaxation of the lower esophageal sphincter because of congenital

absence of ganglia at the sphincter (The ganglia when present releases VIP (Vaso-IntestinalshyPeptide) which relaxes the sphincter)

Mid-Gut Rotation ~ Because of the 90 degree rotation of the primitive stomach all of the following events occur ~ Lesser curvature comes to the right Therefore lesser omentum also comes to the right ~ Greater curvature comes to the left Therefore greater omentum also comes to the left ~ Right side vagal trunk becomes posterior vagal trunk ~ Left side vagal trunk becomes anterior vagal trunk ~ The left side peritoneal cavity comes to the anterior aspect of the stomach and will later be called

as the greater sac ~ The right side peritoneal cavity comes to the posterior aspect of the stomach and is (relatively a

small sac because the liver is on the right) called the lesser sacomental bursaepiploic sac ~ Epiploic foramen of Winslow (the lower free margin of the ventral mesentry) wiII be the

communication between the greater and lesser sac ~ The Liver moves to the right and therefore actually causes the 90 degree rotation of the stomach

The spleen comes to lie on the left side ~ Axis Antero-posterior axis around the superior mesenteric artery

bull Counterclockwise bull Approximately 270deg bull During herniation (about 90deg) bull During return (remaining 180deg)

Duodenum ~ Becomes retroperitoneal (except the first part which is still suspended by the hepato-duodenal

part of lesser omentum) ~ Glands (of Brunner) go submucosal ~ An imaginary line drawn below the opening of the major duodenal papilla represents the junction

between the foregut and midgut ~ Duodenal atresia in Downs syndrome Liver ~ 3rd week

bull liver bud grow bull into the septum bull transversum

~ 10th week bull hematopoietic bull function

bull 10 of the total bull body weight

~ 12th week bull bile is formed

Pancreas ~ In about 10 of cases the duct system fails to fuse and the original double system persists ~ 3rd month

bull pancreatic islets (Langerhans) ~ 5th month

bull Insulin secretion ~ Annular pancreas

bull The right portiCn of the ventralbud migrates along its normal route but the left migrates in the opposite direction

~ Complete obstruction of duodenum ~ Accessory pancreatic tissue Polyhydramnios (Amniotic fluidgt 1500-2000 ml)

~ Congenital defects including central nervous system disorders (eg Anencephaly) and gastrointestinal defects (atresias ego Duodenal esophageal) prevent the infant from swallowing the amniotic fluid (failure of recanalization)

Oligohydramnios (Amniotic fluid lt 400 mt) ~ Cl~ldberenal-agenesis

bull Midgut _-_

~ Primary Midgut intestinal loop gives rise to bull Distal duodenum bull Jejunum bull Ileum bull Ascending colon bull Transverse colon - proximal two-thirds of the bull Transverse colon with the distal third

~ Primary intestinaltoop bull ncephalic limb distal part of the duodenum the jejunum and part of the ileum bull ncaudal limb lower portion of the ileum the cecum the appendix the ascending colon and

the proximal two-thirds of the transverse colon bull 6th week

bull Rapid elongation of the cephalic limb bull Rapid growth of the liver bull Intestinal loops enter the extraembryonic cavity in the umbilical cord

bull 10th week bull loops begin to return bull regression of the mesonephric kidney reduced growth of the liver expansion of the

abdominal cavity bull Jejunum -left bull Loops - more to the right

bull Cecal bud -last part (temporarily below the right lobe of the liver) ~ qIDlthaloseJe (Structures COlHLoArts9V~1tion)

bull Through umbilical ring bull 6th to 10th weeks

bull Associated with a high rate of mortality (25) and severe malformations bull Associated with chromosome abnormalities

~ Gastroschisis (Structures coming out are not covered by Amnion) bull herniation through the body wall ----=---=-shybull Into the amniotic cavity bull Lateral right of the umbilicus bull Sometimes the inferior wall fails to develop as a result lower abdominal structures like the

bladder would be exposed to the exterior not associated with chromosome abnormalities ~ Abnormalities of the Mesenteries

bull Mobile cecum persistence of mesocolon bull Extreme form - long mesentery bull Volvulus

~ Distal third of the transverse colon ~ Descending colon ~ Sigmoid colon ~ Rectum ~ Upper part of the anal canal ~ Primitive anorectal canal

bull 7th week cloacal membrane ruptures bull Tip of the urorectal septum perineal body bull Pectinate line

~ Hindgut anamolies bull Rectoanal atresias and fistulas bull Imperforate anus bull Congenital megacolon (aganglionic megacolon Hirschsprung disease)

bull

bull Hindgut

Chirags Abdomen Review - Part 2

Understanding Embryo makes learning blood supply EASY

I I

I t

~ -

)

Table l1r-~ L Adult SUmiddotuctu~SDrj~l Froln Each of he Three Dhisions of be Pringttive GUl Tube t-middot-----middotmiddotmiddotmiddot-

Foregu(

I_ (Celiac Trunk)

Ir-slt-gtphgus

S101na(b

I h -= LiJ~r

Pancre=l S

bull 1 i Biliary apparntu5

Gall bladdshy

i Pha11~Cal pltgtuchcs

LullSS-I

Mjig ---- bull __ _- ----n--duct----~---middot-------l--n

(Superior Jldesen1eric Artery)_-1I-(I_~__ middot __ O-=-)_in_middoto_r_M_e_se_n_t_e_r_i_c_An__

Uuodenu rn 2nd_ 3 lt141h V4Tt

Jejunun-~

nc-un]

tCCUJ11

AppltgtndLX

Transver5e -o1on (p~oxiln1l1 ~O Tbird)

bull__hytgtid~ _ _ ______ L _ __

Tr-dn~llt~se colon (diStul h lTd) I

)

i

Aa ca-nal -( uppeT patt) i

I I

_____ __ _ _ _ ___ - - rhe~ a(t clcriVOkt iV(5 opound~lt prbn1rC ~ nlQC blft TlI)( 134tof r~ tIonoinf~ i 1 ~l l1rd c- P Cle

Now Lets see how much youve learned

Questions

1) A pt receives a general anesthetic in preparation for a c~t~~my A right subcostal incision is made which begins near the xyphoid process runs along and immediately beneath the costal margin to an anterior axillary line and transects the rectus abdominus muscle and rectus sheath At the level of the transpyloric plane the anterior wall of the

-~~-~=--- _eco---shysheath of the rectus abdominus muscle receives contributions from which of the following

a Aponeuroses of the in~ande~tef-Ilal o~ues

b Aponeuroses of the transversus abdominis and internal oblique muscles c Aponeuroses of the transversus abdominis and internal and external oblique

muscles d Transversalis fascia e Transversalis fascia and aponeurosis of the transversus abdominus muscle

A

2) The lat~raJJJ11QjJt~gLfgJlLoneach side of the inner surface of the anterior abdominal wall is created by which of the following structures

K Falx inguinalis (~) Inferior epigastric a

c Lateral border of the rectus sheath d Obliterated umbilical a e Urachus

B

3) A man the victim of several knife wounds to the abdomen during a brawl at the Lobster Shack subsequently developed a direct inguinal hernY Damage to which of the following nerves is most likely responsible for the predisposing weakness of the abdominal wall

~ Genitofemoral nerve ( b) Ilioinguinal nerve ~-t Tenth intercostal nerve

d Subcostal nerve e Pelvic splanchnic nerve

B

4) Which of the following statements concerning a direct inguinal hernia is correct a It is the most common type of abdominal hernia b It transverses the entire length of the inguinal canal c It contains all3 fascia layers of the spermatic cord d It exits the inguinal canal via the superficial ingeJinal ring e It protrudes through H~acb strJg e

~(

1fltbS w E

tl

5) The conjoint tendon is

a Important in preventing indirect inguinal hernias b The fused aponeurotic layers of internal abdominal oblique and transversus

abdominus muscles c Posterior to the deep inguinal ring

d Medial fibers of the inguinal ligament

B

6) A 25 year old male is brought in to the ER after being involved in a car accident in which he received a crushed internal injury in his abdomen Examination reveals a lesion of parasympathetic fibers in the vagJsnerve which interferes with glandular secretory or

smooth muscle functions in which of the foliowingorgans a Bladder b Transverse coloiW c Descending colOO d Prostrate gland e Rectum

B

7) The spermatic cord includes all of the following contents except a Il ioinguinal nerve b Pampin iform plexus of veins c Vas deferens d Genitofemoral nerve

A

8 Which abdominal structure gives rise to the internal spermatic fascia (muscle) following the descent of testes in development

a External abdominal oblique aponeurosis b Transversalis fascia c Transversus abdominis muscle d Peritoneum e Internal abdominal oblique

B

9 Which abdominal structure gives rise to the tunica vaginalis fotlowing the descent of testes during development shy

a External abdominal oblique aponeurosis b Transversalis fascia c Transversus abdominis muscle d Peritoneum e Internal abdominal oblique

D

10) The lesser omentum is a peritoneal fold which is su bdivided into the a Hepatogastric and gastrosplenic ligaments b Hepatoduodenal and gastroomentalligaments c Hepatoduodenal and gastrosplenic ligaments d Hepatogastric and hepatoduoden9-jrj igaments

D

11) A posteriorly perforating ulcer in the pyloric antrum of the stomach is most likely to produce initiallocalized peritonitis or abcess formation in which ofthS fQllowing

a Great-sac - -- -

b Paracolic recess

c Omental bursa

d Right subphrenic space

c

The inferior mesenteric artery arises from the abdominal aorta ilm_ediill~y_J-Qs1eriQLto which of the foowing org~ns A-F~t~filie duodenum B Head of the pan~eis C Neck of the pandeas

D Second part of the duodenum

E Third part of the duooenum_shylaquoshy

shy

The correct answer is E The inferior mesenteric artery arises from the anterior surface of the aorta at the level of the third lumbar vertebra The third part of the duodenum crosses the midline at the level of the third lumbar vertebra and passes anterior to the aorta at the origin of the inferior mesenteric artery The

first part of the duodenum (choice A) lies horizontally to the right of the midline at the level of the first

lumbar vertebra The head of the pancreas (choice B) is to the right of the midline and extends from the

level of the first lumbar vertebra to the third lumbar vertebra It lies within the concavity of the

duodenum The neck of the pancreas (choice C) lies in the midline at the level of the first lumbar

vertebra It lies on the anterior surface of the aorta at the origin of the superior mesenteric artery The second part of the duodenum (choice D) lies vertically to the right of the midline and extends from the

level of the first lumbar vertebra to the level of the third lumbar vertebra

The left adrenaLvein drains directly into which of the following veins A Hemiazygos vein

B Inferior vena cavaee C Left renal veiri -

D Splenic vein

E Superior mesenteric vein

a

The correct answer is C The left adrenal vein and the left gonadal vein (either testicular or ovarian) drain into the left renal vein TheTeft renal vein t~ains intothe- inferior vena cava In contrast the right

adrenal ~~inandnght gonadal veindrai~ gLr~ctJy iQtoJhe iilferiQ[ Vencava -- -

ThehemTazygoS7ein- (~h-~i-~ A)~~c~i~es the venous drainage from the body wall on the left side of the

thorax and abdomen No visceral organs drain directly to the azygos or hemiazygos veins The inferior vena cava (choice B) receives the direct venous drainage from the right adrenal vein but not

the left adrenal vein Remember the inferior vena cava is on the right side of the abdomen The splenic

vein (choice D) receives the venous drainage from the spleen and part of the pancreas and stomach The splenic vein is part of the portal venous system

The superior mesenteric vein (choice E) receives venous drainage from much of the intestinal tract It is part of the portal venous system and joins with the splenic vein to form the portal vein

A 43-year-old man presents complaining of pain in the groin On examination his physician palpates a

bulge in the region of the superficial inguinal ring which he diagnoses as a direct inguinal hernia The hernial sac most likely

A is covered by all three layers of the spennatic fascia B passes medial to the inferior epi gastric artery

C passes medial to the lateral border of the rectus abdominis muscle

D passes posterior to the inguinal ligament E passes through the deep inguinal ring

The correct answer is B Direct inguinal hernias enter the inguinal canal by tearing through the posterior

wall of that structure The typical location for this type of hernia is through the inguinal triangle bounded

laterally by the inferior epigastric artery medially by the lateral border of the rectus abdominis and

inferiorly by the inguinal ligament Direct inguinal hernias pass medial to the inferior epigastric artery

whereas indirect inguinal hernias pass lateral to the inferior epigastric artery because the deep inguinal

ring is lateral to the artery Indirect inguinal hernias are covered by all three layers of the spermatic fascia (choice A) Direct inguinal hernias are covered by fewer than all three layers because the direct inguinal

hernia tears through one or more layers of fascia as it emerges though the abdominal wall The lateral

border of the rectus abdominis muscle (choice C) forms the medial border of the inguinal triangle All

inguinal hernias pass lateral to the rectus abdominis Femoral hernias pass posterior to the inguinal ligament (choice D) Inguinal hernias emerge through the superficial inguinal ring which is superior to the inguinal ligament Inguinal hernias that descend below the inguinal ligament pass anterior to the

ligament Indirect inguinal hernias pass through the deep inguinal ring (choice H) direct inguinal hernias

do not Both types of inguinal hernias pass through the superficial inguinal ring

During a gastric resection in a patient with stomach cancer a surgeon wants to remove the lesser

omentum because of tumor extension into it Which of the following structures lie in the free edge of the

l~~g omentum and consequently must be dissected out in order to be preserved

A Common bile duct cystic duct and hepatic artery 6

B Cystic duct hepatic artery and hepatic vein

e Hepatic vein and cystic duct

Portal vein common bile duct and hepatic artery

E Portal vein hepatic artery and hepatic vein

The correct answer is D The free edge of the lesser omentum contains three important structures the

common bile duct the hepatic artery and the portal vein Nei ther the cystic duct (choices A B and C) nor the hepatic vein (choices B C and E) lies in the free

edge of the lesser omentum

A 55-year-old male patient with chronic liver disease has portal hypertension To relieve the pressure in the portal system a porto-caval shunt is performed Which of the following veins may by anastomosed to

accomplish this porto-caval shunt A Left renal vein-left testicular veingt

B Right renal vein-right suprarenal vein I shy

e Splenic vein -left renal vein J

D Superior mesenteric vein-inferior mesenteric vein E Superior mesenteric vein-splenic vein

The correct answer is C The splenic vein drains directly into the portal vein The left renal vein drains

directly into the inferior vena cava Anastomosis of these veins would allow blood from the portal vein to

drain retrograde though the splenic vein into the renal vein and then into the inferior vena cava The left

renal vein (choice A) drains directly into the inferior vena cava The left testicular vein drains directly into

the left renal vein Thus these veins are already in communication and neither vein is part of the portal venous system The right renal vein (choice B) drains directly into the inferior vena cava The right

suprarenal vein also drains directly into the inferior vena cava Thus neither vein is part of the portal

venous system The superior mesenteric vein (choice D) drains directly into the portal vein The inferior

mesenteric vein drains into the splenic vein which then drains into the portal vein Thus neither vein is

part of the caval venous system The superior mesenteric vein (choice E) drains directly into the portal

vein The splenic vein also drains directly into the portal vein Thus neither vein is part of the caval

venous system

A 12 year old boy has fever vomiting and para-umbilical pain After examining the patient the doctor

makes an initial diagnosis of appendicitis Appendicular pain which is initially referred to the umbilicus goes to the dorsal root ganglion of

a TI b TI2 c L1 d T7

(e I TIO

A 59-year-old male undergoes a neurological examination which reveals that when the abdominal wall is

stroked the muscles of the abdominal wall of the side of the body stimulated failed to contract Other

neurological tests appeared normal The likely region affected includes

a CI - C5 spinal segments b C6 - TI c T2-TI ~T8-T12

e Ll- L5

The surgery done to relive portal hypertension is done by connecting two veins Which of the following veins would be suitable for connection

a Inferior vena cava and portal vein b Superior vena cava and portal vein c Splenic vein and right renal vein d Splenic vein and left renal vein e Superior mesenteric vein and Inferior vena cava

A mother brings her 3-week-old infant to the pediatric clinic reporting a new scrotal bulge that she found -~-

while changing a diaper yesterday The infant is afebrile Physical examination reveals a palpable mass in

the scrotum while in the standing position resolution of the mass in the supine position and no

transillumination of the scrotal sac What is the most likely diagnOSiS

a Cryptorchidism b Direct inguinal hernia c Hydrocele d Indirect inguinal hernia ~ e varicocele

The Vagal trunks enter the abdomen by passing through which of the following openings in the

diaphragm

a Right crus b Esophageal hiatus ~ c Vena caval hiatus d Aortic hiatus e Left crus

2 The anterior boundary of the epiploic foramen of Winslow is bounded by

a) First part of duodenum b) Lesser curvature of stomach c) Liver d) Hepato-duodenalligament v ~

3 The ilio-inguinal nerve is derived from

a TI2 ry b LI c L2 d L3 e L23

15 Surgically the structure used to suspend the kidney to the diaphragm is

a) Renal fascia b) True capsule c) Perinephric fat d) Paranephric fat

6 If there is portal obstruction because of carcinoma affecting the pancreas which of these of the

following signs would be present

a Caput medusae b Esophageal varices c Rectal varices c

d Pulmonary edema

7 In a sliding hernia the gastro-esophageal junction lies

a) At its normal position b) Below the normal position c) Above the normal position V d) None of the above

8 Which of the following structures is retroperi toneal

A transverse colon B spleen IJ2f6 C ileum D descending colon v r 1pound1111111

9 The renal angle is fonned lgtetween the 12th rib and ______ muscle

a Psoas major -middotshyb Erector spinae c Quadratus Iumborum d Diaphragm

10 The anterior structure at the hilum of the kidney is

a) Renal vein ~

b) Renal artery I middot~ I

c) Ureter d) Accessory renal artery

11 Because of origin of the muscle from the lateral one third of the inguinal ligament it

could not fonn the anterior wall of the inguinal ligament

a) External oblique b) Internal oblique c) Transversus abdominis_ d) Rectus abdominis

12 A large tumor mass impinges on the splenic artery and its branches as the artery pass out from below

the greater curvature of the stomach Branches o(which of the following arteries would most likely to

effected by the pressure on the splenic artery

a Left gastric b Left gastro-epipJoic c Right gastric d Right gastro-epipoloic e Short gastric_

13 A new born baby has projectile vomiting after each feeding It is determined that there is obstruction

of the digestive tract as a result of annular pancreas Annular pancreas is as a result of an abnormality in which of the following process

a Rotation of the dorsal pancreatic bud around the first part of duodenum b Rotation of the dorsal pancreatic bud around the second part of duodenum c Rotation of the dorsal pancreatic bud around the third part of duodenum d Rotation of the ventral pancreatic bud around the first part of duodenum y Rotation of the ventral pancreatic bud around the second part of duodenum

14 As the liver bud enters the ventral mesogastrium the region of the mesogastrium stretching from the

liver to the anterior abdominal wall is called

a Lesser Omentum b Greater Omentum ~ Falcifrom ligament d Lacunar ligament e Ligamentum teres of liver

16 A patient has absence of his 12th rib In such a patient if the doctor makes an incision to approach his

kidney mistaking the 11 th rib for the 12t he would end up injuring

Which of the following arteries is a direct branch of the gastroduodenal artery The

A right gastric artery

B left gastric artery

C inferior pancreaticoduodenal artery D left gastroepiploic artery

i E)right gastroepiploic artery --

E x pI a nation The right gastric artery is typically a branch of the proper hepatic artery The left gastric artery is a direct

branch of the celiac trunk The right and left gastric arteries anastomose along the lesser curvature of the

stomach The inferior pancreaticoduodenal artery is a branch of the superior mesenteric artery it

anastomoses with the superior pancreaticoduodenal in the head of the pancreas The left gastroepiploic

artery is a branch of the splenic artery it anastomoses with the right gastroepiploic artery along the greater

curvature of the stomach The right gastroepiploic artery is a branch of the gastroduodenal artery The

other branch of the gastroduodenal artery is the superior pancreaticoduodenal artery

Which of the following pairs of veins join together to form the portal vein The

A superior mesenteric vein and inferior mesenteric vein

B inferior mesenteric vein and splenic vein

C superior mesenteric vein and splenic vein

Ip)splenic vein and left gastric vein E superior mesenteric vein and left gastric vein

Explanation

The portal vein is formed behind the neck of the pancreas by the union of the superior mesenteric vein

and the splenic vein The inferior mesenteric vein drains into the splenic vein The left gastric vein drains

directly into the portal vein After the portal vein forms it enters the hepatoduodenalligament of the

lesser omentum to reach the liver The portal vein is the most posterior structure in the hepatoduodenal

ligament

At which of the following vertebral levels does the duodenum pass anterior to the aorta - _- shy

All ~

B L2 7~

CL3 I

~DL4

E L5

Explanation

The duodenum begins at the pyloric sphincter at the level of Ll The second (or descending) portion of

the duodenum is to the right of the aorta and extends inferiorly from the level of Ll to the level of L3 The third part of the duodenum crosses the aorta from the right side to the left side at the level of L3 The

fourth (ascending) portion of the duodenum extends from the level of LJ to the level of L2 The

duodenum ends at the duodenojejunal flexure The superior mesenteric artery passes anterior to the

duodenum as the duodenum passes anterior to the aorta The duodenum can be constricted at this level

In which of the following locations will perforation of the digestive tract result in the spilling of luminal

contents into the - lesser peritoneal sac

A Anterior wall of the second portion of the duodenum B Posterior wall of the second portion of the duodenum

C Anterior wall of the stomach

~Posterior wall of the stomach E Posterior wall of the transverse colon

Explanation

The posterior wall of the stomach is related to the lesser peritoneal sac The anterior wall of the stomach is related to the greater peritoneal sac The anterior wall of the second portion of the duodenum is related to the greater peritoneal sac The posterior wall of the second portion of the duodenum is related to the retroperitoneal space The posterior wall of the transverse colon is related to the greater peritoneal sac

The ureter lies against the anterior surface of which of the following muscles shyA Crus oftne diaphragm B Quadratus lumborum

0 Psoas major D Transversus abdominis

E Iliacus

Explanation The ureter exits the renal pelvis at about the level of vertebra L2 As it descends along the posterior abdominal wall it lies on the anterior surface of the psoas major The psoas major muscle arises from the bodies of the lower lumbar vertebrae The psoas major muscle is joined by the iliacus to fonn the

iliopsoas muscle The iliopsoas muscle then attaches to the lesser trochanter of the femur and is the major

flexor of the hip

As the right ureter passes the pelvic brim it lies against the anterior surface of which of the following

blood vessels

A Gonadal artery B Inferiorvena cava C Internal iliac artery

rJ- External Iliac artery

E Inferior mesenteric artery

Explanation

The ureter lies in the extraperitoneal space in the posterior abdominal wall Alter leaving the kidney it

passes inferiorly on the anterior surface of the psoas major muscle At the pelvic brim the ureter passes

into the pelvis At this point the common iliac artery is dividing into the external and iliac arteries The

ureter lies on the anterior surface of the external iliac artery immediately distal to the bifurcation This is a useful landmark for a surgeon to locate the ureter

When extravasated urine passes from the superficial perineal space into the anterior abdominal wall it is

found immediately deep to which of the following layers of the anterior abdominal wall

-ltScarpas fascia

B External oblique muscle

C Internal oblique muscle D Transversus abdominis muscle

E Transversalis fascia

Explanation

The superficial perineal space is bound by Colles fascia the fibrous portion of the superficial fascia This

layer of fascia is continuous with Scarpas fascia the fibrous portion of the superficial fascia of the anterior abdominal wall Therefore urine that is deep to Colles fascia will remain deep to Scarpa s fascia The urine will spread in the plane between Scarpas fascia and the external oblique layer

When a horseshoe kidney develops the ascent of the kidney is restricted by the A internal iliac artery B external Iliac artery

C common iliac artery

inferior mesenteric artery

E superior mesenteric artery

Explanation

A horseshoe kidney develops when the inferior poles of the to kidneys fuse together as they ascend into

the abdomen from the pelvis The first anterior midline vessel that is encountered by the horseshoe kidney

is the inferior mesenteric artery This artery prevents the kidney from continuing its ascent

The left testicular vein drains into which of the following veins

A Left internal iliac vein B Left common iliac vein

bflnferior vena cava D Left renal vein I

E Left internal pudendal vein

Explanation

The left testicular vein drains into the left renal vein The right testicular ~i~[~nsltjectlY into the

inferior vena cava This difference in venous drainage is believed to explain the greater incidence of

varicocele on the left side than on the right The venous drainage from the penis is to the internal vein

which then drains into the internal Iliac vein

The spinal nerve that provides cutaneous branches to the skin around the umbilicus is

A TS B TW-shy

C TI2

DL2 EtA

Explanation

The tenth intercostal nerve is the anterior ramus of the TIO spinal nerve After passing through the tenth

intercostal space the nerve continues forward in the anterolateral abdominal wall in the plane between

the internal oblique muscle and the transversus abdominis muscle In the abdominal wall the nerve innervates to the abdominal wall muscles as well as the skin and the parietal peritoneum The umbilicus is

a useful landmark for the region of distribution of the tenth thoracic nerve

The ligament of the vertebral column that resists its extension is the Aligamentum flavum

B supraspinous ligament

C posterior longitudinal ligament

D anterior longitudinal ligament

E interspinous ligament

Explanation

The ligaments of the vertebral column that resist flexion of the column include the supraspinous ligament

interspinous ligament ligamentum fiavum and posterior longitudinal ligament The ligament that resists

extension is the anterior longitudinal ligament This longitudinal ligament is very broad and strong It

covers the anterior and anterolateral surfaces of the vertebral bodies and the intervertebral disks In

addition to resisting extension the anterior longitudinal ligament provides reinforcement to the anterior

and anterolateral surfaces of the intervertebral disk The posterior longitudinal ligament is relatively

narrow and covers the posterior surface of the vertebral bodies and the intervertebral disks This ligament

reinforces the posterior surface of the disk The posterolateral surface of the disk is not reinforced and it

is through this region that herniation of the nucleus pulposus usually occurs

A patient presents with epigastric and right upper quadrant pain The pain is most intense 2-4 hours after

eating and is reduced by the ingestion of antacids The patient states that he has passed black tarry stools

(melena) within the last week Fiberoptic endoscopy reveals a yellowish crater surrounded by a rim of

erythema that is 3 cm distal to the pylorus Accordingly an ulcer has been identified in the patients

A fundus

B antrum

C duodenum

D jejunum

E ileum

A number of physiologic genetic and other factors increase the risk of gastric (and duodenal) peptic

ulcers The evidence that H pylori plays a principle role is compelling Smoking and caffeine are known to adversely affect the morbidity mortality and healing rates of peptic ulcers In general first-degree

relatives of peptic ulcer patients as well as males have a threefold to fourfold increased risk of developing this disorder Paradoxically in gastric ulcer disease acid secretion is not elevated It is possible that

excess secreted hydrogen ion is reabsorbed across the injured gastric mucosa In general a defect in gastric mucosal defense is the more important local physiologic

A patient presents with symptoms of duodenal obstruction caused by an annular pancreas Annular pancreas is caused by

A rotation of the dorsal pancreatic bud into the ventral mesentery B rotation of the ventral pancreatic bud into the dorsal mesentery

fJ failure of the major and minor pancreatic ducts to fuse ~ ~ cleavage of the ventral pancreatic bud and rotation of the two portions in opposite directions around -the duodenum E formation of one pancreatic bud instead of two

Explanation Normally the ventral pancreatic bud rotates around the gut tube to reach the dorsal pancreatic bud The two buds fuse to form a single pancreas and the distal portions of the two ducts fuse The ventral pancreatic bud forms the inferior portion of the head of the pancreas the uncinate process and the major pancreatic duct (of Wirsung) The dorsal pancreatic bud forms the superior part of the head the neck body and tail and the minor pancreatic duct (of Santorini) Annular pancreas is the result of the ventral pancreatic bud dividing into two portions before it rotates into the dorsal mesentery Each portion rotates in opposite directions to get to the dorsal mesentery thus encircling the duodenum The presence of annular pancreas can constrict the duodenum thus obstructing its lumen

In n _ phranlc----

Gon ~l ----_1 Lum bltano

~~--- CornmQ1t bull ac

+-~4--- lnlllirnaJ ilic

xtem iliac

OBJECTIVE - Identify the blood supply to each of the structures listed in the table on the previous page

Ill give you a head start

FOREGUT - Supplied bV Celiac Tru nk (T12)

Proper hepatic

GastiooUod 13Jafter

1nferlor pancreaticoduodenal artery

Common epatlc

Lett gas ric iiirtery

Spfen artery

shy Gastroepiphgtic artery

~ Superior mesenteric 8rtfry

~

1 Esophagus is a derivative of the foregut so its blood supply originates from the celiac trunk

(T12) The predominant blood supply to abdominal portion of the esophagus is the Esophageal

A (Branch of L Gastric) The venous drainage of the esophagus is particularly important because

it is 1 of 3 clinically relevant sites of Portal Caval anastamoses The Portal Esophageal Vein

meets the Caval Azygos System Persistent bleeding manifests as Esophageal Varices - a fata I

condition

2 The Stomach is also a derivative of the foregut has EXTENSIVE blood supply and is very high

yield on anatomy exams The lesser curvature is supplied superiorly by the L Gastric A (1 of 3

major branches ofthe Celiac trunk) and inferiorly by the R Gastric A ( a branch ofthe proper

Hepatic A) The greater curvature is supplied superiorly by the L Gastroepiploic A (a major

branch of the splenic A) and inferiorly by the R Gastroepiploic A

The Short Gastric arteries (branches of Splenic Artery) supply the fundus of the stomach and

are referred to as EIID ARTERIES because they have no collateral blood supply Therefore if the

splenic artery were occluded (ex - increased pressure in the ommental bursa) - there would be

ischemia to the fundus of the stomach Venous drainage of the stomach is extensive via various

veins lead ing to the portal system Posterior to the stomach the IMV joins the splenic V which

joins the SMV to form the PORTAL VEIN ADAMS

3 Duodenum blood supply has high clinical relevance because it is the junction of the foregut and

midgut and therefore is the site of anastamoses between branches ofthe Celiac Trunk (main

foregut artery) and the Superior Messenteric Artery (main midgut artery) The Proper hepatic

artery gives off the gastroduodenal artery which travels behind the 1st part of the duodenum

This point has high clin ical relevance because duodenal ulcers are very common and a posterior

rupture of the 1st part of the duodenum could rupture the gastroduodenal artery causing

traumatic abdominal bleeding The Gastroduodenal artery first gives off the R Gastroepiploic A

(mentioned above) and proceeds as the Superior pancreatico duodenal artery (supplies the

pancreas and duodenum) which anastamoses with the inferior pancreatico duodenal A (branch

of the SMA) This is the junction of foregut and midgut and occurs near the opening of the

bil iary system into the duodenum (ampula of vater) Portal venous drainage here is responsible

for delivering nutrients from digestion to the liver for metabolism Appreciate that the Superior

mesenteric artery (artery of the midgut) branches from the aorta at Ll travels posterior to the

pancreas than moves anteriorly (at the jxn of the pancreatic headbody) and comes over the

3rd4th part of the duodenum Tumor of the head of the pancreas can compress the SMA

4 Jiver blood supply is via the common hepatic artery (major branch of the cel iac trunk) The

common hepatiC becomes the proper hepatic gives off the R gastric A and the Gastroduodenal

A and then joins the common bile duct and the portal vein in the portal triad Clinical- if a

patient were bleeding from the hepatic A a surgeon can stick his fingers in the epiplOic foramen

and squeeze the free edge of the hepatoduodenalligament in order to stop bleeding to the

area Please note that the hepatic a branches into Rand L hepatic A The Right hepatic artery

gives off the cystic artery which supplies the gallbladder Afferent venous supply is via the

Portal vein which is bringing nutrient rich blood to the liver After metabolism takes place

venous blood leaves the liver through the hepatic veins into the IVC PLEASE UNDERSTAND THE

RELATIONSHIP OF THESE STRUCTURES - ADAMSNETIERSNH Etc

5 Pancreas - Head is supplied via the superior and inferior pancreaticoduodenal arteries

(mentioned above) The tail (situated towards the hilum of the spleen) is supplied via the

pancreatic branches of the splenic artery (END ARTERIES) This blood supply is very important

because the endocrine Alpha and Beta Cells from the pancreatic islets of lagerhans are located

towards the tail This is where Insulin and Glucagon is released to the blood

Now complete this for mid and hindgut structures Make sure to note clinically relevant arterial

anastomoses as well as portal caval anastomoses FYI Appendix blood supply SMA + IMA

anastamoses marginal artery Portalcaval rectal veins fhemmorhoids) and periumbilical caput

medusa are high yield THE BUTT THE GUT and THE CAPUT

Abdominal Development

Liver

Ij1f

II wall b

oh liN ~ VltJrti n be- bull

Pancreas

Secondary Retroperitonealization e I~tl r 1 a v-mtrai m ellter

Rotations of the Gut I i Ij (lIl1UtIJ f~ l r tilt

()l td 10 me l-ft and he v

--~--- -~ -~-~

i

I AolaijonjoI~guf I

STOMACH BED (IDENTIFY IN ADAMS)- the structures posterior to the ommental bursa which

support the stomach in the supine position

Abdomnal JQrUI

Splnic vein

OmQ-oul tv~ ) O(s(Jroa)

Lojt(r o m nturrt (hpJtodu o d~n31 Hid

Gadrl)SplerH (g3stroll~nal) IIgam~nt

hiad h~~atogrtricent IIQdmiddotcrt~)

Lt Dome of Diaphragm (why left Look this up in Adams)

Spleen (What is the blood supply)

Left Kidney (What is the blood supply - AND how is it different from the R kidney)

Suprarenal Gland (What is the Arterial AND Venous Blood supply - how are they different)

Pancreas (How does supply differ from Head to Tail What is the SMA Relationship)

Transverse Mesocolon

liver - ADAMSWET - Make sure you look at the liver in wet lab

Left triangular nl1am~nt

ComoaDj ligamnt

Erophg~1 impre$ioo

Hepatio veins

In1erior -ifena middotr3)Ia

Fibrous appendix o-t

live

impr~j on

Heprorendl p~rtion of Q)(Qllary ligament

Righllri~n9ul r 1I~met

(Common) bile quol

Gr)mmCtr~ hepatic dlJct

Ccentic duct

Duodenal impression

GaJdate p-fr)~S

Hepatic artgtrl prop-f iiiiila - Faloiform ligament

_ - shy Round ligamen liver

~--F-- CoJio imprgt-ssi-on

Prta heptis

Identify the lobes impressions and embryonic remnants associated with the liver

Caudate Lobe Quadrate Lobe Right Lobe Left Lobe Round ligament Falciform Ligament

Ligamentum Venosum (what is its fxn in embryonic life) Hepatic Veins (NOT PART OF THE

PORTAL TRIAD) IVC PORTAL TRIAD - Contents relationship cross section etc Know the

Galbladder relationship to the lobes of the liver

Biliary Duct System - Make sure you understand the sequence of these structures - BE ABLE TO

DRAW A FLOW CHART

TPVd i

t

I t

1 __ Cm-(r

patk GlJet

I

J

Clinical = JAUNDICE is caused by anything that prevents delivery of bile to intestine Tumor of the

head of the pancreas Stones etc Patient will have pale stools and yellowish colored mucus

membranes

Clinical- Any scenario that tells you the patient has BILLOUS VOMIT means that the obstruction to

the flow of digestive contents is after the Ampulla of Vater (Site of Entry of Billiary system to the

duodenum) - ie Duodenal Atresia

Spleen -located posterior to the mid axillary line between ribs 9 and 11 Make sure you know that

the 10th rib is the main axis of the spleen and this organ is susceptible to injury (stab wound errant

thoracoce ntesis etc)

The spleen is derived from mesodermal cells - NOT THE GUT TUBE

The spleen rests on the left colic flexure associates with the tail of the pancreas Know the

structures entering the Hilum of the spleen

Sh rt O~-t~ic 1 0(0 10 rtiltSPIric Iloa nt

(cut)

Peritoneum - similar concept to Pleura - think of a fist in a balloon

Visceral Peritoneum - Layer of balloon touching your fist

Parietal Peritoneum - Layer of balloon not touching your fist

Your fist represents the organ your wrist is the hilum and your arm contains the blood supply

entering the organ

Appreciate that there will never be organs in the peritoneal cavity - rather these organs invaginate

the cavity Kaplan videos

RULES OF NOMENCLATUREshy

1 Organ completely surrounded by peritoneum - peritoneal organ

2 Organ partially surrounded by peritoneum- Retroperitoneal

3 Peritoneum surrounding peritoneal organ is VISCERAL peritoneum

4 Peritoneum surrounding retroperitoneal organ is PARIETAL peritoneum

5 Peritoneum connecting visceral to parietal is called messentary 2 messentaries in the

gut Dorsal (to the gut tube) and ventral (to the gut tube) messentary

Aorta is in Retro peritoneal position - but blood must reach peritoneal position - vessels travel through

messentary All peritoneal organs will have blood supply reaching through messentary

-Mesentery is a 2 layer peritoneum with a neurovascular communication between body wall and organ

- Ligament connects one organ with another or to the abdominal wall (Ommentum = ligament)

lesser Ommentum (attach lesser curvature of stomach and duodenum to liver) =Hepatoduodenal

Ligament and Hepatogastric Ligament

Has a Superior and Inferior Recess (Accumulation of Fluid in Ascites)

Communicates with the greater sac through the epiplic foramen (what structures pass through

this foramen)

Boundaries - you must be able to visualize this

o Anterior - stomach

o Posterior - parietal peritoneum pancreas

o Superior - superior recess (bw diaphragm and coronary ligament)

o Inferior -Inferior recess (bw layers or greater momentum

Greater Ommentum (attach greater curvature of stomach) Gastrophrenic ligament Gastrosplenic

ligament gastrocolic ligament

The greater omentum is the largest peritoneal fold It consists of a double sheet of peritoneum folded on itself so that it is made up of four layers The two layers which descend from the greater curvature of the stomach and commencement of the duodenum pass in front of the small intestines sometimes as low down as the pelvis they then turn upon themselves and ascend again as far as the transverse colon where they separate and enclose that part of the intestine

ABDOMINAL PAIN

Parietal Peritoneum - supplied by same vasculature lymphatics and nerves supplying body wall it

lines and diaphragm Sensitive to pain pressure heat cold well localized

Visceral Peritoneum - supplied by same vasculature lymphatics and somatic nerve of organ it covers

Insensitive to touch heat cold and laceration - referred to dermatome of spinal ganglia providing

sensory fibers Where does appendicitis refer to

Foregut pain - epigastric area (ie - cholycystitis)

Midgut pain - periumbilical area (ie - appendicitis)

Hindgut Pain - suprapubic area (ie - diverticulitis)

Extra ImagesConcepts

ll~_____-

FalifCtrm ligament oind r~ud ligamet f Ilver

Blood from splenio gastriC and inferiof rne$e-rteri v~ins

Ca-I tributaries

Lett gastrio Ifein

Posterior superior pan~reatioodul)denal vaihS

Lott gamo-om~nlal (9aropip lomiddotic) -in

Poq_~ tjol imerl-9-r panCJertlcorllJod-nal veiopound --amp----I- - ~J Right grtr~-omntal

Anwrior interi (gartroepiploic) Jjn

pan euaii cod vl)denal veins middot Inf~Ji (t r mesentric vein

Miqdle (olic vein

Right cl)licvein Sigmoid and rectosigml)id (ei ns

IhH)Collc(~io

--- Mi~dl laquooLJl gtjrltgt

PoM ca vl1 illasto)moses -----shyampoptoageal 2 Paraumbilie-lt11 Inferi or Fectal vei ns

3 Recial 4 REuoperHonea1

Know how the Portal vein is formed I 4 sites of portal caval anastamoses and 1 clinical shunt

Col li t ltt-~ otTl~tI ~nj pc~ 1lt1 turJoG

Ltf 14i1 tImiddot~ artoftl9 on tj phtAt$

L-oftqf 4t t~r 1=laquoIran d 1 bull shy~p l ci rj o fOOOts

Nerves follow the arteries - appreciate the splanchnic nervous system I

Uet~ric branch of left ~nal art

Ureterie branch of righi renal artelY

Left Zld lumbar in and co mlTlunication to as)erdin9 lumbar l(~in Hi ~ht tEZ1~~t~ t3r j t itn ~ nJ l1t- rlnd lfe i r1

Inferior me5nteri~ artery

Notice that the right testicular vein drains directly into the IVC and the right testicular artery drains

directly into the aorta However the left testicular vein drains into the L renal vein at a right angleshy

reason left testicle is lower and more susceptible to varicocele (bag of worms)

Also notice that the left renal vein has a longer course because the IVC is on the right side whereas

the right renal artery has a longer course because the aorta is on the left side

Appreciate the anterior to posterior relationship of structures in the hilum of the kidney - VAP - Vein

Artery Renal Pelvis (Ureter)

11____ __ L_ L_ n VJ __ _ _ t_L I I_ _ L __ L_ I -pound1 bull LI_~-I ____

Posterior View of Head of Pancreas in ( of Duodenum

Celiao hunk

Co mmon ~L~jJth art~ry

GastNduQdonal artrf (partilly in phantn)

P1)Sterior $Up~Jior panCflaticuduodfmal art~r~t

(Co mm on) bile duct

middot~1t~~t-1l---~-~- Right gshomiddotomental (gastoe plp lolc) 3rte (phantomost)

Grener paocre atic art-ry

1n1~rjor pancr-iatlc artery

Jtrifll supejo r pal)oreailcento)dJodenal artr1 (phantom)

Anastomotlo branch

POostetlor bJanch of jo f~ri of pan-reatir(lduodensl drttnj

Anterio r branch of i flferior palcreati~)duodenal art~(phan1om)

Notice the extensive blood supply to the pancreas and duodenum via the branches of the celiac trunk

Notice collateral supply from SMA branches - makes sense bc this is the jxn of foregutmidgut

Identify the vessels in this arteriogram

Hiltid i)f N~ck oi B)dvof Tail 01 pa nereas pan cent~as P-nmiddot-reas panCtCas

I nferie v~na cava

jHept1iic p(lrlai v~in

Port1 tnd H~pti lt a ftH prol

Comm on) bll duct

Ouodtnum

~ft colic (sio)Atta~ hmtrlt jt~xJr-ofha~elSe

muo(IIQn

Right ~lIc (h~j)tic)

il~gtture

In1triol m~oten lIein (rttr op~ritoMdO

SlJp efl or mes~n~fiC amrV and lipln

KNOW YOUR NEIGHBORHOOD

Questions

vVhiJh structure supplied by a bnmdlof the cclia( artery is not derivcd from foregut LemCJUCrITI

(A) Head of the pancte-a5

CD) Pyloric duolenum

Cystkduct

( Liver hepatocyt~~

~F) Body of the spleen

An infant presents with an omrhaJucele at birth -hi oJ the [oHm illg applies to his cM1-dition

(A) It is 31so seen ill p4titnts with aganghonic megacolon

(11) ft reuirs from a fal1ure of resorption of theviteUine d let

(C) It results from herniation at the-site of regression of the right umbilk vein

DJ It is caustd by faihtrc of recanalization of the midgut part of the duodenum

~ It ill camioo by a failuIt vf the midgul to return to the abGQminal uity after herniashytion in-n the urnbilk s l stalk

Ot er than the spleen occlusion Cif the spit-Ilk artery at its odgin wm most likely affect die blood supply to jllch st cnud

(A) Jejunum

(B) Body of th pal1~lltas

(C) LeSStT Cllmiddotlaturc of tl )toma-ch

(D Duodenum dista to the entrance of the Ornmou bile duct

E Fundus of the stomach

A 38-yeu-old batL~er with a history of heartburn suddenly experiences excluciating pain in the (plgastric region of th~ abdomeu SurgCry is perf~rme immediard y upon admisshysion to the 1IlcrgCJliy tuomh~re i~ evidence uf a ruptured ulcer in the posterior waU of the stomach Vhere will a surgeon first fi nd the stomach contenlSf

A) Greater p4ritoneal sac

rB) Cul~de-s~c of Douglas (--

C Omental bursa ~

--D) Paracolic gutter

rEj Between -he panttal perimltum and the posterior body wal1

At birth an infant presents with a st()ma~ rb~tbas~njJled jfltotb~diaplfagru 1A1ltre is the defect thatresulied iiitJle heini~t()n shy~tsophagealbiatus

7 - rH-- Hiatus for the inferior vena cava

( Pleuroperitoneal membrane -(0) Septum transvcrsum

(E) Right Crlt~

An infant born with DOVv7l syndrome presents with bili()u~ vomiting Ahat congenital defect does the infant have

(A) Pyloric stenosis

(B) Meckel diverticulum C) Ornphaloce1e

(D) Gastroschisis

( ~ ) Duodenal atresia y A patient with cirrhosis of the liver presents with ~ bacalvaricestnlreased retrograde pressure in which veins caused the varices

(A) Paraumuilical

(B) Splenic

(ct AzygltJus

(15))G~trk ( (-F) Superior mesemeric

A htaltby 3-year~old male patient experiences a hernial sa protruding from the anterior abdominal wall about halfway between me anterior superior ilia spine and the pubk tuberde Pulsations of al1 artery are palpated medial to the protrusion site through the abdominal walL Which layer of the anterior abdominal wall will first be traversed by the

1hctma

fA) Rectus sheath (B) External oblique aponeurosis

(C) Inguinal ligament

lD) Transversalis fusda

(E) Cremasteric fa~cia

After 5urgi(aj ffpair of a hernia the patient tXperienccs mtmlgtness in the skin on the anteshyrior aspect of the S(Totum_ Vhaf nerve may have been lesioned during thehemiorrhaphy

(A) Femoral

(B) Obturator

(C) Ilioinguinal

(D) lliohypogastrk

(E) Pudendal

A 23~year-LJld female secretary il1 good health ~-uddcn1) doubles over with pain in the a ea of the 1JmbRicu$ Sbe feels vartn and ltneasy and has no appetite That night the pain seems to have mQved to the tower right abdominal regjol1 and she calls her family doctor who then arranges for an ambulance to pk-k her up and take her to the hospitaL Wh ell ntn~ perceived in the area of the urnbilirus most Hkely carried lhe pairfu I sensations into the eNS

tA) Vagus nerves I~

V B)

) Lessersplanchnk nerves

tC) Pudendal nerves

(D) lIiohpogastrk nerves

(E) Greater splam ic l erves

A CT reveals carcinoma in the bOod of the ancreas Vhich blood vessel trut ourses ----~- - -bull ------ --shy

immediately poftterior to the body ofthe pancreas is the m~t likely to be oompressed

(A) Splenk artery

(B) Abdominal aorta (C) Portal vein

(1) Splenic vein

(E) Renal vein

A patient has a penrln1l1ng uker of the posterior wall ot the br~l part ot the (lUooenmn llkh blood vessel is subject to erosion

(A) Common hepatic artery

(B) Gastroouodenal artery

(C) Proper hevatic artery

(D) Celiac artery

(E) Anterior inferior 11amrelltlcoduodcnal attery

Your patient has been diagnosed -ith a carcinoma locallted to the head and l~e(k of the pancreas Another clinical sign would be

A esophageal varices

(8) hemorrhoids

C) a caput medusa

(D) increased pra Teuro n th~ hepatic veins

(E) enlarged right supra lavkular lymph nodes

Wltkh of the foUowing structures develops in the ventral mesentery

(A) Spleen

(B) Jeiunum (C) Head of1ht pancreas (D) Transverse colon (E) Stomach

ti l Uw ~ littwin~ f( S-t lil oai Imdge ~ hi(h or tbt la~)d J truetur tgt liJ llntn nl) he hl p UC iJd [IIi ell

c o

A) drains Ie tht infCrior a La aI

R t middot~nfl0 ~ill to th~ lunlgtn of h i dtlndCrlllfH

(e) m t bull JiJattd on tl l J n T ~H

D ) sup Lc O VSlt I Hlid bhtu l 1 li - -I un oid

( ) U~tpli(t tr j middottUh~ 1 v(( b~nt rfK n1ilc~Zm

ANSWERS AND EXPLANATIONS

Answer E The spleen is t hlttnopodicand lymph organ demlted from mesoderm

Answ~ R Al1 tlmphalocele is caused by it failure of the nlidgut to return to the ahdomir nat cavity after herniation into the umbiliau Stalk Choices Aand D maybe seen in infants with Down syndrome choice D ~s the specific CBuse ofduudcnal JtiCSitt Choice C is (ile cause of gclstrosbisis and Choice B nsults iu a Meurolktldivertku1-tlB

Answer B The fundus ofthe stomach is suppHed by soort gastric brunches of the splenic altery The splenic artery supplies the body and tail of the pancreas part of the greater curvature of the sttmla(h and the spleen Te jejunum part of the head of the pancreas and tht~ duodenum distal to the entrance of the commOll bile duct are supplied by the superior mesenterk artery clll~l ~be less r ctlt1ature cmd the pylQric antrum are supplied by the right and lei gastric art(ries

AnSWftt C Tbeomental bursa or lesser ~ritoneaj sac lies direcdy posterior to the proxshyimal part of the duodeTtlm and the stomach and would be the first site where stomach contents ~Ott1d be fpoundluncL

Answer C A defect in a llleuropcritoneal membrane (uswlly the left) is the typical site of i1 cc-ngenitlI diilphragluatic hemia llere the membr4ne fails to dose ()pound( of the perishycCirdiopcritulleal canals

Answer E DuoJenal atresia and aganglionic megacoion are congwitaI defects S~Il in patients with Dowmiddotnsyndrome

Answer D RulaTgemt~llt of and retrograde flow in g~lstrk vel_ns in particlJl~r the kft gas~ tricveins dilates the capillary bed in rhe wall of the esophagus in (ases of porta yper~

tension Blood flow would increase in and dilampte tribntarkgts of the (lZygOUS vein on the other side of the capiUary bed but flow in this vein is in the typical direction t()ward the superior vena cava Paraumbiii(ltU vein eilgorgement contributes to a caput medusH Splenic ~nlargement might prc~nt with 5plcnonlegaly and balt-kflow in to tlu superior m~~ntclic vein occurs but is asymptomatic

Answer D The patient hagt an indirect inguinal hernia whi~h emerges from the antt-rior abdominal wall through the deep inguinltilling Theeep ring is a fault in the transv~rshysaUs fascia this I~yer wiIJ be penetrated first by the hernia

An~Wer C The ilioinguinal nenc which provides sens~llion to the lnedlal thigh ltmclanteshytior SClotunl pass~lt th rough the 5uperfh_ial inguinal ring ind $subject to inj i1T) becaus-e

it is in the operatitm Held of the erniorrhapny

Auswer B The leMHr splanchnic nerves are sympathdic nerVlts that carry viscera l sensashytlltgtrogt ftom illtllt1m~d ()J stietched gust (itinteitinal ~tructures (in this case the pprndix) into tnt eNS Lesser splanchnic ntTYcsarisc from thmiddot T9--T12 spinal cord segments lt1nd provide sympathetic innenation tD rnidgut siruc1ures whiCh include CLe app~JldD Viscera] Pain arising from affecLed Inidgut ampt 1C1ure is referred over the same dl- matorne~ of spinal segrnertts v-hich provide the sympathetic Innervation n this G1SC of appendicitis the invohen~n t of the ltire) of t e unlhHku indud s the T 10 dermatome

Answer B Of the five choices onty the dscending olon is retroperiton~al aldwould be a lik ~ ( choice to be seen immediately a(~jilcent to t11e posterior abdominal middotn~L

Amwen D The SpltftlC ~-ein ourses posterior to the body of the panneas m its way tt drain into the superior mCSfttltlri( vein

Answcr B TILt glstrodllolticnal artery 1 direct hIamh of the comrootl hepatic artery courses immediately pt))iwri() to the duodenum and is slbject to erosion

Answer B Carcinoma of th pan middott3S in the 1tilt1 may compreampgt the portltil vein at irs orishygill The poTtai vcin is fomled when the splenic vein jQiaswith tfie superior meStllt eric vein The inferiot mesenteric vein joins the ~plenjc vein just priOT to tlli~ point at which the splenic joins the superior Jlleit1ltcri( vein Increescd venous presslu in the inferior mesenteric vein is a cause of emo hoid~

Answer C The- velltral pancreas wilich forms most of the head of the p ~ncr as develops in the ventral mes(ntery as antutgrowth of the hepatic diverticulum Th~ hepatic divershyticulull induding the biIJary appa~atus develops in tbe ventral mesentery of the foregut

Answer~ A The superior mesenteric ~in joins with the spienkvein to form the hepatic portal vciu

Answer D The structure at gttlK is the proper hepatic artery~ whkh suppUesoxygenated b middotood to the liver

MAKE SURE YOU KNOW the diff bw Rectus Sheath above and below the arcuate line

ABOVE

Aponeurosis of xiiltmal obllque musclo

Extemll f)biquw musde

Reotln ilbdomlnls musole S~in

Internal 9bliquQ mY~QI

AponeUfOsi$ of hJH$V~~S Lir9a a lb lbdolTlin~ musolo Tri OJV6 rUi

atldomlnis mUS(loe

Sub cutanlilous tiue (tatty ye r)

BElOW

A POrl lJfosis 01 etemal oblique muscle

Aponeul~)sis 01 Internal oblique mU$cl~

Anteriol lay~ of r~ltdus st~ath EXttom1 oblique rnu$cll

Rectus Jbdominis muscle Intoernal Aponeurc-sis of tra~fersU$ oblique muscle-

at-domlnis muscentl ~ Skio

Tra nsvitSus abdomioLs ml)ZClt

TralSVersaHs fascia Medial umQil iegtt1 1i9Jment -and folj

Uldchus Peritoneum (ir median Umbilj~al Suboutane ous

Extraprftone 11ascia

Ymbilimiddot~1 fold)

preu9poundiea1 fascia

tissue (fatty 4nd m~mbr3n(iUS layers)

o Above the arcuate line (A horizontal line 13 of the distance bw the umbilicus and the

pubic symphysis) -10 Aponeurosis divides into an AntPost Laminae

o The Ant Laminae joins EO and Post Laminae joins Trans Abdominis = Ant and Post

RECTUS SHEATH respectively

o BElOW the arcuate line - all 3 aponeurosis join ANTERIOR to rectus muscle to meet its

counterpart in the midline (linea Alba)

o Take away Msg - The abdomen is devoid of a posterior rectus sheath below the

arcuate line and is therefore more vulnerable to herniasinjuries

Question - A physician makes a deep incision in the patients midline immediately superior to

the pubic symphysis which of the following layers is his knife least likely to pass

Rectus Abdominis External Oblique Ant Rectus Sheath Posterior Rectus Sheath All of the

Above

Answer - All of the above None of the other answer choices are midline structures -LINEA

ALBA

Linea Alba has very poor blood supply - doesnt heal well after surgery Therefore this is a

common site for incisional hernias

a Spleen b Transverse colon c Descending colon d Stomach e Pleura

17 Meckels diverticulum is normally found 2 feet proximal from the

a Pyloric sphincter b Lower esophageal sphincter c Ileo-cecal valve d Middle valve of Huston e Anal valve

18 Ulcer in the posterior wall of the first part of the duodenum would erode ___ artery and would cause bleeding

a Left gastric b Right gastric c Hepatic artery proper d Gastroduodenal artery e Middle colic artery

19 An inflamed appendix is identified by a surgeon on the operation table by noting

a The appendicies epiploicae b The convergence of tenia c The artery of Drummond d The mesocolon e The mesosalphinx

20 The nerve which emerges through the psoas major is

a Femoral b Ilio-inguinal c Ilio-hypogastric d Pudendal e Subcostal

21 The right gonadal vein drains into the

a Azygos b Hemiazygos c Inferior Vena Cava d Right renal vein e Left renal vein

22 The hepatocytes in the liver is derived from

a Ectoderm b Endoderm c Mesoderm

d Neural ectoderm

23 Abscess in the lumbar vertebrae due to tuberculosis would spread to the adjacent muscle which is

a Psoas Major b Iliacus c Quadratus lumborum d Tranversus Abdominis

24 The anterior wall of the inguinal canal is formed by

a External oblique and transverses abdominis b External oblique and fascia transversalis c Internal oblique and external oblique d Internal oblique and transverses abdominis e Fascia transversalis and peritoneum

Meckels diverticulum is a result of which of the following developmental abnormalities shy

A Failure of the vitelline duct to close

B Failure of the herniated intestinal loop to retract into the abdomen

C Failure of the urachus to close

D Failure of the midgut to rotate

E Failure of the hepatic duct to close

Explanation

Meckels diverticulum is a result of the persistence of the proximal part of the vitelline duct This

diverticulum is usually found about 2 feet proximal to the ileocecal junction and is usually about 2 inches

long It is present in about 2 of the popUlation It may be the site of ectopic pancreatic tissue or gastric

mucosa and may develop inflammatory processes and ulcerations Acute Meckels diverticulitis

simulates appendicitis

Which of the following veins carries blood from the esophagus to the portal vein The

A right gastric vein

B left gastric vein c splenic vein D azygos vein

E left gastroepiploic vein

Explanation

The left gastric vein a direct branch of the portal vein drains blood from the lesser curvature of the

stomach and the inferior portion of the esophagus Because branches of the portal vein do not have

valves blood can flow in a retrograde path when there is an obstruction to flow through the portal system or liveL Rlooci Cln then flow from the nortl] vein thr()1Ph the left PRstric vein to the esonhlPlIS lno

through venous communications within the submucosa of the esophagus to esophageal veins that drain

into the azygos vein The increase in blood flow through the esophageal submucosal veins results in esophageal varices

On the posterior wall of the abdomen the celiac ganglion A contains cell bodies of postganglionic parasympathetic neurons B is synapsed upon by neurons in the posterior vagal trunk C is synapsed upon by neurons in the greater splanchnic nerve D contains sensory cell bodies of lumbar spinal nerves E contains cell bodies of neurons that cause an increase in the rate of peristasis

Explanation The celiac ganglion is one of the preaortic ganglia of the sympathetic nervous system It contains cell bodies of postganglionic sympathetic neurons The sympathetic splanchnic nerves contain preganglionic sympathetic neurons that pass through the sympathetic chain without synapsing These splanchnic nerves go to the preaortic ganglia to synapse The greater splanchnic nerve contains preganglionic neurons from spinal cord segments T5-T9 This nerve synapses in the celiac ganglion The nerve fibers in the vagal trunks are preganglionic parasympathetic fibers that go to the walls of the organs that they will innervate and synapse on postganglionic parasympathetic neurons in the walls of those organs Cell bodies of sensory neurons in the abdomen are found in the dorsal root ganglia or the sensory ganglia of the vagus nerve Sympathetic innervation decreases the rate of peristalsis parasympathetic innervation increases the rate of peristalsis

Which of the following pairs of arteries will allow blood to bypass an occlusion of the celiac trunk

A Left gastric artery-right gastric artery

B Left gastroepiploic artery-right gastroepiploic artery

C Superior pancreaticoduodenal artery-inferior pancreaticoduodenal artery

D Splenic artery-common hepatic artery

E Left gastric artery - proper hepatic artery

Explanation The anastoOlosis of a branch of the celiac trunk and a branch of the superior mesenteric artery will

provide collateral circulation around an occlusion of the celiac trunk Each of the other choices pair

branches of the celiac trunk therefore these will not provide collateral flow around the obstruction of the

celiac trunk The left gastric splenic and common hepatic arteries are direct branches of the celiac trunk

The right gastric artery is a branch of the proper hepatic artery which is a branch of the common hepatic artery The left gastroepiploic artery is a branch of the splenic artery The right gastroepiploic artery is a

branch of the gastroduodenal artery whlch is a branch of the common hepatic artery

Which of the following organs has appendices epiploica The

A sigmoid colon

Bjejunum

C duodenum

D stomach E esophagus

Explanation Appendices epiploica are characteristic of the colon Appendices epiploica are subserosal accumulations

of fat None of the organs of the gastrointestinal tract has appendices epiploica except the colon

Page 8: Chirag's Abdomen Review

CLINICALS -

Trac heoesophageal Almocml COll 11ction htteell esophalIs ll nd tmlthe1 fistula Jost lI)OUllul1 StLblypt is hlind up tr () hngtls with lOler esophagus tO)HHcted to

lrac hea RcsulJ in (3 11 $($ (ho tng anc vo mitin with ftt(~~lS lir bllblllpoundQll CXR and )Qlvl V -~-

I ~ i I Esophageal y i J atresia - - Trachea

----shy

Z

Congenital pyloric l1l perlrnphy (J f le pylurus camt~ obstruti I) Pa Ipablc uhve mass in epigl~tric n~ion stenosis and nOl1biliolts PfOiL mnit+rrg- - - wetk of ltl4C Tn)tment is suriltai ineisi n

Ottmi in 160(llit births ofteIl in I ~ t-bOfl malts --

Peptk ulcer djsease G~ltri( ulcer Pain can he greater vdth mellls--Welghtl - ft(H OCClIlS in older putient

a Mlori tnfedion in 70 chronic NSAID use also implicated Dm to llHue-oml protection a~lllns t gfltrh l id

Duodella) ulcer Pain Deu(QSe5 wilh menh--weighl ~aln_ Almost IOO have H priori infection Dut to 1~ gastfk acid cerct 11 f( 1 2011 inpmiddotr-Ell lson )~md rol11t i ( r L m(lcos~ ll

protction Hyprttophy of Bntnllcr glands Tend to ha ve ltIeall pullcheJ-ouf margim unlike the tais(dJirre~lllar margins of

cnCinOl1lH P-te1itiai eomplications include bl eedin~ penetrnu(n into pnnerelS perfomtion and obs tmctiQH (not intrinsltllly pr((JIlcerom) (see Image 11-+

Question - A 57 year old obese chronic alcoholic presents with an ulcer which has ruptured the cI~wnpastelgrly Surgical investigation reveals blood in the peritoneum Which of shy

the following arteries is most likely responsible for the bleeding

a Splenic Artery

b Gastroduodenal Artery

c R Gastric Artery

d L Gastric Artery

e L Gastroepiploic Artery

Appendicitis All age ~rol1ps most common indication for em er(nl ilxkminai)lJrger) in ehildrefL huLial diffmc periumbilical pain --7 localized pain JtMcBmm~ point N~ltlSC~l kvef

r)13)- perfor~te - pcntouitis Difftr~ntiI1 divertieuli[i (elderly ectopic pr(~gnanl to [3-hCG 10 rule ont)o

-----

PcmiddotrhtltllCf of the ) lcllille dud Of ~Olk sh31k ~br lontli n e( topic ~l cid-seefehllg gatric m 1(0]

andor pmcrcatic ii~~lle Lllost common c(lllgenital anomliy aftlle CT tract Cm CllHt hkfciing illtusm~(~pjjon Dlnllus or nbstrudion nelr the tcrrnin)l ileum Contraslwitll QIB~efic nmiddot = cvtk dilalaUon of ittllilC dJet

------~-

The ile 2$

2Jpound~11 11~ 2 feet frolll the iie( middoteCill vke l~ QfiJ~ at 1~

CIllIn nly prcsenfltgt III rll~2

llf~ of lifc- by ilwe 2 ty )ts of

epilheH8 19ls-trie- pal1elli(i

Hirschsplungs disease

Congenital tnt91(middotolon characterized by lack of Think of a gian ~pring that ganliml ttlLJcnJ(rk ~~gJeXllS~( (~lihs and lei ~sner plCxpstTlrlsgIllent on inbstinal biop y ()It to-iIure of U(middotural t~restpoundtU migration =

has s nl

Presents as ronic comtipnHoll tHly in life Dilllted pOltioll of the colon proximal 10 the aganiionic

segment resulting in a middot trmsjtillll ZQl1t rnvolt~

rectum [huany farlur( to P~$ meconium

High Yield WetDrylTheory List-

Suggestion - in your study group try to write a question for each of these points and then

exchange with a friend and try to answer each others questions

Abdomen Blood Supply - Reference viks picture posted on my google group - this is THE MOST HIGH

YIELD TOPIC IN ALL OF ANATOMY - expect 5 questions on your mini and 5-10 questions on your shelf

Make sure you can draw the blood supply and answer tertiary questions

Example - If the patient had an occlusion of the celiac trunk - which of the following areas would

experience ischemia

Portal hypertension - Know the 3 clinically relevant sites of portal caval anastamoses

Testes vs Scrotum lymph drainage

Where are paraumbillicai veins located

Omalophcele - failure of the gutto come back in (if in yolk sac - fatal)

Marginal Arteries

Superior Messenteric Artery is in front of 3d part of duodenum

Caput medusa (Distended paraumbillical veins secondary to portal hypertension)

Kidney Constrictors -1 Renal Pelvis 2 Crossing Pelvic Brim 3 Entering Urinary Bladder

Ureter - wet lab

Vagus is PIIJS supply up to 23 trans colon than pelvic splanch N up to the ass

Hirshsprungs disease - baby cant poop - dilated colon

Meckels Diverticulum - rule of 2s - 2 feet prox from ileocecal ju nc

Urachal Fistula - weeping belly

Gall stones - common bile duct

Jaundice relation to tumor of the head of the pancreas

Hepatopancreatic ampulla

Blood supply of renal gland - s superrenal art m s suprarenal a abd aorta inf suprarenal art

R Kidney - Tl2-L3 L Kidney - Tl1-L2

Renal Artery - L2

Epiploic foramen - know the borders and contents

Alantois diverticulum - urachus - medial umbilical

Lateral Medial and Median umbillical Folds (know the contents)

Directindirect hernia - know how to diagnose where they enter and exit the inguinal region and which

one is congenital

Anular pancreas - projectile vomiting

Pyloric Stenosis - projectile vomiting (non bilous)

Duodenal Atresia - projectile vomiting (bilous)

Dry Lab - know x rays

Vitteline Fistula - food out of umbilicus

Major duodenal papilla - junc of foregutmidgut

Arcuate line - relationship to rectus sheath

Mcburneys point -13 from ASIS bw umbilicus

Parietal pain - what is the nerve supply

Internal Oblique - cremasteric relationship

Know spermatic fasia

Processes Vaginalis - connection bw peritoneum and gubernaculums

Umbillicus - TlO dermatome

Deep Inguinal-l25 cm above mid inguinal ligament

Superficial Inguinal Ligament- superolateral to pubic symphysis

Variocele - veins engorged in scrotum (bag of worms)

bull

bull External spermatic fascia derived from external obliques EO II Cremasteric fascia ~ from internal obliques fO bull Internal spermatic fascia derived from fascia transversalis bull Tunica vaginalis derived from processes vaginalis directly rests on testes bull know order from testes out to skin

note reflex o ilioinguinal nerve o Efferent =genital branch of the genitofemoral nerve

--lt gt-- info important anastamoses which connects thorax to abdomen

bull Sup

o Sup epigastric branch of internal thoracic o Inf branch external iliac

Venous drainage o Above umbilicus aXillary v o Below umbilicus veins in triangle o At level of umbilicus Paraumbilical veins -gt drain into the portal V

II Important in Portal Caval Venous system Venous drainage of testes

o Clinical correlation Varicocele 11 vein drains into IVC 11 Left testicular vein ~ drains into left renal v

bag of rmlt

for lymph drainage T10 axillary lymph nodes

ill Below T10 superficial inguinal lymph nodes (lateral

Umbilical Folds

Lateral umbilical folds inferior vessels

Medial umbillcial folds umbilical (fetal remnant)

Median umbilical fold urachus (fetal remnant)

Between these folds fossas o Supervesical fossa between median and medial folds

11 bladder o hesselbachs between medial and I folds

II DIRECT HERNIAS HERE Borders

Medial semilunar line

Lateral info Epigastric

Inferior inguinallig o Lateral Inguinal Fossa beyond lateral fold

INDIRECT HERNIAS HERE II Deep inguinal ring (lateral to inferior epigastric a)

Indirect inguinal hernia o Lateral to inferior epigastric a o more common o When inserting finger in superficial inguinal ring will feel on tip of finger (since it goes

throueh ineuinal canall

----

Dry Lab - Label subcostal iliohypogastric Ll Ilioinguinal (Ll)

Horesshoe Kidney - stuck under IMA

Renal Agenesis -failure of the ureter bud to develop

Double Ureter

Unilateral Agenesis -1 kidney

Kidneys - Metanephros

Fetal kidneys are at sacral level

Look at 3rd part of duodenum

Some of this stuff is repeated I know just copied and pasted a bunch of stuff I had copy

Dermatomes

bull T4 nipples

bull no umbilicus v o Pain referred to no in appendicitis o Pain referred to T7ITS in gastritis ~

Inguinal ligament = external abdominal oblique aponeurosis

bull Inserts at anterior superior iliac spine to the pubic tubercle o Why important to know -7 visualizing this line allows us to properly diagnose a hernia

Below the inguinallig femoral hernia Above the inguinallig =inguinal hernia

Also to palpate the deep inguinal ring you go about 12Scm above the mid-inguinal

point

bull Modifications to ligament o Pectinate ligament o Lacunar ligament -7 cut this ligament to relieve strain i~ stran ul~tEd hernia

Inguinal canal

bull in males -7 transmits spermatic cord o important structures of spermatic cord ductus deferens testicular a genital branch of

the genitofemoral n pampiniform plexus of veins bull in females -7 transmits round ligament

Borders

bull Floor -7 inguinal ligament + lacunar ligament bull Anterior -7 aponeurosis of external oblique + internal oblique bull Roof -7 internal oblique and traverse abdominal bull Posterior -7 transverse abdominal + transversalis fascia

o Reinforced by conjoint tendon bull Aponeurosis of internal abdominal obliques and transverse abdominus bull Lies immediately behind the superficial inguinal ring in what would otherwise be

a weak point in the abdominal wall bull Innervated by ilioinguinal nerve (Ll) ~why important

bull In appendicitis Ll can be injured which will injure this nerve and in turn

the conjoint tendon With loss of innervation to this supportive structure the patient is now predisposed to a direct inguinal hernia

o Only hernia that can transverse the inguinal canal o Associated with congenital condition persistent tunica vaginalis

bull Direct inguinal hernia o Medial to inferior epigastric a o When inserting finger in superficial inguinal ring will feel on back of finger o Associated w old age or recent surgery

Muscles (Abdomen RECTUS SHEATH)

bull Arcuate line at level of ASISor 13rd distance between pubis and umbilicus bull Above arcuate line rectus abdominus is surrounded by a rectus sheath anteriorly and

posteriorly

o EO and 10 lie over rectus abdominus o 10 and TA lie behind rectus abdominus

bull Below arcuate line rectus abdominus has no rectus sheath posteriorly o EO 10 and TA lie over rectus abdominus o Transversalis fascia lies behind rectus abdominus o Inf EpIgastric vessels pierces the rectus sheath here

Peritoneum serous sac which encloses most of the abdominal structures

bull Ovary =only intraperitoneal organ o Oocyte ejected from ovary then captured by fallopian tubes o Why impt Women more prone to infection that can enter peritoneum

Peritoneum forms

bull Mesentery double layered fold of peritoneum formed as the organ was pulled in

bull Ligament between 2 organs in general bull Omentum between stomach and another organ bull Bare area area of no peritoneum bull

Viscera innervation

bull Pa rasympathetics 11 o Afferents sense hunger o Efferents l peristalsis relaxes sphincters gland secretion

bull Sympathetics o Efferents do opposite o Afferents CARRY PAIN SENSATION OF THE VISCERA (dull stretching pain)

bull PARASYMPATHETIC INNERVATION o Vagus nerve 7 _1l to 23rd unct ion of la rgej nte~tine oJ)elVrcspla~~~)~~ic~rYe~ IJiU- ~rd aJ~lpoteotiD~ IMPT

Gut Embryology

Gut ~ We say that the gut is derived from endoderm We often forget that when we say so we mean

that only the mucosa is derived from the endoderm The submucosa and the muscle layer is actually derived from the splanchnopleuric mesoderm and the serosa is derived from the visceral peritoneum

~ The main function of the gut is to digest the food which is done by the glands derived (and are) in the mucosa (endoderm) The only two exceptions in the Gut where glands though derived from the endoderm do not stay there but migrate down into the submucosa are esophagus and duodenum These glands however have their ducts opening to the swface of the mucosa

bull

~ Lungs liver amp gall bladder and pancreas are off-shoots from the foregut Esophagusshy~ The region of the tube from the laryngeal diverticulum to the beginning of the stomach elongates

to form the esophagus ~ The glands which form in the endoderm (mucosa) migrate down into the submucosa The path

whlch it took migrate becomes the duct of the glands which open to the mucosa ~ Achalasia Cardia - Failure of relaxation of the lower esophageal sphincter because of congenital

absence of ganglia at the sphincter (The ganglia when present releases VIP (Vaso-IntestinalshyPeptide) which relaxes the sphincter)

Mid-Gut Rotation ~ Because of the 90 degree rotation of the primitive stomach all of the following events occur ~ Lesser curvature comes to the right Therefore lesser omentum also comes to the right ~ Greater curvature comes to the left Therefore greater omentum also comes to the left ~ Right side vagal trunk becomes posterior vagal trunk ~ Left side vagal trunk becomes anterior vagal trunk ~ The left side peritoneal cavity comes to the anterior aspect of the stomach and will later be called

as the greater sac ~ The right side peritoneal cavity comes to the posterior aspect of the stomach and is (relatively a

small sac because the liver is on the right) called the lesser sacomental bursaepiploic sac ~ Epiploic foramen of Winslow (the lower free margin of the ventral mesentry) wiII be the

communication between the greater and lesser sac ~ The Liver moves to the right and therefore actually causes the 90 degree rotation of the stomach

The spleen comes to lie on the left side ~ Axis Antero-posterior axis around the superior mesenteric artery

bull Counterclockwise bull Approximately 270deg bull During herniation (about 90deg) bull During return (remaining 180deg)

Duodenum ~ Becomes retroperitoneal (except the first part which is still suspended by the hepato-duodenal

part of lesser omentum) ~ Glands (of Brunner) go submucosal ~ An imaginary line drawn below the opening of the major duodenal papilla represents the junction

between the foregut and midgut ~ Duodenal atresia in Downs syndrome Liver ~ 3rd week

bull liver bud grow bull into the septum bull transversum

~ 10th week bull hematopoietic bull function

bull 10 of the total bull body weight

~ 12th week bull bile is formed

Pancreas ~ In about 10 of cases the duct system fails to fuse and the original double system persists ~ 3rd month

bull pancreatic islets (Langerhans) ~ 5th month

bull Insulin secretion ~ Annular pancreas

bull The right portiCn of the ventralbud migrates along its normal route but the left migrates in the opposite direction

~ Complete obstruction of duodenum ~ Accessory pancreatic tissue Polyhydramnios (Amniotic fluidgt 1500-2000 ml)

~ Congenital defects including central nervous system disorders (eg Anencephaly) and gastrointestinal defects (atresias ego Duodenal esophageal) prevent the infant from swallowing the amniotic fluid (failure of recanalization)

Oligohydramnios (Amniotic fluid lt 400 mt) ~ Cl~ldberenal-agenesis

bull Midgut _-_

~ Primary Midgut intestinal loop gives rise to bull Distal duodenum bull Jejunum bull Ileum bull Ascending colon bull Transverse colon - proximal two-thirds of the bull Transverse colon with the distal third

~ Primary intestinaltoop bull ncephalic limb distal part of the duodenum the jejunum and part of the ileum bull ncaudal limb lower portion of the ileum the cecum the appendix the ascending colon and

the proximal two-thirds of the transverse colon bull 6th week

bull Rapid elongation of the cephalic limb bull Rapid growth of the liver bull Intestinal loops enter the extraembryonic cavity in the umbilical cord

bull 10th week bull loops begin to return bull regression of the mesonephric kidney reduced growth of the liver expansion of the

abdominal cavity bull Jejunum -left bull Loops - more to the right

bull Cecal bud -last part (temporarily below the right lobe of the liver) ~ qIDlthaloseJe (Structures COlHLoArts9V~1tion)

bull Through umbilical ring bull 6th to 10th weeks

bull Associated with a high rate of mortality (25) and severe malformations bull Associated with chromosome abnormalities

~ Gastroschisis (Structures coming out are not covered by Amnion) bull herniation through the body wall ----=---=-shybull Into the amniotic cavity bull Lateral right of the umbilicus bull Sometimes the inferior wall fails to develop as a result lower abdominal structures like the

bladder would be exposed to the exterior not associated with chromosome abnormalities ~ Abnormalities of the Mesenteries

bull Mobile cecum persistence of mesocolon bull Extreme form - long mesentery bull Volvulus

~ Distal third of the transverse colon ~ Descending colon ~ Sigmoid colon ~ Rectum ~ Upper part of the anal canal ~ Primitive anorectal canal

bull 7th week cloacal membrane ruptures bull Tip of the urorectal septum perineal body bull Pectinate line

~ Hindgut anamolies bull Rectoanal atresias and fistulas bull Imperforate anus bull Congenital megacolon (aganglionic megacolon Hirschsprung disease)

bull

bull Hindgut

Chirags Abdomen Review - Part 2

Understanding Embryo makes learning blood supply EASY

I I

I t

~ -

)

Table l1r-~ L Adult SUmiddotuctu~SDrj~l Froln Each of he Three Dhisions of be Pringttive GUl Tube t-middot-----middotmiddotmiddotmiddot-

Foregu(

I_ (Celiac Trunk)

Ir-slt-gtphgus

S101na(b

I h -= LiJ~r

Pancre=l S

bull 1 i Biliary apparntu5

Gall bladdshy

i Pha11~Cal pltgtuchcs

LullSS-I

Mjig ---- bull __ _- ----n--duct----~---middot-------l--n

(Superior Jldesen1eric Artery)_-1I-(I_~__ middot __ O-=-)_in_middoto_r_M_e_se_n_t_e_r_i_c_An__

Uuodenu rn 2nd_ 3 lt141h V4Tt

Jejunun-~

nc-un]

tCCUJ11

AppltgtndLX

Transver5e -o1on (p~oxiln1l1 ~O Tbird)

bull__hytgtid~ _ _ ______ L _ __

Tr-dn~llt~se colon (diStul h lTd) I

)

i

Aa ca-nal -( uppeT patt) i

I I

_____ __ _ _ _ ___ - - rhe~ a(t clcriVOkt iV(5 opound~lt prbn1rC ~ nlQC blft TlI)( 134tof r~ tIonoinf~ i 1 ~l l1rd c- P Cle

Now Lets see how much youve learned

Questions

1) A pt receives a general anesthetic in preparation for a c~t~~my A right subcostal incision is made which begins near the xyphoid process runs along and immediately beneath the costal margin to an anterior axillary line and transects the rectus abdominus muscle and rectus sheath At the level of the transpyloric plane the anterior wall of the

-~~-~=--- _eco---shysheath of the rectus abdominus muscle receives contributions from which of the following

a Aponeuroses of the in~ande~tef-Ilal o~ues

b Aponeuroses of the transversus abdominis and internal oblique muscles c Aponeuroses of the transversus abdominis and internal and external oblique

muscles d Transversalis fascia e Transversalis fascia and aponeurosis of the transversus abdominus muscle

A

2) The lat~raJJJ11QjJt~gLfgJlLoneach side of the inner surface of the anterior abdominal wall is created by which of the following structures

K Falx inguinalis (~) Inferior epigastric a

c Lateral border of the rectus sheath d Obliterated umbilical a e Urachus

B

3) A man the victim of several knife wounds to the abdomen during a brawl at the Lobster Shack subsequently developed a direct inguinal hernY Damage to which of the following nerves is most likely responsible for the predisposing weakness of the abdominal wall

~ Genitofemoral nerve ( b) Ilioinguinal nerve ~-t Tenth intercostal nerve

d Subcostal nerve e Pelvic splanchnic nerve

B

4) Which of the following statements concerning a direct inguinal hernia is correct a It is the most common type of abdominal hernia b It transverses the entire length of the inguinal canal c It contains all3 fascia layers of the spermatic cord d It exits the inguinal canal via the superficial ingeJinal ring e It protrudes through H~acb strJg e

~(

1fltbS w E

tl

5) The conjoint tendon is

a Important in preventing indirect inguinal hernias b The fused aponeurotic layers of internal abdominal oblique and transversus

abdominus muscles c Posterior to the deep inguinal ring

d Medial fibers of the inguinal ligament

B

6) A 25 year old male is brought in to the ER after being involved in a car accident in which he received a crushed internal injury in his abdomen Examination reveals a lesion of parasympathetic fibers in the vagJsnerve which interferes with glandular secretory or

smooth muscle functions in which of the foliowingorgans a Bladder b Transverse coloiW c Descending colOO d Prostrate gland e Rectum

B

7) The spermatic cord includes all of the following contents except a Il ioinguinal nerve b Pampin iform plexus of veins c Vas deferens d Genitofemoral nerve

A

8 Which abdominal structure gives rise to the internal spermatic fascia (muscle) following the descent of testes in development

a External abdominal oblique aponeurosis b Transversalis fascia c Transversus abdominis muscle d Peritoneum e Internal abdominal oblique

B

9 Which abdominal structure gives rise to the tunica vaginalis fotlowing the descent of testes during development shy

a External abdominal oblique aponeurosis b Transversalis fascia c Transversus abdominis muscle d Peritoneum e Internal abdominal oblique

D

10) The lesser omentum is a peritoneal fold which is su bdivided into the a Hepatogastric and gastrosplenic ligaments b Hepatoduodenal and gastroomentalligaments c Hepatoduodenal and gastrosplenic ligaments d Hepatogastric and hepatoduoden9-jrj igaments

D

11) A posteriorly perforating ulcer in the pyloric antrum of the stomach is most likely to produce initiallocalized peritonitis or abcess formation in which ofthS fQllowing

a Great-sac - -- -

b Paracolic recess

c Omental bursa

d Right subphrenic space

c

The inferior mesenteric artery arises from the abdominal aorta ilm_ediill~y_J-Qs1eriQLto which of the foowing org~ns A-F~t~filie duodenum B Head of the pan~eis C Neck of the pandeas

D Second part of the duodenum

E Third part of the duooenum_shylaquoshy

shy

The correct answer is E The inferior mesenteric artery arises from the anterior surface of the aorta at the level of the third lumbar vertebra The third part of the duodenum crosses the midline at the level of the third lumbar vertebra and passes anterior to the aorta at the origin of the inferior mesenteric artery The

first part of the duodenum (choice A) lies horizontally to the right of the midline at the level of the first

lumbar vertebra The head of the pancreas (choice B) is to the right of the midline and extends from the

level of the first lumbar vertebra to the third lumbar vertebra It lies within the concavity of the

duodenum The neck of the pancreas (choice C) lies in the midline at the level of the first lumbar

vertebra It lies on the anterior surface of the aorta at the origin of the superior mesenteric artery The second part of the duodenum (choice D) lies vertically to the right of the midline and extends from the

level of the first lumbar vertebra to the level of the third lumbar vertebra

The left adrenaLvein drains directly into which of the following veins A Hemiazygos vein

B Inferior vena cavaee C Left renal veiri -

D Splenic vein

E Superior mesenteric vein

a

The correct answer is C The left adrenal vein and the left gonadal vein (either testicular or ovarian) drain into the left renal vein TheTeft renal vein t~ains intothe- inferior vena cava In contrast the right

adrenal ~~inandnght gonadal veindrai~ gLr~ctJy iQtoJhe iilferiQ[ Vencava -- -

ThehemTazygoS7ein- (~h-~i-~ A)~~c~i~es the venous drainage from the body wall on the left side of the

thorax and abdomen No visceral organs drain directly to the azygos or hemiazygos veins The inferior vena cava (choice B) receives the direct venous drainage from the right adrenal vein but not

the left adrenal vein Remember the inferior vena cava is on the right side of the abdomen The splenic

vein (choice D) receives the venous drainage from the spleen and part of the pancreas and stomach The splenic vein is part of the portal venous system

The superior mesenteric vein (choice E) receives venous drainage from much of the intestinal tract It is part of the portal venous system and joins with the splenic vein to form the portal vein

A 43-year-old man presents complaining of pain in the groin On examination his physician palpates a

bulge in the region of the superficial inguinal ring which he diagnoses as a direct inguinal hernia The hernial sac most likely

A is covered by all three layers of the spennatic fascia B passes medial to the inferior epi gastric artery

C passes medial to the lateral border of the rectus abdominis muscle

D passes posterior to the inguinal ligament E passes through the deep inguinal ring

The correct answer is B Direct inguinal hernias enter the inguinal canal by tearing through the posterior

wall of that structure The typical location for this type of hernia is through the inguinal triangle bounded

laterally by the inferior epigastric artery medially by the lateral border of the rectus abdominis and

inferiorly by the inguinal ligament Direct inguinal hernias pass medial to the inferior epigastric artery

whereas indirect inguinal hernias pass lateral to the inferior epigastric artery because the deep inguinal

ring is lateral to the artery Indirect inguinal hernias are covered by all three layers of the spermatic fascia (choice A) Direct inguinal hernias are covered by fewer than all three layers because the direct inguinal

hernia tears through one or more layers of fascia as it emerges though the abdominal wall The lateral

border of the rectus abdominis muscle (choice C) forms the medial border of the inguinal triangle All

inguinal hernias pass lateral to the rectus abdominis Femoral hernias pass posterior to the inguinal ligament (choice D) Inguinal hernias emerge through the superficial inguinal ring which is superior to the inguinal ligament Inguinal hernias that descend below the inguinal ligament pass anterior to the

ligament Indirect inguinal hernias pass through the deep inguinal ring (choice H) direct inguinal hernias

do not Both types of inguinal hernias pass through the superficial inguinal ring

During a gastric resection in a patient with stomach cancer a surgeon wants to remove the lesser

omentum because of tumor extension into it Which of the following structures lie in the free edge of the

l~~g omentum and consequently must be dissected out in order to be preserved

A Common bile duct cystic duct and hepatic artery 6

B Cystic duct hepatic artery and hepatic vein

e Hepatic vein and cystic duct

Portal vein common bile duct and hepatic artery

E Portal vein hepatic artery and hepatic vein

The correct answer is D The free edge of the lesser omentum contains three important structures the

common bile duct the hepatic artery and the portal vein Nei ther the cystic duct (choices A B and C) nor the hepatic vein (choices B C and E) lies in the free

edge of the lesser omentum

A 55-year-old male patient with chronic liver disease has portal hypertension To relieve the pressure in the portal system a porto-caval shunt is performed Which of the following veins may by anastomosed to

accomplish this porto-caval shunt A Left renal vein-left testicular veingt

B Right renal vein-right suprarenal vein I shy

e Splenic vein -left renal vein J

D Superior mesenteric vein-inferior mesenteric vein E Superior mesenteric vein-splenic vein

The correct answer is C The splenic vein drains directly into the portal vein The left renal vein drains

directly into the inferior vena cava Anastomosis of these veins would allow blood from the portal vein to

drain retrograde though the splenic vein into the renal vein and then into the inferior vena cava The left

renal vein (choice A) drains directly into the inferior vena cava The left testicular vein drains directly into

the left renal vein Thus these veins are already in communication and neither vein is part of the portal venous system The right renal vein (choice B) drains directly into the inferior vena cava The right

suprarenal vein also drains directly into the inferior vena cava Thus neither vein is part of the portal

venous system The superior mesenteric vein (choice D) drains directly into the portal vein The inferior

mesenteric vein drains into the splenic vein which then drains into the portal vein Thus neither vein is

part of the caval venous system The superior mesenteric vein (choice E) drains directly into the portal

vein The splenic vein also drains directly into the portal vein Thus neither vein is part of the caval

venous system

A 12 year old boy has fever vomiting and para-umbilical pain After examining the patient the doctor

makes an initial diagnosis of appendicitis Appendicular pain which is initially referred to the umbilicus goes to the dorsal root ganglion of

a TI b TI2 c L1 d T7

(e I TIO

A 59-year-old male undergoes a neurological examination which reveals that when the abdominal wall is

stroked the muscles of the abdominal wall of the side of the body stimulated failed to contract Other

neurological tests appeared normal The likely region affected includes

a CI - C5 spinal segments b C6 - TI c T2-TI ~T8-T12

e Ll- L5

The surgery done to relive portal hypertension is done by connecting two veins Which of the following veins would be suitable for connection

a Inferior vena cava and portal vein b Superior vena cava and portal vein c Splenic vein and right renal vein d Splenic vein and left renal vein e Superior mesenteric vein and Inferior vena cava

A mother brings her 3-week-old infant to the pediatric clinic reporting a new scrotal bulge that she found -~-

while changing a diaper yesterday The infant is afebrile Physical examination reveals a palpable mass in

the scrotum while in the standing position resolution of the mass in the supine position and no

transillumination of the scrotal sac What is the most likely diagnOSiS

a Cryptorchidism b Direct inguinal hernia c Hydrocele d Indirect inguinal hernia ~ e varicocele

The Vagal trunks enter the abdomen by passing through which of the following openings in the

diaphragm

a Right crus b Esophageal hiatus ~ c Vena caval hiatus d Aortic hiatus e Left crus

2 The anterior boundary of the epiploic foramen of Winslow is bounded by

a) First part of duodenum b) Lesser curvature of stomach c) Liver d) Hepato-duodenalligament v ~

3 The ilio-inguinal nerve is derived from

a TI2 ry b LI c L2 d L3 e L23

15 Surgically the structure used to suspend the kidney to the diaphragm is

a) Renal fascia b) True capsule c) Perinephric fat d) Paranephric fat

6 If there is portal obstruction because of carcinoma affecting the pancreas which of these of the

following signs would be present

a Caput medusae b Esophageal varices c Rectal varices c

d Pulmonary edema

7 In a sliding hernia the gastro-esophageal junction lies

a) At its normal position b) Below the normal position c) Above the normal position V d) None of the above

8 Which of the following structures is retroperi toneal

A transverse colon B spleen IJ2f6 C ileum D descending colon v r 1pound1111111

9 The renal angle is fonned lgtetween the 12th rib and ______ muscle

a Psoas major -middotshyb Erector spinae c Quadratus Iumborum d Diaphragm

10 The anterior structure at the hilum of the kidney is

a) Renal vein ~

b) Renal artery I middot~ I

c) Ureter d) Accessory renal artery

11 Because of origin of the muscle from the lateral one third of the inguinal ligament it

could not fonn the anterior wall of the inguinal ligament

a) External oblique b) Internal oblique c) Transversus abdominis_ d) Rectus abdominis

12 A large tumor mass impinges on the splenic artery and its branches as the artery pass out from below

the greater curvature of the stomach Branches o(which of the following arteries would most likely to

effected by the pressure on the splenic artery

a Left gastric b Left gastro-epipJoic c Right gastric d Right gastro-epipoloic e Short gastric_

13 A new born baby has projectile vomiting after each feeding It is determined that there is obstruction

of the digestive tract as a result of annular pancreas Annular pancreas is as a result of an abnormality in which of the following process

a Rotation of the dorsal pancreatic bud around the first part of duodenum b Rotation of the dorsal pancreatic bud around the second part of duodenum c Rotation of the dorsal pancreatic bud around the third part of duodenum d Rotation of the ventral pancreatic bud around the first part of duodenum y Rotation of the ventral pancreatic bud around the second part of duodenum

14 As the liver bud enters the ventral mesogastrium the region of the mesogastrium stretching from the

liver to the anterior abdominal wall is called

a Lesser Omentum b Greater Omentum ~ Falcifrom ligament d Lacunar ligament e Ligamentum teres of liver

16 A patient has absence of his 12th rib In such a patient if the doctor makes an incision to approach his

kidney mistaking the 11 th rib for the 12t he would end up injuring

Which of the following arteries is a direct branch of the gastroduodenal artery The

A right gastric artery

B left gastric artery

C inferior pancreaticoduodenal artery D left gastroepiploic artery

i E)right gastroepiploic artery --

E x pI a nation The right gastric artery is typically a branch of the proper hepatic artery The left gastric artery is a direct

branch of the celiac trunk The right and left gastric arteries anastomose along the lesser curvature of the

stomach The inferior pancreaticoduodenal artery is a branch of the superior mesenteric artery it

anastomoses with the superior pancreaticoduodenal in the head of the pancreas The left gastroepiploic

artery is a branch of the splenic artery it anastomoses with the right gastroepiploic artery along the greater

curvature of the stomach The right gastroepiploic artery is a branch of the gastroduodenal artery The

other branch of the gastroduodenal artery is the superior pancreaticoduodenal artery

Which of the following pairs of veins join together to form the portal vein The

A superior mesenteric vein and inferior mesenteric vein

B inferior mesenteric vein and splenic vein

C superior mesenteric vein and splenic vein

Ip)splenic vein and left gastric vein E superior mesenteric vein and left gastric vein

Explanation

The portal vein is formed behind the neck of the pancreas by the union of the superior mesenteric vein

and the splenic vein The inferior mesenteric vein drains into the splenic vein The left gastric vein drains

directly into the portal vein After the portal vein forms it enters the hepatoduodenalligament of the

lesser omentum to reach the liver The portal vein is the most posterior structure in the hepatoduodenal

ligament

At which of the following vertebral levels does the duodenum pass anterior to the aorta - _- shy

All ~

B L2 7~

CL3 I

~DL4

E L5

Explanation

The duodenum begins at the pyloric sphincter at the level of Ll The second (or descending) portion of

the duodenum is to the right of the aorta and extends inferiorly from the level of Ll to the level of L3 The third part of the duodenum crosses the aorta from the right side to the left side at the level of L3 The

fourth (ascending) portion of the duodenum extends from the level of LJ to the level of L2 The

duodenum ends at the duodenojejunal flexure The superior mesenteric artery passes anterior to the

duodenum as the duodenum passes anterior to the aorta The duodenum can be constricted at this level

In which of the following locations will perforation of the digestive tract result in the spilling of luminal

contents into the - lesser peritoneal sac

A Anterior wall of the second portion of the duodenum B Posterior wall of the second portion of the duodenum

C Anterior wall of the stomach

~Posterior wall of the stomach E Posterior wall of the transverse colon

Explanation

The posterior wall of the stomach is related to the lesser peritoneal sac The anterior wall of the stomach is related to the greater peritoneal sac The anterior wall of the second portion of the duodenum is related to the greater peritoneal sac The posterior wall of the second portion of the duodenum is related to the retroperitoneal space The posterior wall of the transverse colon is related to the greater peritoneal sac

The ureter lies against the anterior surface of which of the following muscles shyA Crus oftne diaphragm B Quadratus lumborum

0 Psoas major D Transversus abdominis

E Iliacus

Explanation The ureter exits the renal pelvis at about the level of vertebra L2 As it descends along the posterior abdominal wall it lies on the anterior surface of the psoas major The psoas major muscle arises from the bodies of the lower lumbar vertebrae The psoas major muscle is joined by the iliacus to fonn the

iliopsoas muscle The iliopsoas muscle then attaches to the lesser trochanter of the femur and is the major

flexor of the hip

As the right ureter passes the pelvic brim it lies against the anterior surface of which of the following

blood vessels

A Gonadal artery B Inferiorvena cava C Internal iliac artery

rJ- External Iliac artery

E Inferior mesenteric artery

Explanation

The ureter lies in the extraperitoneal space in the posterior abdominal wall Alter leaving the kidney it

passes inferiorly on the anterior surface of the psoas major muscle At the pelvic brim the ureter passes

into the pelvis At this point the common iliac artery is dividing into the external and iliac arteries The

ureter lies on the anterior surface of the external iliac artery immediately distal to the bifurcation This is a useful landmark for a surgeon to locate the ureter

When extravasated urine passes from the superficial perineal space into the anterior abdominal wall it is

found immediately deep to which of the following layers of the anterior abdominal wall

-ltScarpas fascia

B External oblique muscle

C Internal oblique muscle D Transversus abdominis muscle

E Transversalis fascia

Explanation

The superficial perineal space is bound by Colles fascia the fibrous portion of the superficial fascia This

layer of fascia is continuous with Scarpas fascia the fibrous portion of the superficial fascia of the anterior abdominal wall Therefore urine that is deep to Colles fascia will remain deep to Scarpa s fascia The urine will spread in the plane between Scarpas fascia and the external oblique layer

When a horseshoe kidney develops the ascent of the kidney is restricted by the A internal iliac artery B external Iliac artery

C common iliac artery

inferior mesenteric artery

E superior mesenteric artery

Explanation

A horseshoe kidney develops when the inferior poles of the to kidneys fuse together as they ascend into

the abdomen from the pelvis The first anterior midline vessel that is encountered by the horseshoe kidney

is the inferior mesenteric artery This artery prevents the kidney from continuing its ascent

The left testicular vein drains into which of the following veins

A Left internal iliac vein B Left common iliac vein

bflnferior vena cava D Left renal vein I

E Left internal pudendal vein

Explanation

The left testicular vein drains into the left renal vein The right testicular ~i~[~nsltjectlY into the

inferior vena cava This difference in venous drainage is believed to explain the greater incidence of

varicocele on the left side than on the right The venous drainage from the penis is to the internal vein

which then drains into the internal Iliac vein

The spinal nerve that provides cutaneous branches to the skin around the umbilicus is

A TS B TW-shy

C TI2

DL2 EtA

Explanation

The tenth intercostal nerve is the anterior ramus of the TIO spinal nerve After passing through the tenth

intercostal space the nerve continues forward in the anterolateral abdominal wall in the plane between

the internal oblique muscle and the transversus abdominis muscle In the abdominal wall the nerve innervates to the abdominal wall muscles as well as the skin and the parietal peritoneum The umbilicus is

a useful landmark for the region of distribution of the tenth thoracic nerve

The ligament of the vertebral column that resists its extension is the Aligamentum flavum

B supraspinous ligament

C posterior longitudinal ligament

D anterior longitudinal ligament

E interspinous ligament

Explanation

The ligaments of the vertebral column that resist flexion of the column include the supraspinous ligament

interspinous ligament ligamentum fiavum and posterior longitudinal ligament The ligament that resists

extension is the anterior longitudinal ligament This longitudinal ligament is very broad and strong It

covers the anterior and anterolateral surfaces of the vertebral bodies and the intervertebral disks In

addition to resisting extension the anterior longitudinal ligament provides reinforcement to the anterior

and anterolateral surfaces of the intervertebral disk The posterior longitudinal ligament is relatively

narrow and covers the posterior surface of the vertebral bodies and the intervertebral disks This ligament

reinforces the posterior surface of the disk The posterolateral surface of the disk is not reinforced and it

is through this region that herniation of the nucleus pulposus usually occurs

A patient presents with epigastric and right upper quadrant pain The pain is most intense 2-4 hours after

eating and is reduced by the ingestion of antacids The patient states that he has passed black tarry stools

(melena) within the last week Fiberoptic endoscopy reveals a yellowish crater surrounded by a rim of

erythema that is 3 cm distal to the pylorus Accordingly an ulcer has been identified in the patients

A fundus

B antrum

C duodenum

D jejunum

E ileum

A number of physiologic genetic and other factors increase the risk of gastric (and duodenal) peptic

ulcers The evidence that H pylori plays a principle role is compelling Smoking and caffeine are known to adversely affect the morbidity mortality and healing rates of peptic ulcers In general first-degree

relatives of peptic ulcer patients as well as males have a threefold to fourfold increased risk of developing this disorder Paradoxically in gastric ulcer disease acid secretion is not elevated It is possible that

excess secreted hydrogen ion is reabsorbed across the injured gastric mucosa In general a defect in gastric mucosal defense is the more important local physiologic

A patient presents with symptoms of duodenal obstruction caused by an annular pancreas Annular pancreas is caused by

A rotation of the dorsal pancreatic bud into the ventral mesentery B rotation of the ventral pancreatic bud into the dorsal mesentery

fJ failure of the major and minor pancreatic ducts to fuse ~ ~ cleavage of the ventral pancreatic bud and rotation of the two portions in opposite directions around -the duodenum E formation of one pancreatic bud instead of two

Explanation Normally the ventral pancreatic bud rotates around the gut tube to reach the dorsal pancreatic bud The two buds fuse to form a single pancreas and the distal portions of the two ducts fuse The ventral pancreatic bud forms the inferior portion of the head of the pancreas the uncinate process and the major pancreatic duct (of Wirsung) The dorsal pancreatic bud forms the superior part of the head the neck body and tail and the minor pancreatic duct (of Santorini) Annular pancreas is the result of the ventral pancreatic bud dividing into two portions before it rotates into the dorsal mesentery Each portion rotates in opposite directions to get to the dorsal mesentery thus encircling the duodenum The presence of annular pancreas can constrict the duodenum thus obstructing its lumen

In n _ phranlc----

Gon ~l ----_1 Lum bltano

~~--- CornmQ1t bull ac

+-~4--- lnlllirnaJ ilic

xtem iliac

OBJECTIVE - Identify the blood supply to each of the structures listed in the table on the previous page

Ill give you a head start

FOREGUT - Supplied bV Celiac Tru nk (T12)

Proper hepatic

GastiooUod 13Jafter

1nferlor pancreaticoduodenal artery

Common epatlc

Lett gas ric iiirtery

Spfen artery

shy Gastroepiphgtic artery

~ Superior mesenteric 8rtfry

~

1 Esophagus is a derivative of the foregut so its blood supply originates from the celiac trunk

(T12) The predominant blood supply to abdominal portion of the esophagus is the Esophageal

A (Branch of L Gastric) The venous drainage of the esophagus is particularly important because

it is 1 of 3 clinically relevant sites of Portal Caval anastamoses The Portal Esophageal Vein

meets the Caval Azygos System Persistent bleeding manifests as Esophageal Varices - a fata I

condition

2 The Stomach is also a derivative of the foregut has EXTENSIVE blood supply and is very high

yield on anatomy exams The lesser curvature is supplied superiorly by the L Gastric A (1 of 3

major branches ofthe Celiac trunk) and inferiorly by the R Gastric A ( a branch ofthe proper

Hepatic A) The greater curvature is supplied superiorly by the L Gastroepiploic A (a major

branch of the splenic A) and inferiorly by the R Gastroepiploic A

The Short Gastric arteries (branches of Splenic Artery) supply the fundus of the stomach and

are referred to as EIID ARTERIES because they have no collateral blood supply Therefore if the

splenic artery were occluded (ex - increased pressure in the ommental bursa) - there would be

ischemia to the fundus of the stomach Venous drainage of the stomach is extensive via various

veins lead ing to the portal system Posterior to the stomach the IMV joins the splenic V which

joins the SMV to form the PORTAL VEIN ADAMS

3 Duodenum blood supply has high clinical relevance because it is the junction of the foregut and

midgut and therefore is the site of anastamoses between branches ofthe Celiac Trunk (main

foregut artery) and the Superior Messenteric Artery (main midgut artery) The Proper hepatic

artery gives off the gastroduodenal artery which travels behind the 1st part of the duodenum

This point has high clin ical relevance because duodenal ulcers are very common and a posterior

rupture of the 1st part of the duodenum could rupture the gastroduodenal artery causing

traumatic abdominal bleeding The Gastroduodenal artery first gives off the R Gastroepiploic A

(mentioned above) and proceeds as the Superior pancreatico duodenal artery (supplies the

pancreas and duodenum) which anastamoses with the inferior pancreatico duodenal A (branch

of the SMA) This is the junction of foregut and midgut and occurs near the opening of the

bil iary system into the duodenum (ampula of vater) Portal venous drainage here is responsible

for delivering nutrients from digestion to the liver for metabolism Appreciate that the Superior

mesenteric artery (artery of the midgut) branches from the aorta at Ll travels posterior to the

pancreas than moves anteriorly (at the jxn of the pancreatic headbody) and comes over the

3rd4th part of the duodenum Tumor of the head of the pancreas can compress the SMA

4 Jiver blood supply is via the common hepatic artery (major branch of the cel iac trunk) The

common hepatiC becomes the proper hepatic gives off the R gastric A and the Gastroduodenal

A and then joins the common bile duct and the portal vein in the portal triad Clinical- if a

patient were bleeding from the hepatic A a surgeon can stick his fingers in the epiplOic foramen

and squeeze the free edge of the hepatoduodenalligament in order to stop bleeding to the

area Please note that the hepatic a branches into Rand L hepatic A The Right hepatic artery

gives off the cystic artery which supplies the gallbladder Afferent venous supply is via the

Portal vein which is bringing nutrient rich blood to the liver After metabolism takes place

venous blood leaves the liver through the hepatic veins into the IVC PLEASE UNDERSTAND THE

RELATIONSHIP OF THESE STRUCTURES - ADAMSNETIERSNH Etc

5 Pancreas - Head is supplied via the superior and inferior pancreaticoduodenal arteries

(mentioned above) The tail (situated towards the hilum of the spleen) is supplied via the

pancreatic branches of the splenic artery (END ARTERIES) This blood supply is very important

because the endocrine Alpha and Beta Cells from the pancreatic islets of lagerhans are located

towards the tail This is where Insulin and Glucagon is released to the blood

Now complete this for mid and hindgut structures Make sure to note clinically relevant arterial

anastomoses as well as portal caval anastomoses FYI Appendix blood supply SMA + IMA

anastamoses marginal artery Portalcaval rectal veins fhemmorhoids) and periumbilical caput

medusa are high yield THE BUTT THE GUT and THE CAPUT

Abdominal Development

Liver

Ij1f

II wall b

oh liN ~ VltJrti n be- bull

Pancreas

Secondary Retroperitonealization e I~tl r 1 a v-mtrai m ellter

Rotations of the Gut I i Ij (lIl1UtIJ f~ l r tilt

()l td 10 me l-ft and he v

--~--- -~ -~-~

i

I AolaijonjoI~guf I

STOMACH BED (IDENTIFY IN ADAMS)- the structures posterior to the ommental bursa which

support the stomach in the supine position

Abdomnal JQrUI

Splnic vein

OmQ-oul tv~ ) O(s(Jroa)

Lojt(r o m nturrt (hpJtodu o d~n31 Hid

Gadrl)SplerH (g3stroll~nal) IIgam~nt

hiad h~~atogrtricent IIQdmiddotcrt~)

Lt Dome of Diaphragm (why left Look this up in Adams)

Spleen (What is the blood supply)

Left Kidney (What is the blood supply - AND how is it different from the R kidney)

Suprarenal Gland (What is the Arterial AND Venous Blood supply - how are they different)

Pancreas (How does supply differ from Head to Tail What is the SMA Relationship)

Transverse Mesocolon

liver - ADAMSWET - Make sure you look at the liver in wet lab

Left triangular nl1am~nt

ComoaDj ligamnt

Erophg~1 impre$ioo

Hepatio veins

In1erior -ifena middotr3)Ia

Fibrous appendix o-t

live

impr~j on

Heprorendl p~rtion of Q)(Qllary ligament

Righllri~n9ul r 1I~met

(Common) bile quol

Gr)mmCtr~ hepatic dlJct

Ccentic duct

Duodenal impression

GaJdate p-fr)~S

Hepatic artgtrl prop-f iiiiila - Faloiform ligament

_ - shy Round ligamen liver

~--F-- CoJio imprgt-ssi-on

Prta heptis

Identify the lobes impressions and embryonic remnants associated with the liver

Caudate Lobe Quadrate Lobe Right Lobe Left Lobe Round ligament Falciform Ligament

Ligamentum Venosum (what is its fxn in embryonic life) Hepatic Veins (NOT PART OF THE

PORTAL TRIAD) IVC PORTAL TRIAD - Contents relationship cross section etc Know the

Galbladder relationship to the lobes of the liver

Biliary Duct System - Make sure you understand the sequence of these structures - BE ABLE TO

DRAW A FLOW CHART

TPVd i

t

I t

1 __ Cm-(r

patk GlJet

I

J

Clinical = JAUNDICE is caused by anything that prevents delivery of bile to intestine Tumor of the

head of the pancreas Stones etc Patient will have pale stools and yellowish colored mucus

membranes

Clinical- Any scenario that tells you the patient has BILLOUS VOMIT means that the obstruction to

the flow of digestive contents is after the Ampulla of Vater (Site of Entry of Billiary system to the

duodenum) - ie Duodenal Atresia

Spleen -located posterior to the mid axillary line between ribs 9 and 11 Make sure you know that

the 10th rib is the main axis of the spleen and this organ is susceptible to injury (stab wound errant

thoracoce ntesis etc)

The spleen is derived from mesodermal cells - NOT THE GUT TUBE

The spleen rests on the left colic flexure associates with the tail of the pancreas Know the

structures entering the Hilum of the spleen

Sh rt O~-t~ic 1 0(0 10 rtiltSPIric Iloa nt

(cut)

Peritoneum - similar concept to Pleura - think of a fist in a balloon

Visceral Peritoneum - Layer of balloon touching your fist

Parietal Peritoneum - Layer of balloon not touching your fist

Your fist represents the organ your wrist is the hilum and your arm contains the blood supply

entering the organ

Appreciate that there will never be organs in the peritoneal cavity - rather these organs invaginate

the cavity Kaplan videos

RULES OF NOMENCLATUREshy

1 Organ completely surrounded by peritoneum - peritoneal organ

2 Organ partially surrounded by peritoneum- Retroperitoneal

3 Peritoneum surrounding peritoneal organ is VISCERAL peritoneum

4 Peritoneum surrounding retroperitoneal organ is PARIETAL peritoneum

5 Peritoneum connecting visceral to parietal is called messentary 2 messentaries in the

gut Dorsal (to the gut tube) and ventral (to the gut tube) messentary

Aorta is in Retro peritoneal position - but blood must reach peritoneal position - vessels travel through

messentary All peritoneal organs will have blood supply reaching through messentary

-Mesentery is a 2 layer peritoneum with a neurovascular communication between body wall and organ

- Ligament connects one organ with another or to the abdominal wall (Ommentum = ligament)

lesser Ommentum (attach lesser curvature of stomach and duodenum to liver) =Hepatoduodenal

Ligament and Hepatogastric Ligament

Has a Superior and Inferior Recess (Accumulation of Fluid in Ascites)

Communicates with the greater sac through the epiplic foramen (what structures pass through

this foramen)

Boundaries - you must be able to visualize this

o Anterior - stomach

o Posterior - parietal peritoneum pancreas

o Superior - superior recess (bw diaphragm and coronary ligament)

o Inferior -Inferior recess (bw layers or greater momentum

Greater Ommentum (attach greater curvature of stomach) Gastrophrenic ligament Gastrosplenic

ligament gastrocolic ligament

The greater omentum is the largest peritoneal fold It consists of a double sheet of peritoneum folded on itself so that it is made up of four layers The two layers which descend from the greater curvature of the stomach and commencement of the duodenum pass in front of the small intestines sometimes as low down as the pelvis they then turn upon themselves and ascend again as far as the transverse colon where they separate and enclose that part of the intestine

ABDOMINAL PAIN

Parietal Peritoneum - supplied by same vasculature lymphatics and nerves supplying body wall it

lines and diaphragm Sensitive to pain pressure heat cold well localized

Visceral Peritoneum - supplied by same vasculature lymphatics and somatic nerve of organ it covers

Insensitive to touch heat cold and laceration - referred to dermatome of spinal ganglia providing

sensory fibers Where does appendicitis refer to

Foregut pain - epigastric area (ie - cholycystitis)

Midgut pain - periumbilical area (ie - appendicitis)

Hindgut Pain - suprapubic area (ie - diverticulitis)

Extra ImagesConcepts

ll~_____-

FalifCtrm ligament oind r~ud ligamet f Ilver

Blood from splenio gastriC and inferiof rne$e-rteri v~ins

Ca-I tributaries

Lett gastrio Ifein

Posterior superior pan~reatioodul)denal vaihS

Lott gamo-om~nlal (9aropip lomiddotic) -in

Poq_~ tjol imerl-9-r panCJertlcorllJod-nal veiopound --amp----I- - ~J Right grtr~-omntal

Anwrior interi (gartroepiploic) Jjn

pan euaii cod vl)denal veins middot Inf~Ji (t r mesentric vein

Miqdle (olic vein

Right cl)licvein Sigmoid and rectosigml)id (ei ns

IhH)Collc(~io

--- Mi~dl laquooLJl gtjrltgt

PoM ca vl1 illasto)moses -----shyampoptoageal 2 Paraumbilie-lt11 Inferi or Fectal vei ns

3 Recial 4 REuoperHonea1

Know how the Portal vein is formed I 4 sites of portal caval anastamoses and 1 clinical shunt

Col li t ltt-~ otTl~tI ~nj pc~ 1lt1 turJoG

Ltf 14i1 tImiddot~ artoftl9 on tj phtAt$

L-oftqf 4t t~r 1=laquoIran d 1 bull shy~p l ci rj o fOOOts

Nerves follow the arteries - appreciate the splanchnic nervous system I

Uet~ric branch of left ~nal art

Ureterie branch of righi renal artelY

Left Zld lumbar in and co mlTlunication to as)erdin9 lumbar l(~in Hi ~ht tEZ1~~t~ t3r j t itn ~ nJ l1t- rlnd lfe i r1

Inferior me5nteri~ artery

Notice that the right testicular vein drains directly into the IVC and the right testicular artery drains

directly into the aorta However the left testicular vein drains into the L renal vein at a right angleshy

reason left testicle is lower and more susceptible to varicocele (bag of worms)

Also notice that the left renal vein has a longer course because the IVC is on the right side whereas

the right renal artery has a longer course because the aorta is on the left side

Appreciate the anterior to posterior relationship of structures in the hilum of the kidney - VAP - Vein

Artery Renal Pelvis (Ureter)

11____ __ L_ L_ n VJ __ _ _ t_L I I_ _ L __ L_ I -pound1 bull LI_~-I ____

Posterior View of Head of Pancreas in ( of Duodenum

Celiao hunk

Co mmon ~L~jJth art~ry

GastNduQdonal artrf (partilly in phantn)

P1)Sterior $Up~Jior panCflaticuduodfmal art~r~t

(Co mm on) bile duct

middot~1t~~t-1l---~-~- Right gshomiddotomental (gastoe plp lolc) 3rte (phantomost)

Grener paocre atic art-ry

1n1~rjor pancr-iatlc artery

Jtrifll supejo r pal)oreailcento)dJodenal artr1 (phantom)

Anastomotlo branch

POostetlor bJanch of jo f~ri of pan-reatir(lduodensl drttnj

Anterio r branch of i flferior palcreati~)duodenal art~(phan1om)

Notice the extensive blood supply to the pancreas and duodenum via the branches of the celiac trunk

Notice collateral supply from SMA branches - makes sense bc this is the jxn of foregutmidgut

Identify the vessels in this arteriogram

Hiltid i)f N~ck oi B)dvof Tail 01 pa nereas pan cent~as P-nmiddot-reas panCtCas

I nferie v~na cava

jHept1iic p(lrlai v~in

Port1 tnd H~pti lt a ftH prol

Comm on) bll duct

Ouodtnum

~ft colic (sio)Atta~ hmtrlt jt~xJr-ofha~elSe

muo(IIQn

Right ~lIc (h~j)tic)

il~gtture

In1triol m~oten lIein (rttr op~ritoMdO

SlJp efl or mes~n~fiC amrV and lipln

KNOW YOUR NEIGHBORHOOD

Questions

vVhiJh structure supplied by a bnmdlof the cclia( artery is not derivcd from foregut LemCJUCrITI

(A) Head of the pancte-a5

CD) Pyloric duolenum

Cystkduct

( Liver hepatocyt~~

~F) Body of the spleen

An infant presents with an omrhaJucele at birth -hi oJ the [oHm illg applies to his cM1-dition

(A) It is 31so seen ill p4titnts with aganghonic megacolon

(11) ft reuirs from a fal1ure of resorption of theviteUine d let

(C) It results from herniation at the-site of regression of the right umbilk vein

DJ It is caustd by faihtrc of recanalization of the midgut part of the duodenum

~ It ill camioo by a failuIt vf the midgul to return to the abGQminal uity after herniashytion in-n the urnbilk s l stalk

Ot er than the spleen occlusion Cif the spit-Ilk artery at its odgin wm most likely affect die blood supply to jllch st cnud

(A) Jejunum

(B) Body of th pal1~lltas

(C) LeSStT Cllmiddotlaturc of tl )toma-ch

(D Duodenum dista to the entrance of the Ornmou bile duct

E Fundus of the stomach

A 38-yeu-old batL~er with a history of heartburn suddenly experiences excluciating pain in the (plgastric region of th~ abdomeu SurgCry is perf~rme immediard y upon admisshysion to the 1IlcrgCJliy tuomh~re i~ evidence uf a ruptured ulcer in the posterior waU of the stomach Vhere will a surgeon first fi nd the stomach contenlSf

A) Greater p4ritoneal sac

rB) Cul~de-s~c of Douglas (--

C Omental bursa ~

--D) Paracolic gutter

rEj Between -he panttal perimltum and the posterior body wal1

At birth an infant presents with a st()ma~ rb~tbas~njJled jfltotb~diaplfagru 1A1ltre is the defect thatresulied iiitJle heini~t()n shy~tsophagealbiatus

7 - rH-- Hiatus for the inferior vena cava

( Pleuroperitoneal membrane -(0) Septum transvcrsum

(E) Right Crlt~

An infant born with DOVv7l syndrome presents with bili()u~ vomiting Ahat congenital defect does the infant have

(A) Pyloric stenosis

(B) Meckel diverticulum C) Ornphaloce1e

(D) Gastroschisis

( ~ ) Duodenal atresia y A patient with cirrhosis of the liver presents with ~ bacalvaricestnlreased retrograde pressure in which veins caused the varices

(A) Paraumuilical

(B) Splenic

(ct AzygltJus

(15))G~trk ( (-F) Superior mesemeric

A htaltby 3-year~old male patient experiences a hernial sa protruding from the anterior abdominal wall about halfway between me anterior superior ilia spine and the pubk tuberde Pulsations of al1 artery are palpated medial to the protrusion site through the abdominal walL Which layer of the anterior abdominal wall will first be traversed by the

1hctma

fA) Rectus sheath (B) External oblique aponeurosis

(C) Inguinal ligament

lD) Transversalis fusda

(E) Cremasteric fa~cia

After 5urgi(aj ffpair of a hernia the patient tXperienccs mtmlgtness in the skin on the anteshyrior aspect of the S(Totum_ Vhaf nerve may have been lesioned during thehemiorrhaphy

(A) Femoral

(B) Obturator

(C) Ilioinguinal

(D) lliohypogastrk

(E) Pudendal

A 23~year-LJld female secretary il1 good health ~-uddcn1) doubles over with pain in the a ea of the 1JmbRicu$ Sbe feels vartn and ltneasy and has no appetite That night the pain seems to have mQved to the tower right abdominal regjol1 and she calls her family doctor who then arranges for an ambulance to pk-k her up and take her to the hospitaL Wh ell ntn~ perceived in the area of the urnbilirus most Hkely carried lhe pairfu I sensations into the eNS

tA) Vagus nerves I~

V B)

) Lessersplanchnk nerves

tC) Pudendal nerves

(D) lIiohpogastrk nerves

(E) Greater splam ic l erves

A CT reveals carcinoma in the bOod of the ancreas Vhich blood vessel trut ourses ----~- - -bull ------ --shy

immediately poftterior to the body ofthe pancreas is the m~t likely to be oompressed

(A) Splenk artery

(B) Abdominal aorta (C) Portal vein

(1) Splenic vein

(E) Renal vein

A patient has a penrln1l1ng uker of the posterior wall ot the br~l part ot the (lUooenmn llkh blood vessel is subject to erosion

(A) Common hepatic artery

(B) Gastroouodenal artery

(C) Proper hevatic artery

(D) Celiac artery

(E) Anterior inferior 11amrelltlcoduodcnal attery

Your patient has been diagnosed -ith a carcinoma locallted to the head and l~e(k of the pancreas Another clinical sign would be

A esophageal varices

(8) hemorrhoids

C) a caput medusa

(D) increased pra Teuro n th~ hepatic veins

(E) enlarged right supra lavkular lymph nodes

Wltkh of the foUowing structures develops in the ventral mesentery

(A) Spleen

(B) Jeiunum (C) Head of1ht pancreas (D) Transverse colon (E) Stomach

ti l Uw ~ littwin~ f( S-t lil oai Imdge ~ hi(h or tbt la~)d J truetur tgt liJ llntn nl) he hl p UC iJd [IIi ell

c o

A) drains Ie tht infCrior a La aI

R t middot~nfl0 ~ill to th~ lunlgtn of h i dtlndCrlllfH

(e) m t bull JiJattd on tl l J n T ~H

D ) sup Lc O VSlt I Hlid bhtu l 1 li - -I un oid

( ) U~tpli(t tr j middottUh~ 1 v(( b~nt rfK n1ilc~Zm

ANSWERS AND EXPLANATIONS

Answer E The spleen is t hlttnopodicand lymph organ demlted from mesoderm

Answ~ R Al1 tlmphalocele is caused by it failure of the nlidgut to return to the ahdomir nat cavity after herniation into the umbiliau Stalk Choices Aand D maybe seen in infants with Down syndrome choice D ~s the specific CBuse ofduudcnal JtiCSitt Choice C is (ile cause of gclstrosbisis and Choice B nsults iu a Meurolktldivertku1-tlB

Answer B The fundus ofthe stomach is suppHed by soort gastric brunches of the splenic altery The splenic artery supplies the body and tail of the pancreas part of the greater curvature of the sttmla(h and the spleen Te jejunum part of the head of the pancreas and tht~ duodenum distal to the entrance of the commOll bile duct are supplied by the superior mesenterk artery clll~l ~be less r ctlt1ature cmd the pylQric antrum are supplied by the right and lei gastric art(ries

AnSWftt C Tbeomental bursa or lesser ~ritoneaj sac lies direcdy posterior to the proxshyimal part of the duodeTtlm and the stomach and would be the first site where stomach contents ~Ott1d be fpoundluncL

Answer C A defect in a llleuropcritoneal membrane (uswlly the left) is the typical site of i1 cc-ngenitlI diilphragluatic hemia llere the membr4ne fails to dose ()pound( of the perishycCirdiopcritulleal canals

Answer E DuoJenal atresia and aganglionic megacoion are congwitaI defects S~Il in patients with Dowmiddotnsyndrome

Answer D RulaTgemt~llt of and retrograde flow in g~lstrk vel_ns in particlJl~r the kft gas~ tricveins dilates the capillary bed in rhe wall of the esophagus in (ases of porta yper~

tension Blood flow would increase in and dilampte tribntarkgts of the (lZygOUS vein on the other side of the capiUary bed but flow in this vein is in the typical direction t()ward the superior vena cava Paraumbiii(ltU vein eilgorgement contributes to a caput medusH Splenic ~nlargement might prc~nt with 5plcnonlegaly and balt-kflow in to tlu superior m~~ntclic vein occurs but is asymptomatic

Answer D The patient hagt an indirect inguinal hernia whi~h emerges from the antt-rior abdominal wall through the deep inguinltilling Theeep ring is a fault in the transv~rshysaUs fascia this I~yer wiIJ be penetrated first by the hernia

An~Wer C The ilioinguinal nenc which provides sens~llion to the lnedlal thigh ltmclanteshytior SClotunl pass~lt th rough the 5uperfh_ial inguinal ring ind $subject to inj i1T) becaus-e

it is in the operatitm Held of the erniorrhapny

Auswer B The leMHr splanchnic nerves are sympathdic nerVlts that carry viscera l sensashytlltgtrogt ftom illtllt1m~d ()J stietched gust (itinteitinal ~tructures (in this case the pprndix) into tnt eNS Lesser splanchnic ntTYcsarisc from thmiddot T9--T12 spinal cord segments lt1nd provide sympathetic innenation tD rnidgut siruc1ures whiCh include CLe app~JldD Viscera] Pain arising from affecLed Inidgut ampt 1C1ure is referred over the same dl- matorne~ of spinal segrnertts v-hich provide the sympathetic Innervation n this G1SC of appendicitis the invohen~n t of the ltire) of t e unlhHku indud s the T 10 dermatome

Answer B Of the five choices onty the dscending olon is retroperiton~al aldwould be a lik ~ ( choice to be seen immediately a(~jilcent to t11e posterior abdominal middotn~L

Amwen D The SpltftlC ~-ein ourses posterior to the body of the panneas m its way tt drain into the superior mCSfttltlri( vein

Answcr B TILt glstrodllolticnal artery 1 direct hIamh of the comrootl hepatic artery courses immediately pt))iwri() to the duodenum and is slbject to erosion

Answer B Carcinoma of th pan middott3S in the 1tilt1 may compreampgt the portltil vein at irs orishygill The poTtai vcin is fomled when the splenic vein jQiaswith tfie superior meStllt eric vein The inferiot mesenteric vein joins the ~plenjc vein just priOT to tlli~ point at which the splenic joins the superior Jlleit1ltcri( vein Increescd venous presslu in the inferior mesenteric vein is a cause of emo hoid~

Answer C The- velltral pancreas wilich forms most of the head of the p ~ncr as develops in the ventral mes(ntery as antutgrowth of the hepatic diverticulum Th~ hepatic divershyticulull induding the biIJary appa~atus develops in tbe ventral mesentery of the foregut

Answer~ A The superior mesenteric ~in joins with the spienkvein to form the hepatic portal vciu

Answer D The structure at gttlK is the proper hepatic artery~ whkh suppUesoxygenated b middotood to the liver

MAKE SURE YOU KNOW the diff bw Rectus Sheath above and below the arcuate line

ABOVE

Aponeurosis of xiiltmal obllque musclo

Extemll f)biquw musde

Reotln ilbdomlnls musole S~in

Internal 9bliquQ mY~QI

AponeUfOsi$ of hJH$V~~S Lir9a a lb lbdolTlin~ musolo Tri OJV6 rUi

atldomlnis mUS(loe

Sub cutanlilous tiue (tatty ye r)

BElOW

A POrl lJfosis 01 etemal oblique muscle

Aponeul~)sis 01 Internal oblique mU$cl~

Anteriol lay~ of r~ltdus st~ath EXttom1 oblique rnu$cll

Rectus Jbdominis muscle Intoernal Aponeurc-sis of tra~fersU$ oblique muscle-

at-domlnis muscentl ~ Skio

Tra nsvitSus abdomioLs ml)ZClt

TralSVersaHs fascia Medial umQil iegtt1 1i9Jment -and folj

Uldchus Peritoneum (ir median Umbilj~al Suboutane ous

Extraprftone 11ascia

Ymbilimiddot~1 fold)

preu9poundiea1 fascia

tissue (fatty 4nd m~mbr3n(iUS layers)

o Above the arcuate line (A horizontal line 13 of the distance bw the umbilicus and the

pubic symphysis) -10 Aponeurosis divides into an AntPost Laminae

o The Ant Laminae joins EO and Post Laminae joins Trans Abdominis = Ant and Post

RECTUS SHEATH respectively

o BElOW the arcuate line - all 3 aponeurosis join ANTERIOR to rectus muscle to meet its

counterpart in the midline (linea Alba)

o Take away Msg - The abdomen is devoid of a posterior rectus sheath below the

arcuate line and is therefore more vulnerable to herniasinjuries

Question - A physician makes a deep incision in the patients midline immediately superior to

the pubic symphysis which of the following layers is his knife least likely to pass

Rectus Abdominis External Oblique Ant Rectus Sheath Posterior Rectus Sheath All of the

Above

Answer - All of the above None of the other answer choices are midline structures -LINEA

ALBA

Linea Alba has very poor blood supply - doesnt heal well after surgery Therefore this is a

common site for incisional hernias

a Spleen b Transverse colon c Descending colon d Stomach e Pleura

17 Meckels diverticulum is normally found 2 feet proximal from the

a Pyloric sphincter b Lower esophageal sphincter c Ileo-cecal valve d Middle valve of Huston e Anal valve

18 Ulcer in the posterior wall of the first part of the duodenum would erode ___ artery and would cause bleeding

a Left gastric b Right gastric c Hepatic artery proper d Gastroduodenal artery e Middle colic artery

19 An inflamed appendix is identified by a surgeon on the operation table by noting

a The appendicies epiploicae b The convergence of tenia c The artery of Drummond d The mesocolon e The mesosalphinx

20 The nerve which emerges through the psoas major is

a Femoral b Ilio-inguinal c Ilio-hypogastric d Pudendal e Subcostal

21 The right gonadal vein drains into the

a Azygos b Hemiazygos c Inferior Vena Cava d Right renal vein e Left renal vein

22 The hepatocytes in the liver is derived from

a Ectoderm b Endoderm c Mesoderm

d Neural ectoderm

23 Abscess in the lumbar vertebrae due to tuberculosis would spread to the adjacent muscle which is

a Psoas Major b Iliacus c Quadratus lumborum d Tranversus Abdominis

24 The anterior wall of the inguinal canal is formed by

a External oblique and transverses abdominis b External oblique and fascia transversalis c Internal oblique and external oblique d Internal oblique and transverses abdominis e Fascia transversalis and peritoneum

Meckels diverticulum is a result of which of the following developmental abnormalities shy

A Failure of the vitelline duct to close

B Failure of the herniated intestinal loop to retract into the abdomen

C Failure of the urachus to close

D Failure of the midgut to rotate

E Failure of the hepatic duct to close

Explanation

Meckels diverticulum is a result of the persistence of the proximal part of the vitelline duct This

diverticulum is usually found about 2 feet proximal to the ileocecal junction and is usually about 2 inches

long It is present in about 2 of the popUlation It may be the site of ectopic pancreatic tissue or gastric

mucosa and may develop inflammatory processes and ulcerations Acute Meckels diverticulitis

simulates appendicitis

Which of the following veins carries blood from the esophagus to the portal vein The

A right gastric vein

B left gastric vein c splenic vein D azygos vein

E left gastroepiploic vein

Explanation

The left gastric vein a direct branch of the portal vein drains blood from the lesser curvature of the

stomach and the inferior portion of the esophagus Because branches of the portal vein do not have

valves blood can flow in a retrograde path when there is an obstruction to flow through the portal system or liveL Rlooci Cln then flow from the nortl] vein thr()1Ph the left PRstric vein to the esonhlPlIS lno

through venous communications within the submucosa of the esophagus to esophageal veins that drain

into the azygos vein The increase in blood flow through the esophageal submucosal veins results in esophageal varices

On the posterior wall of the abdomen the celiac ganglion A contains cell bodies of postganglionic parasympathetic neurons B is synapsed upon by neurons in the posterior vagal trunk C is synapsed upon by neurons in the greater splanchnic nerve D contains sensory cell bodies of lumbar spinal nerves E contains cell bodies of neurons that cause an increase in the rate of peristasis

Explanation The celiac ganglion is one of the preaortic ganglia of the sympathetic nervous system It contains cell bodies of postganglionic sympathetic neurons The sympathetic splanchnic nerves contain preganglionic sympathetic neurons that pass through the sympathetic chain without synapsing These splanchnic nerves go to the preaortic ganglia to synapse The greater splanchnic nerve contains preganglionic neurons from spinal cord segments T5-T9 This nerve synapses in the celiac ganglion The nerve fibers in the vagal trunks are preganglionic parasympathetic fibers that go to the walls of the organs that they will innervate and synapse on postganglionic parasympathetic neurons in the walls of those organs Cell bodies of sensory neurons in the abdomen are found in the dorsal root ganglia or the sensory ganglia of the vagus nerve Sympathetic innervation decreases the rate of peristalsis parasympathetic innervation increases the rate of peristalsis

Which of the following pairs of arteries will allow blood to bypass an occlusion of the celiac trunk

A Left gastric artery-right gastric artery

B Left gastroepiploic artery-right gastroepiploic artery

C Superior pancreaticoduodenal artery-inferior pancreaticoduodenal artery

D Splenic artery-common hepatic artery

E Left gastric artery - proper hepatic artery

Explanation The anastoOlosis of a branch of the celiac trunk and a branch of the superior mesenteric artery will

provide collateral circulation around an occlusion of the celiac trunk Each of the other choices pair

branches of the celiac trunk therefore these will not provide collateral flow around the obstruction of the

celiac trunk The left gastric splenic and common hepatic arteries are direct branches of the celiac trunk

The right gastric artery is a branch of the proper hepatic artery which is a branch of the common hepatic artery The left gastroepiploic artery is a branch of the splenic artery The right gastroepiploic artery is a

branch of the gastroduodenal artery whlch is a branch of the common hepatic artery

Which of the following organs has appendices epiploica The

A sigmoid colon

Bjejunum

C duodenum

D stomach E esophagus

Explanation Appendices epiploica are characteristic of the colon Appendices epiploica are subserosal accumulations

of fat None of the organs of the gastrointestinal tract has appendices epiploica except the colon

Page 9: Chirag's Abdomen Review

-----

PcmiddotrhtltllCf of the ) lcllille dud Of ~Olk sh31k ~br lontli n e( topic ~l cid-seefehllg gatric m 1(0]

andor pmcrcatic ii~~lle Lllost common c(lllgenital anomliy aftlle CT tract Cm CllHt hkfciing illtusm~(~pjjon Dlnllus or nbstrudion nelr the tcrrnin)l ileum Contraslwitll QIB~efic nmiddot = cvtk dilalaUon of ittllilC dJet

------~-

The ile 2$

2Jpound~11 11~ 2 feet frolll the iie( middoteCill vke l~ QfiJ~ at 1~

CIllIn nly prcsenfltgt III rll~2

llf~ of lifc- by ilwe 2 ty )ts of

epilheH8 19ls-trie- pal1elli(i

Hirschsplungs disease

Congenital tnt91(middotolon characterized by lack of Think of a gian ~pring that ganliml ttlLJcnJ(rk ~~gJeXllS~( (~lihs and lei ~sner plCxpstTlrlsgIllent on inbstinal biop y ()It to-iIure of U(middotural t~restpoundtU migration =

has s nl

Presents as ronic comtipnHoll tHly in life Dilllted pOltioll of the colon proximal 10 the aganiionic

segment resulting in a middot trmsjtillll ZQl1t rnvolt~

rectum [huany farlur( to P~$ meconium

High Yield WetDrylTheory List-

Suggestion - in your study group try to write a question for each of these points and then

exchange with a friend and try to answer each others questions

Abdomen Blood Supply - Reference viks picture posted on my google group - this is THE MOST HIGH

YIELD TOPIC IN ALL OF ANATOMY - expect 5 questions on your mini and 5-10 questions on your shelf

Make sure you can draw the blood supply and answer tertiary questions

Example - If the patient had an occlusion of the celiac trunk - which of the following areas would

experience ischemia

Portal hypertension - Know the 3 clinically relevant sites of portal caval anastamoses

Testes vs Scrotum lymph drainage

Where are paraumbillicai veins located

Omalophcele - failure of the gutto come back in (if in yolk sac - fatal)

Marginal Arteries

Superior Messenteric Artery is in front of 3d part of duodenum

Caput medusa (Distended paraumbillical veins secondary to portal hypertension)

Kidney Constrictors -1 Renal Pelvis 2 Crossing Pelvic Brim 3 Entering Urinary Bladder

Ureter - wet lab

Vagus is PIIJS supply up to 23 trans colon than pelvic splanch N up to the ass

Hirshsprungs disease - baby cant poop - dilated colon

Meckels Diverticulum - rule of 2s - 2 feet prox from ileocecal ju nc

Urachal Fistula - weeping belly

Gall stones - common bile duct

Jaundice relation to tumor of the head of the pancreas

Hepatopancreatic ampulla

Blood supply of renal gland - s superrenal art m s suprarenal a abd aorta inf suprarenal art

R Kidney - Tl2-L3 L Kidney - Tl1-L2

Renal Artery - L2

Epiploic foramen - know the borders and contents

Alantois diverticulum - urachus - medial umbilical

Lateral Medial and Median umbillical Folds (know the contents)

Directindirect hernia - know how to diagnose where they enter and exit the inguinal region and which

one is congenital

Anular pancreas - projectile vomiting

Pyloric Stenosis - projectile vomiting (non bilous)

Duodenal Atresia - projectile vomiting (bilous)

Dry Lab - know x rays

Vitteline Fistula - food out of umbilicus

Major duodenal papilla - junc of foregutmidgut

Arcuate line - relationship to rectus sheath

Mcburneys point -13 from ASIS bw umbilicus

Parietal pain - what is the nerve supply

Internal Oblique - cremasteric relationship

Know spermatic fasia

Processes Vaginalis - connection bw peritoneum and gubernaculums

Umbillicus - TlO dermatome

Deep Inguinal-l25 cm above mid inguinal ligament

Superficial Inguinal Ligament- superolateral to pubic symphysis

Variocele - veins engorged in scrotum (bag of worms)

bull

bull External spermatic fascia derived from external obliques EO II Cremasteric fascia ~ from internal obliques fO bull Internal spermatic fascia derived from fascia transversalis bull Tunica vaginalis derived from processes vaginalis directly rests on testes bull know order from testes out to skin

note reflex o ilioinguinal nerve o Efferent =genital branch of the genitofemoral nerve

--lt gt-- info important anastamoses which connects thorax to abdomen

bull Sup

o Sup epigastric branch of internal thoracic o Inf branch external iliac

Venous drainage o Above umbilicus aXillary v o Below umbilicus veins in triangle o At level of umbilicus Paraumbilical veins -gt drain into the portal V

II Important in Portal Caval Venous system Venous drainage of testes

o Clinical correlation Varicocele 11 vein drains into IVC 11 Left testicular vein ~ drains into left renal v

bag of rmlt

for lymph drainage T10 axillary lymph nodes

ill Below T10 superficial inguinal lymph nodes (lateral

Umbilical Folds

Lateral umbilical folds inferior vessels

Medial umbillcial folds umbilical (fetal remnant)

Median umbilical fold urachus (fetal remnant)

Between these folds fossas o Supervesical fossa between median and medial folds

11 bladder o hesselbachs between medial and I folds

II DIRECT HERNIAS HERE Borders

Medial semilunar line

Lateral info Epigastric

Inferior inguinallig o Lateral Inguinal Fossa beyond lateral fold

INDIRECT HERNIAS HERE II Deep inguinal ring (lateral to inferior epigastric a)

Indirect inguinal hernia o Lateral to inferior epigastric a o more common o When inserting finger in superficial inguinal ring will feel on tip of finger (since it goes

throueh ineuinal canall

----

Dry Lab - Label subcostal iliohypogastric Ll Ilioinguinal (Ll)

Horesshoe Kidney - stuck under IMA

Renal Agenesis -failure of the ureter bud to develop

Double Ureter

Unilateral Agenesis -1 kidney

Kidneys - Metanephros

Fetal kidneys are at sacral level

Look at 3rd part of duodenum

Some of this stuff is repeated I know just copied and pasted a bunch of stuff I had copy

Dermatomes

bull T4 nipples

bull no umbilicus v o Pain referred to no in appendicitis o Pain referred to T7ITS in gastritis ~

Inguinal ligament = external abdominal oblique aponeurosis

bull Inserts at anterior superior iliac spine to the pubic tubercle o Why important to know -7 visualizing this line allows us to properly diagnose a hernia

Below the inguinallig femoral hernia Above the inguinallig =inguinal hernia

Also to palpate the deep inguinal ring you go about 12Scm above the mid-inguinal

point

bull Modifications to ligament o Pectinate ligament o Lacunar ligament -7 cut this ligament to relieve strain i~ stran ul~tEd hernia

Inguinal canal

bull in males -7 transmits spermatic cord o important structures of spermatic cord ductus deferens testicular a genital branch of

the genitofemoral n pampiniform plexus of veins bull in females -7 transmits round ligament

Borders

bull Floor -7 inguinal ligament + lacunar ligament bull Anterior -7 aponeurosis of external oblique + internal oblique bull Roof -7 internal oblique and traverse abdominal bull Posterior -7 transverse abdominal + transversalis fascia

o Reinforced by conjoint tendon bull Aponeurosis of internal abdominal obliques and transverse abdominus bull Lies immediately behind the superficial inguinal ring in what would otherwise be

a weak point in the abdominal wall bull Innervated by ilioinguinal nerve (Ll) ~why important

bull In appendicitis Ll can be injured which will injure this nerve and in turn

the conjoint tendon With loss of innervation to this supportive structure the patient is now predisposed to a direct inguinal hernia

o Only hernia that can transverse the inguinal canal o Associated with congenital condition persistent tunica vaginalis

bull Direct inguinal hernia o Medial to inferior epigastric a o When inserting finger in superficial inguinal ring will feel on back of finger o Associated w old age or recent surgery

Muscles (Abdomen RECTUS SHEATH)

bull Arcuate line at level of ASISor 13rd distance between pubis and umbilicus bull Above arcuate line rectus abdominus is surrounded by a rectus sheath anteriorly and

posteriorly

o EO and 10 lie over rectus abdominus o 10 and TA lie behind rectus abdominus

bull Below arcuate line rectus abdominus has no rectus sheath posteriorly o EO 10 and TA lie over rectus abdominus o Transversalis fascia lies behind rectus abdominus o Inf EpIgastric vessels pierces the rectus sheath here

Peritoneum serous sac which encloses most of the abdominal structures

bull Ovary =only intraperitoneal organ o Oocyte ejected from ovary then captured by fallopian tubes o Why impt Women more prone to infection that can enter peritoneum

Peritoneum forms

bull Mesentery double layered fold of peritoneum formed as the organ was pulled in

bull Ligament between 2 organs in general bull Omentum between stomach and another organ bull Bare area area of no peritoneum bull

Viscera innervation

bull Pa rasympathetics 11 o Afferents sense hunger o Efferents l peristalsis relaxes sphincters gland secretion

bull Sympathetics o Efferents do opposite o Afferents CARRY PAIN SENSATION OF THE VISCERA (dull stretching pain)

bull PARASYMPATHETIC INNERVATION o Vagus nerve 7 _1l to 23rd unct ion of la rgej nte~tine oJ)elVrcspla~~~)~~ic~rYe~ IJiU- ~rd aJ~lpoteotiD~ IMPT

Gut Embryology

Gut ~ We say that the gut is derived from endoderm We often forget that when we say so we mean

that only the mucosa is derived from the endoderm The submucosa and the muscle layer is actually derived from the splanchnopleuric mesoderm and the serosa is derived from the visceral peritoneum

~ The main function of the gut is to digest the food which is done by the glands derived (and are) in the mucosa (endoderm) The only two exceptions in the Gut where glands though derived from the endoderm do not stay there but migrate down into the submucosa are esophagus and duodenum These glands however have their ducts opening to the swface of the mucosa

bull

~ Lungs liver amp gall bladder and pancreas are off-shoots from the foregut Esophagusshy~ The region of the tube from the laryngeal diverticulum to the beginning of the stomach elongates

to form the esophagus ~ The glands which form in the endoderm (mucosa) migrate down into the submucosa The path

whlch it took migrate becomes the duct of the glands which open to the mucosa ~ Achalasia Cardia - Failure of relaxation of the lower esophageal sphincter because of congenital

absence of ganglia at the sphincter (The ganglia when present releases VIP (Vaso-IntestinalshyPeptide) which relaxes the sphincter)

Mid-Gut Rotation ~ Because of the 90 degree rotation of the primitive stomach all of the following events occur ~ Lesser curvature comes to the right Therefore lesser omentum also comes to the right ~ Greater curvature comes to the left Therefore greater omentum also comes to the left ~ Right side vagal trunk becomes posterior vagal trunk ~ Left side vagal trunk becomes anterior vagal trunk ~ The left side peritoneal cavity comes to the anterior aspect of the stomach and will later be called

as the greater sac ~ The right side peritoneal cavity comes to the posterior aspect of the stomach and is (relatively a

small sac because the liver is on the right) called the lesser sacomental bursaepiploic sac ~ Epiploic foramen of Winslow (the lower free margin of the ventral mesentry) wiII be the

communication between the greater and lesser sac ~ The Liver moves to the right and therefore actually causes the 90 degree rotation of the stomach

The spleen comes to lie on the left side ~ Axis Antero-posterior axis around the superior mesenteric artery

bull Counterclockwise bull Approximately 270deg bull During herniation (about 90deg) bull During return (remaining 180deg)

Duodenum ~ Becomes retroperitoneal (except the first part which is still suspended by the hepato-duodenal

part of lesser omentum) ~ Glands (of Brunner) go submucosal ~ An imaginary line drawn below the opening of the major duodenal papilla represents the junction

between the foregut and midgut ~ Duodenal atresia in Downs syndrome Liver ~ 3rd week

bull liver bud grow bull into the septum bull transversum

~ 10th week bull hematopoietic bull function

bull 10 of the total bull body weight

~ 12th week bull bile is formed

Pancreas ~ In about 10 of cases the duct system fails to fuse and the original double system persists ~ 3rd month

bull pancreatic islets (Langerhans) ~ 5th month

bull Insulin secretion ~ Annular pancreas

bull The right portiCn of the ventralbud migrates along its normal route but the left migrates in the opposite direction

~ Complete obstruction of duodenum ~ Accessory pancreatic tissue Polyhydramnios (Amniotic fluidgt 1500-2000 ml)

~ Congenital defects including central nervous system disorders (eg Anencephaly) and gastrointestinal defects (atresias ego Duodenal esophageal) prevent the infant from swallowing the amniotic fluid (failure of recanalization)

Oligohydramnios (Amniotic fluid lt 400 mt) ~ Cl~ldberenal-agenesis

bull Midgut _-_

~ Primary Midgut intestinal loop gives rise to bull Distal duodenum bull Jejunum bull Ileum bull Ascending colon bull Transverse colon - proximal two-thirds of the bull Transverse colon with the distal third

~ Primary intestinaltoop bull ncephalic limb distal part of the duodenum the jejunum and part of the ileum bull ncaudal limb lower portion of the ileum the cecum the appendix the ascending colon and

the proximal two-thirds of the transverse colon bull 6th week

bull Rapid elongation of the cephalic limb bull Rapid growth of the liver bull Intestinal loops enter the extraembryonic cavity in the umbilical cord

bull 10th week bull loops begin to return bull regression of the mesonephric kidney reduced growth of the liver expansion of the

abdominal cavity bull Jejunum -left bull Loops - more to the right

bull Cecal bud -last part (temporarily below the right lobe of the liver) ~ qIDlthaloseJe (Structures COlHLoArts9V~1tion)

bull Through umbilical ring bull 6th to 10th weeks

bull Associated with a high rate of mortality (25) and severe malformations bull Associated with chromosome abnormalities

~ Gastroschisis (Structures coming out are not covered by Amnion) bull herniation through the body wall ----=---=-shybull Into the amniotic cavity bull Lateral right of the umbilicus bull Sometimes the inferior wall fails to develop as a result lower abdominal structures like the

bladder would be exposed to the exterior not associated with chromosome abnormalities ~ Abnormalities of the Mesenteries

bull Mobile cecum persistence of mesocolon bull Extreme form - long mesentery bull Volvulus

~ Distal third of the transverse colon ~ Descending colon ~ Sigmoid colon ~ Rectum ~ Upper part of the anal canal ~ Primitive anorectal canal

bull 7th week cloacal membrane ruptures bull Tip of the urorectal septum perineal body bull Pectinate line

~ Hindgut anamolies bull Rectoanal atresias and fistulas bull Imperforate anus bull Congenital megacolon (aganglionic megacolon Hirschsprung disease)

bull

bull Hindgut

Chirags Abdomen Review - Part 2

Understanding Embryo makes learning blood supply EASY

I I

I t

~ -

)

Table l1r-~ L Adult SUmiddotuctu~SDrj~l Froln Each of he Three Dhisions of be Pringttive GUl Tube t-middot-----middotmiddotmiddotmiddot-

Foregu(

I_ (Celiac Trunk)

Ir-slt-gtphgus

S101na(b

I h -= LiJ~r

Pancre=l S

bull 1 i Biliary apparntu5

Gall bladdshy

i Pha11~Cal pltgtuchcs

LullSS-I

Mjig ---- bull __ _- ----n--duct----~---middot-------l--n

(Superior Jldesen1eric Artery)_-1I-(I_~__ middot __ O-=-)_in_middoto_r_M_e_se_n_t_e_r_i_c_An__

Uuodenu rn 2nd_ 3 lt141h V4Tt

Jejunun-~

nc-un]

tCCUJ11

AppltgtndLX

Transver5e -o1on (p~oxiln1l1 ~O Tbird)

bull__hytgtid~ _ _ ______ L _ __

Tr-dn~llt~se colon (diStul h lTd) I

)

i

Aa ca-nal -( uppeT patt) i

I I

_____ __ _ _ _ ___ - - rhe~ a(t clcriVOkt iV(5 opound~lt prbn1rC ~ nlQC blft TlI)( 134tof r~ tIonoinf~ i 1 ~l l1rd c- P Cle

Now Lets see how much youve learned

Questions

1) A pt receives a general anesthetic in preparation for a c~t~~my A right subcostal incision is made which begins near the xyphoid process runs along and immediately beneath the costal margin to an anterior axillary line and transects the rectus abdominus muscle and rectus sheath At the level of the transpyloric plane the anterior wall of the

-~~-~=--- _eco---shysheath of the rectus abdominus muscle receives contributions from which of the following

a Aponeuroses of the in~ande~tef-Ilal o~ues

b Aponeuroses of the transversus abdominis and internal oblique muscles c Aponeuroses of the transversus abdominis and internal and external oblique

muscles d Transversalis fascia e Transversalis fascia and aponeurosis of the transversus abdominus muscle

A

2) The lat~raJJJ11QjJt~gLfgJlLoneach side of the inner surface of the anterior abdominal wall is created by which of the following structures

K Falx inguinalis (~) Inferior epigastric a

c Lateral border of the rectus sheath d Obliterated umbilical a e Urachus

B

3) A man the victim of several knife wounds to the abdomen during a brawl at the Lobster Shack subsequently developed a direct inguinal hernY Damage to which of the following nerves is most likely responsible for the predisposing weakness of the abdominal wall

~ Genitofemoral nerve ( b) Ilioinguinal nerve ~-t Tenth intercostal nerve

d Subcostal nerve e Pelvic splanchnic nerve

B

4) Which of the following statements concerning a direct inguinal hernia is correct a It is the most common type of abdominal hernia b It transverses the entire length of the inguinal canal c It contains all3 fascia layers of the spermatic cord d It exits the inguinal canal via the superficial ingeJinal ring e It protrudes through H~acb strJg e

~(

1fltbS w E

tl

5) The conjoint tendon is

a Important in preventing indirect inguinal hernias b The fused aponeurotic layers of internal abdominal oblique and transversus

abdominus muscles c Posterior to the deep inguinal ring

d Medial fibers of the inguinal ligament

B

6) A 25 year old male is brought in to the ER after being involved in a car accident in which he received a crushed internal injury in his abdomen Examination reveals a lesion of parasympathetic fibers in the vagJsnerve which interferes with glandular secretory or

smooth muscle functions in which of the foliowingorgans a Bladder b Transverse coloiW c Descending colOO d Prostrate gland e Rectum

B

7) The spermatic cord includes all of the following contents except a Il ioinguinal nerve b Pampin iform plexus of veins c Vas deferens d Genitofemoral nerve

A

8 Which abdominal structure gives rise to the internal spermatic fascia (muscle) following the descent of testes in development

a External abdominal oblique aponeurosis b Transversalis fascia c Transversus abdominis muscle d Peritoneum e Internal abdominal oblique

B

9 Which abdominal structure gives rise to the tunica vaginalis fotlowing the descent of testes during development shy

a External abdominal oblique aponeurosis b Transversalis fascia c Transversus abdominis muscle d Peritoneum e Internal abdominal oblique

D

10) The lesser omentum is a peritoneal fold which is su bdivided into the a Hepatogastric and gastrosplenic ligaments b Hepatoduodenal and gastroomentalligaments c Hepatoduodenal and gastrosplenic ligaments d Hepatogastric and hepatoduoden9-jrj igaments

D

11) A posteriorly perforating ulcer in the pyloric antrum of the stomach is most likely to produce initiallocalized peritonitis or abcess formation in which ofthS fQllowing

a Great-sac - -- -

b Paracolic recess

c Omental bursa

d Right subphrenic space

c

The inferior mesenteric artery arises from the abdominal aorta ilm_ediill~y_J-Qs1eriQLto which of the foowing org~ns A-F~t~filie duodenum B Head of the pan~eis C Neck of the pandeas

D Second part of the duodenum

E Third part of the duooenum_shylaquoshy

shy

The correct answer is E The inferior mesenteric artery arises from the anterior surface of the aorta at the level of the third lumbar vertebra The third part of the duodenum crosses the midline at the level of the third lumbar vertebra and passes anterior to the aorta at the origin of the inferior mesenteric artery The

first part of the duodenum (choice A) lies horizontally to the right of the midline at the level of the first

lumbar vertebra The head of the pancreas (choice B) is to the right of the midline and extends from the

level of the first lumbar vertebra to the third lumbar vertebra It lies within the concavity of the

duodenum The neck of the pancreas (choice C) lies in the midline at the level of the first lumbar

vertebra It lies on the anterior surface of the aorta at the origin of the superior mesenteric artery The second part of the duodenum (choice D) lies vertically to the right of the midline and extends from the

level of the first lumbar vertebra to the level of the third lumbar vertebra

The left adrenaLvein drains directly into which of the following veins A Hemiazygos vein

B Inferior vena cavaee C Left renal veiri -

D Splenic vein

E Superior mesenteric vein

a

The correct answer is C The left adrenal vein and the left gonadal vein (either testicular or ovarian) drain into the left renal vein TheTeft renal vein t~ains intothe- inferior vena cava In contrast the right

adrenal ~~inandnght gonadal veindrai~ gLr~ctJy iQtoJhe iilferiQ[ Vencava -- -

ThehemTazygoS7ein- (~h-~i-~ A)~~c~i~es the venous drainage from the body wall on the left side of the

thorax and abdomen No visceral organs drain directly to the azygos or hemiazygos veins The inferior vena cava (choice B) receives the direct venous drainage from the right adrenal vein but not

the left adrenal vein Remember the inferior vena cava is on the right side of the abdomen The splenic

vein (choice D) receives the venous drainage from the spleen and part of the pancreas and stomach The splenic vein is part of the portal venous system

The superior mesenteric vein (choice E) receives venous drainage from much of the intestinal tract It is part of the portal venous system and joins with the splenic vein to form the portal vein

A 43-year-old man presents complaining of pain in the groin On examination his physician palpates a

bulge in the region of the superficial inguinal ring which he diagnoses as a direct inguinal hernia The hernial sac most likely

A is covered by all three layers of the spennatic fascia B passes medial to the inferior epi gastric artery

C passes medial to the lateral border of the rectus abdominis muscle

D passes posterior to the inguinal ligament E passes through the deep inguinal ring

The correct answer is B Direct inguinal hernias enter the inguinal canal by tearing through the posterior

wall of that structure The typical location for this type of hernia is through the inguinal triangle bounded

laterally by the inferior epigastric artery medially by the lateral border of the rectus abdominis and

inferiorly by the inguinal ligament Direct inguinal hernias pass medial to the inferior epigastric artery

whereas indirect inguinal hernias pass lateral to the inferior epigastric artery because the deep inguinal

ring is lateral to the artery Indirect inguinal hernias are covered by all three layers of the spermatic fascia (choice A) Direct inguinal hernias are covered by fewer than all three layers because the direct inguinal

hernia tears through one or more layers of fascia as it emerges though the abdominal wall The lateral

border of the rectus abdominis muscle (choice C) forms the medial border of the inguinal triangle All

inguinal hernias pass lateral to the rectus abdominis Femoral hernias pass posterior to the inguinal ligament (choice D) Inguinal hernias emerge through the superficial inguinal ring which is superior to the inguinal ligament Inguinal hernias that descend below the inguinal ligament pass anterior to the

ligament Indirect inguinal hernias pass through the deep inguinal ring (choice H) direct inguinal hernias

do not Both types of inguinal hernias pass through the superficial inguinal ring

During a gastric resection in a patient with stomach cancer a surgeon wants to remove the lesser

omentum because of tumor extension into it Which of the following structures lie in the free edge of the

l~~g omentum and consequently must be dissected out in order to be preserved

A Common bile duct cystic duct and hepatic artery 6

B Cystic duct hepatic artery and hepatic vein

e Hepatic vein and cystic duct

Portal vein common bile duct and hepatic artery

E Portal vein hepatic artery and hepatic vein

The correct answer is D The free edge of the lesser omentum contains three important structures the

common bile duct the hepatic artery and the portal vein Nei ther the cystic duct (choices A B and C) nor the hepatic vein (choices B C and E) lies in the free

edge of the lesser omentum

A 55-year-old male patient with chronic liver disease has portal hypertension To relieve the pressure in the portal system a porto-caval shunt is performed Which of the following veins may by anastomosed to

accomplish this porto-caval shunt A Left renal vein-left testicular veingt

B Right renal vein-right suprarenal vein I shy

e Splenic vein -left renal vein J

D Superior mesenteric vein-inferior mesenteric vein E Superior mesenteric vein-splenic vein

The correct answer is C The splenic vein drains directly into the portal vein The left renal vein drains

directly into the inferior vena cava Anastomosis of these veins would allow blood from the portal vein to

drain retrograde though the splenic vein into the renal vein and then into the inferior vena cava The left

renal vein (choice A) drains directly into the inferior vena cava The left testicular vein drains directly into

the left renal vein Thus these veins are already in communication and neither vein is part of the portal venous system The right renal vein (choice B) drains directly into the inferior vena cava The right

suprarenal vein also drains directly into the inferior vena cava Thus neither vein is part of the portal

venous system The superior mesenteric vein (choice D) drains directly into the portal vein The inferior

mesenteric vein drains into the splenic vein which then drains into the portal vein Thus neither vein is

part of the caval venous system The superior mesenteric vein (choice E) drains directly into the portal

vein The splenic vein also drains directly into the portal vein Thus neither vein is part of the caval

venous system

A 12 year old boy has fever vomiting and para-umbilical pain After examining the patient the doctor

makes an initial diagnosis of appendicitis Appendicular pain which is initially referred to the umbilicus goes to the dorsal root ganglion of

a TI b TI2 c L1 d T7

(e I TIO

A 59-year-old male undergoes a neurological examination which reveals that when the abdominal wall is

stroked the muscles of the abdominal wall of the side of the body stimulated failed to contract Other

neurological tests appeared normal The likely region affected includes

a CI - C5 spinal segments b C6 - TI c T2-TI ~T8-T12

e Ll- L5

The surgery done to relive portal hypertension is done by connecting two veins Which of the following veins would be suitable for connection

a Inferior vena cava and portal vein b Superior vena cava and portal vein c Splenic vein and right renal vein d Splenic vein and left renal vein e Superior mesenteric vein and Inferior vena cava

A mother brings her 3-week-old infant to the pediatric clinic reporting a new scrotal bulge that she found -~-

while changing a diaper yesterday The infant is afebrile Physical examination reveals a palpable mass in

the scrotum while in the standing position resolution of the mass in the supine position and no

transillumination of the scrotal sac What is the most likely diagnOSiS

a Cryptorchidism b Direct inguinal hernia c Hydrocele d Indirect inguinal hernia ~ e varicocele

The Vagal trunks enter the abdomen by passing through which of the following openings in the

diaphragm

a Right crus b Esophageal hiatus ~ c Vena caval hiatus d Aortic hiatus e Left crus

2 The anterior boundary of the epiploic foramen of Winslow is bounded by

a) First part of duodenum b) Lesser curvature of stomach c) Liver d) Hepato-duodenalligament v ~

3 The ilio-inguinal nerve is derived from

a TI2 ry b LI c L2 d L3 e L23

15 Surgically the structure used to suspend the kidney to the diaphragm is

a) Renal fascia b) True capsule c) Perinephric fat d) Paranephric fat

6 If there is portal obstruction because of carcinoma affecting the pancreas which of these of the

following signs would be present

a Caput medusae b Esophageal varices c Rectal varices c

d Pulmonary edema

7 In a sliding hernia the gastro-esophageal junction lies

a) At its normal position b) Below the normal position c) Above the normal position V d) None of the above

8 Which of the following structures is retroperi toneal

A transverse colon B spleen IJ2f6 C ileum D descending colon v r 1pound1111111

9 The renal angle is fonned lgtetween the 12th rib and ______ muscle

a Psoas major -middotshyb Erector spinae c Quadratus Iumborum d Diaphragm

10 The anterior structure at the hilum of the kidney is

a) Renal vein ~

b) Renal artery I middot~ I

c) Ureter d) Accessory renal artery

11 Because of origin of the muscle from the lateral one third of the inguinal ligament it

could not fonn the anterior wall of the inguinal ligament

a) External oblique b) Internal oblique c) Transversus abdominis_ d) Rectus abdominis

12 A large tumor mass impinges on the splenic artery and its branches as the artery pass out from below

the greater curvature of the stomach Branches o(which of the following arteries would most likely to

effected by the pressure on the splenic artery

a Left gastric b Left gastro-epipJoic c Right gastric d Right gastro-epipoloic e Short gastric_

13 A new born baby has projectile vomiting after each feeding It is determined that there is obstruction

of the digestive tract as a result of annular pancreas Annular pancreas is as a result of an abnormality in which of the following process

a Rotation of the dorsal pancreatic bud around the first part of duodenum b Rotation of the dorsal pancreatic bud around the second part of duodenum c Rotation of the dorsal pancreatic bud around the third part of duodenum d Rotation of the ventral pancreatic bud around the first part of duodenum y Rotation of the ventral pancreatic bud around the second part of duodenum

14 As the liver bud enters the ventral mesogastrium the region of the mesogastrium stretching from the

liver to the anterior abdominal wall is called

a Lesser Omentum b Greater Omentum ~ Falcifrom ligament d Lacunar ligament e Ligamentum teres of liver

16 A patient has absence of his 12th rib In such a patient if the doctor makes an incision to approach his

kidney mistaking the 11 th rib for the 12t he would end up injuring

Which of the following arteries is a direct branch of the gastroduodenal artery The

A right gastric artery

B left gastric artery

C inferior pancreaticoduodenal artery D left gastroepiploic artery

i E)right gastroepiploic artery --

E x pI a nation The right gastric artery is typically a branch of the proper hepatic artery The left gastric artery is a direct

branch of the celiac trunk The right and left gastric arteries anastomose along the lesser curvature of the

stomach The inferior pancreaticoduodenal artery is a branch of the superior mesenteric artery it

anastomoses with the superior pancreaticoduodenal in the head of the pancreas The left gastroepiploic

artery is a branch of the splenic artery it anastomoses with the right gastroepiploic artery along the greater

curvature of the stomach The right gastroepiploic artery is a branch of the gastroduodenal artery The

other branch of the gastroduodenal artery is the superior pancreaticoduodenal artery

Which of the following pairs of veins join together to form the portal vein The

A superior mesenteric vein and inferior mesenteric vein

B inferior mesenteric vein and splenic vein

C superior mesenteric vein and splenic vein

Ip)splenic vein and left gastric vein E superior mesenteric vein and left gastric vein

Explanation

The portal vein is formed behind the neck of the pancreas by the union of the superior mesenteric vein

and the splenic vein The inferior mesenteric vein drains into the splenic vein The left gastric vein drains

directly into the portal vein After the portal vein forms it enters the hepatoduodenalligament of the

lesser omentum to reach the liver The portal vein is the most posterior structure in the hepatoduodenal

ligament

At which of the following vertebral levels does the duodenum pass anterior to the aorta - _- shy

All ~

B L2 7~

CL3 I

~DL4

E L5

Explanation

The duodenum begins at the pyloric sphincter at the level of Ll The second (or descending) portion of

the duodenum is to the right of the aorta and extends inferiorly from the level of Ll to the level of L3 The third part of the duodenum crosses the aorta from the right side to the left side at the level of L3 The

fourth (ascending) portion of the duodenum extends from the level of LJ to the level of L2 The

duodenum ends at the duodenojejunal flexure The superior mesenteric artery passes anterior to the

duodenum as the duodenum passes anterior to the aorta The duodenum can be constricted at this level

In which of the following locations will perforation of the digestive tract result in the spilling of luminal

contents into the - lesser peritoneal sac

A Anterior wall of the second portion of the duodenum B Posterior wall of the second portion of the duodenum

C Anterior wall of the stomach

~Posterior wall of the stomach E Posterior wall of the transverse colon

Explanation

The posterior wall of the stomach is related to the lesser peritoneal sac The anterior wall of the stomach is related to the greater peritoneal sac The anterior wall of the second portion of the duodenum is related to the greater peritoneal sac The posterior wall of the second portion of the duodenum is related to the retroperitoneal space The posterior wall of the transverse colon is related to the greater peritoneal sac

The ureter lies against the anterior surface of which of the following muscles shyA Crus oftne diaphragm B Quadratus lumborum

0 Psoas major D Transversus abdominis

E Iliacus

Explanation The ureter exits the renal pelvis at about the level of vertebra L2 As it descends along the posterior abdominal wall it lies on the anterior surface of the psoas major The psoas major muscle arises from the bodies of the lower lumbar vertebrae The psoas major muscle is joined by the iliacus to fonn the

iliopsoas muscle The iliopsoas muscle then attaches to the lesser trochanter of the femur and is the major

flexor of the hip

As the right ureter passes the pelvic brim it lies against the anterior surface of which of the following

blood vessels

A Gonadal artery B Inferiorvena cava C Internal iliac artery

rJ- External Iliac artery

E Inferior mesenteric artery

Explanation

The ureter lies in the extraperitoneal space in the posterior abdominal wall Alter leaving the kidney it

passes inferiorly on the anterior surface of the psoas major muscle At the pelvic brim the ureter passes

into the pelvis At this point the common iliac artery is dividing into the external and iliac arteries The

ureter lies on the anterior surface of the external iliac artery immediately distal to the bifurcation This is a useful landmark for a surgeon to locate the ureter

When extravasated urine passes from the superficial perineal space into the anterior abdominal wall it is

found immediately deep to which of the following layers of the anterior abdominal wall

-ltScarpas fascia

B External oblique muscle

C Internal oblique muscle D Transversus abdominis muscle

E Transversalis fascia

Explanation

The superficial perineal space is bound by Colles fascia the fibrous portion of the superficial fascia This

layer of fascia is continuous with Scarpas fascia the fibrous portion of the superficial fascia of the anterior abdominal wall Therefore urine that is deep to Colles fascia will remain deep to Scarpa s fascia The urine will spread in the plane between Scarpas fascia and the external oblique layer

When a horseshoe kidney develops the ascent of the kidney is restricted by the A internal iliac artery B external Iliac artery

C common iliac artery

inferior mesenteric artery

E superior mesenteric artery

Explanation

A horseshoe kidney develops when the inferior poles of the to kidneys fuse together as they ascend into

the abdomen from the pelvis The first anterior midline vessel that is encountered by the horseshoe kidney

is the inferior mesenteric artery This artery prevents the kidney from continuing its ascent

The left testicular vein drains into which of the following veins

A Left internal iliac vein B Left common iliac vein

bflnferior vena cava D Left renal vein I

E Left internal pudendal vein

Explanation

The left testicular vein drains into the left renal vein The right testicular ~i~[~nsltjectlY into the

inferior vena cava This difference in venous drainage is believed to explain the greater incidence of

varicocele on the left side than on the right The venous drainage from the penis is to the internal vein

which then drains into the internal Iliac vein

The spinal nerve that provides cutaneous branches to the skin around the umbilicus is

A TS B TW-shy

C TI2

DL2 EtA

Explanation

The tenth intercostal nerve is the anterior ramus of the TIO spinal nerve After passing through the tenth

intercostal space the nerve continues forward in the anterolateral abdominal wall in the plane between

the internal oblique muscle and the transversus abdominis muscle In the abdominal wall the nerve innervates to the abdominal wall muscles as well as the skin and the parietal peritoneum The umbilicus is

a useful landmark for the region of distribution of the tenth thoracic nerve

The ligament of the vertebral column that resists its extension is the Aligamentum flavum

B supraspinous ligament

C posterior longitudinal ligament

D anterior longitudinal ligament

E interspinous ligament

Explanation

The ligaments of the vertebral column that resist flexion of the column include the supraspinous ligament

interspinous ligament ligamentum fiavum and posterior longitudinal ligament The ligament that resists

extension is the anterior longitudinal ligament This longitudinal ligament is very broad and strong It

covers the anterior and anterolateral surfaces of the vertebral bodies and the intervertebral disks In

addition to resisting extension the anterior longitudinal ligament provides reinforcement to the anterior

and anterolateral surfaces of the intervertebral disk The posterior longitudinal ligament is relatively

narrow and covers the posterior surface of the vertebral bodies and the intervertebral disks This ligament

reinforces the posterior surface of the disk The posterolateral surface of the disk is not reinforced and it

is through this region that herniation of the nucleus pulposus usually occurs

A patient presents with epigastric and right upper quadrant pain The pain is most intense 2-4 hours after

eating and is reduced by the ingestion of antacids The patient states that he has passed black tarry stools

(melena) within the last week Fiberoptic endoscopy reveals a yellowish crater surrounded by a rim of

erythema that is 3 cm distal to the pylorus Accordingly an ulcer has been identified in the patients

A fundus

B antrum

C duodenum

D jejunum

E ileum

A number of physiologic genetic and other factors increase the risk of gastric (and duodenal) peptic

ulcers The evidence that H pylori plays a principle role is compelling Smoking and caffeine are known to adversely affect the morbidity mortality and healing rates of peptic ulcers In general first-degree

relatives of peptic ulcer patients as well as males have a threefold to fourfold increased risk of developing this disorder Paradoxically in gastric ulcer disease acid secretion is not elevated It is possible that

excess secreted hydrogen ion is reabsorbed across the injured gastric mucosa In general a defect in gastric mucosal defense is the more important local physiologic

A patient presents with symptoms of duodenal obstruction caused by an annular pancreas Annular pancreas is caused by

A rotation of the dorsal pancreatic bud into the ventral mesentery B rotation of the ventral pancreatic bud into the dorsal mesentery

fJ failure of the major and minor pancreatic ducts to fuse ~ ~ cleavage of the ventral pancreatic bud and rotation of the two portions in opposite directions around -the duodenum E formation of one pancreatic bud instead of two

Explanation Normally the ventral pancreatic bud rotates around the gut tube to reach the dorsal pancreatic bud The two buds fuse to form a single pancreas and the distal portions of the two ducts fuse The ventral pancreatic bud forms the inferior portion of the head of the pancreas the uncinate process and the major pancreatic duct (of Wirsung) The dorsal pancreatic bud forms the superior part of the head the neck body and tail and the minor pancreatic duct (of Santorini) Annular pancreas is the result of the ventral pancreatic bud dividing into two portions before it rotates into the dorsal mesentery Each portion rotates in opposite directions to get to the dorsal mesentery thus encircling the duodenum The presence of annular pancreas can constrict the duodenum thus obstructing its lumen

In n _ phranlc----

Gon ~l ----_1 Lum bltano

~~--- CornmQ1t bull ac

+-~4--- lnlllirnaJ ilic

xtem iliac

OBJECTIVE - Identify the blood supply to each of the structures listed in the table on the previous page

Ill give you a head start

FOREGUT - Supplied bV Celiac Tru nk (T12)

Proper hepatic

GastiooUod 13Jafter

1nferlor pancreaticoduodenal artery

Common epatlc

Lett gas ric iiirtery

Spfen artery

shy Gastroepiphgtic artery

~ Superior mesenteric 8rtfry

~

1 Esophagus is a derivative of the foregut so its blood supply originates from the celiac trunk

(T12) The predominant blood supply to abdominal portion of the esophagus is the Esophageal

A (Branch of L Gastric) The venous drainage of the esophagus is particularly important because

it is 1 of 3 clinically relevant sites of Portal Caval anastamoses The Portal Esophageal Vein

meets the Caval Azygos System Persistent bleeding manifests as Esophageal Varices - a fata I

condition

2 The Stomach is also a derivative of the foregut has EXTENSIVE blood supply and is very high

yield on anatomy exams The lesser curvature is supplied superiorly by the L Gastric A (1 of 3

major branches ofthe Celiac trunk) and inferiorly by the R Gastric A ( a branch ofthe proper

Hepatic A) The greater curvature is supplied superiorly by the L Gastroepiploic A (a major

branch of the splenic A) and inferiorly by the R Gastroepiploic A

The Short Gastric arteries (branches of Splenic Artery) supply the fundus of the stomach and

are referred to as EIID ARTERIES because they have no collateral blood supply Therefore if the

splenic artery were occluded (ex - increased pressure in the ommental bursa) - there would be

ischemia to the fundus of the stomach Venous drainage of the stomach is extensive via various

veins lead ing to the portal system Posterior to the stomach the IMV joins the splenic V which

joins the SMV to form the PORTAL VEIN ADAMS

3 Duodenum blood supply has high clinical relevance because it is the junction of the foregut and

midgut and therefore is the site of anastamoses between branches ofthe Celiac Trunk (main

foregut artery) and the Superior Messenteric Artery (main midgut artery) The Proper hepatic

artery gives off the gastroduodenal artery which travels behind the 1st part of the duodenum

This point has high clin ical relevance because duodenal ulcers are very common and a posterior

rupture of the 1st part of the duodenum could rupture the gastroduodenal artery causing

traumatic abdominal bleeding The Gastroduodenal artery first gives off the R Gastroepiploic A

(mentioned above) and proceeds as the Superior pancreatico duodenal artery (supplies the

pancreas and duodenum) which anastamoses with the inferior pancreatico duodenal A (branch

of the SMA) This is the junction of foregut and midgut and occurs near the opening of the

bil iary system into the duodenum (ampula of vater) Portal venous drainage here is responsible

for delivering nutrients from digestion to the liver for metabolism Appreciate that the Superior

mesenteric artery (artery of the midgut) branches from the aorta at Ll travels posterior to the

pancreas than moves anteriorly (at the jxn of the pancreatic headbody) and comes over the

3rd4th part of the duodenum Tumor of the head of the pancreas can compress the SMA

4 Jiver blood supply is via the common hepatic artery (major branch of the cel iac trunk) The

common hepatiC becomes the proper hepatic gives off the R gastric A and the Gastroduodenal

A and then joins the common bile duct and the portal vein in the portal triad Clinical- if a

patient were bleeding from the hepatic A a surgeon can stick his fingers in the epiplOic foramen

and squeeze the free edge of the hepatoduodenalligament in order to stop bleeding to the

area Please note that the hepatic a branches into Rand L hepatic A The Right hepatic artery

gives off the cystic artery which supplies the gallbladder Afferent venous supply is via the

Portal vein which is bringing nutrient rich blood to the liver After metabolism takes place

venous blood leaves the liver through the hepatic veins into the IVC PLEASE UNDERSTAND THE

RELATIONSHIP OF THESE STRUCTURES - ADAMSNETIERSNH Etc

5 Pancreas - Head is supplied via the superior and inferior pancreaticoduodenal arteries

(mentioned above) The tail (situated towards the hilum of the spleen) is supplied via the

pancreatic branches of the splenic artery (END ARTERIES) This blood supply is very important

because the endocrine Alpha and Beta Cells from the pancreatic islets of lagerhans are located

towards the tail This is where Insulin and Glucagon is released to the blood

Now complete this for mid and hindgut structures Make sure to note clinically relevant arterial

anastomoses as well as portal caval anastomoses FYI Appendix blood supply SMA + IMA

anastamoses marginal artery Portalcaval rectal veins fhemmorhoids) and periumbilical caput

medusa are high yield THE BUTT THE GUT and THE CAPUT

Abdominal Development

Liver

Ij1f

II wall b

oh liN ~ VltJrti n be- bull

Pancreas

Secondary Retroperitonealization e I~tl r 1 a v-mtrai m ellter

Rotations of the Gut I i Ij (lIl1UtIJ f~ l r tilt

()l td 10 me l-ft and he v

--~--- -~ -~-~

i

I AolaijonjoI~guf I

STOMACH BED (IDENTIFY IN ADAMS)- the structures posterior to the ommental bursa which

support the stomach in the supine position

Abdomnal JQrUI

Splnic vein

OmQ-oul tv~ ) O(s(Jroa)

Lojt(r o m nturrt (hpJtodu o d~n31 Hid

Gadrl)SplerH (g3stroll~nal) IIgam~nt

hiad h~~atogrtricent IIQdmiddotcrt~)

Lt Dome of Diaphragm (why left Look this up in Adams)

Spleen (What is the blood supply)

Left Kidney (What is the blood supply - AND how is it different from the R kidney)

Suprarenal Gland (What is the Arterial AND Venous Blood supply - how are they different)

Pancreas (How does supply differ from Head to Tail What is the SMA Relationship)

Transverse Mesocolon

liver - ADAMSWET - Make sure you look at the liver in wet lab

Left triangular nl1am~nt

ComoaDj ligamnt

Erophg~1 impre$ioo

Hepatio veins

In1erior -ifena middotr3)Ia

Fibrous appendix o-t

live

impr~j on

Heprorendl p~rtion of Q)(Qllary ligament

Righllri~n9ul r 1I~met

(Common) bile quol

Gr)mmCtr~ hepatic dlJct

Ccentic duct

Duodenal impression

GaJdate p-fr)~S

Hepatic artgtrl prop-f iiiiila - Faloiform ligament

_ - shy Round ligamen liver

~--F-- CoJio imprgt-ssi-on

Prta heptis

Identify the lobes impressions and embryonic remnants associated with the liver

Caudate Lobe Quadrate Lobe Right Lobe Left Lobe Round ligament Falciform Ligament

Ligamentum Venosum (what is its fxn in embryonic life) Hepatic Veins (NOT PART OF THE

PORTAL TRIAD) IVC PORTAL TRIAD - Contents relationship cross section etc Know the

Galbladder relationship to the lobes of the liver

Biliary Duct System - Make sure you understand the sequence of these structures - BE ABLE TO

DRAW A FLOW CHART

TPVd i

t

I t

1 __ Cm-(r

patk GlJet

I

J

Clinical = JAUNDICE is caused by anything that prevents delivery of bile to intestine Tumor of the

head of the pancreas Stones etc Patient will have pale stools and yellowish colored mucus

membranes

Clinical- Any scenario that tells you the patient has BILLOUS VOMIT means that the obstruction to

the flow of digestive contents is after the Ampulla of Vater (Site of Entry of Billiary system to the

duodenum) - ie Duodenal Atresia

Spleen -located posterior to the mid axillary line between ribs 9 and 11 Make sure you know that

the 10th rib is the main axis of the spleen and this organ is susceptible to injury (stab wound errant

thoracoce ntesis etc)

The spleen is derived from mesodermal cells - NOT THE GUT TUBE

The spleen rests on the left colic flexure associates with the tail of the pancreas Know the

structures entering the Hilum of the spleen

Sh rt O~-t~ic 1 0(0 10 rtiltSPIric Iloa nt

(cut)

Peritoneum - similar concept to Pleura - think of a fist in a balloon

Visceral Peritoneum - Layer of balloon touching your fist

Parietal Peritoneum - Layer of balloon not touching your fist

Your fist represents the organ your wrist is the hilum and your arm contains the blood supply

entering the organ

Appreciate that there will never be organs in the peritoneal cavity - rather these organs invaginate

the cavity Kaplan videos

RULES OF NOMENCLATUREshy

1 Organ completely surrounded by peritoneum - peritoneal organ

2 Organ partially surrounded by peritoneum- Retroperitoneal

3 Peritoneum surrounding peritoneal organ is VISCERAL peritoneum

4 Peritoneum surrounding retroperitoneal organ is PARIETAL peritoneum

5 Peritoneum connecting visceral to parietal is called messentary 2 messentaries in the

gut Dorsal (to the gut tube) and ventral (to the gut tube) messentary

Aorta is in Retro peritoneal position - but blood must reach peritoneal position - vessels travel through

messentary All peritoneal organs will have blood supply reaching through messentary

-Mesentery is a 2 layer peritoneum with a neurovascular communication between body wall and organ

- Ligament connects one organ with another or to the abdominal wall (Ommentum = ligament)

lesser Ommentum (attach lesser curvature of stomach and duodenum to liver) =Hepatoduodenal

Ligament and Hepatogastric Ligament

Has a Superior and Inferior Recess (Accumulation of Fluid in Ascites)

Communicates with the greater sac through the epiplic foramen (what structures pass through

this foramen)

Boundaries - you must be able to visualize this

o Anterior - stomach

o Posterior - parietal peritoneum pancreas

o Superior - superior recess (bw diaphragm and coronary ligament)

o Inferior -Inferior recess (bw layers or greater momentum

Greater Ommentum (attach greater curvature of stomach) Gastrophrenic ligament Gastrosplenic

ligament gastrocolic ligament

The greater omentum is the largest peritoneal fold It consists of a double sheet of peritoneum folded on itself so that it is made up of four layers The two layers which descend from the greater curvature of the stomach and commencement of the duodenum pass in front of the small intestines sometimes as low down as the pelvis they then turn upon themselves and ascend again as far as the transverse colon where they separate and enclose that part of the intestine

ABDOMINAL PAIN

Parietal Peritoneum - supplied by same vasculature lymphatics and nerves supplying body wall it

lines and diaphragm Sensitive to pain pressure heat cold well localized

Visceral Peritoneum - supplied by same vasculature lymphatics and somatic nerve of organ it covers

Insensitive to touch heat cold and laceration - referred to dermatome of spinal ganglia providing

sensory fibers Where does appendicitis refer to

Foregut pain - epigastric area (ie - cholycystitis)

Midgut pain - periumbilical area (ie - appendicitis)

Hindgut Pain - suprapubic area (ie - diverticulitis)

Extra ImagesConcepts

ll~_____-

FalifCtrm ligament oind r~ud ligamet f Ilver

Blood from splenio gastriC and inferiof rne$e-rteri v~ins

Ca-I tributaries

Lett gastrio Ifein

Posterior superior pan~reatioodul)denal vaihS

Lott gamo-om~nlal (9aropip lomiddotic) -in

Poq_~ tjol imerl-9-r panCJertlcorllJod-nal veiopound --amp----I- - ~J Right grtr~-omntal

Anwrior interi (gartroepiploic) Jjn

pan euaii cod vl)denal veins middot Inf~Ji (t r mesentric vein

Miqdle (olic vein

Right cl)licvein Sigmoid and rectosigml)id (ei ns

IhH)Collc(~io

--- Mi~dl laquooLJl gtjrltgt

PoM ca vl1 illasto)moses -----shyampoptoageal 2 Paraumbilie-lt11 Inferi or Fectal vei ns

3 Recial 4 REuoperHonea1

Know how the Portal vein is formed I 4 sites of portal caval anastamoses and 1 clinical shunt

Col li t ltt-~ otTl~tI ~nj pc~ 1lt1 turJoG

Ltf 14i1 tImiddot~ artoftl9 on tj phtAt$

L-oftqf 4t t~r 1=laquoIran d 1 bull shy~p l ci rj o fOOOts

Nerves follow the arteries - appreciate the splanchnic nervous system I

Uet~ric branch of left ~nal art

Ureterie branch of righi renal artelY

Left Zld lumbar in and co mlTlunication to as)erdin9 lumbar l(~in Hi ~ht tEZ1~~t~ t3r j t itn ~ nJ l1t- rlnd lfe i r1

Inferior me5nteri~ artery

Notice that the right testicular vein drains directly into the IVC and the right testicular artery drains

directly into the aorta However the left testicular vein drains into the L renal vein at a right angleshy

reason left testicle is lower and more susceptible to varicocele (bag of worms)

Also notice that the left renal vein has a longer course because the IVC is on the right side whereas

the right renal artery has a longer course because the aorta is on the left side

Appreciate the anterior to posterior relationship of structures in the hilum of the kidney - VAP - Vein

Artery Renal Pelvis (Ureter)

11____ __ L_ L_ n VJ __ _ _ t_L I I_ _ L __ L_ I -pound1 bull LI_~-I ____

Posterior View of Head of Pancreas in ( of Duodenum

Celiao hunk

Co mmon ~L~jJth art~ry

GastNduQdonal artrf (partilly in phantn)

P1)Sterior $Up~Jior panCflaticuduodfmal art~r~t

(Co mm on) bile duct

middot~1t~~t-1l---~-~- Right gshomiddotomental (gastoe plp lolc) 3rte (phantomost)

Grener paocre atic art-ry

1n1~rjor pancr-iatlc artery

Jtrifll supejo r pal)oreailcento)dJodenal artr1 (phantom)

Anastomotlo branch

POostetlor bJanch of jo f~ri of pan-reatir(lduodensl drttnj

Anterio r branch of i flferior palcreati~)duodenal art~(phan1om)

Notice the extensive blood supply to the pancreas and duodenum via the branches of the celiac trunk

Notice collateral supply from SMA branches - makes sense bc this is the jxn of foregutmidgut

Identify the vessels in this arteriogram

Hiltid i)f N~ck oi B)dvof Tail 01 pa nereas pan cent~as P-nmiddot-reas panCtCas

I nferie v~na cava

jHept1iic p(lrlai v~in

Port1 tnd H~pti lt a ftH prol

Comm on) bll duct

Ouodtnum

~ft colic (sio)Atta~ hmtrlt jt~xJr-ofha~elSe

muo(IIQn

Right ~lIc (h~j)tic)

il~gtture

In1triol m~oten lIein (rttr op~ritoMdO

SlJp efl or mes~n~fiC amrV and lipln

KNOW YOUR NEIGHBORHOOD

Questions

vVhiJh structure supplied by a bnmdlof the cclia( artery is not derivcd from foregut LemCJUCrITI

(A) Head of the pancte-a5

CD) Pyloric duolenum

Cystkduct

( Liver hepatocyt~~

~F) Body of the spleen

An infant presents with an omrhaJucele at birth -hi oJ the [oHm illg applies to his cM1-dition

(A) It is 31so seen ill p4titnts with aganghonic megacolon

(11) ft reuirs from a fal1ure of resorption of theviteUine d let

(C) It results from herniation at the-site of regression of the right umbilk vein

DJ It is caustd by faihtrc of recanalization of the midgut part of the duodenum

~ It ill camioo by a failuIt vf the midgul to return to the abGQminal uity after herniashytion in-n the urnbilk s l stalk

Ot er than the spleen occlusion Cif the spit-Ilk artery at its odgin wm most likely affect die blood supply to jllch st cnud

(A) Jejunum

(B) Body of th pal1~lltas

(C) LeSStT Cllmiddotlaturc of tl )toma-ch

(D Duodenum dista to the entrance of the Ornmou bile duct

E Fundus of the stomach

A 38-yeu-old batL~er with a history of heartburn suddenly experiences excluciating pain in the (plgastric region of th~ abdomeu SurgCry is perf~rme immediard y upon admisshysion to the 1IlcrgCJliy tuomh~re i~ evidence uf a ruptured ulcer in the posterior waU of the stomach Vhere will a surgeon first fi nd the stomach contenlSf

A) Greater p4ritoneal sac

rB) Cul~de-s~c of Douglas (--

C Omental bursa ~

--D) Paracolic gutter

rEj Between -he panttal perimltum and the posterior body wal1

At birth an infant presents with a st()ma~ rb~tbas~njJled jfltotb~diaplfagru 1A1ltre is the defect thatresulied iiitJle heini~t()n shy~tsophagealbiatus

7 - rH-- Hiatus for the inferior vena cava

( Pleuroperitoneal membrane -(0) Septum transvcrsum

(E) Right Crlt~

An infant born with DOVv7l syndrome presents with bili()u~ vomiting Ahat congenital defect does the infant have

(A) Pyloric stenosis

(B) Meckel diverticulum C) Ornphaloce1e

(D) Gastroschisis

( ~ ) Duodenal atresia y A patient with cirrhosis of the liver presents with ~ bacalvaricestnlreased retrograde pressure in which veins caused the varices

(A) Paraumuilical

(B) Splenic

(ct AzygltJus

(15))G~trk ( (-F) Superior mesemeric

A htaltby 3-year~old male patient experiences a hernial sa protruding from the anterior abdominal wall about halfway between me anterior superior ilia spine and the pubk tuberde Pulsations of al1 artery are palpated medial to the protrusion site through the abdominal walL Which layer of the anterior abdominal wall will first be traversed by the

1hctma

fA) Rectus sheath (B) External oblique aponeurosis

(C) Inguinal ligament

lD) Transversalis fusda

(E) Cremasteric fa~cia

After 5urgi(aj ffpair of a hernia the patient tXperienccs mtmlgtness in the skin on the anteshyrior aspect of the S(Totum_ Vhaf nerve may have been lesioned during thehemiorrhaphy

(A) Femoral

(B) Obturator

(C) Ilioinguinal

(D) lliohypogastrk

(E) Pudendal

A 23~year-LJld female secretary il1 good health ~-uddcn1) doubles over with pain in the a ea of the 1JmbRicu$ Sbe feels vartn and ltneasy and has no appetite That night the pain seems to have mQved to the tower right abdominal regjol1 and she calls her family doctor who then arranges for an ambulance to pk-k her up and take her to the hospitaL Wh ell ntn~ perceived in the area of the urnbilirus most Hkely carried lhe pairfu I sensations into the eNS

tA) Vagus nerves I~

V B)

) Lessersplanchnk nerves

tC) Pudendal nerves

(D) lIiohpogastrk nerves

(E) Greater splam ic l erves

A CT reveals carcinoma in the bOod of the ancreas Vhich blood vessel trut ourses ----~- - -bull ------ --shy

immediately poftterior to the body ofthe pancreas is the m~t likely to be oompressed

(A) Splenk artery

(B) Abdominal aorta (C) Portal vein

(1) Splenic vein

(E) Renal vein

A patient has a penrln1l1ng uker of the posterior wall ot the br~l part ot the (lUooenmn llkh blood vessel is subject to erosion

(A) Common hepatic artery

(B) Gastroouodenal artery

(C) Proper hevatic artery

(D) Celiac artery

(E) Anterior inferior 11amrelltlcoduodcnal attery

Your patient has been diagnosed -ith a carcinoma locallted to the head and l~e(k of the pancreas Another clinical sign would be

A esophageal varices

(8) hemorrhoids

C) a caput medusa

(D) increased pra Teuro n th~ hepatic veins

(E) enlarged right supra lavkular lymph nodes

Wltkh of the foUowing structures develops in the ventral mesentery

(A) Spleen

(B) Jeiunum (C) Head of1ht pancreas (D) Transverse colon (E) Stomach

ti l Uw ~ littwin~ f( S-t lil oai Imdge ~ hi(h or tbt la~)d J truetur tgt liJ llntn nl) he hl p UC iJd [IIi ell

c o

A) drains Ie tht infCrior a La aI

R t middot~nfl0 ~ill to th~ lunlgtn of h i dtlndCrlllfH

(e) m t bull JiJattd on tl l J n T ~H

D ) sup Lc O VSlt I Hlid bhtu l 1 li - -I un oid

( ) U~tpli(t tr j middottUh~ 1 v(( b~nt rfK n1ilc~Zm

ANSWERS AND EXPLANATIONS

Answer E The spleen is t hlttnopodicand lymph organ demlted from mesoderm

Answ~ R Al1 tlmphalocele is caused by it failure of the nlidgut to return to the ahdomir nat cavity after herniation into the umbiliau Stalk Choices Aand D maybe seen in infants with Down syndrome choice D ~s the specific CBuse ofduudcnal JtiCSitt Choice C is (ile cause of gclstrosbisis and Choice B nsults iu a Meurolktldivertku1-tlB

Answer B The fundus ofthe stomach is suppHed by soort gastric brunches of the splenic altery The splenic artery supplies the body and tail of the pancreas part of the greater curvature of the sttmla(h and the spleen Te jejunum part of the head of the pancreas and tht~ duodenum distal to the entrance of the commOll bile duct are supplied by the superior mesenterk artery clll~l ~be less r ctlt1ature cmd the pylQric antrum are supplied by the right and lei gastric art(ries

AnSWftt C Tbeomental bursa or lesser ~ritoneaj sac lies direcdy posterior to the proxshyimal part of the duodeTtlm and the stomach and would be the first site where stomach contents ~Ott1d be fpoundluncL

Answer C A defect in a llleuropcritoneal membrane (uswlly the left) is the typical site of i1 cc-ngenitlI diilphragluatic hemia llere the membr4ne fails to dose ()pound( of the perishycCirdiopcritulleal canals

Answer E DuoJenal atresia and aganglionic megacoion are congwitaI defects S~Il in patients with Dowmiddotnsyndrome

Answer D RulaTgemt~llt of and retrograde flow in g~lstrk vel_ns in particlJl~r the kft gas~ tricveins dilates the capillary bed in rhe wall of the esophagus in (ases of porta yper~

tension Blood flow would increase in and dilampte tribntarkgts of the (lZygOUS vein on the other side of the capiUary bed but flow in this vein is in the typical direction t()ward the superior vena cava Paraumbiii(ltU vein eilgorgement contributes to a caput medusH Splenic ~nlargement might prc~nt with 5plcnonlegaly and balt-kflow in to tlu superior m~~ntclic vein occurs but is asymptomatic

Answer D The patient hagt an indirect inguinal hernia whi~h emerges from the antt-rior abdominal wall through the deep inguinltilling Theeep ring is a fault in the transv~rshysaUs fascia this I~yer wiIJ be penetrated first by the hernia

An~Wer C The ilioinguinal nenc which provides sens~llion to the lnedlal thigh ltmclanteshytior SClotunl pass~lt th rough the 5uperfh_ial inguinal ring ind $subject to inj i1T) becaus-e

it is in the operatitm Held of the erniorrhapny

Auswer B The leMHr splanchnic nerves are sympathdic nerVlts that carry viscera l sensashytlltgtrogt ftom illtllt1m~d ()J stietched gust (itinteitinal ~tructures (in this case the pprndix) into tnt eNS Lesser splanchnic ntTYcsarisc from thmiddot T9--T12 spinal cord segments lt1nd provide sympathetic innenation tD rnidgut siruc1ures whiCh include CLe app~JldD Viscera] Pain arising from affecLed Inidgut ampt 1C1ure is referred over the same dl- matorne~ of spinal segrnertts v-hich provide the sympathetic Innervation n this G1SC of appendicitis the invohen~n t of the ltire) of t e unlhHku indud s the T 10 dermatome

Answer B Of the five choices onty the dscending olon is retroperiton~al aldwould be a lik ~ ( choice to be seen immediately a(~jilcent to t11e posterior abdominal middotn~L

Amwen D The SpltftlC ~-ein ourses posterior to the body of the panneas m its way tt drain into the superior mCSfttltlri( vein

Answcr B TILt glstrodllolticnal artery 1 direct hIamh of the comrootl hepatic artery courses immediately pt))iwri() to the duodenum and is slbject to erosion

Answer B Carcinoma of th pan middott3S in the 1tilt1 may compreampgt the portltil vein at irs orishygill The poTtai vcin is fomled when the splenic vein jQiaswith tfie superior meStllt eric vein The inferiot mesenteric vein joins the ~plenjc vein just priOT to tlli~ point at which the splenic joins the superior Jlleit1ltcri( vein Increescd venous presslu in the inferior mesenteric vein is a cause of emo hoid~

Answer C The- velltral pancreas wilich forms most of the head of the p ~ncr as develops in the ventral mes(ntery as antutgrowth of the hepatic diverticulum Th~ hepatic divershyticulull induding the biIJary appa~atus develops in tbe ventral mesentery of the foregut

Answer~ A The superior mesenteric ~in joins with the spienkvein to form the hepatic portal vciu

Answer D The structure at gttlK is the proper hepatic artery~ whkh suppUesoxygenated b middotood to the liver

MAKE SURE YOU KNOW the diff bw Rectus Sheath above and below the arcuate line

ABOVE

Aponeurosis of xiiltmal obllque musclo

Extemll f)biquw musde

Reotln ilbdomlnls musole S~in

Internal 9bliquQ mY~QI

AponeUfOsi$ of hJH$V~~S Lir9a a lb lbdolTlin~ musolo Tri OJV6 rUi

atldomlnis mUS(loe

Sub cutanlilous tiue (tatty ye r)

BElOW

A POrl lJfosis 01 etemal oblique muscle

Aponeul~)sis 01 Internal oblique mU$cl~

Anteriol lay~ of r~ltdus st~ath EXttom1 oblique rnu$cll

Rectus Jbdominis muscle Intoernal Aponeurc-sis of tra~fersU$ oblique muscle-

at-domlnis muscentl ~ Skio

Tra nsvitSus abdomioLs ml)ZClt

TralSVersaHs fascia Medial umQil iegtt1 1i9Jment -and folj

Uldchus Peritoneum (ir median Umbilj~al Suboutane ous

Extraprftone 11ascia

Ymbilimiddot~1 fold)

preu9poundiea1 fascia

tissue (fatty 4nd m~mbr3n(iUS layers)

o Above the arcuate line (A horizontal line 13 of the distance bw the umbilicus and the

pubic symphysis) -10 Aponeurosis divides into an AntPost Laminae

o The Ant Laminae joins EO and Post Laminae joins Trans Abdominis = Ant and Post

RECTUS SHEATH respectively

o BElOW the arcuate line - all 3 aponeurosis join ANTERIOR to rectus muscle to meet its

counterpart in the midline (linea Alba)

o Take away Msg - The abdomen is devoid of a posterior rectus sheath below the

arcuate line and is therefore more vulnerable to herniasinjuries

Question - A physician makes a deep incision in the patients midline immediately superior to

the pubic symphysis which of the following layers is his knife least likely to pass

Rectus Abdominis External Oblique Ant Rectus Sheath Posterior Rectus Sheath All of the

Above

Answer - All of the above None of the other answer choices are midline structures -LINEA

ALBA

Linea Alba has very poor blood supply - doesnt heal well after surgery Therefore this is a

common site for incisional hernias

a Spleen b Transverse colon c Descending colon d Stomach e Pleura

17 Meckels diverticulum is normally found 2 feet proximal from the

a Pyloric sphincter b Lower esophageal sphincter c Ileo-cecal valve d Middle valve of Huston e Anal valve

18 Ulcer in the posterior wall of the first part of the duodenum would erode ___ artery and would cause bleeding

a Left gastric b Right gastric c Hepatic artery proper d Gastroduodenal artery e Middle colic artery

19 An inflamed appendix is identified by a surgeon on the operation table by noting

a The appendicies epiploicae b The convergence of tenia c The artery of Drummond d The mesocolon e The mesosalphinx

20 The nerve which emerges through the psoas major is

a Femoral b Ilio-inguinal c Ilio-hypogastric d Pudendal e Subcostal

21 The right gonadal vein drains into the

a Azygos b Hemiazygos c Inferior Vena Cava d Right renal vein e Left renal vein

22 The hepatocytes in the liver is derived from

a Ectoderm b Endoderm c Mesoderm

d Neural ectoderm

23 Abscess in the lumbar vertebrae due to tuberculosis would spread to the adjacent muscle which is

a Psoas Major b Iliacus c Quadratus lumborum d Tranversus Abdominis

24 The anterior wall of the inguinal canal is formed by

a External oblique and transverses abdominis b External oblique and fascia transversalis c Internal oblique and external oblique d Internal oblique and transverses abdominis e Fascia transversalis and peritoneum

Meckels diverticulum is a result of which of the following developmental abnormalities shy

A Failure of the vitelline duct to close

B Failure of the herniated intestinal loop to retract into the abdomen

C Failure of the urachus to close

D Failure of the midgut to rotate

E Failure of the hepatic duct to close

Explanation

Meckels diverticulum is a result of the persistence of the proximal part of the vitelline duct This

diverticulum is usually found about 2 feet proximal to the ileocecal junction and is usually about 2 inches

long It is present in about 2 of the popUlation It may be the site of ectopic pancreatic tissue or gastric

mucosa and may develop inflammatory processes and ulcerations Acute Meckels diverticulitis

simulates appendicitis

Which of the following veins carries blood from the esophagus to the portal vein The

A right gastric vein

B left gastric vein c splenic vein D azygos vein

E left gastroepiploic vein

Explanation

The left gastric vein a direct branch of the portal vein drains blood from the lesser curvature of the

stomach and the inferior portion of the esophagus Because branches of the portal vein do not have

valves blood can flow in a retrograde path when there is an obstruction to flow through the portal system or liveL Rlooci Cln then flow from the nortl] vein thr()1Ph the left PRstric vein to the esonhlPlIS lno

through venous communications within the submucosa of the esophagus to esophageal veins that drain

into the azygos vein The increase in blood flow through the esophageal submucosal veins results in esophageal varices

On the posterior wall of the abdomen the celiac ganglion A contains cell bodies of postganglionic parasympathetic neurons B is synapsed upon by neurons in the posterior vagal trunk C is synapsed upon by neurons in the greater splanchnic nerve D contains sensory cell bodies of lumbar spinal nerves E contains cell bodies of neurons that cause an increase in the rate of peristasis

Explanation The celiac ganglion is one of the preaortic ganglia of the sympathetic nervous system It contains cell bodies of postganglionic sympathetic neurons The sympathetic splanchnic nerves contain preganglionic sympathetic neurons that pass through the sympathetic chain without synapsing These splanchnic nerves go to the preaortic ganglia to synapse The greater splanchnic nerve contains preganglionic neurons from spinal cord segments T5-T9 This nerve synapses in the celiac ganglion The nerve fibers in the vagal trunks are preganglionic parasympathetic fibers that go to the walls of the organs that they will innervate and synapse on postganglionic parasympathetic neurons in the walls of those organs Cell bodies of sensory neurons in the abdomen are found in the dorsal root ganglia or the sensory ganglia of the vagus nerve Sympathetic innervation decreases the rate of peristalsis parasympathetic innervation increases the rate of peristalsis

Which of the following pairs of arteries will allow blood to bypass an occlusion of the celiac trunk

A Left gastric artery-right gastric artery

B Left gastroepiploic artery-right gastroepiploic artery

C Superior pancreaticoduodenal artery-inferior pancreaticoduodenal artery

D Splenic artery-common hepatic artery

E Left gastric artery - proper hepatic artery

Explanation The anastoOlosis of a branch of the celiac trunk and a branch of the superior mesenteric artery will

provide collateral circulation around an occlusion of the celiac trunk Each of the other choices pair

branches of the celiac trunk therefore these will not provide collateral flow around the obstruction of the

celiac trunk The left gastric splenic and common hepatic arteries are direct branches of the celiac trunk

The right gastric artery is a branch of the proper hepatic artery which is a branch of the common hepatic artery The left gastroepiploic artery is a branch of the splenic artery The right gastroepiploic artery is a

branch of the gastroduodenal artery whlch is a branch of the common hepatic artery

Which of the following organs has appendices epiploica The

A sigmoid colon

Bjejunum

C duodenum

D stomach E esophagus

Explanation Appendices epiploica are characteristic of the colon Appendices epiploica are subserosal accumulations

of fat None of the organs of the gastrointestinal tract has appendices epiploica except the colon

Page 10: Chirag's Abdomen Review

Hirshsprungs disease - baby cant poop - dilated colon

Meckels Diverticulum - rule of 2s - 2 feet prox from ileocecal ju nc

Urachal Fistula - weeping belly

Gall stones - common bile duct

Jaundice relation to tumor of the head of the pancreas

Hepatopancreatic ampulla

Blood supply of renal gland - s superrenal art m s suprarenal a abd aorta inf suprarenal art

R Kidney - Tl2-L3 L Kidney - Tl1-L2

Renal Artery - L2

Epiploic foramen - know the borders and contents

Alantois diverticulum - urachus - medial umbilical

Lateral Medial and Median umbillical Folds (know the contents)

Directindirect hernia - know how to diagnose where they enter and exit the inguinal region and which

one is congenital

Anular pancreas - projectile vomiting

Pyloric Stenosis - projectile vomiting (non bilous)

Duodenal Atresia - projectile vomiting (bilous)

Dry Lab - know x rays

Vitteline Fistula - food out of umbilicus

Major duodenal papilla - junc of foregutmidgut

Arcuate line - relationship to rectus sheath

Mcburneys point -13 from ASIS bw umbilicus

Parietal pain - what is the nerve supply

Internal Oblique - cremasteric relationship

Know spermatic fasia

Processes Vaginalis - connection bw peritoneum and gubernaculums

Umbillicus - TlO dermatome

Deep Inguinal-l25 cm above mid inguinal ligament

Superficial Inguinal Ligament- superolateral to pubic symphysis

Variocele - veins engorged in scrotum (bag of worms)

bull

bull External spermatic fascia derived from external obliques EO II Cremasteric fascia ~ from internal obliques fO bull Internal spermatic fascia derived from fascia transversalis bull Tunica vaginalis derived from processes vaginalis directly rests on testes bull know order from testes out to skin

note reflex o ilioinguinal nerve o Efferent =genital branch of the genitofemoral nerve

--lt gt-- info important anastamoses which connects thorax to abdomen

bull Sup

o Sup epigastric branch of internal thoracic o Inf branch external iliac

Venous drainage o Above umbilicus aXillary v o Below umbilicus veins in triangle o At level of umbilicus Paraumbilical veins -gt drain into the portal V

II Important in Portal Caval Venous system Venous drainage of testes

o Clinical correlation Varicocele 11 vein drains into IVC 11 Left testicular vein ~ drains into left renal v

bag of rmlt

for lymph drainage T10 axillary lymph nodes

ill Below T10 superficial inguinal lymph nodes (lateral

Umbilical Folds

Lateral umbilical folds inferior vessels

Medial umbillcial folds umbilical (fetal remnant)

Median umbilical fold urachus (fetal remnant)

Between these folds fossas o Supervesical fossa between median and medial folds

11 bladder o hesselbachs between medial and I folds

II DIRECT HERNIAS HERE Borders

Medial semilunar line

Lateral info Epigastric

Inferior inguinallig o Lateral Inguinal Fossa beyond lateral fold

INDIRECT HERNIAS HERE II Deep inguinal ring (lateral to inferior epigastric a)

Indirect inguinal hernia o Lateral to inferior epigastric a o more common o When inserting finger in superficial inguinal ring will feel on tip of finger (since it goes

throueh ineuinal canall

----

Dry Lab - Label subcostal iliohypogastric Ll Ilioinguinal (Ll)

Horesshoe Kidney - stuck under IMA

Renal Agenesis -failure of the ureter bud to develop

Double Ureter

Unilateral Agenesis -1 kidney

Kidneys - Metanephros

Fetal kidneys are at sacral level

Look at 3rd part of duodenum

Some of this stuff is repeated I know just copied and pasted a bunch of stuff I had copy

Dermatomes

bull T4 nipples

bull no umbilicus v o Pain referred to no in appendicitis o Pain referred to T7ITS in gastritis ~

Inguinal ligament = external abdominal oblique aponeurosis

bull Inserts at anterior superior iliac spine to the pubic tubercle o Why important to know -7 visualizing this line allows us to properly diagnose a hernia

Below the inguinallig femoral hernia Above the inguinallig =inguinal hernia

Also to palpate the deep inguinal ring you go about 12Scm above the mid-inguinal

point

bull Modifications to ligament o Pectinate ligament o Lacunar ligament -7 cut this ligament to relieve strain i~ stran ul~tEd hernia

Inguinal canal

bull in males -7 transmits spermatic cord o important structures of spermatic cord ductus deferens testicular a genital branch of

the genitofemoral n pampiniform plexus of veins bull in females -7 transmits round ligament

Borders

bull Floor -7 inguinal ligament + lacunar ligament bull Anterior -7 aponeurosis of external oblique + internal oblique bull Roof -7 internal oblique and traverse abdominal bull Posterior -7 transverse abdominal + transversalis fascia

o Reinforced by conjoint tendon bull Aponeurosis of internal abdominal obliques and transverse abdominus bull Lies immediately behind the superficial inguinal ring in what would otherwise be

a weak point in the abdominal wall bull Innervated by ilioinguinal nerve (Ll) ~why important

bull In appendicitis Ll can be injured which will injure this nerve and in turn

the conjoint tendon With loss of innervation to this supportive structure the patient is now predisposed to a direct inguinal hernia

o Only hernia that can transverse the inguinal canal o Associated with congenital condition persistent tunica vaginalis

bull Direct inguinal hernia o Medial to inferior epigastric a o When inserting finger in superficial inguinal ring will feel on back of finger o Associated w old age or recent surgery

Muscles (Abdomen RECTUS SHEATH)

bull Arcuate line at level of ASISor 13rd distance between pubis and umbilicus bull Above arcuate line rectus abdominus is surrounded by a rectus sheath anteriorly and

posteriorly

o EO and 10 lie over rectus abdominus o 10 and TA lie behind rectus abdominus

bull Below arcuate line rectus abdominus has no rectus sheath posteriorly o EO 10 and TA lie over rectus abdominus o Transversalis fascia lies behind rectus abdominus o Inf EpIgastric vessels pierces the rectus sheath here

Peritoneum serous sac which encloses most of the abdominal structures

bull Ovary =only intraperitoneal organ o Oocyte ejected from ovary then captured by fallopian tubes o Why impt Women more prone to infection that can enter peritoneum

Peritoneum forms

bull Mesentery double layered fold of peritoneum formed as the organ was pulled in

bull Ligament between 2 organs in general bull Omentum between stomach and another organ bull Bare area area of no peritoneum bull

Viscera innervation

bull Pa rasympathetics 11 o Afferents sense hunger o Efferents l peristalsis relaxes sphincters gland secretion

bull Sympathetics o Efferents do opposite o Afferents CARRY PAIN SENSATION OF THE VISCERA (dull stretching pain)

bull PARASYMPATHETIC INNERVATION o Vagus nerve 7 _1l to 23rd unct ion of la rgej nte~tine oJ)elVrcspla~~~)~~ic~rYe~ IJiU- ~rd aJ~lpoteotiD~ IMPT

Gut Embryology

Gut ~ We say that the gut is derived from endoderm We often forget that when we say so we mean

that only the mucosa is derived from the endoderm The submucosa and the muscle layer is actually derived from the splanchnopleuric mesoderm and the serosa is derived from the visceral peritoneum

~ The main function of the gut is to digest the food which is done by the glands derived (and are) in the mucosa (endoderm) The only two exceptions in the Gut where glands though derived from the endoderm do not stay there but migrate down into the submucosa are esophagus and duodenum These glands however have their ducts opening to the swface of the mucosa

bull

~ Lungs liver amp gall bladder and pancreas are off-shoots from the foregut Esophagusshy~ The region of the tube from the laryngeal diverticulum to the beginning of the stomach elongates

to form the esophagus ~ The glands which form in the endoderm (mucosa) migrate down into the submucosa The path

whlch it took migrate becomes the duct of the glands which open to the mucosa ~ Achalasia Cardia - Failure of relaxation of the lower esophageal sphincter because of congenital

absence of ganglia at the sphincter (The ganglia when present releases VIP (Vaso-IntestinalshyPeptide) which relaxes the sphincter)

Mid-Gut Rotation ~ Because of the 90 degree rotation of the primitive stomach all of the following events occur ~ Lesser curvature comes to the right Therefore lesser omentum also comes to the right ~ Greater curvature comes to the left Therefore greater omentum also comes to the left ~ Right side vagal trunk becomes posterior vagal trunk ~ Left side vagal trunk becomes anterior vagal trunk ~ The left side peritoneal cavity comes to the anterior aspect of the stomach and will later be called

as the greater sac ~ The right side peritoneal cavity comes to the posterior aspect of the stomach and is (relatively a

small sac because the liver is on the right) called the lesser sacomental bursaepiploic sac ~ Epiploic foramen of Winslow (the lower free margin of the ventral mesentry) wiII be the

communication between the greater and lesser sac ~ The Liver moves to the right and therefore actually causes the 90 degree rotation of the stomach

The spleen comes to lie on the left side ~ Axis Antero-posterior axis around the superior mesenteric artery

bull Counterclockwise bull Approximately 270deg bull During herniation (about 90deg) bull During return (remaining 180deg)

Duodenum ~ Becomes retroperitoneal (except the first part which is still suspended by the hepato-duodenal

part of lesser omentum) ~ Glands (of Brunner) go submucosal ~ An imaginary line drawn below the opening of the major duodenal papilla represents the junction

between the foregut and midgut ~ Duodenal atresia in Downs syndrome Liver ~ 3rd week

bull liver bud grow bull into the septum bull transversum

~ 10th week bull hematopoietic bull function

bull 10 of the total bull body weight

~ 12th week bull bile is formed

Pancreas ~ In about 10 of cases the duct system fails to fuse and the original double system persists ~ 3rd month

bull pancreatic islets (Langerhans) ~ 5th month

bull Insulin secretion ~ Annular pancreas

bull The right portiCn of the ventralbud migrates along its normal route but the left migrates in the opposite direction

~ Complete obstruction of duodenum ~ Accessory pancreatic tissue Polyhydramnios (Amniotic fluidgt 1500-2000 ml)

~ Congenital defects including central nervous system disorders (eg Anencephaly) and gastrointestinal defects (atresias ego Duodenal esophageal) prevent the infant from swallowing the amniotic fluid (failure of recanalization)

Oligohydramnios (Amniotic fluid lt 400 mt) ~ Cl~ldberenal-agenesis

bull Midgut _-_

~ Primary Midgut intestinal loop gives rise to bull Distal duodenum bull Jejunum bull Ileum bull Ascending colon bull Transverse colon - proximal two-thirds of the bull Transverse colon with the distal third

~ Primary intestinaltoop bull ncephalic limb distal part of the duodenum the jejunum and part of the ileum bull ncaudal limb lower portion of the ileum the cecum the appendix the ascending colon and

the proximal two-thirds of the transverse colon bull 6th week

bull Rapid elongation of the cephalic limb bull Rapid growth of the liver bull Intestinal loops enter the extraembryonic cavity in the umbilical cord

bull 10th week bull loops begin to return bull regression of the mesonephric kidney reduced growth of the liver expansion of the

abdominal cavity bull Jejunum -left bull Loops - more to the right

bull Cecal bud -last part (temporarily below the right lobe of the liver) ~ qIDlthaloseJe (Structures COlHLoArts9V~1tion)

bull Through umbilical ring bull 6th to 10th weeks

bull Associated with a high rate of mortality (25) and severe malformations bull Associated with chromosome abnormalities

~ Gastroschisis (Structures coming out are not covered by Amnion) bull herniation through the body wall ----=---=-shybull Into the amniotic cavity bull Lateral right of the umbilicus bull Sometimes the inferior wall fails to develop as a result lower abdominal structures like the

bladder would be exposed to the exterior not associated with chromosome abnormalities ~ Abnormalities of the Mesenteries

bull Mobile cecum persistence of mesocolon bull Extreme form - long mesentery bull Volvulus

~ Distal third of the transverse colon ~ Descending colon ~ Sigmoid colon ~ Rectum ~ Upper part of the anal canal ~ Primitive anorectal canal

bull 7th week cloacal membrane ruptures bull Tip of the urorectal septum perineal body bull Pectinate line

~ Hindgut anamolies bull Rectoanal atresias and fistulas bull Imperforate anus bull Congenital megacolon (aganglionic megacolon Hirschsprung disease)

bull

bull Hindgut

Chirags Abdomen Review - Part 2

Understanding Embryo makes learning blood supply EASY

I I

I t

~ -

)

Table l1r-~ L Adult SUmiddotuctu~SDrj~l Froln Each of he Three Dhisions of be Pringttive GUl Tube t-middot-----middotmiddotmiddotmiddot-

Foregu(

I_ (Celiac Trunk)

Ir-slt-gtphgus

S101na(b

I h -= LiJ~r

Pancre=l S

bull 1 i Biliary apparntu5

Gall bladdshy

i Pha11~Cal pltgtuchcs

LullSS-I

Mjig ---- bull __ _- ----n--duct----~---middot-------l--n

(Superior Jldesen1eric Artery)_-1I-(I_~__ middot __ O-=-)_in_middoto_r_M_e_se_n_t_e_r_i_c_An__

Uuodenu rn 2nd_ 3 lt141h V4Tt

Jejunun-~

nc-un]

tCCUJ11

AppltgtndLX

Transver5e -o1on (p~oxiln1l1 ~O Tbird)

bull__hytgtid~ _ _ ______ L _ __

Tr-dn~llt~se colon (diStul h lTd) I

)

i

Aa ca-nal -( uppeT patt) i

I I

_____ __ _ _ _ ___ - - rhe~ a(t clcriVOkt iV(5 opound~lt prbn1rC ~ nlQC blft TlI)( 134tof r~ tIonoinf~ i 1 ~l l1rd c- P Cle

Now Lets see how much youve learned

Questions

1) A pt receives a general anesthetic in preparation for a c~t~~my A right subcostal incision is made which begins near the xyphoid process runs along and immediately beneath the costal margin to an anterior axillary line and transects the rectus abdominus muscle and rectus sheath At the level of the transpyloric plane the anterior wall of the

-~~-~=--- _eco---shysheath of the rectus abdominus muscle receives contributions from which of the following

a Aponeuroses of the in~ande~tef-Ilal o~ues

b Aponeuroses of the transversus abdominis and internal oblique muscles c Aponeuroses of the transversus abdominis and internal and external oblique

muscles d Transversalis fascia e Transversalis fascia and aponeurosis of the transversus abdominus muscle

A

2) The lat~raJJJ11QjJt~gLfgJlLoneach side of the inner surface of the anterior abdominal wall is created by which of the following structures

K Falx inguinalis (~) Inferior epigastric a

c Lateral border of the rectus sheath d Obliterated umbilical a e Urachus

B

3) A man the victim of several knife wounds to the abdomen during a brawl at the Lobster Shack subsequently developed a direct inguinal hernY Damage to which of the following nerves is most likely responsible for the predisposing weakness of the abdominal wall

~ Genitofemoral nerve ( b) Ilioinguinal nerve ~-t Tenth intercostal nerve

d Subcostal nerve e Pelvic splanchnic nerve

B

4) Which of the following statements concerning a direct inguinal hernia is correct a It is the most common type of abdominal hernia b It transverses the entire length of the inguinal canal c It contains all3 fascia layers of the spermatic cord d It exits the inguinal canal via the superficial ingeJinal ring e It protrudes through H~acb strJg e

~(

1fltbS w E

tl

5) The conjoint tendon is

a Important in preventing indirect inguinal hernias b The fused aponeurotic layers of internal abdominal oblique and transversus

abdominus muscles c Posterior to the deep inguinal ring

d Medial fibers of the inguinal ligament

B

6) A 25 year old male is brought in to the ER after being involved in a car accident in which he received a crushed internal injury in his abdomen Examination reveals a lesion of parasympathetic fibers in the vagJsnerve which interferes with glandular secretory or

smooth muscle functions in which of the foliowingorgans a Bladder b Transverse coloiW c Descending colOO d Prostrate gland e Rectum

B

7) The spermatic cord includes all of the following contents except a Il ioinguinal nerve b Pampin iform plexus of veins c Vas deferens d Genitofemoral nerve

A

8 Which abdominal structure gives rise to the internal spermatic fascia (muscle) following the descent of testes in development

a External abdominal oblique aponeurosis b Transversalis fascia c Transversus abdominis muscle d Peritoneum e Internal abdominal oblique

B

9 Which abdominal structure gives rise to the tunica vaginalis fotlowing the descent of testes during development shy

a External abdominal oblique aponeurosis b Transversalis fascia c Transversus abdominis muscle d Peritoneum e Internal abdominal oblique

D

10) The lesser omentum is a peritoneal fold which is su bdivided into the a Hepatogastric and gastrosplenic ligaments b Hepatoduodenal and gastroomentalligaments c Hepatoduodenal and gastrosplenic ligaments d Hepatogastric and hepatoduoden9-jrj igaments

D

11) A posteriorly perforating ulcer in the pyloric antrum of the stomach is most likely to produce initiallocalized peritonitis or abcess formation in which ofthS fQllowing

a Great-sac - -- -

b Paracolic recess

c Omental bursa

d Right subphrenic space

c

The inferior mesenteric artery arises from the abdominal aorta ilm_ediill~y_J-Qs1eriQLto which of the foowing org~ns A-F~t~filie duodenum B Head of the pan~eis C Neck of the pandeas

D Second part of the duodenum

E Third part of the duooenum_shylaquoshy

shy

The correct answer is E The inferior mesenteric artery arises from the anterior surface of the aorta at the level of the third lumbar vertebra The third part of the duodenum crosses the midline at the level of the third lumbar vertebra and passes anterior to the aorta at the origin of the inferior mesenteric artery The

first part of the duodenum (choice A) lies horizontally to the right of the midline at the level of the first

lumbar vertebra The head of the pancreas (choice B) is to the right of the midline and extends from the

level of the first lumbar vertebra to the third lumbar vertebra It lies within the concavity of the

duodenum The neck of the pancreas (choice C) lies in the midline at the level of the first lumbar

vertebra It lies on the anterior surface of the aorta at the origin of the superior mesenteric artery The second part of the duodenum (choice D) lies vertically to the right of the midline and extends from the

level of the first lumbar vertebra to the level of the third lumbar vertebra

The left adrenaLvein drains directly into which of the following veins A Hemiazygos vein

B Inferior vena cavaee C Left renal veiri -

D Splenic vein

E Superior mesenteric vein

a

The correct answer is C The left adrenal vein and the left gonadal vein (either testicular or ovarian) drain into the left renal vein TheTeft renal vein t~ains intothe- inferior vena cava In contrast the right

adrenal ~~inandnght gonadal veindrai~ gLr~ctJy iQtoJhe iilferiQ[ Vencava -- -

ThehemTazygoS7ein- (~h-~i-~ A)~~c~i~es the venous drainage from the body wall on the left side of the

thorax and abdomen No visceral organs drain directly to the azygos or hemiazygos veins The inferior vena cava (choice B) receives the direct venous drainage from the right adrenal vein but not

the left adrenal vein Remember the inferior vena cava is on the right side of the abdomen The splenic

vein (choice D) receives the venous drainage from the spleen and part of the pancreas and stomach The splenic vein is part of the portal venous system

The superior mesenteric vein (choice E) receives venous drainage from much of the intestinal tract It is part of the portal venous system and joins with the splenic vein to form the portal vein

A 43-year-old man presents complaining of pain in the groin On examination his physician palpates a

bulge in the region of the superficial inguinal ring which he diagnoses as a direct inguinal hernia The hernial sac most likely

A is covered by all three layers of the spennatic fascia B passes medial to the inferior epi gastric artery

C passes medial to the lateral border of the rectus abdominis muscle

D passes posterior to the inguinal ligament E passes through the deep inguinal ring

The correct answer is B Direct inguinal hernias enter the inguinal canal by tearing through the posterior

wall of that structure The typical location for this type of hernia is through the inguinal triangle bounded

laterally by the inferior epigastric artery medially by the lateral border of the rectus abdominis and

inferiorly by the inguinal ligament Direct inguinal hernias pass medial to the inferior epigastric artery

whereas indirect inguinal hernias pass lateral to the inferior epigastric artery because the deep inguinal

ring is lateral to the artery Indirect inguinal hernias are covered by all three layers of the spermatic fascia (choice A) Direct inguinal hernias are covered by fewer than all three layers because the direct inguinal

hernia tears through one or more layers of fascia as it emerges though the abdominal wall The lateral

border of the rectus abdominis muscle (choice C) forms the medial border of the inguinal triangle All

inguinal hernias pass lateral to the rectus abdominis Femoral hernias pass posterior to the inguinal ligament (choice D) Inguinal hernias emerge through the superficial inguinal ring which is superior to the inguinal ligament Inguinal hernias that descend below the inguinal ligament pass anterior to the

ligament Indirect inguinal hernias pass through the deep inguinal ring (choice H) direct inguinal hernias

do not Both types of inguinal hernias pass through the superficial inguinal ring

During a gastric resection in a patient with stomach cancer a surgeon wants to remove the lesser

omentum because of tumor extension into it Which of the following structures lie in the free edge of the

l~~g omentum and consequently must be dissected out in order to be preserved

A Common bile duct cystic duct and hepatic artery 6

B Cystic duct hepatic artery and hepatic vein

e Hepatic vein and cystic duct

Portal vein common bile duct and hepatic artery

E Portal vein hepatic artery and hepatic vein

The correct answer is D The free edge of the lesser omentum contains three important structures the

common bile duct the hepatic artery and the portal vein Nei ther the cystic duct (choices A B and C) nor the hepatic vein (choices B C and E) lies in the free

edge of the lesser omentum

A 55-year-old male patient with chronic liver disease has portal hypertension To relieve the pressure in the portal system a porto-caval shunt is performed Which of the following veins may by anastomosed to

accomplish this porto-caval shunt A Left renal vein-left testicular veingt

B Right renal vein-right suprarenal vein I shy

e Splenic vein -left renal vein J

D Superior mesenteric vein-inferior mesenteric vein E Superior mesenteric vein-splenic vein

The correct answer is C The splenic vein drains directly into the portal vein The left renal vein drains

directly into the inferior vena cava Anastomosis of these veins would allow blood from the portal vein to

drain retrograde though the splenic vein into the renal vein and then into the inferior vena cava The left

renal vein (choice A) drains directly into the inferior vena cava The left testicular vein drains directly into

the left renal vein Thus these veins are already in communication and neither vein is part of the portal venous system The right renal vein (choice B) drains directly into the inferior vena cava The right

suprarenal vein also drains directly into the inferior vena cava Thus neither vein is part of the portal

venous system The superior mesenteric vein (choice D) drains directly into the portal vein The inferior

mesenteric vein drains into the splenic vein which then drains into the portal vein Thus neither vein is

part of the caval venous system The superior mesenteric vein (choice E) drains directly into the portal

vein The splenic vein also drains directly into the portal vein Thus neither vein is part of the caval

venous system

A 12 year old boy has fever vomiting and para-umbilical pain After examining the patient the doctor

makes an initial diagnosis of appendicitis Appendicular pain which is initially referred to the umbilicus goes to the dorsal root ganglion of

a TI b TI2 c L1 d T7

(e I TIO

A 59-year-old male undergoes a neurological examination which reveals that when the abdominal wall is

stroked the muscles of the abdominal wall of the side of the body stimulated failed to contract Other

neurological tests appeared normal The likely region affected includes

a CI - C5 spinal segments b C6 - TI c T2-TI ~T8-T12

e Ll- L5

The surgery done to relive portal hypertension is done by connecting two veins Which of the following veins would be suitable for connection

a Inferior vena cava and portal vein b Superior vena cava and portal vein c Splenic vein and right renal vein d Splenic vein and left renal vein e Superior mesenteric vein and Inferior vena cava

A mother brings her 3-week-old infant to the pediatric clinic reporting a new scrotal bulge that she found -~-

while changing a diaper yesterday The infant is afebrile Physical examination reveals a palpable mass in

the scrotum while in the standing position resolution of the mass in the supine position and no

transillumination of the scrotal sac What is the most likely diagnOSiS

a Cryptorchidism b Direct inguinal hernia c Hydrocele d Indirect inguinal hernia ~ e varicocele

The Vagal trunks enter the abdomen by passing through which of the following openings in the

diaphragm

a Right crus b Esophageal hiatus ~ c Vena caval hiatus d Aortic hiatus e Left crus

2 The anterior boundary of the epiploic foramen of Winslow is bounded by

a) First part of duodenum b) Lesser curvature of stomach c) Liver d) Hepato-duodenalligament v ~

3 The ilio-inguinal nerve is derived from

a TI2 ry b LI c L2 d L3 e L23

15 Surgically the structure used to suspend the kidney to the diaphragm is

a) Renal fascia b) True capsule c) Perinephric fat d) Paranephric fat

6 If there is portal obstruction because of carcinoma affecting the pancreas which of these of the

following signs would be present

a Caput medusae b Esophageal varices c Rectal varices c

d Pulmonary edema

7 In a sliding hernia the gastro-esophageal junction lies

a) At its normal position b) Below the normal position c) Above the normal position V d) None of the above

8 Which of the following structures is retroperi toneal

A transverse colon B spleen IJ2f6 C ileum D descending colon v r 1pound1111111

9 The renal angle is fonned lgtetween the 12th rib and ______ muscle

a Psoas major -middotshyb Erector spinae c Quadratus Iumborum d Diaphragm

10 The anterior structure at the hilum of the kidney is

a) Renal vein ~

b) Renal artery I middot~ I

c) Ureter d) Accessory renal artery

11 Because of origin of the muscle from the lateral one third of the inguinal ligament it

could not fonn the anterior wall of the inguinal ligament

a) External oblique b) Internal oblique c) Transversus abdominis_ d) Rectus abdominis

12 A large tumor mass impinges on the splenic artery and its branches as the artery pass out from below

the greater curvature of the stomach Branches o(which of the following arteries would most likely to

effected by the pressure on the splenic artery

a Left gastric b Left gastro-epipJoic c Right gastric d Right gastro-epipoloic e Short gastric_

13 A new born baby has projectile vomiting after each feeding It is determined that there is obstruction

of the digestive tract as a result of annular pancreas Annular pancreas is as a result of an abnormality in which of the following process

a Rotation of the dorsal pancreatic bud around the first part of duodenum b Rotation of the dorsal pancreatic bud around the second part of duodenum c Rotation of the dorsal pancreatic bud around the third part of duodenum d Rotation of the ventral pancreatic bud around the first part of duodenum y Rotation of the ventral pancreatic bud around the second part of duodenum

14 As the liver bud enters the ventral mesogastrium the region of the mesogastrium stretching from the

liver to the anterior abdominal wall is called

a Lesser Omentum b Greater Omentum ~ Falcifrom ligament d Lacunar ligament e Ligamentum teres of liver

16 A patient has absence of his 12th rib In such a patient if the doctor makes an incision to approach his

kidney mistaking the 11 th rib for the 12t he would end up injuring

Which of the following arteries is a direct branch of the gastroduodenal artery The

A right gastric artery

B left gastric artery

C inferior pancreaticoduodenal artery D left gastroepiploic artery

i E)right gastroepiploic artery --

E x pI a nation The right gastric artery is typically a branch of the proper hepatic artery The left gastric artery is a direct

branch of the celiac trunk The right and left gastric arteries anastomose along the lesser curvature of the

stomach The inferior pancreaticoduodenal artery is a branch of the superior mesenteric artery it

anastomoses with the superior pancreaticoduodenal in the head of the pancreas The left gastroepiploic

artery is a branch of the splenic artery it anastomoses with the right gastroepiploic artery along the greater

curvature of the stomach The right gastroepiploic artery is a branch of the gastroduodenal artery The

other branch of the gastroduodenal artery is the superior pancreaticoduodenal artery

Which of the following pairs of veins join together to form the portal vein The

A superior mesenteric vein and inferior mesenteric vein

B inferior mesenteric vein and splenic vein

C superior mesenteric vein and splenic vein

Ip)splenic vein and left gastric vein E superior mesenteric vein and left gastric vein

Explanation

The portal vein is formed behind the neck of the pancreas by the union of the superior mesenteric vein

and the splenic vein The inferior mesenteric vein drains into the splenic vein The left gastric vein drains

directly into the portal vein After the portal vein forms it enters the hepatoduodenalligament of the

lesser omentum to reach the liver The portal vein is the most posterior structure in the hepatoduodenal

ligament

At which of the following vertebral levels does the duodenum pass anterior to the aorta - _- shy

All ~

B L2 7~

CL3 I

~DL4

E L5

Explanation

The duodenum begins at the pyloric sphincter at the level of Ll The second (or descending) portion of

the duodenum is to the right of the aorta and extends inferiorly from the level of Ll to the level of L3 The third part of the duodenum crosses the aorta from the right side to the left side at the level of L3 The

fourth (ascending) portion of the duodenum extends from the level of LJ to the level of L2 The

duodenum ends at the duodenojejunal flexure The superior mesenteric artery passes anterior to the

duodenum as the duodenum passes anterior to the aorta The duodenum can be constricted at this level

In which of the following locations will perforation of the digestive tract result in the spilling of luminal

contents into the - lesser peritoneal sac

A Anterior wall of the second portion of the duodenum B Posterior wall of the second portion of the duodenum

C Anterior wall of the stomach

~Posterior wall of the stomach E Posterior wall of the transverse colon

Explanation

The posterior wall of the stomach is related to the lesser peritoneal sac The anterior wall of the stomach is related to the greater peritoneal sac The anterior wall of the second portion of the duodenum is related to the greater peritoneal sac The posterior wall of the second portion of the duodenum is related to the retroperitoneal space The posterior wall of the transverse colon is related to the greater peritoneal sac

The ureter lies against the anterior surface of which of the following muscles shyA Crus oftne diaphragm B Quadratus lumborum

0 Psoas major D Transversus abdominis

E Iliacus

Explanation The ureter exits the renal pelvis at about the level of vertebra L2 As it descends along the posterior abdominal wall it lies on the anterior surface of the psoas major The psoas major muscle arises from the bodies of the lower lumbar vertebrae The psoas major muscle is joined by the iliacus to fonn the

iliopsoas muscle The iliopsoas muscle then attaches to the lesser trochanter of the femur and is the major

flexor of the hip

As the right ureter passes the pelvic brim it lies against the anterior surface of which of the following

blood vessels

A Gonadal artery B Inferiorvena cava C Internal iliac artery

rJ- External Iliac artery

E Inferior mesenteric artery

Explanation

The ureter lies in the extraperitoneal space in the posterior abdominal wall Alter leaving the kidney it

passes inferiorly on the anterior surface of the psoas major muscle At the pelvic brim the ureter passes

into the pelvis At this point the common iliac artery is dividing into the external and iliac arteries The

ureter lies on the anterior surface of the external iliac artery immediately distal to the bifurcation This is a useful landmark for a surgeon to locate the ureter

When extravasated urine passes from the superficial perineal space into the anterior abdominal wall it is

found immediately deep to which of the following layers of the anterior abdominal wall

-ltScarpas fascia

B External oblique muscle

C Internal oblique muscle D Transversus abdominis muscle

E Transversalis fascia

Explanation

The superficial perineal space is bound by Colles fascia the fibrous portion of the superficial fascia This

layer of fascia is continuous with Scarpas fascia the fibrous portion of the superficial fascia of the anterior abdominal wall Therefore urine that is deep to Colles fascia will remain deep to Scarpa s fascia The urine will spread in the plane between Scarpas fascia and the external oblique layer

When a horseshoe kidney develops the ascent of the kidney is restricted by the A internal iliac artery B external Iliac artery

C common iliac artery

inferior mesenteric artery

E superior mesenteric artery

Explanation

A horseshoe kidney develops when the inferior poles of the to kidneys fuse together as they ascend into

the abdomen from the pelvis The first anterior midline vessel that is encountered by the horseshoe kidney

is the inferior mesenteric artery This artery prevents the kidney from continuing its ascent

The left testicular vein drains into which of the following veins

A Left internal iliac vein B Left common iliac vein

bflnferior vena cava D Left renal vein I

E Left internal pudendal vein

Explanation

The left testicular vein drains into the left renal vein The right testicular ~i~[~nsltjectlY into the

inferior vena cava This difference in venous drainage is believed to explain the greater incidence of

varicocele on the left side than on the right The venous drainage from the penis is to the internal vein

which then drains into the internal Iliac vein

The spinal nerve that provides cutaneous branches to the skin around the umbilicus is

A TS B TW-shy

C TI2

DL2 EtA

Explanation

The tenth intercostal nerve is the anterior ramus of the TIO spinal nerve After passing through the tenth

intercostal space the nerve continues forward in the anterolateral abdominal wall in the plane between

the internal oblique muscle and the transversus abdominis muscle In the abdominal wall the nerve innervates to the abdominal wall muscles as well as the skin and the parietal peritoneum The umbilicus is

a useful landmark for the region of distribution of the tenth thoracic nerve

The ligament of the vertebral column that resists its extension is the Aligamentum flavum

B supraspinous ligament

C posterior longitudinal ligament

D anterior longitudinal ligament

E interspinous ligament

Explanation

The ligaments of the vertebral column that resist flexion of the column include the supraspinous ligament

interspinous ligament ligamentum fiavum and posterior longitudinal ligament The ligament that resists

extension is the anterior longitudinal ligament This longitudinal ligament is very broad and strong It

covers the anterior and anterolateral surfaces of the vertebral bodies and the intervertebral disks In

addition to resisting extension the anterior longitudinal ligament provides reinforcement to the anterior

and anterolateral surfaces of the intervertebral disk The posterior longitudinal ligament is relatively

narrow and covers the posterior surface of the vertebral bodies and the intervertebral disks This ligament

reinforces the posterior surface of the disk The posterolateral surface of the disk is not reinforced and it

is through this region that herniation of the nucleus pulposus usually occurs

A patient presents with epigastric and right upper quadrant pain The pain is most intense 2-4 hours after

eating and is reduced by the ingestion of antacids The patient states that he has passed black tarry stools

(melena) within the last week Fiberoptic endoscopy reveals a yellowish crater surrounded by a rim of

erythema that is 3 cm distal to the pylorus Accordingly an ulcer has been identified in the patients

A fundus

B antrum

C duodenum

D jejunum

E ileum

A number of physiologic genetic and other factors increase the risk of gastric (and duodenal) peptic

ulcers The evidence that H pylori plays a principle role is compelling Smoking and caffeine are known to adversely affect the morbidity mortality and healing rates of peptic ulcers In general first-degree

relatives of peptic ulcer patients as well as males have a threefold to fourfold increased risk of developing this disorder Paradoxically in gastric ulcer disease acid secretion is not elevated It is possible that

excess secreted hydrogen ion is reabsorbed across the injured gastric mucosa In general a defect in gastric mucosal defense is the more important local physiologic

A patient presents with symptoms of duodenal obstruction caused by an annular pancreas Annular pancreas is caused by

A rotation of the dorsal pancreatic bud into the ventral mesentery B rotation of the ventral pancreatic bud into the dorsal mesentery

fJ failure of the major and minor pancreatic ducts to fuse ~ ~ cleavage of the ventral pancreatic bud and rotation of the two portions in opposite directions around -the duodenum E formation of one pancreatic bud instead of two

Explanation Normally the ventral pancreatic bud rotates around the gut tube to reach the dorsal pancreatic bud The two buds fuse to form a single pancreas and the distal portions of the two ducts fuse The ventral pancreatic bud forms the inferior portion of the head of the pancreas the uncinate process and the major pancreatic duct (of Wirsung) The dorsal pancreatic bud forms the superior part of the head the neck body and tail and the minor pancreatic duct (of Santorini) Annular pancreas is the result of the ventral pancreatic bud dividing into two portions before it rotates into the dorsal mesentery Each portion rotates in opposite directions to get to the dorsal mesentery thus encircling the duodenum The presence of annular pancreas can constrict the duodenum thus obstructing its lumen

In n _ phranlc----

Gon ~l ----_1 Lum bltano

~~--- CornmQ1t bull ac

+-~4--- lnlllirnaJ ilic

xtem iliac

OBJECTIVE - Identify the blood supply to each of the structures listed in the table on the previous page

Ill give you a head start

FOREGUT - Supplied bV Celiac Tru nk (T12)

Proper hepatic

GastiooUod 13Jafter

1nferlor pancreaticoduodenal artery

Common epatlc

Lett gas ric iiirtery

Spfen artery

shy Gastroepiphgtic artery

~ Superior mesenteric 8rtfry

~

1 Esophagus is a derivative of the foregut so its blood supply originates from the celiac trunk

(T12) The predominant blood supply to abdominal portion of the esophagus is the Esophageal

A (Branch of L Gastric) The venous drainage of the esophagus is particularly important because

it is 1 of 3 clinically relevant sites of Portal Caval anastamoses The Portal Esophageal Vein

meets the Caval Azygos System Persistent bleeding manifests as Esophageal Varices - a fata I

condition

2 The Stomach is also a derivative of the foregut has EXTENSIVE blood supply and is very high

yield on anatomy exams The lesser curvature is supplied superiorly by the L Gastric A (1 of 3

major branches ofthe Celiac trunk) and inferiorly by the R Gastric A ( a branch ofthe proper

Hepatic A) The greater curvature is supplied superiorly by the L Gastroepiploic A (a major

branch of the splenic A) and inferiorly by the R Gastroepiploic A

The Short Gastric arteries (branches of Splenic Artery) supply the fundus of the stomach and

are referred to as EIID ARTERIES because they have no collateral blood supply Therefore if the

splenic artery were occluded (ex - increased pressure in the ommental bursa) - there would be

ischemia to the fundus of the stomach Venous drainage of the stomach is extensive via various

veins lead ing to the portal system Posterior to the stomach the IMV joins the splenic V which

joins the SMV to form the PORTAL VEIN ADAMS

3 Duodenum blood supply has high clinical relevance because it is the junction of the foregut and

midgut and therefore is the site of anastamoses between branches ofthe Celiac Trunk (main

foregut artery) and the Superior Messenteric Artery (main midgut artery) The Proper hepatic

artery gives off the gastroduodenal artery which travels behind the 1st part of the duodenum

This point has high clin ical relevance because duodenal ulcers are very common and a posterior

rupture of the 1st part of the duodenum could rupture the gastroduodenal artery causing

traumatic abdominal bleeding The Gastroduodenal artery first gives off the R Gastroepiploic A

(mentioned above) and proceeds as the Superior pancreatico duodenal artery (supplies the

pancreas and duodenum) which anastamoses with the inferior pancreatico duodenal A (branch

of the SMA) This is the junction of foregut and midgut and occurs near the opening of the

bil iary system into the duodenum (ampula of vater) Portal venous drainage here is responsible

for delivering nutrients from digestion to the liver for metabolism Appreciate that the Superior

mesenteric artery (artery of the midgut) branches from the aorta at Ll travels posterior to the

pancreas than moves anteriorly (at the jxn of the pancreatic headbody) and comes over the

3rd4th part of the duodenum Tumor of the head of the pancreas can compress the SMA

4 Jiver blood supply is via the common hepatic artery (major branch of the cel iac trunk) The

common hepatiC becomes the proper hepatic gives off the R gastric A and the Gastroduodenal

A and then joins the common bile duct and the portal vein in the portal triad Clinical- if a

patient were bleeding from the hepatic A a surgeon can stick his fingers in the epiplOic foramen

and squeeze the free edge of the hepatoduodenalligament in order to stop bleeding to the

area Please note that the hepatic a branches into Rand L hepatic A The Right hepatic artery

gives off the cystic artery which supplies the gallbladder Afferent venous supply is via the

Portal vein which is bringing nutrient rich blood to the liver After metabolism takes place

venous blood leaves the liver through the hepatic veins into the IVC PLEASE UNDERSTAND THE

RELATIONSHIP OF THESE STRUCTURES - ADAMSNETIERSNH Etc

5 Pancreas - Head is supplied via the superior and inferior pancreaticoduodenal arteries

(mentioned above) The tail (situated towards the hilum of the spleen) is supplied via the

pancreatic branches of the splenic artery (END ARTERIES) This blood supply is very important

because the endocrine Alpha and Beta Cells from the pancreatic islets of lagerhans are located

towards the tail This is where Insulin and Glucagon is released to the blood

Now complete this for mid and hindgut structures Make sure to note clinically relevant arterial

anastomoses as well as portal caval anastomoses FYI Appendix blood supply SMA + IMA

anastamoses marginal artery Portalcaval rectal veins fhemmorhoids) and periumbilical caput

medusa are high yield THE BUTT THE GUT and THE CAPUT

Abdominal Development

Liver

Ij1f

II wall b

oh liN ~ VltJrti n be- bull

Pancreas

Secondary Retroperitonealization e I~tl r 1 a v-mtrai m ellter

Rotations of the Gut I i Ij (lIl1UtIJ f~ l r tilt

()l td 10 me l-ft and he v

--~--- -~ -~-~

i

I AolaijonjoI~guf I

STOMACH BED (IDENTIFY IN ADAMS)- the structures posterior to the ommental bursa which

support the stomach in the supine position

Abdomnal JQrUI

Splnic vein

OmQ-oul tv~ ) O(s(Jroa)

Lojt(r o m nturrt (hpJtodu o d~n31 Hid

Gadrl)SplerH (g3stroll~nal) IIgam~nt

hiad h~~atogrtricent IIQdmiddotcrt~)

Lt Dome of Diaphragm (why left Look this up in Adams)

Spleen (What is the blood supply)

Left Kidney (What is the blood supply - AND how is it different from the R kidney)

Suprarenal Gland (What is the Arterial AND Venous Blood supply - how are they different)

Pancreas (How does supply differ from Head to Tail What is the SMA Relationship)

Transverse Mesocolon

liver - ADAMSWET - Make sure you look at the liver in wet lab

Left triangular nl1am~nt

ComoaDj ligamnt

Erophg~1 impre$ioo

Hepatio veins

In1erior -ifena middotr3)Ia

Fibrous appendix o-t

live

impr~j on

Heprorendl p~rtion of Q)(Qllary ligament

Righllri~n9ul r 1I~met

(Common) bile quol

Gr)mmCtr~ hepatic dlJct

Ccentic duct

Duodenal impression

GaJdate p-fr)~S

Hepatic artgtrl prop-f iiiiila - Faloiform ligament

_ - shy Round ligamen liver

~--F-- CoJio imprgt-ssi-on

Prta heptis

Identify the lobes impressions and embryonic remnants associated with the liver

Caudate Lobe Quadrate Lobe Right Lobe Left Lobe Round ligament Falciform Ligament

Ligamentum Venosum (what is its fxn in embryonic life) Hepatic Veins (NOT PART OF THE

PORTAL TRIAD) IVC PORTAL TRIAD - Contents relationship cross section etc Know the

Galbladder relationship to the lobes of the liver

Biliary Duct System - Make sure you understand the sequence of these structures - BE ABLE TO

DRAW A FLOW CHART

TPVd i

t

I t

1 __ Cm-(r

patk GlJet

I

J

Clinical = JAUNDICE is caused by anything that prevents delivery of bile to intestine Tumor of the

head of the pancreas Stones etc Patient will have pale stools and yellowish colored mucus

membranes

Clinical- Any scenario that tells you the patient has BILLOUS VOMIT means that the obstruction to

the flow of digestive contents is after the Ampulla of Vater (Site of Entry of Billiary system to the

duodenum) - ie Duodenal Atresia

Spleen -located posterior to the mid axillary line between ribs 9 and 11 Make sure you know that

the 10th rib is the main axis of the spleen and this organ is susceptible to injury (stab wound errant

thoracoce ntesis etc)

The spleen is derived from mesodermal cells - NOT THE GUT TUBE

The spleen rests on the left colic flexure associates with the tail of the pancreas Know the

structures entering the Hilum of the spleen

Sh rt O~-t~ic 1 0(0 10 rtiltSPIric Iloa nt

(cut)

Peritoneum - similar concept to Pleura - think of a fist in a balloon

Visceral Peritoneum - Layer of balloon touching your fist

Parietal Peritoneum - Layer of balloon not touching your fist

Your fist represents the organ your wrist is the hilum and your arm contains the blood supply

entering the organ

Appreciate that there will never be organs in the peritoneal cavity - rather these organs invaginate

the cavity Kaplan videos

RULES OF NOMENCLATUREshy

1 Organ completely surrounded by peritoneum - peritoneal organ

2 Organ partially surrounded by peritoneum- Retroperitoneal

3 Peritoneum surrounding peritoneal organ is VISCERAL peritoneum

4 Peritoneum surrounding retroperitoneal organ is PARIETAL peritoneum

5 Peritoneum connecting visceral to parietal is called messentary 2 messentaries in the

gut Dorsal (to the gut tube) and ventral (to the gut tube) messentary

Aorta is in Retro peritoneal position - but blood must reach peritoneal position - vessels travel through

messentary All peritoneal organs will have blood supply reaching through messentary

-Mesentery is a 2 layer peritoneum with a neurovascular communication between body wall and organ

- Ligament connects one organ with another or to the abdominal wall (Ommentum = ligament)

lesser Ommentum (attach lesser curvature of stomach and duodenum to liver) =Hepatoduodenal

Ligament and Hepatogastric Ligament

Has a Superior and Inferior Recess (Accumulation of Fluid in Ascites)

Communicates with the greater sac through the epiplic foramen (what structures pass through

this foramen)

Boundaries - you must be able to visualize this

o Anterior - stomach

o Posterior - parietal peritoneum pancreas

o Superior - superior recess (bw diaphragm and coronary ligament)

o Inferior -Inferior recess (bw layers or greater momentum

Greater Ommentum (attach greater curvature of stomach) Gastrophrenic ligament Gastrosplenic

ligament gastrocolic ligament

The greater omentum is the largest peritoneal fold It consists of a double sheet of peritoneum folded on itself so that it is made up of four layers The two layers which descend from the greater curvature of the stomach and commencement of the duodenum pass in front of the small intestines sometimes as low down as the pelvis they then turn upon themselves and ascend again as far as the transverse colon where they separate and enclose that part of the intestine

ABDOMINAL PAIN

Parietal Peritoneum - supplied by same vasculature lymphatics and nerves supplying body wall it

lines and diaphragm Sensitive to pain pressure heat cold well localized

Visceral Peritoneum - supplied by same vasculature lymphatics and somatic nerve of organ it covers

Insensitive to touch heat cold and laceration - referred to dermatome of spinal ganglia providing

sensory fibers Where does appendicitis refer to

Foregut pain - epigastric area (ie - cholycystitis)

Midgut pain - periumbilical area (ie - appendicitis)

Hindgut Pain - suprapubic area (ie - diverticulitis)

Extra ImagesConcepts

ll~_____-

FalifCtrm ligament oind r~ud ligamet f Ilver

Blood from splenio gastriC and inferiof rne$e-rteri v~ins

Ca-I tributaries

Lett gastrio Ifein

Posterior superior pan~reatioodul)denal vaihS

Lott gamo-om~nlal (9aropip lomiddotic) -in

Poq_~ tjol imerl-9-r panCJertlcorllJod-nal veiopound --amp----I- - ~J Right grtr~-omntal

Anwrior interi (gartroepiploic) Jjn

pan euaii cod vl)denal veins middot Inf~Ji (t r mesentric vein

Miqdle (olic vein

Right cl)licvein Sigmoid and rectosigml)id (ei ns

IhH)Collc(~io

--- Mi~dl laquooLJl gtjrltgt

PoM ca vl1 illasto)moses -----shyampoptoageal 2 Paraumbilie-lt11 Inferi or Fectal vei ns

3 Recial 4 REuoperHonea1

Know how the Portal vein is formed I 4 sites of portal caval anastamoses and 1 clinical shunt

Col li t ltt-~ otTl~tI ~nj pc~ 1lt1 turJoG

Ltf 14i1 tImiddot~ artoftl9 on tj phtAt$

L-oftqf 4t t~r 1=laquoIran d 1 bull shy~p l ci rj o fOOOts

Nerves follow the arteries - appreciate the splanchnic nervous system I

Uet~ric branch of left ~nal art

Ureterie branch of righi renal artelY

Left Zld lumbar in and co mlTlunication to as)erdin9 lumbar l(~in Hi ~ht tEZ1~~t~ t3r j t itn ~ nJ l1t- rlnd lfe i r1

Inferior me5nteri~ artery

Notice that the right testicular vein drains directly into the IVC and the right testicular artery drains

directly into the aorta However the left testicular vein drains into the L renal vein at a right angleshy

reason left testicle is lower and more susceptible to varicocele (bag of worms)

Also notice that the left renal vein has a longer course because the IVC is on the right side whereas

the right renal artery has a longer course because the aorta is on the left side

Appreciate the anterior to posterior relationship of structures in the hilum of the kidney - VAP - Vein

Artery Renal Pelvis (Ureter)

11____ __ L_ L_ n VJ __ _ _ t_L I I_ _ L __ L_ I -pound1 bull LI_~-I ____

Posterior View of Head of Pancreas in ( of Duodenum

Celiao hunk

Co mmon ~L~jJth art~ry

GastNduQdonal artrf (partilly in phantn)

P1)Sterior $Up~Jior panCflaticuduodfmal art~r~t

(Co mm on) bile duct

middot~1t~~t-1l---~-~- Right gshomiddotomental (gastoe plp lolc) 3rte (phantomost)

Grener paocre atic art-ry

1n1~rjor pancr-iatlc artery

Jtrifll supejo r pal)oreailcento)dJodenal artr1 (phantom)

Anastomotlo branch

POostetlor bJanch of jo f~ri of pan-reatir(lduodensl drttnj

Anterio r branch of i flferior palcreati~)duodenal art~(phan1om)

Notice the extensive blood supply to the pancreas and duodenum via the branches of the celiac trunk

Notice collateral supply from SMA branches - makes sense bc this is the jxn of foregutmidgut

Identify the vessels in this arteriogram

Hiltid i)f N~ck oi B)dvof Tail 01 pa nereas pan cent~as P-nmiddot-reas panCtCas

I nferie v~na cava

jHept1iic p(lrlai v~in

Port1 tnd H~pti lt a ftH prol

Comm on) bll duct

Ouodtnum

~ft colic (sio)Atta~ hmtrlt jt~xJr-ofha~elSe

muo(IIQn

Right ~lIc (h~j)tic)

il~gtture

In1triol m~oten lIein (rttr op~ritoMdO

SlJp efl or mes~n~fiC amrV and lipln

KNOW YOUR NEIGHBORHOOD

Questions

vVhiJh structure supplied by a bnmdlof the cclia( artery is not derivcd from foregut LemCJUCrITI

(A) Head of the pancte-a5

CD) Pyloric duolenum

Cystkduct

( Liver hepatocyt~~

~F) Body of the spleen

An infant presents with an omrhaJucele at birth -hi oJ the [oHm illg applies to his cM1-dition

(A) It is 31so seen ill p4titnts with aganghonic megacolon

(11) ft reuirs from a fal1ure of resorption of theviteUine d let

(C) It results from herniation at the-site of regression of the right umbilk vein

DJ It is caustd by faihtrc of recanalization of the midgut part of the duodenum

~ It ill camioo by a failuIt vf the midgul to return to the abGQminal uity after herniashytion in-n the urnbilk s l stalk

Ot er than the spleen occlusion Cif the spit-Ilk artery at its odgin wm most likely affect die blood supply to jllch st cnud

(A) Jejunum

(B) Body of th pal1~lltas

(C) LeSStT Cllmiddotlaturc of tl )toma-ch

(D Duodenum dista to the entrance of the Ornmou bile duct

E Fundus of the stomach

A 38-yeu-old batL~er with a history of heartburn suddenly experiences excluciating pain in the (plgastric region of th~ abdomeu SurgCry is perf~rme immediard y upon admisshysion to the 1IlcrgCJliy tuomh~re i~ evidence uf a ruptured ulcer in the posterior waU of the stomach Vhere will a surgeon first fi nd the stomach contenlSf

A) Greater p4ritoneal sac

rB) Cul~de-s~c of Douglas (--

C Omental bursa ~

--D) Paracolic gutter

rEj Between -he panttal perimltum and the posterior body wal1

At birth an infant presents with a st()ma~ rb~tbas~njJled jfltotb~diaplfagru 1A1ltre is the defect thatresulied iiitJle heini~t()n shy~tsophagealbiatus

7 - rH-- Hiatus for the inferior vena cava

( Pleuroperitoneal membrane -(0) Septum transvcrsum

(E) Right Crlt~

An infant born with DOVv7l syndrome presents with bili()u~ vomiting Ahat congenital defect does the infant have

(A) Pyloric stenosis

(B) Meckel diverticulum C) Ornphaloce1e

(D) Gastroschisis

( ~ ) Duodenal atresia y A patient with cirrhosis of the liver presents with ~ bacalvaricestnlreased retrograde pressure in which veins caused the varices

(A) Paraumuilical

(B) Splenic

(ct AzygltJus

(15))G~trk ( (-F) Superior mesemeric

A htaltby 3-year~old male patient experiences a hernial sa protruding from the anterior abdominal wall about halfway between me anterior superior ilia spine and the pubk tuberde Pulsations of al1 artery are palpated medial to the protrusion site through the abdominal walL Which layer of the anterior abdominal wall will first be traversed by the

1hctma

fA) Rectus sheath (B) External oblique aponeurosis

(C) Inguinal ligament

lD) Transversalis fusda

(E) Cremasteric fa~cia

After 5urgi(aj ffpair of a hernia the patient tXperienccs mtmlgtness in the skin on the anteshyrior aspect of the S(Totum_ Vhaf nerve may have been lesioned during thehemiorrhaphy

(A) Femoral

(B) Obturator

(C) Ilioinguinal

(D) lliohypogastrk

(E) Pudendal

A 23~year-LJld female secretary il1 good health ~-uddcn1) doubles over with pain in the a ea of the 1JmbRicu$ Sbe feels vartn and ltneasy and has no appetite That night the pain seems to have mQved to the tower right abdominal regjol1 and she calls her family doctor who then arranges for an ambulance to pk-k her up and take her to the hospitaL Wh ell ntn~ perceived in the area of the urnbilirus most Hkely carried lhe pairfu I sensations into the eNS

tA) Vagus nerves I~

V B)

) Lessersplanchnk nerves

tC) Pudendal nerves

(D) lIiohpogastrk nerves

(E) Greater splam ic l erves

A CT reveals carcinoma in the bOod of the ancreas Vhich blood vessel trut ourses ----~- - -bull ------ --shy

immediately poftterior to the body ofthe pancreas is the m~t likely to be oompressed

(A) Splenk artery

(B) Abdominal aorta (C) Portal vein

(1) Splenic vein

(E) Renal vein

A patient has a penrln1l1ng uker of the posterior wall ot the br~l part ot the (lUooenmn llkh blood vessel is subject to erosion

(A) Common hepatic artery

(B) Gastroouodenal artery

(C) Proper hevatic artery

(D) Celiac artery

(E) Anterior inferior 11amrelltlcoduodcnal attery

Your patient has been diagnosed -ith a carcinoma locallted to the head and l~e(k of the pancreas Another clinical sign would be

A esophageal varices

(8) hemorrhoids

C) a caput medusa

(D) increased pra Teuro n th~ hepatic veins

(E) enlarged right supra lavkular lymph nodes

Wltkh of the foUowing structures develops in the ventral mesentery

(A) Spleen

(B) Jeiunum (C) Head of1ht pancreas (D) Transverse colon (E) Stomach

ti l Uw ~ littwin~ f( S-t lil oai Imdge ~ hi(h or tbt la~)d J truetur tgt liJ llntn nl) he hl p UC iJd [IIi ell

c o

A) drains Ie tht infCrior a La aI

R t middot~nfl0 ~ill to th~ lunlgtn of h i dtlndCrlllfH

(e) m t bull JiJattd on tl l J n T ~H

D ) sup Lc O VSlt I Hlid bhtu l 1 li - -I un oid

( ) U~tpli(t tr j middottUh~ 1 v(( b~nt rfK n1ilc~Zm

ANSWERS AND EXPLANATIONS

Answer E The spleen is t hlttnopodicand lymph organ demlted from mesoderm

Answ~ R Al1 tlmphalocele is caused by it failure of the nlidgut to return to the ahdomir nat cavity after herniation into the umbiliau Stalk Choices Aand D maybe seen in infants with Down syndrome choice D ~s the specific CBuse ofduudcnal JtiCSitt Choice C is (ile cause of gclstrosbisis and Choice B nsults iu a Meurolktldivertku1-tlB

Answer B The fundus ofthe stomach is suppHed by soort gastric brunches of the splenic altery The splenic artery supplies the body and tail of the pancreas part of the greater curvature of the sttmla(h and the spleen Te jejunum part of the head of the pancreas and tht~ duodenum distal to the entrance of the commOll bile duct are supplied by the superior mesenterk artery clll~l ~be less r ctlt1ature cmd the pylQric antrum are supplied by the right and lei gastric art(ries

AnSWftt C Tbeomental bursa or lesser ~ritoneaj sac lies direcdy posterior to the proxshyimal part of the duodeTtlm and the stomach and would be the first site where stomach contents ~Ott1d be fpoundluncL

Answer C A defect in a llleuropcritoneal membrane (uswlly the left) is the typical site of i1 cc-ngenitlI diilphragluatic hemia llere the membr4ne fails to dose ()pound( of the perishycCirdiopcritulleal canals

Answer E DuoJenal atresia and aganglionic megacoion are congwitaI defects S~Il in patients with Dowmiddotnsyndrome

Answer D RulaTgemt~llt of and retrograde flow in g~lstrk vel_ns in particlJl~r the kft gas~ tricveins dilates the capillary bed in rhe wall of the esophagus in (ases of porta yper~

tension Blood flow would increase in and dilampte tribntarkgts of the (lZygOUS vein on the other side of the capiUary bed but flow in this vein is in the typical direction t()ward the superior vena cava Paraumbiii(ltU vein eilgorgement contributes to a caput medusH Splenic ~nlargement might prc~nt with 5plcnonlegaly and balt-kflow in to tlu superior m~~ntclic vein occurs but is asymptomatic

Answer D The patient hagt an indirect inguinal hernia whi~h emerges from the antt-rior abdominal wall through the deep inguinltilling Theeep ring is a fault in the transv~rshysaUs fascia this I~yer wiIJ be penetrated first by the hernia

An~Wer C The ilioinguinal nenc which provides sens~llion to the lnedlal thigh ltmclanteshytior SClotunl pass~lt th rough the 5uperfh_ial inguinal ring ind $subject to inj i1T) becaus-e

it is in the operatitm Held of the erniorrhapny

Auswer B The leMHr splanchnic nerves are sympathdic nerVlts that carry viscera l sensashytlltgtrogt ftom illtllt1m~d ()J stietched gust (itinteitinal ~tructures (in this case the pprndix) into tnt eNS Lesser splanchnic ntTYcsarisc from thmiddot T9--T12 spinal cord segments lt1nd provide sympathetic innenation tD rnidgut siruc1ures whiCh include CLe app~JldD Viscera] Pain arising from affecLed Inidgut ampt 1C1ure is referred over the same dl- matorne~ of spinal segrnertts v-hich provide the sympathetic Innervation n this G1SC of appendicitis the invohen~n t of the ltire) of t e unlhHku indud s the T 10 dermatome

Answer B Of the five choices onty the dscending olon is retroperiton~al aldwould be a lik ~ ( choice to be seen immediately a(~jilcent to t11e posterior abdominal middotn~L

Amwen D The SpltftlC ~-ein ourses posterior to the body of the panneas m its way tt drain into the superior mCSfttltlri( vein

Answcr B TILt glstrodllolticnal artery 1 direct hIamh of the comrootl hepatic artery courses immediately pt))iwri() to the duodenum and is slbject to erosion

Answer B Carcinoma of th pan middott3S in the 1tilt1 may compreampgt the portltil vein at irs orishygill The poTtai vcin is fomled when the splenic vein jQiaswith tfie superior meStllt eric vein The inferiot mesenteric vein joins the ~plenjc vein just priOT to tlli~ point at which the splenic joins the superior Jlleit1ltcri( vein Increescd venous presslu in the inferior mesenteric vein is a cause of emo hoid~

Answer C The- velltral pancreas wilich forms most of the head of the p ~ncr as develops in the ventral mes(ntery as antutgrowth of the hepatic diverticulum Th~ hepatic divershyticulull induding the biIJary appa~atus develops in tbe ventral mesentery of the foregut

Answer~ A The superior mesenteric ~in joins with the spienkvein to form the hepatic portal vciu

Answer D The structure at gttlK is the proper hepatic artery~ whkh suppUesoxygenated b middotood to the liver

MAKE SURE YOU KNOW the diff bw Rectus Sheath above and below the arcuate line

ABOVE

Aponeurosis of xiiltmal obllque musclo

Extemll f)biquw musde

Reotln ilbdomlnls musole S~in

Internal 9bliquQ mY~QI

AponeUfOsi$ of hJH$V~~S Lir9a a lb lbdolTlin~ musolo Tri OJV6 rUi

atldomlnis mUS(loe

Sub cutanlilous tiue (tatty ye r)

BElOW

A POrl lJfosis 01 etemal oblique muscle

Aponeul~)sis 01 Internal oblique mU$cl~

Anteriol lay~ of r~ltdus st~ath EXttom1 oblique rnu$cll

Rectus Jbdominis muscle Intoernal Aponeurc-sis of tra~fersU$ oblique muscle-

at-domlnis muscentl ~ Skio

Tra nsvitSus abdomioLs ml)ZClt

TralSVersaHs fascia Medial umQil iegtt1 1i9Jment -and folj

Uldchus Peritoneum (ir median Umbilj~al Suboutane ous

Extraprftone 11ascia

Ymbilimiddot~1 fold)

preu9poundiea1 fascia

tissue (fatty 4nd m~mbr3n(iUS layers)

o Above the arcuate line (A horizontal line 13 of the distance bw the umbilicus and the

pubic symphysis) -10 Aponeurosis divides into an AntPost Laminae

o The Ant Laminae joins EO and Post Laminae joins Trans Abdominis = Ant and Post

RECTUS SHEATH respectively

o BElOW the arcuate line - all 3 aponeurosis join ANTERIOR to rectus muscle to meet its

counterpart in the midline (linea Alba)

o Take away Msg - The abdomen is devoid of a posterior rectus sheath below the

arcuate line and is therefore more vulnerable to herniasinjuries

Question - A physician makes a deep incision in the patients midline immediately superior to

the pubic symphysis which of the following layers is his knife least likely to pass

Rectus Abdominis External Oblique Ant Rectus Sheath Posterior Rectus Sheath All of the

Above

Answer - All of the above None of the other answer choices are midline structures -LINEA

ALBA

Linea Alba has very poor blood supply - doesnt heal well after surgery Therefore this is a

common site for incisional hernias

a Spleen b Transverse colon c Descending colon d Stomach e Pleura

17 Meckels diverticulum is normally found 2 feet proximal from the

a Pyloric sphincter b Lower esophageal sphincter c Ileo-cecal valve d Middle valve of Huston e Anal valve

18 Ulcer in the posterior wall of the first part of the duodenum would erode ___ artery and would cause bleeding

a Left gastric b Right gastric c Hepatic artery proper d Gastroduodenal artery e Middle colic artery

19 An inflamed appendix is identified by a surgeon on the operation table by noting

a The appendicies epiploicae b The convergence of tenia c The artery of Drummond d The mesocolon e The mesosalphinx

20 The nerve which emerges through the psoas major is

a Femoral b Ilio-inguinal c Ilio-hypogastric d Pudendal e Subcostal

21 The right gonadal vein drains into the

a Azygos b Hemiazygos c Inferior Vena Cava d Right renal vein e Left renal vein

22 The hepatocytes in the liver is derived from

a Ectoderm b Endoderm c Mesoderm

d Neural ectoderm

23 Abscess in the lumbar vertebrae due to tuberculosis would spread to the adjacent muscle which is

a Psoas Major b Iliacus c Quadratus lumborum d Tranversus Abdominis

24 The anterior wall of the inguinal canal is formed by

a External oblique and transverses abdominis b External oblique and fascia transversalis c Internal oblique and external oblique d Internal oblique and transverses abdominis e Fascia transversalis and peritoneum

Meckels diverticulum is a result of which of the following developmental abnormalities shy

A Failure of the vitelline duct to close

B Failure of the herniated intestinal loop to retract into the abdomen

C Failure of the urachus to close

D Failure of the midgut to rotate

E Failure of the hepatic duct to close

Explanation

Meckels diverticulum is a result of the persistence of the proximal part of the vitelline duct This

diverticulum is usually found about 2 feet proximal to the ileocecal junction and is usually about 2 inches

long It is present in about 2 of the popUlation It may be the site of ectopic pancreatic tissue or gastric

mucosa and may develop inflammatory processes and ulcerations Acute Meckels diverticulitis

simulates appendicitis

Which of the following veins carries blood from the esophagus to the portal vein The

A right gastric vein

B left gastric vein c splenic vein D azygos vein

E left gastroepiploic vein

Explanation

The left gastric vein a direct branch of the portal vein drains blood from the lesser curvature of the

stomach and the inferior portion of the esophagus Because branches of the portal vein do not have

valves blood can flow in a retrograde path when there is an obstruction to flow through the portal system or liveL Rlooci Cln then flow from the nortl] vein thr()1Ph the left PRstric vein to the esonhlPlIS lno

through venous communications within the submucosa of the esophagus to esophageal veins that drain

into the azygos vein The increase in blood flow through the esophageal submucosal veins results in esophageal varices

On the posterior wall of the abdomen the celiac ganglion A contains cell bodies of postganglionic parasympathetic neurons B is synapsed upon by neurons in the posterior vagal trunk C is synapsed upon by neurons in the greater splanchnic nerve D contains sensory cell bodies of lumbar spinal nerves E contains cell bodies of neurons that cause an increase in the rate of peristasis

Explanation The celiac ganglion is one of the preaortic ganglia of the sympathetic nervous system It contains cell bodies of postganglionic sympathetic neurons The sympathetic splanchnic nerves contain preganglionic sympathetic neurons that pass through the sympathetic chain without synapsing These splanchnic nerves go to the preaortic ganglia to synapse The greater splanchnic nerve contains preganglionic neurons from spinal cord segments T5-T9 This nerve synapses in the celiac ganglion The nerve fibers in the vagal trunks are preganglionic parasympathetic fibers that go to the walls of the organs that they will innervate and synapse on postganglionic parasympathetic neurons in the walls of those organs Cell bodies of sensory neurons in the abdomen are found in the dorsal root ganglia or the sensory ganglia of the vagus nerve Sympathetic innervation decreases the rate of peristalsis parasympathetic innervation increases the rate of peristalsis

Which of the following pairs of arteries will allow blood to bypass an occlusion of the celiac trunk

A Left gastric artery-right gastric artery

B Left gastroepiploic artery-right gastroepiploic artery

C Superior pancreaticoduodenal artery-inferior pancreaticoduodenal artery

D Splenic artery-common hepatic artery

E Left gastric artery - proper hepatic artery

Explanation The anastoOlosis of a branch of the celiac trunk and a branch of the superior mesenteric artery will

provide collateral circulation around an occlusion of the celiac trunk Each of the other choices pair

branches of the celiac trunk therefore these will not provide collateral flow around the obstruction of the

celiac trunk The left gastric splenic and common hepatic arteries are direct branches of the celiac trunk

The right gastric artery is a branch of the proper hepatic artery which is a branch of the common hepatic artery The left gastroepiploic artery is a branch of the splenic artery The right gastroepiploic artery is a

branch of the gastroduodenal artery whlch is a branch of the common hepatic artery

Which of the following organs has appendices epiploica The

A sigmoid colon

Bjejunum

C duodenum

D stomach E esophagus

Explanation Appendices epiploica are characteristic of the colon Appendices epiploica are subserosal accumulations

of fat None of the organs of the gastrointestinal tract has appendices epiploica except the colon

Page 11: Chirag's Abdomen Review

bull

bull External spermatic fascia derived from external obliques EO II Cremasteric fascia ~ from internal obliques fO bull Internal spermatic fascia derived from fascia transversalis bull Tunica vaginalis derived from processes vaginalis directly rests on testes bull know order from testes out to skin

note reflex o ilioinguinal nerve o Efferent =genital branch of the genitofemoral nerve

--lt gt-- info important anastamoses which connects thorax to abdomen

bull Sup

o Sup epigastric branch of internal thoracic o Inf branch external iliac

Venous drainage o Above umbilicus aXillary v o Below umbilicus veins in triangle o At level of umbilicus Paraumbilical veins -gt drain into the portal V

II Important in Portal Caval Venous system Venous drainage of testes

o Clinical correlation Varicocele 11 vein drains into IVC 11 Left testicular vein ~ drains into left renal v

bag of rmlt

for lymph drainage T10 axillary lymph nodes

ill Below T10 superficial inguinal lymph nodes (lateral

Umbilical Folds

Lateral umbilical folds inferior vessels

Medial umbillcial folds umbilical (fetal remnant)

Median umbilical fold urachus (fetal remnant)

Between these folds fossas o Supervesical fossa between median and medial folds

11 bladder o hesselbachs between medial and I folds

II DIRECT HERNIAS HERE Borders

Medial semilunar line

Lateral info Epigastric

Inferior inguinallig o Lateral Inguinal Fossa beyond lateral fold

INDIRECT HERNIAS HERE II Deep inguinal ring (lateral to inferior epigastric a)

Indirect inguinal hernia o Lateral to inferior epigastric a o more common o When inserting finger in superficial inguinal ring will feel on tip of finger (since it goes

throueh ineuinal canall

----

Dry Lab - Label subcostal iliohypogastric Ll Ilioinguinal (Ll)

Horesshoe Kidney - stuck under IMA

Renal Agenesis -failure of the ureter bud to develop

Double Ureter

Unilateral Agenesis -1 kidney

Kidneys - Metanephros

Fetal kidneys are at sacral level

Look at 3rd part of duodenum

Some of this stuff is repeated I know just copied and pasted a bunch of stuff I had copy

Dermatomes

bull T4 nipples

bull no umbilicus v o Pain referred to no in appendicitis o Pain referred to T7ITS in gastritis ~

Inguinal ligament = external abdominal oblique aponeurosis

bull Inserts at anterior superior iliac spine to the pubic tubercle o Why important to know -7 visualizing this line allows us to properly diagnose a hernia

Below the inguinallig femoral hernia Above the inguinallig =inguinal hernia

Also to palpate the deep inguinal ring you go about 12Scm above the mid-inguinal

point

bull Modifications to ligament o Pectinate ligament o Lacunar ligament -7 cut this ligament to relieve strain i~ stran ul~tEd hernia

Inguinal canal

bull in males -7 transmits spermatic cord o important structures of spermatic cord ductus deferens testicular a genital branch of

the genitofemoral n pampiniform plexus of veins bull in females -7 transmits round ligament

Borders

bull Floor -7 inguinal ligament + lacunar ligament bull Anterior -7 aponeurosis of external oblique + internal oblique bull Roof -7 internal oblique and traverse abdominal bull Posterior -7 transverse abdominal + transversalis fascia

o Reinforced by conjoint tendon bull Aponeurosis of internal abdominal obliques and transverse abdominus bull Lies immediately behind the superficial inguinal ring in what would otherwise be

a weak point in the abdominal wall bull Innervated by ilioinguinal nerve (Ll) ~why important

bull In appendicitis Ll can be injured which will injure this nerve and in turn

the conjoint tendon With loss of innervation to this supportive structure the patient is now predisposed to a direct inguinal hernia

o Only hernia that can transverse the inguinal canal o Associated with congenital condition persistent tunica vaginalis

bull Direct inguinal hernia o Medial to inferior epigastric a o When inserting finger in superficial inguinal ring will feel on back of finger o Associated w old age or recent surgery

Muscles (Abdomen RECTUS SHEATH)

bull Arcuate line at level of ASISor 13rd distance between pubis and umbilicus bull Above arcuate line rectus abdominus is surrounded by a rectus sheath anteriorly and

posteriorly

o EO and 10 lie over rectus abdominus o 10 and TA lie behind rectus abdominus

bull Below arcuate line rectus abdominus has no rectus sheath posteriorly o EO 10 and TA lie over rectus abdominus o Transversalis fascia lies behind rectus abdominus o Inf EpIgastric vessels pierces the rectus sheath here

Peritoneum serous sac which encloses most of the abdominal structures

bull Ovary =only intraperitoneal organ o Oocyte ejected from ovary then captured by fallopian tubes o Why impt Women more prone to infection that can enter peritoneum

Peritoneum forms

bull Mesentery double layered fold of peritoneum formed as the organ was pulled in

bull Ligament between 2 organs in general bull Omentum between stomach and another organ bull Bare area area of no peritoneum bull

Viscera innervation

bull Pa rasympathetics 11 o Afferents sense hunger o Efferents l peristalsis relaxes sphincters gland secretion

bull Sympathetics o Efferents do opposite o Afferents CARRY PAIN SENSATION OF THE VISCERA (dull stretching pain)

bull PARASYMPATHETIC INNERVATION o Vagus nerve 7 _1l to 23rd unct ion of la rgej nte~tine oJ)elVrcspla~~~)~~ic~rYe~ IJiU- ~rd aJ~lpoteotiD~ IMPT

Gut Embryology

Gut ~ We say that the gut is derived from endoderm We often forget that when we say so we mean

that only the mucosa is derived from the endoderm The submucosa and the muscle layer is actually derived from the splanchnopleuric mesoderm and the serosa is derived from the visceral peritoneum

~ The main function of the gut is to digest the food which is done by the glands derived (and are) in the mucosa (endoderm) The only two exceptions in the Gut where glands though derived from the endoderm do not stay there but migrate down into the submucosa are esophagus and duodenum These glands however have their ducts opening to the swface of the mucosa

bull

~ Lungs liver amp gall bladder and pancreas are off-shoots from the foregut Esophagusshy~ The region of the tube from the laryngeal diverticulum to the beginning of the stomach elongates

to form the esophagus ~ The glands which form in the endoderm (mucosa) migrate down into the submucosa The path

whlch it took migrate becomes the duct of the glands which open to the mucosa ~ Achalasia Cardia - Failure of relaxation of the lower esophageal sphincter because of congenital

absence of ganglia at the sphincter (The ganglia when present releases VIP (Vaso-IntestinalshyPeptide) which relaxes the sphincter)

Mid-Gut Rotation ~ Because of the 90 degree rotation of the primitive stomach all of the following events occur ~ Lesser curvature comes to the right Therefore lesser omentum also comes to the right ~ Greater curvature comes to the left Therefore greater omentum also comes to the left ~ Right side vagal trunk becomes posterior vagal trunk ~ Left side vagal trunk becomes anterior vagal trunk ~ The left side peritoneal cavity comes to the anterior aspect of the stomach and will later be called

as the greater sac ~ The right side peritoneal cavity comes to the posterior aspect of the stomach and is (relatively a

small sac because the liver is on the right) called the lesser sacomental bursaepiploic sac ~ Epiploic foramen of Winslow (the lower free margin of the ventral mesentry) wiII be the

communication between the greater and lesser sac ~ The Liver moves to the right and therefore actually causes the 90 degree rotation of the stomach

The spleen comes to lie on the left side ~ Axis Antero-posterior axis around the superior mesenteric artery

bull Counterclockwise bull Approximately 270deg bull During herniation (about 90deg) bull During return (remaining 180deg)

Duodenum ~ Becomes retroperitoneal (except the first part which is still suspended by the hepato-duodenal

part of lesser omentum) ~ Glands (of Brunner) go submucosal ~ An imaginary line drawn below the opening of the major duodenal papilla represents the junction

between the foregut and midgut ~ Duodenal atresia in Downs syndrome Liver ~ 3rd week

bull liver bud grow bull into the septum bull transversum

~ 10th week bull hematopoietic bull function

bull 10 of the total bull body weight

~ 12th week bull bile is formed

Pancreas ~ In about 10 of cases the duct system fails to fuse and the original double system persists ~ 3rd month

bull pancreatic islets (Langerhans) ~ 5th month

bull Insulin secretion ~ Annular pancreas

bull The right portiCn of the ventralbud migrates along its normal route but the left migrates in the opposite direction

~ Complete obstruction of duodenum ~ Accessory pancreatic tissue Polyhydramnios (Amniotic fluidgt 1500-2000 ml)

~ Congenital defects including central nervous system disorders (eg Anencephaly) and gastrointestinal defects (atresias ego Duodenal esophageal) prevent the infant from swallowing the amniotic fluid (failure of recanalization)

Oligohydramnios (Amniotic fluid lt 400 mt) ~ Cl~ldberenal-agenesis

bull Midgut _-_

~ Primary Midgut intestinal loop gives rise to bull Distal duodenum bull Jejunum bull Ileum bull Ascending colon bull Transverse colon - proximal two-thirds of the bull Transverse colon with the distal third

~ Primary intestinaltoop bull ncephalic limb distal part of the duodenum the jejunum and part of the ileum bull ncaudal limb lower portion of the ileum the cecum the appendix the ascending colon and

the proximal two-thirds of the transverse colon bull 6th week

bull Rapid elongation of the cephalic limb bull Rapid growth of the liver bull Intestinal loops enter the extraembryonic cavity in the umbilical cord

bull 10th week bull loops begin to return bull regression of the mesonephric kidney reduced growth of the liver expansion of the

abdominal cavity bull Jejunum -left bull Loops - more to the right

bull Cecal bud -last part (temporarily below the right lobe of the liver) ~ qIDlthaloseJe (Structures COlHLoArts9V~1tion)

bull Through umbilical ring bull 6th to 10th weeks

bull Associated with a high rate of mortality (25) and severe malformations bull Associated with chromosome abnormalities

~ Gastroschisis (Structures coming out are not covered by Amnion) bull herniation through the body wall ----=---=-shybull Into the amniotic cavity bull Lateral right of the umbilicus bull Sometimes the inferior wall fails to develop as a result lower abdominal structures like the

bladder would be exposed to the exterior not associated with chromosome abnormalities ~ Abnormalities of the Mesenteries

bull Mobile cecum persistence of mesocolon bull Extreme form - long mesentery bull Volvulus

~ Distal third of the transverse colon ~ Descending colon ~ Sigmoid colon ~ Rectum ~ Upper part of the anal canal ~ Primitive anorectal canal

bull 7th week cloacal membrane ruptures bull Tip of the urorectal septum perineal body bull Pectinate line

~ Hindgut anamolies bull Rectoanal atresias and fistulas bull Imperforate anus bull Congenital megacolon (aganglionic megacolon Hirschsprung disease)

bull

bull Hindgut

Chirags Abdomen Review - Part 2

Understanding Embryo makes learning blood supply EASY

I I

I t

~ -

)

Table l1r-~ L Adult SUmiddotuctu~SDrj~l Froln Each of he Three Dhisions of be Pringttive GUl Tube t-middot-----middotmiddotmiddotmiddot-

Foregu(

I_ (Celiac Trunk)

Ir-slt-gtphgus

S101na(b

I h -= LiJ~r

Pancre=l S

bull 1 i Biliary apparntu5

Gall bladdshy

i Pha11~Cal pltgtuchcs

LullSS-I

Mjig ---- bull __ _- ----n--duct----~---middot-------l--n

(Superior Jldesen1eric Artery)_-1I-(I_~__ middot __ O-=-)_in_middoto_r_M_e_se_n_t_e_r_i_c_An__

Uuodenu rn 2nd_ 3 lt141h V4Tt

Jejunun-~

nc-un]

tCCUJ11

AppltgtndLX

Transver5e -o1on (p~oxiln1l1 ~O Tbird)

bull__hytgtid~ _ _ ______ L _ __

Tr-dn~llt~se colon (diStul h lTd) I

)

i

Aa ca-nal -( uppeT patt) i

I I

_____ __ _ _ _ ___ - - rhe~ a(t clcriVOkt iV(5 opound~lt prbn1rC ~ nlQC blft TlI)( 134tof r~ tIonoinf~ i 1 ~l l1rd c- P Cle

Now Lets see how much youve learned

Questions

1) A pt receives a general anesthetic in preparation for a c~t~~my A right subcostal incision is made which begins near the xyphoid process runs along and immediately beneath the costal margin to an anterior axillary line and transects the rectus abdominus muscle and rectus sheath At the level of the transpyloric plane the anterior wall of the

-~~-~=--- _eco---shysheath of the rectus abdominus muscle receives contributions from which of the following

a Aponeuroses of the in~ande~tef-Ilal o~ues

b Aponeuroses of the transversus abdominis and internal oblique muscles c Aponeuroses of the transversus abdominis and internal and external oblique

muscles d Transversalis fascia e Transversalis fascia and aponeurosis of the transversus abdominus muscle

A

2) The lat~raJJJ11QjJt~gLfgJlLoneach side of the inner surface of the anterior abdominal wall is created by which of the following structures

K Falx inguinalis (~) Inferior epigastric a

c Lateral border of the rectus sheath d Obliterated umbilical a e Urachus

B

3) A man the victim of several knife wounds to the abdomen during a brawl at the Lobster Shack subsequently developed a direct inguinal hernY Damage to which of the following nerves is most likely responsible for the predisposing weakness of the abdominal wall

~ Genitofemoral nerve ( b) Ilioinguinal nerve ~-t Tenth intercostal nerve

d Subcostal nerve e Pelvic splanchnic nerve

B

4) Which of the following statements concerning a direct inguinal hernia is correct a It is the most common type of abdominal hernia b It transverses the entire length of the inguinal canal c It contains all3 fascia layers of the spermatic cord d It exits the inguinal canal via the superficial ingeJinal ring e It protrudes through H~acb strJg e

~(

1fltbS w E

tl

5) The conjoint tendon is

a Important in preventing indirect inguinal hernias b The fused aponeurotic layers of internal abdominal oblique and transversus

abdominus muscles c Posterior to the deep inguinal ring

d Medial fibers of the inguinal ligament

B

6) A 25 year old male is brought in to the ER after being involved in a car accident in which he received a crushed internal injury in his abdomen Examination reveals a lesion of parasympathetic fibers in the vagJsnerve which interferes with glandular secretory or

smooth muscle functions in which of the foliowingorgans a Bladder b Transverse coloiW c Descending colOO d Prostrate gland e Rectum

B

7) The spermatic cord includes all of the following contents except a Il ioinguinal nerve b Pampin iform plexus of veins c Vas deferens d Genitofemoral nerve

A

8 Which abdominal structure gives rise to the internal spermatic fascia (muscle) following the descent of testes in development

a External abdominal oblique aponeurosis b Transversalis fascia c Transversus abdominis muscle d Peritoneum e Internal abdominal oblique

B

9 Which abdominal structure gives rise to the tunica vaginalis fotlowing the descent of testes during development shy

a External abdominal oblique aponeurosis b Transversalis fascia c Transversus abdominis muscle d Peritoneum e Internal abdominal oblique

D

10) The lesser omentum is a peritoneal fold which is su bdivided into the a Hepatogastric and gastrosplenic ligaments b Hepatoduodenal and gastroomentalligaments c Hepatoduodenal and gastrosplenic ligaments d Hepatogastric and hepatoduoden9-jrj igaments

D

11) A posteriorly perforating ulcer in the pyloric antrum of the stomach is most likely to produce initiallocalized peritonitis or abcess formation in which ofthS fQllowing

a Great-sac - -- -

b Paracolic recess

c Omental bursa

d Right subphrenic space

c

The inferior mesenteric artery arises from the abdominal aorta ilm_ediill~y_J-Qs1eriQLto which of the foowing org~ns A-F~t~filie duodenum B Head of the pan~eis C Neck of the pandeas

D Second part of the duodenum

E Third part of the duooenum_shylaquoshy

shy

The correct answer is E The inferior mesenteric artery arises from the anterior surface of the aorta at the level of the third lumbar vertebra The third part of the duodenum crosses the midline at the level of the third lumbar vertebra and passes anterior to the aorta at the origin of the inferior mesenteric artery The

first part of the duodenum (choice A) lies horizontally to the right of the midline at the level of the first

lumbar vertebra The head of the pancreas (choice B) is to the right of the midline and extends from the

level of the first lumbar vertebra to the third lumbar vertebra It lies within the concavity of the

duodenum The neck of the pancreas (choice C) lies in the midline at the level of the first lumbar

vertebra It lies on the anterior surface of the aorta at the origin of the superior mesenteric artery The second part of the duodenum (choice D) lies vertically to the right of the midline and extends from the

level of the first lumbar vertebra to the level of the third lumbar vertebra

The left adrenaLvein drains directly into which of the following veins A Hemiazygos vein

B Inferior vena cavaee C Left renal veiri -

D Splenic vein

E Superior mesenteric vein

a

The correct answer is C The left adrenal vein and the left gonadal vein (either testicular or ovarian) drain into the left renal vein TheTeft renal vein t~ains intothe- inferior vena cava In contrast the right

adrenal ~~inandnght gonadal veindrai~ gLr~ctJy iQtoJhe iilferiQ[ Vencava -- -

ThehemTazygoS7ein- (~h-~i-~ A)~~c~i~es the venous drainage from the body wall on the left side of the

thorax and abdomen No visceral organs drain directly to the azygos or hemiazygos veins The inferior vena cava (choice B) receives the direct venous drainage from the right adrenal vein but not

the left adrenal vein Remember the inferior vena cava is on the right side of the abdomen The splenic

vein (choice D) receives the venous drainage from the spleen and part of the pancreas and stomach The splenic vein is part of the portal venous system

The superior mesenteric vein (choice E) receives venous drainage from much of the intestinal tract It is part of the portal venous system and joins with the splenic vein to form the portal vein

A 43-year-old man presents complaining of pain in the groin On examination his physician palpates a

bulge in the region of the superficial inguinal ring which he diagnoses as a direct inguinal hernia The hernial sac most likely

A is covered by all three layers of the spennatic fascia B passes medial to the inferior epi gastric artery

C passes medial to the lateral border of the rectus abdominis muscle

D passes posterior to the inguinal ligament E passes through the deep inguinal ring

The correct answer is B Direct inguinal hernias enter the inguinal canal by tearing through the posterior

wall of that structure The typical location for this type of hernia is through the inguinal triangle bounded

laterally by the inferior epigastric artery medially by the lateral border of the rectus abdominis and

inferiorly by the inguinal ligament Direct inguinal hernias pass medial to the inferior epigastric artery

whereas indirect inguinal hernias pass lateral to the inferior epigastric artery because the deep inguinal

ring is lateral to the artery Indirect inguinal hernias are covered by all three layers of the spermatic fascia (choice A) Direct inguinal hernias are covered by fewer than all three layers because the direct inguinal

hernia tears through one or more layers of fascia as it emerges though the abdominal wall The lateral

border of the rectus abdominis muscle (choice C) forms the medial border of the inguinal triangle All

inguinal hernias pass lateral to the rectus abdominis Femoral hernias pass posterior to the inguinal ligament (choice D) Inguinal hernias emerge through the superficial inguinal ring which is superior to the inguinal ligament Inguinal hernias that descend below the inguinal ligament pass anterior to the

ligament Indirect inguinal hernias pass through the deep inguinal ring (choice H) direct inguinal hernias

do not Both types of inguinal hernias pass through the superficial inguinal ring

During a gastric resection in a patient with stomach cancer a surgeon wants to remove the lesser

omentum because of tumor extension into it Which of the following structures lie in the free edge of the

l~~g omentum and consequently must be dissected out in order to be preserved

A Common bile duct cystic duct and hepatic artery 6

B Cystic duct hepatic artery and hepatic vein

e Hepatic vein and cystic duct

Portal vein common bile duct and hepatic artery

E Portal vein hepatic artery and hepatic vein

The correct answer is D The free edge of the lesser omentum contains three important structures the

common bile duct the hepatic artery and the portal vein Nei ther the cystic duct (choices A B and C) nor the hepatic vein (choices B C and E) lies in the free

edge of the lesser omentum

A 55-year-old male patient with chronic liver disease has portal hypertension To relieve the pressure in the portal system a porto-caval shunt is performed Which of the following veins may by anastomosed to

accomplish this porto-caval shunt A Left renal vein-left testicular veingt

B Right renal vein-right suprarenal vein I shy

e Splenic vein -left renal vein J

D Superior mesenteric vein-inferior mesenteric vein E Superior mesenteric vein-splenic vein

The correct answer is C The splenic vein drains directly into the portal vein The left renal vein drains

directly into the inferior vena cava Anastomosis of these veins would allow blood from the portal vein to

drain retrograde though the splenic vein into the renal vein and then into the inferior vena cava The left

renal vein (choice A) drains directly into the inferior vena cava The left testicular vein drains directly into

the left renal vein Thus these veins are already in communication and neither vein is part of the portal venous system The right renal vein (choice B) drains directly into the inferior vena cava The right

suprarenal vein also drains directly into the inferior vena cava Thus neither vein is part of the portal

venous system The superior mesenteric vein (choice D) drains directly into the portal vein The inferior

mesenteric vein drains into the splenic vein which then drains into the portal vein Thus neither vein is

part of the caval venous system The superior mesenteric vein (choice E) drains directly into the portal

vein The splenic vein also drains directly into the portal vein Thus neither vein is part of the caval

venous system

A 12 year old boy has fever vomiting and para-umbilical pain After examining the patient the doctor

makes an initial diagnosis of appendicitis Appendicular pain which is initially referred to the umbilicus goes to the dorsal root ganglion of

a TI b TI2 c L1 d T7

(e I TIO

A 59-year-old male undergoes a neurological examination which reveals that when the abdominal wall is

stroked the muscles of the abdominal wall of the side of the body stimulated failed to contract Other

neurological tests appeared normal The likely region affected includes

a CI - C5 spinal segments b C6 - TI c T2-TI ~T8-T12

e Ll- L5

The surgery done to relive portal hypertension is done by connecting two veins Which of the following veins would be suitable for connection

a Inferior vena cava and portal vein b Superior vena cava and portal vein c Splenic vein and right renal vein d Splenic vein and left renal vein e Superior mesenteric vein and Inferior vena cava

A mother brings her 3-week-old infant to the pediatric clinic reporting a new scrotal bulge that she found -~-

while changing a diaper yesterday The infant is afebrile Physical examination reveals a palpable mass in

the scrotum while in the standing position resolution of the mass in the supine position and no

transillumination of the scrotal sac What is the most likely diagnOSiS

a Cryptorchidism b Direct inguinal hernia c Hydrocele d Indirect inguinal hernia ~ e varicocele

The Vagal trunks enter the abdomen by passing through which of the following openings in the

diaphragm

a Right crus b Esophageal hiatus ~ c Vena caval hiatus d Aortic hiatus e Left crus

2 The anterior boundary of the epiploic foramen of Winslow is bounded by

a) First part of duodenum b) Lesser curvature of stomach c) Liver d) Hepato-duodenalligament v ~

3 The ilio-inguinal nerve is derived from

a TI2 ry b LI c L2 d L3 e L23

15 Surgically the structure used to suspend the kidney to the diaphragm is

a) Renal fascia b) True capsule c) Perinephric fat d) Paranephric fat

6 If there is portal obstruction because of carcinoma affecting the pancreas which of these of the

following signs would be present

a Caput medusae b Esophageal varices c Rectal varices c

d Pulmonary edema

7 In a sliding hernia the gastro-esophageal junction lies

a) At its normal position b) Below the normal position c) Above the normal position V d) None of the above

8 Which of the following structures is retroperi toneal

A transverse colon B spleen IJ2f6 C ileum D descending colon v r 1pound1111111

9 The renal angle is fonned lgtetween the 12th rib and ______ muscle

a Psoas major -middotshyb Erector spinae c Quadratus Iumborum d Diaphragm

10 The anterior structure at the hilum of the kidney is

a) Renal vein ~

b) Renal artery I middot~ I

c) Ureter d) Accessory renal artery

11 Because of origin of the muscle from the lateral one third of the inguinal ligament it

could not fonn the anterior wall of the inguinal ligament

a) External oblique b) Internal oblique c) Transversus abdominis_ d) Rectus abdominis

12 A large tumor mass impinges on the splenic artery and its branches as the artery pass out from below

the greater curvature of the stomach Branches o(which of the following arteries would most likely to

effected by the pressure on the splenic artery

a Left gastric b Left gastro-epipJoic c Right gastric d Right gastro-epipoloic e Short gastric_

13 A new born baby has projectile vomiting after each feeding It is determined that there is obstruction

of the digestive tract as a result of annular pancreas Annular pancreas is as a result of an abnormality in which of the following process

a Rotation of the dorsal pancreatic bud around the first part of duodenum b Rotation of the dorsal pancreatic bud around the second part of duodenum c Rotation of the dorsal pancreatic bud around the third part of duodenum d Rotation of the ventral pancreatic bud around the first part of duodenum y Rotation of the ventral pancreatic bud around the second part of duodenum

14 As the liver bud enters the ventral mesogastrium the region of the mesogastrium stretching from the

liver to the anterior abdominal wall is called

a Lesser Omentum b Greater Omentum ~ Falcifrom ligament d Lacunar ligament e Ligamentum teres of liver

16 A patient has absence of his 12th rib In such a patient if the doctor makes an incision to approach his

kidney mistaking the 11 th rib for the 12t he would end up injuring

Which of the following arteries is a direct branch of the gastroduodenal artery The

A right gastric artery

B left gastric artery

C inferior pancreaticoduodenal artery D left gastroepiploic artery

i E)right gastroepiploic artery --

E x pI a nation The right gastric artery is typically a branch of the proper hepatic artery The left gastric artery is a direct

branch of the celiac trunk The right and left gastric arteries anastomose along the lesser curvature of the

stomach The inferior pancreaticoduodenal artery is a branch of the superior mesenteric artery it

anastomoses with the superior pancreaticoduodenal in the head of the pancreas The left gastroepiploic

artery is a branch of the splenic artery it anastomoses with the right gastroepiploic artery along the greater

curvature of the stomach The right gastroepiploic artery is a branch of the gastroduodenal artery The

other branch of the gastroduodenal artery is the superior pancreaticoduodenal artery

Which of the following pairs of veins join together to form the portal vein The

A superior mesenteric vein and inferior mesenteric vein

B inferior mesenteric vein and splenic vein

C superior mesenteric vein and splenic vein

Ip)splenic vein and left gastric vein E superior mesenteric vein and left gastric vein

Explanation

The portal vein is formed behind the neck of the pancreas by the union of the superior mesenteric vein

and the splenic vein The inferior mesenteric vein drains into the splenic vein The left gastric vein drains

directly into the portal vein After the portal vein forms it enters the hepatoduodenalligament of the

lesser omentum to reach the liver The portal vein is the most posterior structure in the hepatoduodenal

ligament

At which of the following vertebral levels does the duodenum pass anterior to the aorta - _- shy

All ~

B L2 7~

CL3 I

~DL4

E L5

Explanation

The duodenum begins at the pyloric sphincter at the level of Ll The second (or descending) portion of

the duodenum is to the right of the aorta and extends inferiorly from the level of Ll to the level of L3 The third part of the duodenum crosses the aorta from the right side to the left side at the level of L3 The

fourth (ascending) portion of the duodenum extends from the level of LJ to the level of L2 The

duodenum ends at the duodenojejunal flexure The superior mesenteric artery passes anterior to the

duodenum as the duodenum passes anterior to the aorta The duodenum can be constricted at this level

In which of the following locations will perforation of the digestive tract result in the spilling of luminal

contents into the - lesser peritoneal sac

A Anterior wall of the second portion of the duodenum B Posterior wall of the second portion of the duodenum

C Anterior wall of the stomach

~Posterior wall of the stomach E Posterior wall of the transverse colon

Explanation

The posterior wall of the stomach is related to the lesser peritoneal sac The anterior wall of the stomach is related to the greater peritoneal sac The anterior wall of the second portion of the duodenum is related to the greater peritoneal sac The posterior wall of the second portion of the duodenum is related to the retroperitoneal space The posterior wall of the transverse colon is related to the greater peritoneal sac

The ureter lies against the anterior surface of which of the following muscles shyA Crus oftne diaphragm B Quadratus lumborum

0 Psoas major D Transversus abdominis

E Iliacus

Explanation The ureter exits the renal pelvis at about the level of vertebra L2 As it descends along the posterior abdominal wall it lies on the anterior surface of the psoas major The psoas major muscle arises from the bodies of the lower lumbar vertebrae The psoas major muscle is joined by the iliacus to fonn the

iliopsoas muscle The iliopsoas muscle then attaches to the lesser trochanter of the femur and is the major

flexor of the hip

As the right ureter passes the pelvic brim it lies against the anterior surface of which of the following

blood vessels

A Gonadal artery B Inferiorvena cava C Internal iliac artery

rJ- External Iliac artery

E Inferior mesenteric artery

Explanation

The ureter lies in the extraperitoneal space in the posterior abdominal wall Alter leaving the kidney it

passes inferiorly on the anterior surface of the psoas major muscle At the pelvic brim the ureter passes

into the pelvis At this point the common iliac artery is dividing into the external and iliac arteries The

ureter lies on the anterior surface of the external iliac artery immediately distal to the bifurcation This is a useful landmark for a surgeon to locate the ureter

When extravasated urine passes from the superficial perineal space into the anterior abdominal wall it is

found immediately deep to which of the following layers of the anterior abdominal wall

-ltScarpas fascia

B External oblique muscle

C Internal oblique muscle D Transversus abdominis muscle

E Transversalis fascia

Explanation

The superficial perineal space is bound by Colles fascia the fibrous portion of the superficial fascia This

layer of fascia is continuous with Scarpas fascia the fibrous portion of the superficial fascia of the anterior abdominal wall Therefore urine that is deep to Colles fascia will remain deep to Scarpa s fascia The urine will spread in the plane between Scarpas fascia and the external oblique layer

When a horseshoe kidney develops the ascent of the kidney is restricted by the A internal iliac artery B external Iliac artery

C common iliac artery

inferior mesenteric artery

E superior mesenteric artery

Explanation

A horseshoe kidney develops when the inferior poles of the to kidneys fuse together as they ascend into

the abdomen from the pelvis The first anterior midline vessel that is encountered by the horseshoe kidney

is the inferior mesenteric artery This artery prevents the kidney from continuing its ascent

The left testicular vein drains into which of the following veins

A Left internal iliac vein B Left common iliac vein

bflnferior vena cava D Left renal vein I

E Left internal pudendal vein

Explanation

The left testicular vein drains into the left renal vein The right testicular ~i~[~nsltjectlY into the

inferior vena cava This difference in venous drainage is believed to explain the greater incidence of

varicocele on the left side than on the right The venous drainage from the penis is to the internal vein

which then drains into the internal Iliac vein

The spinal nerve that provides cutaneous branches to the skin around the umbilicus is

A TS B TW-shy

C TI2

DL2 EtA

Explanation

The tenth intercostal nerve is the anterior ramus of the TIO spinal nerve After passing through the tenth

intercostal space the nerve continues forward in the anterolateral abdominal wall in the plane between

the internal oblique muscle and the transversus abdominis muscle In the abdominal wall the nerve innervates to the abdominal wall muscles as well as the skin and the parietal peritoneum The umbilicus is

a useful landmark for the region of distribution of the tenth thoracic nerve

The ligament of the vertebral column that resists its extension is the Aligamentum flavum

B supraspinous ligament

C posterior longitudinal ligament

D anterior longitudinal ligament

E interspinous ligament

Explanation

The ligaments of the vertebral column that resist flexion of the column include the supraspinous ligament

interspinous ligament ligamentum fiavum and posterior longitudinal ligament The ligament that resists

extension is the anterior longitudinal ligament This longitudinal ligament is very broad and strong It

covers the anterior and anterolateral surfaces of the vertebral bodies and the intervertebral disks In

addition to resisting extension the anterior longitudinal ligament provides reinforcement to the anterior

and anterolateral surfaces of the intervertebral disk The posterior longitudinal ligament is relatively

narrow and covers the posterior surface of the vertebral bodies and the intervertebral disks This ligament

reinforces the posterior surface of the disk The posterolateral surface of the disk is not reinforced and it

is through this region that herniation of the nucleus pulposus usually occurs

A patient presents with epigastric and right upper quadrant pain The pain is most intense 2-4 hours after

eating and is reduced by the ingestion of antacids The patient states that he has passed black tarry stools

(melena) within the last week Fiberoptic endoscopy reveals a yellowish crater surrounded by a rim of

erythema that is 3 cm distal to the pylorus Accordingly an ulcer has been identified in the patients

A fundus

B antrum

C duodenum

D jejunum

E ileum

A number of physiologic genetic and other factors increase the risk of gastric (and duodenal) peptic

ulcers The evidence that H pylori plays a principle role is compelling Smoking and caffeine are known to adversely affect the morbidity mortality and healing rates of peptic ulcers In general first-degree

relatives of peptic ulcer patients as well as males have a threefold to fourfold increased risk of developing this disorder Paradoxically in gastric ulcer disease acid secretion is not elevated It is possible that

excess secreted hydrogen ion is reabsorbed across the injured gastric mucosa In general a defect in gastric mucosal defense is the more important local physiologic

A patient presents with symptoms of duodenal obstruction caused by an annular pancreas Annular pancreas is caused by

A rotation of the dorsal pancreatic bud into the ventral mesentery B rotation of the ventral pancreatic bud into the dorsal mesentery

fJ failure of the major and minor pancreatic ducts to fuse ~ ~ cleavage of the ventral pancreatic bud and rotation of the two portions in opposite directions around -the duodenum E formation of one pancreatic bud instead of two

Explanation Normally the ventral pancreatic bud rotates around the gut tube to reach the dorsal pancreatic bud The two buds fuse to form a single pancreas and the distal portions of the two ducts fuse The ventral pancreatic bud forms the inferior portion of the head of the pancreas the uncinate process and the major pancreatic duct (of Wirsung) The dorsal pancreatic bud forms the superior part of the head the neck body and tail and the minor pancreatic duct (of Santorini) Annular pancreas is the result of the ventral pancreatic bud dividing into two portions before it rotates into the dorsal mesentery Each portion rotates in opposite directions to get to the dorsal mesentery thus encircling the duodenum The presence of annular pancreas can constrict the duodenum thus obstructing its lumen

In n _ phranlc----

Gon ~l ----_1 Lum bltano

~~--- CornmQ1t bull ac

+-~4--- lnlllirnaJ ilic

xtem iliac

OBJECTIVE - Identify the blood supply to each of the structures listed in the table on the previous page

Ill give you a head start

FOREGUT - Supplied bV Celiac Tru nk (T12)

Proper hepatic

GastiooUod 13Jafter

1nferlor pancreaticoduodenal artery

Common epatlc

Lett gas ric iiirtery

Spfen artery

shy Gastroepiphgtic artery

~ Superior mesenteric 8rtfry

~

1 Esophagus is a derivative of the foregut so its blood supply originates from the celiac trunk

(T12) The predominant blood supply to abdominal portion of the esophagus is the Esophageal

A (Branch of L Gastric) The venous drainage of the esophagus is particularly important because

it is 1 of 3 clinically relevant sites of Portal Caval anastamoses The Portal Esophageal Vein

meets the Caval Azygos System Persistent bleeding manifests as Esophageal Varices - a fata I

condition

2 The Stomach is also a derivative of the foregut has EXTENSIVE blood supply and is very high

yield on anatomy exams The lesser curvature is supplied superiorly by the L Gastric A (1 of 3

major branches ofthe Celiac trunk) and inferiorly by the R Gastric A ( a branch ofthe proper

Hepatic A) The greater curvature is supplied superiorly by the L Gastroepiploic A (a major

branch of the splenic A) and inferiorly by the R Gastroepiploic A

The Short Gastric arteries (branches of Splenic Artery) supply the fundus of the stomach and

are referred to as EIID ARTERIES because they have no collateral blood supply Therefore if the

splenic artery were occluded (ex - increased pressure in the ommental bursa) - there would be

ischemia to the fundus of the stomach Venous drainage of the stomach is extensive via various

veins lead ing to the portal system Posterior to the stomach the IMV joins the splenic V which

joins the SMV to form the PORTAL VEIN ADAMS

3 Duodenum blood supply has high clinical relevance because it is the junction of the foregut and

midgut and therefore is the site of anastamoses between branches ofthe Celiac Trunk (main

foregut artery) and the Superior Messenteric Artery (main midgut artery) The Proper hepatic

artery gives off the gastroduodenal artery which travels behind the 1st part of the duodenum

This point has high clin ical relevance because duodenal ulcers are very common and a posterior

rupture of the 1st part of the duodenum could rupture the gastroduodenal artery causing

traumatic abdominal bleeding The Gastroduodenal artery first gives off the R Gastroepiploic A

(mentioned above) and proceeds as the Superior pancreatico duodenal artery (supplies the

pancreas and duodenum) which anastamoses with the inferior pancreatico duodenal A (branch

of the SMA) This is the junction of foregut and midgut and occurs near the opening of the

bil iary system into the duodenum (ampula of vater) Portal venous drainage here is responsible

for delivering nutrients from digestion to the liver for metabolism Appreciate that the Superior

mesenteric artery (artery of the midgut) branches from the aorta at Ll travels posterior to the

pancreas than moves anteriorly (at the jxn of the pancreatic headbody) and comes over the

3rd4th part of the duodenum Tumor of the head of the pancreas can compress the SMA

4 Jiver blood supply is via the common hepatic artery (major branch of the cel iac trunk) The

common hepatiC becomes the proper hepatic gives off the R gastric A and the Gastroduodenal

A and then joins the common bile duct and the portal vein in the portal triad Clinical- if a

patient were bleeding from the hepatic A a surgeon can stick his fingers in the epiplOic foramen

and squeeze the free edge of the hepatoduodenalligament in order to stop bleeding to the

area Please note that the hepatic a branches into Rand L hepatic A The Right hepatic artery

gives off the cystic artery which supplies the gallbladder Afferent venous supply is via the

Portal vein which is bringing nutrient rich blood to the liver After metabolism takes place

venous blood leaves the liver through the hepatic veins into the IVC PLEASE UNDERSTAND THE

RELATIONSHIP OF THESE STRUCTURES - ADAMSNETIERSNH Etc

5 Pancreas - Head is supplied via the superior and inferior pancreaticoduodenal arteries

(mentioned above) The tail (situated towards the hilum of the spleen) is supplied via the

pancreatic branches of the splenic artery (END ARTERIES) This blood supply is very important

because the endocrine Alpha and Beta Cells from the pancreatic islets of lagerhans are located

towards the tail This is where Insulin and Glucagon is released to the blood

Now complete this for mid and hindgut structures Make sure to note clinically relevant arterial

anastomoses as well as portal caval anastomoses FYI Appendix blood supply SMA + IMA

anastamoses marginal artery Portalcaval rectal veins fhemmorhoids) and periumbilical caput

medusa are high yield THE BUTT THE GUT and THE CAPUT

Abdominal Development

Liver

Ij1f

II wall b

oh liN ~ VltJrti n be- bull

Pancreas

Secondary Retroperitonealization e I~tl r 1 a v-mtrai m ellter

Rotations of the Gut I i Ij (lIl1UtIJ f~ l r tilt

()l td 10 me l-ft and he v

--~--- -~ -~-~

i

I AolaijonjoI~guf I

STOMACH BED (IDENTIFY IN ADAMS)- the structures posterior to the ommental bursa which

support the stomach in the supine position

Abdomnal JQrUI

Splnic vein

OmQ-oul tv~ ) O(s(Jroa)

Lojt(r o m nturrt (hpJtodu o d~n31 Hid

Gadrl)SplerH (g3stroll~nal) IIgam~nt

hiad h~~atogrtricent IIQdmiddotcrt~)

Lt Dome of Diaphragm (why left Look this up in Adams)

Spleen (What is the blood supply)

Left Kidney (What is the blood supply - AND how is it different from the R kidney)

Suprarenal Gland (What is the Arterial AND Venous Blood supply - how are they different)

Pancreas (How does supply differ from Head to Tail What is the SMA Relationship)

Transverse Mesocolon

liver - ADAMSWET - Make sure you look at the liver in wet lab

Left triangular nl1am~nt

ComoaDj ligamnt

Erophg~1 impre$ioo

Hepatio veins

In1erior -ifena middotr3)Ia

Fibrous appendix o-t

live

impr~j on

Heprorendl p~rtion of Q)(Qllary ligament

Righllri~n9ul r 1I~met

(Common) bile quol

Gr)mmCtr~ hepatic dlJct

Ccentic duct

Duodenal impression

GaJdate p-fr)~S

Hepatic artgtrl prop-f iiiiila - Faloiform ligament

_ - shy Round ligamen liver

~--F-- CoJio imprgt-ssi-on

Prta heptis

Identify the lobes impressions and embryonic remnants associated with the liver

Caudate Lobe Quadrate Lobe Right Lobe Left Lobe Round ligament Falciform Ligament

Ligamentum Venosum (what is its fxn in embryonic life) Hepatic Veins (NOT PART OF THE

PORTAL TRIAD) IVC PORTAL TRIAD - Contents relationship cross section etc Know the

Galbladder relationship to the lobes of the liver

Biliary Duct System - Make sure you understand the sequence of these structures - BE ABLE TO

DRAW A FLOW CHART

TPVd i

t

I t

1 __ Cm-(r

patk GlJet

I

J

Clinical = JAUNDICE is caused by anything that prevents delivery of bile to intestine Tumor of the

head of the pancreas Stones etc Patient will have pale stools and yellowish colored mucus

membranes

Clinical- Any scenario that tells you the patient has BILLOUS VOMIT means that the obstruction to

the flow of digestive contents is after the Ampulla of Vater (Site of Entry of Billiary system to the

duodenum) - ie Duodenal Atresia

Spleen -located posterior to the mid axillary line between ribs 9 and 11 Make sure you know that

the 10th rib is the main axis of the spleen and this organ is susceptible to injury (stab wound errant

thoracoce ntesis etc)

The spleen is derived from mesodermal cells - NOT THE GUT TUBE

The spleen rests on the left colic flexure associates with the tail of the pancreas Know the

structures entering the Hilum of the spleen

Sh rt O~-t~ic 1 0(0 10 rtiltSPIric Iloa nt

(cut)

Peritoneum - similar concept to Pleura - think of a fist in a balloon

Visceral Peritoneum - Layer of balloon touching your fist

Parietal Peritoneum - Layer of balloon not touching your fist

Your fist represents the organ your wrist is the hilum and your arm contains the blood supply

entering the organ

Appreciate that there will never be organs in the peritoneal cavity - rather these organs invaginate

the cavity Kaplan videos

RULES OF NOMENCLATUREshy

1 Organ completely surrounded by peritoneum - peritoneal organ

2 Organ partially surrounded by peritoneum- Retroperitoneal

3 Peritoneum surrounding peritoneal organ is VISCERAL peritoneum

4 Peritoneum surrounding retroperitoneal organ is PARIETAL peritoneum

5 Peritoneum connecting visceral to parietal is called messentary 2 messentaries in the

gut Dorsal (to the gut tube) and ventral (to the gut tube) messentary

Aorta is in Retro peritoneal position - but blood must reach peritoneal position - vessels travel through

messentary All peritoneal organs will have blood supply reaching through messentary

-Mesentery is a 2 layer peritoneum with a neurovascular communication between body wall and organ

- Ligament connects one organ with another or to the abdominal wall (Ommentum = ligament)

lesser Ommentum (attach lesser curvature of stomach and duodenum to liver) =Hepatoduodenal

Ligament and Hepatogastric Ligament

Has a Superior and Inferior Recess (Accumulation of Fluid in Ascites)

Communicates with the greater sac through the epiplic foramen (what structures pass through

this foramen)

Boundaries - you must be able to visualize this

o Anterior - stomach

o Posterior - parietal peritoneum pancreas

o Superior - superior recess (bw diaphragm and coronary ligament)

o Inferior -Inferior recess (bw layers or greater momentum

Greater Ommentum (attach greater curvature of stomach) Gastrophrenic ligament Gastrosplenic

ligament gastrocolic ligament

The greater omentum is the largest peritoneal fold It consists of a double sheet of peritoneum folded on itself so that it is made up of four layers The two layers which descend from the greater curvature of the stomach and commencement of the duodenum pass in front of the small intestines sometimes as low down as the pelvis they then turn upon themselves and ascend again as far as the transverse colon where they separate and enclose that part of the intestine

ABDOMINAL PAIN

Parietal Peritoneum - supplied by same vasculature lymphatics and nerves supplying body wall it

lines and diaphragm Sensitive to pain pressure heat cold well localized

Visceral Peritoneum - supplied by same vasculature lymphatics and somatic nerve of organ it covers

Insensitive to touch heat cold and laceration - referred to dermatome of spinal ganglia providing

sensory fibers Where does appendicitis refer to

Foregut pain - epigastric area (ie - cholycystitis)

Midgut pain - periumbilical area (ie - appendicitis)

Hindgut Pain - suprapubic area (ie - diverticulitis)

Extra ImagesConcepts

ll~_____-

FalifCtrm ligament oind r~ud ligamet f Ilver

Blood from splenio gastriC and inferiof rne$e-rteri v~ins

Ca-I tributaries

Lett gastrio Ifein

Posterior superior pan~reatioodul)denal vaihS

Lott gamo-om~nlal (9aropip lomiddotic) -in

Poq_~ tjol imerl-9-r panCJertlcorllJod-nal veiopound --amp----I- - ~J Right grtr~-omntal

Anwrior interi (gartroepiploic) Jjn

pan euaii cod vl)denal veins middot Inf~Ji (t r mesentric vein

Miqdle (olic vein

Right cl)licvein Sigmoid and rectosigml)id (ei ns

IhH)Collc(~io

--- Mi~dl laquooLJl gtjrltgt

PoM ca vl1 illasto)moses -----shyampoptoageal 2 Paraumbilie-lt11 Inferi or Fectal vei ns

3 Recial 4 REuoperHonea1

Know how the Portal vein is formed I 4 sites of portal caval anastamoses and 1 clinical shunt

Col li t ltt-~ otTl~tI ~nj pc~ 1lt1 turJoG

Ltf 14i1 tImiddot~ artoftl9 on tj phtAt$

L-oftqf 4t t~r 1=laquoIran d 1 bull shy~p l ci rj o fOOOts

Nerves follow the arteries - appreciate the splanchnic nervous system I

Uet~ric branch of left ~nal art

Ureterie branch of righi renal artelY

Left Zld lumbar in and co mlTlunication to as)erdin9 lumbar l(~in Hi ~ht tEZ1~~t~ t3r j t itn ~ nJ l1t- rlnd lfe i r1

Inferior me5nteri~ artery

Notice that the right testicular vein drains directly into the IVC and the right testicular artery drains

directly into the aorta However the left testicular vein drains into the L renal vein at a right angleshy

reason left testicle is lower and more susceptible to varicocele (bag of worms)

Also notice that the left renal vein has a longer course because the IVC is on the right side whereas

the right renal artery has a longer course because the aorta is on the left side

Appreciate the anterior to posterior relationship of structures in the hilum of the kidney - VAP - Vein

Artery Renal Pelvis (Ureter)

11____ __ L_ L_ n VJ __ _ _ t_L I I_ _ L __ L_ I -pound1 bull LI_~-I ____

Posterior View of Head of Pancreas in ( of Duodenum

Celiao hunk

Co mmon ~L~jJth art~ry

GastNduQdonal artrf (partilly in phantn)

P1)Sterior $Up~Jior panCflaticuduodfmal art~r~t

(Co mm on) bile duct

middot~1t~~t-1l---~-~- Right gshomiddotomental (gastoe plp lolc) 3rte (phantomost)

Grener paocre atic art-ry

1n1~rjor pancr-iatlc artery

Jtrifll supejo r pal)oreailcento)dJodenal artr1 (phantom)

Anastomotlo branch

POostetlor bJanch of jo f~ri of pan-reatir(lduodensl drttnj

Anterio r branch of i flferior palcreati~)duodenal art~(phan1om)

Notice the extensive blood supply to the pancreas and duodenum via the branches of the celiac trunk

Notice collateral supply from SMA branches - makes sense bc this is the jxn of foregutmidgut

Identify the vessels in this arteriogram

Hiltid i)f N~ck oi B)dvof Tail 01 pa nereas pan cent~as P-nmiddot-reas panCtCas

I nferie v~na cava

jHept1iic p(lrlai v~in

Port1 tnd H~pti lt a ftH prol

Comm on) bll duct

Ouodtnum

~ft colic (sio)Atta~ hmtrlt jt~xJr-ofha~elSe

muo(IIQn

Right ~lIc (h~j)tic)

il~gtture

In1triol m~oten lIein (rttr op~ritoMdO

SlJp efl or mes~n~fiC amrV and lipln

KNOW YOUR NEIGHBORHOOD

Questions

vVhiJh structure supplied by a bnmdlof the cclia( artery is not derivcd from foregut LemCJUCrITI

(A) Head of the pancte-a5

CD) Pyloric duolenum

Cystkduct

( Liver hepatocyt~~

~F) Body of the spleen

An infant presents with an omrhaJucele at birth -hi oJ the [oHm illg applies to his cM1-dition

(A) It is 31so seen ill p4titnts with aganghonic megacolon

(11) ft reuirs from a fal1ure of resorption of theviteUine d let

(C) It results from herniation at the-site of regression of the right umbilk vein

DJ It is caustd by faihtrc of recanalization of the midgut part of the duodenum

~ It ill camioo by a failuIt vf the midgul to return to the abGQminal uity after herniashytion in-n the urnbilk s l stalk

Ot er than the spleen occlusion Cif the spit-Ilk artery at its odgin wm most likely affect die blood supply to jllch st cnud

(A) Jejunum

(B) Body of th pal1~lltas

(C) LeSStT Cllmiddotlaturc of tl )toma-ch

(D Duodenum dista to the entrance of the Ornmou bile duct

E Fundus of the stomach

A 38-yeu-old batL~er with a history of heartburn suddenly experiences excluciating pain in the (plgastric region of th~ abdomeu SurgCry is perf~rme immediard y upon admisshysion to the 1IlcrgCJliy tuomh~re i~ evidence uf a ruptured ulcer in the posterior waU of the stomach Vhere will a surgeon first fi nd the stomach contenlSf

A) Greater p4ritoneal sac

rB) Cul~de-s~c of Douglas (--

C Omental bursa ~

--D) Paracolic gutter

rEj Between -he panttal perimltum and the posterior body wal1

At birth an infant presents with a st()ma~ rb~tbas~njJled jfltotb~diaplfagru 1A1ltre is the defect thatresulied iiitJle heini~t()n shy~tsophagealbiatus

7 - rH-- Hiatus for the inferior vena cava

( Pleuroperitoneal membrane -(0) Septum transvcrsum

(E) Right Crlt~

An infant born with DOVv7l syndrome presents with bili()u~ vomiting Ahat congenital defect does the infant have

(A) Pyloric stenosis

(B) Meckel diverticulum C) Ornphaloce1e

(D) Gastroschisis

( ~ ) Duodenal atresia y A patient with cirrhosis of the liver presents with ~ bacalvaricestnlreased retrograde pressure in which veins caused the varices

(A) Paraumuilical

(B) Splenic

(ct AzygltJus

(15))G~trk ( (-F) Superior mesemeric

A htaltby 3-year~old male patient experiences a hernial sa protruding from the anterior abdominal wall about halfway between me anterior superior ilia spine and the pubk tuberde Pulsations of al1 artery are palpated medial to the protrusion site through the abdominal walL Which layer of the anterior abdominal wall will first be traversed by the

1hctma

fA) Rectus sheath (B) External oblique aponeurosis

(C) Inguinal ligament

lD) Transversalis fusda

(E) Cremasteric fa~cia

After 5urgi(aj ffpair of a hernia the patient tXperienccs mtmlgtness in the skin on the anteshyrior aspect of the S(Totum_ Vhaf nerve may have been lesioned during thehemiorrhaphy

(A) Femoral

(B) Obturator

(C) Ilioinguinal

(D) lliohypogastrk

(E) Pudendal

A 23~year-LJld female secretary il1 good health ~-uddcn1) doubles over with pain in the a ea of the 1JmbRicu$ Sbe feels vartn and ltneasy and has no appetite That night the pain seems to have mQved to the tower right abdominal regjol1 and she calls her family doctor who then arranges for an ambulance to pk-k her up and take her to the hospitaL Wh ell ntn~ perceived in the area of the urnbilirus most Hkely carried lhe pairfu I sensations into the eNS

tA) Vagus nerves I~

V B)

) Lessersplanchnk nerves

tC) Pudendal nerves

(D) lIiohpogastrk nerves

(E) Greater splam ic l erves

A CT reveals carcinoma in the bOod of the ancreas Vhich blood vessel trut ourses ----~- - -bull ------ --shy

immediately poftterior to the body ofthe pancreas is the m~t likely to be oompressed

(A) Splenk artery

(B) Abdominal aorta (C) Portal vein

(1) Splenic vein

(E) Renal vein

A patient has a penrln1l1ng uker of the posterior wall ot the br~l part ot the (lUooenmn llkh blood vessel is subject to erosion

(A) Common hepatic artery

(B) Gastroouodenal artery

(C) Proper hevatic artery

(D) Celiac artery

(E) Anterior inferior 11amrelltlcoduodcnal attery

Your patient has been diagnosed -ith a carcinoma locallted to the head and l~e(k of the pancreas Another clinical sign would be

A esophageal varices

(8) hemorrhoids

C) a caput medusa

(D) increased pra Teuro n th~ hepatic veins

(E) enlarged right supra lavkular lymph nodes

Wltkh of the foUowing structures develops in the ventral mesentery

(A) Spleen

(B) Jeiunum (C) Head of1ht pancreas (D) Transverse colon (E) Stomach

ti l Uw ~ littwin~ f( S-t lil oai Imdge ~ hi(h or tbt la~)d J truetur tgt liJ llntn nl) he hl p UC iJd [IIi ell

c o

A) drains Ie tht infCrior a La aI

R t middot~nfl0 ~ill to th~ lunlgtn of h i dtlndCrlllfH

(e) m t bull JiJattd on tl l J n T ~H

D ) sup Lc O VSlt I Hlid bhtu l 1 li - -I un oid

( ) U~tpli(t tr j middottUh~ 1 v(( b~nt rfK n1ilc~Zm

ANSWERS AND EXPLANATIONS

Answer E The spleen is t hlttnopodicand lymph organ demlted from mesoderm

Answ~ R Al1 tlmphalocele is caused by it failure of the nlidgut to return to the ahdomir nat cavity after herniation into the umbiliau Stalk Choices Aand D maybe seen in infants with Down syndrome choice D ~s the specific CBuse ofduudcnal JtiCSitt Choice C is (ile cause of gclstrosbisis and Choice B nsults iu a Meurolktldivertku1-tlB

Answer B The fundus ofthe stomach is suppHed by soort gastric brunches of the splenic altery The splenic artery supplies the body and tail of the pancreas part of the greater curvature of the sttmla(h and the spleen Te jejunum part of the head of the pancreas and tht~ duodenum distal to the entrance of the commOll bile duct are supplied by the superior mesenterk artery clll~l ~be less r ctlt1ature cmd the pylQric antrum are supplied by the right and lei gastric art(ries

AnSWftt C Tbeomental bursa or lesser ~ritoneaj sac lies direcdy posterior to the proxshyimal part of the duodeTtlm and the stomach and would be the first site where stomach contents ~Ott1d be fpoundluncL

Answer C A defect in a llleuropcritoneal membrane (uswlly the left) is the typical site of i1 cc-ngenitlI diilphragluatic hemia llere the membr4ne fails to dose ()pound( of the perishycCirdiopcritulleal canals

Answer E DuoJenal atresia and aganglionic megacoion are congwitaI defects S~Il in patients with Dowmiddotnsyndrome

Answer D RulaTgemt~llt of and retrograde flow in g~lstrk vel_ns in particlJl~r the kft gas~ tricveins dilates the capillary bed in rhe wall of the esophagus in (ases of porta yper~

tension Blood flow would increase in and dilampte tribntarkgts of the (lZygOUS vein on the other side of the capiUary bed but flow in this vein is in the typical direction t()ward the superior vena cava Paraumbiii(ltU vein eilgorgement contributes to a caput medusH Splenic ~nlargement might prc~nt with 5plcnonlegaly and balt-kflow in to tlu superior m~~ntclic vein occurs but is asymptomatic

Answer D The patient hagt an indirect inguinal hernia whi~h emerges from the antt-rior abdominal wall through the deep inguinltilling Theeep ring is a fault in the transv~rshysaUs fascia this I~yer wiIJ be penetrated first by the hernia

An~Wer C The ilioinguinal nenc which provides sens~llion to the lnedlal thigh ltmclanteshytior SClotunl pass~lt th rough the 5uperfh_ial inguinal ring ind $subject to inj i1T) becaus-e

it is in the operatitm Held of the erniorrhapny

Auswer B The leMHr splanchnic nerves are sympathdic nerVlts that carry viscera l sensashytlltgtrogt ftom illtllt1m~d ()J stietched gust (itinteitinal ~tructures (in this case the pprndix) into tnt eNS Lesser splanchnic ntTYcsarisc from thmiddot T9--T12 spinal cord segments lt1nd provide sympathetic innenation tD rnidgut siruc1ures whiCh include CLe app~JldD Viscera] Pain arising from affecLed Inidgut ampt 1C1ure is referred over the same dl- matorne~ of spinal segrnertts v-hich provide the sympathetic Innervation n this G1SC of appendicitis the invohen~n t of the ltire) of t e unlhHku indud s the T 10 dermatome

Answer B Of the five choices onty the dscending olon is retroperiton~al aldwould be a lik ~ ( choice to be seen immediately a(~jilcent to t11e posterior abdominal middotn~L

Amwen D The SpltftlC ~-ein ourses posterior to the body of the panneas m its way tt drain into the superior mCSfttltlri( vein

Answcr B TILt glstrodllolticnal artery 1 direct hIamh of the comrootl hepatic artery courses immediately pt))iwri() to the duodenum and is slbject to erosion

Answer B Carcinoma of th pan middott3S in the 1tilt1 may compreampgt the portltil vein at irs orishygill The poTtai vcin is fomled when the splenic vein jQiaswith tfie superior meStllt eric vein The inferiot mesenteric vein joins the ~plenjc vein just priOT to tlli~ point at which the splenic joins the superior Jlleit1ltcri( vein Increescd venous presslu in the inferior mesenteric vein is a cause of emo hoid~

Answer C The- velltral pancreas wilich forms most of the head of the p ~ncr as develops in the ventral mes(ntery as antutgrowth of the hepatic diverticulum Th~ hepatic divershyticulull induding the biIJary appa~atus develops in tbe ventral mesentery of the foregut

Answer~ A The superior mesenteric ~in joins with the spienkvein to form the hepatic portal vciu

Answer D The structure at gttlK is the proper hepatic artery~ whkh suppUesoxygenated b middotood to the liver

MAKE SURE YOU KNOW the diff bw Rectus Sheath above and below the arcuate line

ABOVE

Aponeurosis of xiiltmal obllque musclo

Extemll f)biquw musde

Reotln ilbdomlnls musole S~in

Internal 9bliquQ mY~QI

AponeUfOsi$ of hJH$V~~S Lir9a a lb lbdolTlin~ musolo Tri OJV6 rUi

atldomlnis mUS(loe

Sub cutanlilous tiue (tatty ye r)

BElOW

A POrl lJfosis 01 etemal oblique muscle

Aponeul~)sis 01 Internal oblique mU$cl~

Anteriol lay~ of r~ltdus st~ath EXttom1 oblique rnu$cll

Rectus Jbdominis muscle Intoernal Aponeurc-sis of tra~fersU$ oblique muscle-

at-domlnis muscentl ~ Skio

Tra nsvitSus abdomioLs ml)ZClt

TralSVersaHs fascia Medial umQil iegtt1 1i9Jment -and folj

Uldchus Peritoneum (ir median Umbilj~al Suboutane ous

Extraprftone 11ascia

Ymbilimiddot~1 fold)

preu9poundiea1 fascia

tissue (fatty 4nd m~mbr3n(iUS layers)

o Above the arcuate line (A horizontal line 13 of the distance bw the umbilicus and the

pubic symphysis) -10 Aponeurosis divides into an AntPost Laminae

o The Ant Laminae joins EO and Post Laminae joins Trans Abdominis = Ant and Post

RECTUS SHEATH respectively

o BElOW the arcuate line - all 3 aponeurosis join ANTERIOR to rectus muscle to meet its

counterpart in the midline (linea Alba)

o Take away Msg - The abdomen is devoid of a posterior rectus sheath below the

arcuate line and is therefore more vulnerable to herniasinjuries

Question - A physician makes a deep incision in the patients midline immediately superior to

the pubic symphysis which of the following layers is his knife least likely to pass

Rectus Abdominis External Oblique Ant Rectus Sheath Posterior Rectus Sheath All of the

Above

Answer - All of the above None of the other answer choices are midline structures -LINEA

ALBA

Linea Alba has very poor blood supply - doesnt heal well after surgery Therefore this is a

common site for incisional hernias

a Spleen b Transverse colon c Descending colon d Stomach e Pleura

17 Meckels diverticulum is normally found 2 feet proximal from the

a Pyloric sphincter b Lower esophageal sphincter c Ileo-cecal valve d Middle valve of Huston e Anal valve

18 Ulcer in the posterior wall of the first part of the duodenum would erode ___ artery and would cause bleeding

a Left gastric b Right gastric c Hepatic artery proper d Gastroduodenal artery e Middle colic artery

19 An inflamed appendix is identified by a surgeon on the operation table by noting

a The appendicies epiploicae b The convergence of tenia c The artery of Drummond d The mesocolon e The mesosalphinx

20 The nerve which emerges through the psoas major is

a Femoral b Ilio-inguinal c Ilio-hypogastric d Pudendal e Subcostal

21 The right gonadal vein drains into the

a Azygos b Hemiazygos c Inferior Vena Cava d Right renal vein e Left renal vein

22 The hepatocytes in the liver is derived from

a Ectoderm b Endoderm c Mesoderm

d Neural ectoderm

23 Abscess in the lumbar vertebrae due to tuberculosis would spread to the adjacent muscle which is

a Psoas Major b Iliacus c Quadratus lumborum d Tranversus Abdominis

24 The anterior wall of the inguinal canal is formed by

a External oblique and transverses abdominis b External oblique and fascia transversalis c Internal oblique and external oblique d Internal oblique and transverses abdominis e Fascia transversalis and peritoneum

Meckels diverticulum is a result of which of the following developmental abnormalities shy

A Failure of the vitelline duct to close

B Failure of the herniated intestinal loop to retract into the abdomen

C Failure of the urachus to close

D Failure of the midgut to rotate

E Failure of the hepatic duct to close

Explanation

Meckels diverticulum is a result of the persistence of the proximal part of the vitelline duct This

diverticulum is usually found about 2 feet proximal to the ileocecal junction and is usually about 2 inches

long It is present in about 2 of the popUlation It may be the site of ectopic pancreatic tissue or gastric

mucosa and may develop inflammatory processes and ulcerations Acute Meckels diverticulitis

simulates appendicitis

Which of the following veins carries blood from the esophagus to the portal vein The

A right gastric vein

B left gastric vein c splenic vein D azygos vein

E left gastroepiploic vein

Explanation

The left gastric vein a direct branch of the portal vein drains blood from the lesser curvature of the

stomach and the inferior portion of the esophagus Because branches of the portal vein do not have

valves blood can flow in a retrograde path when there is an obstruction to flow through the portal system or liveL Rlooci Cln then flow from the nortl] vein thr()1Ph the left PRstric vein to the esonhlPlIS lno

through venous communications within the submucosa of the esophagus to esophageal veins that drain

into the azygos vein The increase in blood flow through the esophageal submucosal veins results in esophageal varices

On the posterior wall of the abdomen the celiac ganglion A contains cell bodies of postganglionic parasympathetic neurons B is synapsed upon by neurons in the posterior vagal trunk C is synapsed upon by neurons in the greater splanchnic nerve D contains sensory cell bodies of lumbar spinal nerves E contains cell bodies of neurons that cause an increase in the rate of peristasis

Explanation The celiac ganglion is one of the preaortic ganglia of the sympathetic nervous system It contains cell bodies of postganglionic sympathetic neurons The sympathetic splanchnic nerves contain preganglionic sympathetic neurons that pass through the sympathetic chain without synapsing These splanchnic nerves go to the preaortic ganglia to synapse The greater splanchnic nerve contains preganglionic neurons from spinal cord segments T5-T9 This nerve synapses in the celiac ganglion The nerve fibers in the vagal trunks are preganglionic parasympathetic fibers that go to the walls of the organs that they will innervate and synapse on postganglionic parasympathetic neurons in the walls of those organs Cell bodies of sensory neurons in the abdomen are found in the dorsal root ganglia or the sensory ganglia of the vagus nerve Sympathetic innervation decreases the rate of peristalsis parasympathetic innervation increases the rate of peristalsis

Which of the following pairs of arteries will allow blood to bypass an occlusion of the celiac trunk

A Left gastric artery-right gastric artery

B Left gastroepiploic artery-right gastroepiploic artery

C Superior pancreaticoduodenal artery-inferior pancreaticoduodenal artery

D Splenic artery-common hepatic artery

E Left gastric artery - proper hepatic artery

Explanation The anastoOlosis of a branch of the celiac trunk and a branch of the superior mesenteric artery will

provide collateral circulation around an occlusion of the celiac trunk Each of the other choices pair

branches of the celiac trunk therefore these will not provide collateral flow around the obstruction of the

celiac trunk The left gastric splenic and common hepatic arteries are direct branches of the celiac trunk

The right gastric artery is a branch of the proper hepatic artery which is a branch of the common hepatic artery The left gastroepiploic artery is a branch of the splenic artery The right gastroepiploic artery is a

branch of the gastroduodenal artery whlch is a branch of the common hepatic artery

Which of the following organs has appendices epiploica The

A sigmoid colon

Bjejunum

C duodenum

D stomach E esophagus

Explanation Appendices epiploica are characteristic of the colon Appendices epiploica are subserosal accumulations

of fat None of the organs of the gastrointestinal tract has appendices epiploica except the colon

Page 12: Chirag's Abdomen Review

----

Dry Lab - Label subcostal iliohypogastric Ll Ilioinguinal (Ll)

Horesshoe Kidney - stuck under IMA

Renal Agenesis -failure of the ureter bud to develop

Double Ureter

Unilateral Agenesis -1 kidney

Kidneys - Metanephros

Fetal kidneys are at sacral level

Look at 3rd part of duodenum

Some of this stuff is repeated I know just copied and pasted a bunch of stuff I had copy

Dermatomes

bull T4 nipples

bull no umbilicus v o Pain referred to no in appendicitis o Pain referred to T7ITS in gastritis ~

Inguinal ligament = external abdominal oblique aponeurosis

bull Inserts at anterior superior iliac spine to the pubic tubercle o Why important to know -7 visualizing this line allows us to properly diagnose a hernia

Below the inguinallig femoral hernia Above the inguinallig =inguinal hernia

Also to palpate the deep inguinal ring you go about 12Scm above the mid-inguinal

point

bull Modifications to ligament o Pectinate ligament o Lacunar ligament -7 cut this ligament to relieve strain i~ stran ul~tEd hernia

Inguinal canal

bull in males -7 transmits spermatic cord o important structures of spermatic cord ductus deferens testicular a genital branch of

the genitofemoral n pampiniform plexus of veins bull in females -7 transmits round ligament

Borders

bull Floor -7 inguinal ligament + lacunar ligament bull Anterior -7 aponeurosis of external oblique + internal oblique bull Roof -7 internal oblique and traverse abdominal bull Posterior -7 transverse abdominal + transversalis fascia

o Reinforced by conjoint tendon bull Aponeurosis of internal abdominal obliques and transverse abdominus bull Lies immediately behind the superficial inguinal ring in what would otherwise be

a weak point in the abdominal wall bull Innervated by ilioinguinal nerve (Ll) ~why important

bull In appendicitis Ll can be injured which will injure this nerve and in turn

the conjoint tendon With loss of innervation to this supportive structure the patient is now predisposed to a direct inguinal hernia

o Only hernia that can transverse the inguinal canal o Associated with congenital condition persistent tunica vaginalis

bull Direct inguinal hernia o Medial to inferior epigastric a o When inserting finger in superficial inguinal ring will feel on back of finger o Associated w old age or recent surgery

Muscles (Abdomen RECTUS SHEATH)

bull Arcuate line at level of ASISor 13rd distance between pubis and umbilicus bull Above arcuate line rectus abdominus is surrounded by a rectus sheath anteriorly and

posteriorly

o EO and 10 lie over rectus abdominus o 10 and TA lie behind rectus abdominus

bull Below arcuate line rectus abdominus has no rectus sheath posteriorly o EO 10 and TA lie over rectus abdominus o Transversalis fascia lies behind rectus abdominus o Inf EpIgastric vessels pierces the rectus sheath here

Peritoneum serous sac which encloses most of the abdominal structures

bull Ovary =only intraperitoneal organ o Oocyte ejected from ovary then captured by fallopian tubes o Why impt Women more prone to infection that can enter peritoneum

Peritoneum forms

bull Mesentery double layered fold of peritoneum formed as the organ was pulled in

bull Ligament between 2 organs in general bull Omentum between stomach and another organ bull Bare area area of no peritoneum bull

Viscera innervation

bull Pa rasympathetics 11 o Afferents sense hunger o Efferents l peristalsis relaxes sphincters gland secretion

bull Sympathetics o Efferents do opposite o Afferents CARRY PAIN SENSATION OF THE VISCERA (dull stretching pain)

bull PARASYMPATHETIC INNERVATION o Vagus nerve 7 _1l to 23rd unct ion of la rgej nte~tine oJ)elVrcspla~~~)~~ic~rYe~ IJiU- ~rd aJ~lpoteotiD~ IMPT

Gut Embryology

Gut ~ We say that the gut is derived from endoderm We often forget that when we say so we mean

that only the mucosa is derived from the endoderm The submucosa and the muscle layer is actually derived from the splanchnopleuric mesoderm and the serosa is derived from the visceral peritoneum

~ The main function of the gut is to digest the food which is done by the glands derived (and are) in the mucosa (endoderm) The only two exceptions in the Gut where glands though derived from the endoderm do not stay there but migrate down into the submucosa are esophagus and duodenum These glands however have their ducts opening to the swface of the mucosa

bull

~ Lungs liver amp gall bladder and pancreas are off-shoots from the foregut Esophagusshy~ The region of the tube from the laryngeal diverticulum to the beginning of the stomach elongates

to form the esophagus ~ The glands which form in the endoderm (mucosa) migrate down into the submucosa The path

whlch it took migrate becomes the duct of the glands which open to the mucosa ~ Achalasia Cardia - Failure of relaxation of the lower esophageal sphincter because of congenital

absence of ganglia at the sphincter (The ganglia when present releases VIP (Vaso-IntestinalshyPeptide) which relaxes the sphincter)

Mid-Gut Rotation ~ Because of the 90 degree rotation of the primitive stomach all of the following events occur ~ Lesser curvature comes to the right Therefore lesser omentum also comes to the right ~ Greater curvature comes to the left Therefore greater omentum also comes to the left ~ Right side vagal trunk becomes posterior vagal trunk ~ Left side vagal trunk becomes anterior vagal trunk ~ The left side peritoneal cavity comes to the anterior aspect of the stomach and will later be called

as the greater sac ~ The right side peritoneal cavity comes to the posterior aspect of the stomach and is (relatively a

small sac because the liver is on the right) called the lesser sacomental bursaepiploic sac ~ Epiploic foramen of Winslow (the lower free margin of the ventral mesentry) wiII be the

communication between the greater and lesser sac ~ The Liver moves to the right and therefore actually causes the 90 degree rotation of the stomach

The spleen comes to lie on the left side ~ Axis Antero-posterior axis around the superior mesenteric artery

bull Counterclockwise bull Approximately 270deg bull During herniation (about 90deg) bull During return (remaining 180deg)

Duodenum ~ Becomes retroperitoneal (except the first part which is still suspended by the hepato-duodenal

part of lesser omentum) ~ Glands (of Brunner) go submucosal ~ An imaginary line drawn below the opening of the major duodenal papilla represents the junction

between the foregut and midgut ~ Duodenal atresia in Downs syndrome Liver ~ 3rd week

bull liver bud grow bull into the septum bull transversum

~ 10th week bull hematopoietic bull function

bull 10 of the total bull body weight

~ 12th week bull bile is formed

Pancreas ~ In about 10 of cases the duct system fails to fuse and the original double system persists ~ 3rd month

bull pancreatic islets (Langerhans) ~ 5th month

bull Insulin secretion ~ Annular pancreas

bull The right portiCn of the ventralbud migrates along its normal route but the left migrates in the opposite direction

~ Complete obstruction of duodenum ~ Accessory pancreatic tissue Polyhydramnios (Amniotic fluidgt 1500-2000 ml)

~ Congenital defects including central nervous system disorders (eg Anencephaly) and gastrointestinal defects (atresias ego Duodenal esophageal) prevent the infant from swallowing the amniotic fluid (failure of recanalization)

Oligohydramnios (Amniotic fluid lt 400 mt) ~ Cl~ldberenal-agenesis

bull Midgut _-_

~ Primary Midgut intestinal loop gives rise to bull Distal duodenum bull Jejunum bull Ileum bull Ascending colon bull Transverse colon - proximal two-thirds of the bull Transverse colon with the distal third

~ Primary intestinaltoop bull ncephalic limb distal part of the duodenum the jejunum and part of the ileum bull ncaudal limb lower portion of the ileum the cecum the appendix the ascending colon and

the proximal two-thirds of the transverse colon bull 6th week

bull Rapid elongation of the cephalic limb bull Rapid growth of the liver bull Intestinal loops enter the extraembryonic cavity in the umbilical cord

bull 10th week bull loops begin to return bull regression of the mesonephric kidney reduced growth of the liver expansion of the

abdominal cavity bull Jejunum -left bull Loops - more to the right

bull Cecal bud -last part (temporarily below the right lobe of the liver) ~ qIDlthaloseJe (Structures COlHLoArts9V~1tion)

bull Through umbilical ring bull 6th to 10th weeks

bull Associated with a high rate of mortality (25) and severe malformations bull Associated with chromosome abnormalities

~ Gastroschisis (Structures coming out are not covered by Amnion) bull herniation through the body wall ----=---=-shybull Into the amniotic cavity bull Lateral right of the umbilicus bull Sometimes the inferior wall fails to develop as a result lower abdominal structures like the

bladder would be exposed to the exterior not associated with chromosome abnormalities ~ Abnormalities of the Mesenteries

bull Mobile cecum persistence of mesocolon bull Extreme form - long mesentery bull Volvulus

~ Distal third of the transverse colon ~ Descending colon ~ Sigmoid colon ~ Rectum ~ Upper part of the anal canal ~ Primitive anorectal canal

bull 7th week cloacal membrane ruptures bull Tip of the urorectal septum perineal body bull Pectinate line

~ Hindgut anamolies bull Rectoanal atresias and fistulas bull Imperforate anus bull Congenital megacolon (aganglionic megacolon Hirschsprung disease)

bull

bull Hindgut

Chirags Abdomen Review - Part 2

Understanding Embryo makes learning blood supply EASY

I I

I t

~ -

)

Table l1r-~ L Adult SUmiddotuctu~SDrj~l Froln Each of he Three Dhisions of be Pringttive GUl Tube t-middot-----middotmiddotmiddotmiddot-

Foregu(

I_ (Celiac Trunk)

Ir-slt-gtphgus

S101na(b

I h -= LiJ~r

Pancre=l S

bull 1 i Biliary apparntu5

Gall bladdshy

i Pha11~Cal pltgtuchcs

LullSS-I

Mjig ---- bull __ _- ----n--duct----~---middot-------l--n

(Superior Jldesen1eric Artery)_-1I-(I_~__ middot __ O-=-)_in_middoto_r_M_e_se_n_t_e_r_i_c_An__

Uuodenu rn 2nd_ 3 lt141h V4Tt

Jejunun-~

nc-un]

tCCUJ11

AppltgtndLX

Transver5e -o1on (p~oxiln1l1 ~O Tbird)

bull__hytgtid~ _ _ ______ L _ __

Tr-dn~llt~se colon (diStul h lTd) I

)

i

Aa ca-nal -( uppeT patt) i

I I

_____ __ _ _ _ ___ - - rhe~ a(t clcriVOkt iV(5 opound~lt prbn1rC ~ nlQC blft TlI)( 134tof r~ tIonoinf~ i 1 ~l l1rd c- P Cle

Now Lets see how much youve learned

Questions

1) A pt receives a general anesthetic in preparation for a c~t~~my A right subcostal incision is made which begins near the xyphoid process runs along and immediately beneath the costal margin to an anterior axillary line and transects the rectus abdominus muscle and rectus sheath At the level of the transpyloric plane the anterior wall of the

-~~-~=--- _eco---shysheath of the rectus abdominus muscle receives contributions from which of the following

a Aponeuroses of the in~ande~tef-Ilal o~ues

b Aponeuroses of the transversus abdominis and internal oblique muscles c Aponeuroses of the transversus abdominis and internal and external oblique

muscles d Transversalis fascia e Transversalis fascia and aponeurosis of the transversus abdominus muscle

A

2) The lat~raJJJ11QjJt~gLfgJlLoneach side of the inner surface of the anterior abdominal wall is created by which of the following structures

K Falx inguinalis (~) Inferior epigastric a

c Lateral border of the rectus sheath d Obliterated umbilical a e Urachus

B

3) A man the victim of several knife wounds to the abdomen during a brawl at the Lobster Shack subsequently developed a direct inguinal hernY Damage to which of the following nerves is most likely responsible for the predisposing weakness of the abdominal wall

~ Genitofemoral nerve ( b) Ilioinguinal nerve ~-t Tenth intercostal nerve

d Subcostal nerve e Pelvic splanchnic nerve

B

4) Which of the following statements concerning a direct inguinal hernia is correct a It is the most common type of abdominal hernia b It transverses the entire length of the inguinal canal c It contains all3 fascia layers of the spermatic cord d It exits the inguinal canal via the superficial ingeJinal ring e It protrudes through H~acb strJg e

~(

1fltbS w E

tl

5) The conjoint tendon is

a Important in preventing indirect inguinal hernias b The fused aponeurotic layers of internal abdominal oblique and transversus

abdominus muscles c Posterior to the deep inguinal ring

d Medial fibers of the inguinal ligament

B

6) A 25 year old male is brought in to the ER after being involved in a car accident in which he received a crushed internal injury in his abdomen Examination reveals a lesion of parasympathetic fibers in the vagJsnerve which interferes with glandular secretory or

smooth muscle functions in which of the foliowingorgans a Bladder b Transverse coloiW c Descending colOO d Prostrate gland e Rectum

B

7) The spermatic cord includes all of the following contents except a Il ioinguinal nerve b Pampin iform plexus of veins c Vas deferens d Genitofemoral nerve

A

8 Which abdominal structure gives rise to the internal spermatic fascia (muscle) following the descent of testes in development

a External abdominal oblique aponeurosis b Transversalis fascia c Transversus abdominis muscle d Peritoneum e Internal abdominal oblique

B

9 Which abdominal structure gives rise to the tunica vaginalis fotlowing the descent of testes during development shy

a External abdominal oblique aponeurosis b Transversalis fascia c Transversus abdominis muscle d Peritoneum e Internal abdominal oblique

D

10) The lesser omentum is a peritoneal fold which is su bdivided into the a Hepatogastric and gastrosplenic ligaments b Hepatoduodenal and gastroomentalligaments c Hepatoduodenal and gastrosplenic ligaments d Hepatogastric and hepatoduoden9-jrj igaments

D

11) A posteriorly perforating ulcer in the pyloric antrum of the stomach is most likely to produce initiallocalized peritonitis or abcess formation in which ofthS fQllowing

a Great-sac - -- -

b Paracolic recess

c Omental bursa

d Right subphrenic space

c

The inferior mesenteric artery arises from the abdominal aorta ilm_ediill~y_J-Qs1eriQLto which of the foowing org~ns A-F~t~filie duodenum B Head of the pan~eis C Neck of the pandeas

D Second part of the duodenum

E Third part of the duooenum_shylaquoshy

shy

The correct answer is E The inferior mesenteric artery arises from the anterior surface of the aorta at the level of the third lumbar vertebra The third part of the duodenum crosses the midline at the level of the third lumbar vertebra and passes anterior to the aorta at the origin of the inferior mesenteric artery The

first part of the duodenum (choice A) lies horizontally to the right of the midline at the level of the first

lumbar vertebra The head of the pancreas (choice B) is to the right of the midline and extends from the

level of the first lumbar vertebra to the third lumbar vertebra It lies within the concavity of the

duodenum The neck of the pancreas (choice C) lies in the midline at the level of the first lumbar

vertebra It lies on the anterior surface of the aorta at the origin of the superior mesenteric artery The second part of the duodenum (choice D) lies vertically to the right of the midline and extends from the

level of the first lumbar vertebra to the level of the third lumbar vertebra

The left adrenaLvein drains directly into which of the following veins A Hemiazygos vein

B Inferior vena cavaee C Left renal veiri -

D Splenic vein

E Superior mesenteric vein

a

The correct answer is C The left adrenal vein and the left gonadal vein (either testicular or ovarian) drain into the left renal vein TheTeft renal vein t~ains intothe- inferior vena cava In contrast the right

adrenal ~~inandnght gonadal veindrai~ gLr~ctJy iQtoJhe iilferiQ[ Vencava -- -

ThehemTazygoS7ein- (~h-~i-~ A)~~c~i~es the venous drainage from the body wall on the left side of the

thorax and abdomen No visceral organs drain directly to the azygos or hemiazygos veins The inferior vena cava (choice B) receives the direct venous drainage from the right adrenal vein but not

the left adrenal vein Remember the inferior vena cava is on the right side of the abdomen The splenic

vein (choice D) receives the venous drainage from the spleen and part of the pancreas and stomach The splenic vein is part of the portal venous system

The superior mesenteric vein (choice E) receives venous drainage from much of the intestinal tract It is part of the portal venous system and joins with the splenic vein to form the portal vein

A 43-year-old man presents complaining of pain in the groin On examination his physician palpates a

bulge in the region of the superficial inguinal ring which he diagnoses as a direct inguinal hernia The hernial sac most likely

A is covered by all three layers of the spennatic fascia B passes medial to the inferior epi gastric artery

C passes medial to the lateral border of the rectus abdominis muscle

D passes posterior to the inguinal ligament E passes through the deep inguinal ring

The correct answer is B Direct inguinal hernias enter the inguinal canal by tearing through the posterior

wall of that structure The typical location for this type of hernia is through the inguinal triangle bounded

laterally by the inferior epigastric artery medially by the lateral border of the rectus abdominis and

inferiorly by the inguinal ligament Direct inguinal hernias pass medial to the inferior epigastric artery

whereas indirect inguinal hernias pass lateral to the inferior epigastric artery because the deep inguinal

ring is lateral to the artery Indirect inguinal hernias are covered by all three layers of the spermatic fascia (choice A) Direct inguinal hernias are covered by fewer than all three layers because the direct inguinal

hernia tears through one or more layers of fascia as it emerges though the abdominal wall The lateral

border of the rectus abdominis muscle (choice C) forms the medial border of the inguinal triangle All

inguinal hernias pass lateral to the rectus abdominis Femoral hernias pass posterior to the inguinal ligament (choice D) Inguinal hernias emerge through the superficial inguinal ring which is superior to the inguinal ligament Inguinal hernias that descend below the inguinal ligament pass anterior to the

ligament Indirect inguinal hernias pass through the deep inguinal ring (choice H) direct inguinal hernias

do not Both types of inguinal hernias pass through the superficial inguinal ring

During a gastric resection in a patient with stomach cancer a surgeon wants to remove the lesser

omentum because of tumor extension into it Which of the following structures lie in the free edge of the

l~~g omentum and consequently must be dissected out in order to be preserved

A Common bile duct cystic duct and hepatic artery 6

B Cystic duct hepatic artery and hepatic vein

e Hepatic vein and cystic duct

Portal vein common bile duct and hepatic artery

E Portal vein hepatic artery and hepatic vein

The correct answer is D The free edge of the lesser omentum contains three important structures the

common bile duct the hepatic artery and the portal vein Nei ther the cystic duct (choices A B and C) nor the hepatic vein (choices B C and E) lies in the free

edge of the lesser omentum

A 55-year-old male patient with chronic liver disease has portal hypertension To relieve the pressure in the portal system a porto-caval shunt is performed Which of the following veins may by anastomosed to

accomplish this porto-caval shunt A Left renal vein-left testicular veingt

B Right renal vein-right suprarenal vein I shy

e Splenic vein -left renal vein J

D Superior mesenteric vein-inferior mesenteric vein E Superior mesenteric vein-splenic vein

The correct answer is C The splenic vein drains directly into the portal vein The left renal vein drains

directly into the inferior vena cava Anastomosis of these veins would allow blood from the portal vein to

drain retrograde though the splenic vein into the renal vein and then into the inferior vena cava The left

renal vein (choice A) drains directly into the inferior vena cava The left testicular vein drains directly into

the left renal vein Thus these veins are already in communication and neither vein is part of the portal venous system The right renal vein (choice B) drains directly into the inferior vena cava The right

suprarenal vein also drains directly into the inferior vena cava Thus neither vein is part of the portal

venous system The superior mesenteric vein (choice D) drains directly into the portal vein The inferior

mesenteric vein drains into the splenic vein which then drains into the portal vein Thus neither vein is

part of the caval venous system The superior mesenteric vein (choice E) drains directly into the portal

vein The splenic vein also drains directly into the portal vein Thus neither vein is part of the caval

venous system

A 12 year old boy has fever vomiting and para-umbilical pain After examining the patient the doctor

makes an initial diagnosis of appendicitis Appendicular pain which is initially referred to the umbilicus goes to the dorsal root ganglion of

a TI b TI2 c L1 d T7

(e I TIO

A 59-year-old male undergoes a neurological examination which reveals that when the abdominal wall is

stroked the muscles of the abdominal wall of the side of the body stimulated failed to contract Other

neurological tests appeared normal The likely region affected includes

a CI - C5 spinal segments b C6 - TI c T2-TI ~T8-T12

e Ll- L5

The surgery done to relive portal hypertension is done by connecting two veins Which of the following veins would be suitable for connection

a Inferior vena cava and portal vein b Superior vena cava and portal vein c Splenic vein and right renal vein d Splenic vein and left renal vein e Superior mesenteric vein and Inferior vena cava

A mother brings her 3-week-old infant to the pediatric clinic reporting a new scrotal bulge that she found -~-

while changing a diaper yesterday The infant is afebrile Physical examination reveals a palpable mass in

the scrotum while in the standing position resolution of the mass in the supine position and no

transillumination of the scrotal sac What is the most likely diagnOSiS

a Cryptorchidism b Direct inguinal hernia c Hydrocele d Indirect inguinal hernia ~ e varicocele

The Vagal trunks enter the abdomen by passing through which of the following openings in the

diaphragm

a Right crus b Esophageal hiatus ~ c Vena caval hiatus d Aortic hiatus e Left crus

2 The anterior boundary of the epiploic foramen of Winslow is bounded by

a) First part of duodenum b) Lesser curvature of stomach c) Liver d) Hepato-duodenalligament v ~

3 The ilio-inguinal nerve is derived from

a TI2 ry b LI c L2 d L3 e L23

15 Surgically the structure used to suspend the kidney to the diaphragm is

a) Renal fascia b) True capsule c) Perinephric fat d) Paranephric fat

6 If there is portal obstruction because of carcinoma affecting the pancreas which of these of the

following signs would be present

a Caput medusae b Esophageal varices c Rectal varices c

d Pulmonary edema

7 In a sliding hernia the gastro-esophageal junction lies

a) At its normal position b) Below the normal position c) Above the normal position V d) None of the above

8 Which of the following structures is retroperi toneal

A transverse colon B spleen IJ2f6 C ileum D descending colon v r 1pound1111111

9 The renal angle is fonned lgtetween the 12th rib and ______ muscle

a Psoas major -middotshyb Erector spinae c Quadratus Iumborum d Diaphragm

10 The anterior structure at the hilum of the kidney is

a) Renal vein ~

b) Renal artery I middot~ I

c) Ureter d) Accessory renal artery

11 Because of origin of the muscle from the lateral one third of the inguinal ligament it

could not fonn the anterior wall of the inguinal ligament

a) External oblique b) Internal oblique c) Transversus abdominis_ d) Rectus abdominis

12 A large tumor mass impinges on the splenic artery and its branches as the artery pass out from below

the greater curvature of the stomach Branches o(which of the following arteries would most likely to

effected by the pressure on the splenic artery

a Left gastric b Left gastro-epipJoic c Right gastric d Right gastro-epipoloic e Short gastric_

13 A new born baby has projectile vomiting after each feeding It is determined that there is obstruction

of the digestive tract as a result of annular pancreas Annular pancreas is as a result of an abnormality in which of the following process

a Rotation of the dorsal pancreatic bud around the first part of duodenum b Rotation of the dorsal pancreatic bud around the second part of duodenum c Rotation of the dorsal pancreatic bud around the third part of duodenum d Rotation of the ventral pancreatic bud around the first part of duodenum y Rotation of the ventral pancreatic bud around the second part of duodenum

14 As the liver bud enters the ventral mesogastrium the region of the mesogastrium stretching from the

liver to the anterior abdominal wall is called

a Lesser Omentum b Greater Omentum ~ Falcifrom ligament d Lacunar ligament e Ligamentum teres of liver

16 A patient has absence of his 12th rib In such a patient if the doctor makes an incision to approach his

kidney mistaking the 11 th rib for the 12t he would end up injuring

Which of the following arteries is a direct branch of the gastroduodenal artery The

A right gastric artery

B left gastric artery

C inferior pancreaticoduodenal artery D left gastroepiploic artery

i E)right gastroepiploic artery --

E x pI a nation The right gastric artery is typically a branch of the proper hepatic artery The left gastric artery is a direct

branch of the celiac trunk The right and left gastric arteries anastomose along the lesser curvature of the

stomach The inferior pancreaticoduodenal artery is a branch of the superior mesenteric artery it

anastomoses with the superior pancreaticoduodenal in the head of the pancreas The left gastroepiploic

artery is a branch of the splenic artery it anastomoses with the right gastroepiploic artery along the greater

curvature of the stomach The right gastroepiploic artery is a branch of the gastroduodenal artery The

other branch of the gastroduodenal artery is the superior pancreaticoduodenal artery

Which of the following pairs of veins join together to form the portal vein The

A superior mesenteric vein and inferior mesenteric vein

B inferior mesenteric vein and splenic vein

C superior mesenteric vein and splenic vein

Ip)splenic vein and left gastric vein E superior mesenteric vein and left gastric vein

Explanation

The portal vein is formed behind the neck of the pancreas by the union of the superior mesenteric vein

and the splenic vein The inferior mesenteric vein drains into the splenic vein The left gastric vein drains

directly into the portal vein After the portal vein forms it enters the hepatoduodenalligament of the

lesser omentum to reach the liver The portal vein is the most posterior structure in the hepatoduodenal

ligament

At which of the following vertebral levels does the duodenum pass anterior to the aorta - _- shy

All ~

B L2 7~

CL3 I

~DL4

E L5

Explanation

The duodenum begins at the pyloric sphincter at the level of Ll The second (or descending) portion of

the duodenum is to the right of the aorta and extends inferiorly from the level of Ll to the level of L3 The third part of the duodenum crosses the aorta from the right side to the left side at the level of L3 The

fourth (ascending) portion of the duodenum extends from the level of LJ to the level of L2 The

duodenum ends at the duodenojejunal flexure The superior mesenteric artery passes anterior to the

duodenum as the duodenum passes anterior to the aorta The duodenum can be constricted at this level

In which of the following locations will perforation of the digestive tract result in the spilling of luminal

contents into the - lesser peritoneal sac

A Anterior wall of the second portion of the duodenum B Posterior wall of the second portion of the duodenum

C Anterior wall of the stomach

~Posterior wall of the stomach E Posterior wall of the transverse colon

Explanation

The posterior wall of the stomach is related to the lesser peritoneal sac The anterior wall of the stomach is related to the greater peritoneal sac The anterior wall of the second portion of the duodenum is related to the greater peritoneal sac The posterior wall of the second portion of the duodenum is related to the retroperitoneal space The posterior wall of the transverse colon is related to the greater peritoneal sac

The ureter lies against the anterior surface of which of the following muscles shyA Crus oftne diaphragm B Quadratus lumborum

0 Psoas major D Transversus abdominis

E Iliacus

Explanation The ureter exits the renal pelvis at about the level of vertebra L2 As it descends along the posterior abdominal wall it lies on the anterior surface of the psoas major The psoas major muscle arises from the bodies of the lower lumbar vertebrae The psoas major muscle is joined by the iliacus to fonn the

iliopsoas muscle The iliopsoas muscle then attaches to the lesser trochanter of the femur and is the major

flexor of the hip

As the right ureter passes the pelvic brim it lies against the anterior surface of which of the following

blood vessels

A Gonadal artery B Inferiorvena cava C Internal iliac artery

rJ- External Iliac artery

E Inferior mesenteric artery

Explanation

The ureter lies in the extraperitoneal space in the posterior abdominal wall Alter leaving the kidney it

passes inferiorly on the anterior surface of the psoas major muscle At the pelvic brim the ureter passes

into the pelvis At this point the common iliac artery is dividing into the external and iliac arteries The

ureter lies on the anterior surface of the external iliac artery immediately distal to the bifurcation This is a useful landmark for a surgeon to locate the ureter

When extravasated urine passes from the superficial perineal space into the anterior abdominal wall it is

found immediately deep to which of the following layers of the anterior abdominal wall

-ltScarpas fascia

B External oblique muscle

C Internal oblique muscle D Transversus abdominis muscle

E Transversalis fascia

Explanation

The superficial perineal space is bound by Colles fascia the fibrous portion of the superficial fascia This

layer of fascia is continuous with Scarpas fascia the fibrous portion of the superficial fascia of the anterior abdominal wall Therefore urine that is deep to Colles fascia will remain deep to Scarpa s fascia The urine will spread in the plane between Scarpas fascia and the external oblique layer

When a horseshoe kidney develops the ascent of the kidney is restricted by the A internal iliac artery B external Iliac artery

C common iliac artery

inferior mesenteric artery

E superior mesenteric artery

Explanation

A horseshoe kidney develops when the inferior poles of the to kidneys fuse together as they ascend into

the abdomen from the pelvis The first anterior midline vessel that is encountered by the horseshoe kidney

is the inferior mesenteric artery This artery prevents the kidney from continuing its ascent

The left testicular vein drains into which of the following veins

A Left internal iliac vein B Left common iliac vein

bflnferior vena cava D Left renal vein I

E Left internal pudendal vein

Explanation

The left testicular vein drains into the left renal vein The right testicular ~i~[~nsltjectlY into the

inferior vena cava This difference in venous drainage is believed to explain the greater incidence of

varicocele on the left side than on the right The venous drainage from the penis is to the internal vein

which then drains into the internal Iliac vein

The spinal nerve that provides cutaneous branches to the skin around the umbilicus is

A TS B TW-shy

C TI2

DL2 EtA

Explanation

The tenth intercostal nerve is the anterior ramus of the TIO spinal nerve After passing through the tenth

intercostal space the nerve continues forward in the anterolateral abdominal wall in the plane between

the internal oblique muscle and the transversus abdominis muscle In the abdominal wall the nerve innervates to the abdominal wall muscles as well as the skin and the parietal peritoneum The umbilicus is

a useful landmark for the region of distribution of the tenth thoracic nerve

The ligament of the vertebral column that resists its extension is the Aligamentum flavum

B supraspinous ligament

C posterior longitudinal ligament

D anterior longitudinal ligament

E interspinous ligament

Explanation

The ligaments of the vertebral column that resist flexion of the column include the supraspinous ligament

interspinous ligament ligamentum fiavum and posterior longitudinal ligament The ligament that resists

extension is the anterior longitudinal ligament This longitudinal ligament is very broad and strong It

covers the anterior and anterolateral surfaces of the vertebral bodies and the intervertebral disks In

addition to resisting extension the anterior longitudinal ligament provides reinforcement to the anterior

and anterolateral surfaces of the intervertebral disk The posterior longitudinal ligament is relatively

narrow and covers the posterior surface of the vertebral bodies and the intervertebral disks This ligament

reinforces the posterior surface of the disk The posterolateral surface of the disk is not reinforced and it

is through this region that herniation of the nucleus pulposus usually occurs

A patient presents with epigastric and right upper quadrant pain The pain is most intense 2-4 hours after

eating and is reduced by the ingestion of antacids The patient states that he has passed black tarry stools

(melena) within the last week Fiberoptic endoscopy reveals a yellowish crater surrounded by a rim of

erythema that is 3 cm distal to the pylorus Accordingly an ulcer has been identified in the patients

A fundus

B antrum

C duodenum

D jejunum

E ileum

A number of physiologic genetic and other factors increase the risk of gastric (and duodenal) peptic

ulcers The evidence that H pylori plays a principle role is compelling Smoking and caffeine are known to adversely affect the morbidity mortality and healing rates of peptic ulcers In general first-degree

relatives of peptic ulcer patients as well as males have a threefold to fourfold increased risk of developing this disorder Paradoxically in gastric ulcer disease acid secretion is not elevated It is possible that

excess secreted hydrogen ion is reabsorbed across the injured gastric mucosa In general a defect in gastric mucosal defense is the more important local physiologic

A patient presents with symptoms of duodenal obstruction caused by an annular pancreas Annular pancreas is caused by

A rotation of the dorsal pancreatic bud into the ventral mesentery B rotation of the ventral pancreatic bud into the dorsal mesentery

fJ failure of the major and minor pancreatic ducts to fuse ~ ~ cleavage of the ventral pancreatic bud and rotation of the two portions in opposite directions around -the duodenum E formation of one pancreatic bud instead of two

Explanation Normally the ventral pancreatic bud rotates around the gut tube to reach the dorsal pancreatic bud The two buds fuse to form a single pancreas and the distal portions of the two ducts fuse The ventral pancreatic bud forms the inferior portion of the head of the pancreas the uncinate process and the major pancreatic duct (of Wirsung) The dorsal pancreatic bud forms the superior part of the head the neck body and tail and the minor pancreatic duct (of Santorini) Annular pancreas is the result of the ventral pancreatic bud dividing into two portions before it rotates into the dorsal mesentery Each portion rotates in opposite directions to get to the dorsal mesentery thus encircling the duodenum The presence of annular pancreas can constrict the duodenum thus obstructing its lumen

In n _ phranlc----

Gon ~l ----_1 Lum bltano

~~--- CornmQ1t bull ac

+-~4--- lnlllirnaJ ilic

xtem iliac

OBJECTIVE - Identify the blood supply to each of the structures listed in the table on the previous page

Ill give you a head start

FOREGUT - Supplied bV Celiac Tru nk (T12)

Proper hepatic

GastiooUod 13Jafter

1nferlor pancreaticoduodenal artery

Common epatlc

Lett gas ric iiirtery

Spfen artery

shy Gastroepiphgtic artery

~ Superior mesenteric 8rtfry

~

1 Esophagus is a derivative of the foregut so its blood supply originates from the celiac trunk

(T12) The predominant blood supply to abdominal portion of the esophagus is the Esophageal

A (Branch of L Gastric) The venous drainage of the esophagus is particularly important because

it is 1 of 3 clinically relevant sites of Portal Caval anastamoses The Portal Esophageal Vein

meets the Caval Azygos System Persistent bleeding manifests as Esophageal Varices - a fata I

condition

2 The Stomach is also a derivative of the foregut has EXTENSIVE blood supply and is very high

yield on anatomy exams The lesser curvature is supplied superiorly by the L Gastric A (1 of 3

major branches ofthe Celiac trunk) and inferiorly by the R Gastric A ( a branch ofthe proper

Hepatic A) The greater curvature is supplied superiorly by the L Gastroepiploic A (a major

branch of the splenic A) and inferiorly by the R Gastroepiploic A

The Short Gastric arteries (branches of Splenic Artery) supply the fundus of the stomach and

are referred to as EIID ARTERIES because they have no collateral blood supply Therefore if the

splenic artery were occluded (ex - increased pressure in the ommental bursa) - there would be

ischemia to the fundus of the stomach Venous drainage of the stomach is extensive via various

veins lead ing to the portal system Posterior to the stomach the IMV joins the splenic V which

joins the SMV to form the PORTAL VEIN ADAMS

3 Duodenum blood supply has high clinical relevance because it is the junction of the foregut and

midgut and therefore is the site of anastamoses between branches ofthe Celiac Trunk (main

foregut artery) and the Superior Messenteric Artery (main midgut artery) The Proper hepatic

artery gives off the gastroduodenal artery which travels behind the 1st part of the duodenum

This point has high clin ical relevance because duodenal ulcers are very common and a posterior

rupture of the 1st part of the duodenum could rupture the gastroduodenal artery causing

traumatic abdominal bleeding The Gastroduodenal artery first gives off the R Gastroepiploic A

(mentioned above) and proceeds as the Superior pancreatico duodenal artery (supplies the

pancreas and duodenum) which anastamoses with the inferior pancreatico duodenal A (branch

of the SMA) This is the junction of foregut and midgut and occurs near the opening of the

bil iary system into the duodenum (ampula of vater) Portal venous drainage here is responsible

for delivering nutrients from digestion to the liver for metabolism Appreciate that the Superior

mesenteric artery (artery of the midgut) branches from the aorta at Ll travels posterior to the

pancreas than moves anteriorly (at the jxn of the pancreatic headbody) and comes over the

3rd4th part of the duodenum Tumor of the head of the pancreas can compress the SMA

4 Jiver blood supply is via the common hepatic artery (major branch of the cel iac trunk) The

common hepatiC becomes the proper hepatic gives off the R gastric A and the Gastroduodenal

A and then joins the common bile duct and the portal vein in the portal triad Clinical- if a

patient were bleeding from the hepatic A a surgeon can stick his fingers in the epiplOic foramen

and squeeze the free edge of the hepatoduodenalligament in order to stop bleeding to the

area Please note that the hepatic a branches into Rand L hepatic A The Right hepatic artery

gives off the cystic artery which supplies the gallbladder Afferent venous supply is via the

Portal vein which is bringing nutrient rich blood to the liver After metabolism takes place

venous blood leaves the liver through the hepatic veins into the IVC PLEASE UNDERSTAND THE

RELATIONSHIP OF THESE STRUCTURES - ADAMSNETIERSNH Etc

5 Pancreas - Head is supplied via the superior and inferior pancreaticoduodenal arteries

(mentioned above) The tail (situated towards the hilum of the spleen) is supplied via the

pancreatic branches of the splenic artery (END ARTERIES) This blood supply is very important

because the endocrine Alpha and Beta Cells from the pancreatic islets of lagerhans are located

towards the tail This is where Insulin and Glucagon is released to the blood

Now complete this for mid and hindgut structures Make sure to note clinically relevant arterial

anastomoses as well as portal caval anastomoses FYI Appendix blood supply SMA + IMA

anastamoses marginal artery Portalcaval rectal veins fhemmorhoids) and periumbilical caput

medusa are high yield THE BUTT THE GUT and THE CAPUT

Abdominal Development

Liver

Ij1f

II wall b

oh liN ~ VltJrti n be- bull

Pancreas

Secondary Retroperitonealization e I~tl r 1 a v-mtrai m ellter

Rotations of the Gut I i Ij (lIl1UtIJ f~ l r tilt

()l td 10 me l-ft and he v

--~--- -~ -~-~

i

I AolaijonjoI~guf I

STOMACH BED (IDENTIFY IN ADAMS)- the structures posterior to the ommental bursa which

support the stomach in the supine position

Abdomnal JQrUI

Splnic vein

OmQ-oul tv~ ) O(s(Jroa)

Lojt(r o m nturrt (hpJtodu o d~n31 Hid

Gadrl)SplerH (g3stroll~nal) IIgam~nt

hiad h~~atogrtricent IIQdmiddotcrt~)

Lt Dome of Diaphragm (why left Look this up in Adams)

Spleen (What is the blood supply)

Left Kidney (What is the blood supply - AND how is it different from the R kidney)

Suprarenal Gland (What is the Arterial AND Venous Blood supply - how are they different)

Pancreas (How does supply differ from Head to Tail What is the SMA Relationship)

Transverse Mesocolon

liver - ADAMSWET - Make sure you look at the liver in wet lab

Left triangular nl1am~nt

ComoaDj ligamnt

Erophg~1 impre$ioo

Hepatio veins

In1erior -ifena middotr3)Ia

Fibrous appendix o-t

live

impr~j on

Heprorendl p~rtion of Q)(Qllary ligament

Righllri~n9ul r 1I~met

(Common) bile quol

Gr)mmCtr~ hepatic dlJct

Ccentic duct

Duodenal impression

GaJdate p-fr)~S

Hepatic artgtrl prop-f iiiiila - Faloiform ligament

_ - shy Round ligamen liver

~--F-- CoJio imprgt-ssi-on

Prta heptis

Identify the lobes impressions and embryonic remnants associated with the liver

Caudate Lobe Quadrate Lobe Right Lobe Left Lobe Round ligament Falciform Ligament

Ligamentum Venosum (what is its fxn in embryonic life) Hepatic Veins (NOT PART OF THE

PORTAL TRIAD) IVC PORTAL TRIAD - Contents relationship cross section etc Know the

Galbladder relationship to the lobes of the liver

Biliary Duct System - Make sure you understand the sequence of these structures - BE ABLE TO

DRAW A FLOW CHART

TPVd i

t

I t

1 __ Cm-(r

patk GlJet

I

J

Clinical = JAUNDICE is caused by anything that prevents delivery of bile to intestine Tumor of the

head of the pancreas Stones etc Patient will have pale stools and yellowish colored mucus

membranes

Clinical- Any scenario that tells you the patient has BILLOUS VOMIT means that the obstruction to

the flow of digestive contents is after the Ampulla of Vater (Site of Entry of Billiary system to the

duodenum) - ie Duodenal Atresia

Spleen -located posterior to the mid axillary line between ribs 9 and 11 Make sure you know that

the 10th rib is the main axis of the spleen and this organ is susceptible to injury (stab wound errant

thoracoce ntesis etc)

The spleen is derived from mesodermal cells - NOT THE GUT TUBE

The spleen rests on the left colic flexure associates with the tail of the pancreas Know the

structures entering the Hilum of the spleen

Sh rt O~-t~ic 1 0(0 10 rtiltSPIric Iloa nt

(cut)

Peritoneum - similar concept to Pleura - think of a fist in a balloon

Visceral Peritoneum - Layer of balloon touching your fist

Parietal Peritoneum - Layer of balloon not touching your fist

Your fist represents the organ your wrist is the hilum and your arm contains the blood supply

entering the organ

Appreciate that there will never be organs in the peritoneal cavity - rather these organs invaginate

the cavity Kaplan videos

RULES OF NOMENCLATUREshy

1 Organ completely surrounded by peritoneum - peritoneal organ

2 Organ partially surrounded by peritoneum- Retroperitoneal

3 Peritoneum surrounding peritoneal organ is VISCERAL peritoneum

4 Peritoneum surrounding retroperitoneal organ is PARIETAL peritoneum

5 Peritoneum connecting visceral to parietal is called messentary 2 messentaries in the

gut Dorsal (to the gut tube) and ventral (to the gut tube) messentary

Aorta is in Retro peritoneal position - but blood must reach peritoneal position - vessels travel through

messentary All peritoneal organs will have blood supply reaching through messentary

-Mesentery is a 2 layer peritoneum with a neurovascular communication between body wall and organ

- Ligament connects one organ with another or to the abdominal wall (Ommentum = ligament)

lesser Ommentum (attach lesser curvature of stomach and duodenum to liver) =Hepatoduodenal

Ligament and Hepatogastric Ligament

Has a Superior and Inferior Recess (Accumulation of Fluid in Ascites)

Communicates with the greater sac through the epiplic foramen (what structures pass through

this foramen)

Boundaries - you must be able to visualize this

o Anterior - stomach

o Posterior - parietal peritoneum pancreas

o Superior - superior recess (bw diaphragm and coronary ligament)

o Inferior -Inferior recess (bw layers or greater momentum

Greater Ommentum (attach greater curvature of stomach) Gastrophrenic ligament Gastrosplenic

ligament gastrocolic ligament

The greater omentum is the largest peritoneal fold It consists of a double sheet of peritoneum folded on itself so that it is made up of four layers The two layers which descend from the greater curvature of the stomach and commencement of the duodenum pass in front of the small intestines sometimes as low down as the pelvis they then turn upon themselves and ascend again as far as the transverse colon where they separate and enclose that part of the intestine

ABDOMINAL PAIN

Parietal Peritoneum - supplied by same vasculature lymphatics and nerves supplying body wall it

lines and diaphragm Sensitive to pain pressure heat cold well localized

Visceral Peritoneum - supplied by same vasculature lymphatics and somatic nerve of organ it covers

Insensitive to touch heat cold and laceration - referred to dermatome of spinal ganglia providing

sensory fibers Where does appendicitis refer to

Foregut pain - epigastric area (ie - cholycystitis)

Midgut pain - periumbilical area (ie - appendicitis)

Hindgut Pain - suprapubic area (ie - diverticulitis)

Extra ImagesConcepts

ll~_____-

FalifCtrm ligament oind r~ud ligamet f Ilver

Blood from splenio gastriC and inferiof rne$e-rteri v~ins

Ca-I tributaries

Lett gastrio Ifein

Posterior superior pan~reatioodul)denal vaihS

Lott gamo-om~nlal (9aropip lomiddotic) -in

Poq_~ tjol imerl-9-r panCJertlcorllJod-nal veiopound --amp----I- - ~J Right grtr~-omntal

Anwrior interi (gartroepiploic) Jjn

pan euaii cod vl)denal veins middot Inf~Ji (t r mesentric vein

Miqdle (olic vein

Right cl)licvein Sigmoid and rectosigml)id (ei ns

IhH)Collc(~io

--- Mi~dl laquooLJl gtjrltgt

PoM ca vl1 illasto)moses -----shyampoptoageal 2 Paraumbilie-lt11 Inferi or Fectal vei ns

3 Recial 4 REuoperHonea1

Know how the Portal vein is formed I 4 sites of portal caval anastamoses and 1 clinical shunt

Col li t ltt-~ otTl~tI ~nj pc~ 1lt1 turJoG

Ltf 14i1 tImiddot~ artoftl9 on tj phtAt$

L-oftqf 4t t~r 1=laquoIran d 1 bull shy~p l ci rj o fOOOts

Nerves follow the arteries - appreciate the splanchnic nervous system I

Uet~ric branch of left ~nal art

Ureterie branch of righi renal artelY

Left Zld lumbar in and co mlTlunication to as)erdin9 lumbar l(~in Hi ~ht tEZ1~~t~ t3r j t itn ~ nJ l1t- rlnd lfe i r1

Inferior me5nteri~ artery

Notice that the right testicular vein drains directly into the IVC and the right testicular artery drains

directly into the aorta However the left testicular vein drains into the L renal vein at a right angleshy

reason left testicle is lower and more susceptible to varicocele (bag of worms)

Also notice that the left renal vein has a longer course because the IVC is on the right side whereas

the right renal artery has a longer course because the aorta is on the left side

Appreciate the anterior to posterior relationship of structures in the hilum of the kidney - VAP - Vein

Artery Renal Pelvis (Ureter)

11____ __ L_ L_ n VJ __ _ _ t_L I I_ _ L __ L_ I -pound1 bull LI_~-I ____

Posterior View of Head of Pancreas in ( of Duodenum

Celiao hunk

Co mmon ~L~jJth art~ry

GastNduQdonal artrf (partilly in phantn)

P1)Sterior $Up~Jior panCflaticuduodfmal art~r~t

(Co mm on) bile duct

middot~1t~~t-1l---~-~- Right gshomiddotomental (gastoe plp lolc) 3rte (phantomost)

Grener paocre atic art-ry

1n1~rjor pancr-iatlc artery

Jtrifll supejo r pal)oreailcento)dJodenal artr1 (phantom)

Anastomotlo branch

POostetlor bJanch of jo f~ri of pan-reatir(lduodensl drttnj

Anterio r branch of i flferior palcreati~)duodenal art~(phan1om)

Notice the extensive blood supply to the pancreas and duodenum via the branches of the celiac trunk

Notice collateral supply from SMA branches - makes sense bc this is the jxn of foregutmidgut

Identify the vessels in this arteriogram

Hiltid i)f N~ck oi B)dvof Tail 01 pa nereas pan cent~as P-nmiddot-reas panCtCas

I nferie v~na cava

jHept1iic p(lrlai v~in

Port1 tnd H~pti lt a ftH prol

Comm on) bll duct

Ouodtnum

~ft colic (sio)Atta~ hmtrlt jt~xJr-ofha~elSe

muo(IIQn

Right ~lIc (h~j)tic)

il~gtture

In1triol m~oten lIein (rttr op~ritoMdO

SlJp efl or mes~n~fiC amrV and lipln

KNOW YOUR NEIGHBORHOOD

Questions

vVhiJh structure supplied by a bnmdlof the cclia( artery is not derivcd from foregut LemCJUCrITI

(A) Head of the pancte-a5

CD) Pyloric duolenum

Cystkduct

( Liver hepatocyt~~

~F) Body of the spleen

An infant presents with an omrhaJucele at birth -hi oJ the [oHm illg applies to his cM1-dition

(A) It is 31so seen ill p4titnts with aganghonic megacolon

(11) ft reuirs from a fal1ure of resorption of theviteUine d let

(C) It results from herniation at the-site of regression of the right umbilk vein

DJ It is caustd by faihtrc of recanalization of the midgut part of the duodenum

~ It ill camioo by a failuIt vf the midgul to return to the abGQminal uity after herniashytion in-n the urnbilk s l stalk

Ot er than the spleen occlusion Cif the spit-Ilk artery at its odgin wm most likely affect die blood supply to jllch st cnud

(A) Jejunum

(B) Body of th pal1~lltas

(C) LeSStT Cllmiddotlaturc of tl )toma-ch

(D Duodenum dista to the entrance of the Ornmou bile duct

E Fundus of the stomach

A 38-yeu-old batL~er with a history of heartburn suddenly experiences excluciating pain in the (plgastric region of th~ abdomeu SurgCry is perf~rme immediard y upon admisshysion to the 1IlcrgCJliy tuomh~re i~ evidence uf a ruptured ulcer in the posterior waU of the stomach Vhere will a surgeon first fi nd the stomach contenlSf

A) Greater p4ritoneal sac

rB) Cul~de-s~c of Douglas (--

C Omental bursa ~

--D) Paracolic gutter

rEj Between -he panttal perimltum and the posterior body wal1

At birth an infant presents with a st()ma~ rb~tbas~njJled jfltotb~diaplfagru 1A1ltre is the defect thatresulied iiitJle heini~t()n shy~tsophagealbiatus

7 - rH-- Hiatus for the inferior vena cava

( Pleuroperitoneal membrane -(0) Septum transvcrsum

(E) Right Crlt~

An infant born with DOVv7l syndrome presents with bili()u~ vomiting Ahat congenital defect does the infant have

(A) Pyloric stenosis

(B) Meckel diverticulum C) Ornphaloce1e

(D) Gastroschisis

( ~ ) Duodenal atresia y A patient with cirrhosis of the liver presents with ~ bacalvaricestnlreased retrograde pressure in which veins caused the varices

(A) Paraumuilical

(B) Splenic

(ct AzygltJus

(15))G~trk ( (-F) Superior mesemeric

A htaltby 3-year~old male patient experiences a hernial sa protruding from the anterior abdominal wall about halfway between me anterior superior ilia spine and the pubk tuberde Pulsations of al1 artery are palpated medial to the protrusion site through the abdominal walL Which layer of the anterior abdominal wall will first be traversed by the

1hctma

fA) Rectus sheath (B) External oblique aponeurosis

(C) Inguinal ligament

lD) Transversalis fusda

(E) Cremasteric fa~cia

After 5urgi(aj ffpair of a hernia the patient tXperienccs mtmlgtness in the skin on the anteshyrior aspect of the S(Totum_ Vhaf nerve may have been lesioned during thehemiorrhaphy

(A) Femoral

(B) Obturator

(C) Ilioinguinal

(D) lliohypogastrk

(E) Pudendal

A 23~year-LJld female secretary il1 good health ~-uddcn1) doubles over with pain in the a ea of the 1JmbRicu$ Sbe feels vartn and ltneasy and has no appetite That night the pain seems to have mQved to the tower right abdominal regjol1 and she calls her family doctor who then arranges for an ambulance to pk-k her up and take her to the hospitaL Wh ell ntn~ perceived in the area of the urnbilirus most Hkely carried lhe pairfu I sensations into the eNS

tA) Vagus nerves I~

V B)

) Lessersplanchnk nerves

tC) Pudendal nerves

(D) lIiohpogastrk nerves

(E) Greater splam ic l erves

A CT reveals carcinoma in the bOod of the ancreas Vhich blood vessel trut ourses ----~- - -bull ------ --shy

immediately poftterior to the body ofthe pancreas is the m~t likely to be oompressed

(A) Splenk artery

(B) Abdominal aorta (C) Portal vein

(1) Splenic vein

(E) Renal vein

A patient has a penrln1l1ng uker of the posterior wall ot the br~l part ot the (lUooenmn llkh blood vessel is subject to erosion

(A) Common hepatic artery

(B) Gastroouodenal artery

(C) Proper hevatic artery

(D) Celiac artery

(E) Anterior inferior 11amrelltlcoduodcnal attery

Your patient has been diagnosed -ith a carcinoma locallted to the head and l~e(k of the pancreas Another clinical sign would be

A esophageal varices

(8) hemorrhoids

C) a caput medusa

(D) increased pra Teuro n th~ hepatic veins

(E) enlarged right supra lavkular lymph nodes

Wltkh of the foUowing structures develops in the ventral mesentery

(A) Spleen

(B) Jeiunum (C) Head of1ht pancreas (D) Transverse colon (E) Stomach

ti l Uw ~ littwin~ f( S-t lil oai Imdge ~ hi(h or tbt la~)d J truetur tgt liJ llntn nl) he hl p UC iJd [IIi ell

c o

A) drains Ie tht infCrior a La aI

R t middot~nfl0 ~ill to th~ lunlgtn of h i dtlndCrlllfH

(e) m t bull JiJattd on tl l J n T ~H

D ) sup Lc O VSlt I Hlid bhtu l 1 li - -I un oid

( ) U~tpli(t tr j middottUh~ 1 v(( b~nt rfK n1ilc~Zm

ANSWERS AND EXPLANATIONS

Answer E The spleen is t hlttnopodicand lymph organ demlted from mesoderm

Answ~ R Al1 tlmphalocele is caused by it failure of the nlidgut to return to the ahdomir nat cavity after herniation into the umbiliau Stalk Choices Aand D maybe seen in infants with Down syndrome choice D ~s the specific CBuse ofduudcnal JtiCSitt Choice C is (ile cause of gclstrosbisis and Choice B nsults iu a Meurolktldivertku1-tlB

Answer B The fundus ofthe stomach is suppHed by soort gastric brunches of the splenic altery The splenic artery supplies the body and tail of the pancreas part of the greater curvature of the sttmla(h and the spleen Te jejunum part of the head of the pancreas and tht~ duodenum distal to the entrance of the commOll bile duct are supplied by the superior mesenterk artery clll~l ~be less r ctlt1ature cmd the pylQric antrum are supplied by the right and lei gastric art(ries

AnSWftt C Tbeomental bursa or lesser ~ritoneaj sac lies direcdy posterior to the proxshyimal part of the duodeTtlm and the stomach and would be the first site where stomach contents ~Ott1d be fpoundluncL

Answer C A defect in a llleuropcritoneal membrane (uswlly the left) is the typical site of i1 cc-ngenitlI diilphragluatic hemia llere the membr4ne fails to dose ()pound( of the perishycCirdiopcritulleal canals

Answer E DuoJenal atresia and aganglionic megacoion are congwitaI defects S~Il in patients with Dowmiddotnsyndrome

Answer D RulaTgemt~llt of and retrograde flow in g~lstrk vel_ns in particlJl~r the kft gas~ tricveins dilates the capillary bed in rhe wall of the esophagus in (ases of porta yper~

tension Blood flow would increase in and dilampte tribntarkgts of the (lZygOUS vein on the other side of the capiUary bed but flow in this vein is in the typical direction t()ward the superior vena cava Paraumbiii(ltU vein eilgorgement contributes to a caput medusH Splenic ~nlargement might prc~nt with 5plcnonlegaly and balt-kflow in to tlu superior m~~ntclic vein occurs but is asymptomatic

Answer D The patient hagt an indirect inguinal hernia whi~h emerges from the antt-rior abdominal wall through the deep inguinltilling Theeep ring is a fault in the transv~rshysaUs fascia this I~yer wiIJ be penetrated first by the hernia

An~Wer C The ilioinguinal nenc which provides sens~llion to the lnedlal thigh ltmclanteshytior SClotunl pass~lt th rough the 5uperfh_ial inguinal ring ind $subject to inj i1T) becaus-e

it is in the operatitm Held of the erniorrhapny

Auswer B The leMHr splanchnic nerves are sympathdic nerVlts that carry viscera l sensashytlltgtrogt ftom illtllt1m~d ()J stietched gust (itinteitinal ~tructures (in this case the pprndix) into tnt eNS Lesser splanchnic ntTYcsarisc from thmiddot T9--T12 spinal cord segments lt1nd provide sympathetic innenation tD rnidgut siruc1ures whiCh include CLe app~JldD Viscera] Pain arising from affecLed Inidgut ampt 1C1ure is referred over the same dl- matorne~ of spinal segrnertts v-hich provide the sympathetic Innervation n this G1SC of appendicitis the invohen~n t of the ltire) of t e unlhHku indud s the T 10 dermatome

Answer B Of the five choices onty the dscending olon is retroperiton~al aldwould be a lik ~ ( choice to be seen immediately a(~jilcent to t11e posterior abdominal middotn~L

Amwen D The SpltftlC ~-ein ourses posterior to the body of the panneas m its way tt drain into the superior mCSfttltlri( vein

Answcr B TILt glstrodllolticnal artery 1 direct hIamh of the comrootl hepatic artery courses immediately pt))iwri() to the duodenum and is slbject to erosion

Answer B Carcinoma of th pan middott3S in the 1tilt1 may compreampgt the portltil vein at irs orishygill The poTtai vcin is fomled when the splenic vein jQiaswith tfie superior meStllt eric vein The inferiot mesenteric vein joins the ~plenjc vein just priOT to tlli~ point at which the splenic joins the superior Jlleit1ltcri( vein Increescd venous presslu in the inferior mesenteric vein is a cause of emo hoid~

Answer C The- velltral pancreas wilich forms most of the head of the p ~ncr as develops in the ventral mes(ntery as antutgrowth of the hepatic diverticulum Th~ hepatic divershyticulull induding the biIJary appa~atus develops in tbe ventral mesentery of the foregut

Answer~ A The superior mesenteric ~in joins with the spienkvein to form the hepatic portal vciu

Answer D The structure at gttlK is the proper hepatic artery~ whkh suppUesoxygenated b middotood to the liver

MAKE SURE YOU KNOW the diff bw Rectus Sheath above and below the arcuate line

ABOVE

Aponeurosis of xiiltmal obllque musclo

Extemll f)biquw musde

Reotln ilbdomlnls musole S~in

Internal 9bliquQ mY~QI

AponeUfOsi$ of hJH$V~~S Lir9a a lb lbdolTlin~ musolo Tri OJV6 rUi

atldomlnis mUS(loe

Sub cutanlilous tiue (tatty ye r)

BElOW

A POrl lJfosis 01 etemal oblique muscle

Aponeul~)sis 01 Internal oblique mU$cl~

Anteriol lay~ of r~ltdus st~ath EXttom1 oblique rnu$cll

Rectus Jbdominis muscle Intoernal Aponeurc-sis of tra~fersU$ oblique muscle-

at-domlnis muscentl ~ Skio

Tra nsvitSus abdomioLs ml)ZClt

TralSVersaHs fascia Medial umQil iegtt1 1i9Jment -and folj

Uldchus Peritoneum (ir median Umbilj~al Suboutane ous

Extraprftone 11ascia

Ymbilimiddot~1 fold)

preu9poundiea1 fascia

tissue (fatty 4nd m~mbr3n(iUS layers)

o Above the arcuate line (A horizontal line 13 of the distance bw the umbilicus and the

pubic symphysis) -10 Aponeurosis divides into an AntPost Laminae

o The Ant Laminae joins EO and Post Laminae joins Trans Abdominis = Ant and Post

RECTUS SHEATH respectively

o BElOW the arcuate line - all 3 aponeurosis join ANTERIOR to rectus muscle to meet its

counterpart in the midline (linea Alba)

o Take away Msg - The abdomen is devoid of a posterior rectus sheath below the

arcuate line and is therefore more vulnerable to herniasinjuries

Question - A physician makes a deep incision in the patients midline immediately superior to

the pubic symphysis which of the following layers is his knife least likely to pass

Rectus Abdominis External Oblique Ant Rectus Sheath Posterior Rectus Sheath All of the

Above

Answer - All of the above None of the other answer choices are midline structures -LINEA

ALBA

Linea Alba has very poor blood supply - doesnt heal well after surgery Therefore this is a

common site for incisional hernias

a Spleen b Transverse colon c Descending colon d Stomach e Pleura

17 Meckels diverticulum is normally found 2 feet proximal from the

a Pyloric sphincter b Lower esophageal sphincter c Ileo-cecal valve d Middle valve of Huston e Anal valve

18 Ulcer in the posterior wall of the first part of the duodenum would erode ___ artery and would cause bleeding

a Left gastric b Right gastric c Hepatic artery proper d Gastroduodenal artery e Middle colic artery

19 An inflamed appendix is identified by a surgeon on the operation table by noting

a The appendicies epiploicae b The convergence of tenia c The artery of Drummond d The mesocolon e The mesosalphinx

20 The nerve which emerges through the psoas major is

a Femoral b Ilio-inguinal c Ilio-hypogastric d Pudendal e Subcostal

21 The right gonadal vein drains into the

a Azygos b Hemiazygos c Inferior Vena Cava d Right renal vein e Left renal vein

22 The hepatocytes in the liver is derived from

a Ectoderm b Endoderm c Mesoderm

d Neural ectoderm

23 Abscess in the lumbar vertebrae due to tuberculosis would spread to the adjacent muscle which is

a Psoas Major b Iliacus c Quadratus lumborum d Tranversus Abdominis

24 The anterior wall of the inguinal canal is formed by

a External oblique and transverses abdominis b External oblique and fascia transversalis c Internal oblique and external oblique d Internal oblique and transverses abdominis e Fascia transversalis and peritoneum

Meckels diverticulum is a result of which of the following developmental abnormalities shy

A Failure of the vitelline duct to close

B Failure of the herniated intestinal loop to retract into the abdomen

C Failure of the urachus to close

D Failure of the midgut to rotate

E Failure of the hepatic duct to close

Explanation

Meckels diverticulum is a result of the persistence of the proximal part of the vitelline duct This

diverticulum is usually found about 2 feet proximal to the ileocecal junction and is usually about 2 inches

long It is present in about 2 of the popUlation It may be the site of ectopic pancreatic tissue or gastric

mucosa and may develop inflammatory processes and ulcerations Acute Meckels diverticulitis

simulates appendicitis

Which of the following veins carries blood from the esophagus to the portal vein The

A right gastric vein

B left gastric vein c splenic vein D azygos vein

E left gastroepiploic vein

Explanation

The left gastric vein a direct branch of the portal vein drains blood from the lesser curvature of the

stomach and the inferior portion of the esophagus Because branches of the portal vein do not have

valves blood can flow in a retrograde path when there is an obstruction to flow through the portal system or liveL Rlooci Cln then flow from the nortl] vein thr()1Ph the left PRstric vein to the esonhlPlIS lno

through venous communications within the submucosa of the esophagus to esophageal veins that drain

into the azygos vein The increase in blood flow through the esophageal submucosal veins results in esophageal varices

On the posterior wall of the abdomen the celiac ganglion A contains cell bodies of postganglionic parasympathetic neurons B is synapsed upon by neurons in the posterior vagal trunk C is synapsed upon by neurons in the greater splanchnic nerve D contains sensory cell bodies of lumbar spinal nerves E contains cell bodies of neurons that cause an increase in the rate of peristasis

Explanation The celiac ganglion is one of the preaortic ganglia of the sympathetic nervous system It contains cell bodies of postganglionic sympathetic neurons The sympathetic splanchnic nerves contain preganglionic sympathetic neurons that pass through the sympathetic chain without synapsing These splanchnic nerves go to the preaortic ganglia to synapse The greater splanchnic nerve contains preganglionic neurons from spinal cord segments T5-T9 This nerve synapses in the celiac ganglion The nerve fibers in the vagal trunks are preganglionic parasympathetic fibers that go to the walls of the organs that they will innervate and synapse on postganglionic parasympathetic neurons in the walls of those organs Cell bodies of sensory neurons in the abdomen are found in the dorsal root ganglia or the sensory ganglia of the vagus nerve Sympathetic innervation decreases the rate of peristalsis parasympathetic innervation increases the rate of peristalsis

Which of the following pairs of arteries will allow blood to bypass an occlusion of the celiac trunk

A Left gastric artery-right gastric artery

B Left gastroepiploic artery-right gastroepiploic artery

C Superior pancreaticoduodenal artery-inferior pancreaticoduodenal artery

D Splenic artery-common hepatic artery

E Left gastric artery - proper hepatic artery

Explanation The anastoOlosis of a branch of the celiac trunk and a branch of the superior mesenteric artery will

provide collateral circulation around an occlusion of the celiac trunk Each of the other choices pair

branches of the celiac trunk therefore these will not provide collateral flow around the obstruction of the

celiac trunk The left gastric splenic and common hepatic arteries are direct branches of the celiac trunk

The right gastric artery is a branch of the proper hepatic artery which is a branch of the common hepatic artery The left gastroepiploic artery is a branch of the splenic artery The right gastroepiploic artery is a

branch of the gastroduodenal artery whlch is a branch of the common hepatic artery

Which of the following organs has appendices epiploica The

A sigmoid colon

Bjejunum

C duodenum

D stomach E esophagus

Explanation Appendices epiploica are characteristic of the colon Appendices epiploica are subserosal accumulations

of fat None of the organs of the gastrointestinal tract has appendices epiploica except the colon

Page 13: Chirag's Abdomen Review

o Only hernia that can transverse the inguinal canal o Associated with congenital condition persistent tunica vaginalis

bull Direct inguinal hernia o Medial to inferior epigastric a o When inserting finger in superficial inguinal ring will feel on back of finger o Associated w old age or recent surgery

Muscles (Abdomen RECTUS SHEATH)

bull Arcuate line at level of ASISor 13rd distance between pubis and umbilicus bull Above arcuate line rectus abdominus is surrounded by a rectus sheath anteriorly and

posteriorly

o EO and 10 lie over rectus abdominus o 10 and TA lie behind rectus abdominus

bull Below arcuate line rectus abdominus has no rectus sheath posteriorly o EO 10 and TA lie over rectus abdominus o Transversalis fascia lies behind rectus abdominus o Inf EpIgastric vessels pierces the rectus sheath here

Peritoneum serous sac which encloses most of the abdominal structures

bull Ovary =only intraperitoneal organ o Oocyte ejected from ovary then captured by fallopian tubes o Why impt Women more prone to infection that can enter peritoneum

Peritoneum forms

bull Mesentery double layered fold of peritoneum formed as the organ was pulled in

bull Ligament between 2 organs in general bull Omentum between stomach and another organ bull Bare area area of no peritoneum bull

Viscera innervation

bull Pa rasympathetics 11 o Afferents sense hunger o Efferents l peristalsis relaxes sphincters gland secretion

bull Sympathetics o Efferents do opposite o Afferents CARRY PAIN SENSATION OF THE VISCERA (dull stretching pain)

bull PARASYMPATHETIC INNERVATION o Vagus nerve 7 _1l to 23rd unct ion of la rgej nte~tine oJ)elVrcspla~~~)~~ic~rYe~ IJiU- ~rd aJ~lpoteotiD~ IMPT

Gut Embryology

Gut ~ We say that the gut is derived from endoderm We often forget that when we say so we mean

that only the mucosa is derived from the endoderm The submucosa and the muscle layer is actually derived from the splanchnopleuric mesoderm and the serosa is derived from the visceral peritoneum

~ The main function of the gut is to digest the food which is done by the glands derived (and are) in the mucosa (endoderm) The only two exceptions in the Gut where glands though derived from the endoderm do not stay there but migrate down into the submucosa are esophagus and duodenum These glands however have their ducts opening to the swface of the mucosa

bull

~ Lungs liver amp gall bladder and pancreas are off-shoots from the foregut Esophagusshy~ The region of the tube from the laryngeal diverticulum to the beginning of the stomach elongates

to form the esophagus ~ The glands which form in the endoderm (mucosa) migrate down into the submucosa The path

whlch it took migrate becomes the duct of the glands which open to the mucosa ~ Achalasia Cardia - Failure of relaxation of the lower esophageal sphincter because of congenital

absence of ganglia at the sphincter (The ganglia when present releases VIP (Vaso-IntestinalshyPeptide) which relaxes the sphincter)

Mid-Gut Rotation ~ Because of the 90 degree rotation of the primitive stomach all of the following events occur ~ Lesser curvature comes to the right Therefore lesser omentum also comes to the right ~ Greater curvature comes to the left Therefore greater omentum also comes to the left ~ Right side vagal trunk becomes posterior vagal trunk ~ Left side vagal trunk becomes anterior vagal trunk ~ The left side peritoneal cavity comes to the anterior aspect of the stomach and will later be called

as the greater sac ~ The right side peritoneal cavity comes to the posterior aspect of the stomach and is (relatively a

small sac because the liver is on the right) called the lesser sacomental bursaepiploic sac ~ Epiploic foramen of Winslow (the lower free margin of the ventral mesentry) wiII be the

communication between the greater and lesser sac ~ The Liver moves to the right and therefore actually causes the 90 degree rotation of the stomach

The spleen comes to lie on the left side ~ Axis Antero-posterior axis around the superior mesenteric artery

bull Counterclockwise bull Approximately 270deg bull During herniation (about 90deg) bull During return (remaining 180deg)

Duodenum ~ Becomes retroperitoneal (except the first part which is still suspended by the hepato-duodenal

part of lesser omentum) ~ Glands (of Brunner) go submucosal ~ An imaginary line drawn below the opening of the major duodenal papilla represents the junction

between the foregut and midgut ~ Duodenal atresia in Downs syndrome Liver ~ 3rd week

bull liver bud grow bull into the septum bull transversum

~ 10th week bull hematopoietic bull function

bull 10 of the total bull body weight

~ 12th week bull bile is formed

Pancreas ~ In about 10 of cases the duct system fails to fuse and the original double system persists ~ 3rd month

bull pancreatic islets (Langerhans) ~ 5th month

bull Insulin secretion ~ Annular pancreas

bull The right portiCn of the ventralbud migrates along its normal route but the left migrates in the opposite direction

~ Complete obstruction of duodenum ~ Accessory pancreatic tissue Polyhydramnios (Amniotic fluidgt 1500-2000 ml)

~ Congenital defects including central nervous system disorders (eg Anencephaly) and gastrointestinal defects (atresias ego Duodenal esophageal) prevent the infant from swallowing the amniotic fluid (failure of recanalization)

Oligohydramnios (Amniotic fluid lt 400 mt) ~ Cl~ldberenal-agenesis

bull Midgut _-_

~ Primary Midgut intestinal loop gives rise to bull Distal duodenum bull Jejunum bull Ileum bull Ascending colon bull Transverse colon - proximal two-thirds of the bull Transverse colon with the distal third

~ Primary intestinaltoop bull ncephalic limb distal part of the duodenum the jejunum and part of the ileum bull ncaudal limb lower portion of the ileum the cecum the appendix the ascending colon and

the proximal two-thirds of the transverse colon bull 6th week

bull Rapid elongation of the cephalic limb bull Rapid growth of the liver bull Intestinal loops enter the extraembryonic cavity in the umbilical cord

bull 10th week bull loops begin to return bull regression of the mesonephric kidney reduced growth of the liver expansion of the

abdominal cavity bull Jejunum -left bull Loops - more to the right

bull Cecal bud -last part (temporarily below the right lobe of the liver) ~ qIDlthaloseJe (Structures COlHLoArts9V~1tion)

bull Through umbilical ring bull 6th to 10th weeks

bull Associated with a high rate of mortality (25) and severe malformations bull Associated with chromosome abnormalities

~ Gastroschisis (Structures coming out are not covered by Amnion) bull herniation through the body wall ----=---=-shybull Into the amniotic cavity bull Lateral right of the umbilicus bull Sometimes the inferior wall fails to develop as a result lower abdominal structures like the

bladder would be exposed to the exterior not associated with chromosome abnormalities ~ Abnormalities of the Mesenteries

bull Mobile cecum persistence of mesocolon bull Extreme form - long mesentery bull Volvulus

~ Distal third of the transverse colon ~ Descending colon ~ Sigmoid colon ~ Rectum ~ Upper part of the anal canal ~ Primitive anorectal canal

bull 7th week cloacal membrane ruptures bull Tip of the urorectal septum perineal body bull Pectinate line

~ Hindgut anamolies bull Rectoanal atresias and fistulas bull Imperforate anus bull Congenital megacolon (aganglionic megacolon Hirschsprung disease)

bull

bull Hindgut

Chirags Abdomen Review - Part 2

Understanding Embryo makes learning blood supply EASY

I I

I t

~ -

)

Table l1r-~ L Adult SUmiddotuctu~SDrj~l Froln Each of he Three Dhisions of be Pringttive GUl Tube t-middot-----middotmiddotmiddotmiddot-

Foregu(

I_ (Celiac Trunk)

Ir-slt-gtphgus

S101na(b

I h -= LiJ~r

Pancre=l S

bull 1 i Biliary apparntu5

Gall bladdshy

i Pha11~Cal pltgtuchcs

LullSS-I

Mjig ---- bull __ _- ----n--duct----~---middot-------l--n

(Superior Jldesen1eric Artery)_-1I-(I_~__ middot __ O-=-)_in_middoto_r_M_e_se_n_t_e_r_i_c_An__

Uuodenu rn 2nd_ 3 lt141h V4Tt

Jejunun-~

nc-un]

tCCUJ11

AppltgtndLX

Transver5e -o1on (p~oxiln1l1 ~O Tbird)

bull__hytgtid~ _ _ ______ L _ __

Tr-dn~llt~se colon (diStul h lTd) I

)

i

Aa ca-nal -( uppeT patt) i

I I

_____ __ _ _ _ ___ - - rhe~ a(t clcriVOkt iV(5 opound~lt prbn1rC ~ nlQC blft TlI)( 134tof r~ tIonoinf~ i 1 ~l l1rd c- P Cle

Now Lets see how much youve learned

Questions

1) A pt receives a general anesthetic in preparation for a c~t~~my A right subcostal incision is made which begins near the xyphoid process runs along and immediately beneath the costal margin to an anterior axillary line and transects the rectus abdominus muscle and rectus sheath At the level of the transpyloric plane the anterior wall of the

-~~-~=--- _eco---shysheath of the rectus abdominus muscle receives contributions from which of the following

a Aponeuroses of the in~ande~tef-Ilal o~ues

b Aponeuroses of the transversus abdominis and internal oblique muscles c Aponeuroses of the transversus abdominis and internal and external oblique

muscles d Transversalis fascia e Transversalis fascia and aponeurosis of the transversus abdominus muscle

A

2) The lat~raJJJ11QjJt~gLfgJlLoneach side of the inner surface of the anterior abdominal wall is created by which of the following structures

K Falx inguinalis (~) Inferior epigastric a

c Lateral border of the rectus sheath d Obliterated umbilical a e Urachus

B

3) A man the victim of several knife wounds to the abdomen during a brawl at the Lobster Shack subsequently developed a direct inguinal hernY Damage to which of the following nerves is most likely responsible for the predisposing weakness of the abdominal wall

~ Genitofemoral nerve ( b) Ilioinguinal nerve ~-t Tenth intercostal nerve

d Subcostal nerve e Pelvic splanchnic nerve

B

4) Which of the following statements concerning a direct inguinal hernia is correct a It is the most common type of abdominal hernia b It transverses the entire length of the inguinal canal c It contains all3 fascia layers of the spermatic cord d It exits the inguinal canal via the superficial ingeJinal ring e It protrudes through H~acb strJg e

~(

1fltbS w E

tl

5) The conjoint tendon is

a Important in preventing indirect inguinal hernias b The fused aponeurotic layers of internal abdominal oblique and transversus

abdominus muscles c Posterior to the deep inguinal ring

d Medial fibers of the inguinal ligament

B

6) A 25 year old male is brought in to the ER after being involved in a car accident in which he received a crushed internal injury in his abdomen Examination reveals a lesion of parasympathetic fibers in the vagJsnerve which interferes with glandular secretory or

smooth muscle functions in which of the foliowingorgans a Bladder b Transverse coloiW c Descending colOO d Prostrate gland e Rectum

B

7) The spermatic cord includes all of the following contents except a Il ioinguinal nerve b Pampin iform plexus of veins c Vas deferens d Genitofemoral nerve

A

8 Which abdominal structure gives rise to the internal spermatic fascia (muscle) following the descent of testes in development

a External abdominal oblique aponeurosis b Transversalis fascia c Transversus abdominis muscle d Peritoneum e Internal abdominal oblique

B

9 Which abdominal structure gives rise to the tunica vaginalis fotlowing the descent of testes during development shy

a External abdominal oblique aponeurosis b Transversalis fascia c Transversus abdominis muscle d Peritoneum e Internal abdominal oblique

D

10) The lesser omentum is a peritoneal fold which is su bdivided into the a Hepatogastric and gastrosplenic ligaments b Hepatoduodenal and gastroomentalligaments c Hepatoduodenal and gastrosplenic ligaments d Hepatogastric and hepatoduoden9-jrj igaments

D

11) A posteriorly perforating ulcer in the pyloric antrum of the stomach is most likely to produce initiallocalized peritonitis or abcess formation in which ofthS fQllowing

a Great-sac - -- -

b Paracolic recess

c Omental bursa

d Right subphrenic space

c

The inferior mesenteric artery arises from the abdominal aorta ilm_ediill~y_J-Qs1eriQLto which of the foowing org~ns A-F~t~filie duodenum B Head of the pan~eis C Neck of the pandeas

D Second part of the duodenum

E Third part of the duooenum_shylaquoshy

shy

The correct answer is E The inferior mesenteric artery arises from the anterior surface of the aorta at the level of the third lumbar vertebra The third part of the duodenum crosses the midline at the level of the third lumbar vertebra and passes anterior to the aorta at the origin of the inferior mesenteric artery The

first part of the duodenum (choice A) lies horizontally to the right of the midline at the level of the first

lumbar vertebra The head of the pancreas (choice B) is to the right of the midline and extends from the

level of the first lumbar vertebra to the third lumbar vertebra It lies within the concavity of the

duodenum The neck of the pancreas (choice C) lies in the midline at the level of the first lumbar

vertebra It lies on the anterior surface of the aorta at the origin of the superior mesenteric artery The second part of the duodenum (choice D) lies vertically to the right of the midline and extends from the

level of the first lumbar vertebra to the level of the third lumbar vertebra

The left adrenaLvein drains directly into which of the following veins A Hemiazygos vein

B Inferior vena cavaee C Left renal veiri -

D Splenic vein

E Superior mesenteric vein

a

The correct answer is C The left adrenal vein and the left gonadal vein (either testicular or ovarian) drain into the left renal vein TheTeft renal vein t~ains intothe- inferior vena cava In contrast the right

adrenal ~~inandnght gonadal veindrai~ gLr~ctJy iQtoJhe iilferiQ[ Vencava -- -

ThehemTazygoS7ein- (~h-~i-~ A)~~c~i~es the venous drainage from the body wall on the left side of the

thorax and abdomen No visceral organs drain directly to the azygos or hemiazygos veins The inferior vena cava (choice B) receives the direct venous drainage from the right adrenal vein but not

the left adrenal vein Remember the inferior vena cava is on the right side of the abdomen The splenic

vein (choice D) receives the venous drainage from the spleen and part of the pancreas and stomach The splenic vein is part of the portal venous system

The superior mesenteric vein (choice E) receives venous drainage from much of the intestinal tract It is part of the portal venous system and joins with the splenic vein to form the portal vein

A 43-year-old man presents complaining of pain in the groin On examination his physician palpates a

bulge in the region of the superficial inguinal ring which he diagnoses as a direct inguinal hernia The hernial sac most likely

A is covered by all three layers of the spennatic fascia B passes medial to the inferior epi gastric artery

C passes medial to the lateral border of the rectus abdominis muscle

D passes posterior to the inguinal ligament E passes through the deep inguinal ring

The correct answer is B Direct inguinal hernias enter the inguinal canal by tearing through the posterior

wall of that structure The typical location for this type of hernia is through the inguinal triangle bounded

laterally by the inferior epigastric artery medially by the lateral border of the rectus abdominis and

inferiorly by the inguinal ligament Direct inguinal hernias pass medial to the inferior epigastric artery

whereas indirect inguinal hernias pass lateral to the inferior epigastric artery because the deep inguinal

ring is lateral to the artery Indirect inguinal hernias are covered by all three layers of the spermatic fascia (choice A) Direct inguinal hernias are covered by fewer than all three layers because the direct inguinal

hernia tears through one or more layers of fascia as it emerges though the abdominal wall The lateral

border of the rectus abdominis muscle (choice C) forms the medial border of the inguinal triangle All

inguinal hernias pass lateral to the rectus abdominis Femoral hernias pass posterior to the inguinal ligament (choice D) Inguinal hernias emerge through the superficial inguinal ring which is superior to the inguinal ligament Inguinal hernias that descend below the inguinal ligament pass anterior to the

ligament Indirect inguinal hernias pass through the deep inguinal ring (choice H) direct inguinal hernias

do not Both types of inguinal hernias pass through the superficial inguinal ring

During a gastric resection in a patient with stomach cancer a surgeon wants to remove the lesser

omentum because of tumor extension into it Which of the following structures lie in the free edge of the

l~~g omentum and consequently must be dissected out in order to be preserved

A Common bile duct cystic duct and hepatic artery 6

B Cystic duct hepatic artery and hepatic vein

e Hepatic vein and cystic duct

Portal vein common bile duct and hepatic artery

E Portal vein hepatic artery and hepatic vein

The correct answer is D The free edge of the lesser omentum contains three important structures the

common bile duct the hepatic artery and the portal vein Nei ther the cystic duct (choices A B and C) nor the hepatic vein (choices B C and E) lies in the free

edge of the lesser omentum

A 55-year-old male patient with chronic liver disease has portal hypertension To relieve the pressure in the portal system a porto-caval shunt is performed Which of the following veins may by anastomosed to

accomplish this porto-caval shunt A Left renal vein-left testicular veingt

B Right renal vein-right suprarenal vein I shy

e Splenic vein -left renal vein J

D Superior mesenteric vein-inferior mesenteric vein E Superior mesenteric vein-splenic vein

The correct answer is C The splenic vein drains directly into the portal vein The left renal vein drains

directly into the inferior vena cava Anastomosis of these veins would allow blood from the portal vein to

drain retrograde though the splenic vein into the renal vein and then into the inferior vena cava The left

renal vein (choice A) drains directly into the inferior vena cava The left testicular vein drains directly into

the left renal vein Thus these veins are already in communication and neither vein is part of the portal venous system The right renal vein (choice B) drains directly into the inferior vena cava The right

suprarenal vein also drains directly into the inferior vena cava Thus neither vein is part of the portal

venous system The superior mesenteric vein (choice D) drains directly into the portal vein The inferior

mesenteric vein drains into the splenic vein which then drains into the portal vein Thus neither vein is

part of the caval venous system The superior mesenteric vein (choice E) drains directly into the portal

vein The splenic vein also drains directly into the portal vein Thus neither vein is part of the caval

venous system

A 12 year old boy has fever vomiting and para-umbilical pain After examining the patient the doctor

makes an initial diagnosis of appendicitis Appendicular pain which is initially referred to the umbilicus goes to the dorsal root ganglion of

a TI b TI2 c L1 d T7

(e I TIO

A 59-year-old male undergoes a neurological examination which reveals that when the abdominal wall is

stroked the muscles of the abdominal wall of the side of the body stimulated failed to contract Other

neurological tests appeared normal The likely region affected includes

a CI - C5 spinal segments b C6 - TI c T2-TI ~T8-T12

e Ll- L5

The surgery done to relive portal hypertension is done by connecting two veins Which of the following veins would be suitable for connection

a Inferior vena cava and portal vein b Superior vena cava and portal vein c Splenic vein and right renal vein d Splenic vein and left renal vein e Superior mesenteric vein and Inferior vena cava

A mother brings her 3-week-old infant to the pediatric clinic reporting a new scrotal bulge that she found -~-

while changing a diaper yesterday The infant is afebrile Physical examination reveals a palpable mass in

the scrotum while in the standing position resolution of the mass in the supine position and no

transillumination of the scrotal sac What is the most likely diagnOSiS

a Cryptorchidism b Direct inguinal hernia c Hydrocele d Indirect inguinal hernia ~ e varicocele

The Vagal trunks enter the abdomen by passing through which of the following openings in the

diaphragm

a Right crus b Esophageal hiatus ~ c Vena caval hiatus d Aortic hiatus e Left crus

2 The anterior boundary of the epiploic foramen of Winslow is bounded by

a) First part of duodenum b) Lesser curvature of stomach c) Liver d) Hepato-duodenalligament v ~

3 The ilio-inguinal nerve is derived from

a TI2 ry b LI c L2 d L3 e L23

15 Surgically the structure used to suspend the kidney to the diaphragm is

a) Renal fascia b) True capsule c) Perinephric fat d) Paranephric fat

6 If there is portal obstruction because of carcinoma affecting the pancreas which of these of the

following signs would be present

a Caput medusae b Esophageal varices c Rectal varices c

d Pulmonary edema

7 In a sliding hernia the gastro-esophageal junction lies

a) At its normal position b) Below the normal position c) Above the normal position V d) None of the above

8 Which of the following structures is retroperi toneal

A transverse colon B spleen IJ2f6 C ileum D descending colon v r 1pound1111111

9 The renal angle is fonned lgtetween the 12th rib and ______ muscle

a Psoas major -middotshyb Erector spinae c Quadratus Iumborum d Diaphragm

10 The anterior structure at the hilum of the kidney is

a) Renal vein ~

b) Renal artery I middot~ I

c) Ureter d) Accessory renal artery

11 Because of origin of the muscle from the lateral one third of the inguinal ligament it

could not fonn the anterior wall of the inguinal ligament

a) External oblique b) Internal oblique c) Transversus abdominis_ d) Rectus abdominis

12 A large tumor mass impinges on the splenic artery and its branches as the artery pass out from below

the greater curvature of the stomach Branches o(which of the following arteries would most likely to

effected by the pressure on the splenic artery

a Left gastric b Left gastro-epipJoic c Right gastric d Right gastro-epipoloic e Short gastric_

13 A new born baby has projectile vomiting after each feeding It is determined that there is obstruction

of the digestive tract as a result of annular pancreas Annular pancreas is as a result of an abnormality in which of the following process

a Rotation of the dorsal pancreatic bud around the first part of duodenum b Rotation of the dorsal pancreatic bud around the second part of duodenum c Rotation of the dorsal pancreatic bud around the third part of duodenum d Rotation of the ventral pancreatic bud around the first part of duodenum y Rotation of the ventral pancreatic bud around the second part of duodenum

14 As the liver bud enters the ventral mesogastrium the region of the mesogastrium stretching from the

liver to the anterior abdominal wall is called

a Lesser Omentum b Greater Omentum ~ Falcifrom ligament d Lacunar ligament e Ligamentum teres of liver

16 A patient has absence of his 12th rib In such a patient if the doctor makes an incision to approach his

kidney mistaking the 11 th rib for the 12t he would end up injuring

Which of the following arteries is a direct branch of the gastroduodenal artery The

A right gastric artery

B left gastric artery

C inferior pancreaticoduodenal artery D left gastroepiploic artery

i E)right gastroepiploic artery --

E x pI a nation The right gastric artery is typically a branch of the proper hepatic artery The left gastric artery is a direct

branch of the celiac trunk The right and left gastric arteries anastomose along the lesser curvature of the

stomach The inferior pancreaticoduodenal artery is a branch of the superior mesenteric artery it

anastomoses with the superior pancreaticoduodenal in the head of the pancreas The left gastroepiploic

artery is a branch of the splenic artery it anastomoses with the right gastroepiploic artery along the greater

curvature of the stomach The right gastroepiploic artery is a branch of the gastroduodenal artery The

other branch of the gastroduodenal artery is the superior pancreaticoduodenal artery

Which of the following pairs of veins join together to form the portal vein The

A superior mesenteric vein and inferior mesenteric vein

B inferior mesenteric vein and splenic vein

C superior mesenteric vein and splenic vein

Ip)splenic vein and left gastric vein E superior mesenteric vein and left gastric vein

Explanation

The portal vein is formed behind the neck of the pancreas by the union of the superior mesenteric vein

and the splenic vein The inferior mesenteric vein drains into the splenic vein The left gastric vein drains

directly into the portal vein After the portal vein forms it enters the hepatoduodenalligament of the

lesser omentum to reach the liver The portal vein is the most posterior structure in the hepatoduodenal

ligament

At which of the following vertebral levels does the duodenum pass anterior to the aorta - _- shy

All ~

B L2 7~

CL3 I

~DL4

E L5

Explanation

The duodenum begins at the pyloric sphincter at the level of Ll The second (or descending) portion of

the duodenum is to the right of the aorta and extends inferiorly from the level of Ll to the level of L3 The third part of the duodenum crosses the aorta from the right side to the left side at the level of L3 The

fourth (ascending) portion of the duodenum extends from the level of LJ to the level of L2 The

duodenum ends at the duodenojejunal flexure The superior mesenteric artery passes anterior to the

duodenum as the duodenum passes anterior to the aorta The duodenum can be constricted at this level

In which of the following locations will perforation of the digestive tract result in the spilling of luminal

contents into the - lesser peritoneal sac

A Anterior wall of the second portion of the duodenum B Posterior wall of the second portion of the duodenum

C Anterior wall of the stomach

~Posterior wall of the stomach E Posterior wall of the transverse colon

Explanation

The posterior wall of the stomach is related to the lesser peritoneal sac The anterior wall of the stomach is related to the greater peritoneal sac The anterior wall of the second portion of the duodenum is related to the greater peritoneal sac The posterior wall of the second portion of the duodenum is related to the retroperitoneal space The posterior wall of the transverse colon is related to the greater peritoneal sac

The ureter lies against the anterior surface of which of the following muscles shyA Crus oftne diaphragm B Quadratus lumborum

0 Psoas major D Transversus abdominis

E Iliacus

Explanation The ureter exits the renal pelvis at about the level of vertebra L2 As it descends along the posterior abdominal wall it lies on the anterior surface of the psoas major The psoas major muscle arises from the bodies of the lower lumbar vertebrae The psoas major muscle is joined by the iliacus to fonn the

iliopsoas muscle The iliopsoas muscle then attaches to the lesser trochanter of the femur and is the major

flexor of the hip

As the right ureter passes the pelvic brim it lies against the anterior surface of which of the following

blood vessels

A Gonadal artery B Inferiorvena cava C Internal iliac artery

rJ- External Iliac artery

E Inferior mesenteric artery

Explanation

The ureter lies in the extraperitoneal space in the posterior abdominal wall Alter leaving the kidney it

passes inferiorly on the anterior surface of the psoas major muscle At the pelvic brim the ureter passes

into the pelvis At this point the common iliac artery is dividing into the external and iliac arteries The

ureter lies on the anterior surface of the external iliac artery immediately distal to the bifurcation This is a useful landmark for a surgeon to locate the ureter

When extravasated urine passes from the superficial perineal space into the anterior abdominal wall it is

found immediately deep to which of the following layers of the anterior abdominal wall

-ltScarpas fascia

B External oblique muscle

C Internal oblique muscle D Transversus abdominis muscle

E Transversalis fascia

Explanation

The superficial perineal space is bound by Colles fascia the fibrous portion of the superficial fascia This

layer of fascia is continuous with Scarpas fascia the fibrous portion of the superficial fascia of the anterior abdominal wall Therefore urine that is deep to Colles fascia will remain deep to Scarpa s fascia The urine will spread in the plane between Scarpas fascia and the external oblique layer

When a horseshoe kidney develops the ascent of the kidney is restricted by the A internal iliac artery B external Iliac artery

C common iliac artery

inferior mesenteric artery

E superior mesenteric artery

Explanation

A horseshoe kidney develops when the inferior poles of the to kidneys fuse together as they ascend into

the abdomen from the pelvis The first anterior midline vessel that is encountered by the horseshoe kidney

is the inferior mesenteric artery This artery prevents the kidney from continuing its ascent

The left testicular vein drains into which of the following veins

A Left internal iliac vein B Left common iliac vein

bflnferior vena cava D Left renal vein I

E Left internal pudendal vein

Explanation

The left testicular vein drains into the left renal vein The right testicular ~i~[~nsltjectlY into the

inferior vena cava This difference in venous drainage is believed to explain the greater incidence of

varicocele on the left side than on the right The venous drainage from the penis is to the internal vein

which then drains into the internal Iliac vein

The spinal nerve that provides cutaneous branches to the skin around the umbilicus is

A TS B TW-shy

C TI2

DL2 EtA

Explanation

The tenth intercostal nerve is the anterior ramus of the TIO spinal nerve After passing through the tenth

intercostal space the nerve continues forward in the anterolateral abdominal wall in the plane between

the internal oblique muscle and the transversus abdominis muscle In the abdominal wall the nerve innervates to the abdominal wall muscles as well as the skin and the parietal peritoneum The umbilicus is

a useful landmark for the region of distribution of the tenth thoracic nerve

The ligament of the vertebral column that resists its extension is the Aligamentum flavum

B supraspinous ligament

C posterior longitudinal ligament

D anterior longitudinal ligament

E interspinous ligament

Explanation

The ligaments of the vertebral column that resist flexion of the column include the supraspinous ligament

interspinous ligament ligamentum fiavum and posterior longitudinal ligament The ligament that resists

extension is the anterior longitudinal ligament This longitudinal ligament is very broad and strong It

covers the anterior and anterolateral surfaces of the vertebral bodies and the intervertebral disks In

addition to resisting extension the anterior longitudinal ligament provides reinforcement to the anterior

and anterolateral surfaces of the intervertebral disk The posterior longitudinal ligament is relatively

narrow and covers the posterior surface of the vertebral bodies and the intervertebral disks This ligament

reinforces the posterior surface of the disk The posterolateral surface of the disk is not reinforced and it

is through this region that herniation of the nucleus pulposus usually occurs

A patient presents with epigastric and right upper quadrant pain The pain is most intense 2-4 hours after

eating and is reduced by the ingestion of antacids The patient states that he has passed black tarry stools

(melena) within the last week Fiberoptic endoscopy reveals a yellowish crater surrounded by a rim of

erythema that is 3 cm distal to the pylorus Accordingly an ulcer has been identified in the patients

A fundus

B antrum

C duodenum

D jejunum

E ileum

A number of physiologic genetic and other factors increase the risk of gastric (and duodenal) peptic

ulcers The evidence that H pylori plays a principle role is compelling Smoking and caffeine are known to adversely affect the morbidity mortality and healing rates of peptic ulcers In general first-degree

relatives of peptic ulcer patients as well as males have a threefold to fourfold increased risk of developing this disorder Paradoxically in gastric ulcer disease acid secretion is not elevated It is possible that

excess secreted hydrogen ion is reabsorbed across the injured gastric mucosa In general a defect in gastric mucosal defense is the more important local physiologic

A patient presents with symptoms of duodenal obstruction caused by an annular pancreas Annular pancreas is caused by

A rotation of the dorsal pancreatic bud into the ventral mesentery B rotation of the ventral pancreatic bud into the dorsal mesentery

fJ failure of the major and minor pancreatic ducts to fuse ~ ~ cleavage of the ventral pancreatic bud and rotation of the two portions in opposite directions around -the duodenum E formation of one pancreatic bud instead of two

Explanation Normally the ventral pancreatic bud rotates around the gut tube to reach the dorsal pancreatic bud The two buds fuse to form a single pancreas and the distal portions of the two ducts fuse The ventral pancreatic bud forms the inferior portion of the head of the pancreas the uncinate process and the major pancreatic duct (of Wirsung) The dorsal pancreatic bud forms the superior part of the head the neck body and tail and the minor pancreatic duct (of Santorini) Annular pancreas is the result of the ventral pancreatic bud dividing into two portions before it rotates into the dorsal mesentery Each portion rotates in opposite directions to get to the dorsal mesentery thus encircling the duodenum The presence of annular pancreas can constrict the duodenum thus obstructing its lumen

In n _ phranlc----

Gon ~l ----_1 Lum bltano

~~--- CornmQ1t bull ac

+-~4--- lnlllirnaJ ilic

xtem iliac

OBJECTIVE - Identify the blood supply to each of the structures listed in the table on the previous page

Ill give you a head start

FOREGUT - Supplied bV Celiac Tru nk (T12)

Proper hepatic

GastiooUod 13Jafter

1nferlor pancreaticoduodenal artery

Common epatlc

Lett gas ric iiirtery

Spfen artery

shy Gastroepiphgtic artery

~ Superior mesenteric 8rtfry

~

1 Esophagus is a derivative of the foregut so its blood supply originates from the celiac trunk

(T12) The predominant blood supply to abdominal portion of the esophagus is the Esophageal

A (Branch of L Gastric) The venous drainage of the esophagus is particularly important because

it is 1 of 3 clinically relevant sites of Portal Caval anastamoses The Portal Esophageal Vein

meets the Caval Azygos System Persistent bleeding manifests as Esophageal Varices - a fata I

condition

2 The Stomach is also a derivative of the foregut has EXTENSIVE blood supply and is very high

yield on anatomy exams The lesser curvature is supplied superiorly by the L Gastric A (1 of 3

major branches ofthe Celiac trunk) and inferiorly by the R Gastric A ( a branch ofthe proper

Hepatic A) The greater curvature is supplied superiorly by the L Gastroepiploic A (a major

branch of the splenic A) and inferiorly by the R Gastroepiploic A

The Short Gastric arteries (branches of Splenic Artery) supply the fundus of the stomach and

are referred to as EIID ARTERIES because they have no collateral blood supply Therefore if the

splenic artery were occluded (ex - increased pressure in the ommental bursa) - there would be

ischemia to the fundus of the stomach Venous drainage of the stomach is extensive via various

veins lead ing to the portal system Posterior to the stomach the IMV joins the splenic V which

joins the SMV to form the PORTAL VEIN ADAMS

3 Duodenum blood supply has high clinical relevance because it is the junction of the foregut and

midgut and therefore is the site of anastamoses between branches ofthe Celiac Trunk (main

foregut artery) and the Superior Messenteric Artery (main midgut artery) The Proper hepatic

artery gives off the gastroduodenal artery which travels behind the 1st part of the duodenum

This point has high clin ical relevance because duodenal ulcers are very common and a posterior

rupture of the 1st part of the duodenum could rupture the gastroduodenal artery causing

traumatic abdominal bleeding The Gastroduodenal artery first gives off the R Gastroepiploic A

(mentioned above) and proceeds as the Superior pancreatico duodenal artery (supplies the

pancreas and duodenum) which anastamoses with the inferior pancreatico duodenal A (branch

of the SMA) This is the junction of foregut and midgut and occurs near the opening of the

bil iary system into the duodenum (ampula of vater) Portal venous drainage here is responsible

for delivering nutrients from digestion to the liver for metabolism Appreciate that the Superior

mesenteric artery (artery of the midgut) branches from the aorta at Ll travels posterior to the

pancreas than moves anteriorly (at the jxn of the pancreatic headbody) and comes over the

3rd4th part of the duodenum Tumor of the head of the pancreas can compress the SMA

4 Jiver blood supply is via the common hepatic artery (major branch of the cel iac trunk) The

common hepatiC becomes the proper hepatic gives off the R gastric A and the Gastroduodenal

A and then joins the common bile duct and the portal vein in the portal triad Clinical- if a

patient were bleeding from the hepatic A a surgeon can stick his fingers in the epiplOic foramen

and squeeze the free edge of the hepatoduodenalligament in order to stop bleeding to the

area Please note that the hepatic a branches into Rand L hepatic A The Right hepatic artery

gives off the cystic artery which supplies the gallbladder Afferent venous supply is via the

Portal vein which is bringing nutrient rich blood to the liver After metabolism takes place

venous blood leaves the liver through the hepatic veins into the IVC PLEASE UNDERSTAND THE

RELATIONSHIP OF THESE STRUCTURES - ADAMSNETIERSNH Etc

5 Pancreas - Head is supplied via the superior and inferior pancreaticoduodenal arteries

(mentioned above) The tail (situated towards the hilum of the spleen) is supplied via the

pancreatic branches of the splenic artery (END ARTERIES) This blood supply is very important

because the endocrine Alpha and Beta Cells from the pancreatic islets of lagerhans are located

towards the tail This is where Insulin and Glucagon is released to the blood

Now complete this for mid and hindgut structures Make sure to note clinically relevant arterial

anastomoses as well as portal caval anastomoses FYI Appendix blood supply SMA + IMA

anastamoses marginal artery Portalcaval rectal veins fhemmorhoids) and periumbilical caput

medusa are high yield THE BUTT THE GUT and THE CAPUT

Abdominal Development

Liver

Ij1f

II wall b

oh liN ~ VltJrti n be- bull

Pancreas

Secondary Retroperitonealization e I~tl r 1 a v-mtrai m ellter

Rotations of the Gut I i Ij (lIl1UtIJ f~ l r tilt

()l td 10 me l-ft and he v

--~--- -~ -~-~

i

I AolaijonjoI~guf I

STOMACH BED (IDENTIFY IN ADAMS)- the structures posterior to the ommental bursa which

support the stomach in the supine position

Abdomnal JQrUI

Splnic vein

OmQ-oul tv~ ) O(s(Jroa)

Lojt(r o m nturrt (hpJtodu o d~n31 Hid

Gadrl)SplerH (g3stroll~nal) IIgam~nt

hiad h~~atogrtricent IIQdmiddotcrt~)

Lt Dome of Diaphragm (why left Look this up in Adams)

Spleen (What is the blood supply)

Left Kidney (What is the blood supply - AND how is it different from the R kidney)

Suprarenal Gland (What is the Arterial AND Venous Blood supply - how are they different)

Pancreas (How does supply differ from Head to Tail What is the SMA Relationship)

Transverse Mesocolon

liver - ADAMSWET - Make sure you look at the liver in wet lab

Left triangular nl1am~nt

ComoaDj ligamnt

Erophg~1 impre$ioo

Hepatio veins

In1erior -ifena middotr3)Ia

Fibrous appendix o-t

live

impr~j on

Heprorendl p~rtion of Q)(Qllary ligament

Righllri~n9ul r 1I~met

(Common) bile quol

Gr)mmCtr~ hepatic dlJct

Ccentic duct

Duodenal impression

GaJdate p-fr)~S

Hepatic artgtrl prop-f iiiiila - Faloiform ligament

_ - shy Round ligamen liver

~--F-- CoJio imprgt-ssi-on

Prta heptis

Identify the lobes impressions and embryonic remnants associated with the liver

Caudate Lobe Quadrate Lobe Right Lobe Left Lobe Round ligament Falciform Ligament

Ligamentum Venosum (what is its fxn in embryonic life) Hepatic Veins (NOT PART OF THE

PORTAL TRIAD) IVC PORTAL TRIAD - Contents relationship cross section etc Know the

Galbladder relationship to the lobes of the liver

Biliary Duct System - Make sure you understand the sequence of these structures - BE ABLE TO

DRAW A FLOW CHART

TPVd i

t

I t

1 __ Cm-(r

patk GlJet

I

J

Clinical = JAUNDICE is caused by anything that prevents delivery of bile to intestine Tumor of the

head of the pancreas Stones etc Patient will have pale stools and yellowish colored mucus

membranes

Clinical- Any scenario that tells you the patient has BILLOUS VOMIT means that the obstruction to

the flow of digestive contents is after the Ampulla of Vater (Site of Entry of Billiary system to the

duodenum) - ie Duodenal Atresia

Spleen -located posterior to the mid axillary line between ribs 9 and 11 Make sure you know that

the 10th rib is the main axis of the spleen and this organ is susceptible to injury (stab wound errant

thoracoce ntesis etc)

The spleen is derived from mesodermal cells - NOT THE GUT TUBE

The spleen rests on the left colic flexure associates with the tail of the pancreas Know the

structures entering the Hilum of the spleen

Sh rt O~-t~ic 1 0(0 10 rtiltSPIric Iloa nt

(cut)

Peritoneum - similar concept to Pleura - think of a fist in a balloon

Visceral Peritoneum - Layer of balloon touching your fist

Parietal Peritoneum - Layer of balloon not touching your fist

Your fist represents the organ your wrist is the hilum and your arm contains the blood supply

entering the organ

Appreciate that there will never be organs in the peritoneal cavity - rather these organs invaginate

the cavity Kaplan videos

RULES OF NOMENCLATUREshy

1 Organ completely surrounded by peritoneum - peritoneal organ

2 Organ partially surrounded by peritoneum- Retroperitoneal

3 Peritoneum surrounding peritoneal organ is VISCERAL peritoneum

4 Peritoneum surrounding retroperitoneal organ is PARIETAL peritoneum

5 Peritoneum connecting visceral to parietal is called messentary 2 messentaries in the

gut Dorsal (to the gut tube) and ventral (to the gut tube) messentary

Aorta is in Retro peritoneal position - but blood must reach peritoneal position - vessels travel through

messentary All peritoneal organs will have blood supply reaching through messentary

-Mesentery is a 2 layer peritoneum with a neurovascular communication between body wall and organ

- Ligament connects one organ with another or to the abdominal wall (Ommentum = ligament)

lesser Ommentum (attach lesser curvature of stomach and duodenum to liver) =Hepatoduodenal

Ligament and Hepatogastric Ligament

Has a Superior and Inferior Recess (Accumulation of Fluid in Ascites)

Communicates with the greater sac through the epiplic foramen (what structures pass through

this foramen)

Boundaries - you must be able to visualize this

o Anterior - stomach

o Posterior - parietal peritoneum pancreas

o Superior - superior recess (bw diaphragm and coronary ligament)

o Inferior -Inferior recess (bw layers or greater momentum

Greater Ommentum (attach greater curvature of stomach) Gastrophrenic ligament Gastrosplenic

ligament gastrocolic ligament

The greater omentum is the largest peritoneal fold It consists of a double sheet of peritoneum folded on itself so that it is made up of four layers The two layers which descend from the greater curvature of the stomach and commencement of the duodenum pass in front of the small intestines sometimes as low down as the pelvis they then turn upon themselves and ascend again as far as the transverse colon where they separate and enclose that part of the intestine

ABDOMINAL PAIN

Parietal Peritoneum - supplied by same vasculature lymphatics and nerves supplying body wall it

lines and diaphragm Sensitive to pain pressure heat cold well localized

Visceral Peritoneum - supplied by same vasculature lymphatics and somatic nerve of organ it covers

Insensitive to touch heat cold and laceration - referred to dermatome of spinal ganglia providing

sensory fibers Where does appendicitis refer to

Foregut pain - epigastric area (ie - cholycystitis)

Midgut pain - periumbilical area (ie - appendicitis)

Hindgut Pain - suprapubic area (ie - diverticulitis)

Extra ImagesConcepts

ll~_____-

FalifCtrm ligament oind r~ud ligamet f Ilver

Blood from splenio gastriC and inferiof rne$e-rteri v~ins

Ca-I tributaries

Lett gastrio Ifein

Posterior superior pan~reatioodul)denal vaihS

Lott gamo-om~nlal (9aropip lomiddotic) -in

Poq_~ tjol imerl-9-r panCJertlcorllJod-nal veiopound --amp----I- - ~J Right grtr~-omntal

Anwrior interi (gartroepiploic) Jjn

pan euaii cod vl)denal veins middot Inf~Ji (t r mesentric vein

Miqdle (olic vein

Right cl)licvein Sigmoid and rectosigml)id (ei ns

IhH)Collc(~io

--- Mi~dl laquooLJl gtjrltgt

PoM ca vl1 illasto)moses -----shyampoptoageal 2 Paraumbilie-lt11 Inferi or Fectal vei ns

3 Recial 4 REuoperHonea1

Know how the Portal vein is formed I 4 sites of portal caval anastamoses and 1 clinical shunt

Col li t ltt-~ otTl~tI ~nj pc~ 1lt1 turJoG

Ltf 14i1 tImiddot~ artoftl9 on tj phtAt$

L-oftqf 4t t~r 1=laquoIran d 1 bull shy~p l ci rj o fOOOts

Nerves follow the arteries - appreciate the splanchnic nervous system I

Uet~ric branch of left ~nal art

Ureterie branch of righi renal artelY

Left Zld lumbar in and co mlTlunication to as)erdin9 lumbar l(~in Hi ~ht tEZ1~~t~ t3r j t itn ~ nJ l1t- rlnd lfe i r1

Inferior me5nteri~ artery

Notice that the right testicular vein drains directly into the IVC and the right testicular artery drains

directly into the aorta However the left testicular vein drains into the L renal vein at a right angleshy

reason left testicle is lower and more susceptible to varicocele (bag of worms)

Also notice that the left renal vein has a longer course because the IVC is on the right side whereas

the right renal artery has a longer course because the aorta is on the left side

Appreciate the anterior to posterior relationship of structures in the hilum of the kidney - VAP - Vein

Artery Renal Pelvis (Ureter)

11____ __ L_ L_ n VJ __ _ _ t_L I I_ _ L __ L_ I -pound1 bull LI_~-I ____

Posterior View of Head of Pancreas in ( of Duodenum

Celiao hunk

Co mmon ~L~jJth art~ry

GastNduQdonal artrf (partilly in phantn)

P1)Sterior $Up~Jior panCflaticuduodfmal art~r~t

(Co mm on) bile duct

middot~1t~~t-1l---~-~- Right gshomiddotomental (gastoe plp lolc) 3rte (phantomost)

Grener paocre atic art-ry

1n1~rjor pancr-iatlc artery

Jtrifll supejo r pal)oreailcento)dJodenal artr1 (phantom)

Anastomotlo branch

POostetlor bJanch of jo f~ri of pan-reatir(lduodensl drttnj

Anterio r branch of i flferior palcreati~)duodenal art~(phan1om)

Notice the extensive blood supply to the pancreas and duodenum via the branches of the celiac trunk

Notice collateral supply from SMA branches - makes sense bc this is the jxn of foregutmidgut

Identify the vessels in this arteriogram

Hiltid i)f N~ck oi B)dvof Tail 01 pa nereas pan cent~as P-nmiddot-reas panCtCas

I nferie v~na cava

jHept1iic p(lrlai v~in

Port1 tnd H~pti lt a ftH prol

Comm on) bll duct

Ouodtnum

~ft colic (sio)Atta~ hmtrlt jt~xJr-ofha~elSe

muo(IIQn

Right ~lIc (h~j)tic)

il~gtture

In1triol m~oten lIein (rttr op~ritoMdO

SlJp efl or mes~n~fiC amrV and lipln

KNOW YOUR NEIGHBORHOOD

Questions

vVhiJh structure supplied by a bnmdlof the cclia( artery is not derivcd from foregut LemCJUCrITI

(A) Head of the pancte-a5

CD) Pyloric duolenum

Cystkduct

( Liver hepatocyt~~

~F) Body of the spleen

An infant presents with an omrhaJucele at birth -hi oJ the [oHm illg applies to his cM1-dition

(A) It is 31so seen ill p4titnts with aganghonic megacolon

(11) ft reuirs from a fal1ure of resorption of theviteUine d let

(C) It results from herniation at the-site of regression of the right umbilk vein

DJ It is caustd by faihtrc of recanalization of the midgut part of the duodenum

~ It ill camioo by a failuIt vf the midgul to return to the abGQminal uity after herniashytion in-n the urnbilk s l stalk

Ot er than the spleen occlusion Cif the spit-Ilk artery at its odgin wm most likely affect die blood supply to jllch st cnud

(A) Jejunum

(B) Body of th pal1~lltas

(C) LeSStT Cllmiddotlaturc of tl )toma-ch

(D Duodenum dista to the entrance of the Ornmou bile duct

E Fundus of the stomach

A 38-yeu-old batL~er with a history of heartburn suddenly experiences excluciating pain in the (plgastric region of th~ abdomeu SurgCry is perf~rme immediard y upon admisshysion to the 1IlcrgCJliy tuomh~re i~ evidence uf a ruptured ulcer in the posterior waU of the stomach Vhere will a surgeon first fi nd the stomach contenlSf

A) Greater p4ritoneal sac

rB) Cul~de-s~c of Douglas (--

C Omental bursa ~

--D) Paracolic gutter

rEj Between -he panttal perimltum and the posterior body wal1

At birth an infant presents with a st()ma~ rb~tbas~njJled jfltotb~diaplfagru 1A1ltre is the defect thatresulied iiitJle heini~t()n shy~tsophagealbiatus

7 - rH-- Hiatus for the inferior vena cava

( Pleuroperitoneal membrane -(0) Septum transvcrsum

(E) Right Crlt~

An infant born with DOVv7l syndrome presents with bili()u~ vomiting Ahat congenital defect does the infant have

(A) Pyloric stenosis

(B) Meckel diverticulum C) Ornphaloce1e

(D) Gastroschisis

( ~ ) Duodenal atresia y A patient with cirrhosis of the liver presents with ~ bacalvaricestnlreased retrograde pressure in which veins caused the varices

(A) Paraumuilical

(B) Splenic

(ct AzygltJus

(15))G~trk ( (-F) Superior mesemeric

A htaltby 3-year~old male patient experiences a hernial sa protruding from the anterior abdominal wall about halfway between me anterior superior ilia spine and the pubk tuberde Pulsations of al1 artery are palpated medial to the protrusion site through the abdominal walL Which layer of the anterior abdominal wall will first be traversed by the

1hctma

fA) Rectus sheath (B) External oblique aponeurosis

(C) Inguinal ligament

lD) Transversalis fusda

(E) Cremasteric fa~cia

After 5urgi(aj ffpair of a hernia the patient tXperienccs mtmlgtness in the skin on the anteshyrior aspect of the S(Totum_ Vhaf nerve may have been lesioned during thehemiorrhaphy

(A) Femoral

(B) Obturator

(C) Ilioinguinal

(D) lliohypogastrk

(E) Pudendal

A 23~year-LJld female secretary il1 good health ~-uddcn1) doubles over with pain in the a ea of the 1JmbRicu$ Sbe feels vartn and ltneasy and has no appetite That night the pain seems to have mQved to the tower right abdominal regjol1 and she calls her family doctor who then arranges for an ambulance to pk-k her up and take her to the hospitaL Wh ell ntn~ perceived in the area of the urnbilirus most Hkely carried lhe pairfu I sensations into the eNS

tA) Vagus nerves I~

V B)

) Lessersplanchnk nerves

tC) Pudendal nerves

(D) lIiohpogastrk nerves

(E) Greater splam ic l erves

A CT reveals carcinoma in the bOod of the ancreas Vhich blood vessel trut ourses ----~- - -bull ------ --shy

immediately poftterior to the body ofthe pancreas is the m~t likely to be oompressed

(A) Splenk artery

(B) Abdominal aorta (C) Portal vein

(1) Splenic vein

(E) Renal vein

A patient has a penrln1l1ng uker of the posterior wall ot the br~l part ot the (lUooenmn llkh blood vessel is subject to erosion

(A) Common hepatic artery

(B) Gastroouodenal artery

(C) Proper hevatic artery

(D) Celiac artery

(E) Anterior inferior 11amrelltlcoduodcnal attery

Your patient has been diagnosed -ith a carcinoma locallted to the head and l~e(k of the pancreas Another clinical sign would be

A esophageal varices

(8) hemorrhoids

C) a caput medusa

(D) increased pra Teuro n th~ hepatic veins

(E) enlarged right supra lavkular lymph nodes

Wltkh of the foUowing structures develops in the ventral mesentery

(A) Spleen

(B) Jeiunum (C) Head of1ht pancreas (D) Transverse colon (E) Stomach

ti l Uw ~ littwin~ f( S-t lil oai Imdge ~ hi(h or tbt la~)d J truetur tgt liJ llntn nl) he hl p UC iJd [IIi ell

c o

A) drains Ie tht infCrior a La aI

R t middot~nfl0 ~ill to th~ lunlgtn of h i dtlndCrlllfH

(e) m t bull JiJattd on tl l J n T ~H

D ) sup Lc O VSlt I Hlid bhtu l 1 li - -I un oid

( ) U~tpli(t tr j middottUh~ 1 v(( b~nt rfK n1ilc~Zm

ANSWERS AND EXPLANATIONS

Answer E The spleen is t hlttnopodicand lymph organ demlted from mesoderm

Answ~ R Al1 tlmphalocele is caused by it failure of the nlidgut to return to the ahdomir nat cavity after herniation into the umbiliau Stalk Choices Aand D maybe seen in infants with Down syndrome choice D ~s the specific CBuse ofduudcnal JtiCSitt Choice C is (ile cause of gclstrosbisis and Choice B nsults iu a Meurolktldivertku1-tlB

Answer B The fundus ofthe stomach is suppHed by soort gastric brunches of the splenic altery The splenic artery supplies the body and tail of the pancreas part of the greater curvature of the sttmla(h and the spleen Te jejunum part of the head of the pancreas and tht~ duodenum distal to the entrance of the commOll bile duct are supplied by the superior mesenterk artery clll~l ~be less r ctlt1ature cmd the pylQric antrum are supplied by the right and lei gastric art(ries

AnSWftt C Tbeomental bursa or lesser ~ritoneaj sac lies direcdy posterior to the proxshyimal part of the duodeTtlm and the stomach and would be the first site where stomach contents ~Ott1d be fpoundluncL

Answer C A defect in a llleuropcritoneal membrane (uswlly the left) is the typical site of i1 cc-ngenitlI diilphragluatic hemia llere the membr4ne fails to dose ()pound( of the perishycCirdiopcritulleal canals

Answer E DuoJenal atresia and aganglionic megacoion are congwitaI defects S~Il in patients with Dowmiddotnsyndrome

Answer D RulaTgemt~llt of and retrograde flow in g~lstrk vel_ns in particlJl~r the kft gas~ tricveins dilates the capillary bed in rhe wall of the esophagus in (ases of porta yper~

tension Blood flow would increase in and dilampte tribntarkgts of the (lZygOUS vein on the other side of the capiUary bed but flow in this vein is in the typical direction t()ward the superior vena cava Paraumbiii(ltU vein eilgorgement contributes to a caput medusH Splenic ~nlargement might prc~nt with 5plcnonlegaly and balt-kflow in to tlu superior m~~ntclic vein occurs but is asymptomatic

Answer D The patient hagt an indirect inguinal hernia whi~h emerges from the antt-rior abdominal wall through the deep inguinltilling Theeep ring is a fault in the transv~rshysaUs fascia this I~yer wiIJ be penetrated first by the hernia

An~Wer C The ilioinguinal nenc which provides sens~llion to the lnedlal thigh ltmclanteshytior SClotunl pass~lt th rough the 5uperfh_ial inguinal ring ind $subject to inj i1T) becaus-e

it is in the operatitm Held of the erniorrhapny

Auswer B The leMHr splanchnic nerves are sympathdic nerVlts that carry viscera l sensashytlltgtrogt ftom illtllt1m~d ()J stietched gust (itinteitinal ~tructures (in this case the pprndix) into tnt eNS Lesser splanchnic ntTYcsarisc from thmiddot T9--T12 spinal cord segments lt1nd provide sympathetic innenation tD rnidgut siruc1ures whiCh include CLe app~JldD Viscera] Pain arising from affecLed Inidgut ampt 1C1ure is referred over the same dl- matorne~ of spinal segrnertts v-hich provide the sympathetic Innervation n this G1SC of appendicitis the invohen~n t of the ltire) of t e unlhHku indud s the T 10 dermatome

Answer B Of the five choices onty the dscending olon is retroperiton~al aldwould be a lik ~ ( choice to be seen immediately a(~jilcent to t11e posterior abdominal middotn~L

Amwen D The SpltftlC ~-ein ourses posterior to the body of the panneas m its way tt drain into the superior mCSfttltlri( vein

Answcr B TILt glstrodllolticnal artery 1 direct hIamh of the comrootl hepatic artery courses immediately pt))iwri() to the duodenum and is slbject to erosion

Answer B Carcinoma of th pan middott3S in the 1tilt1 may compreampgt the portltil vein at irs orishygill The poTtai vcin is fomled when the splenic vein jQiaswith tfie superior meStllt eric vein The inferiot mesenteric vein joins the ~plenjc vein just priOT to tlli~ point at which the splenic joins the superior Jlleit1ltcri( vein Increescd venous presslu in the inferior mesenteric vein is a cause of emo hoid~

Answer C The- velltral pancreas wilich forms most of the head of the p ~ncr as develops in the ventral mes(ntery as antutgrowth of the hepatic diverticulum Th~ hepatic divershyticulull induding the biIJary appa~atus develops in tbe ventral mesentery of the foregut

Answer~ A The superior mesenteric ~in joins with the spienkvein to form the hepatic portal vciu

Answer D The structure at gttlK is the proper hepatic artery~ whkh suppUesoxygenated b middotood to the liver

MAKE SURE YOU KNOW the diff bw Rectus Sheath above and below the arcuate line

ABOVE

Aponeurosis of xiiltmal obllque musclo

Extemll f)biquw musde

Reotln ilbdomlnls musole S~in

Internal 9bliquQ mY~QI

AponeUfOsi$ of hJH$V~~S Lir9a a lb lbdolTlin~ musolo Tri OJV6 rUi

atldomlnis mUS(loe

Sub cutanlilous tiue (tatty ye r)

BElOW

A POrl lJfosis 01 etemal oblique muscle

Aponeul~)sis 01 Internal oblique mU$cl~

Anteriol lay~ of r~ltdus st~ath EXttom1 oblique rnu$cll

Rectus Jbdominis muscle Intoernal Aponeurc-sis of tra~fersU$ oblique muscle-

at-domlnis muscentl ~ Skio

Tra nsvitSus abdomioLs ml)ZClt

TralSVersaHs fascia Medial umQil iegtt1 1i9Jment -and folj

Uldchus Peritoneum (ir median Umbilj~al Suboutane ous

Extraprftone 11ascia

Ymbilimiddot~1 fold)

preu9poundiea1 fascia

tissue (fatty 4nd m~mbr3n(iUS layers)

o Above the arcuate line (A horizontal line 13 of the distance bw the umbilicus and the

pubic symphysis) -10 Aponeurosis divides into an AntPost Laminae

o The Ant Laminae joins EO and Post Laminae joins Trans Abdominis = Ant and Post

RECTUS SHEATH respectively

o BElOW the arcuate line - all 3 aponeurosis join ANTERIOR to rectus muscle to meet its

counterpart in the midline (linea Alba)

o Take away Msg - The abdomen is devoid of a posterior rectus sheath below the

arcuate line and is therefore more vulnerable to herniasinjuries

Question - A physician makes a deep incision in the patients midline immediately superior to

the pubic symphysis which of the following layers is his knife least likely to pass

Rectus Abdominis External Oblique Ant Rectus Sheath Posterior Rectus Sheath All of the

Above

Answer - All of the above None of the other answer choices are midline structures -LINEA

ALBA

Linea Alba has very poor blood supply - doesnt heal well after surgery Therefore this is a

common site for incisional hernias

a Spleen b Transverse colon c Descending colon d Stomach e Pleura

17 Meckels diverticulum is normally found 2 feet proximal from the

a Pyloric sphincter b Lower esophageal sphincter c Ileo-cecal valve d Middle valve of Huston e Anal valve

18 Ulcer in the posterior wall of the first part of the duodenum would erode ___ artery and would cause bleeding

a Left gastric b Right gastric c Hepatic artery proper d Gastroduodenal artery e Middle colic artery

19 An inflamed appendix is identified by a surgeon on the operation table by noting

a The appendicies epiploicae b The convergence of tenia c The artery of Drummond d The mesocolon e The mesosalphinx

20 The nerve which emerges through the psoas major is

a Femoral b Ilio-inguinal c Ilio-hypogastric d Pudendal e Subcostal

21 The right gonadal vein drains into the

a Azygos b Hemiazygos c Inferior Vena Cava d Right renal vein e Left renal vein

22 The hepatocytes in the liver is derived from

a Ectoderm b Endoderm c Mesoderm

d Neural ectoderm

23 Abscess in the lumbar vertebrae due to tuberculosis would spread to the adjacent muscle which is

a Psoas Major b Iliacus c Quadratus lumborum d Tranversus Abdominis

24 The anterior wall of the inguinal canal is formed by

a External oblique and transverses abdominis b External oblique and fascia transversalis c Internal oblique and external oblique d Internal oblique and transverses abdominis e Fascia transversalis and peritoneum

Meckels diverticulum is a result of which of the following developmental abnormalities shy

A Failure of the vitelline duct to close

B Failure of the herniated intestinal loop to retract into the abdomen

C Failure of the urachus to close

D Failure of the midgut to rotate

E Failure of the hepatic duct to close

Explanation

Meckels diverticulum is a result of the persistence of the proximal part of the vitelline duct This

diverticulum is usually found about 2 feet proximal to the ileocecal junction and is usually about 2 inches

long It is present in about 2 of the popUlation It may be the site of ectopic pancreatic tissue or gastric

mucosa and may develop inflammatory processes and ulcerations Acute Meckels diverticulitis

simulates appendicitis

Which of the following veins carries blood from the esophagus to the portal vein The

A right gastric vein

B left gastric vein c splenic vein D azygos vein

E left gastroepiploic vein

Explanation

The left gastric vein a direct branch of the portal vein drains blood from the lesser curvature of the

stomach and the inferior portion of the esophagus Because branches of the portal vein do not have

valves blood can flow in a retrograde path when there is an obstruction to flow through the portal system or liveL Rlooci Cln then flow from the nortl] vein thr()1Ph the left PRstric vein to the esonhlPlIS lno

through venous communications within the submucosa of the esophagus to esophageal veins that drain

into the azygos vein The increase in blood flow through the esophageal submucosal veins results in esophageal varices

On the posterior wall of the abdomen the celiac ganglion A contains cell bodies of postganglionic parasympathetic neurons B is synapsed upon by neurons in the posterior vagal trunk C is synapsed upon by neurons in the greater splanchnic nerve D contains sensory cell bodies of lumbar spinal nerves E contains cell bodies of neurons that cause an increase in the rate of peristasis

Explanation The celiac ganglion is one of the preaortic ganglia of the sympathetic nervous system It contains cell bodies of postganglionic sympathetic neurons The sympathetic splanchnic nerves contain preganglionic sympathetic neurons that pass through the sympathetic chain without synapsing These splanchnic nerves go to the preaortic ganglia to synapse The greater splanchnic nerve contains preganglionic neurons from spinal cord segments T5-T9 This nerve synapses in the celiac ganglion The nerve fibers in the vagal trunks are preganglionic parasympathetic fibers that go to the walls of the organs that they will innervate and synapse on postganglionic parasympathetic neurons in the walls of those organs Cell bodies of sensory neurons in the abdomen are found in the dorsal root ganglia or the sensory ganglia of the vagus nerve Sympathetic innervation decreases the rate of peristalsis parasympathetic innervation increases the rate of peristalsis

Which of the following pairs of arteries will allow blood to bypass an occlusion of the celiac trunk

A Left gastric artery-right gastric artery

B Left gastroepiploic artery-right gastroepiploic artery

C Superior pancreaticoduodenal artery-inferior pancreaticoduodenal artery

D Splenic artery-common hepatic artery

E Left gastric artery - proper hepatic artery

Explanation The anastoOlosis of a branch of the celiac trunk and a branch of the superior mesenteric artery will

provide collateral circulation around an occlusion of the celiac trunk Each of the other choices pair

branches of the celiac trunk therefore these will not provide collateral flow around the obstruction of the

celiac trunk The left gastric splenic and common hepatic arteries are direct branches of the celiac trunk

The right gastric artery is a branch of the proper hepatic artery which is a branch of the common hepatic artery The left gastroepiploic artery is a branch of the splenic artery The right gastroepiploic artery is a

branch of the gastroduodenal artery whlch is a branch of the common hepatic artery

Which of the following organs has appendices epiploica The

A sigmoid colon

Bjejunum

C duodenum

D stomach E esophagus

Explanation Appendices epiploica are characteristic of the colon Appendices epiploica are subserosal accumulations

of fat None of the organs of the gastrointestinal tract has appendices epiploica except the colon

Page 14: Chirag's Abdomen Review

bull

~ Lungs liver amp gall bladder and pancreas are off-shoots from the foregut Esophagusshy~ The region of the tube from the laryngeal diverticulum to the beginning of the stomach elongates

to form the esophagus ~ The glands which form in the endoderm (mucosa) migrate down into the submucosa The path

whlch it took migrate becomes the duct of the glands which open to the mucosa ~ Achalasia Cardia - Failure of relaxation of the lower esophageal sphincter because of congenital

absence of ganglia at the sphincter (The ganglia when present releases VIP (Vaso-IntestinalshyPeptide) which relaxes the sphincter)

Mid-Gut Rotation ~ Because of the 90 degree rotation of the primitive stomach all of the following events occur ~ Lesser curvature comes to the right Therefore lesser omentum also comes to the right ~ Greater curvature comes to the left Therefore greater omentum also comes to the left ~ Right side vagal trunk becomes posterior vagal trunk ~ Left side vagal trunk becomes anterior vagal trunk ~ The left side peritoneal cavity comes to the anterior aspect of the stomach and will later be called

as the greater sac ~ The right side peritoneal cavity comes to the posterior aspect of the stomach and is (relatively a

small sac because the liver is on the right) called the lesser sacomental bursaepiploic sac ~ Epiploic foramen of Winslow (the lower free margin of the ventral mesentry) wiII be the

communication between the greater and lesser sac ~ The Liver moves to the right and therefore actually causes the 90 degree rotation of the stomach

The spleen comes to lie on the left side ~ Axis Antero-posterior axis around the superior mesenteric artery

bull Counterclockwise bull Approximately 270deg bull During herniation (about 90deg) bull During return (remaining 180deg)

Duodenum ~ Becomes retroperitoneal (except the first part which is still suspended by the hepato-duodenal

part of lesser omentum) ~ Glands (of Brunner) go submucosal ~ An imaginary line drawn below the opening of the major duodenal papilla represents the junction

between the foregut and midgut ~ Duodenal atresia in Downs syndrome Liver ~ 3rd week

bull liver bud grow bull into the septum bull transversum

~ 10th week bull hematopoietic bull function

bull 10 of the total bull body weight

~ 12th week bull bile is formed

Pancreas ~ In about 10 of cases the duct system fails to fuse and the original double system persists ~ 3rd month

bull pancreatic islets (Langerhans) ~ 5th month

bull Insulin secretion ~ Annular pancreas

bull The right portiCn of the ventralbud migrates along its normal route but the left migrates in the opposite direction

~ Complete obstruction of duodenum ~ Accessory pancreatic tissue Polyhydramnios (Amniotic fluidgt 1500-2000 ml)

~ Congenital defects including central nervous system disorders (eg Anencephaly) and gastrointestinal defects (atresias ego Duodenal esophageal) prevent the infant from swallowing the amniotic fluid (failure of recanalization)

Oligohydramnios (Amniotic fluid lt 400 mt) ~ Cl~ldberenal-agenesis

bull Midgut _-_

~ Primary Midgut intestinal loop gives rise to bull Distal duodenum bull Jejunum bull Ileum bull Ascending colon bull Transverse colon - proximal two-thirds of the bull Transverse colon with the distal third

~ Primary intestinaltoop bull ncephalic limb distal part of the duodenum the jejunum and part of the ileum bull ncaudal limb lower portion of the ileum the cecum the appendix the ascending colon and

the proximal two-thirds of the transverse colon bull 6th week

bull Rapid elongation of the cephalic limb bull Rapid growth of the liver bull Intestinal loops enter the extraembryonic cavity in the umbilical cord

bull 10th week bull loops begin to return bull regression of the mesonephric kidney reduced growth of the liver expansion of the

abdominal cavity bull Jejunum -left bull Loops - more to the right

bull Cecal bud -last part (temporarily below the right lobe of the liver) ~ qIDlthaloseJe (Structures COlHLoArts9V~1tion)

bull Through umbilical ring bull 6th to 10th weeks

bull Associated with a high rate of mortality (25) and severe malformations bull Associated with chromosome abnormalities

~ Gastroschisis (Structures coming out are not covered by Amnion) bull herniation through the body wall ----=---=-shybull Into the amniotic cavity bull Lateral right of the umbilicus bull Sometimes the inferior wall fails to develop as a result lower abdominal structures like the

bladder would be exposed to the exterior not associated with chromosome abnormalities ~ Abnormalities of the Mesenteries

bull Mobile cecum persistence of mesocolon bull Extreme form - long mesentery bull Volvulus

~ Distal third of the transverse colon ~ Descending colon ~ Sigmoid colon ~ Rectum ~ Upper part of the anal canal ~ Primitive anorectal canal

bull 7th week cloacal membrane ruptures bull Tip of the urorectal septum perineal body bull Pectinate line

~ Hindgut anamolies bull Rectoanal atresias and fistulas bull Imperforate anus bull Congenital megacolon (aganglionic megacolon Hirschsprung disease)

bull

bull Hindgut

Chirags Abdomen Review - Part 2

Understanding Embryo makes learning blood supply EASY

I I

I t

~ -

)

Table l1r-~ L Adult SUmiddotuctu~SDrj~l Froln Each of he Three Dhisions of be Pringttive GUl Tube t-middot-----middotmiddotmiddotmiddot-

Foregu(

I_ (Celiac Trunk)

Ir-slt-gtphgus

S101na(b

I h -= LiJ~r

Pancre=l S

bull 1 i Biliary apparntu5

Gall bladdshy

i Pha11~Cal pltgtuchcs

LullSS-I

Mjig ---- bull __ _- ----n--duct----~---middot-------l--n

(Superior Jldesen1eric Artery)_-1I-(I_~__ middot __ O-=-)_in_middoto_r_M_e_se_n_t_e_r_i_c_An__

Uuodenu rn 2nd_ 3 lt141h V4Tt

Jejunun-~

nc-un]

tCCUJ11

AppltgtndLX

Transver5e -o1on (p~oxiln1l1 ~O Tbird)

bull__hytgtid~ _ _ ______ L _ __

Tr-dn~llt~se colon (diStul h lTd) I

)

i

Aa ca-nal -( uppeT patt) i

I I

_____ __ _ _ _ ___ - - rhe~ a(t clcriVOkt iV(5 opound~lt prbn1rC ~ nlQC blft TlI)( 134tof r~ tIonoinf~ i 1 ~l l1rd c- P Cle

Now Lets see how much youve learned

Questions

1) A pt receives a general anesthetic in preparation for a c~t~~my A right subcostal incision is made which begins near the xyphoid process runs along and immediately beneath the costal margin to an anterior axillary line and transects the rectus abdominus muscle and rectus sheath At the level of the transpyloric plane the anterior wall of the

-~~-~=--- _eco---shysheath of the rectus abdominus muscle receives contributions from which of the following

a Aponeuroses of the in~ande~tef-Ilal o~ues

b Aponeuroses of the transversus abdominis and internal oblique muscles c Aponeuroses of the transversus abdominis and internal and external oblique

muscles d Transversalis fascia e Transversalis fascia and aponeurosis of the transversus abdominus muscle

A

2) The lat~raJJJ11QjJt~gLfgJlLoneach side of the inner surface of the anterior abdominal wall is created by which of the following structures

K Falx inguinalis (~) Inferior epigastric a

c Lateral border of the rectus sheath d Obliterated umbilical a e Urachus

B

3) A man the victim of several knife wounds to the abdomen during a brawl at the Lobster Shack subsequently developed a direct inguinal hernY Damage to which of the following nerves is most likely responsible for the predisposing weakness of the abdominal wall

~ Genitofemoral nerve ( b) Ilioinguinal nerve ~-t Tenth intercostal nerve

d Subcostal nerve e Pelvic splanchnic nerve

B

4) Which of the following statements concerning a direct inguinal hernia is correct a It is the most common type of abdominal hernia b It transverses the entire length of the inguinal canal c It contains all3 fascia layers of the spermatic cord d It exits the inguinal canal via the superficial ingeJinal ring e It protrudes through H~acb strJg e

~(

1fltbS w E

tl

5) The conjoint tendon is

a Important in preventing indirect inguinal hernias b The fused aponeurotic layers of internal abdominal oblique and transversus

abdominus muscles c Posterior to the deep inguinal ring

d Medial fibers of the inguinal ligament

B

6) A 25 year old male is brought in to the ER after being involved in a car accident in which he received a crushed internal injury in his abdomen Examination reveals a lesion of parasympathetic fibers in the vagJsnerve which interferes with glandular secretory or

smooth muscle functions in which of the foliowingorgans a Bladder b Transverse coloiW c Descending colOO d Prostrate gland e Rectum

B

7) The spermatic cord includes all of the following contents except a Il ioinguinal nerve b Pampin iform plexus of veins c Vas deferens d Genitofemoral nerve

A

8 Which abdominal structure gives rise to the internal spermatic fascia (muscle) following the descent of testes in development

a External abdominal oblique aponeurosis b Transversalis fascia c Transversus abdominis muscle d Peritoneum e Internal abdominal oblique

B

9 Which abdominal structure gives rise to the tunica vaginalis fotlowing the descent of testes during development shy

a External abdominal oblique aponeurosis b Transversalis fascia c Transversus abdominis muscle d Peritoneum e Internal abdominal oblique

D

10) The lesser omentum is a peritoneal fold which is su bdivided into the a Hepatogastric and gastrosplenic ligaments b Hepatoduodenal and gastroomentalligaments c Hepatoduodenal and gastrosplenic ligaments d Hepatogastric and hepatoduoden9-jrj igaments

D

11) A posteriorly perforating ulcer in the pyloric antrum of the stomach is most likely to produce initiallocalized peritonitis or abcess formation in which ofthS fQllowing

a Great-sac - -- -

b Paracolic recess

c Omental bursa

d Right subphrenic space

c

The inferior mesenteric artery arises from the abdominal aorta ilm_ediill~y_J-Qs1eriQLto which of the foowing org~ns A-F~t~filie duodenum B Head of the pan~eis C Neck of the pandeas

D Second part of the duodenum

E Third part of the duooenum_shylaquoshy

shy

The correct answer is E The inferior mesenteric artery arises from the anterior surface of the aorta at the level of the third lumbar vertebra The third part of the duodenum crosses the midline at the level of the third lumbar vertebra and passes anterior to the aorta at the origin of the inferior mesenteric artery The

first part of the duodenum (choice A) lies horizontally to the right of the midline at the level of the first

lumbar vertebra The head of the pancreas (choice B) is to the right of the midline and extends from the

level of the first lumbar vertebra to the third lumbar vertebra It lies within the concavity of the

duodenum The neck of the pancreas (choice C) lies in the midline at the level of the first lumbar

vertebra It lies on the anterior surface of the aorta at the origin of the superior mesenteric artery The second part of the duodenum (choice D) lies vertically to the right of the midline and extends from the

level of the first lumbar vertebra to the level of the third lumbar vertebra

The left adrenaLvein drains directly into which of the following veins A Hemiazygos vein

B Inferior vena cavaee C Left renal veiri -

D Splenic vein

E Superior mesenteric vein

a

The correct answer is C The left adrenal vein and the left gonadal vein (either testicular or ovarian) drain into the left renal vein TheTeft renal vein t~ains intothe- inferior vena cava In contrast the right

adrenal ~~inandnght gonadal veindrai~ gLr~ctJy iQtoJhe iilferiQ[ Vencava -- -

ThehemTazygoS7ein- (~h-~i-~ A)~~c~i~es the venous drainage from the body wall on the left side of the

thorax and abdomen No visceral organs drain directly to the azygos or hemiazygos veins The inferior vena cava (choice B) receives the direct venous drainage from the right adrenal vein but not

the left adrenal vein Remember the inferior vena cava is on the right side of the abdomen The splenic

vein (choice D) receives the venous drainage from the spleen and part of the pancreas and stomach The splenic vein is part of the portal venous system

The superior mesenteric vein (choice E) receives venous drainage from much of the intestinal tract It is part of the portal venous system and joins with the splenic vein to form the portal vein

A 43-year-old man presents complaining of pain in the groin On examination his physician palpates a

bulge in the region of the superficial inguinal ring which he diagnoses as a direct inguinal hernia The hernial sac most likely

A is covered by all three layers of the spennatic fascia B passes medial to the inferior epi gastric artery

C passes medial to the lateral border of the rectus abdominis muscle

D passes posterior to the inguinal ligament E passes through the deep inguinal ring

The correct answer is B Direct inguinal hernias enter the inguinal canal by tearing through the posterior

wall of that structure The typical location for this type of hernia is through the inguinal triangle bounded

laterally by the inferior epigastric artery medially by the lateral border of the rectus abdominis and

inferiorly by the inguinal ligament Direct inguinal hernias pass medial to the inferior epigastric artery

whereas indirect inguinal hernias pass lateral to the inferior epigastric artery because the deep inguinal

ring is lateral to the artery Indirect inguinal hernias are covered by all three layers of the spermatic fascia (choice A) Direct inguinal hernias are covered by fewer than all three layers because the direct inguinal

hernia tears through one or more layers of fascia as it emerges though the abdominal wall The lateral

border of the rectus abdominis muscle (choice C) forms the medial border of the inguinal triangle All

inguinal hernias pass lateral to the rectus abdominis Femoral hernias pass posterior to the inguinal ligament (choice D) Inguinal hernias emerge through the superficial inguinal ring which is superior to the inguinal ligament Inguinal hernias that descend below the inguinal ligament pass anterior to the

ligament Indirect inguinal hernias pass through the deep inguinal ring (choice H) direct inguinal hernias

do not Both types of inguinal hernias pass through the superficial inguinal ring

During a gastric resection in a patient with stomach cancer a surgeon wants to remove the lesser

omentum because of tumor extension into it Which of the following structures lie in the free edge of the

l~~g omentum and consequently must be dissected out in order to be preserved

A Common bile duct cystic duct and hepatic artery 6

B Cystic duct hepatic artery and hepatic vein

e Hepatic vein and cystic duct

Portal vein common bile duct and hepatic artery

E Portal vein hepatic artery and hepatic vein

The correct answer is D The free edge of the lesser omentum contains three important structures the

common bile duct the hepatic artery and the portal vein Nei ther the cystic duct (choices A B and C) nor the hepatic vein (choices B C and E) lies in the free

edge of the lesser omentum

A 55-year-old male patient with chronic liver disease has portal hypertension To relieve the pressure in the portal system a porto-caval shunt is performed Which of the following veins may by anastomosed to

accomplish this porto-caval shunt A Left renal vein-left testicular veingt

B Right renal vein-right suprarenal vein I shy

e Splenic vein -left renal vein J

D Superior mesenteric vein-inferior mesenteric vein E Superior mesenteric vein-splenic vein

The correct answer is C The splenic vein drains directly into the portal vein The left renal vein drains

directly into the inferior vena cava Anastomosis of these veins would allow blood from the portal vein to

drain retrograde though the splenic vein into the renal vein and then into the inferior vena cava The left

renal vein (choice A) drains directly into the inferior vena cava The left testicular vein drains directly into

the left renal vein Thus these veins are already in communication and neither vein is part of the portal venous system The right renal vein (choice B) drains directly into the inferior vena cava The right

suprarenal vein also drains directly into the inferior vena cava Thus neither vein is part of the portal

venous system The superior mesenteric vein (choice D) drains directly into the portal vein The inferior

mesenteric vein drains into the splenic vein which then drains into the portal vein Thus neither vein is

part of the caval venous system The superior mesenteric vein (choice E) drains directly into the portal

vein The splenic vein also drains directly into the portal vein Thus neither vein is part of the caval

venous system

A 12 year old boy has fever vomiting and para-umbilical pain After examining the patient the doctor

makes an initial diagnosis of appendicitis Appendicular pain which is initially referred to the umbilicus goes to the dorsal root ganglion of

a TI b TI2 c L1 d T7

(e I TIO

A 59-year-old male undergoes a neurological examination which reveals that when the abdominal wall is

stroked the muscles of the abdominal wall of the side of the body stimulated failed to contract Other

neurological tests appeared normal The likely region affected includes

a CI - C5 spinal segments b C6 - TI c T2-TI ~T8-T12

e Ll- L5

The surgery done to relive portal hypertension is done by connecting two veins Which of the following veins would be suitable for connection

a Inferior vena cava and portal vein b Superior vena cava and portal vein c Splenic vein and right renal vein d Splenic vein and left renal vein e Superior mesenteric vein and Inferior vena cava

A mother brings her 3-week-old infant to the pediatric clinic reporting a new scrotal bulge that she found -~-

while changing a diaper yesterday The infant is afebrile Physical examination reveals a palpable mass in

the scrotum while in the standing position resolution of the mass in the supine position and no

transillumination of the scrotal sac What is the most likely diagnOSiS

a Cryptorchidism b Direct inguinal hernia c Hydrocele d Indirect inguinal hernia ~ e varicocele

The Vagal trunks enter the abdomen by passing through which of the following openings in the

diaphragm

a Right crus b Esophageal hiatus ~ c Vena caval hiatus d Aortic hiatus e Left crus

2 The anterior boundary of the epiploic foramen of Winslow is bounded by

a) First part of duodenum b) Lesser curvature of stomach c) Liver d) Hepato-duodenalligament v ~

3 The ilio-inguinal nerve is derived from

a TI2 ry b LI c L2 d L3 e L23

15 Surgically the structure used to suspend the kidney to the diaphragm is

a) Renal fascia b) True capsule c) Perinephric fat d) Paranephric fat

6 If there is portal obstruction because of carcinoma affecting the pancreas which of these of the

following signs would be present

a Caput medusae b Esophageal varices c Rectal varices c

d Pulmonary edema

7 In a sliding hernia the gastro-esophageal junction lies

a) At its normal position b) Below the normal position c) Above the normal position V d) None of the above

8 Which of the following structures is retroperi toneal

A transverse colon B spleen IJ2f6 C ileum D descending colon v r 1pound1111111

9 The renal angle is fonned lgtetween the 12th rib and ______ muscle

a Psoas major -middotshyb Erector spinae c Quadratus Iumborum d Diaphragm

10 The anterior structure at the hilum of the kidney is

a) Renal vein ~

b) Renal artery I middot~ I

c) Ureter d) Accessory renal artery

11 Because of origin of the muscle from the lateral one third of the inguinal ligament it

could not fonn the anterior wall of the inguinal ligament

a) External oblique b) Internal oblique c) Transversus abdominis_ d) Rectus abdominis

12 A large tumor mass impinges on the splenic artery and its branches as the artery pass out from below

the greater curvature of the stomach Branches o(which of the following arteries would most likely to

effected by the pressure on the splenic artery

a Left gastric b Left gastro-epipJoic c Right gastric d Right gastro-epipoloic e Short gastric_

13 A new born baby has projectile vomiting after each feeding It is determined that there is obstruction

of the digestive tract as a result of annular pancreas Annular pancreas is as a result of an abnormality in which of the following process

a Rotation of the dorsal pancreatic bud around the first part of duodenum b Rotation of the dorsal pancreatic bud around the second part of duodenum c Rotation of the dorsal pancreatic bud around the third part of duodenum d Rotation of the ventral pancreatic bud around the first part of duodenum y Rotation of the ventral pancreatic bud around the second part of duodenum

14 As the liver bud enters the ventral mesogastrium the region of the mesogastrium stretching from the

liver to the anterior abdominal wall is called

a Lesser Omentum b Greater Omentum ~ Falcifrom ligament d Lacunar ligament e Ligamentum teres of liver

16 A patient has absence of his 12th rib In such a patient if the doctor makes an incision to approach his

kidney mistaking the 11 th rib for the 12t he would end up injuring

Which of the following arteries is a direct branch of the gastroduodenal artery The

A right gastric artery

B left gastric artery

C inferior pancreaticoduodenal artery D left gastroepiploic artery

i E)right gastroepiploic artery --

E x pI a nation The right gastric artery is typically a branch of the proper hepatic artery The left gastric artery is a direct

branch of the celiac trunk The right and left gastric arteries anastomose along the lesser curvature of the

stomach The inferior pancreaticoduodenal artery is a branch of the superior mesenteric artery it

anastomoses with the superior pancreaticoduodenal in the head of the pancreas The left gastroepiploic

artery is a branch of the splenic artery it anastomoses with the right gastroepiploic artery along the greater

curvature of the stomach The right gastroepiploic artery is a branch of the gastroduodenal artery The

other branch of the gastroduodenal artery is the superior pancreaticoduodenal artery

Which of the following pairs of veins join together to form the portal vein The

A superior mesenteric vein and inferior mesenteric vein

B inferior mesenteric vein and splenic vein

C superior mesenteric vein and splenic vein

Ip)splenic vein and left gastric vein E superior mesenteric vein and left gastric vein

Explanation

The portal vein is formed behind the neck of the pancreas by the union of the superior mesenteric vein

and the splenic vein The inferior mesenteric vein drains into the splenic vein The left gastric vein drains

directly into the portal vein After the portal vein forms it enters the hepatoduodenalligament of the

lesser omentum to reach the liver The portal vein is the most posterior structure in the hepatoduodenal

ligament

At which of the following vertebral levels does the duodenum pass anterior to the aorta - _- shy

All ~

B L2 7~

CL3 I

~DL4

E L5

Explanation

The duodenum begins at the pyloric sphincter at the level of Ll The second (or descending) portion of

the duodenum is to the right of the aorta and extends inferiorly from the level of Ll to the level of L3 The third part of the duodenum crosses the aorta from the right side to the left side at the level of L3 The

fourth (ascending) portion of the duodenum extends from the level of LJ to the level of L2 The

duodenum ends at the duodenojejunal flexure The superior mesenteric artery passes anterior to the

duodenum as the duodenum passes anterior to the aorta The duodenum can be constricted at this level

In which of the following locations will perforation of the digestive tract result in the spilling of luminal

contents into the - lesser peritoneal sac

A Anterior wall of the second portion of the duodenum B Posterior wall of the second portion of the duodenum

C Anterior wall of the stomach

~Posterior wall of the stomach E Posterior wall of the transverse colon

Explanation

The posterior wall of the stomach is related to the lesser peritoneal sac The anterior wall of the stomach is related to the greater peritoneal sac The anterior wall of the second portion of the duodenum is related to the greater peritoneal sac The posterior wall of the second portion of the duodenum is related to the retroperitoneal space The posterior wall of the transverse colon is related to the greater peritoneal sac

The ureter lies against the anterior surface of which of the following muscles shyA Crus oftne diaphragm B Quadratus lumborum

0 Psoas major D Transversus abdominis

E Iliacus

Explanation The ureter exits the renal pelvis at about the level of vertebra L2 As it descends along the posterior abdominal wall it lies on the anterior surface of the psoas major The psoas major muscle arises from the bodies of the lower lumbar vertebrae The psoas major muscle is joined by the iliacus to fonn the

iliopsoas muscle The iliopsoas muscle then attaches to the lesser trochanter of the femur and is the major

flexor of the hip

As the right ureter passes the pelvic brim it lies against the anterior surface of which of the following

blood vessels

A Gonadal artery B Inferiorvena cava C Internal iliac artery

rJ- External Iliac artery

E Inferior mesenteric artery

Explanation

The ureter lies in the extraperitoneal space in the posterior abdominal wall Alter leaving the kidney it

passes inferiorly on the anterior surface of the psoas major muscle At the pelvic brim the ureter passes

into the pelvis At this point the common iliac artery is dividing into the external and iliac arteries The

ureter lies on the anterior surface of the external iliac artery immediately distal to the bifurcation This is a useful landmark for a surgeon to locate the ureter

When extravasated urine passes from the superficial perineal space into the anterior abdominal wall it is

found immediately deep to which of the following layers of the anterior abdominal wall

-ltScarpas fascia

B External oblique muscle

C Internal oblique muscle D Transversus abdominis muscle

E Transversalis fascia

Explanation

The superficial perineal space is bound by Colles fascia the fibrous portion of the superficial fascia This

layer of fascia is continuous with Scarpas fascia the fibrous portion of the superficial fascia of the anterior abdominal wall Therefore urine that is deep to Colles fascia will remain deep to Scarpa s fascia The urine will spread in the plane between Scarpas fascia and the external oblique layer

When a horseshoe kidney develops the ascent of the kidney is restricted by the A internal iliac artery B external Iliac artery

C common iliac artery

inferior mesenteric artery

E superior mesenteric artery

Explanation

A horseshoe kidney develops when the inferior poles of the to kidneys fuse together as they ascend into

the abdomen from the pelvis The first anterior midline vessel that is encountered by the horseshoe kidney

is the inferior mesenteric artery This artery prevents the kidney from continuing its ascent

The left testicular vein drains into which of the following veins

A Left internal iliac vein B Left common iliac vein

bflnferior vena cava D Left renal vein I

E Left internal pudendal vein

Explanation

The left testicular vein drains into the left renal vein The right testicular ~i~[~nsltjectlY into the

inferior vena cava This difference in venous drainage is believed to explain the greater incidence of

varicocele on the left side than on the right The venous drainage from the penis is to the internal vein

which then drains into the internal Iliac vein

The spinal nerve that provides cutaneous branches to the skin around the umbilicus is

A TS B TW-shy

C TI2

DL2 EtA

Explanation

The tenth intercostal nerve is the anterior ramus of the TIO spinal nerve After passing through the tenth

intercostal space the nerve continues forward in the anterolateral abdominal wall in the plane between

the internal oblique muscle and the transversus abdominis muscle In the abdominal wall the nerve innervates to the abdominal wall muscles as well as the skin and the parietal peritoneum The umbilicus is

a useful landmark for the region of distribution of the tenth thoracic nerve

The ligament of the vertebral column that resists its extension is the Aligamentum flavum

B supraspinous ligament

C posterior longitudinal ligament

D anterior longitudinal ligament

E interspinous ligament

Explanation

The ligaments of the vertebral column that resist flexion of the column include the supraspinous ligament

interspinous ligament ligamentum fiavum and posterior longitudinal ligament The ligament that resists

extension is the anterior longitudinal ligament This longitudinal ligament is very broad and strong It

covers the anterior and anterolateral surfaces of the vertebral bodies and the intervertebral disks In

addition to resisting extension the anterior longitudinal ligament provides reinforcement to the anterior

and anterolateral surfaces of the intervertebral disk The posterior longitudinal ligament is relatively

narrow and covers the posterior surface of the vertebral bodies and the intervertebral disks This ligament

reinforces the posterior surface of the disk The posterolateral surface of the disk is not reinforced and it

is through this region that herniation of the nucleus pulposus usually occurs

A patient presents with epigastric and right upper quadrant pain The pain is most intense 2-4 hours after

eating and is reduced by the ingestion of antacids The patient states that he has passed black tarry stools

(melena) within the last week Fiberoptic endoscopy reveals a yellowish crater surrounded by a rim of

erythema that is 3 cm distal to the pylorus Accordingly an ulcer has been identified in the patients

A fundus

B antrum

C duodenum

D jejunum

E ileum

A number of physiologic genetic and other factors increase the risk of gastric (and duodenal) peptic

ulcers The evidence that H pylori plays a principle role is compelling Smoking and caffeine are known to adversely affect the morbidity mortality and healing rates of peptic ulcers In general first-degree

relatives of peptic ulcer patients as well as males have a threefold to fourfold increased risk of developing this disorder Paradoxically in gastric ulcer disease acid secretion is not elevated It is possible that

excess secreted hydrogen ion is reabsorbed across the injured gastric mucosa In general a defect in gastric mucosal defense is the more important local physiologic

A patient presents with symptoms of duodenal obstruction caused by an annular pancreas Annular pancreas is caused by

A rotation of the dorsal pancreatic bud into the ventral mesentery B rotation of the ventral pancreatic bud into the dorsal mesentery

fJ failure of the major and minor pancreatic ducts to fuse ~ ~ cleavage of the ventral pancreatic bud and rotation of the two portions in opposite directions around -the duodenum E formation of one pancreatic bud instead of two

Explanation Normally the ventral pancreatic bud rotates around the gut tube to reach the dorsal pancreatic bud The two buds fuse to form a single pancreas and the distal portions of the two ducts fuse The ventral pancreatic bud forms the inferior portion of the head of the pancreas the uncinate process and the major pancreatic duct (of Wirsung) The dorsal pancreatic bud forms the superior part of the head the neck body and tail and the minor pancreatic duct (of Santorini) Annular pancreas is the result of the ventral pancreatic bud dividing into two portions before it rotates into the dorsal mesentery Each portion rotates in opposite directions to get to the dorsal mesentery thus encircling the duodenum The presence of annular pancreas can constrict the duodenum thus obstructing its lumen

In n _ phranlc----

Gon ~l ----_1 Lum bltano

~~--- CornmQ1t bull ac

+-~4--- lnlllirnaJ ilic

xtem iliac

OBJECTIVE - Identify the blood supply to each of the structures listed in the table on the previous page

Ill give you a head start

FOREGUT - Supplied bV Celiac Tru nk (T12)

Proper hepatic

GastiooUod 13Jafter

1nferlor pancreaticoduodenal artery

Common epatlc

Lett gas ric iiirtery

Spfen artery

shy Gastroepiphgtic artery

~ Superior mesenteric 8rtfry

~

1 Esophagus is a derivative of the foregut so its blood supply originates from the celiac trunk

(T12) The predominant blood supply to abdominal portion of the esophagus is the Esophageal

A (Branch of L Gastric) The venous drainage of the esophagus is particularly important because

it is 1 of 3 clinically relevant sites of Portal Caval anastamoses The Portal Esophageal Vein

meets the Caval Azygos System Persistent bleeding manifests as Esophageal Varices - a fata I

condition

2 The Stomach is also a derivative of the foregut has EXTENSIVE blood supply and is very high

yield on anatomy exams The lesser curvature is supplied superiorly by the L Gastric A (1 of 3

major branches ofthe Celiac trunk) and inferiorly by the R Gastric A ( a branch ofthe proper

Hepatic A) The greater curvature is supplied superiorly by the L Gastroepiploic A (a major

branch of the splenic A) and inferiorly by the R Gastroepiploic A

The Short Gastric arteries (branches of Splenic Artery) supply the fundus of the stomach and

are referred to as EIID ARTERIES because they have no collateral blood supply Therefore if the

splenic artery were occluded (ex - increased pressure in the ommental bursa) - there would be

ischemia to the fundus of the stomach Venous drainage of the stomach is extensive via various

veins lead ing to the portal system Posterior to the stomach the IMV joins the splenic V which

joins the SMV to form the PORTAL VEIN ADAMS

3 Duodenum blood supply has high clinical relevance because it is the junction of the foregut and

midgut and therefore is the site of anastamoses between branches ofthe Celiac Trunk (main

foregut artery) and the Superior Messenteric Artery (main midgut artery) The Proper hepatic

artery gives off the gastroduodenal artery which travels behind the 1st part of the duodenum

This point has high clin ical relevance because duodenal ulcers are very common and a posterior

rupture of the 1st part of the duodenum could rupture the gastroduodenal artery causing

traumatic abdominal bleeding The Gastroduodenal artery first gives off the R Gastroepiploic A

(mentioned above) and proceeds as the Superior pancreatico duodenal artery (supplies the

pancreas and duodenum) which anastamoses with the inferior pancreatico duodenal A (branch

of the SMA) This is the junction of foregut and midgut and occurs near the opening of the

bil iary system into the duodenum (ampula of vater) Portal venous drainage here is responsible

for delivering nutrients from digestion to the liver for metabolism Appreciate that the Superior

mesenteric artery (artery of the midgut) branches from the aorta at Ll travels posterior to the

pancreas than moves anteriorly (at the jxn of the pancreatic headbody) and comes over the

3rd4th part of the duodenum Tumor of the head of the pancreas can compress the SMA

4 Jiver blood supply is via the common hepatic artery (major branch of the cel iac trunk) The

common hepatiC becomes the proper hepatic gives off the R gastric A and the Gastroduodenal

A and then joins the common bile duct and the portal vein in the portal triad Clinical- if a

patient were bleeding from the hepatic A a surgeon can stick his fingers in the epiplOic foramen

and squeeze the free edge of the hepatoduodenalligament in order to stop bleeding to the

area Please note that the hepatic a branches into Rand L hepatic A The Right hepatic artery

gives off the cystic artery which supplies the gallbladder Afferent venous supply is via the

Portal vein which is bringing nutrient rich blood to the liver After metabolism takes place

venous blood leaves the liver through the hepatic veins into the IVC PLEASE UNDERSTAND THE

RELATIONSHIP OF THESE STRUCTURES - ADAMSNETIERSNH Etc

5 Pancreas - Head is supplied via the superior and inferior pancreaticoduodenal arteries

(mentioned above) The tail (situated towards the hilum of the spleen) is supplied via the

pancreatic branches of the splenic artery (END ARTERIES) This blood supply is very important

because the endocrine Alpha and Beta Cells from the pancreatic islets of lagerhans are located

towards the tail This is where Insulin and Glucagon is released to the blood

Now complete this for mid and hindgut structures Make sure to note clinically relevant arterial

anastomoses as well as portal caval anastomoses FYI Appendix blood supply SMA + IMA

anastamoses marginal artery Portalcaval rectal veins fhemmorhoids) and periumbilical caput

medusa are high yield THE BUTT THE GUT and THE CAPUT

Abdominal Development

Liver

Ij1f

II wall b

oh liN ~ VltJrti n be- bull

Pancreas

Secondary Retroperitonealization e I~tl r 1 a v-mtrai m ellter

Rotations of the Gut I i Ij (lIl1UtIJ f~ l r tilt

()l td 10 me l-ft and he v

--~--- -~ -~-~

i

I AolaijonjoI~guf I

STOMACH BED (IDENTIFY IN ADAMS)- the structures posterior to the ommental bursa which

support the stomach in the supine position

Abdomnal JQrUI

Splnic vein

OmQ-oul tv~ ) O(s(Jroa)

Lojt(r o m nturrt (hpJtodu o d~n31 Hid

Gadrl)SplerH (g3stroll~nal) IIgam~nt

hiad h~~atogrtricent IIQdmiddotcrt~)

Lt Dome of Diaphragm (why left Look this up in Adams)

Spleen (What is the blood supply)

Left Kidney (What is the blood supply - AND how is it different from the R kidney)

Suprarenal Gland (What is the Arterial AND Venous Blood supply - how are they different)

Pancreas (How does supply differ from Head to Tail What is the SMA Relationship)

Transverse Mesocolon

liver - ADAMSWET - Make sure you look at the liver in wet lab

Left triangular nl1am~nt

ComoaDj ligamnt

Erophg~1 impre$ioo

Hepatio veins

In1erior -ifena middotr3)Ia

Fibrous appendix o-t

live

impr~j on

Heprorendl p~rtion of Q)(Qllary ligament

Righllri~n9ul r 1I~met

(Common) bile quol

Gr)mmCtr~ hepatic dlJct

Ccentic duct

Duodenal impression

GaJdate p-fr)~S

Hepatic artgtrl prop-f iiiiila - Faloiform ligament

_ - shy Round ligamen liver

~--F-- CoJio imprgt-ssi-on

Prta heptis

Identify the lobes impressions and embryonic remnants associated with the liver

Caudate Lobe Quadrate Lobe Right Lobe Left Lobe Round ligament Falciform Ligament

Ligamentum Venosum (what is its fxn in embryonic life) Hepatic Veins (NOT PART OF THE

PORTAL TRIAD) IVC PORTAL TRIAD - Contents relationship cross section etc Know the

Galbladder relationship to the lobes of the liver

Biliary Duct System - Make sure you understand the sequence of these structures - BE ABLE TO

DRAW A FLOW CHART

TPVd i

t

I t

1 __ Cm-(r

patk GlJet

I

J

Clinical = JAUNDICE is caused by anything that prevents delivery of bile to intestine Tumor of the

head of the pancreas Stones etc Patient will have pale stools and yellowish colored mucus

membranes

Clinical- Any scenario that tells you the patient has BILLOUS VOMIT means that the obstruction to

the flow of digestive contents is after the Ampulla of Vater (Site of Entry of Billiary system to the

duodenum) - ie Duodenal Atresia

Spleen -located posterior to the mid axillary line between ribs 9 and 11 Make sure you know that

the 10th rib is the main axis of the spleen and this organ is susceptible to injury (stab wound errant

thoracoce ntesis etc)

The spleen is derived from mesodermal cells - NOT THE GUT TUBE

The spleen rests on the left colic flexure associates with the tail of the pancreas Know the

structures entering the Hilum of the spleen

Sh rt O~-t~ic 1 0(0 10 rtiltSPIric Iloa nt

(cut)

Peritoneum - similar concept to Pleura - think of a fist in a balloon

Visceral Peritoneum - Layer of balloon touching your fist

Parietal Peritoneum - Layer of balloon not touching your fist

Your fist represents the organ your wrist is the hilum and your arm contains the blood supply

entering the organ

Appreciate that there will never be organs in the peritoneal cavity - rather these organs invaginate

the cavity Kaplan videos

RULES OF NOMENCLATUREshy

1 Organ completely surrounded by peritoneum - peritoneal organ

2 Organ partially surrounded by peritoneum- Retroperitoneal

3 Peritoneum surrounding peritoneal organ is VISCERAL peritoneum

4 Peritoneum surrounding retroperitoneal organ is PARIETAL peritoneum

5 Peritoneum connecting visceral to parietal is called messentary 2 messentaries in the

gut Dorsal (to the gut tube) and ventral (to the gut tube) messentary

Aorta is in Retro peritoneal position - but blood must reach peritoneal position - vessels travel through

messentary All peritoneal organs will have blood supply reaching through messentary

-Mesentery is a 2 layer peritoneum with a neurovascular communication between body wall and organ

- Ligament connects one organ with another or to the abdominal wall (Ommentum = ligament)

lesser Ommentum (attach lesser curvature of stomach and duodenum to liver) =Hepatoduodenal

Ligament and Hepatogastric Ligament

Has a Superior and Inferior Recess (Accumulation of Fluid in Ascites)

Communicates with the greater sac through the epiplic foramen (what structures pass through

this foramen)

Boundaries - you must be able to visualize this

o Anterior - stomach

o Posterior - parietal peritoneum pancreas

o Superior - superior recess (bw diaphragm and coronary ligament)

o Inferior -Inferior recess (bw layers or greater momentum

Greater Ommentum (attach greater curvature of stomach) Gastrophrenic ligament Gastrosplenic

ligament gastrocolic ligament

The greater omentum is the largest peritoneal fold It consists of a double sheet of peritoneum folded on itself so that it is made up of four layers The two layers which descend from the greater curvature of the stomach and commencement of the duodenum pass in front of the small intestines sometimes as low down as the pelvis they then turn upon themselves and ascend again as far as the transverse colon where they separate and enclose that part of the intestine

ABDOMINAL PAIN

Parietal Peritoneum - supplied by same vasculature lymphatics and nerves supplying body wall it

lines and diaphragm Sensitive to pain pressure heat cold well localized

Visceral Peritoneum - supplied by same vasculature lymphatics and somatic nerve of organ it covers

Insensitive to touch heat cold and laceration - referred to dermatome of spinal ganglia providing

sensory fibers Where does appendicitis refer to

Foregut pain - epigastric area (ie - cholycystitis)

Midgut pain - periumbilical area (ie - appendicitis)

Hindgut Pain - suprapubic area (ie - diverticulitis)

Extra ImagesConcepts

ll~_____-

FalifCtrm ligament oind r~ud ligamet f Ilver

Blood from splenio gastriC and inferiof rne$e-rteri v~ins

Ca-I tributaries

Lett gastrio Ifein

Posterior superior pan~reatioodul)denal vaihS

Lott gamo-om~nlal (9aropip lomiddotic) -in

Poq_~ tjol imerl-9-r panCJertlcorllJod-nal veiopound --amp----I- - ~J Right grtr~-omntal

Anwrior interi (gartroepiploic) Jjn

pan euaii cod vl)denal veins middot Inf~Ji (t r mesentric vein

Miqdle (olic vein

Right cl)licvein Sigmoid and rectosigml)id (ei ns

IhH)Collc(~io

--- Mi~dl laquooLJl gtjrltgt

PoM ca vl1 illasto)moses -----shyampoptoageal 2 Paraumbilie-lt11 Inferi or Fectal vei ns

3 Recial 4 REuoperHonea1

Know how the Portal vein is formed I 4 sites of portal caval anastamoses and 1 clinical shunt

Col li t ltt-~ otTl~tI ~nj pc~ 1lt1 turJoG

Ltf 14i1 tImiddot~ artoftl9 on tj phtAt$

L-oftqf 4t t~r 1=laquoIran d 1 bull shy~p l ci rj o fOOOts

Nerves follow the arteries - appreciate the splanchnic nervous system I

Uet~ric branch of left ~nal art

Ureterie branch of righi renal artelY

Left Zld lumbar in and co mlTlunication to as)erdin9 lumbar l(~in Hi ~ht tEZ1~~t~ t3r j t itn ~ nJ l1t- rlnd lfe i r1

Inferior me5nteri~ artery

Notice that the right testicular vein drains directly into the IVC and the right testicular artery drains

directly into the aorta However the left testicular vein drains into the L renal vein at a right angleshy

reason left testicle is lower and more susceptible to varicocele (bag of worms)

Also notice that the left renal vein has a longer course because the IVC is on the right side whereas

the right renal artery has a longer course because the aorta is on the left side

Appreciate the anterior to posterior relationship of structures in the hilum of the kidney - VAP - Vein

Artery Renal Pelvis (Ureter)

11____ __ L_ L_ n VJ __ _ _ t_L I I_ _ L __ L_ I -pound1 bull LI_~-I ____

Posterior View of Head of Pancreas in ( of Duodenum

Celiao hunk

Co mmon ~L~jJth art~ry

GastNduQdonal artrf (partilly in phantn)

P1)Sterior $Up~Jior panCflaticuduodfmal art~r~t

(Co mm on) bile duct

middot~1t~~t-1l---~-~- Right gshomiddotomental (gastoe plp lolc) 3rte (phantomost)

Grener paocre atic art-ry

1n1~rjor pancr-iatlc artery

Jtrifll supejo r pal)oreailcento)dJodenal artr1 (phantom)

Anastomotlo branch

POostetlor bJanch of jo f~ri of pan-reatir(lduodensl drttnj

Anterio r branch of i flferior palcreati~)duodenal art~(phan1om)

Notice the extensive blood supply to the pancreas and duodenum via the branches of the celiac trunk

Notice collateral supply from SMA branches - makes sense bc this is the jxn of foregutmidgut

Identify the vessels in this arteriogram

Hiltid i)f N~ck oi B)dvof Tail 01 pa nereas pan cent~as P-nmiddot-reas panCtCas

I nferie v~na cava

jHept1iic p(lrlai v~in

Port1 tnd H~pti lt a ftH prol

Comm on) bll duct

Ouodtnum

~ft colic (sio)Atta~ hmtrlt jt~xJr-ofha~elSe

muo(IIQn

Right ~lIc (h~j)tic)

il~gtture

In1triol m~oten lIein (rttr op~ritoMdO

SlJp efl or mes~n~fiC amrV and lipln

KNOW YOUR NEIGHBORHOOD

Questions

vVhiJh structure supplied by a bnmdlof the cclia( artery is not derivcd from foregut LemCJUCrITI

(A) Head of the pancte-a5

CD) Pyloric duolenum

Cystkduct

( Liver hepatocyt~~

~F) Body of the spleen

An infant presents with an omrhaJucele at birth -hi oJ the [oHm illg applies to his cM1-dition

(A) It is 31so seen ill p4titnts with aganghonic megacolon

(11) ft reuirs from a fal1ure of resorption of theviteUine d let

(C) It results from herniation at the-site of regression of the right umbilk vein

DJ It is caustd by faihtrc of recanalization of the midgut part of the duodenum

~ It ill camioo by a failuIt vf the midgul to return to the abGQminal uity after herniashytion in-n the urnbilk s l stalk

Ot er than the spleen occlusion Cif the spit-Ilk artery at its odgin wm most likely affect die blood supply to jllch st cnud

(A) Jejunum

(B) Body of th pal1~lltas

(C) LeSStT Cllmiddotlaturc of tl )toma-ch

(D Duodenum dista to the entrance of the Ornmou bile duct

E Fundus of the stomach

A 38-yeu-old batL~er with a history of heartburn suddenly experiences excluciating pain in the (plgastric region of th~ abdomeu SurgCry is perf~rme immediard y upon admisshysion to the 1IlcrgCJliy tuomh~re i~ evidence uf a ruptured ulcer in the posterior waU of the stomach Vhere will a surgeon first fi nd the stomach contenlSf

A) Greater p4ritoneal sac

rB) Cul~de-s~c of Douglas (--

C Omental bursa ~

--D) Paracolic gutter

rEj Between -he panttal perimltum and the posterior body wal1

At birth an infant presents with a st()ma~ rb~tbas~njJled jfltotb~diaplfagru 1A1ltre is the defect thatresulied iiitJle heini~t()n shy~tsophagealbiatus

7 - rH-- Hiatus for the inferior vena cava

( Pleuroperitoneal membrane -(0) Septum transvcrsum

(E) Right Crlt~

An infant born with DOVv7l syndrome presents with bili()u~ vomiting Ahat congenital defect does the infant have

(A) Pyloric stenosis

(B) Meckel diverticulum C) Ornphaloce1e

(D) Gastroschisis

( ~ ) Duodenal atresia y A patient with cirrhosis of the liver presents with ~ bacalvaricestnlreased retrograde pressure in which veins caused the varices

(A) Paraumuilical

(B) Splenic

(ct AzygltJus

(15))G~trk ( (-F) Superior mesemeric

A htaltby 3-year~old male patient experiences a hernial sa protruding from the anterior abdominal wall about halfway between me anterior superior ilia spine and the pubk tuberde Pulsations of al1 artery are palpated medial to the protrusion site through the abdominal walL Which layer of the anterior abdominal wall will first be traversed by the

1hctma

fA) Rectus sheath (B) External oblique aponeurosis

(C) Inguinal ligament

lD) Transversalis fusda

(E) Cremasteric fa~cia

After 5urgi(aj ffpair of a hernia the patient tXperienccs mtmlgtness in the skin on the anteshyrior aspect of the S(Totum_ Vhaf nerve may have been lesioned during thehemiorrhaphy

(A) Femoral

(B) Obturator

(C) Ilioinguinal

(D) lliohypogastrk

(E) Pudendal

A 23~year-LJld female secretary il1 good health ~-uddcn1) doubles over with pain in the a ea of the 1JmbRicu$ Sbe feels vartn and ltneasy and has no appetite That night the pain seems to have mQved to the tower right abdominal regjol1 and she calls her family doctor who then arranges for an ambulance to pk-k her up and take her to the hospitaL Wh ell ntn~ perceived in the area of the urnbilirus most Hkely carried lhe pairfu I sensations into the eNS

tA) Vagus nerves I~

V B)

) Lessersplanchnk nerves

tC) Pudendal nerves

(D) lIiohpogastrk nerves

(E) Greater splam ic l erves

A CT reveals carcinoma in the bOod of the ancreas Vhich blood vessel trut ourses ----~- - -bull ------ --shy

immediately poftterior to the body ofthe pancreas is the m~t likely to be oompressed

(A) Splenk artery

(B) Abdominal aorta (C) Portal vein

(1) Splenic vein

(E) Renal vein

A patient has a penrln1l1ng uker of the posterior wall ot the br~l part ot the (lUooenmn llkh blood vessel is subject to erosion

(A) Common hepatic artery

(B) Gastroouodenal artery

(C) Proper hevatic artery

(D) Celiac artery

(E) Anterior inferior 11amrelltlcoduodcnal attery

Your patient has been diagnosed -ith a carcinoma locallted to the head and l~e(k of the pancreas Another clinical sign would be

A esophageal varices

(8) hemorrhoids

C) a caput medusa

(D) increased pra Teuro n th~ hepatic veins

(E) enlarged right supra lavkular lymph nodes

Wltkh of the foUowing structures develops in the ventral mesentery

(A) Spleen

(B) Jeiunum (C) Head of1ht pancreas (D) Transverse colon (E) Stomach

ti l Uw ~ littwin~ f( S-t lil oai Imdge ~ hi(h or tbt la~)d J truetur tgt liJ llntn nl) he hl p UC iJd [IIi ell

c o

A) drains Ie tht infCrior a La aI

R t middot~nfl0 ~ill to th~ lunlgtn of h i dtlndCrlllfH

(e) m t bull JiJattd on tl l J n T ~H

D ) sup Lc O VSlt I Hlid bhtu l 1 li - -I un oid

( ) U~tpli(t tr j middottUh~ 1 v(( b~nt rfK n1ilc~Zm

ANSWERS AND EXPLANATIONS

Answer E The spleen is t hlttnopodicand lymph organ demlted from mesoderm

Answ~ R Al1 tlmphalocele is caused by it failure of the nlidgut to return to the ahdomir nat cavity after herniation into the umbiliau Stalk Choices Aand D maybe seen in infants with Down syndrome choice D ~s the specific CBuse ofduudcnal JtiCSitt Choice C is (ile cause of gclstrosbisis and Choice B nsults iu a Meurolktldivertku1-tlB

Answer B The fundus ofthe stomach is suppHed by soort gastric brunches of the splenic altery The splenic artery supplies the body and tail of the pancreas part of the greater curvature of the sttmla(h and the spleen Te jejunum part of the head of the pancreas and tht~ duodenum distal to the entrance of the commOll bile duct are supplied by the superior mesenterk artery clll~l ~be less r ctlt1ature cmd the pylQric antrum are supplied by the right and lei gastric art(ries

AnSWftt C Tbeomental bursa or lesser ~ritoneaj sac lies direcdy posterior to the proxshyimal part of the duodeTtlm and the stomach and would be the first site where stomach contents ~Ott1d be fpoundluncL

Answer C A defect in a llleuropcritoneal membrane (uswlly the left) is the typical site of i1 cc-ngenitlI diilphragluatic hemia llere the membr4ne fails to dose ()pound( of the perishycCirdiopcritulleal canals

Answer E DuoJenal atresia and aganglionic megacoion are congwitaI defects S~Il in patients with Dowmiddotnsyndrome

Answer D RulaTgemt~llt of and retrograde flow in g~lstrk vel_ns in particlJl~r the kft gas~ tricveins dilates the capillary bed in rhe wall of the esophagus in (ases of porta yper~

tension Blood flow would increase in and dilampte tribntarkgts of the (lZygOUS vein on the other side of the capiUary bed but flow in this vein is in the typical direction t()ward the superior vena cava Paraumbiii(ltU vein eilgorgement contributes to a caput medusH Splenic ~nlargement might prc~nt with 5plcnonlegaly and balt-kflow in to tlu superior m~~ntclic vein occurs but is asymptomatic

Answer D The patient hagt an indirect inguinal hernia whi~h emerges from the antt-rior abdominal wall through the deep inguinltilling Theeep ring is a fault in the transv~rshysaUs fascia this I~yer wiIJ be penetrated first by the hernia

An~Wer C The ilioinguinal nenc which provides sens~llion to the lnedlal thigh ltmclanteshytior SClotunl pass~lt th rough the 5uperfh_ial inguinal ring ind $subject to inj i1T) becaus-e

it is in the operatitm Held of the erniorrhapny

Auswer B The leMHr splanchnic nerves are sympathdic nerVlts that carry viscera l sensashytlltgtrogt ftom illtllt1m~d ()J stietched gust (itinteitinal ~tructures (in this case the pprndix) into tnt eNS Lesser splanchnic ntTYcsarisc from thmiddot T9--T12 spinal cord segments lt1nd provide sympathetic innenation tD rnidgut siruc1ures whiCh include CLe app~JldD Viscera] Pain arising from affecLed Inidgut ampt 1C1ure is referred over the same dl- matorne~ of spinal segrnertts v-hich provide the sympathetic Innervation n this G1SC of appendicitis the invohen~n t of the ltire) of t e unlhHku indud s the T 10 dermatome

Answer B Of the five choices onty the dscending olon is retroperiton~al aldwould be a lik ~ ( choice to be seen immediately a(~jilcent to t11e posterior abdominal middotn~L

Amwen D The SpltftlC ~-ein ourses posterior to the body of the panneas m its way tt drain into the superior mCSfttltlri( vein

Answcr B TILt glstrodllolticnal artery 1 direct hIamh of the comrootl hepatic artery courses immediately pt))iwri() to the duodenum and is slbject to erosion

Answer B Carcinoma of th pan middott3S in the 1tilt1 may compreampgt the portltil vein at irs orishygill The poTtai vcin is fomled when the splenic vein jQiaswith tfie superior meStllt eric vein The inferiot mesenteric vein joins the ~plenjc vein just priOT to tlli~ point at which the splenic joins the superior Jlleit1ltcri( vein Increescd venous presslu in the inferior mesenteric vein is a cause of emo hoid~

Answer C The- velltral pancreas wilich forms most of the head of the p ~ncr as develops in the ventral mes(ntery as antutgrowth of the hepatic diverticulum Th~ hepatic divershyticulull induding the biIJary appa~atus develops in tbe ventral mesentery of the foregut

Answer~ A The superior mesenteric ~in joins with the spienkvein to form the hepatic portal vciu

Answer D The structure at gttlK is the proper hepatic artery~ whkh suppUesoxygenated b middotood to the liver

MAKE SURE YOU KNOW the diff bw Rectus Sheath above and below the arcuate line

ABOVE

Aponeurosis of xiiltmal obllque musclo

Extemll f)biquw musde

Reotln ilbdomlnls musole S~in

Internal 9bliquQ mY~QI

AponeUfOsi$ of hJH$V~~S Lir9a a lb lbdolTlin~ musolo Tri OJV6 rUi

atldomlnis mUS(loe

Sub cutanlilous tiue (tatty ye r)

BElOW

A POrl lJfosis 01 etemal oblique muscle

Aponeul~)sis 01 Internal oblique mU$cl~

Anteriol lay~ of r~ltdus st~ath EXttom1 oblique rnu$cll

Rectus Jbdominis muscle Intoernal Aponeurc-sis of tra~fersU$ oblique muscle-

at-domlnis muscentl ~ Skio

Tra nsvitSus abdomioLs ml)ZClt

TralSVersaHs fascia Medial umQil iegtt1 1i9Jment -and folj

Uldchus Peritoneum (ir median Umbilj~al Suboutane ous

Extraprftone 11ascia

Ymbilimiddot~1 fold)

preu9poundiea1 fascia

tissue (fatty 4nd m~mbr3n(iUS layers)

o Above the arcuate line (A horizontal line 13 of the distance bw the umbilicus and the

pubic symphysis) -10 Aponeurosis divides into an AntPost Laminae

o The Ant Laminae joins EO and Post Laminae joins Trans Abdominis = Ant and Post

RECTUS SHEATH respectively

o BElOW the arcuate line - all 3 aponeurosis join ANTERIOR to rectus muscle to meet its

counterpart in the midline (linea Alba)

o Take away Msg - The abdomen is devoid of a posterior rectus sheath below the

arcuate line and is therefore more vulnerable to herniasinjuries

Question - A physician makes a deep incision in the patients midline immediately superior to

the pubic symphysis which of the following layers is his knife least likely to pass

Rectus Abdominis External Oblique Ant Rectus Sheath Posterior Rectus Sheath All of the

Above

Answer - All of the above None of the other answer choices are midline structures -LINEA

ALBA

Linea Alba has very poor blood supply - doesnt heal well after surgery Therefore this is a

common site for incisional hernias

a Spleen b Transverse colon c Descending colon d Stomach e Pleura

17 Meckels diverticulum is normally found 2 feet proximal from the

a Pyloric sphincter b Lower esophageal sphincter c Ileo-cecal valve d Middle valve of Huston e Anal valve

18 Ulcer in the posterior wall of the first part of the duodenum would erode ___ artery and would cause bleeding

a Left gastric b Right gastric c Hepatic artery proper d Gastroduodenal artery e Middle colic artery

19 An inflamed appendix is identified by a surgeon on the operation table by noting

a The appendicies epiploicae b The convergence of tenia c The artery of Drummond d The mesocolon e The mesosalphinx

20 The nerve which emerges through the psoas major is

a Femoral b Ilio-inguinal c Ilio-hypogastric d Pudendal e Subcostal

21 The right gonadal vein drains into the

a Azygos b Hemiazygos c Inferior Vena Cava d Right renal vein e Left renal vein

22 The hepatocytes in the liver is derived from

a Ectoderm b Endoderm c Mesoderm

d Neural ectoderm

23 Abscess in the lumbar vertebrae due to tuberculosis would spread to the adjacent muscle which is

a Psoas Major b Iliacus c Quadratus lumborum d Tranversus Abdominis

24 The anterior wall of the inguinal canal is formed by

a External oblique and transverses abdominis b External oblique and fascia transversalis c Internal oblique and external oblique d Internal oblique and transverses abdominis e Fascia transversalis and peritoneum

Meckels diverticulum is a result of which of the following developmental abnormalities shy

A Failure of the vitelline duct to close

B Failure of the herniated intestinal loop to retract into the abdomen

C Failure of the urachus to close

D Failure of the midgut to rotate

E Failure of the hepatic duct to close

Explanation

Meckels diverticulum is a result of the persistence of the proximal part of the vitelline duct This

diverticulum is usually found about 2 feet proximal to the ileocecal junction and is usually about 2 inches

long It is present in about 2 of the popUlation It may be the site of ectopic pancreatic tissue or gastric

mucosa and may develop inflammatory processes and ulcerations Acute Meckels diverticulitis

simulates appendicitis

Which of the following veins carries blood from the esophagus to the portal vein The

A right gastric vein

B left gastric vein c splenic vein D azygos vein

E left gastroepiploic vein

Explanation

The left gastric vein a direct branch of the portal vein drains blood from the lesser curvature of the

stomach and the inferior portion of the esophagus Because branches of the portal vein do not have

valves blood can flow in a retrograde path when there is an obstruction to flow through the portal system or liveL Rlooci Cln then flow from the nortl] vein thr()1Ph the left PRstric vein to the esonhlPlIS lno

through venous communications within the submucosa of the esophagus to esophageal veins that drain

into the azygos vein The increase in blood flow through the esophageal submucosal veins results in esophageal varices

On the posterior wall of the abdomen the celiac ganglion A contains cell bodies of postganglionic parasympathetic neurons B is synapsed upon by neurons in the posterior vagal trunk C is synapsed upon by neurons in the greater splanchnic nerve D contains sensory cell bodies of lumbar spinal nerves E contains cell bodies of neurons that cause an increase in the rate of peristasis

Explanation The celiac ganglion is one of the preaortic ganglia of the sympathetic nervous system It contains cell bodies of postganglionic sympathetic neurons The sympathetic splanchnic nerves contain preganglionic sympathetic neurons that pass through the sympathetic chain without synapsing These splanchnic nerves go to the preaortic ganglia to synapse The greater splanchnic nerve contains preganglionic neurons from spinal cord segments T5-T9 This nerve synapses in the celiac ganglion The nerve fibers in the vagal trunks are preganglionic parasympathetic fibers that go to the walls of the organs that they will innervate and synapse on postganglionic parasympathetic neurons in the walls of those organs Cell bodies of sensory neurons in the abdomen are found in the dorsal root ganglia or the sensory ganglia of the vagus nerve Sympathetic innervation decreases the rate of peristalsis parasympathetic innervation increases the rate of peristalsis

Which of the following pairs of arteries will allow blood to bypass an occlusion of the celiac trunk

A Left gastric artery-right gastric artery

B Left gastroepiploic artery-right gastroepiploic artery

C Superior pancreaticoduodenal artery-inferior pancreaticoduodenal artery

D Splenic artery-common hepatic artery

E Left gastric artery - proper hepatic artery

Explanation The anastoOlosis of a branch of the celiac trunk and a branch of the superior mesenteric artery will

provide collateral circulation around an occlusion of the celiac trunk Each of the other choices pair

branches of the celiac trunk therefore these will not provide collateral flow around the obstruction of the

celiac trunk The left gastric splenic and common hepatic arteries are direct branches of the celiac trunk

The right gastric artery is a branch of the proper hepatic artery which is a branch of the common hepatic artery The left gastroepiploic artery is a branch of the splenic artery The right gastroepiploic artery is a

branch of the gastroduodenal artery whlch is a branch of the common hepatic artery

Which of the following organs has appendices epiploica The

A sigmoid colon

Bjejunum

C duodenum

D stomach E esophagus

Explanation Appendices epiploica are characteristic of the colon Appendices epiploica are subserosal accumulations

of fat None of the organs of the gastrointestinal tract has appendices epiploica except the colon

Page 15: Chirag's Abdomen Review

~ Congenital defects including central nervous system disorders (eg Anencephaly) and gastrointestinal defects (atresias ego Duodenal esophageal) prevent the infant from swallowing the amniotic fluid (failure of recanalization)

Oligohydramnios (Amniotic fluid lt 400 mt) ~ Cl~ldberenal-agenesis

bull Midgut _-_

~ Primary Midgut intestinal loop gives rise to bull Distal duodenum bull Jejunum bull Ileum bull Ascending colon bull Transverse colon - proximal two-thirds of the bull Transverse colon with the distal third

~ Primary intestinaltoop bull ncephalic limb distal part of the duodenum the jejunum and part of the ileum bull ncaudal limb lower portion of the ileum the cecum the appendix the ascending colon and

the proximal two-thirds of the transverse colon bull 6th week

bull Rapid elongation of the cephalic limb bull Rapid growth of the liver bull Intestinal loops enter the extraembryonic cavity in the umbilical cord

bull 10th week bull loops begin to return bull regression of the mesonephric kidney reduced growth of the liver expansion of the

abdominal cavity bull Jejunum -left bull Loops - more to the right

bull Cecal bud -last part (temporarily below the right lobe of the liver) ~ qIDlthaloseJe (Structures COlHLoArts9V~1tion)

bull Through umbilical ring bull 6th to 10th weeks

bull Associated with a high rate of mortality (25) and severe malformations bull Associated with chromosome abnormalities

~ Gastroschisis (Structures coming out are not covered by Amnion) bull herniation through the body wall ----=---=-shybull Into the amniotic cavity bull Lateral right of the umbilicus bull Sometimes the inferior wall fails to develop as a result lower abdominal structures like the

bladder would be exposed to the exterior not associated with chromosome abnormalities ~ Abnormalities of the Mesenteries

bull Mobile cecum persistence of mesocolon bull Extreme form - long mesentery bull Volvulus

~ Distal third of the transverse colon ~ Descending colon ~ Sigmoid colon ~ Rectum ~ Upper part of the anal canal ~ Primitive anorectal canal

bull 7th week cloacal membrane ruptures bull Tip of the urorectal septum perineal body bull Pectinate line

~ Hindgut anamolies bull Rectoanal atresias and fistulas bull Imperforate anus bull Congenital megacolon (aganglionic megacolon Hirschsprung disease)

bull

bull Hindgut

Chirags Abdomen Review - Part 2

Understanding Embryo makes learning blood supply EASY

I I

I t

~ -

)

Table l1r-~ L Adult SUmiddotuctu~SDrj~l Froln Each of he Three Dhisions of be Pringttive GUl Tube t-middot-----middotmiddotmiddotmiddot-

Foregu(

I_ (Celiac Trunk)

Ir-slt-gtphgus

S101na(b

I h -= LiJ~r

Pancre=l S

bull 1 i Biliary apparntu5

Gall bladdshy

i Pha11~Cal pltgtuchcs

LullSS-I

Mjig ---- bull __ _- ----n--duct----~---middot-------l--n

(Superior Jldesen1eric Artery)_-1I-(I_~__ middot __ O-=-)_in_middoto_r_M_e_se_n_t_e_r_i_c_An__

Uuodenu rn 2nd_ 3 lt141h V4Tt

Jejunun-~

nc-un]

tCCUJ11

AppltgtndLX

Transver5e -o1on (p~oxiln1l1 ~O Tbird)

bull__hytgtid~ _ _ ______ L _ __

Tr-dn~llt~se colon (diStul h lTd) I

)

i

Aa ca-nal -( uppeT patt) i

I I

_____ __ _ _ _ ___ - - rhe~ a(t clcriVOkt iV(5 opound~lt prbn1rC ~ nlQC blft TlI)( 134tof r~ tIonoinf~ i 1 ~l l1rd c- P Cle

Now Lets see how much youve learned

Questions

1) A pt receives a general anesthetic in preparation for a c~t~~my A right subcostal incision is made which begins near the xyphoid process runs along and immediately beneath the costal margin to an anterior axillary line and transects the rectus abdominus muscle and rectus sheath At the level of the transpyloric plane the anterior wall of the

-~~-~=--- _eco---shysheath of the rectus abdominus muscle receives contributions from which of the following

a Aponeuroses of the in~ande~tef-Ilal o~ues

b Aponeuroses of the transversus abdominis and internal oblique muscles c Aponeuroses of the transversus abdominis and internal and external oblique

muscles d Transversalis fascia e Transversalis fascia and aponeurosis of the transversus abdominus muscle

A

2) The lat~raJJJ11QjJt~gLfgJlLoneach side of the inner surface of the anterior abdominal wall is created by which of the following structures

K Falx inguinalis (~) Inferior epigastric a

c Lateral border of the rectus sheath d Obliterated umbilical a e Urachus

B

3) A man the victim of several knife wounds to the abdomen during a brawl at the Lobster Shack subsequently developed a direct inguinal hernY Damage to which of the following nerves is most likely responsible for the predisposing weakness of the abdominal wall

~ Genitofemoral nerve ( b) Ilioinguinal nerve ~-t Tenth intercostal nerve

d Subcostal nerve e Pelvic splanchnic nerve

B

4) Which of the following statements concerning a direct inguinal hernia is correct a It is the most common type of abdominal hernia b It transverses the entire length of the inguinal canal c It contains all3 fascia layers of the spermatic cord d It exits the inguinal canal via the superficial ingeJinal ring e It protrudes through H~acb strJg e

~(

1fltbS w E

tl

5) The conjoint tendon is

a Important in preventing indirect inguinal hernias b The fused aponeurotic layers of internal abdominal oblique and transversus

abdominus muscles c Posterior to the deep inguinal ring

d Medial fibers of the inguinal ligament

B

6) A 25 year old male is brought in to the ER after being involved in a car accident in which he received a crushed internal injury in his abdomen Examination reveals a lesion of parasympathetic fibers in the vagJsnerve which interferes with glandular secretory or

smooth muscle functions in which of the foliowingorgans a Bladder b Transverse coloiW c Descending colOO d Prostrate gland e Rectum

B

7) The spermatic cord includes all of the following contents except a Il ioinguinal nerve b Pampin iform plexus of veins c Vas deferens d Genitofemoral nerve

A

8 Which abdominal structure gives rise to the internal spermatic fascia (muscle) following the descent of testes in development

a External abdominal oblique aponeurosis b Transversalis fascia c Transversus abdominis muscle d Peritoneum e Internal abdominal oblique

B

9 Which abdominal structure gives rise to the tunica vaginalis fotlowing the descent of testes during development shy

a External abdominal oblique aponeurosis b Transversalis fascia c Transversus abdominis muscle d Peritoneum e Internal abdominal oblique

D

10) The lesser omentum is a peritoneal fold which is su bdivided into the a Hepatogastric and gastrosplenic ligaments b Hepatoduodenal and gastroomentalligaments c Hepatoduodenal and gastrosplenic ligaments d Hepatogastric and hepatoduoden9-jrj igaments

D

11) A posteriorly perforating ulcer in the pyloric antrum of the stomach is most likely to produce initiallocalized peritonitis or abcess formation in which ofthS fQllowing

a Great-sac - -- -

b Paracolic recess

c Omental bursa

d Right subphrenic space

c

The inferior mesenteric artery arises from the abdominal aorta ilm_ediill~y_J-Qs1eriQLto which of the foowing org~ns A-F~t~filie duodenum B Head of the pan~eis C Neck of the pandeas

D Second part of the duodenum

E Third part of the duooenum_shylaquoshy

shy

The correct answer is E The inferior mesenteric artery arises from the anterior surface of the aorta at the level of the third lumbar vertebra The third part of the duodenum crosses the midline at the level of the third lumbar vertebra and passes anterior to the aorta at the origin of the inferior mesenteric artery The

first part of the duodenum (choice A) lies horizontally to the right of the midline at the level of the first

lumbar vertebra The head of the pancreas (choice B) is to the right of the midline and extends from the

level of the first lumbar vertebra to the third lumbar vertebra It lies within the concavity of the

duodenum The neck of the pancreas (choice C) lies in the midline at the level of the first lumbar

vertebra It lies on the anterior surface of the aorta at the origin of the superior mesenteric artery The second part of the duodenum (choice D) lies vertically to the right of the midline and extends from the

level of the first lumbar vertebra to the level of the third lumbar vertebra

The left adrenaLvein drains directly into which of the following veins A Hemiazygos vein

B Inferior vena cavaee C Left renal veiri -

D Splenic vein

E Superior mesenteric vein

a

The correct answer is C The left adrenal vein and the left gonadal vein (either testicular or ovarian) drain into the left renal vein TheTeft renal vein t~ains intothe- inferior vena cava In contrast the right

adrenal ~~inandnght gonadal veindrai~ gLr~ctJy iQtoJhe iilferiQ[ Vencava -- -

ThehemTazygoS7ein- (~h-~i-~ A)~~c~i~es the venous drainage from the body wall on the left side of the

thorax and abdomen No visceral organs drain directly to the azygos or hemiazygos veins The inferior vena cava (choice B) receives the direct venous drainage from the right adrenal vein but not

the left adrenal vein Remember the inferior vena cava is on the right side of the abdomen The splenic

vein (choice D) receives the venous drainage from the spleen and part of the pancreas and stomach The splenic vein is part of the portal venous system

The superior mesenteric vein (choice E) receives venous drainage from much of the intestinal tract It is part of the portal venous system and joins with the splenic vein to form the portal vein

A 43-year-old man presents complaining of pain in the groin On examination his physician palpates a

bulge in the region of the superficial inguinal ring which he diagnoses as a direct inguinal hernia The hernial sac most likely

A is covered by all three layers of the spennatic fascia B passes medial to the inferior epi gastric artery

C passes medial to the lateral border of the rectus abdominis muscle

D passes posterior to the inguinal ligament E passes through the deep inguinal ring

The correct answer is B Direct inguinal hernias enter the inguinal canal by tearing through the posterior

wall of that structure The typical location for this type of hernia is through the inguinal triangle bounded

laterally by the inferior epigastric artery medially by the lateral border of the rectus abdominis and

inferiorly by the inguinal ligament Direct inguinal hernias pass medial to the inferior epigastric artery

whereas indirect inguinal hernias pass lateral to the inferior epigastric artery because the deep inguinal

ring is lateral to the artery Indirect inguinal hernias are covered by all three layers of the spermatic fascia (choice A) Direct inguinal hernias are covered by fewer than all three layers because the direct inguinal

hernia tears through one or more layers of fascia as it emerges though the abdominal wall The lateral

border of the rectus abdominis muscle (choice C) forms the medial border of the inguinal triangle All

inguinal hernias pass lateral to the rectus abdominis Femoral hernias pass posterior to the inguinal ligament (choice D) Inguinal hernias emerge through the superficial inguinal ring which is superior to the inguinal ligament Inguinal hernias that descend below the inguinal ligament pass anterior to the

ligament Indirect inguinal hernias pass through the deep inguinal ring (choice H) direct inguinal hernias

do not Both types of inguinal hernias pass through the superficial inguinal ring

During a gastric resection in a patient with stomach cancer a surgeon wants to remove the lesser

omentum because of tumor extension into it Which of the following structures lie in the free edge of the

l~~g omentum and consequently must be dissected out in order to be preserved

A Common bile duct cystic duct and hepatic artery 6

B Cystic duct hepatic artery and hepatic vein

e Hepatic vein and cystic duct

Portal vein common bile duct and hepatic artery

E Portal vein hepatic artery and hepatic vein

The correct answer is D The free edge of the lesser omentum contains three important structures the

common bile duct the hepatic artery and the portal vein Nei ther the cystic duct (choices A B and C) nor the hepatic vein (choices B C and E) lies in the free

edge of the lesser omentum

A 55-year-old male patient with chronic liver disease has portal hypertension To relieve the pressure in the portal system a porto-caval shunt is performed Which of the following veins may by anastomosed to

accomplish this porto-caval shunt A Left renal vein-left testicular veingt

B Right renal vein-right suprarenal vein I shy

e Splenic vein -left renal vein J

D Superior mesenteric vein-inferior mesenteric vein E Superior mesenteric vein-splenic vein

The correct answer is C The splenic vein drains directly into the portal vein The left renal vein drains

directly into the inferior vena cava Anastomosis of these veins would allow blood from the portal vein to

drain retrograde though the splenic vein into the renal vein and then into the inferior vena cava The left

renal vein (choice A) drains directly into the inferior vena cava The left testicular vein drains directly into

the left renal vein Thus these veins are already in communication and neither vein is part of the portal venous system The right renal vein (choice B) drains directly into the inferior vena cava The right

suprarenal vein also drains directly into the inferior vena cava Thus neither vein is part of the portal

venous system The superior mesenteric vein (choice D) drains directly into the portal vein The inferior

mesenteric vein drains into the splenic vein which then drains into the portal vein Thus neither vein is

part of the caval venous system The superior mesenteric vein (choice E) drains directly into the portal

vein The splenic vein also drains directly into the portal vein Thus neither vein is part of the caval

venous system

A 12 year old boy has fever vomiting and para-umbilical pain After examining the patient the doctor

makes an initial diagnosis of appendicitis Appendicular pain which is initially referred to the umbilicus goes to the dorsal root ganglion of

a TI b TI2 c L1 d T7

(e I TIO

A 59-year-old male undergoes a neurological examination which reveals that when the abdominal wall is

stroked the muscles of the abdominal wall of the side of the body stimulated failed to contract Other

neurological tests appeared normal The likely region affected includes

a CI - C5 spinal segments b C6 - TI c T2-TI ~T8-T12

e Ll- L5

The surgery done to relive portal hypertension is done by connecting two veins Which of the following veins would be suitable for connection

a Inferior vena cava and portal vein b Superior vena cava and portal vein c Splenic vein and right renal vein d Splenic vein and left renal vein e Superior mesenteric vein and Inferior vena cava

A mother brings her 3-week-old infant to the pediatric clinic reporting a new scrotal bulge that she found -~-

while changing a diaper yesterday The infant is afebrile Physical examination reveals a palpable mass in

the scrotum while in the standing position resolution of the mass in the supine position and no

transillumination of the scrotal sac What is the most likely diagnOSiS

a Cryptorchidism b Direct inguinal hernia c Hydrocele d Indirect inguinal hernia ~ e varicocele

The Vagal trunks enter the abdomen by passing through which of the following openings in the

diaphragm

a Right crus b Esophageal hiatus ~ c Vena caval hiatus d Aortic hiatus e Left crus

2 The anterior boundary of the epiploic foramen of Winslow is bounded by

a) First part of duodenum b) Lesser curvature of stomach c) Liver d) Hepato-duodenalligament v ~

3 The ilio-inguinal nerve is derived from

a TI2 ry b LI c L2 d L3 e L23

15 Surgically the structure used to suspend the kidney to the diaphragm is

a) Renal fascia b) True capsule c) Perinephric fat d) Paranephric fat

6 If there is portal obstruction because of carcinoma affecting the pancreas which of these of the

following signs would be present

a Caput medusae b Esophageal varices c Rectal varices c

d Pulmonary edema

7 In a sliding hernia the gastro-esophageal junction lies

a) At its normal position b) Below the normal position c) Above the normal position V d) None of the above

8 Which of the following structures is retroperi toneal

A transverse colon B spleen IJ2f6 C ileum D descending colon v r 1pound1111111

9 The renal angle is fonned lgtetween the 12th rib and ______ muscle

a Psoas major -middotshyb Erector spinae c Quadratus Iumborum d Diaphragm

10 The anterior structure at the hilum of the kidney is

a) Renal vein ~

b) Renal artery I middot~ I

c) Ureter d) Accessory renal artery

11 Because of origin of the muscle from the lateral one third of the inguinal ligament it

could not fonn the anterior wall of the inguinal ligament

a) External oblique b) Internal oblique c) Transversus abdominis_ d) Rectus abdominis

12 A large tumor mass impinges on the splenic artery and its branches as the artery pass out from below

the greater curvature of the stomach Branches o(which of the following arteries would most likely to

effected by the pressure on the splenic artery

a Left gastric b Left gastro-epipJoic c Right gastric d Right gastro-epipoloic e Short gastric_

13 A new born baby has projectile vomiting after each feeding It is determined that there is obstruction

of the digestive tract as a result of annular pancreas Annular pancreas is as a result of an abnormality in which of the following process

a Rotation of the dorsal pancreatic bud around the first part of duodenum b Rotation of the dorsal pancreatic bud around the second part of duodenum c Rotation of the dorsal pancreatic bud around the third part of duodenum d Rotation of the ventral pancreatic bud around the first part of duodenum y Rotation of the ventral pancreatic bud around the second part of duodenum

14 As the liver bud enters the ventral mesogastrium the region of the mesogastrium stretching from the

liver to the anterior abdominal wall is called

a Lesser Omentum b Greater Omentum ~ Falcifrom ligament d Lacunar ligament e Ligamentum teres of liver

16 A patient has absence of his 12th rib In such a patient if the doctor makes an incision to approach his

kidney mistaking the 11 th rib for the 12t he would end up injuring

Which of the following arteries is a direct branch of the gastroduodenal artery The

A right gastric artery

B left gastric artery

C inferior pancreaticoduodenal artery D left gastroepiploic artery

i E)right gastroepiploic artery --

E x pI a nation The right gastric artery is typically a branch of the proper hepatic artery The left gastric artery is a direct

branch of the celiac trunk The right and left gastric arteries anastomose along the lesser curvature of the

stomach The inferior pancreaticoduodenal artery is a branch of the superior mesenteric artery it

anastomoses with the superior pancreaticoduodenal in the head of the pancreas The left gastroepiploic

artery is a branch of the splenic artery it anastomoses with the right gastroepiploic artery along the greater

curvature of the stomach The right gastroepiploic artery is a branch of the gastroduodenal artery The

other branch of the gastroduodenal artery is the superior pancreaticoduodenal artery

Which of the following pairs of veins join together to form the portal vein The

A superior mesenteric vein and inferior mesenteric vein

B inferior mesenteric vein and splenic vein

C superior mesenteric vein and splenic vein

Ip)splenic vein and left gastric vein E superior mesenteric vein and left gastric vein

Explanation

The portal vein is formed behind the neck of the pancreas by the union of the superior mesenteric vein

and the splenic vein The inferior mesenteric vein drains into the splenic vein The left gastric vein drains

directly into the portal vein After the portal vein forms it enters the hepatoduodenalligament of the

lesser omentum to reach the liver The portal vein is the most posterior structure in the hepatoduodenal

ligament

At which of the following vertebral levels does the duodenum pass anterior to the aorta - _- shy

All ~

B L2 7~

CL3 I

~DL4

E L5

Explanation

The duodenum begins at the pyloric sphincter at the level of Ll The second (or descending) portion of

the duodenum is to the right of the aorta and extends inferiorly from the level of Ll to the level of L3 The third part of the duodenum crosses the aorta from the right side to the left side at the level of L3 The

fourth (ascending) portion of the duodenum extends from the level of LJ to the level of L2 The

duodenum ends at the duodenojejunal flexure The superior mesenteric artery passes anterior to the

duodenum as the duodenum passes anterior to the aorta The duodenum can be constricted at this level

In which of the following locations will perforation of the digestive tract result in the spilling of luminal

contents into the - lesser peritoneal sac

A Anterior wall of the second portion of the duodenum B Posterior wall of the second portion of the duodenum

C Anterior wall of the stomach

~Posterior wall of the stomach E Posterior wall of the transverse colon

Explanation

The posterior wall of the stomach is related to the lesser peritoneal sac The anterior wall of the stomach is related to the greater peritoneal sac The anterior wall of the second portion of the duodenum is related to the greater peritoneal sac The posterior wall of the second portion of the duodenum is related to the retroperitoneal space The posterior wall of the transverse colon is related to the greater peritoneal sac

The ureter lies against the anterior surface of which of the following muscles shyA Crus oftne diaphragm B Quadratus lumborum

0 Psoas major D Transversus abdominis

E Iliacus

Explanation The ureter exits the renal pelvis at about the level of vertebra L2 As it descends along the posterior abdominal wall it lies on the anterior surface of the psoas major The psoas major muscle arises from the bodies of the lower lumbar vertebrae The psoas major muscle is joined by the iliacus to fonn the

iliopsoas muscle The iliopsoas muscle then attaches to the lesser trochanter of the femur and is the major

flexor of the hip

As the right ureter passes the pelvic brim it lies against the anterior surface of which of the following

blood vessels

A Gonadal artery B Inferiorvena cava C Internal iliac artery

rJ- External Iliac artery

E Inferior mesenteric artery

Explanation

The ureter lies in the extraperitoneal space in the posterior abdominal wall Alter leaving the kidney it

passes inferiorly on the anterior surface of the psoas major muscle At the pelvic brim the ureter passes

into the pelvis At this point the common iliac artery is dividing into the external and iliac arteries The

ureter lies on the anterior surface of the external iliac artery immediately distal to the bifurcation This is a useful landmark for a surgeon to locate the ureter

When extravasated urine passes from the superficial perineal space into the anterior abdominal wall it is

found immediately deep to which of the following layers of the anterior abdominal wall

-ltScarpas fascia

B External oblique muscle

C Internal oblique muscle D Transversus abdominis muscle

E Transversalis fascia

Explanation

The superficial perineal space is bound by Colles fascia the fibrous portion of the superficial fascia This

layer of fascia is continuous with Scarpas fascia the fibrous portion of the superficial fascia of the anterior abdominal wall Therefore urine that is deep to Colles fascia will remain deep to Scarpa s fascia The urine will spread in the plane between Scarpas fascia and the external oblique layer

When a horseshoe kidney develops the ascent of the kidney is restricted by the A internal iliac artery B external Iliac artery

C common iliac artery

inferior mesenteric artery

E superior mesenteric artery

Explanation

A horseshoe kidney develops when the inferior poles of the to kidneys fuse together as they ascend into

the abdomen from the pelvis The first anterior midline vessel that is encountered by the horseshoe kidney

is the inferior mesenteric artery This artery prevents the kidney from continuing its ascent

The left testicular vein drains into which of the following veins

A Left internal iliac vein B Left common iliac vein

bflnferior vena cava D Left renal vein I

E Left internal pudendal vein

Explanation

The left testicular vein drains into the left renal vein The right testicular ~i~[~nsltjectlY into the

inferior vena cava This difference in venous drainage is believed to explain the greater incidence of

varicocele on the left side than on the right The venous drainage from the penis is to the internal vein

which then drains into the internal Iliac vein

The spinal nerve that provides cutaneous branches to the skin around the umbilicus is

A TS B TW-shy

C TI2

DL2 EtA

Explanation

The tenth intercostal nerve is the anterior ramus of the TIO spinal nerve After passing through the tenth

intercostal space the nerve continues forward in the anterolateral abdominal wall in the plane between

the internal oblique muscle and the transversus abdominis muscle In the abdominal wall the nerve innervates to the abdominal wall muscles as well as the skin and the parietal peritoneum The umbilicus is

a useful landmark for the region of distribution of the tenth thoracic nerve

The ligament of the vertebral column that resists its extension is the Aligamentum flavum

B supraspinous ligament

C posterior longitudinal ligament

D anterior longitudinal ligament

E interspinous ligament

Explanation

The ligaments of the vertebral column that resist flexion of the column include the supraspinous ligament

interspinous ligament ligamentum fiavum and posterior longitudinal ligament The ligament that resists

extension is the anterior longitudinal ligament This longitudinal ligament is very broad and strong It

covers the anterior and anterolateral surfaces of the vertebral bodies and the intervertebral disks In

addition to resisting extension the anterior longitudinal ligament provides reinforcement to the anterior

and anterolateral surfaces of the intervertebral disk The posterior longitudinal ligament is relatively

narrow and covers the posterior surface of the vertebral bodies and the intervertebral disks This ligament

reinforces the posterior surface of the disk The posterolateral surface of the disk is not reinforced and it

is through this region that herniation of the nucleus pulposus usually occurs

A patient presents with epigastric and right upper quadrant pain The pain is most intense 2-4 hours after

eating and is reduced by the ingestion of antacids The patient states that he has passed black tarry stools

(melena) within the last week Fiberoptic endoscopy reveals a yellowish crater surrounded by a rim of

erythema that is 3 cm distal to the pylorus Accordingly an ulcer has been identified in the patients

A fundus

B antrum

C duodenum

D jejunum

E ileum

A number of physiologic genetic and other factors increase the risk of gastric (and duodenal) peptic

ulcers The evidence that H pylori plays a principle role is compelling Smoking and caffeine are known to adversely affect the morbidity mortality and healing rates of peptic ulcers In general first-degree

relatives of peptic ulcer patients as well as males have a threefold to fourfold increased risk of developing this disorder Paradoxically in gastric ulcer disease acid secretion is not elevated It is possible that

excess secreted hydrogen ion is reabsorbed across the injured gastric mucosa In general a defect in gastric mucosal defense is the more important local physiologic

A patient presents with symptoms of duodenal obstruction caused by an annular pancreas Annular pancreas is caused by

A rotation of the dorsal pancreatic bud into the ventral mesentery B rotation of the ventral pancreatic bud into the dorsal mesentery

fJ failure of the major and minor pancreatic ducts to fuse ~ ~ cleavage of the ventral pancreatic bud and rotation of the two portions in opposite directions around -the duodenum E formation of one pancreatic bud instead of two

Explanation Normally the ventral pancreatic bud rotates around the gut tube to reach the dorsal pancreatic bud The two buds fuse to form a single pancreas and the distal portions of the two ducts fuse The ventral pancreatic bud forms the inferior portion of the head of the pancreas the uncinate process and the major pancreatic duct (of Wirsung) The dorsal pancreatic bud forms the superior part of the head the neck body and tail and the minor pancreatic duct (of Santorini) Annular pancreas is the result of the ventral pancreatic bud dividing into two portions before it rotates into the dorsal mesentery Each portion rotates in opposite directions to get to the dorsal mesentery thus encircling the duodenum The presence of annular pancreas can constrict the duodenum thus obstructing its lumen

In n _ phranlc----

Gon ~l ----_1 Lum bltano

~~--- CornmQ1t bull ac

+-~4--- lnlllirnaJ ilic

xtem iliac

OBJECTIVE - Identify the blood supply to each of the structures listed in the table on the previous page

Ill give you a head start

FOREGUT - Supplied bV Celiac Tru nk (T12)

Proper hepatic

GastiooUod 13Jafter

1nferlor pancreaticoduodenal artery

Common epatlc

Lett gas ric iiirtery

Spfen artery

shy Gastroepiphgtic artery

~ Superior mesenteric 8rtfry

~

1 Esophagus is a derivative of the foregut so its blood supply originates from the celiac trunk

(T12) The predominant blood supply to abdominal portion of the esophagus is the Esophageal

A (Branch of L Gastric) The venous drainage of the esophagus is particularly important because

it is 1 of 3 clinically relevant sites of Portal Caval anastamoses The Portal Esophageal Vein

meets the Caval Azygos System Persistent bleeding manifests as Esophageal Varices - a fata I

condition

2 The Stomach is also a derivative of the foregut has EXTENSIVE blood supply and is very high

yield on anatomy exams The lesser curvature is supplied superiorly by the L Gastric A (1 of 3

major branches ofthe Celiac trunk) and inferiorly by the R Gastric A ( a branch ofthe proper

Hepatic A) The greater curvature is supplied superiorly by the L Gastroepiploic A (a major

branch of the splenic A) and inferiorly by the R Gastroepiploic A

The Short Gastric arteries (branches of Splenic Artery) supply the fundus of the stomach and

are referred to as EIID ARTERIES because they have no collateral blood supply Therefore if the

splenic artery were occluded (ex - increased pressure in the ommental bursa) - there would be

ischemia to the fundus of the stomach Venous drainage of the stomach is extensive via various

veins lead ing to the portal system Posterior to the stomach the IMV joins the splenic V which

joins the SMV to form the PORTAL VEIN ADAMS

3 Duodenum blood supply has high clinical relevance because it is the junction of the foregut and

midgut and therefore is the site of anastamoses between branches ofthe Celiac Trunk (main

foregut artery) and the Superior Messenteric Artery (main midgut artery) The Proper hepatic

artery gives off the gastroduodenal artery which travels behind the 1st part of the duodenum

This point has high clin ical relevance because duodenal ulcers are very common and a posterior

rupture of the 1st part of the duodenum could rupture the gastroduodenal artery causing

traumatic abdominal bleeding The Gastroduodenal artery first gives off the R Gastroepiploic A

(mentioned above) and proceeds as the Superior pancreatico duodenal artery (supplies the

pancreas and duodenum) which anastamoses with the inferior pancreatico duodenal A (branch

of the SMA) This is the junction of foregut and midgut and occurs near the opening of the

bil iary system into the duodenum (ampula of vater) Portal venous drainage here is responsible

for delivering nutrients from digestion to the liver for metabolism Appreciate that the Superior

mesenteric artery (artery of the midgut) branches from the aorta at Ll travels posterior to the

pancreas than moves anteriorly (at the jxn of the pancreatic headbody) and comes over the

3rd4th part of the duodenum Tumor of the head of the pancreas can compress the SMA

4 Jiver blood supply is via the common hepatic artery (major branch of the cel iac trunk) The

common hepatiC becomes the proper hepatic gives off the R gastric A and the Gastroduodenal

A and then joins the common bile duct and the portal vein in the portal triad Clinical- if a

patient were bleeding from the hepatic A a surgeon can stick his fingers in the epiplOic foramen

and squeeze the free edge of the hepatoduodenalligament in order to stop bleeding to the

area Please note that the hepatic a branches into Rand L hepatic A The Right hepatic artery

gives off the cystic artery which supplies the gallbladder Afferent venous supply is via the

Portal vein which is bringing nutrient rich blood to the liver After metabolism takes place

venous blood leaves the liver through the hepatic veins into the IVC PLEASE UNDERSTAND THE

RELATIONSHIP OF THESE STRUCTURES - ADAMSNETIERSNH Etc

5 Pancreas - Head is supplied via the superior and inferior pancreaticoduodenal arteries

(mentioned above) The tail (situated towards the hilum of the spleen) is supplied via the

pancreatic branches of the splenic artery (END ARTERIES) This blood supply is very important

because the endocrine Alpha and Beta Cells from the pancreatic islets of lagerhans are located

towards the tail This is where Insulin and Glucagon is released to the blood

Now complete this for mid and hindgut structures Make sure to note clinically relevant arterial

anastomoses as well as portal caval anastomoses FYI Appendix blood supply SMA + IMA

anastamoses marginal artery Portalcaval rectal veins fhemmorhoids) and periumbilical caput

medusa are high yield THE BUTT THE GUT and THE CAPUT

Abdominal Development

Liver

Ij1f

II wall b

oh liN ~ VltJrti n be- bull

Pancreas

Secondary Retroperitonealization e I~tl r 1 a v-mtrai m ellter

Rotations of the Gut I i Ij (lIl1UtIJ f~ l r tilt

()l td 10 me l-ft and he v

--~--- -~ -~-~

i

I AolaijonjoI~guf I

STOMACH BED (IDENTIFY IN ADAMS)- the structures posterior to the ommental bursa which

support the stomach in the supine position

Abdomnal JQrUI

Splnic vein

OmQ-oul tv~ ) O(s(Jroa)

Lojt(r o m nturrt (hpJtodu o d~n31 Hid

Gadrl)SplerH (g3stroll~nal) IIgam~nt

hiad h~~atogrtricent IIQdmiddotcrt~)

Lt Dome of Diaphragm (why left Look this up in Adams)

Spleen (What is the blood supply)

Left Kidney (What is the blood supply - AND how is it different from the R kidney)

Suprarenal Gland (What is the Arterial AND Venous Blood supply - how are they different)

Pancreas (How does supply differ from Head to Tail What is the SMA Relationship)

Transverse Mesocolon

liver - ADAMSWET - Make sure you look at the liver in wet lab

Left triangular nl1am~nt

ComoaDj ligamnt

Erophg~1 impre$ioo

Hepatio veins

In1erior -ifena middotr3)Ia

Fibrous appendix o-t

live

impr~j on

Heprorendl p~rtion of Q)(Qllary ligament

Righllri~n9ul r 1I~met

(Common) bile quol

Gr)mmCtr~ hepatic dlJct

Ccentic duct

Duodenal impression

GaJdate p-fr)~S

Hepatic artgtrl prop-f iiiiila - Faloiform ligament

_ - shy Round ligamen liver

~--F-- CoJio imprgt-ssi-on

Prta heptis

Identify the lobes impressions and embryonic remnants associated with the liver

Caudate Lobe Quadrate Lobe Right Lobe Left Lobe Round ligament Falciform Ligament

Ligamentum Venosum (what is its fxn in embryonic life) Hepatic Veins (NOT PART OF THE

PORTAL TRIAD) IVC PORTAL TRIAD - Contents relationship cross section etc Know the

Galbladder relationship to the lobes of the liver

Biliary Duct System - Make sure you understand the sequence of these structures - BE ABLE TO

DRAW A FLOW CHART

TPVd i

t

I t

1 __ Cm-(r

patk GlJet

I

J

Clinical = JAUNDICE is caused by anything that prevents delivery of bile to intestine Tumor of the

head of the pancreas Stones etc Patient will have pale stools and yellowish colored mucus

membranes

Clinical- Any scenario that tells you the patient has BILLOUS VOMIT means that the obstruction to

the flow of digestive contents is after the Ampulla of Vater (Site of Entry of Billiary system to the

duodenum) - ie Duodenal Atresia

Spleen -located posterior to the mid axillary line between ribs 9 and 11 Make sure you know that

the 10th rib is the main axis of the spleen and this organ is susceptible to injury (stab wound errant

thoracoce ntesis etc)

The spleen is derived from mesodermal cells - NOT THE GUT TUBE

The spleen rests on the left colic flexure associates with the tail of the pancreas Know the

structures entering the Hilum of the spleen

Sh rt O~-t~ic 1 0(0 10 rtiltSPIric Iloa nt

(cut)

Peritoneum - similar concept to Pleura - think of a fist in a balloon

Visceral Peritoneum - Layer of balloon touching your fist

Parietal Peritoneum - Layer of balloon not touching your fist

Your fist represents the organ your wrist is the hilum and your arm contains the blood supply

entering the organ

Appreciate that there will never be organs in the peritoneal cavity - rather these organs invaginate

the cavity Kaplan videos

RULES OF NOMENCLATUREshy

1 Organ completely surrounded by peritoneum - peritoneal organ

2 Organ partially surrounded by peritoneum- Retroperitoneal

3 Peritoneum surrounding peritoneal organ is VISCERAL peritoneum

4 Peritoneum surrounding retroperitoneal organ is PARIETAL peritoneum

5 Peritoneum connecting visceral to parietal is called messentary 2 messentaries in the

gut Dorsal (to the gut tube) and ventral (to the gut tube) messentary

Aorta is in Retro peritoneal position - but blood must reach peritoneal position - vessels travel through

messentary All peritoneal organs will have blood supply reaching through messentary

-Mesentery is a 2 layer peritoneum with a neurovascular communication between body wall and organ

- Ligament connects one organ with another or to the abdominal wall (Ommentum = ligament)

lesser Ommentum (attach lesser curvature of stomach and duodenum to liver) =Hepatoduodenal

Ligament and Hepatogastric Ligament

Has a Superior and Inferior Recess (Accumulation of Fluid in Ascites)

Communicates with the greater sac through the epiplic foramen (what structures pass through

this foramen)

Boundaries - you must be able to visualize this

o Anterior - stomach

o Posterior - parietal peritoneum pancreas

o Superior - superior recess (bw diaphragm and coronary ligament)

o Inferior -Inferior recess (bw layers or greater momentum

Greater Ommentum (attach greater curvature of stomach) Gastrophrenic ligament Gastrosplenic

ligament gastrocolic ligament

The greater omentum is the largest peritoneal fold It consists of a double sheet of peritoneum folded on itself so that it is made up of four layers The two layers which descend from the greater curvature of the stomach and commencement of the duodenum pass in front of the small intestines sometimes as low down as the pelvis they then turn upon themselves and ascend again as far as the transverse colon where they separate and enclose that part of the intestine

ABDOMINAL PAIN

Parietal Peritoneum - supplied by same vasculature lymphatics and nerves supplying body wall it

lines and diaphragm Sensitive to pain pressure heat cold well localized

Visceral Peritoneum - supplied by same vasculature lymphatics and somatic nerve of organ it covers

Insensitive to touch heat cold and laceration - referred to dermatome of spinal ganglia providing

sensory fibers Where does appendicitis refer to

Foregut pain - epigastric area (ie - cholycystitis)

Midgut pain - periumbilical area (ie - appendicitis)

Hindgut Pain - suprapubic area (ie - diverticulitis)

Extra ImagesConcepts

ll~_____-

FalifCtrm ligament oind r~ud ligamet f Ilver

Blood from splenio gastriC and inferiof rne$e-rteri v~ins

Ca-I tributaries

Lett gastrio Ifein

Posterior superior pan~reatioodul)denal vaihS

Lott gamo-om~nlal (9aropip lomiddotic) -in

Poq_~ tjol imerl-9-r panCJertlcorllJod-nal veiopound --amp----I- - ~J Right grtr~-omntal

Anwrior interi (gartroepiploic) Jjn

pan euaii cod vl)denal veins middot Inf~Ji (t r mesentric vein

Miqdle (olic vein

Right cl)licvein Sigmoid and rectosigml)id (ei ns

IhH)Collc(~io

--- Mi~dl laquooLJl gtjrltgt

PoM ca vl1 illasto)moses -----shyampoptoageal 2 Paraumbilie-lt11 Inferi or Fectal vei ns

3 Recial 4 REuoperHonea1

Know how the Portal vein is formed I 4 sites of portal caval anastamoses and 1 clinical shunt

Col li t ltt-~ otTl~tI ~nj pc~ 1lt1 turJoG

Ltf 14i1 tImiddot~ artoftl9 on tj phtAt$

L-oftqf 4t t~r 1=laquoIran d 1 bull shy~p l ci rj o fOOOts

Nerves follow the arteries - appreciate the splanchnic nervous system I

Uet~ric branch of left ~nal art

Ureterie branch of righi renal artelY

Left Zld lumbar in and co mlTlunication to as)erdin9 lumbar l(~in Hi ~ht tEZ1~~t~ t3r j t itn ~ nJ l1t- rlnd lfe i r1

Inferior me5nteri~ artery

Notice that the right testicular vein drains directly into the IVC and the right testicular artery drains

directly into the aorta However the left testicular vein drains into the L renal vein at a right angleshy

reason left testicle is lower and more susceptible to varicocele (bag of worms)

Also notice that the left renal vein has a longer course because the IVC is on the right side whereas

the right renal artery has a longer course because the aorta is on the left side

Appreciate the anterior to posterior relationship of structures in the hilum of the kidney - VAP - Vein

Artery Renal Pelvis (Ureter)

11____ __ L_ L_ n VJ __ _ _ t_L I I_ _ L __ L_ I -pound1 bull LI_~-I ____

Posterior View of Head of Pancreas in ( of Duodenum

Celiao hunk

Co mmon ~L~jJth art~ry

GastNduQdonal artrf (partilly in phantn)

P1)Sterior $Up~Jior panCflaticuduodfmal art~r~t

(Co mm on) bile duct

middot~1t~~t-1l---~-~- Right gshomiddotomental (gastoe plp lolc) 3rte (phantomost)

Grener paocre atic art-ry

1n1~rjor pancr-iatlc artery

Jtrifll supejo r pal)oreailcento)dJodenal artr1 (phantom)

Anastomotlo branch

POostetlor bJanch of jo f~ri of pan-reatir(lduodensl drttnj

Anterio r branch of i flferior palcreati~)duodenal art~(phan1om)

Notice the extensive blood supply to the pancreas and duodenum via the branches of the celiac trunk

Notice collateral supply from SMA branches - makes sense bc this is the jxn of foregutmidgut

Identify the vessels in this arteriogram

Hiltid i)f N~ck oi B)dvof Tail 01 pa nereas pan cent~as P-nmiddot-reas panCtCas

I nferie v~na cava

jHept1iic p(lrlai v~in

Port1 tnd H~pti lt a ftH prol

Comm on) bll duct

Ouodtnum

~ft colic (sio)Atta~ hmtrlt jt~xJr-ofha~elSe

muo(IIQn

Right ~lIc (h~j)tic)

il~gtture

In1triol m~oten lIein (rttr op~ritoMdO

SlJp efl or mes~n~fiC amrV and lipln

KNOW YOUR NEIGHBORHOOD

Questions

vVhiJh structure supplied by a bnmdlof the cclia( artery is not derivcd from foregut LemCJUCrITI

(A) Head of the pancte-a5

CD) Pyloric duolenum

Cystkduct

( Liver hepatocyt~~

~F) Body of the spleen

An infant presents with an omrhaJucele at birth -hi oJ the [oHm illg applies to his cM1-dition

(A) It is 31so seen ill p4titnts with aganghonic megacolon

(11) ft reuirs from a fal1ure of resorption of theviteUine d let

(C) It results from herniation at the-site of regression of the right umbilk vein

DJ It is caustd by faihtrc of recanalization of the midgut part of the duodenum

~ It ill camioo by a failuIt vf the midgul to return to the abGQminal uity after herniashytion in-n the urnbilk s l stalk

Ot er than the spleen occlusion Cif the spit-Ilk artery at its odgin wm most likely affect die blood supply to jllch st cnud

(A) Jejunum

(B) Body of th pal1~lltas

(C) LeSStT Cllmiddotlaturc of tl )toma-ch

(D Duodenum dista to the entrance of the Ornmou bile duct

E Fundus of the stomach

A 38-yeu-old batL~er with a history of heartburn suddenly experiences excluciating pain in the (plgastric region of th~ abdomeu SurgCry is perf~rme immediard y upon admisshysion to the 1IlcrgCJliy tuomh~re i~ evidence uf a ruptured ulcer in the posterior waU of the stomach Vhere will a surgeon first fi nd the stomach contenlSf

A) Greater p4ritoneal sac

rB) Cul~de-s~c of Douglas (--

C Omental bursa ~

--D) Paracolic gutter

rEj Between -he panttal perimltum and the posterior body wal1

At birth an infant presents with a st()ma~ rb~tbas~njJled jfltotb~diaplfagru 1A1ltre is the defect thatresulied iiitJle heini~t()n shy~tsophagealbiatus

7 - rH-- Hiatus for the inferior vena cava

( Pleuroperitoneal membrane -(0) Septum transvcrsum

(E) Right Crlt~

An infant born with DOVv7l syndrome presents with bili()u~ vomiting Ahat congenital defect does the infant have

(A) Pyloric stenosis

(B) Meckel diverticulum C) Ornphaloce1e

(D) Gastroschisis

( ~ ) Duodenal atresia y A patient with cirrhosis of the liver presents with ~ bacalvaricestnlreased retrograde pressure in which veins caused the varices

(A) Paraumuilical

(B) Splenic

(ct AzygltJus

(15))G~trk ( (-F) Superior mesemeric

A htaltby 3-year~old male patient experiences a hernial sa protruding from the anterior abdominal wall about halfway between me anterior superior ilia spine and the pubk tuberde Pulsations of al1 artery are palpated medial to the protrusion site through the abdominal walL Which layer of the anterior abdominal wall will first be traversed by the

1hctma

fA) Rectus sheath (B) External oblique aponeurosis

(C) Inguinal ligament

lD) Transversalis fusda

(E) Cremasteric fa~cia

After 5urgi(aj ffpair of a hernia the patient tXperienccs mtmlgtness in the skin on the anteshyrior aspect of the S(Totum_ Vhaf nerve may have been lesioned during thehemiorrhaphy

(A) Femoral

(B) Obturator

(C) Ilioinguinal

(D) lliohypogastrk

(E) Pudendal

A 23~year-LJld female secretary il1 good health ~-uddcn1) doubles over with pain in the a ea of the 1JmbRicu$ Sbe feels vartn and ltneasy and has no appetite That night the pain seems to have mQved to the tower right abdominal regjol1 and she calls her family doctor who then arranges for an ambulance to pk-k her up and take her to the hospitaL Wh ell ntn~ perceived in the area of the urnbilirus most Hkely carried lhe pairfu I sensations into the eNS

tA) Vagus nerves I~

V B)

) Lessersplanchnk nerves

tC) Pudendal nerves

(D) lIiohpogastrk nerves

(E) Greater splam ic l erves

A CT reveals carcinoma in the bOod of the ancreas Vhich blood vessel trut ourses ----~- - -bull ------ --shy

immediately poftterior to the body ofthe pancreas is the m~t likely to be oompressed

(A) Splenk artery

(B) Abdominal aorta (C) Portal vein

(1) Splenic vein

(E) Renal vein

A patient has a penrln1l1ng uker of the posterior wall ot the br~l part ot the (lUooenmn llkh blood vessel is subject to erosion

(A) Common hepatic artery

(B) Gastroouodenal artery

(C) Proper hevatic artery

(D) Celiac artery

(E) Anterior inferior 11amrelltlcoduodcnal attery

Your patient has been diagnosed -ith a carcinoma locallted to the head and l~e(k of the pancreas Another clinical sign would be

A esophageal varices

(8) hemorrhoids

C) a caput medusa

(D) increased pra Teuro n th~ hepatic veins

(E) enlarged right supra lavkular lymph nodes

Wltkh of the foUowing structures develops in the ventral mesentery

(A) Spleen

(B) Jeiunum (C) Head of1ht pancreas (D) Transverse colon (E) Stomach

ti l Uw ~ littwin~ f( S-t lil oai Imdge ~ hi(h or tbt la~)d J truetur tgt liJ llntn nl) he hl p UC iJd [IIi ell

c o

A) drains Ie tht infCrior a La aI

R t middot~nfl0 ~ill to th~ lunlgtn of h i dtlndCrlllfH

(e) m t bull JiJattd on tl l J n T ~H

D ) sup Lc O VSlt I Hlid bhtu l 1 li - -I un oid

( ) U~tpli(t tr j middottUh~ 1 v(( b~nt rfK n1ilc~Zm

ANSWERS AND EXPLANATIONS

Answer E The spleen is t hlttnopodicand lymph organ demlted from mesoderm

Answ~ R Al1 tlmphalocele is caused by it failure of the nlidgut to return to the ahdomir nat cavity after herniation into the umbiliau Stalk Choices Aand D maybe seen in infants with Down syndrome choice D ~s the specific CBuse ofduudcnal JtiCSitt Choice C is (ile cause of gclstrosbisis and Choice B nsults iu a Meurolktldivertku1-tlB

Answer B The fundus ofthe stomach is suppHed by soort gastric brunches of the splenic altery The splenic artery supplies the body and tail of the pancreas part of the greater curvature of the sttmla(h and the spleen Te jejunum part of the head of the pancreas and tht~ duodenum distal to the entrance of the commOll bile duct are supplied by the superior mesenterk artery clll~l ~be less r ctlt1ature cmd the pylQric antrum are supplied by the right and lei gastric art(ries

AnSWftt C Tbeomental bursa or lesser ~ritoneaj sac lies direcdy posterior to the proxshyimal part of the duodeTtlm and the stomach and would be the first site where stomach contents ~Ott1d be fpoundluncL

Answer C A defect in a llleuropcritoneal membrane (uswlly the left) is the typical site of i1 cc-ngenitlI diilphragluatic hemia llere the membr4ne fails to dose ()pound( of the perishycCirdiopcritulleal canals

Answer E DuoJenal atresia and aganglionic megacoion are congwitaI defects S~Il in patients with Dowmiddotnsyndrome

Answer D RulaTgemt~llt of and retrograde flow in g~lstrk vel_ns in particlJl~r the kft gas~ tricveins dilates the capillary bed in rhe wall of the esophagus in (ases of porta yper~

tension Blood flow would increase in and dilampte tribntarkgts of the (lZygOUS vein on the other side of the capiUary bed but flow in this vein is in the typical direction t()ward the superior vena cava Paraumbiii(ltU vein eilgorgement contributes to a caput medusH Splenic ~nlargement might prc~nt with 5plcnonlegaly and balt-kflow in to tlu superior m~~ntclic vein occurs but is asymptomatic

Answer D The patient hagt an indirect inguinal hernia whi~h emerges from the antt-rior abdominal wall through the deep inguinltilling Theeep ring is a fault in the transv~rshysaUs fascia this I~yer wiIJ be penetrated first by the hernia

An~Wer C The ilioinguinal nenc which provides sens~llion to the lnedlal thigh ltmclanteshytior SClotunl pass~lt th rough the 5uperfh_ial inguinal ring ind $subject to inj i1T) becaus-e

it is in the operatitm Held of the erniorrhapny

Auswer B The leMHr splanchnic nerves are sympathdic nerVlts that carry viscera l sensashytlltgtrogt ftom illtllt1m~d ()J stietched gust (itinteitinal ~tructures (in this case the pprndix) into tnt eNS Lesser splanchnic ntTYcsarisc from thmiddot T9--T12 spinal cord segments lt1nd provide sympathetic innenation tD rnidgut siruc1ures whiCh include CLe app~JldD Viscera] Pain arising from affecLed Inidgut ampt 1C1ure is referred over the same dl- matorne~ of spinal segrnertts v-hich provide the sympathetic Innervation n this G1SC of appendicitis the invohen~n t of the ltire) of t e unlhHku indud s the T 10 dermatome

Answer B Of the five choices onty the dscending olon is retroperiton~al aldwould be a lik ~ ( choice to be seen immediately a(~jilcent to t11e posterior abdominal middotn~L

Amwen D The SpltftlC ~-ein ourses posterior to the body of the panneas m its way tt drain into the superior mCSfttltlri( vein

Answcr B TILt glstrodllolticnal artery 1 direct hIamh of the comrootl hepatic artery courses immediately pt))iwri() to the duodenum and is slbject to erosion

Answer B Carcinoma of th pan middott3S in the 1tilt1 may compreampgt the portltil vein at irs orishygill The poTtai vcin is fomled when the splenic vein jQiaswith tfie superior meStllt eric vein The inferiot mesenteric vein joins the ~plenjc vein just priOT to tlli~ point at which the splenic joins the superior Jlleit1ltcri( vein Increescd venous presslu in the inferior mesenteric vein is a cause of emo hoid~

Answer C The- velltral pancreas wilich forms most of the head of the p ~ncr as develops in the ventral mes(ntery as antutgrowth of the hepatic diverticulum Th~ hepatic divershyticulull induding the biIJary appa~atus develops in tbe ventral mesentery of the foregut

Answer~ A The superior mesenteric ~in joins with the spienkvein to form the hepatic portal vciu

Answer D The structure at gttlK is the proper hepatic artery~ whkh suppUesoxygenated b middotood to the liver

MAKE SURE YOU KNOW the diff bw Rectus Sheath above and below the arcuate line

ABOVE

Aponeurosis of xiiltmal obllque musclo

Extemll f)biquw musde

Reotln ilbdomlnls musole S~in

Internal 9bliquQ mY~QI

AponeUfOsi$ of hJH$V~~S Lir9a a lb lbdolTlin~ musolo Tri OJV6 rUi

atldomlnis mUS(loe

Sub cutanlilous tiue (tatty ye r)

BElOW

A POrl lJfosis 01 etemal oblique muscle

Aponeul~)sis 01 Internal oblique mU$cl~

Anteriol lay~ of r~ltdus st~ath EXttom1 oblique rnu$cll

Rectus Jbdominis muscle Intoernal Aponeurc-sis of tra~fersU$ oblique muscle-

at-domlnis muscentl ~ Skio

Tra nsvitSus abdomioLs ml)ZClt

TralSVersaHs fascia Medial umQil iegtt1 1i9Jment -and folj

Uldchus Peritoneum (ir median Umbilj~al Suboutane ous

Extraprftone 11ascia

Ymbilimiddot~1 fold)

preu9poundiea1 fascia

tissue (fatty 4nd m~mbr3n(iUS layers)

o Above the arcuate line (A horizontal line 13 of the distance bw the umbilicus and the

pubic symphysis) -10 Aponeurosis divides into an AntPost Laminae

o The Ant Laminae joins EO and Post Laminae joins Trans Abdominis = Ant and Post

RECTUS SHEATH respectively

o BElOW the arcuate line - all 3 aponeurosis join ANTERIOR to rectus muscle to meet its

counterpart in the midline (linea Alba)

o Take away Msg - The abdomen is devoid of a posterior rectus sheath below the

arcuate line and is therefore more vulnerable to herniasinjuries

Question - A physician makes a deep incision in the patients midline immediately superior to

the pubic symphysis which of the following layers is his knife least likely to pass

Rectus Abdominis External Oblique Ant Rectus Sheath Posterior Rectus Sheath All of the

Above

Answer - All of the above None of the other answer choices are midline structures -LINEA

ALBA

Linea Alba has very poor blood supply - doesnt heal well after surgery Therefore this is a

common site for incisional hernias

a Spleen b Transverse colon c Descending colon d Stomach e Pleura

17 Meckels diverticulum is normally found 2 feet proximal from the

a Pyloric sphincter b Lower esophageal sphincter c Ileo-cecal valve d Middle valve of Huston e Anal valve

18 Ulcer in the posterior wall of the first part of the duodenum would erode ___ artery and would cause bleeding

a Left gastric b Right gastric c Hepatic artery proper d Gastroduodenal artery e Middle colic artery

19 An inflamed appendix is identified by a surgeon on the operation table by noting

a The appendicies epiploicae b The convergence of tenia c The artery of Drummond d The mesocolon e The mesosalphinx

20 The nerve which emerges through the psoas major is

a Femoral b Ilio-inguinal c Ilio-hypogastric d Pudendal e Subcostal

21 The right gonadal vein drains into the

a Azygos b Hemiazygos c Inferior Vena Cava d Right renal vein e Left renal vein

22 The hepatocytes in the liver is derived from

a Ectoderm b Endoderm c Mesoderm

d Neural ectoderm

23 Abscess in the lumbar vertebrae due to tuberculosis would spread to the adjacent muscle which is

a Psoas Major b Iliacus c Quadratus lumborum d Tranversus Abdominis

24 The anterior wall of the inguinal canal is formed by

a External oblique and transverses abdominis b External oblique and fascia transversalis c Internal oblique and external oblique d Internal oblique and transverses abdominis e Fascia transversalis and peritoneum

Meckels diverticulum is a result of which of the following developmental abnormalities shy

A Failure of the vitelline duct to close

B Failure of the herniated intestinal loop to retract into the abdomen

C Failure of the urachus to close

D Failure of the midgut to rotate

E Failure of the hepatic duct to close

Explanation

Meckels diverticulum is a result of the persistence of the proximal part of the vitelline duct This

diverticulum is usually found about 2 feet proximal to the ileocecal junction and is usually about 2 inches

long It is present in about 2 of the popUlation It may be the site of ectopic pancreatic tissue or gastric

mucosa and may develop inflammatory processes and ulcerations Acute Meckels diverticulitis

simulates appendicitis

Which of the following veins carries blood from the esophagus to the portal vein The

A right gastric vein

B left gastric vein c splenic vein D azygos vein

E left gastroepiploic vein

Explanation

The left gastric vein a direct branch of the portal vein drains blood from the lesser curvature of the

stomach and the inferior portion of the esophagus Because branches of the portal vein do not have

valves blood can flow in a retrograde path when there is an obstruction to flow through the portal system or liveL Rlooci Cln then flow from the nortl] vein thr()1Ph the left PRstric vein to the esonhlPlIS lno

through venous communications within the submucosa of the esophagus to esophageal veins that drain

into the azygos vein The increase in blood flow through the esophageal submucosal veins results in esophageal varices

On the posterior wall of the abdomen the celiac ganglion A contains cell bodies of postganglionic parasympathetic neurons B is synapsed upon by neurons in the posterior vagal trunk C is synapsed upon by neurons in the greater splanchnic nerve D contains sensory cell bodies of lumbar spinal nerves E contains cell bodies of neurons that cause an increase in the rate of peristasis

Explanation The celiac ganglion is one of the preaortic ganglia of the sympathetic nervous system It contains cell bodies of postganglionic sympathetic neurons The sympathetic splanchnic nerves contain preganglionic sympathetic neurons that pass through the sympathetic chain without synapsing These splanchnic nerves go to the preaortic ganglia to synapse The greater splanchnic nerve contains preganglionic neurons from spinal cord segments T5-T9 This nerve synapses in the celiac ganglion The nerve fibers in the vagal trunks are preganglionic parasympathetic fibers that go to the walls of the organs that they will innervate and synapse on postganglionic parasympathetic neurons in the walls of those organs Cell bodies of sensory neurons in the abdomen are found in the dorsal root ganglia or the sensory ganglia of the vagus nerve Sympathetic innervation decreases the rate of peristalsis parasympathetic innervation increases the rate of peristalsis

Which of the following pairs of arteries will allow blood to bypass an occlusion of the celiac trunk

A Left gastric artery-right gastric artery

B Left gastroepiploic artery-right gastroepiploic artery

C Superior pancreaticoduodenal artery-inferior pancreaticoduodenal artery

D Splenic artery-common hepatic artery

E Left gastric artery - proper hepatic artery

Explanation The anastoOlosis of a branch of the celiac trunk and a branch of the superior mesenteric artery will

provide collateral circulation around an occlusion of the celiac trunk Each of the other choices pair

branches of the celiac trunk therefore these will not provide collateral flow around the obstruction of the

celiac trunk The left gastric splenic and common hepatic arteries are direct branches of the celiac trunk

The right gastric artery is a branch of the proper hepatic artery which is a branch of the common hepatic artery The left gastroepiploic artery is a branch of the splenic artery The right gastroepiploic artery is a

branch of the gastroduodenal artery whlch is a branch of the common hepatic artery

Which of the following organs has appendices epiploica The

A sigmoid colon

Bjejunum

C duodenum

D stomach E esophagus

Explanation Appendices epiploica are characteristic of the colon Appendices epiploica are subserosal accumulations

of fat None of the organs of the gastrointestinal tract has appendices epiploica except the colon

Page 16: Chirag's Abdomen Review

Chirags Abdomen Review - Part 2

Understanding Embryo makes learning blood supply EASY

I I

I t

~ -

)

Table l1r-~ L Adult SUmiddotuctu~SDrj~l Froln Each of he Three Dhisions of be Pringttive GUl Tube t-middot-----middotmiddotmiddotmiddot-

Foregu(

I_ (Celiac Trunk)

Ir-slt-gtphgus

S101na(b

I h -= LiJ~r

Pancre=l S

bull 1 i Biliary apparntu5

Gall bladdshy

i Pha11~Cal pltgtuchcs

LullSS-I

Mjig ---- bull __ _- ----n--duct----~---middot-------l--n

(Superior Jldesen1eric Artery)_-1I-(I_~__ middot __ O-=-)_in_middoto_r_M_e_se_n_t_e_r_i_c_An__

Uuodenu rn 2nd_ 3 lt141h V4Tt

Jejunun-~

nc-un]

tCCUJ11

AppltgtndLX

Transver5e -o1on (p~oxiln1l1 ~O Tbird)

bull__hytgtid~ _ _ ______ L _ __

Tr-dn~llt~se colon (diStul h lTd) I

)

i

Aa ca-nal -( uppeT patt) i

I I

_____ __ _ _ _ ___ - - rhe~ a(t clcriVOkt iV(5 opound~lt prbn1rC ~ nlQC blft TlI)( 134tof r~ tIonoinf~ i 1 ~l l1rd c- P Cle

Now Lets see how much youve learned

Questions

1) A pt receives a general anesthetic in preparation for a c~t~~my A right subcostal incision is made which begins near the xyphoid process runs along and immediately beneath the costal margin to an anterior axillary line and transects the rectus abdominus muscle and rectus sheath At the level of the transpyloric plane the anterior wall of the

-~~-~=--- _eco---shysheath of the rectus abdominus muscle receives contributions from which of the following

a Aponeuroses of the in~ande~tef-Ilal o~ues

b Aponeuroses of the transversus abdominis and internal oblique muscles c Aponeuroses of the transversus abdominis and internal and external oblique

muscles d Transversalis fascia e Transversalis fascia and aponeurosis of the transversus abdominus muscle

A

2) The lat~raJJJ11QjJt~gLfgJlLoneach side of the inner surface of the anterior abdominal wall is created by which of the following structures

K Falx inguinalis (~) Inferior epigastric a

c Lateral border of the rectus sheath d Obliterated umbilical a e Urachus

B

3) A man the victim of several knife wounds to the abdomen during a brawl at the Lobster Shack subsequently developed a direct inguinal hernY Damage to which of the following nerves is most likely responsible for the predisposing weakness of the abdominal wall

~ Genitofemoral nerve ( b) Ilioinguinal nerve ~-t Tenth intercostal nerve

d Subcostal nerve e Pelvic splanchnic nerve

B

4) Which of the following statements concerning a direct inguinal hernia is correct a It is the most common type of abdominal hernia b It transverses the entire length of the inguinal canal c It contains all3 fascia layers of the spermatic cord d It exits the inguinal canal via the superficial ingeJinal ring e It protrudes through H~acb strJg e

~(

1fltbS w E

tl

5) The conjoint tendon is

a Important in preventing indirect inguinal hernias b The fused aponeurotic layers of internal abdominal oblique and transversus

abdominus muscles c Posterior to the deep inguinal ring

d Medial fibers of the inguinal ligament

B

6) A 25 year old male is brought in to the ER after being involved in a car accident in which he received a crushed internal injury in his abdomen Examination reveals a lesion of parasympathetic fibers in the vagJsnerve which interferes with glandular secretory or

smooth muscle functions in which of the foliowingorgans a Bladder b Transverse coloiW c Descending colOO d Prostrate gland e Rectum

B

7) The spermatic cord includes all of the following contents except a Il ioinguinal nerve b Pampin iform plexus of veins c Vas deferens d Genitofemoral nerve

A

8 Which abdominal structure gives rise to the internal spermatic fascia (muscle) following the descent of testes in development

a External abdominal oblique aponeurosis b Transversalis fascia c Transversus abdominis muscle d Peritoneum e Internal abdominal oblique

B

9 Which abdominal structure gives rise to the tunica vaginalis fotlowing the descent of testes during development shy

a External abdominal oblique aponeurosis b Transversalis fascia c Transversus abdominis muscle d Peritoneum e Internal abdominal oblique

D

10) The lesser omentum is a peritoneal fold which is su bdivided into the a Hepatogastric and gastrosplenic ligaments b Hepatoduodenal and gastroomentalligaments c Hepatoduodenal and gastrosplenic ligaments d Hepatogastric and hepatoduoden9-jrj igaments

D

11) A posteriorly perforating ulcer in the pyloric antrum of the stomach is most likely to produce initiallocalized peritonitis or abcess formation in which ofthS fQllowing

a Great-sac - -- -

b Paracolic recess

c Omental bursa

d Right subphrenic space

c

The inferior mesenteric artery arises from the abdominal aorta ilm_ediill~y_J-Qs1eriQLto which of the foowing org~ns A-F~t~filie duodenum B Head of the pan~eis C Neck of the pandeas

D Second part of the duodenum

E Third part of the duooenum_shylaquoshy

shy

The correct answer is E The inferior mesenteric artery arises from the anterior surface of the aorta at the level of the third lumbar vertebra The third part of the duodenum crosses the midline at the level of the third lumbar vertebra and passes anterior to the aorta at the origin of the inferior mesenteric artery The

first part of the duodenum (choice A) lies horizontally to the right of the midline at the level of the first

lumbar vertebra The head of the pancreas (choice B) is to the right of the midline and extends from the

level of the first lumbar vertebra to the third lumbar vertebra It lies within the concavity of the

duodenum The neck of the pancreas (choice C) lies in the midline at the level of the first lumbar

vertebra It lies on the anterior surface of the aorta at the origin of the superior mesenteric artery The second part of the duodenum (choice D) lies vertically to the right of the midline and extends from the

level of the first lumbar vertebra to the level of the third lumbar vertebra

The left adrenaLvein drains directly into which of the following veins A Hemiazygos vein

B Inferior vena cavaee C Left renal veiri -

D Splenic vein

E Superior mesenteric vein

a

The correct answer is C The left adrenal vein and the left gonadal vein (either testicular or ovarian) drain into the left renal vein TheTeft renal vein t~ains intothe- inferior vena cava In contrast the right

adrenal ~~inandnght gonadal veindrai~ gLr~ctJy iQtoJhe iilferiQ[ Vencava -- -

ThehemTazygoS7ein- (~h-~i-~ A)~~c~i~es the venous drainage from the body wall on the left side of the

thorax and abdomen No visceral organs drain directly to the azygos or hemiazygos veins The inferior vena cava (choice B) receives the direct venous drainage from the right adrenal vein but not

the left adrenal vein Remember the inferior vena cava is on the right side of the abdomen The splenic

vein (choice D) receives the venous drainage from the spleen and part of the pancreas and stomach The splenic vein is part of the portal venous system

The superior mesenteric vein (choice E) receives venous drainage from much of the intestinal tract It is part of the portal venous system and joins with the splenic vein to form the portal vein

A 43-year-old man presents complaining of pain in the groin On examination his physician palpates a

bulge in the region of the superficial inguinal ring which he diagnoses as a direct inguinal hernia The hernial sac most likely

A is covered by all three layers of the spennatic fascia B passes medial to the inferior epi gastric artery

C passes medial to the lateral border of the rectus abdominis muscle

D passes posterior to the inguinal ligament E passes through the deep inguinal ring

The correct answer is B Direct inguinal hernias enter the inguinal canal by tearing through the posterior

wall of that structure The typical location for this type of hernia is through the inguinal triangle bounded

laterally by the inferior epigastric artery medially by the lateral border of the rectus abdominis and

inferiorly by the inguinal ligament Direct inguinal hernias pass medial to the inferior epigastric artery

whereas indirect inguinal hernias pass lateral to the inferior epigastric artery because the deep inguinal

ring is lateral to the artery Indirect inguinal hernias are covered by all three layers of the spermatic fascia (choice A) Direct inguinal hernias are covered by fewer than all three layers because the direct inguinal

hernia tears through one or more layers of fascia as it emerges though the abdominal wall The lateral

border of the rectus abdominis muscle (choice C) forms the medial border of the inguinal triangle All

inguinal hernias pass lateral to the rectus abdominis Femoral hernias pass posterior to the inguinal ligament (choice D) Inguinal hernias emerge through the superficial inguinal ring which is superior to the inguinal ligament Inguinal hernias that descend below the inguinal ligament pass anterior to the

ligament Indirect inguinal hernias pass through the deep inguinal ring (choice H) direct inguinal hernias

do not Both types of inguinal hernias pass through the superficial inguinal ring

During a gastric resection in a patient with stomach cancer a surgeon wants to remove the lesser

omentum because of tumor extension into it Which of the following structures lie in the free edge of the

l~~g omentum and consequently must be dissected out in order to be preserved

A Common bile duct cystic duct and hepatic artery 6

B Cystic duct hepatic artery and hepatic vein

e Hepatic vein and cystic duct

Portal vein common bile duct and hepatic artery

E Portal vein hepatic artery and hepatic vein

The correct answer is D The free edge of the lesser omentum contains three important structures the

common bile duct the hepatic artery and the portal vein Nei ther the cystic duct (choices A B and C) nor the hepatic vein (choices B C and E) lies in the free

edge of the lesser omentum

A 55-year-old male patient with chronic liver disease has portal hypertension To relieve the pressure in the portal system a porto-caval shunt is performed Which of the following veins may by anastomosed to

accomplish this porto-caval shunt A Left renal vein-left testicular veingt

B Right renal vein-right suprarenal vein I shy

e Splenic vein -left renal vein J

D Superior mesenteric vein-inferior mesenteric vein E Superior mesenteric vein-splenic vein

The correct answer is C The splenic vein drains directly into the portal vein The left renal vein drains

directly into the inferior vena cava Anastomosis of these veins would allow blood from the portal vein to

drain retrograde though the splenic vein into the renal vein and then into the inferior vena cava The left

renal vein (choice A) drains directly into the inferior vena cava The left testicular vein drains directly into

the left renal vein Thus these veins are already in communication and neither vein is part of the portal venous system The right renal vein (choice B) drains directly into the inferior vena cava The right

suprarenal vein also drains directly into the inferior vena cava Thus neither vein is part of the portal

venous system The superior mesenteric vein (choice D) drains directly into the portal vein The inferior

mesenteric vein drains into the splenic vein which then drains into the portal vein Thus neither vein is

part of the caval venous system The superior mesenteric vein (choice E) drains directly into the portal

vein The splenic vein also drains directly into the portal vein Thus neither vein is part of the caval

venous system

A 12 year old boy has fever vomiting and para-umbilical pain After examining the patient the doctor

makes an initial diagnosis of appendicitis Appendicular pain which is initially referred to the umbilicus goes to the dorsal root ganglion of

a TI b TI2 c L1 d T7

(e I TIO

A 59-year-old male undergoes a neurological examination which reveals that when the abdominal wall is

stroked the muscles of the abdominal wall of the side of the body stimulated failed to contract Other

neurological tests appeared normal The likely region affected includes

a CI - C5 spinal segments b C6 - TI c T2-TI ~T8-T12

e Ll- L5

The surgery done to relive portal hypertension is done by connecting two veins Which of the following veins would be suitable for connection

a Inferior vena cava and portal vein b Superior vena cava and portal vein c Splenic vein and right renal vein d Splenic vein and left renal vein e Superior mesenteric vein and Inferior vena cava

A mother brings her 3-week-old infant to the pediatric clinic reporting a new scrotal bulge that she found -~-

while changing a diaper yesterday The infant is afebrile Physical examination reveals a palpable mass in

the scrotum while in the standing position resolution of the mass in the supine position and no

transillumination of the scrotal sac What is the most likely diagnOSiS

a Cryptorchidism b Direct inguinal hernia c Hydrocele d Indirect inguinal hernia ~ e varicocele

The Vagal trunks enter the abdomen by passing through which of the following openings in the

diaphragm

a Right crus b Esophageal hiatus ~ c Vena caval hiatus d Aortic hiatus e Left crus

2 The anterior boundary of the epiploic foramen of Winslow is bounded by

a) First part of duodenum b) Lesser curvature of stomach c) Liver d) Hepato-duodenalligament v ~

3 The ilio-inguinal nerve is derived from

a TI2 ry b LI c L2 d L3 e L23

15 Surgically the structure used to suspend the kidney to the diaphragm is

a) Renal fascia b) True capsule c) Perinephric fat d) Paranephric fat

6 If there is portal obstruction because of carcinoma affecting the pancreas which of these of the

following signs would be present

a Caput medusae b Esophageal varices c Rectal varices c

d Pulmonary edema

7 In a sliding hernia the gastro-esophageal junction lies

a) At its normal position b) Below the normal position c) Above the normal position V d) None of the above

8 Which of the following structures is retroperi toneal

A transverse colon B spleen IJ2f6 C ileum D descending colon v r 1pound1111111

9 The renal angle is fonned lgtetween the 12th rib and ______ muscle

a Psoas major -middotshyb Erector spinae c Quadratus Iumborum d Diaphragm

10 The anterior structure at the hilum of the kidney is

a) Renal vein ~

b) Renal artery I middot~ I

c) Ureter d) Accessory renal artery

11 Because of origin of the muscle from the lateral one third of the inguinal ligament it

could not fonn the anterior wall of the inguinal ligament

a) External oblique b) Internal oblique c) Transversus abdominis_ d) Rectus abdominis

12 A large tumor mass impinges on the splenic artery and its branches as the artery pass out from below

the greater curvature of the stomach Branches o(which of the following arteries would most likely to

effected by the pressure on the splenic artery

a Left gastric b Left gastro-epipJoic c Right gastric d Right gastro-epipoloic e Short gastric_

13 A new born baby has projectile vomiting after each feeding It is determined that there is obstruction

of the digestive tract as a result of annular pancreas Annular pancreas is as a result of an abnormality in which of the following process

a Rotation of the dorsal pancreatic bud around the first part of duodenum b Rotation of the dorsal pancreatic bud around the second part of duodenum c Rotation of the dorsal pancreatic bud around the third part of duodenum d Rotation of the ventral pancreatic bud around the first part of duodenum y Rotation of the ventral pancreatic bud around the second part of duodenum

14 As the liver bud enters the ventral mesogastrium the region of the mesogastrium stretching from the

liver to the anterior abdominal wall is called

a Lesser Omentum b Greater Omentum ~ Falcifrom ligament d Lacunar ligament e Ligamentum teres of liver

16 A patient has absence of his 12th rib In such a patient if the doctor makes an incision to approach his

kidney mistaking the 11 th rib for the 12t he would end up injuring

Which of the following arteries is a direct branch of the gastroduodenal artery The

A right gastric artery

B left gastric artery

C inferior pancreaticoduodenal artery D left gastroepiploic artery

i E)right gastroepiploic artery --

E x pI a nation The right gastric artery is typically a branch of the proper hepatic artery The left gastric artery is a direct

branch of the celiac trunk The right and left gastric arteries anastomose along the lesser curvature of the

stomach The inferior pancreaticoduodenal artery is a branch of the superior mesenteric artery it

anastomoses with the superior pancreaticoduodenal in the head of the pancreas The left gastroepiploic

artery is a branch of the splenic artery it anastomoses with the right gastroepiploic artery along the greater

curvature of the stomach The right gastroepiploic artery is a branch of the gastroduodenal artery The

other branch of the gastroduodenal artery is the superior pancreaticoduodenal artery

Which of the following pairs of veins join together to form the portal vein The

A superior mesenteric vein and inferior mesenteric vein

B inferior mesenteric vein and splenic vein

C superior mesenteric vein and splenic vein

Ip)splenic vein and left gastric vein E superior mesenteric vein and left gastric vein

Explanation

The portal vein is formed behind the neck of the pancreas by the union of the superior mesenteric vein

and the splenic vein The inferior mesenteric vein drains into the splenic vein The left gastric vein drains

directly into the portal vein After the portal vein forms it enters the hepatoduodenalligament of the

lesser omentum to reach the liver The portal vein is the most posterior structure in the hepatoduodenal

ligament

At which of the following vertebral levels does the duodenum pass anterior to the aorta - _- shy

All ~

B L2 7~

CL3 I

~DL4

E L5

Explanation

The duodenum begins at the pyloric sphincter at the level of Ll The second (or descending) portion of

the duodenum is to the right of the aorta and extends inferiorly from the level of Ll to the level of L3 The third part of the duodenum crosses the aorta from the right side to the left side at the level of L3 The

fourth (ascending) portion of the duodenum extends from the level of LJ to the level of L2 The

duodenum ends at the duodenojejunal flexure The superior mesenteric artery passes anterior to the

duodenum as the duodenum passes anterior to the aorta The duodenum can be constricted at this level

In which of the following locations will perforation of the digestive tract result in the spilling of luminal

contents into the - lesser peritoneal sac

A Anterior wall of the second portion of the duodenum B Posterior wall of the second portion of the duodenum

C Anterior wall of the stomach

~Posterior wall of the stomach E Posterior wall of the transverse colon

Explanation

The posterior wall of the stomach is related to the lesser peritoneal sac The anterior wall of the stomach is related to the greater peritoneal sac The anterior wall of the second portion of the duodenum is related to the greater peritoneal sac The posterior wall of the second portion of the duodenum is related to the retroperitoneal space The posterior wall of the transverse colon is related to the greater peritoneal sac

The ureter lies against the anterior surface of which of the following muscles shyA Crus oftne diaphragm B Quadratus lumborum

0 Psoas major D Transversus abdominis

E Iliacus

Explanation The ureter exits the renal pelvis at about the level of vertebra L2 As it descends along the posterior abdominal wall it lies on the anterior surface of the psoas major The psoas major muscle arises from the bodies of the lower lumbar vertebrae The psoas major muscle is joined by the iliacus to fonn the

iliopsoas muscle The iliopsoas muscle then attaches to the lesser trochanter of the femur and is the major

flexor of the hip

As the right ureter passes the pelvic brim it lies against the anterior surface of which of the following

blood vessels

A Gonadal artery B Inferiorvena cava C Internal iliac artery

rJ- External Iliac artery

E Inferior mesenteric artery

Explanation

The ureter lies in the extraperitoneal space in the posterior abdominal wall Alter leaving the kidney it

passes inferiorly on the anterior surface of the psoas major muscle At the pelvic brim the ureter passes

into the pelvis At this point the common iliac artery is dividing into the external and iliac arteries The

ureter lies on the anterior surface of the external iliac artery immediately distal to the bifurcation This is a useful landmark for a surgeon to locate the ureter

When extravasated urine passes from the superficial perineal space into the anterior abdominal wall it is

found immediately deep to which of the following layers of the anterior abdominal wall

-ltScarpas fascia

B External oblique muscle

C Internal oblique muscle D Transversus abdominis muscle

E Transversalis fascia

Explanation

The superficial perineal space is bound by Colles fascia the fibrous portion of the superficial fascia This

layer of fascia is continuous with Scarpas fascia the fibrous portion of the superficial fascia of the anterior abdominal wall Therefore urine that is deep to Colles fascia will remain deep to Scarpa s fascia The urine will spread in the plane between Scarpas fascia and the external oblique layer

When a horseshoe kidney develops the ascent of the kidney is restricted by the A internal iliac artery B external Iliac artery

C common iliac artery

inferior mesenteric artery

E superior mesenteric artery

Explanation

A horseshoe kidney develops when the inferior poles of the to kidneys fuse together as they ascend into

the abdomen from the pelvis The first anterior midline vessel that is encountered by the horseshoe kidney

is the inferior mesenteric artery This artery prevents the kidney from continuing its ascent

The left testicular vein drains into which of the following veins

A Left internal iliac vein B Left common iliac vein

bflnferior vena cava D Left renal vein I

E Left internal pudendal vein

Explanation

The left testicular vein drains into the left renal vein The right testicular ~i~[~nsltjectlY into the

inferior vena cava This difference in venous drainage is believed to explain the greater incidence of

varicocele on the left side than on the right The venous drainage from the penis is to the internal vein

which then drains into the internal Iliac vein

The spinal nerve that provides cutaneous branches to the skin around the umbilicus is

A TS B TW-shy

C TI2

DL2 EtA

Explanation

The tenth intercostal nerve is the anterior ramus of the TIO spinal nerve After passing through the tenth

intercostal space the nerve continues forward in the anterolateral abdominal wall in the plane between

the internal oblique muscle and the transversus abdominis muscle In the abdominal wall the nerve innervates to the abdominal wall muscles as well as the skin and the parietal peritoneum The umbilicus is

a useful landmark for the region of distribution of the tenth thoracic nerve

The ligament of the vertebral column that resists its extension is the Aligamentum flavum

B supraspinous ligament

C posterior longitudinal ligament

D anterior longitudinal ligament

E interspinous ligament

Explanation

The ligaments of the vertebral column that resist flexion of the column include the supraspinous ligament

interspinous ligament ligamentum fiavum and posterior longitudinal ligament The ligament that resists

extension is the anterior longitudinal ligament This longitudinal ligament is very broad and strong It

covers the anterior and anterolateral surfaces of the vertebral bodies and the intervertebral disks In

addition to resisting extension the anterior longitudinal ligament provides reinforcement to the anterior

and anterolateral surfaces of the intervertebral disk The posterior longitudinal ligament is relatively

narrow and covers the posterior surface of the vertebral bodies and the intervertebral disks This ligament

reinforces the posterior surface of the disk The posterolateral surface of the disk is not reinforced and it

is through this region that herniation of the nucleus pulposus usually occurs

A patient presents with epigastric and right upper quadrant pain The pain is most intense 2-4 hours after

eating and is reduced by the ingestion of antacids The patient states that he has passed black tarry stools

(melena) within the last week Fiberoptic endoscopy reveals a yellowish crater surrounded by a rim of

erythema that is 3 cm distal to the pylorus Accordingly an ulcer has been identified in the patients

A fundus

B antrum

C duodenum

D jejunum

E ileum

A number of physiologic genetic and other factors increase the risk of gastric (and duodenal) peptic

ulcers The evidence that H pylori plays a principle role is compelling Smoking and caffeine are known to adversely affect the morbidity mortality and healing rates of peptic ulcers In general first-degree

relatives of peptic ulcer patients as well as males have a threefold to fourfold increased risk of developing this disorder Paradoxically in gastric ulcer disease acid secretion is not elevated It is possible that

excess secreted hydrogen ion is reabsorbed across the injured gastric mucosa In general a defect in gastric mucosal defense is the more important local physiologic

A patient presents with symptoms of duodenal obstruction caused by an annular pancreas Annular pancreas is caused by

A rotation of the dorsal pancreatic bud into the ventral mesentery B rotation of the ventral pancreatic bud into the dorsal mesentery

fJ failure of the major and minor pancreatic ducts to fuse ~ ~ cleavage of the ventral pancreatic bud and rotation of the two portions in opposite directions around -the duodenum E formation of one pancreatic bud instead of two

Explanation Normally the ventral pancreatic bud rotates around the gut tube to reach the dorsal pancreatic bud The two buds fuse to form a single pancreas and the distal portions of the two ducts fuse The ventral pancreatic bud forms the inferior portion of the head of the pancreas the uncinate process and the major pancreatic duct (of Wirsung) The dorsal pancreatic bud forms the superior part of the head the neck body and tail and the minor pancreatic duct (of Santorini) Annular pancreas is the result of the ventral pancreatic bud dividing into two portions before it rotates into the dorsal mesentery Each portion rotates in opposite directions to get to the dorsal mesentery thus encircling the duodenum The presence of annular pancreas can constrict the duodenum thus obstructing its lumen

In n _ phranlc----

Gon ~l ----_1 Lum bltano

~~--- CornmQ1t bull ac

+-~4--- lnlllirnaJ ilic

xtem iliac

OBJECTIVE - Identify the blood supply to each of the structures listed in the table on the previous page

Ill give you a head start

FOREGUT - Supplied bV Celiac Tru nk (T12)

Proper hepatic

GastiooUod 13Jafter

1nferlor pancreaticoduodenal artery

Common epatlc

Lett gas ric iiirtery

Spfen artery

shy Gastroepiphgtic artery

~ Superior mesenteric 8rtfry

~

1 Esophagus is a derivative of the foregut so its blood supply originates from the celiac trunk

(T12) The predominant blood supply to abdominal portion of the esophagus is the Esophageal

A (Branch of L Gastric) The venous drainage of the esophagus is particularly important because

it is 1 of 3 clinically relevant sites of Portal Caval anastamoses The Portal Esophageal Vein

meets the Caval Azygos System Persistent bleeding manifests as Esophageal Varices - a fata I

condition

2 The Stomach is also a derivative of the foregut has EXTENSIVE blood supply and is very high

yield on anatomy exams The lesser curvature is supplied superiorly by the L Gastric A (1 of 3

major branches ofthe Celiac trunk) and inferiorly by the R Gastric A ( a branch ofthe proper

Hepatic A) The greater curvature is supplied superiorly by the L Gastroepiploic A (a major

branch of the splenic A) and inferiorly by the R Gastroepiploic A

The Short Gastric arteries (branches of Splenic Artery) supply the fundus of the stomach and

are referred to as EIID ARTERIES because they have no collateral blood supply Therefore if the

splenic artery were occluded (ex - increased pressure in the ommental bursa) - there would be

ischemia to the fundus of the stomach Venous drainage of the stomach is extensive via various

veins lead ing to the portal system Posterior to the stomach the IMV joins the splenic V which

joins the SMV to form the PORTAL VEIN ADAMS

3 Duodenum blood supply has high clinical relevance because it is the junction of the foregut and

midgut and therefore is the site of anastamoses between branches ofthe Celiac Trunk (main

foregut artery) and the Superior Messenteric Artery (main midgut artery) The Proper hepatic

artery gives off the gastroduodenal artery which travels behind the 1st part of the duodenum

This point has high clin ical relevance because duodenal ulcers are very common and a posterior

rupture of the 1st part of the duodenum could rupture the gastroduodenal artery causing

traumatic abdominal bleeding The Gastroduodenal artery first gives off the R Gastroepiploic A

(mentioned above) and proceeds as the Superior pancreatico duodenal artery (supplies the

pancreas and duodenum) which anastamoses with the inferior pancreatico duodenal A (branch

of the SMA) This is the junction of foregut and midgut and occurs near the opening of the

bil iary system into the duodenum (ampula of vater) Portal venous drainage here is responsible

for delivering nutrients from digestion to the liver for metabolism Appreciate that the Superior

mesenteric artery (artery of the midgut) branches from the aorta at Ll travels posterior to the

pancreas than moves anteriorly (at the jxn of the pancreatic headbody) and comes over the

3rd4th part of the duodenum Tumor of the head of the pancreas can compress the SMA

4 Jiver blood supply is via the common hepatic artery (major branch of the cel iac trunk) The

common hepatiC becomes the proper hepatic gives off the R gastric A and the Gastroduodenal

A and then joins the common bile duct and the portal vein in the portal triad Clinical- if a

patient were bleeding from the hepatic A a surgeon can stick his fingers in the epiplOic foramen

and squeeze the free edge of the hepatoduodenalligament in order to stop bleeding to the

area Please note that the hepatic a branches into Rand L hepatic A The Right hepatic artery

gives off the cystic artery which supplies the gallbladder Afferent venous supply is via the

Portal vein which is bringing nutrient rich blood to the liver After metabolism takes place

venous blood leaves the liver through the hepatic veins into the IVC PLEASE UNDERSTAND THE

RELATIONSHIP OF THESE STRUCTURES - ADAMSNETIERSNH Etc

5 Pancreas - Head is supplied via the superior and inferior pancreaticoduodenal arteries

(mentioned above) The tail (situated towards the hilum of the spleen) is supplied via the

pancreatic branches of the splenic artery (END ARTERIES) This blood supply is very important

because the endocrine Alpha and Beta Cells from the pancreatic islets of lagerhans are located

towards the tail This is where Insulin and Glucagon is released to the blood

Now complete this for mid and hindgut structures Make sure to note clinically relevant arterial

anastomoses as well as portal caval anastomoses FYI Appendix blood supply SMA + IMA

anastamoses marginal artery Portalcaval rectal veins fhemmorhoids) and periumbilical caput

medusa are high yield THE BUTT THE GUT and THE CAPUT

Abdominal Development

Liver

Ij1f

II wall b

oh liN ~ VltJrti n be- bull

Pancreas

Secondary Retroperitonealization e I~tl r 1 a v-mtrai m ellter

Rotations of the Gut I i Ij (lIl1UtIJ f~ l r tilt

()l td 10 me l-ft and he v

--~--- -~ -~-~

i

I AolaijonjoI~guf I

STOMACH BED (IDENTIFY IN ADAMS)- the structures posterior to the ommental bursa which

support the stomach in the supine position

Abdomnal JQrUI

Splnic vein

OmQ-oul tv~ ) O(s(Jroa)

Lojt(r o m nturrt (hpJtodu o d~n31 Hid

Gadrl)SplerH (g3stroll~nal) IIgam~nt

hiad h~~atogrtricent IIQdmiddotcrt~)

Lt Dome of Diaphragm (why left Look this up in Adams)

Spleen (What is the blood supply)

Left Kidney (What is the blood supply - AND how is it different from the R kidney)

Suprarenal Gland (What is the Arterial AND Venous Blood supply - how are they different)

Pancreas (How does supply differ from Head to Tail What is the SMA Relationship)

Transverse Mesocolon

liver - ADAMSWET - Make sure you look at the liver in wet lab

Left triangular nl1am~nt

ComoaDj ligamnt

Erophg~1 impre$ioo

Hepatio veins

In1erior -ifena middotr3)Ia

Fibrous appendix o-t

live

impr~j on

Heprorendl p~rtion of Q)(Qllary ligament

Righllri~n9ul r 1I~met

(Common) bile quol

Gr)mmCtr~ hepatic dlJct

Ccentic duct

Duodenal impression

GaJdate p-fr)~S

Hepatic artgtrl prop-f iiiiila - Faloiform ligament

_ - shy Round ligamen liver

~--F-- CoJio imprgt-ssi-on

Prta heptis

Identify the lobes impressions and embryonic remnants associated with the liver

Caudate Lobe Quadrate Lobe Right Lobe Left Lobe Round ligament Falciform Ligament

Ligamentum Venosum (what is its fxn in embryonic life) Hepatic Veins (NOT PART OF THE

PORTAL TRIAD) IVC PORTAL TRIAD - Contents relationship cross section etc Know the

Galbladder relationship to the lobes of the liver

Biliary Duct System - Make sure you understand the sequence of these structures - BE ABLE TO

DRAW A FLOW CHART

TPVd i

t

I t

1 __ Cm-(r

patk GlJet

I

J

Clinical = JAUNDICE is caused by anything that prevents delivery of bile to intestine Tumor of the

head of the pancreas Stones etc Patient will have pale stools and yellowish colored mucus

membranes

Clinical- Any scenario that tells you the patient has BILLOUS VOMIT means that the obstruction to

the flow of digestive contents is after the Ampulla of Vater (Site of Entry of Billiary system to the

duodenum) - ie Duodenal Atresia

Spleen -located posterior to the mid axillary line between ribs 9 and 11 Make sure you know that

the 10th rib is the main axis of the spleen and this organ is susceptible to injury (stab wound errant

thoracoce ntesis etc)

The spleen is derived from mesodermal cells - NOT THE GUT TUBE

The spleen rests on the left colic flexure associates with the tail of the pancreas Know the

structures entering the Hilum of the spleen

Sh rt O~-t~ic 1 0(0 10 rtiltSPIric Iloa nt

(cut)

Peritoneum - similar concept to Pleura - think of a fist in a balloon

Visceral Peritoneum - Layer of balloon touching your fist

Parietal Peritoneum - Layer of balloon not touching your fist

Your fist represents the organ your wrist is the hilum and your arm contains the blood supply

entering the organ

Appreciate that there will never be organs in the peritoneal cavity - rather these organs invaginate

the cavity Kaplan videos

RULES OF NOMENCLATUREshy

1 Organ completely surrounded by peritoneum - peritoneal organ

2 Organ partially surrounded by peritoneum- Retroperitoneal

3 Peritoneum surrounding peritoneal organ is VISCERAL peritoneum

4 Peritoneum surrounding retroperitoneal organ is PARIETAL peritoneum

5 Peritoneum connecting visceral to parietal is called messentary 2 messentaries in the

gut Dorsal (to the gut tube) and ventral (to the gut tube) messentary

Aorta is in Retro peritoneal position - but blood must reach peritoneal position - vessels travel through

messentary All peritoneal organs will have blood supply reaching through messentary

-Mesentery is a 2 layer peritoneum with a neurovascular communication between body wall and organ

- Ligament connects one organ with another or to the abdominal wall (Ommentum = ligament)

lesser Ommentum (attach lesser curvature of stomach and duodenum to liver) =Hepatoduodenal

Ligament and Hepatogastric Ligament

Has a Superior and Inferior Recess (Accumulation of Fluid in Ascites)

Communicates with the greater sac through the epiplic foramen (what structures pass through

this foramen)

Boundaries - you must be able to visualize this

o Anterior - stomach

o Posterior - parietal peritoneum pancreas

o Superior - superior recess (bw diaphragm and coronary ligament)

o Inferior -Inferior recess (bw layers or greater momentum

Greater Ommentum (attach greater curvature of stomach) Gastrophrenic ligament Gastrosplenic

ligament gastrocolic ligament

The greater omentum is the largest peritoneal fold It consists of a double sheet of peritoneum folded on itself so that it is made up of four layers The two layers which descend from the greater curvature of the stomach and commencement of the duodenum pass in front of the small intestines sometimes as low down as the pelvis they then turn upon themselves and ascend again as far as the transverse colon where they separate and enclose that part of the intestine

ABDOMINAL PAIN

Parietal Peritoneum - supplied by same vasculature lymphatics and nerves supplying body wall it

lines and diaphragm Sensitive to pain pressure heat cold well localized

Visceral Peritoneum - supplied by same vasculature lymphatics and somatic nerve of organ it covers

Insensitive to touch heat cold and laceration - referred to dermatome of spinal ganglia providing

sensory fibers Where does appendicitis refer to

Foregut pain - epigastric area (ie - cholycystitis)

Midgut pain - periumbilical area (ie - appendicitis)

Hindgut Pain - suprapubic area (ie - diverticulitis)

Extra ImagesConcepts

ll~_____-

FalifCtrm ligament oind r~ud ligamet f Ilver

Blood from splenio gastriC and inferiof rne$e-rteri v~ins

Ca-I tributaries

Lett gastrio Ifein

Posterior superior pan~reatioodul)denal vaihS

Lott gamo-om~nlal (9aropip lomiddotic) -in

Poq_~ tjol imerl-9-r panCJertlcorllJod-nal veiopound --amp----I- - ~J Right grtr~-omntal

Anwrior interi (gartroepiploic) Jjn

pan euaii cod vl)denal veins middot Inf~Ji (t r mesentric vein

Miqdle (olic vein

Right cl)licvein Sigmoid and rectosigml)id (ei ns

IhH)Collc(~io

--- Mi~dl laquooLJl gtjrltgt

PoM ca vl1 illasto)moses -----shyampoptoageal 2 Paraumbilie-lt11 Inferi or Fectal vei ns

3 Recial 4 REuoperHonea1

Know how the Portal vein is formed I 4 sites of portal caval anastamoses and 1 clinical shunt

Col li t ltt-~ otTl~tI ~nj pc~ 1lt1 turJoG

Ltf 14i1 tImiddot~ artoftl9 on tj phtAt$

L-oftqf 4t t~r 1=laquoIran d 1 bull shy~p l ci rj o fOOOts

Nerves follow the arteries - appreciate the splanchnic nervous system I

Uet~ric branch of left ~nal art

Ureterie branch of righi renal artelY

Left Zld lumbar in and co mlTlunication to as)erdin9 lumbar l(~in Hi ~ht tEZ1~~t~ t3r j t itn ~ nJ l1t- rlnd lfe i r1

Inferior me5nteri~ artery

Notice that the right testicular vein drains directly into the IVC and the right testicular artery drains

directly into the aorta However the left testicular vein drains into the L renal vein at a right angleshy

reason left testicle is lower and more susceptible to varicocele (bag of worms)

Also notice that the left renal vein has a longer course because the IVC is on the right side whereas

the right renal artery has a longer course because the aorta is on the left side

Appreciate the anterior to posterior relationship of structures in the hilum of the kidney - VAP - Vein

Artery Renal Pelvis (Ureter)

11____ __ L_ L_ n VJ __ _ _ t_L I I_ _ L __ L_ I -pound1 bull LI_~-I ____

Posterior View of Head of Pancreas in ( of Duodenum

Celiao hunk

Co mmon ~L~jJth art~ry

GastNduQdonal artrf (partilly in phantn)

P1)Sterior $Up~Jior panCflaticuduodfmal art~r~t

(Co mm on) bile duct

middot~1t~~t-1l---~-~- Right gshomiddotomental (gastoe plp lolc) 3rte (phantomost)

Grener paocre atic art-ry

1n1~rjor pancr-iatlc artery

Jtrifll supejo r pal)oreailcento)dJodenal artr1 (phantom)

Anastomotlo branch

POostetlor bJanch of jo f~ri of pan-reatir(lduodensl drttnj

Anterio r branch of i flferior palcreati~)duodenal art~(phan1om)

Notice the extensive blood supply to the pancreas and duodenum via the branches of the celiac trunk

Notice collateral supply from SMA branches - makes sense bc this is the jxn of foregutmidgut

Identify the vessels in this arteriogram

Hiltid i)f N~ck oi B)dvof Tail 01 pa nereas pan cent~as P-nmiddot-reas panCtCas

I nferie v~na cava

jHept1iic p(lrlai v~in

Port1 tnd H~pti lt a ftH prol

Comm on) bll duct

Ouodtnum

~ft colic (sio)Atta~ hmtrlt jt~xJr-ofha~elSe

muo(IIQn

Right ~lIc (h~j)tic)

il~gtture

In1triol m~oten lIein (rttr op~ritoMdO

SlJp efl or mes~n~fiC amrV and lipln

KNOW YOUR NEIGHBORHOOD

Questions

vVhiJh structure supplied by a bnmdlof the cclia( artery is not derivcd from foregut LemCJUCrITI

(A) Head of the pancte-a5

CD) Pyloric duolenum

Cystkduct

( Liver hepatocyt~~

~F) Body of the spleen

An infant presents with an omrhaJucele at birth -hi oJ the [oHm illg applies to his cM1-dition

(A) It is 31so seen ill p4titnts with aganghonic megacolon

(11) ft reuirs from a fal1ure of resorption of theviteUine d let

(C) It results from herniation at the-site of regression of the right umbilk vein

DJ It is caustd by faihtrc of recanalization of the midgut part of the duodenum

~ It ill camioo by a failuIt vf the midgul to return to the abGQminal uity after herniashytion in-n the urnbilk s l stalk

Ot er than the spleen occlusion Cif the spit-Ilk artery at its odgin wm most likely affect die blood supply to jllch st cnud

(A) Jejunum

(B) Body of th pal1~lltas

(C) LeSStT Cllmiddotlaturc of tl )toma-ch

(D Duodenum dista to the entrance of the Ornmou bile duct

E Fundus of the stomach

A 38-yeu-old batL~er with a history of heartburn suddenly experiences excluciating pain in the (plgastric region of th~ abdomeu SurgCry is perf~rme immediard y upon admisshysion to the 1IlcrgCJliy tuomh~re i~ evidence uf a ruptured ulcer in the posterior waU of the stomach Vhere will a surgeon first fi nd the stomach contenlSf

A) Greater p4ritoneal sac

rB) Cul~de-s~c of Douglas (--

C Omental bursa ~

--D) Paracolic gutter

rEj Between -he panttal perimltum and the posterior body wal1

At birth an infant presents with a st()ma~ rb~tbas~njJled jfltotb~diaplfagru 1A1ltre is the defect thatresulied iiitJle heini~t()n shy~tsophagealbiatus

7 - rH-- Hiatus for the inferior vena cava

( Pleuroperitoneal membrane -(0) Septum transvcrsum

(E) Right Crlt~

An infant born with DOVv7l syndrome presents with bili()u~ vomiting Ahat congenital defect does the infant have

(A) Pyloric stenosis

(B) Meckel diverticulum C) Ornphaloce1e

(D) Gastroschisis

( ~ ) Duodenal atresia y A patient with cirrhosis of the liver presents with ~ bacalvaricestnlreased retrograde pressure in which veins caused the varices

(A) Paraumuilical

(B) Splenic

(ct AzygltJus

(15))G~trk ( (-F) Superior mesemeric

A htaltby 3-year~old male patient experiences a hernial sa protruding from the anterior abdominal wall about halfway between me anterior superior ilia spine and the pubk tuberde Pulsations of al1 artery are palpated medial to the protrusion site through the abdominal walL Which layer of the anterior abdominal wall will first be traversed by the

1hctma

fA) Rectus sheath (B) External oblique aponeurosis

(C) Inguinal ligament

lD) Transversalis fusda

(E) Cremasteric fa~cia

After 5urgi(aj ffpair of a hernia the patient tXperienccs mtmlgtness in the skin on the anteshyrior aspect of the S(Totum_ Vhaf nerve may have been lesioned during thehemiorrhaphy

(A) Femoral

(B) Obturator

(C) Ilioinguinal

(D) lliohypogastrk

(E) Pudendal

A 23~year-LJld female secretary il1 good health ~-uddcn1) doubles over with pain in the a ea of the 1JmbRicu$ Sbe feels vartn and ltneasy and has no appetite That night the pain seems to have mQved to the tower right abdominal regjol1 and she calls her family doctor who then arranges for an ambulance to pk-k her up and take her to the hospitaL Wh ell ntn~ perceived in the area of the urnbilirus most Hkely carried lhe pairfu I sensations into the eNS

tA) Vagus nerves I~

V B)

) Lessersplanchnk nerves

tC) Pudendal nerves

(D) lIiohpogastrk nerves

(E) Greater splam ic l erves

A CT reveals carcinoma in the bOod of the ancreas Vhich blood vessel trut ourses ----~- - -bull ------ --shy

immediately poftterior to the body ofthe pancreas is the m~t likely to be oompressed

(A) Splenk artery

(B) Abdominal aorta (C) Portal vein

(1) Splenic vein

(E) Renal vein

A patient has a penrln1l1ng uker of the posterior wall ot the br~l part ot the (lUooenmn llkh blood vessel is subject to erosion

(A) Common hepatic artery

(B) Gastroouodenal artery

(C) Proper hevatic artery

(D) Celiac artery

(E) Anterior inferior 11amrelltlcoduodcnal attery

Your patient has been diagnosed -ith a carcinoma locallted to the head and l~e(k of the pancreas Another clinical sign would be

A esophageal varices

(8) hemorrhoids

C) a caput medusa

(D) increased pra Teuro n th~ hepatic veins

(E) enlarged right supra lavkular lymph nodes

Wltkh of the foUowing structures develops in the ventral mesentery

(A) Spleen

(B) Jeiunum (C) Head of1ht pancreas (D) Transverse colon (E) Stomach

ti l Uw ~ littwin~ f( S-t lil oai Imdge ~ hi(h or tbt la~)d J truetur tgt liJ llntn nl) he hl p UC iJd [IIi ell

c o

A) drains Ie tht infCrior a La aI

R t middot~nfl0 ~ill to th~ lunlgtn of h i dtlndCrlllfH

(e) m t bull JiJattd on tl l J n T ~H

D ) sup Lc O VSlt I Hlid bhtu l 1 li - -I un oid

( ) U~tpli(t tr j middottUh~ 1 v(( b~nt rfK n1ilc~Zm

ANSWERS AND EXPLANATIONS

Answer E The spleen is t hlttnopodicand lymph organ demlted from mesoderm

Answ~ R Al1 tlmphalocele is caused by it failure of the nlidgut to return to the ahdomir nat cavity after herniation into the umbiliau Stalk Choices Aand D maybe seen in infants with Down syndrome choice D ~s the specific CBuse ofduudcnal JtiCSitt Choice C is (ile cause of gclstrosbisis and Choice B nsults iu a Meurolktldivertku1-tlB

Answer B The fundus ofthe stomach is suppHed by soort gastric brunches of the splenic altery The splenic artery supplies the body and tail of the pancreas part of the greater curvature of the sttmla(h and the spleen Te jejunum part of the head of the pancreas and tht~ duodenum distal to the entrance of the commOll bile duct are supplied by the superior mesenterk artery clll~l ~be less r ctlt1ature cmd the pylQric antrum are supplied by the right and lei gastric art(ries

AnSWftt C Tbeomental bursa or lesser ~ritoneaj sac lies direcdy posterior to the proxshyimal part of the duodeTtlm and the stomach and would be the first site where stomach contents ~Ott1d be fpoundluncL

Answer C A defect in a llleuropcritoneal membrane (uswlly the left) is the typical site of i1 cc-ngenitlI diilphragluatic hemia llere the membr4ne fails to dose ()pound( of the perishycCirdiopcritulleal canals

Answer E DuoJenal atresia and aganglionic megacoion are congwitaI defects S~Il in patients with Dowmiddotnsyndrome

Answer D RulaTgemt~llt of and retrograde flow in g~lstrk vel_ns in particlJl~r the kft gas~ tricveins dilates the capillary bed in rhe wall of the esophagus in (ases of porta yper~

tension Blood flow would increase in and dilampte tribntarkgts of the (lZygOUS vein on the other side of the capiUary bed but flow in this vein is in the typical direction t()ward the superior vena cava Paraumbiii(ltU vein eilgorgement contributes to a caput medusH Splenic ~nlargement might prc~nt with 5plcnonlegaly and balt-kflow in to tlu superior m~~ntclic vein occurs but is asymptomatic

Answer D The patient hagt an indirect inguinal hernia whi~h emerges from the antt-rior abdominal wall through the deep inguinltilling Theeep ring is a fault in the transv~rshysaUs fascia this I~yer wiIJ be penetrated first by the hernia

An~Wer C The ilioinguinal nenc which provides sens~llion to the lnedlal thigh ltmclanteshytior SClotunl pass~lt th rough the 5uperfh_ial inguinal ring ind $subject to inj i1T) becaus-e

it is in the operatitm Held of the erniorrhapny

Auswer B The leMHr splanchnic nerves are sympathdic nerVlts that carry viscera l sensashytlltgtrogt ftom illtllt1m~d ()J stietched gust (itinteitinal ~tructures (in this case the pprndix) into tnt eNS Lesser splanchnic ntTYcsarisc from thmiddot T9--T12 spinal cord segments lt1nd provide sympathetic innenation tD rnidgut siruc1ures whiCh include CLe app~JldD Viscera] Pain arising from affecLed Inidgut ampt 1C1ure is referred over the same dl- matorne~ of spinal segrnertts v-hich provide the sympathetic Innervation n this G1SC of appendicitis the invohen~n t of the ltire) of t e unlhHku indud s the T 10 dermatome

Answer B Of the five choices onty the dscending olon is retroperiton~al aldwould be a lik ~ ( choice to be seen immediately a(~jilcent to t11e posterior abdominal middotn~L

Amwen D The SpltftlC ~-ein ourses posterior to the body of the panneas m its way tt drain into the superior mCSfttltlri( vein

Answcr B TILt glstrodllolticnal artery 1 direct hIamh of the comrootl hepatic artery courses immediately pt))iwri() to the duodenum and is slbject to erosion

Answer B Carcinoma of th pan middott3S in the 1tilt1 may compreampgt the portltil vein at irs orishygill The poTtai vcin is fomled when the splenic vein jQiaswith tfie superior meStllt eric vein The inferiot mesenteric vein joins the ~plenjc vein just priOT to tlli~ point at which the splenic joins the superior Jlleit1ltcri( vein Increescd venous presslu in the inferior mesenteric vein is a cause of emo hoid~

Answer C The- velltral pancreas wilich forms most of the head of the p ~ncr as develops in the ventral mes(ntery as antutgrowth of the hepatic diverticulum Th~ hepatic divershyticulull induding the biIJary appa~atus develops in tbe ventral mesentery of the foregut

Answer~ A The superior mesenteric ~in joins with the spienkvein to form the hepatic portal vciu

Answer D The structure at gttlK is the proper hepatic artery~ whkh suppUesoxygenated b middotood to the liver

MAKE SURE YOU KNOW the diff bw Rectus Sheath above and below the arcuate line

ABOVE

Aponeurosis of xiiltmal obllque musclo

Extemll f)biquw musde

Reotln ilbdomlnls musole S~in

Internal 9bliquQ mY~QI

AponeUfOsi$ of hJH$V~~S Lir9a a lb lbdolTlin~ musolo Tri OJV6 rUi

atldomlnis mUS(loe

Sub cutanlilous tiue (tatty ye r)

BElOW

A POrl lJfosis 01 etemal oblique muscle

Aponeul~)sis 01 Internal oblique mU$cl~

Anteriol lay~ of r~ltdus st~ath EXttom1 oblique rnu$cll

Rectus Jbdominis muscle Intoernal Aponeurc-sis of tra~fersU$ oblique muscle-

at-domlnis muscentl ~ Skio

Tra nsvitSus abdomioLs ml)ZClt

TralSVersaHs fascia Medial umQil iegtt1 1i9Jment -and folj

Uldchus Peritoneum (ir median Umbilj~al Suboutane ous

Extraprftone 11ascia

Ymbilimiddot~1 fold)

preu9poundiea1 fascia

tissue (fatty 4nd m~mbr3n(iUS layers)

o Above the arcuate line (A horizontal line 13 of the distance bw the umbilicus and the

pubic symphysis) -10 Aponeurosis divides into an AntPost Laminae

o The Ant Laminae joins EO and Post Laminae joins Trans Abdominis = Ant and Post

RECTUS SHEATH respectively

o BElOW the arcuate line - all 3 aponeurosis join ANTERIOR to rectus muscle to meet its

counterpart in the midline (linea Alba)

o Take away Msg - The abdomen is devoid of a posterior rectus sheath below the

arcuate line and is therefore more vulnerable to herniasinjuries

Question - A physician makes a deep incision in the patients midline immediately superior to

the pubic symphysis which of the following layers is his knife least likely to pass

Rectus Abdominis External Oblique Ant Rectus Sheath Posterior Rectus Sheath All of the

Above

Answer - All of the above None of the other answer choices are midline structures -LINEA

ALBA

Linea Alba has very poor blood supply - doesnt heal well after surgery Therefore this is a

common site for incisional hernias

a Spleen b Transverse colon c Descending colon d Stomach e Pleura

17 Meckels diverticulum is normally found 2 feet proximal from the

a Pyloric sphincter b Lower esophageal sphincter c Ileo-cecal valve d Middle valve of Huston e Anal valve

18 Ulcer in the posterior wall of the first part of the duodenum would erode ___ artery and would cause bleeding

a Left gastric b Right gastric c Hepatic artery proper d Gastroduodenal artery e Middle colic artery

19 An inflamed appendix is identified by a surgeon on the operation table by noting

a The appendicies epiploicae b The convergence of tenia c The artery of Drummond d The mesocolon e The mesosalphinx

20 The nerve which emerges through the psoas major is

a Femoral b Ilio-inguinal c Ilio-hypogastric d Pudendal e Subcostal

21 The right gonadal vein drains into the

a Azygos b Hemiazygos c Inferior Vena Cava d Right renal vein e Left renal vein

22 The hepatocytes in the liver is derived from

a Ectoderm b Endoderm c Mesoderm

d Neural ectoderm

23 Abscess in the lumbar vertebrae due to tuberculosis would spread to the adjacent muscle which is

a Psoas Major b Iliacus c Quadratus lumborum d Tranversus Abdominis

24 The anterior wall of the inguinal canal is formed by

a External oblique and transverses abdominis b External oblique and fascia transversalis c Internal oblique and external oblique d Internal oblique and transverses abdominis e Fascia transversalis and peritoneum

Meckels diverticulum is a result of which of the following developmental abnormalities shy

A Failure of the vitelline duct to close

B Failure of the herniated intestinal loop to retract into the abdomen

C Failure of the urachus to close

D Failure of the midgut to rotate

E Failure of the hepatic duct to close

Explanation

Meckels diverticulum is a result of the persistence of the proximal part of the vitelline duct This

diverticulum is usually found about 2 feet proximal to the ileocecal junction and is usually about 2 inches

long It is present in about 2 of the popUlation It may be the site of ectopic pancreatic tissue or gastric

mucosa and may develop inflammatory processes and ulcerations Acute Meckels diverticulitis

simulates appendicitis

Which of the following veins carries blood from the esophagus to the portal vein The

A right gastric vein

B left gastric vein c splenic vein D azygos vein

E left gastroepiploic vein

Explanation

The left gastric vein a direct branch of the portal vein drains blood from the lesser curvature of the

stomach and the inferior portion of the esophagus Because branches of the portal vein do not have

valves blood can flow in a retrograde path when there is an obstruction to flow through the portal system or liveL Rlooci Cln then flow from the nortl] vein thr()1Ph the left PRstric vein to the esonhlPlIS lno

through venous communications within the submucosa of the esophagus to esophageal veins that drain

into the azygos vein The increase in blood flow through the esophageal submucosal veins results in esophageal varices

On the posterior wall of the abdomen the celiac ganglion A contains cell bodies of postganglionic parasympathetic neurons B is synapsed upon by neurons in the posterior vagal trunk C is synapsed upon by neurons in the greater splanchnic nerve D contains sensory cell bodies of lumbar spinal nerves E contains cell bodies of neurons that cause an increase in the rate of peristasis

Explanation The celiac ganglion is one of the preaortic ganglia of the sympathetic nervous system It contains cell bodies of postganglionic sympathetic neurons The sympathetic splanchnic nerves contain preganglionic sympathetic neurons that pass through the sympathetic chain without synapsing These splanchnic nerves go to the preaortic ganglia to synapse The greater splanchnic nerve contains preganglionic neurons from spinal cord segments T5-T9 This nerve synapses in the celiac ganglion The nerve fibers in the vagal trunks are preganglionic parasympathetic fibers that go to the walls of the organs that they will innervate and synapse on postganglionic parasympathetic neurons in the walls of those organs Cell bodies of sensory neurons in the abdomen are found in the dorsal root ganglia or the sensory ganglia of the vagus nerve Sympathetic innervation decreases the rate of peristalsis parasympathetic innervation increases the rate of peristalsis

Which of the following pairs of arteries will allow blood to bypass an occlusion of the celiac trunk

A Left gastric artery-right gastric artery

B Left gastroepiploic artery-right gastroepiploic artery

C Superior pancreaticoduodenal artery-inferior pancreaticoduodenal artery

D Splenic artery-common hepatic artery

E Left gastric artery - proper hepatic artery

Explanation The anastoOlosis of a branch of the celiac trunk and a branch of the superior mesenteric artery will

provide collateral circulation around an occlusion of the celiac trunk Each of the other choices pair

branches of the celiac trunk therefore these will not provide collateral flow around the obstruction of the

celiac trunk The left gastric splenic and common hepatic arteries are direct branches of the celiac trunk

The right gastric artery is a branch of the proper hepatic artery which is a branch of the common hepatic artery The left gastroepiploic artery is a branch of the splenic artery The right gastroepiploic artery is a

branch of the gastroduodenal artery whlch is a branch of the common hepatic artery

Which of the following organs has appendices epiploica The

A sigmoid colon

Bjejunum

C duodenum

D stomach E esophagus

Explanation Appendices epiploica are characteristic of the colon Appendices epiploica are subserosal accumulations

of fat None of the organs of the gastrointestinal tract has appendices epiploica except the colon

Page 17: Chirag's Abdomen Review

Now Lets see how much youve learned

Questions

1) A pt receives a general anesthetic in preparation for a c~t~~my A right subcostal incision is made which begins near the xyphoid process runs along and immediately beneath the costal margin to an anterior axillary line and transects the rectus abdominus muscle and rectus sheath At the level of the transpyloric plane the anterior wall of the

-~~-~=--- _eco---shysheath of the rectus abdominus muscle receives contributions from which of the following

a Aponeuroses of the in~ande~tef-Ilal o~ues

b Aponeuroses of the transversus abdominis and internal oblique muscles c Aponeuroses of the transversus abdominis and internal and external oblique

muscles d Transversalis fascia e Transversalis fascia and aponeurosis of the transversus abdominus muscle

A

2) The lat~raJJJ11QjJt~gLfgJlLoneach side of the inner surface of the anterior abdominal wall is created by which of the following structures

K Falx inguinalis (~) Inferior epigastric a

c Lateral border of the rectus sheath d Obliterated umbilical a e Urachus

B

3) A man the victim of several knife wounds to the abdomen during a brawl at the Lobster Shack subsequently developed a direct inguinal hernY Damage to which of the following nerves is most likely responsible for the predisposing weakness of the abdominal wall

~ Genitofemoral nerve ( b) Ilioinguinal nerve ~-t Tenth intercostal nerve

d Subcostal nerve e Pelvic splanchnic nerve

B

4) Which of the following statements concerning a direct inguinal hernia is correct a It is the most common type of abdominal hernia b It transverses the entire length of the inguinal canal c It contains all3 fascia layers of the spermatic cord d It exits the inguinal canal via the superficial ingeJinal ring e It protrudes through H~acb strJg e

~(

1fltbS w E

tl

5) The conjoint tendon is

a Important in preventing indirect inguinal hernias b The fused aponeurotic layers of internal abdominal oblique and transversus

abdominus muscles c Posterior to the deep inguinal ring

d Medial fibers of the inguinal ligament

B

6) A 25 year old male is brought in to the ER after being involved in a car accident in which he received a crushed internal injury in his abdomen Examination reveals a lesion of parasympathetic fibers in the vagJsnerve which interferes with glandular secretory or

smooth muscle functions in which of the foliowingorgans a Bladder b Transverse coloiW c Descending colOO d Prostrate gland e Rectum

B

7) The spermatic cord includes all of the following contents except a Il ioinguinal nerve b Pampin iform plexus of veins c Vas deferens d Genitofemoral nerve

A

8 Which abdominal structure gives rise to the internal spermatic fascia (muscle) following the descent of testes in development

a External abdominal oblique aponeurosis b Transversalis fascia c Transversus abdominis muscle d Peritoneum e Internal abdominal oblique

B

9 Which abdominal structure gives rise to the tunica vaginalis fotlowing the descent of testes during development shy

a External abdominal oblique aponeurosis b Transversalis fascia c Transversus abdominis muscle d Peritoneum e Internal abdominal oblique

D

10) The lesser omentum is a peritoneal fold which is su bdivided into the a Hepatogastric and gastrosplenic ligaments b Hepatoduodenal and gastroomentalligaments c Hepatoduodenal and gastrosplenic ligaments d Hepatogastric and hepatoduoden9-jrj igaments

D

11) A posteriorly perforating ulcer in the pyloric antrum of the stomach is most likely to produce initiallocalized peritonitis or abcess formation in which ofthS fQllowing

a Great-sac - -- -

b Paracolic recess

c Omental bursa

d Right subphrenic space

c

The inferior mesenteric artery arises from the abdominal aorta ilm_ediill~y_J-Qs1eriQLto which of the foowing org~ns A-F~t~filie duodenum B Head of the pan~eis C Neck of the pandeas

D Second part of the duodenum

E Third part of the duooenum_shylaquoshy

shy

The correct answer is E The inferior mesenteric artery arises from the anterior surface of the aorta at the level of the third lumbar vertebra The third part of the duodenum crosses the midline at the level of the third lumbar vertebra and passes anterior to the aorta at the origin of the inferior mesenteric artery The

first part of the duodenum (choice A) lies horizontally to the right of the midline at the level of the first

lumbar vertebra The head of the pancreas (choice B) is to the right of the midline and extends from the

level of the first lumbar vertebra to the third lumbar vertebra It lies within the concavity of the

duodenum The neck of the pancreas (choice C) lies in the midline at the level of the first lumbar

vertebra It lies on the anterior surface of the aorta at the origin of the superior mesenteric artery The second part of the duodenum (choice D) lies vertically to the right of the midline and extends from the

level of the first lumbar vertebra to the level of the third lumbar vertebra

The left adrenaLvein drains directly into which of the following veins A Hemiazygos vein

B Inferior vena cavaee C Left renal veiri -

D Splenic vein

E Superior mesenteric vein

a

The correct answer is C The left adrenal vein and the left gonadal vein (either testicular or ovarian) drain into the left renal vein TheTeft renal vein t~ains intothe- inferior vena cava In contrast the right

adrenal ~~inandnght gonadal veindrai~ gLr~ctJy iQtoJhe iilferiQ[ Vencava -- -

ThehemTazygoS7ein- (~h-~i-~ A)~~c~i~es the venous drainage from the body wall on the left side of the

thorax and abdomen No visceral organs drain directly to the azygos or hemiazygos veins The inferior vena cava (choice B) receives the direct venous drainage from the right adrenal vein but not

the left adrenal vein Remember the inferior vena cava is on the right side of the abdomen The splenic

vein (choice D) receives the venous drainage from the spleen and part of the pancreas and stomach The splenic vein is part of the portal venous system

The superior mesenteric vein (choice E) receives venous drainage from much of the intestinal tract It is part of the portal venous system and joins with the splenic vein to form the portal vein

A 43-year-old man presents complaining of pain in the groin On examination his physician palpates a

bulge in the region of the superficial inguinal ring which he diagnoses as a direct inguinal hernia The hernial sac most likely

A is covered by all three layers of the spennatic fascia B passes medial to the inferior epi gastric artery

C passes medial to the lateral border of the rectus abdominis muscle

D passes posterior to the inguinal ligament E passes through the deep inguinal ring

The correct answer is B Direct inguinal hernias enter the inguinal canal by tearing through the posterior

wall of that structure The typical location for this type of hernia is through the inguinal triangle bounded

laterally by the inferior epigastric artery medially by the lateral border of the rectus abdominis and

inferiorly by the inguinal ligament Direct inguinal hernias pass medial to the inferior epigastric artery

whereas indirect inguinal hernias pass lateral to the inferior epigastric artery because the deep inguinal

ring is lateral to the artery Indirect inguinal hernias are covered by all three layers of the spermatic fascia (choice A) Direct inguinal hernias are covered by fewer than all three layers because the direct inguinal

hernia tears through one or more layers of fascia as it emerges though the abdominal wall The lateral

border of the rectus abdominis muscle (choice C) forms the medial border of the inguinal triangle All

inguinal hernias pass lateral to the rectus abdominis Femoral hernias pass posterior to the inguinal ligament (choice D) Inguinal hernias emerge through the superficial inguinal ring which is superior to the inguinal ligament Inguinal hernias that descend below the inguinal ligament pass anterior to the

ligament Indirect inguinal hernias pass through the deep inguinal ring (choice H) direct inguinal hernias

do not Both types of inguinal hernias pass through the superficial inguinal ring

During a gastric resection in a patient with stomach cancer a surgeon wants to remove the lesser

omentum because of tumor extension into it Which of the following structures lie in the free edge of the

l~~g omentum and consequently must be dissected out in order to be preserved

A Common bile duct cystic duct and hepatic artery 6

B Cystic duct hepatic artery and hepatic vein

e Hepatic vein and cystic duct

Portal vein common bile duct and hepatic artery

E Portal vein hepatic artery and hepatic vein

The correct answer is D The free edge of the lesser omentum contains three important structures the

common bile duct the hepatic artery and the portal vein Nei ther the cystic duct (choices A B and C) nor the hepatic vein (choices B C and E) lies in the free

edge of the lesser omentum

A 55-year-old male patient with chronic liver disease has portal hypertension To relieve the pressure in the portal system a porto-caval shunt is performed Which of the following veins may by anastomosed to

accomplish this porto-caval shunt A Left renal vein-left testicular veingt

B Right renal vein-right suprarenal vein I shy

e Splenic vein -left renal vein J

D Superior mesenteric vein-inferior mesenteric vein E Superior mesenteric vein-splenic vein

The correct answer is C The splenic vein drains directly into the portal vein The left renal vein drains

directly into the inferior vena cava Anastomosis of these veins would allow blood from the portal vein to

drain retrograde though the splenic vein into the renal vein and then into the inferior vena cava The left

renal vein (choice A) drains directly into the inferior vena cava The left testicular vein drains directly into

the left renal vein Thus these veins are already in communication and neither vein is part of the portal venous system The right renal vein (choice B) drains directly into the inferior vena cava The right

suprarenal vein also drains directly into the inferior vena cava Thus neither vein is part of the portal

venous system The superior mesenteric vein (choice D) drains directly into the portal vein The inferior

mesenteric vein drains into the splenic vein which then drains into the portal vein Thus neither vein is

part of the caval venous system The superior mesenteric vein (choice E) drains directly into the portal

vein The splenic vein also drains directly into the portal vein Thus neither vein is part of the caval

venous system

A 12 year old boy has fever vomiting and para-umbilical pain After examining the patient the doctor

makes an initial diagnosis of appendicitis Appendicular pain which is initially referred to the umbilicus goes to the dorsal root ganglion of

a TI b TI2 c L1 d T7

(e I TIO

A 59-year-old male undergoes a neurological examination which reveals that when the abdominal wall is

stroked the muscles of the abdominal wall of the side of the body stimulated failed to contract Other

neurological tests appeared normal The likely region affected includes

a CI - C5 spinal segments b C6 - TI c T2-TI ~T8-T12

e Ll- L5

The surgery done to relive portal hypertension is done by connecting two veins Which of the following veins would be suitable for connection

a Inferior vena cava and portal vein b Superior vena cava and portal vein c Splenic vein and right renal vein d Splenic vein and left renal vein e Superior mesenteric vein and Inferior vena cava

A mother brings her 3-week-old infant to the pediatric clinic reporting a new scrotal bulge that she found -~-

while changing a diaper yesterday The infant is afebrile Physical examination reveals a palpable mass in

the scrotum while in the standing position resolution of the mass in the supine position and no

transillumination of the scrotal sac What is the most likely diagnOSiS

a Cryptorchidism b Direct inguinal hernia c Hydrocele d Indirect inguinal hernia ~ e varicocele

The Vagal trunks enter the abdomen by passing through which of the following openings in the

diaphragm

a Right crus b Esophageal hiatus ~ c Vena caval hiatus d Aortic hiatus e Left crus

2 The anterior boundary of the epiploic foramen of Winslow is bounded by

a) First part of duodenum b) Lesser curvature of stomach c) Liver d) Hepato-duodenalligament v ~

3 The ilio-inguinal nerve is derived from

a TI2 ry b LI c L2 d L3 e L23

15 Surgically the structure used to suspend the kidney to the diaphragm is

a) Renal fascia b) True capsule c) Perinephric fat d) Paranephric fat

6 If there is portal obstruction because of carcinoma affecting the pancreas which of these of the

following signs would be present

a Caput medusae b Esophageal varices c Rectal varices c

d Pulmonary edema

7 In a sliding hernia the gastro-esophageal junction lies

a) At its normal position b) Below the normal position c) Above the normal position V d) None of the above

8 Which of the following structures is retroperi toneal

A transverse colon B spleen IJ2f6 C ileum D descending colon v r 1pound1111111

9 The renal angle is fonned lgtetween the 12th rib and ______ muscle

a Psoas major -middotshyb Erector spinae c Quadratus Iumborum d Diaphragm

10 The anterior structure at the hilum of the kidney is

a) Renal vein ~

b) Renal artery I middot~ I

c) Ureter d) Accessory renal artery

11 Because of origin of the muscle from the lateral one third of the inguinal ligament it

could not fonn the anterior wall of the inguinal ligament

a) External oblique b) Internal oblique c) Transversus abdominis_ d) Rectus abdominis

12 A large tumor mass impinges on the splenic artery and its branches as the artery pass out from below

the greater curvature of the stomach Branches o(which of the following arteries would most likely to

effected by the pressure on the splenic artery

a Left gastric b Left gastro-epipJoic c Right gastric d Right gastro-epipoloic e Short gastric_

13 A new born baby has projectile vomiting after each feeding It is determined that there is obstruction

of the digestive tract as a result of annular pancreas Annular pancreas is as a result of an abnormality in which of the following process

a Rotation of the dorsal pancreatic bud around the first part of duodenum b Rotation of the dorsal pancreatic bud around the second part of duodenum c Rotation of the dorsal pancreatic bud around the third part of duodenum d Rotation of the ventral pancreatic bud around the first part of duodenum y Rotation of the ventral pancreatic bud around the second part of duodenum

14 As the liver bud enters the ventral mesogastrium the region of the mesogastrium stretching from the

liver to the anterior abdominal wall is called

a Lesser Omentum b Greater Omentum ~ Falcifrom ligament d Lacunar ligament e Ligamentum teres of liver

16 A patient has absence of his 12th rib In such a patient if the doctor makes an incision to approach his

kidney mistaking the 11 th rib for the 12t he would end up injuring

Which of the following arteries is a direct branch of the gastroduodenal artery The

A right gastric artery

B left gastric artery

C inferior pancreaticoduodenal artery D left gastroepiploic artery

i E)right gastroepiploic artery --

E x pI a nation The right gastric artery is typically a branch of the proper hepatic artery The left gastric artery is a direct

branch of the celiac trunk The right and left gastric arteries anastomose along the lesser curvature of the

stomach The inferior pancreaticoduodenal artery is a branch of the superior mesenteric artery it

anastomoses with the superior pancreaticoduodenal in the head of the pancreas The left gastroepiploic

artery is a branch of the splenic artery it anastomoses with the right gastroepiploic artery along the greater

curvature of the stomach The right gastroepiploic artery is a branch of the gastroduodenal artery The

other branch of the gastroduodenal artery is the superior pancreaticoduodenal artery

Which of the following pairs of veins join together to form the portal vein The

A superior mesenteric vein and inferior mesenteric vein

B inferior mesenteric vein and splenic vein

C superior mesenteric vein and splenic vein

Ip)splenic vein and left gastric vein E superior mesenteric vein and left gastric vein

Explanation

The portal vein is formed behind the neck of the pancreas by the union of the superior mesenteric vein

and the splenic vein The inferior mesenteric vein drains into the splenic vein The left gastric vein drains

directly into the portal vein After the portal vein forms it enters the hepatoduodenalligament of the

lesser omentum to reach the liver The portal vein is the most posterior structure in the hepatoduodenal

ligament

At which of the following vertebral levels does the duodenum pass anterior to the aorta - _- shy

All ~

B L2 7~

CL3 I

~DL4

E L5

Explanation

The duodenum begins at the pyloric sphincter at the level of Ll The second (or descending) portion of

the duodenum is to the right of the aorta and extends inferiorly from the level of Ll to the level of L3 The third part of the duodenum crosses the aorta from the right side to the left side at the level of L3 The

fourth (ascending) portion of the duodenum extends from the level of LJ to the level of L2 The

duodenum ends at the duodenojejunal flexure The superior mesenteric artery passes anterior to the

duodenum as the duodenum passes anterior to the aorta The duodenum can be constricted at this level

In which of the following locations will perforation of the digestive tract result in the spilling of luminal

contents into the - lesser peritoneal sac

A Anterior wall of the second portion of the duodenum B Posterior wall of the second portion of the duodenum

C Anterior wall of the stomach

~Posterior wall of the stomach E Posterior wall of the transverse colon

Explanation

The posterior wall of the stomach is related to the lesser peritoneal sac The anterior wall of the stomach is related to the greater peritoneal sac The anterior wall of the second portion of the duodenum is related to the greater peritoneal sac The posterior wall of the second portion of the duodenum is related to the retroperitoneal space The posterior wall of the transverse colon is related to the greater peritoneal sac

The ureter lies against the anterior surface of which of the following muscles shyA Crus oftne diaphragm B Quadratus lumborum

0 Psoas major D Transversus abdominis

E Iliacus

Explanation The ureter exits the renal pelvis at about the level of vertebra L2 As it descends along the posterior abdominal wall it lies on the anterior surface of the psoas major The psoas major muscle arises from the bodies of the lower lumbar vertebrae The psoas major muscle is joined by the iliacus to fonn the

iliopsoas muscle The iliopsoas muscle then attaches to the lesser trochanter of the femur and is the major

flexor of the hip

As the right ureter passes the pelvic brim it lies against the anterior surface of which of the following

blood vessels

A Gonadal artery B Inferiorvena cava C Internal iliac artery

rJ- External Iliac artery

E Inferior mesenteric artery

Explanation

The ureter lies in the extraperitoneal space in the posterior abdominal wall Alter leaving the kidney it

passes inferiorly on the anterior surface of the psoas major muscle At the pelvic brim the ureter passes

into the pelvis At this point the common iliac artery is dividing into the external and iliac arteries The

ureter lies on the anterior surface of the external iliac artery immediately distal to the bifurcation This is a useful landmark for a surgeon to locate the ureter

When extravasated urine passes from the superficial perineal space into the anterior abdominal wall it is

found immediately deep to which of the following layers of the anterior abdominal wall

-ltScarpas fascia

B External oblique muscle

C Internal oblique muscle D Transversus abdominis muscle

E Transversalis fascia

Explanation

The superficial perineal space is bound by Colles fascia the fibrous portion of the superficial fascia This

layer of fascia is continuous with Scarpas fascia the fibrous portion of the superficial fascia of the anterior abdominal wall Therefore urine that is deep to Colles fascia will remain deep to Scarpa s fascia The urine will spread in the plane between Scarpas fascia and the external oblique layer

When a horseshoe kidney develops the ascent of the kidney is restricted by the A internal iliac artery B external Iliac artery

C common iliac artery

inferior mesenteric artery

E superior mesenteric artery

Explanation

A horseshoe kidney develops when the inferior poles of the to kidneys fuse together as they ascend into

the abdomen from the pelvis The first anterior midline vessel that is encountered by the horseshoe kidney

is the inferior mesenteric artery This artery prevents the kidney from continuing its ascent

The left testicular vein drains into which of the following veins

A Left internal iliac vein B Left common iliac vein

bflnferior vena cava D Left renal vein I

E Left internal pudendal vein

Explanation

The left testicular vein drains into the left renal vein The right testicular ~i~[~nsltjectlY into the

inferior vena cava This difference in venous drainage is believed to explain the greater incidence of

varicocele on the left side than on the right The venous drainage from the penis is to the internal vein

which then drains into the internal Iliac vein

The spinal nerve that provides cutaneous branches to the skin around the umbilicus is

A TS B TW-shy

C TI2

DL2 EtA

Explanation

The tenth intercostal nerve is the anterior ramus of the TIO spinal nerve After passing through the tenth

intercostal space the nerve continues forward in the anterolateral abdominal wall in the plane between

the internal oblique muscle and the transversus abdominis muscle In the abdominal wall the nerve innervates to the abdominal wall muscles as well as the skin and the parietal peritoneum The umbilicus is

a useful landmark for the region of distribution of the tenth thoracic nerve

The ligament of the vertebral column that resists its extension is the Aligamentum flavum

B supraspinous ligament

C posterior longitudinal ligament

D anterior longitudinal ligament

E interspinous ligament

Explanation

The ligaments of the vertebral column that resist flexion of the column include the supraspinous ligament

interspinous ligament ligamentum fiavum and posterior longitudinal ligament The ligament that resists

extension is the anterior longitudinal ligament This longitudinal ligament is very broad and strong It

covers the anterior and anterolateral surfaces of the vertebral bodies and the intervertebral disks In

addition to resisting extension the anterior longitudinal ligament provides reinforcement to the anterior

and anterolateral surfaces of the intervertebral disk The posterior longitudinal ligament is relatively

narrow and covers the posterior surface of the vertebral bodies and the intervertebral disks This ligament

reinforces the posterior surface of the disk The posterolateral surface of the disk is not reinforced and it

is through this region that herniation of the nucleus pulposus usually occurs

A patient presents with epigastric and right upper quadrant pain The pain is most intense 2-4 hours after

eating and is reduced by the ingestion of antacids The patient states that he has passed black tarry stools

(melena) within the last week Fiberoptic endoscopy reveals a yellowish crater surrounded by a rim of

erythema that is 3 cm distal to the pylorus Accordingly an ulcer has been identified in the patients

A fundus

B antrum

C duodenum

D jejunum

E ileum

A number of physiologic genetic and other factors increase the risk of gastric (and duodenal) peptic

ulcers The evidence that H pylori plays a principle role is compelling Smoking and caffeine are known to adversely affect the morbidity mortality and healing rates of peptic ulcers In general first-degree

relatives of peptic ulcer patients as well as males have a threefold to fourfold increased risk of developing this disorder Paradoxically in gastric ulcer disease acid secretion is not elevated It is possible that

excess secreted hydrogen ion is reabsorbed across the injured gastric mucosa In general a defect in gastric mucosal defense is the more important local physiologic

A patient presents with symptoms of duodenal obstruction caused by an annular pancreas Annular pancreas is caused by

A rotation of the dorsal pancreatic bud into the ventral mesentery B rotation of the ventral pancreatic bud into the dorsal mesentery

fJ failure of the major and minor pancreatic ducts to fuse ~ ~ cleavage of the ventral pancreatic bud and rotation of the two portions in opposite directions around -the duodenum E formation of one pancreatic bud instead of two

Explanation Normally the ventral pancreatic bud rotates around the gut tube to reach the dorsal pancreatic bud The two buds fuse to form a single pancreas and the distal portions of the two ducts fuse The ventral pancreatic bud forms the inferior portion of the head of the pancreas the uncinate process and the major pancreatic duct (of Wirsung) The dorsal pancreatic bud forms the superior part of the head the neck body and tail and the minor pancreatic duct (of Santorini) Annular pancreas is the result of the ventral pancreatic bud dividing into two portions before it rotates into the dorsal mesentery Each portion rotates in opposite directions to get to the dorsal mesentery thus encircling the duodenum The presence of annular pancreas can constrict the duodenum thus obstructing its lumen

In n _ phranlc----

Gon ~l ----_1 Lum bltano

~~--- CornmQ1t bull ac

+-~4--- lnlllirnaJ ilic

xtem iliac

OBJECTIVE - Identify the blood supply to each of the structures listed in the table on the previous page

Ill give you a head start

FOREGUT - Supplied bV Celiac Tru nk (T12)

Proper hepatic

GastiooUod 13Jafter

1nferlor pancreaticoduodenal artery

Common epatlc

Lett gas ric iiirtery

Spfen artery

shy Gastroepiphgtic artery

~ Superior mesenteric 8rtfry

~

1 Esophagus is a derivative of the foregut so its blood supply originates from the celiac trunk

(T12) The predominant blood supply to abdominal portion of the esophagus is the Esophageal

A (Branch of L Gastric) The venous drainage of the esophagus is particularly important because

it is 1 of 3 clinically relevant sites of Portal Caval anastamoses The Portal Esophageal Vein

meets the Caval Azygos System Persistent bleeding manifests as Esophageal Varices - a fata I

condition

2 The Stomach is also a derivative of the foregut has EXTENSIVE blood supply and is very high

yield on anatomy exams The lesser curvature is supplied superiorly by the L Gastric A (1 of 3

major branches ofthe Celiac trunk) and inferiorly by the R Gastric A ( a branch ofthe proper

Hepatic A) The greater curvature is supplied superiorly by the L Gastroepiploic A (a major

branch of the splenic A) and inferiorly by the R Gastroepiploic A

The Short Gastric arteries (branches of Splenic Artery) supply the fundus of the stomach and

are referred to as EIID ARTERIES because they have no collateral blood supply Therefore if the

splenic artery were occluded (ex - increased pressure in the ommental bursa) - there would be

ischemia to the fundus of the stomach Venous drainage of the stomach is extensive via various

veins lead ing to the portal system Posterior to the stomach the IMV joins the splenic V which

joins the SMV to form the PORTAL VEIN ADAMS

3 Duodenum blood supply has high clinical relevance because it is the junction of the foregut and

midgut and therefore is the site of anastamoses between branches ofthe Celiac Trunk (main

foregut artery) and the Superior Messenteric Artery (main midgut artery) The Proper hepatic

artery gives off the gastroduodenal artery which travels behind the 1st part of the duodenum

This point has high clin ical relevance because duodenal ulcers are very common and a posterior

rupture of the 1st part of the duodenum could rupture the gastroduodenal artery causing

traumatic abdominal bleeding The Gastroduodenal artery first gives off the R Gastroepiploic A

(mentioned above) and proceeds as the Superior pancreatico duodenal artery (supplies the

pancreas and duodenum) which anastamoses with the inferior pancreatico duodenal A (branch

of the SMA) This is the junction of foregut and midgut and occurs near the opening of the

bil iary system into the duodenum (ampula of vater) Portal venous drainage here is responsible

for delivering nutrients from digestion to the liver for metabolism Appreciate that the Superior

mesenteric artery (artery of the midgut) branches from the aorta at Ll travels posterior to the

pancreas than moves anteriorly (at the jxn of the pancreatic headbody) and comes over the

3rd4th part of the duodenum Tumor of the head of the pancreas can compress the SMA

4 Jiver blood supply is via the common hepatic artery (major branch of the cel iac trunk) The

common hepatiC becomes the proper hepatic gives off the R gastric A and the Gastroduodenal

A and then joins the common bile duct and the portal vein in the portal triad Clinical- if a

patient were bleeding from the hepatic A a surgeon can stick his fingers in the epiplOic foramen

and squeeze the free edge of the hepatoduodenalligament in order to stop bleeding to the

area Please note that the hepatic a branches into Rand L hepatic A The Right hepatic artery

gives off the cystic artery which supplies the gallbladder Afferent venous supply is via the

Portal vein which is bringing nutrient rich blood to the liver After metabolism takes place

venous blood leaves the liver through the hepatic veins into the IVC PLEASE UNDERSTAND THE

RELATIONSHIP OF THESE STRUCTURES - ADAMSNETIERSNH Etc

5 Pancreas - Head is supplied via the superior and inferior pancreaticoduodenal arteries

(mentioned above) The tail (situated towards the hilum of the spleen) is supplied via the

pancreatic branches of the splenic artery (END ARTERIES) This blood supply is very important

because the endocrine Alpha and Beta Cells from the pancreatic islets of lagerhans are located

towards the tail This is where Insulin and Glucagon is released to the blood

Now complete this for mid and hindgut structures Make sure to note clinically relevant arterial

anastomoses as well as portal caval anastomoses FYI Appendix blood supply SMA + IMA

anastamoses marginal artery Portalcaval rectal veins fhemmorhoids) and periumbilical caput

medusa are high yield THE BUTT THE GUT and THE CAPUT

Abdominal Development

Liver

Ij1f

II wall b

oh liN ~ VltJrti n be- bull

Pancreas

Secondary Retroperitonealization e I~tl r 1 a v-mtrai m ellter

Rotations of the Gut I i Ij (lIl1UtIJ f~ l r tilt

()l td 10 me l-ft and he v

--~--- -~ -~-~

i

I AolaijonjoI~guf I

STOMACH BED (IDENTIFY IN ADAMS)- the structures posterior to the ommental bursa which

support the stomach in the supine position

Abdomnal JQrUI

Splnic vein

OmQ-oul tv~ ) O(s(Jroa)

Lojt(r o m nturrt (hpJtodu o d~n31 Hid

Gadrl)SplerH (g3stroll~nal) IIgam~nt

hiad h~~atogrtricent IIQdmiddotcrt~)

Lt Dome of Diaphragm (why left Look this up in Adams)

Spleen (What is the blood supply)

Left Kidney (What is the blood supply - AND how is it different from the R kidney)

Suprarenal Gland (What is the Arterial AND Venous Blood supply - how are they different)

Pancreas (How does supply differ from Head to Tail What is the SMA Relationship)

Transverse Mesocolon

liver - ADAMSWET - Make sure you look at the liver in wet lab

Left triangular nl1am~nt

ComoaDj ligamnt

Erophg~1 impre$ioo

Hepatio veins

In1erior -ifena middotr3)Ia

Fibrous appendix o-t

live

impr~j on

Heprorendl p~rtion of Q)(Qllary ligament

Righllri~n9ul r 1I~met

(Common) bile quol

Gr)mmCtr~ hepatic dlJct

Ccentic duct

Duodenal impression

GaJdate p-fr)~S

Hepatic artgtrl prop-f iiiiila - Faloiform ligament

_ - shy Round ligamen liver

~--F-- CoJio imprgt-ssi-on

Prta heptis

Identify the lobes impressions and embryonic remnants associated with the liver

Caudate Lobe Quadrate Lobe Right Lobe Left Lobe Round ligament Falciform Ligament

Ligamentum Venosum (what is its fxn in embryonic life) Hepatic Veins (NOT PART OF THE

PORTAL TRIAD) IVC PORTAL TRIAD - Contents relationship cross section etc Know the

Galbladder relationship to the lobes of the liver

Biliary Duct System - Make sure you understand the sequence of these structures - BE ABLE TO

DRAW A FLOW CHART

TPVd i

t

I t

1 __ Cm-(r

patk GlJet

I

J

Clinical = JAUNDICE is caused by anything that prevents delivery of bile to intestine Tumor of the

head of the pancreas Stones etc Patient will have pale stools and yellowish colored mucus

membranes

Clinical- Any scenario that tells you the patient has BILLOUS VOMIT means that the obstruction to

the flow of digestive contents is after the Ampulla of Vater (Site of Entry of Billiary system to the

duodenum) - ie Duodenal Atresia

Spleen -located posterior to the mid axillary line between ribs 9 and 11 Make sure you know that

the 10th rib is the main axis of the spleen and this organ is susceptible to injury (stab wound errant

thoracoce ntesis etc)

The spleen is derived from mesodermal cells - NOT THE GUT TUBE

The spleen rests on the left colic flexure associates with the tail of the pancreas Know the

structures entering the Hilum of the spleen

Sh rt O~-t~ic 1 0(0 10 rtiltSPIric Iloa nt

(cut)

Peritoneum - similar concept to Pleura - think of a fist in a balloon

Visceral Peritoneum - Layer of balloon touching your fist

Parietal Peritoneum - Layer of balloon not touching your fist

Your fist represents the organ your wrist is the hilum and your arm contains the blood supply

entering the organ

Appreciate that there will never be organs in the peritoneal cavity - rather these organs invaginate

the cavity Kaplan videos

RULES OF NOMENCLATUREshy

1 Organ completely surrounded by peritoneum - peritoneal organ

2 Organ partially surrounded by peritoneum- Retroperitoneal

3 Peritoneum surrounding peritoneal organ is VISCERAL peritoneum

4 Peritoneum surrounding retroperitoneal organ is PARIETAL peritoneum

5 Peritoneum connecting visceral to parietal is called messentary 2 messentaries in the

gut Dorsal (to the gut tube) and ventral (to the gut tube) messentary

Aorta is in Retro peritoneal position - but blood must reach peritoneal position - vessels travel through

messentary All peritoneal organs will have blood supply reaching through messentary

-Mesentery is a 2 layer peritoneum with a neurovascular communication between body wall and organ

- Ligament connects one organ with another or to the abdominal wall (Ommentum = ligament)

lesser Ommentum (attach lesser curvature of stomach and duodenum to liver) =Hepatoduodenal

Ligament and Hepatogastric Ligament

Has a Superior and Inferior Recess (Accumulation of Fluid in Ascites)

Communicates with the greater sac through the epiplic foramen (what structures pass through

this foramen)

Boundaries - you must be able to visualize this

o Anterior - stomach

o Posterior - parietal peritoneum pancreas

o Superior - superior recess (bw diaphragm and coronary ligament)

o Inferior -Inferior recess (bw layers or greater momentum

Greater Ommentum (attach greater curvature of stomach) Gastrophrenic ligament Gastrosplenic

ligament gastrocolic ligament

The greater omentum is the largest peritoneal fold It consists of a double sheet of peritoneum folded on itself so that it is made up of four layers The two layers which descend from the greater curvature of the stomach and commencement of the duodenum pass in front of the small intestines sometimes as low down as the pelvis they then turn upon themselves and ascend again as far as the transverse colon where they separate and enclose that part of the intestine

ABDOMINAL PAIN

Parietal Peritoneum - supplied by same vasculature lymphatics and nerves supplying body wall it

lines and diaphragm Sensitive to pain pressure heat cold well localized

Visceral Peritoneum - supplied by same vasculature lymphatics and somatic nerve of organ it covers

Insensitive to touch heat cold and laceration - referred to dermatome of spinal ganglia providing

sensory fibers Where does appendicitis refer to

Foregut pain - epigastric area (ie - cholycystitis)

Midgut pain - periumbilical area (ie - appendicitis)

Hindgut Pain - suprapubic area (ie - diverticulitis)

Extra ImagesConcepts

ll~_____-

FalifCtrm ligament oind r~ud ligamet f Ilver

Blood from splenio gastriC and inferiof rne$e-rteri v~ins

Ca-I tributaries

Lett gastrio Ifein

Posterior superior pan~reatioodul)denal vaihS

Lott gamo-om~nlal (9aropip lomiddotic) -in

Poq_~ tjol imerl-9-r panCJertlcorllJod-nal veiopound --amp----I- - ~J Right grtr~-omntal

Anwrior interi (gartroepiploic) Jjn

pan euaii cod vl)denal veins middot Inf~Ji (t r mesentric vein

Miqdle (olic vein

Right cl)licvein Sigmoid and rectosigml)id (ei ns

IhH)Collc(~io

--- Mi~dl laquooLJl gtjrltgt

PoM ca vl1 illasto)moses -----shyampoptoageal 2 Paraumbilie-lt11 Inferi or Fectal vei ns

3 Recial 4 REuoperHonea1

Know how the Portal vein is formed I 4 sites of portal caval anastamoses and 1 clinical shunt

Col li t ltt-~ otTl~tI ~nj pc~ 1lt1 turJoG

Ltf 14i1 tImiddot~ artoftl9 on tj phtAt$

L-oftqf 4t t~r 1=laquoIran d 1 bull shy~p l ci rj o fOOOts

Nerves follow the arteries - appreciate the splanchnic nervous system I

Uet~ric branch of left ~nal art

Ureterie branch of righi renal artelY

Left Zld lumbar in and co mlTlunication to as)erdin9 lumbar l(~in Hi ~ht tEZ1~~t~ t3r j t itn ~ nJ l1t- rlnd lfe i r1

Inferior me5nteri~ artery

Notice that the right testicular vein drains directly into the IVC and the right testicular artery drains

directly into the aorta However the left testicular vein drains into the L renal vein at a right angleshy

reason left testicle is lower and more susceptible to varicocele (bag of worms)

Also notice that the left renal vein has a longer course because the IVC is on the right side whereas

the right renal artery has a longer course because the aorta is on the left side

Appreciate the anterior to posterior relationship of structures in the hilum of the kidney - VAP - Vein

Artery Renal Pelvis (Ureter)

11____ __ L_ L_ n VJ __ _ _ t_L I I_ _ L __ L_ I -pound1 bull LI_~-I ____

Posterior View of Head of Pancreas in ( of Duodenum

Celiao hunk

Co mmon ~L~jJth art~ry

GastNduQdonal artrf (partilly in phantn)

P1)Sterior $Up~Jior panCflaticuduodfmal art~r~t

(Co mm on) bile duct

middot~1t~~t-1l---~-~- Right gshomiddotomental (gastoe plp lolc) 3rte (phantomost)

Grener paocre atic art-ry

1n1~rjor pancr-iatlc artery

Jtrifll supejo r pal)oreailcento)dJodenal artr1 (phantom)

Anastomotlo branch

POostetlor bJanch of jo f~ri of pan-reatir(lduodensl drttnj

Anterio r branch of i flferior palcreati~)duodenal art~(phan1om)

Notice the extensive blood supply to the pancreas and duodenum via the branches of the celiac trunk

Notice collateral supply from SMA branches - makes sense bc this is the jxn of foregutmidgut

Identify the vessels in this arteriogram

Hiltid i)f N~ck oi B)dvof Tail 01 pa nereas pan cent~as P-nmiddot-reas panCtCas

I nferie v~na cava

jHept1iic p(lrlai v~in

Port1 tnd H~pti lt a ftH prol

Comm on) bll duct

Ouodtnum

~ft colic (sio)Atta~ hmtrlt jt~xJr-ofha~elSe

muo(IIQn

Right ~lIc (h~j)tic)

il~gtture

In1triol m~oten lIein (rttr op~ritoMdO

SlJp efl or mes~n~fiC amrV and lipln

KNOW YOUR NEIGHBORHOOD

Questions

vVhiJh structure supplied by a bnmdlof the cclia( artery is not derivcd from foregut LemCJUCrITI

(A) Head of the pancte-a5

CD) Pyloric duolenum

Cystkduct

( Liver hepatocyt~~

~F) Body of the spleen

An infant presents with an omrhaJucele at birth -hi oJ the [oHm illg applies to his cM1-dition

(A) It is 31so seen ill p4titnts with aganghonic megacolon

(11) ft reuirs from a fal1ure of resorption of theviteUine d let

(C) It results from herniation at the-site of regression of the right umbilk vein

DJ It is caustd by faihtrc of recanalization of the midgut part of the duodenum

~ It ill camioo by a failuIt vf the midgul to return to the abGQminal uity after herniashytion in-n the urnbilk s l stalk

Ot er than the spleen occlusion Cif the spit-Ilk artery at its odgin wm most likely affect die blood supply to jllch st cnud

(A) Jejunum

(B) Body of th pal1~lltas

(C) LeSStT Cllmiddotlaturc of tl )toma-ch

(D Duodenum dista to the entrance of the Ornmou bile duct

E Fundus of the stomach

A 38-yeu-old batL~er with a history of heartburn suddenly experiences excluciating pain in the (plgastric region of th~ abdomeu SurgCry is perf~rme immediard y upon admisshysion to the 1IlcrgCJliy tuomh~re i~ evidence uf a ruptured ulcer in the posterior waU of the stomach Vhere will a surgeon first fi nd the stomach contenlSf

A) Greater p4ritoneal sac

rB) Cul~de-s~c of Douglas (--

C Omental bursa ~

--D) Paracolic gutter

rEj Between -he panttal perimltum and the posterior body wal1

At birth an infant presents with a st()ma~ rb~tbas~njJled jfltotb~diaplfagru 1A1ltre is the defect thatresulied iiitJle heini~t()n shy~tsophagealbiatus

7 - rH-- Hiatus for the inferior vena cava

( Pleuroperitoneal membrane -(0) Septum transvcrsum

(E) Right Crlt~

An infant born with DOVv7l syndrome presents with bili()u~ vomiting Ahat congenital defect does the infant have

(A) Pyloric stenosis

(B) Meckel diverticulum C) Ornphaloce1e

(D) Gastroschisis

( ~ ) Duodenal atresia y A patient with cirrhosis of the liver presents with ~ bacalvaricestnlreased retrograde pressure in which veins caused the varices

(A) Paraumuilical

(B) Splenic

(ct AzygltJus

(15))G~trk ( (-F) Superior mesemeric

A htaltby 3-year~old male patient experiences a hernial sa protruding from the anterior abdominal wall about halfway between me anterior superior ilia spine and the pubk tuberde Pulsations of al1 artery are palpated medial to the protrusion site through the abdominal walL Which layer of the anterior abdominal wall will first be traversed by the

1hctma

fA) Rectus sheath (B) External oblique aponeurosis

(C) Inguinal ligament

lD) Transversalis fusda

(E) Cremasteric fa~cia

After 5urgi(aj ffpair of a hernia the patient tXperienccs mtmlgtness in the skin on the anteshyrior aspect of the S(Totum_ Vhaf nerve may have been lesioned during thehemiorrhaphy

(A) Femoral

(B) Obturator

(C) Ilioinguinal

(D) lliohypogastrk

(E) Pudendal

A 23~year-LJld female secretary il1 good health ~-uddcn1) doubles over with pain in the a ea of the 1JmbRicu$ Sbe feels vartn and ltneasy and has no appetite That night the pain seems to have mQved to the tower right abdominal regjol1 and she calls her family doctor who then arranges for an ambulance to pk-k her up and take her to the hospitaL Wh ell ntn~ perceived in the area of the urnbilirus most Hkely carried lhe pairfu I sensations into the eNS

tA) Vagus nerves I~

V B)

) Lessersplanchnk nerves

tC) Pudendal nerves

(D) lIiohpogastrk nerves

(E) Greater splam ic l erves

A CT reveals carcinoma in the bOod of the ancreas Vhich blood vessel trut ourses ----~- - -bull ------ --shy

immediately poftterior to the body ofthe pancreas is the m~t likely to be oompressed

(A) Splenk artery

(B) Abdominal aorta (C) Portal vein

(1) Splenic vein

(E) Renal vein

A patient has a penrln1l1ng uker of the posterior wall ot the br~l part ot the (lUooenmn llkh blood vessel is subject to erosion

(A) Common hepatic artery

(B) Gastroouodenal artery

(C) Proper hevatic artery

(D) Celiac artery

(E) Anterior inferior 11amrelltlcoduodcnal attery

Your patient has been diagnosed -ith a carcinoma locallted to the head and l~e(k of the pancreas Another clinical sign would be

A esophageal varices

(8) hemorrhoids

C) a caput medusa

(D) increased pra Teuro n th~ hepatic veins

(E) enlarged right supra lavkular lymph nodes

Wltkh of the foUowing structures develops in the ventral mesentery

(A) Spleen

(B) Jeiunum (C) Head of1ht pancreas (D) Transverse colon (E) Stomach

ti l Uw ~ littwin~ f( S-t lil oai Imdge ~ hi(h or tbt la~)d J truetur tgt liJ llntn nl) he hl p UC iJd [IIi ell

c o

A) drains Ie tht infCrior a La aI

R t middot~nfl0 ~ill to th~ lunlgtn of h i dtlndCrlllfH

(e) m t bull JiJattd on tl l J n T ~H

D ) sup Lc O VSlt I Hlid bhtu l 1 li - -I un oid

( ) U~tpli(t tr j middottUh~ 1 v(( b~nt rfK n1ilc~Zm

ANSWERS AND EXPLANATIONS

Answer E The spleen is t hlttnopodicand lymph organ demlted from mesoderm

Answ~ R Al1 tlmphalocele is caused by it failure of the nlidgut to return to the ahdomir nat cavity after herniation into the umbiliau Stalk Choices Aand D maybe seen in infants with Down syndrome choice D ~s the specific CBuse ofduudcnal JtiCSitt Choice C is (ile cause of gclstrosbisis and Choice B nsults iu a Meurolktldivertku1-tlB

Answer B The fundus ofthe stomach is suppHed by soort gastric brunches of the splenic altery The splenic artery supplies the body and tail of the pancreas part of the greater curvature of the sttmla(h and the spleen Te jejunum part of the head of the pancreas and tht~ duodenum distal to the entrance of the commOll bile duct are supplied by the superior mesenterk artery clll~l ~be less r ctlt1ature cmd the pylQric antrum are supplied by the right and lei gastric art(ries

AnSWftt C Tbeomental bursa or lesser ~ritoneaj sac lies direcdy posterior to the proxshyimal part of the duodeTtlm and the stomach and would be the first site where stomach contents ~Ott1d be fpoundluncL

Answer C A defect in a llleuropcritoneal membrane (uswlly the left) is the typical site of i1 cc-ngenitlI diilphragluatic hemia llere the membr4ne fails to dose ()pound( of the perishycCirdiopcritulleal canals

Answer E DuoJenal atresia and aganglionic megacoion are congwitaI defects S~Il in patients with Dowmiddotnsyndrome

Answer D RulaTgemt~llt of and retrograde flow in g~lstrk vel_ns in particlJl~r the kft gas~ tricveins dilates the capillary bed in rhe wall of the esophagus in (ases of porta yper~

tension Blood flow would increase in and dilampte tribntarkgts of the (lZygOUS vein on the other side of the capiUary bed but flow in this vein is in the typical direction t()ward the superior vena cava Paraumbiii(ltU vein eilgorgement contributes to a caput medusH Splenic ~nlargement might prc~nt with 5plcnonlegaly and balt-kflow in to tlu superior m~~ntclic vein occurs but is asymptomatic

Answer D The patient hagt an indirect inguinal hernia whi~h emerges from the antt-rior abdominal wall through the deep inguinltilling Theeep ring is a fault in the transv~rshysaUs fascia this I~yer wiIJ be penetrated first by the hernia

An~Wer C The ilioinguinal nenc which provides sens~llion to the lnedlal thigh ltmclanteshytior SClotunl pass~lt th rough the 5uperfh_ial inguinal ring ind $subject to inj i1T) becaus-e

it is in the operatitm Held of the erniorrhapny

Auswer B The leMHr splanchnic nerves are sympathdic nerVlts that carry viscera l sensashytlltgtrogt ftom illtllt1m~d ()J stietched gust (itinteitinal ~tructures (in this case the pprndix) into tnt eNS Lesser splanchnic ntTYcsarisc from thmiddot T9--T12 spinal cord segments lt1nd provide sympathetic innenation tD rnidgut siruc1ures whiCh include CLe app~JldD Viscera] Pain arising from affecLed Inidgut ampt 1C1ure is referred over the same dl- matorne~ of spinal segrnertts v-hich provide the sympathetic Innervation n this G1SC of appendicitis the invohen~n t of the ltire) of t e unlhHku indud s the T 10 dermatome

Answer B Of the five choices onty the dscending olon is retroperiton~al aldwould be a lik ~ ( choice to be seen immediately a(~jilcent to t11e posterior abdominal middotn~L

Amwen D The SpltftlC ~-ein ourses posterior to the body of the panneas m its way tt drain into the superior mCSfttltlri( vein

Answcr B TILt glstrodllolticnal artery 1 direct hIamh of the comrootl hepatic artery courses immediately pt))iwri() to the duodenum and is slbject to erosion

Answer B Carcinoma of th pan middott3S in the 1tilt1 may compreampgt the portltil vein at irs orishygill The poTtai vcin is fomled when the splenic vein jQiaswith tfie superior meStllt eric vein The inferiot mesenteric vein joins the ~plenjc vein just priOT to tlli~ point at which the splenic joins the superior Jlleit1ltcri( vein Increescd venous presslu in the inferior mesenteric vein is a cause of emo hoid~

Answer C The- velltral pancreas wilich forms most of the head of the p ~ncr as develops in the ventral mes(ntery as antutgrowth of the hepatic diverticulum Th~ hepatic divershyticulull induding the biIJary appa~atus develops in tbe ventral mesentery of the foregut

Answer~ A The superior mesenteric ~in joins with the spienkvein to form the hepatic portal vciu

Answer D The structure at gttlK is the proper hepatic artery~ whkh suppUesoxygenated b middotood to the liver

MAKE SURE YOU KNOW the diff bw Rectus Sheath above and below the arcuate line

ABOVE

Aponeurosis of xiiltmal obllque musclo

Extemll f)biquw musde

Reotln ilbdomlnls musole S~in

Internal 9bliquQ mY~QI

AponeUfOsi$ of hJH$V~~S Lir9a a lb lbdolTlin~ musolo Tri OJV6 rUi

atldomlnis mUS(loe

Sub cutanlilous tiue (tatty ye r)

BElOW

A POrl lJfosis 01 etemal oblique muscle

Aponeul~)sis 01 Internal oblique mU$cl~

Anteriol lay~ of r~ltdus st~ath EXttom1 oblique rnu$cll

Rectus Jbdominis muscle Intoernal Aponeurc-sis of tra~fersU$ oblique muscle-

at-domlnis muscentl ~ Skio

Tra nsvitSus abdomioLs ml)ZClt

TralSVersaHs fascia Medial umQil iegtt1 1i9Jment -and folj

Uldchus Peritoneum (ir median Umbilj~al Suboutane ous

Extraprftone 11ascia

Ymbilimiddot~1 fold)

preu9poundiea1 fascia

tissue (fatty 4nd m~mbr3n(iUS layers)

o Above the arcuate line (A horizontal line 13 of the distance bw the umbilicus and the

pubic symphysis) -10 Aponeurosis divides into an AntPost Laminae

o The Ant Laminae joins EO and Post Laminae joins Trans Abdominis = Ant and Post

RECTUS SHEATH respectively

o BElOW the arcuate line - all 3 aponeurosis join ANTERIOR to rectus muscle to meet its

counterpart in the midline (linea Alba)

o Take away Msg - The abdomen is devoid of a posterior rectus sheath below the

arcuate line and is therefore more vulnerable to herniasinjuries

Question - A physician makes a deep incision in the patients midline immediately superior to

the pubic symphysis which of the following layers is his knife least likely to pass

Rectus Abdominis External Oblique Ant Rectus Sheath Posterior Rectus Sheath All of the

Above

Answer - All of the above None of the other answer choices are midline structures -LINEA

ALBA

Linea Alba has very poor blood supply - doesnt heal well after surgery Therefore this is a

common site for incisional hernias

a Spleen b Transverse colon c Descending colon d Stomach e Pleura

17 Meckels diverticulum is normally found 2 feet proximal from the

a Pyloric sphincter b Lower esophageal sphincter c Ileo-cecal valve d Middle valve of Huston e Anal valve

18 Ulcer in the posterior wall of the first part of the duodenum would erode ___ artery and would cause bleeding

a Left gastric b Right gastric c Hepatic artery proper d Gastroduodenal artery e Middle colic artery

19 An inflamed appendix is identified by a surgeon on the operation table by noting

a The appendicies epiploicae b The convergence of tenia c The artery of Drummond d The mesocolon e The mesosalphinx

20 The nerve which emerges through the psoas major is

a Femoral b Ilio-inguinal c Ilio-hypogastric d Pudendal e Subcostal

21 The right gonadal vein drains into the

a Azygos b Hemiazygos c Inferior Vena Cava d Right renal vein e Left renal vein

22 The hepatocytes in the liver is derived from

a Ectoderm b Endoderm c Mesoderm

d Neural ectoderm

23 Abscess in the lumbar vertebrae due to tuberculosis would spread to the adjacent muscle which is

a Psoas Major b Iliacus c Quadratus lumborum d Tranversus Abdominis

24 The anterior wall of the inguinal canal is formed by

a External oblique and transverses abdominis b External oblique and fascia transversalis c Internal oblique and external oblique d Internal oblique and transverses abdominis e Fascia transversalis and peritoneum

Meckels diverticulum is a result of which of the following developmental abnormalities shy

A Failure of the vitelline duct to close

B Failure of the herniated intestinal loop to retract into the abdomen

C Failure of the urachus to close

D Failure of the midgut to rotate

E Failure of the hepatic duct to close

Explanation

Meckels diverticulum is a result of the persistence of the proximal part of the vitelline duct This

diverticulum is usually found about 2 feet proximal to the ileocecal junction and is usually about 2 inches

long It is present in about 2 of the popUlation It may be the site of ectopic pancreatic tissue or gastric

mucosa and may develop inflammatory processes and ulcerations Acute Meckels diverticulitis

simulates appendicitis

Which of the following veins carries blood from the esophagus to the portal vein The

A right gastric vein

B left gastric vein c splenic vein D azygos vein

E left gastroepiploic vein

Explanation

The left gastric vein a direct branch of the portal vein drains blood from the lesser curvature of the

stomach and the inferior portion of the esophagus Because branches of the portal vein do not have

valves blood can flow in a retrograde path when there is an obstruction to flow through the portal system or liveL Rlooci Cln then flow from the nortl] vein thr()1Ph the left PRstric vein to the esonhlPlIS lno

through venous communications within the submucosa of the esophagus to esophageal veins that drain

into the azygos vein The increase in blood flow through the esophageal submucosal veins results in esophageal varices

On the posterior wall of the abdomen the celiac ganglion A contains cell bodies of postganglionic parasympathetic neurons B is synapsed upon by neurons in the posterior vagal trunk C is synapsed upon by neurons in the greater splanchnic nerve D contains sensory cell bodies of lumbar spinal nerves E contains cell bodies of neurons that cause an increase in the rate of peristasis

Explanation The celiac ganglion is one of the preaortic ganglia of the sympathetic nervous system It contains cell bodies of postganglionic sympathetic neurons The sympathetic splanchnic nerves contain preganglionic sympathetic neurons that pass through the sympathetic chain without synapsing These splanchnic nerves go to the preaortic ganglia to synapse The greater splanchnic nerve contains preganglionic neurons from spinal cord segments T5-T9 This nerve synapses in the celiac ganglion The nerve fibers in the vagal trunks are preganglionic parasympathetic fibers that go to the walls of the organs that they will innervate and synapse on postganglionic parasympathetic neurons in the walls of those organs Cell bodies of sensory neurons in the abdomen are found in the dorsal root ganglia or the sensory ganglia of the vagus nerve Sympathetic innervation decreases the rate of peristalsis parasympathetic innervation increases the rate of peristalsis

Which of the following pairs of arteries will allow blood to bypass an occlusion of the celiac trunk

A Left gastric artery-right gastric artery

B Left gastroepiploic artery-right gastroepiploic artery

C Superior pancreaticoduodenal artery-inferior pancreaticoduodenal artery

D Splenic artery-common hepatic artery

E Left gastric artery - proper hepatic artery

Explanation The anastoOlosis of a branch of the celiac trunk and a branch of the superior mesenteric artery will

provide collateral circulation around an occlusion of the celiac trunk Each of the other choices pair

branches of the celiac trunk therefore these will not provide collateral flow around the obstruction of the

celiac trunk The left gastric splenic and common hepatic arteries are direct branches of the celiac trunk

The right gastric artery is a branch of the proper hepatic artery which is a branch of the common hepatic artery The left gastroepiploic artery is a branch of the splenic artery The right gastroepiploic artery is a

branch of the gastroduodenal artery whlch is a branch of the common hepatic artery

Which of the following organs has appendices epiploica The

A sigmoid colon

Bjejunum

C duodenum

D stomach E esophagus

Explanation Appendices epiploica are characteristic of the colon Appendices epiploica are subserosal accumulations

of fat None of the organs of the gastrointestinal tract has appendices epiploica except the colon

Page 18: Chirag's Abdomen Review

d Medial fibers of the inguinal ligament

B

6) A 25 year old male is brought in to the ER after being involved in a car accident in which he received a crushed internal injury in his abdomen Examination reveals a lesion of parasympathetic fibers in the vagJsnerve which interferes with glandular secretory or

smooth muscle functions in which of the foliowingorgans a Bladder b Transverse coloiW c Descending colOO d Prostrate gland e Rectum

B

7) The spermatic cord includes all of the following contents except a Il ioinguinal nerve b Pampin iform plexus of veins c Vas deferens d Genitofemoral nerve

A

8 Which abdominal structure gives rise to the internal spermatic fascia (muscle) following the descent of testes in development

a External abdominal oblique aponeurosis b Transversalis fascia c Transversus abdominis muscle d Peritoneum e Internal abdominal oblique

B

9 Which abdominal structure gives rise to the tunica vaginalis fotlowing the descent of testes during development shy

a External abdominal oblique aponeurosis b Transversalis fascia c Transversus abdominis muscle d Peritoneum e Internal abdominal oblique

D

10) The lesser omentum is a peritoneal fold which is su bdivided into the a Hepatogastric and gastrosplenic ligaments b Hepatoduodenal and gastroomentalligaments c Hepatoduodenal and gastrosplenic ligaments d Hepatogastric and hepatoduoden9-jrj igaments

D

11) A posteriorly perforating ulcer in the pyloric antrum of the stomach is most likely to produce initiallocalized peritonitis or abcess formation in which ofthS fQllowing

a Great-sac - -- -

b Paracolic recess

c Omental bursa

d Right subphrenic space

c

The inferior mesenteric artery arises from the abdominal aorta ilm_ediill~y_J-Qs1eriQLto which of the foowing org~ns A-F~t~filie duodenum B Head of the pan~eis C Neck of the pandeas

D Second part of the duodenum

E Third part of the duooenum_shylaquoshy

shy

The correct answer is E The inferior mesenteric artery arises from the anterior surface of the aorta at the level of the third lumbar vertebra The third part of the duodenum crosses the midline at the level of the third lumbar vertebra and passes anterior to the aorta at the origin of the inferior mesenteric artery The

first part of the duodenum (choice A) lies horizontally to the right of the midline at the level of the first

lumbar vertebra The head of the pancreas (choice B) is to the right of the midline and extends from the

level of the first lumbar vertebra to the third lumbar vertebra It lies within the concavity of the

duodenum The neck of the pancreas (choice C) lies in the midline at the level of the first lumbar

vertebra It lies on the anterior surface of the aorta at the origin of the superior mesenteric artery The second part of the duodenum (choice D) lies vertically to the right of the midline and extends from the

level of the first lumbar vertebra to the level of the third lumbar vertebra

The left adrenaLvein drains directly into which of the following veins A Hemiazygos vein

B Inferior vena cavaee C Left renal veiri -

D Splenic vein

E Superior mesenteric vein

a

The correct answer is C The left adrenal vein and the left gonadal vein (either testicular or ovarian) drain into the left renal vein TheTeft renal vein t~ains intothe- inferior vena cava In contrast the right

adrenal ~~inandnght gonadal veindrai~ gLr~ctJy iQtoJhe iilferiQ[ Vencava -- -

ThehemTazygoS7ein- (~h-~i-~ A)~~c~i~es the venous drainage from the body wall on the left side of the

thorax and abdomen No visceral organs drain directly to the azygos or hemiazygos veins The inferior vena cava (choice B) receives the direct venous drainage from the right adrenal vein but not

the left adrenal vein Remember the inferior vena cava is on the right side of the abdomen The splenic

vein (choice D) receives the venous drainage from the spleen and part of the pancreas and stomach The splenic vein is part of the portal venous system

The superior mesenteric vein (choice E) receives venous drainage from much of the intestinal tract It is part of the portal venous system and joins with the splenic vein to form the portal vein

A 43-year-old man presents complaining of pain in the groin On examination his physician palpates a

bulge in the region of the superficial inguinal ring which he diagnoses as a direct inguinal hernia The hernial sac most likely

A is covered by all three layers of the spennatic fascia B passes medial to the inferior epi gastric artery

C passes medial to the lateral border of the rectus abdominis muscle

D passes posterior to the inguinal ligament E passes through the deep inguinal ring

The correct answer is B Direct inguinal hernias enter the inguinal canal by tearing through the posterior

wall of that structure The typical location for this type of hernia is through the inguinal triangle bounded

laterally by the inferior epigastric artery medially by the lateral border of the rectus abdominis and

inferiorly by the inguinal ligament Direct inguinal hernias pass medial to the inferior epigastric artery

whereas indirect inguinal hernias pass lateral to the inferior epigastric artery because the deep inguinal

ring is lateral to the artery Indirect inguinal hernias are covered by all three layers of the spermatic fascia (choice A) Direct inguinal hernias are covered by fewer than all three layers because the direct inguinal

hernia tears through one or more layers of fascia as it emerges though the abdominal wall The lateral

border of the rectus abdominis muscle (choice C) forms the medial border of the inguinal triangle All

inguinal hernias pass lateral to the rectus abdominis Femoral hernias pass posterior to the inguinal ligament (choice D) Inguinal hernias emerge through the superficial inguinal ring which is superior to the inguinal ligament Inguinal hernias that descend below the inguinal ligament pass anterior to the

ligament Indirect inguinal hernias pass through the deep inguinal ring (choice H) direct inguinal hernias

do not Both types of inguinal hernias pass through the superficial inguinal ring

During a gastric resection in a patient with stomach cancer a surgeon wants to remove the lesser

omentum because of tumor extension into it Which of the following structures lie in the free edge of the

l~~g omentum and consequently must be dissected out in order to be preserved

A Common bile duct cystic duct and hepatic artery 6

B Cystic duct hepatic artery and hepatic vein

e Hepatic vein and cystic duct

Portal vein common bile duct and hepatic artery

E Portal vein hepatic artery and hepatic vein

The correct answer is D The free edge of the lesser omentum contains three important structures the

common bile duct the hepatic artery and the portal vein Nei ther the cystic duct (choices A B and C) nor the hepatic vein (choices B C and E) lies in the free

edge of the lesser omentum

A 55-year-old male patient with chronic liver disease has portal hypertension To relieve the pressure in the portal system a porto-caval shunt is performed Which of the following veins may by anastomosed to

accomplish this porto-caval shunt A Left renal vein-left testicular veingt

B Right renal vein-right suprarenal vein I shy

e Splenic vein -left renal vein J

D Superior mesenteric vein-inferior mesenteric vein E Superior mesenteric vein-splenic vein

The correct answer is C The splenic vein drains directly into the portal vein The left renal vein drains

directly into the inferior vena cava Anastomosis of these veins would allow blood from the portal vein to

drain retrograde though the splenic vein into the renal vein and then into the inferior vena cava The left

renal vein (choice A) drains directly into the inferior vena cava The left testicular vein drains directly into

the left renal vein Thus these veins are already in communication and neither vein is part of the portal venous system The right renal vein (choice B) drains directly into the inferior vena cava The right

suprarenal vein also drains directly into the inferior vena cava Thus neither vein is part of the portal

venous system The superior mesenteric vein (choice D) drains directly into the portal vein The inferior

mesenteric vein drains into the splenic vein which then drains into the portal vein Thus neither vein is

part of the caval venous system The superior mesenteric vein (choice E) drains directly into the portal

vein The splenic vein also drains directly into the portal vein Thus neither vein is part of the caval

venous system

A 12 year old boy has fever vomiting and para-umbilical pain After examining the patient the doctor

makes an initial diagnosis of appendicitis Appendicular pain which is initially referred to the umbilicus goes to the dorsal root ganglion of

a TI b TI2 c L1 d T7

(e I TIO

A 59-year-old male undergoes a neurological examination which reveals that when the abdominal wall is

stroked the muscles of the abdominal wall of the side of the body stimulated failed to contract Other

neurological tests appeared normal The likely region affected includes

a CI - C5 spinal segments b C6 - TI c T2-TI ~T8-T12

e Ll- L5

The surgery done to relive portal hypertension is done by connecting two veins Which of the following veins would be suitable for connection

a Inferior vena cava and portal vein b Superior vena cava and portal vein c Splenic vein and right renal vein d Splenic vein and left renal vein e Superior mesenteric vein and Inferior vena cava

A mother brings her 3-week-old infant to the pediatric clinic reporting a new scrotal bulge that she found -~-

while changing a diaper yesterday The infant is afebrile Physical examination reveals a palpable mass in

the scrotum while in the standing position resolution of the mass in the supine position and no

transillumination of the scrotal sac What is the most likely diagnOSiS

a Cryptorchidism b Direct inguinal hernia c Hydrocele d Indirect inguinal hernia ~ e varicocele

The Vagal trunks enter the abdomen by passing through which of the following openings in the

diaphragm

a Right crus b Esophageal hiatus ~ c Vena caval hiatus d Aortic hiatus e Left crus

2 The anterior boundary of the epiploic foramen of Winslow is bounded by

a) First part of duodenum b) Lesser curvature of stomach c) Liver d) Hepato-duodenalligament v ~

3 The ilio-inguinal nerve is derived from

a TI2 ry b LI c L2 d L3 e L23

15 Surgically the structure used to suspend the kidney to the diaphragm is

a) Renal fascia b) True capsule c) Perinephric fat d) Paranephric fat

6 If there is portal obstruction because of carcinoma affecting the pancreas which of these of the

following signs would be present

a Caput medusae b Esophageal varices c Rectal varices c

d Pulmonary edema

7 In a sliding hernia the gastro-esophageal junction lies

a) At its normal position b) Below the normal position c) Above the normal position V d) None of the above

8 Which of the following structures is retroperi toneal

A transverse colon B spleen IJ2f6 C ileum D descending colon v r 1pound1111111

9 The renal angle is fonned lgtetween the 12th rib and ______ muscle

a Psoas major -middotshyb Erector spinae c Quadratus Iumborum d Diaphragm

10 The anterior structure at the hilum of the kidney is

a) Renal vein ~

b) Renal artery I middot~ I

c) Ureter d) Accessory renal artery

11 Because of origin of the muscle from the lateral one third of the inguinal ligament it

could not fonn the anterior wall of the inguinal ligament

a) External oblique b) Internal oblique c) Transversus abdominis_ d) Rectus abdominis

12 A large tumor mass impinges on the splenic artery and its branches as the artery pass out from below

the greater curvature of the stomach Branches o(which of the following arteries would most likely to

effected by the pressure on the splenic artery

a Left gastric b Left gastro-epipJoic c Right gastric d Right gastro-epipoloic e Short gastric_

13 A new born baby has projectile vomiting after each feeding It is determined that there is obstruction

of the digestive tract as a result of annular pancreas Annular pancreas is as a result of an abnormality in which of the following process

a Rotation of the dorsal pancreatic bud around the first part of duodenum b Rotation of the dorsal pancreatic bud around the second part of duodenum c Rotation of the dorsal pancreatic bud around the third part of duodenum d Rotation of the ventral pancreatic bud around the first part of duodenum y Rotation of the ventral pancreatic bud around the second part of duodenum

14 As the liver bud enters the ventral mesogastrium the region of the mesogastrium stretching from the

liver to the anterior abdominal wall is called

a Lesser Omentum b Greater Omentum ~ Falcifrom ligament d Lacunar ligament e Ligamentum teres of liver

16 A patient has absence of his 12th rib In such a patient if the doctor makes an incision to approach his

kidney mistaking the 11 th rib for the 12t he would end up injuring

Which of the following arteries is a direct branch of the gastroduodenal artery The

A right gastric artery

B left gastric artery

C inferior pancreaticoduodenal artery D left gastroepiploic artery

i E)right gastroepiploic artery --

E x pI a nation The right gastric artery is typically a branch of the proper hepatic artery The left gastric artery is a direct

branch of the celiac trunk The right and left gastric arteries anastomose along the lesser curvature of the

stomach The inferior pancreaticoduodenal artery is a branch of the superior mesenteric artery it

anastomoses with the superior pancreaticoduodenal in the head of the pancreas The left gastroepiploic

artery is a branch of the splenic artery it anastomoses with the right gastroepiploic artery along the greater

curvature of the stomach The right gastroepiploic artery is a branch of the gastroduodenal artery The

other branch of the gastroduodenal artery is the superior pancreaticoduodenal artery

Which of the following pairs of veins join together to form the portal vein The

A superior mesenteric vein and inferior mesenteric vein

B inferior mesenteric vein and splenic vein

C superior mesenteric vein and splenic vein

Ip)splenic vein and left gastric vein E superior mesenteric vein and left gastric vein

Explanation

The portal vein is formed behind the neck of the pancreas by the union of the superior mesenteric vein

and the splenic vein The inferior mesenteric vein drains into the splenic vein The left gastric vein drains

directly into the portal vein After the portal vein forms it enters the hepatoduodenalligament of the

lesser omentum to reach the liver The portal vein is the most posterior structure in the hepatoduodenal

ligament

At which of the following vertebral levels does the duodenum pass anterior to the aorta - _- shy

All ~

B L2 7~

CL3 I

~DL4

E L5

Explanation

The duodenum begins at the pyloric sphincter at the level of Ll The second (or descending) portion of

the duodenum is to the right of the aorta and extends inferiorly from the level of Ll to the level of L3 The third part of the duodenum crosses the aorta from the right side to the left side at the level of L3 The

fourth (ascending) portion of the duodenum extends from the level of LJ to the level of L2 The

duodenum ends at the duodenojejunal flexure The superior mesenteric artery passes anterior to the

duodenum as the duodenum passes anterior to the aorta The duodenum can be constricted at this level

In which of the following locations will perforation of the digestive tract result in the spilling of luminal

contents into the - lesser peritoneal sac

A Anterior wall of the second portion of the duodenum B Posterior wall of the second portion of the duodenum

C Anterior wall of the stomach

~Posterior wall of the stomach E Posterior wall of the transverse colon

Explanation

The posterior wall of the stomach is related to the lesser peritoneal sac The anterior wall of the stomach is related to the greater peritoneal sac The anterior wall of the second portion of the duodenum is related to the greater peritoneal sac The posterior wall of the second portion of the duodenum is related to the retroperitoneal space The posterior wall of the transverse colon is related to the greater peritoneal sac

The ureter lies against the anterior surface of which of the following muscles shyA Crus oftne diaphragm B Quadratus lumborum

0 Psoas major D Transversus abdominis

E Iliacus

Explanation The ureter exits the renal pelvis at about the level of vertebra L2 As it descends along the posterior abdominal wall it lies on the anterior surface of the psoas major The psoas major muscle arises from the bodies of the lower lumbar vertebrae The psoas major muscle is joined by the iliacus to fonn the

iliopsoas muscle The iliopsoas muscle then attaches to the lesser trochanter of the femur and is the major

flexor of the hip

As the right ureter passes the pelvic brim it lies against the anterior surface of which of the following

blood vessels

A Gonadal artery B Inferiorvena cava C Internal iliac artery

rJ- External Iliac artery

E Inferior mesenteric artery

Explanation

The ureter lies in the extraperitoneal space in the posterior abdominal wall Alter leaving the kidney it

passes inferiorly on the anterior surface of the psoas major muscle At the pelvic brim the ureter passes

into the pelvis At this point the common iliac artery is dividing into the external and iliac arteries The

ureter lies on the anterior surface of the external iliac artery immediately distal to the bifurcation This is a useful landmark for a surgeon to locate the ureter

When extravasated urine passes from the superficial perineal space into the anterior abdominal wall it is

found immediately deep to which of the following layers of the anterior abdominal wall

-ltScarpas fascia

B External oblique muscle

C Internal oblique muscle D Transversus abdominis muscle

E Transversalis fascia

Explanation

The superficial perineal space is bound by Colles fascia the fibrous portion of the superficial fascia This

layer of fascia is continuous with Scarpas fascia the fibrous portion of the superficial fascia of the anterior abdominal wall Therefore urine that is deep to Colles fascia will remain deep to Scarpa s fascia The urine will spread in the plane between Scarpas fascia and the external oblique layer

When a horseshoe kidney develops the ascent of the kidney is restricted by the A internal iliac artery B external Iliac artery

C common iliac artery

inferior mesenteric artery

E superior mesenteric artery

Explanation

A horseshoe kidney develops when the inferior poles of the to kidneys fuse together as they ascend into

the abdomen from the pelvis The first anterior midline vessel that is encountered by the horseshoe kidney

is the inferior mesenteric artery This artery prevents the kidney from continuing its ascent

The left testicular vein drains into which of the following veins

A Left internal iliac vein B Left common iliac vein

bflnferior vena cava D Left renal vein I

E Left internal pudendal vein

Explanation

The left testicular vein drains into the left renal vein The right testicular ~i~[~nsltjectlY into the

inferior vena cava This difference in venous drainage is believed to explain the greater incidence of

varicocele on the left side than on the right The venous drainage from the penis is to the internal vein

which then drains into the internal Iliac vein

The spinal nerve that provides cutaneous branches to the skin around the umbilicus is

A TS B TW-shy

C TI2

DL2 EtA

Explanation

The tenth intercostal nerve is the anterior ramus of the TIO spinal nerve After passing through the tenth

intercostal space the nerve continues forward in the anterolateral abdominal wall in the plane between

the internal oblique muscle and the transversus abdominis muscle In the abdominal wall the nerve innervates to the abdominal wall muscles as well as the skin and the parietal peritoneum The umbilicus is

a useful landmark for the region of distribution of the tenth thoracic nerve

The ligament of the vertebral column that resists its extension is the Aligamentum flavum

B supraspinous ligament

C posterior longitudinal ligament

D anterior longitudinal ligament

E interspinous ligament

Explanation

The ligaments of the vertebral column that resist flexion of the column include the supraspinous ligament

interspinous ligament ligamentum fiavum and posterior longitudinal ligament The ligament that resists

extension is the anterior longitudinal ligament This longitudinal ligament is very broad and strong It

covers the anterior and anterolateral surfaces of the vertebral bodies and the intervertebral disks In

addition to resisting extension the anterior longitudinal ligament provides reinforcement to the anterior

and anterolateral surfaces of the intervertebral disk The posterior longitudinal ligament is relatively

narrow and covers the posterior surface of the vertebral bodies and the intervertebral disks This ligament

reinforces the posterior surface of the disk The posterolateral surface of the disk is not reinforced and it

is through this region that herniation of the nucleus pulposus usually occurs

A patient presents with epigastric and right upper quadrant pain The pain is most intense 2-4 hours after

eating and is reduced by the ingestion of antacids The patient states that he has passed black tarry stools

(melena) within the last week Fiberoptic endoscopy reveals a yellowish crater surrounded by a rim of

erythema that is 3 cm distal to the pylorus Accordingly an ulcer has been identified in the patients

A fundus

B antrum

C duodenum

D jejunum

E ileum

A number of physiologic genetic and other factors increase the risk of gastric (and duodenal) peptic

ulcers The evidence that H pylori plays a principle role is compelling Smoking and caffeine are known to adversely affect the morbidity mortality and healing rates of peptic ulcers In general first-degree

relatives of peptic ulcer patients as well as males have a threefold to fourfold increased risk of developing this disorder Paradoxically in gastric ulcer disease acid secretion is not elevated It is possible that

excess secreted hydrogen ion is reabsorbed across the injured gastric mucosa In general a defect in gastric mucosal defense is the more important local physiologic

A patient presents with symptoms of duodenal obstruction caused by an annular pancreas Annular pancreas is caused by

A rotation of the dorsal pancreatic bud into the ventral mesentery B rotation of the ventral pancreatic bud into the dorsal mesentery

fJ failure of the major and minor pancreatic ducts to fuse ~ ~ cleavage of the ventral pancreatic bud and rotation of the two portions in opposite directions around -the duodenum E formation of one pancreatic bud instead of two

Explanation Normally the ventral pancreatic bud rotates around the gut tube to reach the dorsal pancreatic bud The two buds fuse to form a single pancreas and the distal portions of the two ducts fuse The ventral pancreatic bud forms the inferior portion of the head of the pancreas the uncinate process and the major pancreatic duct (of Wirsung) The dorsal pancreatic bud forms the superior part of the head the neck body and tail and the minor pancreatic duct (of Santorini) Annular pancreas is the result of the ventral pancreatic bud dividing into two portions before it rotates into the dorsal mesentery Each portion rotates in opposite directions to get to the dorsal mesentery thus encircling the duodenum The presence of annular pancreas can constrict the duodenum thus obstructing its lumen

In n _ phranlc----

Gon ~l ----_1 Lum bltano

~~--- CornmQ1t bull ac

+-~4--- lnlllirnaJ ilic

xtem iliac

OBJECTIVE - Identify the blood supply to each of the structures listed in the table on the previous page

Ill give you a head start

FOREGUT - Supplied bV Celiac Tru nk (T12)

Proper hepatic

GastiooUod 13Jafter

1nferlor pancreaticoduodenal artery

Common epatlc

Lett gas ric iiirtery

Spfen artery

shy Gastroepiphgtic artery

~ Superior mesenteric 8rtfry

~

1 Esophagus is a derivative of the foregut so its blood supply originates from the celiac trunk

(T12) The predominant blood supply to abdominal portion of the esophagus is the Esophageal

A (Branch of L Gastric) The venous drainage of the esophagus is particularly important because

it is 1 of 3 clinically relevant sites of Portal Caval anastamoses The Portal Esophageal Vein

meets the Caval Azygos System Persistent bleeding manifests as Esophageal Varices - a fata I

condition

2 The Stomach is also a derivative of the foregut has EXTENSIVE blood supply and is very high

yield on anatomy exams The lesser curvature is supplied superiorly by the L Gastric A (1 of 3

major branches ofthe Celiac trunk) and inferiorly by the R Gastric A ( a branch ofthe proper

Hepatic A) The greater curvature is supplied superiorly by the L Gastroepiploic A (a major

branch of the splenic A) and inferiorly by the R Gastroepiploic A

The Short Gastric arteries (branches of Splenic Artery) supply the fundus of the stomach and

are referred to as EIID ARTERIES because they have no collateral blood supply Therefore if the

splenic artery were occluded (ex - increased pressure in the ommental bursa) - there would be

ischemia to the fundus of the stomach Venous drainage of the stomach is extensive via various

veins lead ing to the portal system Posterior to the stomach the IMV joins the splenic V which

joins the SMV to form the PORTAL VEIN ADAMS

3 Duodenum blood supply has high clinical relevance because it is the junction of the foregut and

midgut and therefore is the site of anastamoses between branches ofthe Celiac Trunk (main

foregut artery) and the Superior Messenteric Artery (main midgut artery) The Proper hepatic

artery gives off the gastroduodenal artery which travels behind the 1st part of the duodenum

This point has high clin ical relevance because duodenal ulcers are very common and a posterior

rupture of the 1st part of the duodenum could rupture the gastroduodenal artery causing

traumatic abdominal bleeding The Gastroduodenal artery first gives off the R Gastroepiploic A

(mentioned above) and proceeds as the Superior pancreatico duodenal artery (supplies the

pancreas and duodenum) which anastamoses with the inferior pancreatico duodenal A (branch

of the SMA) This is the junction of foregut and midgut and occurs near the opening of the

bil iary system into the duodenum (ampula of vater) Portal venous drainage here is responsible

for delivering nutrients from digestion to the liver for metabolism Appreciate that the Superior

mesenteric artery (artery of the midgut) branches from the aorta at Ll travels posterior to the

pancreas than moves anteriorly (at the jxn of the pancreatic headbody) and comes over the

3rd4th part of the duodenum Tumor of the head of the pancreas can compress the SMA

4 Jiver blood supply is via the common hepatic artery (major branch of the cel iac trunk) The

common hepatiC becomes the proper hepatic gives off the R gastric A and the Gastroduodenal

A and then joins the common bile duct and the portal vein in the portal triad Clinical- if a

patient were bleeding from the hepatic A a surgeon can stick his fingers in the epiplOic foramen

and squeeze the free edge of the hepatoduodenalligament in order to stop bleeding to the

area Please note that the hepatic a branches into Rand L hepatic A The Right hepatic artery

gives off the cystic artery which supplies the gallbladder Afferent venous supply is via the

Portal vein which is bringing nutrient rich blood to the liver After metabolism takes place

venous blood leaves the liver through the hepatic veins into the IVC PLEASE UNDERSTAND THE

RELATIONSHIP OF THESE STRUCTURES - ADAMSNETIERSNH Etc

5 Pancreas - Head is supplied via the superior and inferior pancreaticoduodenal arteries

(mentioned above) The tail (situated towards the hilum of the spleen) is supplied via the

pancreatic branches of the splenic artery (END ARTERIES) This blood supply is very important

because the endocrine Alpha and Beta Cells from the pancreatic islets of lagerhans are located

towards the tail This is where Insulin and Glucagon is released to the blood

Now complete this for mid and hindgut structures Make sure to note clinically relevant arterial

anastomoses as well as portal caval anastomoses FYI Appendix blood supply SMA + IMA

anastamoses marginal artery Portalcaval rectal veins fhemmorhoids) and periumbilical caput

medusa are high yield THE BUTT THE GUT and THE CAPUT

Abdominal Development

Liver

Ij1f

II wall b

oh liN ~ VltJrti n be- bull

Pancreas

Secondary Retroperitonealization e I~tl r 1 a v-mtrai m ellter

Rotations of the Gut I i Ij (lIl1UtIJ f~ l r tilt

()l td 10 me l-ft and he v

--~--- -~ -~-~

i

I AolaijonjoI~guf I

STOMACH BED (IDENTIFY IN ADAMS)- the structures posterior to the ommental bursa which

support the stomach in the supine position

Abdomnal JQrUI

Splnic vein

OmQ-oul tv~ ) O(s(Jroa)

Lojt(r o m nturrt (hpJtodu o d~n31 Hid

Gadrl)SplerH (g3stroll~nal) IIgam~nt

hiad h~~atogrtricent IIQdmiddotcrt~)

Lt Dome of Diaphragm (why left Look this up in Adams)

Spleen (What is the blood supply)

Left Kidney (What is the blood supply - AND how is it different from the R kidney)

Suprarenal Gland (What is the Arterial AND Venous Blood supply - how are they different)

Pancreas (How does supply differ from Head to Tail What is the SMA Relationship)

Transverse Mesocolon

liver - ADAMSWET - Make sure you look at the liver in wet lab

Left triangular nl1am~nt

ComoaDj ligamnt

Erophg~1 impre$ioo

Hepatio veins

In1erior -ifena middotr3)Ia

Fibrous appendix o-t

live

impr~j on

Heprorendl p~rtion of Q)(Qllary ligament

Righllri~n9ul r 1I~met

(Common) bile quol

Gr)mmCtr~ hepatic dlJct

Ccentic duct

Duodenal impression

GaJdate p-fr)~S

Hepatic artgtrl prop-f iiiiila - Faloiform ligament

_ - shy Round ligamen liver

~--F-- CoJio imprgt-ssi-on

Prta heptis

Identify the lobes impressions and embryonic remnants associated with the liver

Caudate Lobe Quadrate Lobe Right Lobe Left Lobe Round ligament Falciform Ligament

Ligamentum Venosum (what is its fxn in embryonic life) Hepatic Veins (NOT PART OF THE

PORTAL TRIAD) IVC PORTAL TRIAD - Contents relationship cross section etc Know the

Galbladder relationship to the lobes of the liver

Biliary Duct System - Make sure you understand the sequence of these structures - BE ABLE TO

DRAW A FLOW CHART

TPVd i

t

I t

1 __ Cm-(r

patk GlJet

I

J

Clinical = JAUNDICE is caused by anything that prevents delivery of bile to intestine Tumor of the

head of the pancreas Stones etc Patient will have pale stools and yellowish colored mucus

membranes

Clinical- Any scenario that tells you the patient has BILLOUS VOMIT means that the obstruction to

the flow of digestive contents is after the Ampulla of Vater (Site of Entry of Billiary system to the

duodenum) - ie Duodenal Atresia

Spleen -located posterior to the mid axillary line between ribs 9 and 11 Make sure you know that

the 10th rib is the main axis of the spleen and this organ is susceptible to injury (stab wound errant

thoracoce ntesis etc)

The spleen is derived from mesodermal cells - NOT THE GUT TUBE

The spleen rests on the left colic flexure associates with the tail of the pancreas Know the

structures entering the Hilum of the spleen

Sh rt O~-t~ic 1 0(0 10 rtiltSPIric Iloa nt

(cut)

Peritoneum - similar concept to Pleura - think of a fist in a balloon

Visceral Peritoneum - Layer of balloon touching your fist

Parietal Peritoneum - Layer of balloon not touching your fist

Your fist represents the organ your wrist is the hilum and your arm contains the blood supply

entering the organ

Appreciate that there will never be organs in the peritoneal cavity - rather these organs invaginate

the cavity Kaplan videos

RULES OF NOMENCLATUREshy

1 Organ completely surrounded by peritoneum - peritoneal organ

2 Organ partially surrounded by peritoneum- Retroperitoneal

3 Peritoneum surrounding peritoneal organ is VISCERAL peritoneum

4 Peritoneum surrounding retroperitoneal organ is PARIETAL peritoneum

5 Peritoneum connecting visceral to parietal is called messentary 2 messentaries in the

gut Dorsal (to the gut tube) and ventral (to the gut tube) messentary

Aorta is in Retro peritoneal position - but blood must reach peritoneal position - vessels travel through

messentary All peritoneal organs will have blood supply reaching through messentary

-Mesentery is a 2 layer peritoneum with a neurovascular communication between body wall and organ

- Ligament connects one organ with another or to the abdominal wall (Ommentum = ligament)

lesser Ommentum (attach lesser curvature of stomach and duodenum to liver) =Hepatoduodenal

Ligament and Hepatogastric Ligament

Has a Superior and Inferior Recess (Accumulation of Fluid in Ascites)

Communicates with the greater sac through the epiplic foramen (what structures pass through

this foramen)

Boundaries - you must be able to visualize this

o Anterior - stomach

o Posterior - parietal peritoneum pancreas

o Superior - superior recess (bw diaphragm and coronary ligament)

o Inferior -Inferior recess (bw layers or greater momentum

Greater Ommentum (attach greater curvature of stomach) Gastrophrenic ligament Gastrosplenic

ligament gastrocolic ligament

The greater omentum is the largest peritoneal fold It consists of a double sheet of peritoneum folded on itself so that it is made up of four layers The two layers which descend from the greater curvature of the stomach and commencement of the duodenum pass in front of the small intestines sometimes as low down as the pelvis they then turn upon themselves and ascend again as far as the transverse colon where they separate and enclose that part of the intestine

ABDOMINAL PAIN

Parietal Peritoneum - supplied by same vasculature lymphatics and nerves supplying body wall it

lines and diaphragm Sensitive to pain pressure heat cold well localized

Visceral Peritoneum - supplied by same vasculature lymphatics and somatic nerve of organ it covers

Insensitive to touch heat cold and laceration - referred to dermatome of spinal ganglia providing

sensory fibers Where does appendicitis refer to

Foregut pain - epigastric area (ie - cholycystitis)

Midgut pain - periumbilical area (ie - appendicitis)

Hindgut Pain - suprapubic area (ie - diverticulitis)

Extra ImagesConcepts

ll~_____-

FalifCtrm ligament oind r~ud ligamet f Ilver

Blood from splenio gastriC and inferiof rne$e-rteri v~ins

Ca-I tributaries

Lett gastrio Ifein

Posterior superior pan~reatioodul)denal vaihS

Lott gamo-om~nlal (9aropip lomiddotic) -in

Poq_~ tjol imerl-9-r panCJertlcorllJod-nal veiopound --amp----I- - ~J Right grtr~-omntal

Anwrior interi (gartroepiploic) Jjn

pan euaii cod vl)denal veins middot Inf~Ji (t r mesentric vein

Miqdle (olic vein

Right cl)licvein Sigmoid and rectosigml)id (ei ns

IhH)Collc(~io

--- Mi~dl laquooLJl gtjrltgt

PoM ca vl1 illasto)moses -----shyampoptoageal 2 Paraumbilie-lt11 Inferi or Fectal vei ns

3 Recial 4 REuoperHonea1

Know how the Portal vein is formed I 4 sites of portal caval anastamoses and 1 clinical shunt

Col li t ltt-~ otTl~tI ~nj pc~ 1lt1 turJoG

Ltf 14i1 tImiddot~ artoftl9 on tj phtAt$

L-oftqf 4t t~r 1=laquoIran d 1 bull shy~p l ci rj o fOOOts

Nerves follow the arteries - appreciate the splanchnic nervous system I

Uet~ric branch of left ~nal art

Ureterie branch of righi renal artelY

Left Zld lumbar in and co mlTlunication to as)erdin9 lumbar l(~in Hi ~ht tEZ1~~t~ t3r j t itn ~ nJ l1t- rlnd lfe i r1

Inferior me5nteri~ artery

Notice that the right testicular vein drains directly into the IVC and the right testicular artery drains

directly into the aorta However the left testicular vein drains into the L renal vein at a right angleshy

reason left testicle is lower and more susceptible to varicocele (bag of worms)

Also notice that the left renal vein has a longer course because the IVC is on the right side whereas

the right renal artery has a longer course because the aorta is on the left side

Appreciate the anterior to posterior relationship of structures in the hilum of the kidney - VAP - Vein

Artery Renal Pelvis (Ureter)

11____ __ L_ L_ n VJ __ _ _ t_L I I_ _ L __ L_ I -pound1 bull LI_~-I ____

Posterior View of Head of Pancreas in ( of Duodenum

Celiao hunk

Co mmon ~L~jJth art~ry

GastNduQdonal artrf (partilly in phantn)

P1)Sterior $Up~Jior panCflaticuduodfmal art~r~t

(Co mm on) bile duct

middot~1t~~t-1l---~-~- Right gshomiddotomental (gastoe plp lolc) 3rte (phantomost)

Grener paocre atic art-ry

1n1~rjor pancr-iatlc artery

Jtrifll supejo r pal)oreailcento)dJodenal artr1 (phantom)

Anastomotlo branch

POostetlor bJanch of jo f~ri of pan-reatir(lduodensl drttnj

Anterio r branch of i flferior palcreati~)duodenal art~(phan1om)

Notice the extensive blood supply to the pancreas and duodenum via the branches of the celiac trunk

Notice collateral supply from SMA branches - makes sense bc this is the jxn of foregutmidgut

Identify the vessels in this arteriogram

Hiltid i)f N~ck oi B)dvof Tail 01 pa nereas pan cent~as P-nmiddot-reas panCtCas

I nferie v~na cava

jHept1iic p(lrlai v~in

Port1 tnd H~pti lt a ftH prol

Comm on) bll duct

Ouodtnum

~ft colic (sio)Atta~ hmtrlt jt~xJr-ofha~elSe

muo(IIQn

Right ~lIc (h~j)tic)

il~gtture

In1triol m~oten lIein (rttr op~ritoMdO

SlJp efl or mes~n~fiC amrV and lipln

KNOW YOUR NEIGHBORHOOD

Questions

vVhiJh structure supplied by a bnmdlof the cclia( artery is not derivcd from foregut LemCJUCrITI

(A) Head of the pancte-a5

CD) Pyloric duolenum

Cystkduct

( Liver hepatocyt~~

~F) Body of the spleen

An infant presents with an omrhaJucele at birth -hi oJ the [oHm illg applies to his cM1-dition

(A) It is 31so seen ill p4titnts with aganghonic megacolon

(11) ft reuirs from a fal1ure of resorption of theviteUine d let

(C) It results from herniation at the-site of regression of the right umbilk vein

DJ It is caustd by faihtrc of recanalization of the midgut part of the duodenum

~ It ill camioo by a failuIt vf the midgul to return to the abGQminal uity after herniashytion in-n the urnbilk s l stalk

Ot er than the spleen occlusion Cif the spit-Ilk artery at its odgin wm most likely affect die blood supply to jllch st cnud

(A) Jejunum

(B) Body of th pal1~lltas

(C) LeSStT Cllmiddotlaturc of tl )toma-ch

(D Duodenum dista to the entrance of the Ornmou bile duct

E Fundus of the stomach

A 38-yeu-old batL~er with a history of heartburn suddenly experiences excluciating pain in the (plgastric region of th~ abdomeu SurgCry is perf~rme immediard y upon admisshysion to the 1IlcrgCJliy tuomh~re i~ evidence uf a ruptured ulcer in the posterior waU of the stomach Vhere will a surgeon first fi nd the stomach contenlSf

A) Greater p4ritoneal sac

rB) Cul~de-s~c of Douglas (--

C Omental bursa ~

--D) Paracolic gutter

rEj Between -he panttal perimltum and the posterior body wal1

At birth an infant presents with a st()ma~ rb~tbas~njJled jfltotb~diaplfagru 1A1ltre is the defect thatresulied iiitJle heini~t()n shy~tsophagealbiatus

7 - rH-- Hiatus for the inferior vena cava

( Pleuroperitoneal membrane -(0) Septum transvcrsum

(E) Right Crlt~

An infant born with DOVv7l syndrome presents with bili()u~ vomiting Ahat congenital defect does the infant have

(A) Pyloric stenosis

(B) Meckel diverticulum C) Ornphaloce1e

(D) Gastroschisis

( ~ ) Duodenal atresia y A patient with cirrhosis of the liver presents with ~ bacalvaricestnlreased retrograde pressure in which veins caused the varices

(A) Paraumuilical

(B) Splenic

(ct AzygltJus

(15))G~trk ( (-F) Superior mesemeric

A htaltby 3-year~old male patient experiences a hernial sa protruding from the anterior abdominal wall about halfway between me anterior superior ilia spine and the pubk tuberde Pulsations of al1 artery are palpated medial to the protrusion site through the abdominal walL Which layer of the anterior abdominal wall will first be traversed by the

1hctma

fA) Rectus sheath (B) External oblique aponeurosis

(C) Inguinal ligament

lD) Transversalis fusda

(E) Cremasteric fa~cia

After 5urgi(aj ffpair of a hernia the patient tXperienccs mtmlgtness in the skin on the anteshyrior aspect of the S(Totum_ Vhaf nerve may have been lesioned during thehemiorrhaphy

(A) Femoral

(B) Obturator

(C) Ilioinguinal

(D) lliohypogastrk

(E) Pudendal

A 23~year-LJld female secretary il1 good health ~-uddcn1) doubles over with pain in the a ea of the 1JmbRicu$ Sbe feels vartn and ltneasy and has no appetite That night the pain seems to have mQved to the tower right abdominal regjol1 and she calls her family doctor who then arranges for an ambulance to pk-k her up and take her to the hospitaL Wh ell ntn~ perceived in the area of the urnbilirus most Hkely carried lhe pairfu I sensations into the eNS

tA) Vagus nerves I~

V B)

) Lessersplanchnk nerves

tC) Pudendal nerves

(D) lIiohpogastrk nerves

(E) Greater splam ic l erves

A CT reveals carcinoma in the bOod of the ancreas Vhich blood vessel trut ourses ----~- - -bull ------ --shy

immediately poftterior to the body ofthe pancreas is the m~t likely to be oompressed

(A) Splenk artery

(B) Abdominal aorta (C) Portal vein

(1) Splenic vein

(E) Renal vein

A patient has a penrln1l1ng uker of the posterior wall ot the br~l part ot the (lUooenmn llkh blood vessel is subject to erosion

(A) Common hepatic artery

(B) Gastroouodenal artery

(C) Proper hevatic artery

(D) Celiac artery

(E) Anterior inferior 11amrelltlcoduodcnal attery

Your patient has been diagnosed -ith a carcinoma locallted to the head and l~e(k of the pancreas Another clinical sign would be

A esophageal varices

(8) hemorrhoids

C) a caput medusa

(D) increased pra Teuro n th~ hepatic veins

(E) enlarged right supra lavkular lymph nodes

Wltkh of the foUowing structures develops in the ventral mesentery

(A) Spleen

(B) Jeiunum (C) Head of1ht pancreas (D) Transverse colon (E) Stomach

ti l Uw ~ littwin~ f( S-t lil oai Imdge ~ hi(h or tbt la~)d J truetur tgt liJ llntn nl) he hl p UC iJd [IIi ell

c o

A) drains Ie tht infCrior a La aI

R t middot~nfl0 ~ill to th~ lunlgtn of h i dtlndCrlllfH

(e) m t bull JiJattd on tl l J n T ~H

D ) sup Lc O VSlt I Hlid bhtu l 1 li - -I un oid

( ) U~tpli(t tr j middottUh~ 1 v(( b~nt rfK n1ilc~Zm

ANSWERS AND EXPLANATIONS

Answer E The spleen is t hlttnopodicand lymph organ demlted from mesoderm

Answ~ R Al1 tlmphalocele is caused by it failure of the nlidgut to return to the ahdomir nat cavity after herniation into the umbiliau Stalk Choices Aand D maybe seen in infants with Down syndrome choice D ~s the specific CBuse ofduudcnal JtiCSitt Choice C is (ile cause of gclstrosbisis and Choice B nsults iu a Meurolktldivertku1-tlB

Answer B The fundus ofthe stomach is suppHed by soort gastric brunches of the splenic altery The splenic artery supplies the body and tail of the pancreas part of the greater curvature of the sttmla(h and the spleen Te jejunum part of the head of the pancreas and tht~ duodenum distal to the entrance of the commOll bile duct are supplied by the superior mesenterk artery clll~l ~be less r ctlt1ature cmd the pylQric antrum are supplied by the right and lei gastric art(ries

AnSWftt C Tbeomental bursa or lesser ~ritoneaj sac lies direcdy posterior to the proxshyimal part of the duodeTtlm and the stomach and would be the first site where stomach contents ~Ott1d be fpoundluncL

Answer C A defect in a llleuropcritoneal membrane (uswlly the left) is the typical site of i1 cc-ngenitlI diilphragluatic hemia llere the membr4ne fails to dose ()pound( of the perishycCirdiopcritulleal canals

Answer E DuoJenal atresia and aganglionic megacoion are congwitaI defects S~Il in patients with Dowmiddotnsyndrome

Answer D RulaTgemt~llt of and retrograde flow in g~lstrk vel_ns in particlJl~r the kft gas~ tricveins dilates the capillary bed in rhe wall of the esophagus in (ases of porta yper~

tension Blood flow would increase in and dilampte tribntarkgts of the (lZygOUS vein on the other side of the capiUary bed but flow in this vein is in the typical direction t()ward the superior vena cava Paraumbiii(ltU vein eilgorgement contributes to a caput medusH Splenic ~nlargement might prc~nt with 5plcnonlegaly and balt-kflow in to tlu superior m~~ntclic vein occurs but is asymptomatic

Answer D The patient hagt an indirect inguinal hernia whi~h emerges from the antt-rior abdominal wall through the deep inguinltilling Theeep ring is a fault in the transv~rshysaUs fascia this I~yer wiIJ be penetrated first by the hernia

An~Wer C The ilioinguinal nenc which provides sens~llion to the lnedlal thigh ltmclanteshytior SClotunl pass~lt th rough the 5uperfh_ial inguinal ring ind $subject to inj i1T) becaus-e

it is in the operatitm Held of the erniorrhapny

Auswer B The leMHr splanchnic nerves are sympathdic nerVlts that carry viscera l sensashytlltgtrogt ftom illtllt1m~d ()J stietched gust (itinteitinal ~tructures (in this case the pprndix) into tnt eNS Lesser splanchnic ntTYcsarisc from thmiddot T9--T12 spinal cord segments lt1nd provide sympathetic innenation tD rnidgut siruc1ures whiCh include CLe app~JldD Viscera] Pain arising from affecLed Inidgut ampt 1C1ure is referred over the same dl- matorne~ of spinal segrnertts v-hich provide the sympathetic Innervation n this G1SC of appendicitis the invohen~n t of the ltire) of t e unlhHku indud s the T 10 dermatome

Answer B Of the five choices onty the dscending olon is retroperiton~al aldwould be a lik ~ ( choice to be seen immediately a(~jilcent to t11e posterior abdominal middotn~L

Amwen D The SpltftlC ~-ein ourses posterior to the body of the panneas m its way tt drain into the superior mCSfttltlri( vein

Answcr B TILt glstrodllolticnal artery 1 direct hIamh of the comrootl hepatic artery courses immediately pt))iwri() to the duodenum and is slbject to erosion

Answer B Carcinoma of th pan middott3S in the 1tilt1 may compreampgt the portltil vein at irs orishygill The poTtai vcin is fomled when the splenic vein jQiaswith tfie superior meStllt eric vein The inferiot mesenteric vein joins the ~plenjc vein just priOT to tlli~ point at which the splenic joins the superior Jlleit1ltcri( vein Increescd venous presslu in the inferior mesenteric vein is a cause of emo hoid~

Answer C The- velltral pancreas wilich forms most of the head of the p ~ncr as develops in the ventral mes(ntery as antutgrowth of the hepatic diverticulum Th~ hepatic divershyticulull induding the biIJary appa~atus develops in tbe ventral mesentery of the foregut

Answer~ A The superior mesenteric ~in joins with the spienkvein to form the hepatic portal vciu

Answer D The structure at gttlK is the proper hepatic artery~ whkh suppUesoxygenated b middotood to the liver

MAKE SURE YOU KNOW the diff bw Rectus Sheath above and below the arcuate line

ABOVE

Aponeurosis of xiiltmal obllque musclo

Extemll f)biquw musde

Reotln ilbdomlnls musole S~in

Internal 9bliquQ mY~QI

AponeUfOsi$ of hJH$V~~S Lir9a a lb lbdolTlin~ musolo Tri OJV6 rUi

atldomlnis mUS(loe

Sub cutanlilous tiue (tatty ye r)

BElOW

A POrl lJfosis 01 etemal oblique muscle

Aponeul~)sis 01 Internal oblique mU$cl~

Anteriol lay~ of r~ltdus st~ath EXttom1 oblique rnu$cll

Rectus Jbdominis muscle Intoernal Aponeurc-sis of tra~fersU$ oblique muscle-

at-domlnis muscentl ~ Skio

Tra nsvitSus abdomioLs ml)ZClt

TralSVersaHs fascia Medial umQil iegtt1 1i9Jment -and folj

Uldchus Peritoneum (ir median Umbilj~al Suboutane ous

Extraprftone 11ascia

Ymbilimiddot~1 fold)

preu9poundiea1 fascia

tissue (fatty 4nd m~mbr3n(iUS layers)

o Above the arcuate line (A horizontal line 13 of the distance bw the umbilicus and the

pubic symphysis) -10 Aponeurosis divides into an AntPost Laminae

o The Ant Laminae joins EO and Post Laminae joins Trans Abdominis = Ant and Post

RECTUS SHEATH respectively

o BElOW the arcuate line - all 3 aponeurosis join ANTERIOR to rectus muscle to meet its

counterpart in the midline (linea Alba)

o Take away Msg - The abdomen is devoid of a posterior rectus sheath below the

arcuate line and is therefore more vulnerable to herniasinjuries

Question - A physician makes a deep incision in the patients midline immediately superior to

the pubic symphysis which of the following layers is his knife least likely to pass

Rectus Abdominis External Oblique Ant Rectus Sheath Posterior Rectus Sheath All of the

Above

Answer - All of the above None of the other answer choices are midline structures -LINEA

ALBA

Linea Alba has very poor blood supply - doesnt heal well after surgery Therefore this is a

common site for incisional hernias

a Spleen b Transverse colon c Descending colon d Stomach e Pleura

17 Meckels diverticulum is normally found 2 feet proximal from the

a Pyloric sphincter b Lower esophageal sphincter c Ileo-cecal valve d Middle valve of Huston e Anal valve

18 Ulcer in the posterior wall of the first part of the duodenum would erode ___ artery and would cause bleeding

a Left gastric b Right gastric c Hepatic artery proper d Gastroduodenal artery e Middle colic artery

19 An inflamed appendix is identified by a surgeon on the operation table by noting

a The appendicies epiploicae b The convergence of tenia c The artery of Drummond d The mesocolon e The mesosalphinx

20 The nerve which emerges through the psoas major is

a Femoral b Ilio-inguinal c Ilio-hypogastric d Pudendal e Subcostal

21 The right gonadal vein drains into the

a Azygos b Hemiazygos c Inferior Vena Cava d Right renal vein e Left renal vein

22 The hepatocytes in the liver is derived from

a Ectoderm b Endoderm c Mesoderm

d Neural ectoderm

23 Abscess in the lumbar vertebrae due to tuberculosis would spread to the adjacent muscle which is

a Psoas Major b Iliacus c Quadratus lumborum d Tranversus Abdominis

24 The anterior wall of the inguinal canal is formed by

a External oblique and transverses abdominis b External oblique and fascia transversalis c Internal oblique and external oblique d Internal oblique and transverses abdominis e Fascia transversalis and peritoneum

Meckels diverticulum is a result of which of the following developmental abnormalities shy

A Failure of the vitelline duct to close

B Failure of the herniated intestinal loop to retract into the abdomen

C Failure of the urachus to close

D Failure of the midgut to rotate

E Failure of the hepatic duct to close

Explanation

Meckels diverticulum is a result of the persistence of the proximal part of the vitelline duct This

diverticulum is usually found about 2 feet proximal to the ileocecal junction and is usually about 2 inches

long It is present in about 2 of the popUlation It may be the site of ectopic pancreatic tissue or gastric

mucosa and may develop inflammatory processes and ulcerations Acute Meckels diverticulitis

simulates appendicitis

Which of the following veins carries blood from the esophagus to the portal vein The

A right gastric vein

B left gastric vein c splenic vein D azygos vein

E left gastroepiploic vein

Explanation

The left gastric vein a direct branch of the portal vein drains blood from the lesser curvature of the

stomach and the inferior portion of the esophagus Because branches of the portal vein do not have

valves blood can flow in a retrograde path when there is an obstruction to flow through the portal system or liveL Rlooci Cln then flow from the nortl] vein thr()1Ph the left PRstric vein to the esonhlPlIS lno

through venous communications within the submucosa of the esophagus to esophageal veins that drain

into the azygos vein The increase in blood flow through the esophageal submucosal veins results in esophageal varices

On the posterior wall of the abdomen the celiac ganglion A contains cell bodies of postganglionic parasympathetic neurons B is synapsed upon by neurons in the posterior vagal trunk C is synapsed upon by neurons in the greater splanchnic nerve D contains sensory cell bodies of lumbar spinal nerves E contains cell bodies of neurons that cause an increase in the rate of peristasis

Explanation The celiac ganglion is one of the preaortic ganglia of the sympathetic nervous system It contains cell bodies of postganglionic sympathetic neurons The sympathetic splanchnic nerves contain preganglionic sympathetic neurons that pass through the sympathetic chain without synapsing These splanchnic nerves go to the preaortic ganglia to synapse The greater splanchnic nerve contains preganglionic neurons from spinal cord segments T5-T9 This nerve synapses in the celiac ganglion The nerve fibers in the vagal trunks are preganglionic parasympathetic fibers that go to the walls of the organs that they will innervate and synapse on postganglionic parasympathetic neurons in the walls of those organs Cell bodies of sensory neurons in the abdomen are found in the dorsal root ganglia or the sensory ganglia of the vagus nerve Sympathetic innervation decreases the rate of peristalsis parasympathetic innervation increases the rate of peristalsis

Which of the following pairs of arteries will allow blood to bypass an occlusion of the celiac trunk

A Left gastric artery-right gastric artery

B Left gastroepiploic artery-right gastroepiploic artery

C Superior pancreaticoduodenal artery-inferior pancreaticoduodenal artery

D Splenic artery-common hepatic artery

E Left gastric artery - proper hepatic artery

Explanation The anastoOlosis of a branch of the celiac trunk and a branch of the superior mesenteric artery will

provide collateral circulation around an occlusion of the celiac trunk Each of the other choices pair

branches of the celiac trunk therefore these will not provide collateral flow around the obstruction of the

celiac trunk The left gastric splenic and common hepatic arteries are direct branches of the celiac trunk

The right gastric artery is a branch of the proper hepatic artery which is a branch of the common hepatic artery The left gastroepiploic artery is a branch of the splenic artery The right gastroepiploic artery is a

branch of the gastroduodenal artery whlch is a branch of the common hepatic artery

Which of the following organs has appendices epiploica The

A sigmoid colon

Bjejunum

C duodenum

D stomach E esophagus

Explanation Appendices epiploica are characteristic of the colon Appendices epiploica are subserosal accumulations

of fat None of the organs of the gastrointestinal tract has appendices epiploica except the colon

Page 19: Chirag's Abdomen Review

c Omental bursa

d Right subphrenic space

c

The inferior mesenteric artery arises from the abdominal aorta ilm_ediill~y_J-Qs1eriQLto which of the foowing org~ns A-F~t~filie duodenum B Head of the pan~eis C Neck of the pandeas

D Second part of the duodenum

E Third part of the duooenum_shylaquoshy

shy

The correct answer is E The inferior mesenteric artery arises from the anterior surface of the aorta at the level of the third lumbar vertebra The third part of the duodenum crosses the midline at the level of the third lumbar vertebra and passes anterior to the aorta at the origin of the inferior mesenteric artery The

first part of the duodenum (choice A) lies horizontally to the right of the midline at the level of the first

lumbar vertebra The head of the pancreas (choice B) is to the right of the midline and extends from the

level of the first lumbar vertebra to the third lumbar vertebra It lies within the concavity of the

duodenum The neck of the pancreas (choice C) lies in the midline at the level of the first lumbar

vertebra It lies on the anterior surface of the aorta at the origin of the superior mesenteric artery The second part of the duodenum (choice D) lies vertically to the right of the midline and extends from the

level of the first lumbar vertebra to the level of the third lumbar vertebra

The left adrenaLvein drains directly into which of the following veins A Hemiazygos vein

B Inferior vena cavaee C Left renal veiri -

D Splenic vein

E Superior mesenteric vein

a

The correct answer is C The left adrenal vein and the left gonadal vein (either testicular or ovarian) drain into the left renal vein TheTeft renal vein t~ains intothe- inferior vena cava In contrast the right

adrenal ~~inandnght gonadal veindrai~ gLr~ctJy iQtoJhe iilferiQ[ Vencava -- -

ThehemTazygoS7ein- (~h-~i-~ A)~~c~i~es the venous drainage from the body wall on the left side of the

thorax and abdomen No visceral organs drain directly to the azygos or hemiazygos veins The inferior vena cava (choice B) receives the direct venous drainage from the right adrenal vein but not

the left adrenal vein Remember the inferior vena cava is on the right side of the abdomen The splenic

vein (choice D) receives the venous drainage from the spleen and part of the pancreas and stomach The splenic vein is part of the portal venous system

The superior mesenteric vein (choice E) receives venous drainage from much of the intestinal tract It is part of the portal venous system and joins with the splenic vein to form the portal vein

A 43-year-old man presents complaining of pain in the groin On examination his physician palpates a

bulge in the region of the superficial inguinal ring which he diagnoses as a direct inguinal hernia The hernial sac most likely

A is covered by all three layers of the spennatic fascia B passes medial to the inferior epi gastric artery

C passes medial to the lateral border of the rectus abdominis muscle

D passes posterior to the inguinal ligament E passes through the deep inguinal ring

The correct answer is B Direct inguinal hernias enter the inguinal canal by tearing through the posterior

wall of that structure The typical location for this type of hernia is through the inguinal triangle bounded

laterally by the inferior epigastric artery medially by the lateral border of the rectus abdominis and

inferiorly by the inguinal ligament Direct inguinal hernias pass medial to the inferior epigastric artery

whereas indirect inguinal hernias pass lateral to the inferior epigastric artery because the deep inguinal

ring is lateral to the artery Indirect inguinal hernias are covered by all three layers of the spermatic fascia (choice A) Direct inguinal hernias are covered by fewer than all three layers because the direct inguinal

hernia tears through one or more layers of fascia as it emerges though the abdominal wall The lateral

border of the rectus abdominis muscle (choice C) forms the medial border of the inguinal triangle All

inguinal hernias pass lateral to the rectus abdominis Femoral hernias pass posterior to the inguinal ligament (choice D) Inguinal hernias emerge through the superficial inguinal ring which is superior to the inguinal ligament Inguinal hernias that descend below the inguinal ligament pass anterior to the

ligament Indirect inguinal hernias pass through the deep inguinal ring (choice H) direct inguinal hernias

do not Both types of inguinal hernias pass through the superficial inguinal ring

During a gastric resection in a patient with stomach cancer a surgeon wants to remove the lesser

omentum because of tumor extension into it Which of the following structures lie in the free edge of the

l~~g omentum and consequently must be dissected out in order to be preserved

A Common bile duct cystic duct and hepatic artery 6

B Cystic duct hepatic artery and hepatic vein

e Hepatic vein and cystic duct

Portal vein common bile duct and hepatic artery

E Portal vein hepatic artery and hepatic vein

The correct answer is D The free edge of the lesser omentum contains three important structures the

common bile duct the hepatic artery and the portal vein Nei ther the cystic duct (choices A B and C) nor the hepatic vein (choices B C and E) lies in the free

edge of the lesser omentum

A 55-year-old male patient with chronic liver disease has portal hypertension To relieve the pressure in the portal system a porto-caval shunt is performed Which of the following veins may by anastomosed to

accomplish this porto-caval shunt A Left renal vein-left testicular veingt

B Right renal vein-right suprarenal vein I shy

e Splenic vein -left renal vein J

D Superior mesenteric vein-inferior mesenteric vein E Superior mesenteric vein-splenic vein

The correct answer is C The splenic vein drains directly into the portal vein The left renal vein drains

directly into the inferior vena cava Anastomosis of these veins would allow blood from the portal vein to

drain retrograde though the splenic vein into the renal vein and then into the inferior vena cava The left

renal vein (choice A) drains directly into the inferior vena cava The left testicular vein drains directly into

the left renal vein Thus these veins are already in communication and neither vein is part of the portal venous system The right renal vein (choice B) drains directly into the inferior vena cava The right

suprarenal vein also drains directly into the inferior vena cava Thus neither vein is part of the portal

venous system The superior mesenteric vein (choice D) drains directly into the portal vein The inferior

mesenteric vein drains into the splenic vein which then drains into the portal vein Thus neither vein is

part of the caval venous system The superior mesenteric vein (choice E) drains directly into the portal

vein The splenic vein also drains directly into the portal vein Thus neither vein is part of the caval

venous system

A 12 year old boy has fever vomiting and para-umbilical pain After examining the patient the doctor

makes an initial diagnosis of appendicitis Appendicular pain which is initially referred to the umbilicus goes to the dorsal root ganglion of

a TI b TI2 c L1 d T7

(e I TIO

A 59-year-old male undergoes a neurological examination which reveals that when the abdominal wall is

stroked the muscles of the abdominal wall of the side of the body stimulated failed to contract Other

neurological tests appeared normal The likely region affected includes

a CI - C5 spinal segments b C6 - TI c T2-TI ~T8-T12

e Ll- L5

The surgery done to relive portal hypertension is done by connecting two veins Which of the following veins would be suitable for connection

a Inferior vena cava and portal vein b Superior vena cava and portal vein c Splenic vein and right renal vein d Splenic vein and left renal vein e Superior mesenteric vein and Inferior vena cava

A mother brings her 3-week-old infant to the pediatric clinic reporting a new scrotal bulge that she found -~-

while changing a diaper yesterday The infant is afebrile Physical examination reveals a palpable mass in

the scrotum while in the standing position resolution of the mass in the supine position and no

transillumination of the scrotal sac What is the most likely diagnOSiS

a Cryptorchidism b Direct inguinal hernia c Hydrocele d Indirect inguinal hernia ~ e varicocele

The Vagal trunks enter the abdomen by passing through which of the following openings in the

diaphragm

a Right crus b Esophageal hiatus ~ c Vena caval hiatus d Aortic hiatus e Left crus

2 The anterior boundary of the epiploic foramen of Winslow is bounded by

a) First part of duodenum b) Lesser curvature of stomach c) Liver d) Hepato-duodenalligament v ~

3 The ilio-inguinal nerve is derived from

a TI2 ry b LI c L2 d L3 e L23

15 Surgically the structure used to suspend the kidney to the diaphragm is

a) Renal fascia b) True capsule c) Perinephric fat d) Paranephric fat

6 If there is portal obstruction because of carcinoma affecting the pancreas which of these of the

following signs would be present

a Caput medusae b Esophageal varices c Rectal varices c

d Pulmonary edema

7 In a sliding hernia the gastro-esophageal junction lies

a) At its normal position b) Below the normal position c) Above the normal position V d) None of the above

8 Which of the following structures is retroperi toneal

A transverse colon B spleen IJ2f6 C ileum D descending colon v r 1pound1111111

9 The renal angle is fonned lgtetween the 12th rib and ______ muscle

a Psoas major -middotshyb Erector spinae c Quadratus Iumborum d Diaphragm

10 The anterior structure at the hilum of the kidney is

a) Renal vein ~

b) Renal artery I middot~ I

c) Ureter d) Accessory renal artery

11 Because of origin of the muscle from the lateral one third of the inguinal ligament it

could not fonn the anterior wall of the inguinal ligament

a) External oblique b) Internal oblique c) Transversus abdominis_ d) Rectus abdominis

12 A large tumor mass impinges on the splenic artery and its branches as the artery pass out from below

the greater curvature of the stomach Branches o(which of the following arteries would most likely to

effected by the pressure on the splenic artery

a Left gastric b Left gastro-epipJoic c Right gastric d Right gastro-epipoloic e Short gastric_

13 A new born baby has projectile vomiting after each feeding It is determined that there is obstruction

of the digestive tract as a result of annular pancreas Annular pancreas is as a result of an abnormality in which of the following process

a Rotation of the dorsal pancreatic bud around the first part of duodenum b Rotation of the dorsal pancreatic bud around the second part of duodenum c Rotation of the dorsal pancreatic bud around the third part of duodenum d Rotation of the ventral pancreatic bud around the first part of duodenum y Rotation of the ventral pancreatic bud around the second part of duodenum

14 As the liver bud enters the ventral mesogastrium the region of the mesogastrium stretching from the

liver to the anterior abdominal wall is called

a Lesser Omentum b Greater Omentum ~ Falcifrom ligament d Lacunar ligament e Ligamentum teres of liver

16 A patient has absence of his 12th rib In such a patient if the doctor makes an incision to approach his

kidney mistaking the 11 th rib for the 12t he would end up injuring

Which of the following arteries is a direct branch of the gastroduodenal artery The

A right gastric artery

B left gastric artery

C inferior pancreaticoduodenal artery D left gastroepiploic artery

i E)right gastroepiploic artery --

E x pI a nation The right gastric artery is typically a branch of the proper hepatic artery The left gastric artery is a direct

branch of the celiac trunk The right and left gastric arteries anastomose along the lesser curvature of the

stomach The inferior pancreaticoduodenal artery is a branch of the superior mesenteric artery it

anastomoses with the superior pancreaticoduodenal in the head of the pancreas The left gastroepiploic

artery is a branch of the splenic artery it anastomoses with the right gastroepiploic artery along the greater

curvature of the stomach The right gastroepiploic artery is a branch of the gastroduodenal artery The

other branch of the gastroduodenal artery is the superior pancreaticoduodenal artery

Which of the following pairs of veins join together to form the portal vein The

A superior mesenteric vein and inferior mesenteric vein

B inferior mesenteric vein and splenic vein

C superior mesenteric vein and splenic vein

Ip)splenic vein and left gastric vein E superior mesenteric vein and left gastric vein

Explanation

The portal vein is formed behind the neck of the pancreas by the union of the superior mesenteric vein

and the splenic vein The inferior mesenteric vein drains into the splenic vein The left gastric vein drains

directly into the portal vein After the portal vein forms it enters the hepatoduodenalligament of the

lesser omentum to reach the liver The portal vein is the most posterior structure in the hepatoduodenal

ligament

At which of the following vertebral levels does the duodenum pass anterior to the aorta - _- shy

All ~

B L2 7~

CL3 I

~DL4

E L5

Explanation

The duodenum begins at the pyloric sphincter at the level of Ll The second (or descending) portion of

the duodenum is to the right of the aorta and extends inferiorly from the level of Ll to the level of L3 The third part of the duodenum crosses the aorta from the right side to the left side at the level of L3 The

fourth (ascending) portion of the duodenum extends from the level of LJ to the level of L2 The

duodenum ends at the duodenojejunal flexure The superior mesenteric artery passes anterior to the

duodenum as the duodenum passes anterior to the aorta The duodenum can be constricted at this level

In which of the following locations will perforation of the digestive tract result in the spilling of luminal

contents into the - lesser peritoneal sac

A Anterior wall of the second portion of the duodenum B Posterior wall of the second portion of the duodenum

C Anterior wall of the stomach

~Posterior wall of the stomach E Posterior wall of the transverse colon

Explanation

The posterior wall of the stomach is related to the lesser peritoneal sac The anterior wall of the stomach is related to the greater peritoneal sac The anterior wall of the second portion of the duodenum is related to the greater peritoneal sac The posterior wall of the second portion of the duodenum is related to the retroperitoneal space The posterior wall of the transverse colon is related to the greater peritoneal sac

The ureter lies against the anterior surface of which of the following muscles shyA Crus oftne diaphragm B Quadratus lumborum

0 Psoas major D Transversus abdominis

E Iliacus

Explanation The ureter exits the renal pelvis at about the level of vertebra L2 As it descends along the posterior abdominal wall it lies on the anterior surface of the psoas major The psoas major muscle arises from the bodies of the lower lumbar vertebrae The psoas major muscle is joined by the iliacus to fonn the

iliopsoas muscle The iliopsoas muscle then attaches to the lesser trochanter of the femur and is the major

flexor of the hip

As the right ureter passes the pelvic brim it lies against the anterior surface of which of the following

blood vessels

A Gonadal artery B Inferiorvena cava C Internal iliac artery

rJ- External Iliac artery

E Inferior mesenteric artery

Explanation

The ureter lies in the extraperitoneal space in the posterior abdominal wall Alter leaving the kidney it

passes inferiorly on the anterior surface of the psoas major muscle At the pelvic brim the ureter passes

into the pelvis At this point the common iliac artery is dividing into the external and iliac arteries The

ureter lies on the anterior surface of the external iliac artery immediately distal to the bifurcation This is a useful landmark for a surgeon to locate the ureter

When extravasated urine passes from the superficial perineal space into the anterior abdominal wall it is

found immediately deep to which of the following layers of the anterior abdominal wall

-ltScarpas fascia

B External oblique muscle

C Internal oblique muscle D Transversus abdominis muscle

E Transversalis fascia

Explanation

The superficial perineal space is bound by Colles fascia the fibrous portion of the superficial fascia This

layer of fascia is continuous with Scarpas fascia the fibrous portion of the superficial fascia of the anterior abdominal wall Therefore urine that is deep to Colles fascia will remain deep to Scarpa s fascia The urine will spread in the plane between Scarpas fascia and the external oblique layer

When a horseshoe kidney develops the ascent of the kidney is restricted by the A internal iliac artery B external Iliac artery

C common iliac artery

inferior mesenteric artery

E superior mesenteric artery

Explanation

A horseshoe kidney develops when the inferior poles of the to kidneys fuse together as they ascend into

the abdomen from the pelvis The first anterior midline vessel that is encountered by the horseshoe kidney

is the inferior mesenteric artery This artery prevents the kidney from continuing its ascent

The left testicular vein drains into which of the following veins

A Left internal iliac vein B Left common iliac vein

bflnferior vena cava D Left renal vein I

E Left internal pudendal vein

Explanation

The left testicular vein drains into the left renal vein The right testicular ~i~[~nsltjectlY into the

inferior vena cava This difference in venous drainage is believed to explain the greater incidence of

varicocele on the left side than on the right The venous drainage from the penis is to the internal vein

which then drains into the internal Iliac vein

The spinal nerve that provides cutaneous branches to the skin around the umbilicus is

A TS B TW-shy

C TI2

DL2 EtA

Explanation

The tenth intercostal nerve is the anterior ramus of the TIO spinal nerve After passing through the tenth

intercostal space the nerve continues forward in the anterolateral abdominal wall in the plane between

the internal oblique muscle and the transversus abdominis muscle In the abdominal wall the nerve innervates to the abdominal wall muscles as well as the skin and the parietal peritoneum The umbilicus is

a useful landmark for the region of distribution of the tenth thoracic nerve

The ligament of the vertebral column that resists its extension is the Aligamentum flavum

B supraspinous ligament

C posterior longitudinal ligament

D anterior longitudinal ligament

E interspinous ligament

Explanation

The ligaments of the vertebral column that resist flexion of the column include the supraspinous ligament

interspinous ligament ligamentum fiavum and posterior longitudinal ligament The ligament that resists

extension is the anterior longitudinal ligament This longitudinal ligament is very broad and strong It

covers the anterior and anterolateral surfaces of the vertebral bodies and the intervertebral disks In

addition to resisting extension the anterior longitudinal ligament provides reinforcement to the anterior

and anterolateral surfaces of the intervertebral disk The posterior longitudinal ligament is relatively

narrow and covers the posterior surface of the vertebral bodies and the intervertebral disks This ligament

reinforces the posterior surface of the disk The posterolateral surface of the disk is not reinforced and it

is through this region that herniation of the nucleus pulposus usually occurs

A patient presents with epigastric and right upper quadrant pain The pain is most intense 2-4 hours after

eating and is reduced by the ingestion of antacids The patient states that he has passed black tarry stools

(melena) within the last week Fiberoptic endoscopy reveals a yellowish crater surrounded by a rim of

erythema that is 3 cm distal to the pylorus Accordingly an ulcer has been identified in the patients

A fundus

B antrum

C duodenum

D jejunum

E ileum

A number of physiologic genetic and other factors increase the risk of gastric (and duodenal) peptic

ulcers The evidence that H pylori plays a principle role is compelling Smoking and caffeine are known to adversely affect the morbidity mortality and healing rates of peptic ulcers In general first-degree

relatives of peptic ulcer patients as well as males have a threefold to fourfold increased risk of developing this disorder Paradoxically in gastric ulcer disease acid secretion is not elevated It is possible that

excess secreted hydrogen ion is reabsorbed across the injured gastric mucosa In general a defect in gastric mucosal defense is the more important local physiologic

A patient presents with symptoms of duodenal obstruction caused by an annular pancreas Annular pancreas is caused by

A rotation of the dorsal pancreatic bud into the ventral mesentery B rotation of the ventral pancreatic bud into the dorsal mesentery

fJ failure of the major and minor pancreatic ducts to fuse ~ ~ cleavage of the ventral pancreatic bud and rotation of the two portions in opposite directions around -the duodenum E formation of one pancreatic bud instead of two

Explanation Normally the ventral pancreatic bud rotates around the gut tube to reach the dorsal pancreatic bud The two buds fuse to form a single pancreas and the distal portions of the two ducts fuse The ventral pancreatic bud forms the inferior portion of the head of the pancreas the uncinate process and the major pancreatic duct (of Wirsung) The dorsal pancreatic bud forms the superior part of the head the neck body and tail and the minor pancreatic duct (of Santorini) Annular pancreas is the result of the ventral pancreatic bud dividing into two portions before it rotates into the dorsal mesentery Each portion rotates in opposite directions to get to the dorsal mesentery thus encircling the duodenum The presence of annular pancreas can constrict the duodenum thus obstructing its lumen

In n _ phranlc----

Gon ~l ----_1 Lum bltano

~~--- CornmQ1t bull ac

+-~4--- lnlllirnaJ ilic

xtem iliac

OBJECTIVE - Identify the blood supply to each of the structures listed in the table on the previous page

Ill give you a head start

FOREGUT - Supplied bV Celiac Tru nk (T12)

Proper hepatic

GastiooUod 13Jafter

1nferlor pancreaticoduodenal artery

Common epatlc

Lett gas ric iiirtery

Spfen artery

shy Gastroepiphgtic artery

~ Superior mesenteric 8rtfry

~

1 Esophagus is a derivative of the foregut so its blood supply originates from the celiac trunk

(T12) The predominant blood supply to abdominal portion of the esophagus is the Esophageal

A (Branch of L Gastric) The venous drainage of the esophagus is particularly important because

it is 1 of 3 clinically relevant sites of Portal Caval anastamoses The Portal Esophageal Vein

meets the Caval Azygos System Persistent bleeding manifests as Esophageal Varices - a fata I

condition

2 The Stomach is also a derivative of the foregut has EXTENSIVE blood supply and is very high

yield on anatomy exams The lesser curvature is supplied superiorly by the L Gastric A (1 of 3

major branches ofthe Celiac trunk) and inferiorly by the R Gastric A ( a branch ofthe proper

Hepatic A) The greater curvature is supplied superiorly by the L Gastroepiploic A (a major

branch of the splenic A) and inferiorly by the R Gastroepiploic A

The Short Gastric arteries (branches of Splenic Artery) supply the fundus of the stomach and

are referred to as EIID ARTERIES because they have no collateral blood supply Therefore if the

splenic artery were occluded (ex - increased pressure in the ommental bursa) - there would be

ischemia to the fundus of the stomach Venous drainage of the stomach is extensive via various

veins lead ing to the portal system Posterior to the stomach the IMV joins the splenic V which

joins the SMV to form the PORTAL VEIN ADAMS

3 Duodenum blood supply has high clinical relevance because it is the junction of the foregut and

midgut and therefore is the site of anastamoses between branches ofthe Celiac Trunk (main

foregut artery) and the Superior Messenteric Artery (main midgut artery) The Proper hepatic

artery gives off the gastroduodenal artery which travels behind the 1st part of the duodenum

This point has high clin ical relevance because duodenal ulcers are very common and a posterior

rupture of the 1st part of the duodenum could rupture the gastroduodenal artery causing

traumatic abdominal bleeding The Gastroduodenal artery first gives off the R Gastroepiploic A

(mentioned above) and proceeds as the Superior pancreatico duodenal artery (supplies the

pancreas and duodenum) which anastamoses with the inferior pancreatico duodenal A (branch

of the SMA) This is the junction of foregut and midgut and occurs near the opening of the

bil iary system into the duodenum (ampula of vater) Portal venous drainage here is responsible

for delivering nutrients from digestion to the liver for metabolism Appreciate that the Superior

mesenteric artery (artery of the midgut) branches from the aorta at Ll travels posterior to the

pancreas than moves anteriorly (at the jxn of the pancreatic headbody) and comes over the

3rd4th part of the duodenum Tumor of the head of the pancreas can compress the SMA

4 Jiver blood supply is via the common hepatic artery (major branch of the cel iac trunk) The

common hepatiC becomes the proper hepatic gives off the R gastric A and the Gastroduodenal

A and then joins the common bile duct and the portal vein in the portal triad Clinical- if a

patient were bleeding from the hepatic A a surgeon can stick his fingers in the epiplOic foramen

and squeeze the free edge of the hepatoduodenalligament in order to stop bleeding to the

area Please note that the hepatic a branches into Rand L hepatic A The Right hepatic artery

gives off the cystic artery which supplies the gallbladder Afferent venous supply is via the

Portal vein which is bringing nutrient rich blood to the liver After metabolism takes place

venous blood leaves the liver through the hepatic veins into the IVC PLEASE UNDERSTAND THE

RELATIONSHIP OF THESE STRUCTURES - ADAMSNETIERSNH Etc

5 Pancreas - Head is supplied via the superior and inferior pancreaticoduodenal arteries

(mentioned above) The tail (situated towards the hilum of the spleen) is supplied via the

pancreatic branches of the splenic artery (END ARTERIES) This blood supply is very important

because the endocrine Alpha and Beta Cells from the pancreatic islets of lagerhans are located

towards the tail This is where Insulin and Glucagon is released to the blood

Now complete this for mid and hindgut structures Make sure to note clinically relevant arterial

anastomoses as well as portal caval anastomoses FYI Appendix blood supply SMA + IMA

anastamoses marginal artery Portalcaval rectal veins fhemmorhoids) and periumbilical caput

medusa are high yield THE BUTT THE GUT and THE CAPUT

Abdominal Development

Liver

Ij1f

II wall b

oh liN ~ VltJrti n be- bull

Pancreas

Secondary Retroperitonealization e I~tl r 1 a v-mtrai m ellter

Rotations of the Gut I i Ij (lIl1UtIJ f~ l r tilt

()l td 10 me l-ft and he v

--~--- -~ -~-~

i

I AolaijonjoI~guf I

STOMACH BED (IDENTIFY IN ADAMS)- the structures posterior to the ommental bursa which

support the stomach in the supine position

Abdomnal JQrUI

Splnic vein

OmQ-oul tv~ ) O(s(Jroa)

Lojt(r o m nturrt (hpJtodu o d~n31 Hid

Gadrl)SplerH (g3stroll~nal) IIgam~nt

hiad h~~atogrtricent IIQdmiddotcrt~)

Lt Dome of Diaphragm (why left Look this up in Adams)

Spleen (What is the blood supply)

Left Kidney (What is the blood supply - AND how is it different from the R kidney)

Suprarenal Gland (What is the Arterial AND Venous Blood supply - how are they different)

Pancreas (How does supply differ from Head to Tail What is the SMA Relationship)

Transverse Mesocolon

liver - ADAMSWET - Make sure you look at the liver in wet lab

Left triangular nl1am~nt

ComoaDj ligamnt

Erophg~1 impre$ioo

Hepatio veins

In1erior -ifena middotr3)Ia

Fibrous appendix o-t

live

impr~j on

Heprorendl p~rtion of Q)(Qllary ligament

Righllri~n9ul r 1I~met

(Common) bile quol

Gr)mmCtr~ hepatic dlJct

Ccentic duct

Duodenal impression

GaJdate p-fr)~S

Hepatic artgtrl prop-f iiiiila - Faloiform ligament

_ - shy Round ligamen liver

~--F-- CoJio imprgt-ssi-on

Prta heptis

Identify the lobes impressions and embryonic remnants associated with the liver

Caudate Lobe Quadrate Lobe Right Lobe Left Lobe Round ligament Falciform Ligament

Ligamentum Venosum (what is its fxn in embryonic life) Hepatic Veins (NOT PART OF THE

PORTAL TRIAD) IVC PORTAL TRIAD - Contents relationship cross section etc Know the

Galbladder relationship to the lobes of the liver

Biliary Duct System - Make sure you understand the sequence of these structures - BE ABLE TO

DRAW A FLOW CHART

TPVd i

t

I t

1 __ Cm-(r

patk GlJet

I

J

Clinical = JAUNDICE is caused by anything that prevents delivery of bile to intestine Tumor of the

head of the pancreas Stones etc Patient will have pale stools and yellowish colored mucus

membranes

Clinical- Any scenario that tells you the patient has BILLOUS VOMIT means that the obstruction to

the flow of digestive contents is after the Ampulla of Vater (Site of Entry of Billiary system to the

duodenum) - ie Duodenal Atresia

Spleen -located posterior to the mid axillary line between ribs 9 and 11 Make sure you know that

the 10th rib is the main axis of the spleen and this organ is susceptible to injury (stab wound errant

thoracoce ntesis etc)

The spleen is derived from mesodermal cells - NOT THE GUT TUBE

The spleen rests on the left colic flexure associates with the tail of the pancreas Know the

structures entering the Hilum of the spleen

Sh rt O~-t~ic 1 0(0 10 rtiltSPIric Iloa nt

(cut)

Peritoneum - similar concept to Pleura - think of a fist in a balloon

Visceral Peritoneum - Layer of balloon touching your fist

Parietal Peritoneum - Layer of balloon not touching your fist

Your fist represents the organ your wrist is the hilum and your arm contains the blood supply

entering the organ

Appreciate that there will never be organs in the peritoneal cavity - rather these organs invaginate

the cavity Kaplan videos

RULES OF NOMENCLATUREshy

1 Organ completely surrounded by peritoneum - peritoneal organ

2 Organ partially surrounded by peritoneum- Retroperitoneal

3 Peritoneum surrounding peritoneal organ is VISCERAL peritoneum

4 Peritoneum surrounding retroperitoneal organ is PARIETAL peritoneum

5 Peritoneum connecting visceral to parietal is called messentary 2 messentaries in the

gut Dorsal (to the gut tube) and ventral (to the gut tube) messentary

Aorta is in Retro peritoneal position - but blood must reach peritoneal position - vessels travel through

messentary All peritoneal organs will have blood supply reaching through messentary

-Mesentery is a 2 layer peritoneum with a neurovascular communication between body wall and organ

- Ligament connects one organ with another or to the abdominal wall (Ommentum = ligament)

lesser Ommentum (attach lesser curvature of stomach and duodenum to liver) =Hepatoduodenal

Ligament and Hepatogastric Ligament

Has a Superior and Inferior Recess (Accumulation of Fluid in Ascites)

Communicates with the greater sac through the epiplic foramen (what structures pass through

this foramen)

Boundaries - you must be able to visualize this

o Anterior - stomach

o Posterior - parietal peritoneum pancreas

o Superior - superior recess (bw diaphragm and coronary ligament)

o Inferior -Inferior recess (bw layers or greater momentum

Greater Ommentum (attach greater curvature of stomach) Gastrophrenic ligament Gastrosplenic

ligament gastrocolic ligament

The greater omentum is the largest peritoneal fold It consists of a double sheet of peritoneum folded on itself so that it is made up of four layers The two layers which descend from the greater curvature of the stomach and commencement of the duodenum pass in front of the small intestines sometimes as low down as the pelvis they then turn upon themselves and ascend again as far as the transverse colon where they separate and enclose that part of the intestine

ABDOMINAL PAIN

Parietal Peritoneum - supplied by same vasculature lymphatics and nerves supplying body wall it

lines and diaphragm Sensitive to pain pressure heat cold well localized

Visceral Peritoneum - supplied by same vasculature lymphatics and somatic nerve of organ it covers

Insensitive to touch heat cold and laceration - referred to dermatome of spinal ganglia providing

sensory fibers Where does appendicitis refer to

Foregut pain - epigastric area (ie - cholycystitis)

Midgut pain - periumbilical area (ie - appendicitis)

Hindgut Pain - suprapubic area (ie - diverticulitis)

Extra ImagesConcepts

ll~_____-

FalifCtrm ligament oind r~ud ligamet f Ilver

Blood from splenio gastriC and inferiof rne$e-rteri v~ins

Ca-I tributaries

Lett gastrio Ifein

Posterior superior pan~reatioodul)denal vaihS

Lott gamo-om~nlal (9aropip lomiddotic) -in

Poq_~ tjol imerl-9-r panCJertlcorllJod-nal veiopound --amp----I- - ~J Right grtr~-omntal

Anwrior interi (gartroepiploic) Jjn

pan euaii cod vl)denal veins middot Inf~Ji (t r mesentric vein

Miqdle (olic vein

Right cl)licvein Sigmoid and rectosigml)id (ei ns

IhH)Collc(~io

--- Mi~dl laquooLJl gtjrltgt

PoM ca vl1 illasto)moses -----shyampoptoageal 2 Paraumbilie-lt11 Inferi or Fectal vei ns

3 Recial 4 REuoperHonea1

Know how the Portal vein is formed I 4 sites of portal caval anastamoses and 1 clinical shunt

Col li t ltt-~ otTl~tI ~nj pc~ 1lt1 turJoG

Ltf 14i1 tImiddot~ artoftl9 on tj phtAt$

L-oftqf 4t t~r 1=laquoIran d 1 bull shy~p l ci rj o fOOOts

Nerves follow the arteries - appreciate the splanchnic nervous system I

Uet~ric branch of left ~nal art

Ureterie branch of righi renal artelY

Left Zld lumbar in and co mlTlunication to as)erdin9 lumbar l(~in Hi ~ht tEZ1~~t~ t3r j t itn ~ nJ l1t- rlnd lfe i r1

Inferior me5nteri~ artery

Notice that the right testicular vein drains directly into the IVC and the right testicular artery drains

directly into the aorta However the left testicular vein drains into the L renal vein at a right angleshy

reason left testicle is lower and more susceptible to varicocele (bag of worms)

Also notice that the left renal vein has a longer course because the IVC is on the right side whereas

the right renal artery has a longer course because the aorta is on the left side

Appreciate the anterior to posterior relationship of structures in the hilum of the kidney - VAP - Vein

Artery Renal Pelvis (Ureter)

11____ __ L_ L_ n VJ __ _ _ t_L I I_ _ L __ L_ I -pound1 bull LI_~-I ____

Posterior View of Head of Pancreas in ( of Duodenum

Celiao hunk

Co mmon ~L~jJth art~ry

GastNduQdonal artrf (partilly in phantn)

P1)Sterior $Up~Jior panCflaticuduodfmal art~r~t

(Co mm on) bile duct

middot~1t~~t-1l---~-~- Right gshomiddotomental (gastoe plp lolc) 3rte (phantomost)

Grener paocre atic art-ry

1n1~rjor pancr-iatlc artery

Jtrifll supejo r pal)oreailcento)dJodenal artr1 (phantom)

Anastomotlo branch

POostetlor bJanch of jo f~ri of pan-reatir(lduodensl drttnj

Anterio r branch of i flferior palcreati~)duodenal art~(phan1om)

Notice the extensive blood supply to the pancreas and duodenum via the branches of the celiac trunk

Notice collateral supply from SMA branches - makes sense bc this is the jxn of foregutmidgut

Identify the vessels in this arteriogram

Hiltid i)f N~ck oi B)dvof Tail 01 pa nereas pan cent~as P-nmiddot-reas panCtCas

I nferie v~na cava

jHept1iic p(lrlai v~in

Port1 tnd H~pti lt a ftH prol

Comm on) bll duct

Ouodtnum

~ft colic (sio)Atta~ hmtrlt jt~xJr-ofha~elSe

muo(IIQn

Right ~lIc (h~j)tic)

il~gtture

In1triol m~oten lIein (rttr op~ritoMdO

SlJp efl or mes~n~fiC amrV and lipln

KNOW YOUR NEIGHBORHOOD

Questions

vVhiJh structure supplied by a bnmdlof the cclia( artery is not derivcd from foregut LemCJUCrITI

(A) Head of the pancte-a5

CD) Pyloric duolenum

Cystkduct

( Liver hepatocyt~~

~F) Body of the spleen

An infant presents with an omrhaJucele at birth -hi oJ the [oHm illg applies to his cM1-dition

(A) It is 31so seen ill p4titnts with aganghonic megacolon

(11) ft reuirs from a fal1ure of resorption of theviteUine d let

(C) It results from herniation at the-site of regression of the right umbilk vein

DJ It is caustd by faihtrc of recanalization of the midgut part of the duodenum

~ It ill camioo by a failuIt vf the midgul to return to the abGQminal uity after herniashytion in-n the urnbilk s l stalk

Ot er than the spleen occlusion Cif the spit-Ilk artery at its odgin wm most likely affect die blood supply to jllch st cnud

(A) Jejunum

(B) Body of th pal1~lltas

(C) LeSStT Cllmiddotlaturc of tl )toma-ch

(D Duodenum dista to the entrance of the Ornmou bile duct

E Fundus of the stomach

A 38-yeu-old batL~er with a history of heartburn suddenly experiences excluciating pain in the (plgastric region of th~ abdomeu SurgCry is perf~rme immediard y upon admisshysion to the 1IlcrgCJliy tuomh~re i~ evidence uf a ruptured ulcer in the posterior waU of the stomach Vhere will a surgeon first fi nd the stomach contenlSf

A) Greater p4ritoneal sac

rB) Cul~de-s~c of Douglas (--

C Omental bursa ~

--D) Paracolic gutter

rEj Between -he panttal perimltum and the posterior body wal1

At birth an infant presents with a st()ma~ rb~tbas~njJled jfltotb~diaplfagru 1A1ltre is the defect thatresulied iiitJle heini~t()n shy~tsophagealbiatus

7 - rH-- Hiatus for the inferior vena cava

( Pleuroperitoneal membrane -(0) Septum transvcrsum

(E) Right Crlt~

An infant born with DOVv7l syndrome presents with bili()u~ vomiting Ahat congenital defect does the infant have

(A) Pyloric stenosis

(B) Meckel diverticulum C) Ornphaloce1e

(D) Gastroschisis

( ~ ) Duodenal atresia y A patient with cirrhosis of the liver presents with ~ bacalvaricestnlreased retrograde pressure in which veins caused the varices

(A) Paraumuilical

(B) Splenic

(ct AzygltJus

(15))G~trk ( (-F) Superior mesemeric

A htaltby 3-year~old male patient experiences a hernial sa protruding from the anterior abdominal wall about halfway between me anterior superior ilia spine and the pubk tuberde Pulsations of al1 artery are palpated medial to the protrusion site through the abdominal walL Which layer of the anterior abdominal wall will first be traversed by the

1hctma

fA) Rectus sheath (B) External oblique aponeurosis

(C) Inguinal ligament

lD) Transversalis fusda

(E) Cremasteric fa~cia

After 5urgi(aj ffpair of a hernia the patient tXperienccs mtmlgtness in the skin on the anteshyrior aspect of the S(Totum_ Vhaf nerve may have been lesioned during thehemiorrhaphy

(A) Femoral

(B) Obturator

(C) Ilioinguinal

(D) lliohypogastrk

(E) Pudendal

A 23~year-LJld female secretary il1 good health ~-uddcn1) doubles over with pain in the a ea of the 1JmbRicu$ Sbe feels vartn and ltneasy and has no appetite That night the pain seems to have mQved to the tower right abdominal regjol1 and she calls her family doctor who then arranges for an ambulance to pk-k her up and take her to the hospitaL Wh ell ntn~ perceived in the area of the urnbilirus most Hkely carried lhe pairfu I sensations into the eNS

tA) Vagus nerves I~

V B)

) Lessersplanchnk nerves

tC) Pudendal nerves

(D) lIiohpogastrk nerves

(E) Greater splam ic l erves

A CT reveals carcinoma in the bOod of the ancreas Vhich blood vessel trut ourses ----~- - -bull ------ --shy

immediately poftterior to the body ofthe pancreas is the m~t likely to be oompressed

(A) Splenk artery

(B) Abdominal aorta (C) Portal vein

(1) Splenic vein

(E) Renal vein

A patient has a penrln1l1ng uker of the posterior wall ot the br~l part ot the (lUooenmn llkh blood vessel is subject to erosion

(A) Common hepatic artery

(B) Gastroouodenal artery

(C) Proper hevatic artery

(D) Celiac artery

(E) Anterior inferior 11amrelltlcoduodcnal attery

Your patient has been diagnosed -ith a carcinoma locallted to the head and l~e(k of the pancreas Another clinical sign would be

A esophageal varices

(8) hemorrhoids

C) a caput medusa

(D) increased pra Teuro n th~ hepatic veins

(E) enlarged right supra lavkular lymph nodes

Wltkh of the foUowing structures develops in the ventral mesentery

(A) Spleen

(B) Jeiunum (C) Head of1ht pancreas (D) Transverse colon (E) Stomach

ti l Uw ~ littwin~ f( S-t lil oai Imdge ~ hi(h or tbt la~)d J truetur tgt liJ llntn nl) he hl p UC iJd [IIi ell

c o

A) drains Ie tht infCrior a La aI

R t middot~nfl0 ~ill to th~ lunlgtn of h i dtlndCrlllfH

(e) m t bull JiJattd on tl l J n T ~H

D ) sup Lc O VSlt I Hlid bhtu l 1 li - -I un oid

( ) U~tpli(t tr j middottUh~ 1 v(( b~nt rfK n1ilc~Zm

ANSWERS AND EXPLANATIONS

Answer E The spleen is t hlttnopodicand lymph organ demlted from mesoderm

Answ~ R Al1 tlmphalocele is caused by it failure of the nlidgut to return to the ahdomir nat cavity after herniation into the umbiliau Stalk Choices Aand D maybe seen in infants with Down syndrome choice D ~s the specific CBuse ofduudcnal JtiCSitt Choice C is (ile cause of gclstrosbisis and Choice B nsults iu a Meurolktldivertku1-tlB

Answer B The fundus ofthe stomach is suppHed by soort gastric brunches of the splenic altery The splenic artery supplies the body and tail of the pancreas part of the greater curvature of the sttmla(h and the spleen Te jejunum part of the head of the pancreas and tht~ duodenum distal to the entrance of the commOll bile duct are supplied by the superior mesenterk artery clll~l ~be less r ctlt1ature cmd the pylQric antrum are supplied by the right and lei gastric art(ries

AnSWftt C Tbeomental bursa or lesser ~ritoneaj sac lies direcdy posterior to the proxshyimal part of the duodeTtlm and the stomach and would be the first site where stomach contents ~Ott1d be fpoundluncL

Answer C A defect in a llleuropcritoneal membrane (uswlly the left) is the typical site of i1 cc-ngenitlI diilphragluatic hemia llere the membr4ne fails to dose ()pound( of the perishycCirdiopcritulleal canals

Answer E DuoJenal atresia and aganglionic megacoion are congwitaI defects S~Il in patients with Dowmiddotnsyndrome

Answer D RulaTgemt~llt of and retrograde flow in g~lstrk vel_ns in particlJl~r the kft gas~ tricveins dilates the capillary bed in rhe wall of the esophagus in (ases of porta yper~

tension Blood flow would increase in and dilampte tribntarkgts of the (lZygOUS vein on the other side of the capiUary bed but flow in this vein is in the typical direction t()ward the superior vena cava Paraumbiii(ltU vein eilgorgement contributes to a caput medusH Splenic ~nlargement might prc~nt with 5plcnonlegaly and balt-kflow in to tlu superior m~~ntclic vein occurs but is asymptomatic

Answer D The patient hagt an indirect inguinal hernia whi~h emerges from the antt-rior abdominal wall through the deep inguinltilling Theeep ring is a fault in the transv~rshysaUs fascia this I~yer wiIJ be penetrated first by the hernia

An~Wer C The ilioinguinal nenc which provides sens~llion to the lnedlal thigh ltmclanteshytior SClotunl pass~lt th rough the 5uperfh_ial inguinal ring ind $subject to inj i1T) becaus-e

it is in the operatitm Held of the erniorrhapny

Auswer B The leMHr splanchnic nerves are sympathdic nerVlts that carry viscera l sensashytlltgtrogt ftom illtllt1m~d ()J stietched gust (itinteitinal ~tructures (in this case the pprndix) into tnt eNS Lesser splanchnic ntTYcsarisc from thmiddot T9--T12 spinal cord segments lt1nd provide sympathetic innenation tD rnidgut siruc1ures whiCh include CLe app~JldD Viscera] Pain arising from affecLed Inidgut ampt 1C1ure is referred over the same dl- matorne~ of spinal segrnertts v-hich provide the sympathetic Innervation n this G1SC of appendicitis the invohen~n t of the ltire) of t e unlhHku indud s the T 10 dermatome

Answer B Of the five choices onty the dscending olon is retroperiton~al aldwould be a lik ~ ( choice to be seen immediately a(~jilcent to t11e posterior abdominal middotn~L

Amwen D The SpltftlC ~-ein ourses posterior to the body of the panneas m its way tt drain into the superior mCSfttltlri( vein

Answcr B TILt glstrodllolticnal artery 1 direct hIamh of the comrootl hepatic artery courses immediately pt))iwri() to the duodenum and is slbject to erosion

Answer B Carcinoma of th pan middott3S in the 1tilt1 may compreampgt the portltil vein at irs orishygill The poTtai vcin is fomled when the splenic vein jQiaswith tfie superior meStllt eric vein The inferiot mesenteric vein joins the ~plenjc vein just priOT to tlli~ point at which the splenic joins the superior Jlleit1ltcri( vein Increescd venous presslu in the inferior mesenteric vein is a cause of emo hoid~

Answer C The- velltral pancreas wilich forms most of the head of the p ~ncr as develops in the ventral mes(ntery as antutgrowth of the hepatic diverticulum Th~ hepatic divershyticulull induding the biIJary appa~atus develops in tbe ventral mesentery of the foregut

Answer~ A The superior mesenteric ~in joins with the spienkvein to form the hepatic portal vciu

Answer D The structure at gttlK is the proper hepatic artery~ whkh suppUesoxygenated b middotood to the liver

MAKE SURE YOU KNOW the diff bw Rectus Sheath above and below the arcuate line

ABOVE

Aponeurosis of xiiltmal obllque musclo

Extemll f)biquw musde

Reotln ilbdomlnls musole S~in

Internal 9bliquQ mY~QI

AponeUfOsi$ of hJH$V~~S Lir9a a lb lbdolTlin~ musolo Tri OJV6 rUi

atldomlnis mUS(loe

Sub cutanlilous tiue (tatty ye r)

BElOW

A POrl lJfosis 01 etemal oblique muscle

Aponeul~)sis 01 Internal oblique mU$cl~

Anteriol lay~ of r~ltdus st~ath EXttom1 oblique rnu$cll

Rectus Jbdominis muscle Intoernal Aponeurc-sis of tra~fersU$ oblique muscle-

at-domlnis muscentl ~ Skio

Tra nsvitSus abdomioLs ml)ZClt

TralSVersaHs fascia Medial umQil iegtt1 1i9Jment -and folj

Uldchus Peritoneum (ir median Umbilj~al Suboutane ous

Extraprftone 11ascia

Ymbilimiddot~1 fold)

preu9poundiea1 fascia

tissue (fatty 4nd m~mbr3n(iUS layers)

o Above the arcuate line (A horizontal line 13 of the distance bw the umbilicus and the

pubic symphysis) -10 Aponeurosis divides into an AntPost Laminae

o The Ant Laminae joins EO and Post Laminae joins Trans Abdominis = Ant and Post

RECTUS SHEATH respectively

o BElOW the arcuate line - all 3 aponeurosis join ANTERIOR to rectus muscle to meet its

counterpart in the midline (linea Alba)

o Take away Msg - The abdomen is devoid of a posterior rectus sheath below the

arcuate line and is therefore more vulnerable to herniasinjuries

Question - A physician makes a deep incision in the patients midline immediately superior to

the pubic symphysis which of the following layers is his knife least likely to pass

Rectus Abdominis External Oblique Ant Rectus Sheath Posterior Rectus Sheath All of the

Above

Answer - All of the above None of the other answer choices are midline structures -LINEA

ALBA

Linea Alba has very poor blood supply - doesnt heal well after surgery Therefore this is a

common site for incisional hernias

a Spleen b Transverse colon c Descending colon d Stomach e Pleura

17 Meckels diverticulum is normally found 2 feet proximal from the

a Pyloric sphincter b Lower esophageal sphincter c Ileo-cecal valve d Middle valve of Huston e Anal valve

18 Ulcer in the posterior wall of the first part of the duodenum would erode ___ artery and would cause bleeding

a Left gastric b Right gastric c Hepatic artery proper d Gastroduodenal artery e Middle colic artery

19 An inflamed appendix is identified by a surgeon on the operation table by noting

a The appendicies epiploicae b The convergence of tenia c The artery of Drummond d The mesocolon e The mesosalphinx

20 The nerve which emerges through the psoas major is

a Femoral b Ilio-inguinal c Ilio-hypogastric d Pudendal e Subcostal

21 The right gonadal vein drains into the

a Azygos b Hemiazygos c Inferior Vena Cava d Right renal vein e Left renal vein

22 The hepatocytes in the liver is derived from

a Ectoderm b Endoderm c Mesoderm

d Neural ectoderm

23 Abscess in the lumbar vertebrae due to tuberculosis would spread to the adjacent muscle which is

a Psoas Major b Iliacus c Quadratus lumborum d Tranversus Abdominis

24 The anterior wall of the inguinal canal is formed by

a External oblique and transverses abdominis b External oblique and fascia transversalis c Internal oblique and external oblique d Internal oblique and transverses abdominis e Fascia transversalis and peritoneum

Meckels diverticulum is a result of which of the following developmental abnormalities shy

A Failure of the vitelline duct to close

B Failure of the herniated intestinal loop to retract into the abdomen

C Failure of the urachus to close

D Failure of the midgut to rotate

E Failure of the hepatic duct to close

Explanation

Meckels diverticulum is a result of the persistence of the proximal part of the vitelline duct This

diverticulum is usually found about 2 feet proximal to the ileocecal junction and is usually about 2 inches

long It is present in about 2 of the popUlation It may be the site of ectopic pancreatic tissue or gastric

mucosa and may develop inflammatory processes and ulcerations Acute Meckels diverticulitis

simulates appendicitis

Which of the following veins carries blood from the esophagus to the portal vein The

A right gastric vein

B left gastric vein c splenic vein D azygos vein

E left gastroepiploic vein

Explanation

The left gastric vein a direct branch of the portal vein drains blood from the lesser curvature of the

stomach and the inferior portion of the esophagus Because branches of the portal vein do not have

valves blood can flow in a retrograde path when there is an obstruction to flow through the portal system or liveL Rlooci Cln then flow from the nortl] vein thr()1Ph the left PRstric vein to the esonhlPlIS lno

through venous communications within the submucosa of the esophagus to esophageal veins that drain

into the azygos vein The increase in blood flow through the esophageal submucosal veins results in esophageal varices

On the posterior wall of the abdomen the celiac ganglion A contains cell bodies of postganglionic parasympathetic neurons B is synapsed upon by neurons in the posterior vagal trunk C is synapsed upon by neurons in the greater splanchnic nerve D contains sensory cell bodies of lumbar spinal nerves E contains cell bodies of neurons that cause an increase in the rate of peristasis

Explanation The celiac ganglion is one of the preaortic ganglia of the sympathetic nervous system It contains cell bodies of postganglionic sympathetic neurons The sympathetic splanchnic nerves contain preganglionic sympathetic neurons that pass through the sympathetic chain without synapsing These splanchnic nerves go to the preaortic ganglia to synapse The greater splanchnic nerve contains preganglionic neurons from spinal cord segments T5-T9 This nerve synapses in the celiac ganglion The nerve fibers in the vagal trunks are preganglionic parasympathetic fibers that go to the walls of the organs that they will innervate and synapse on postganglionic parasympathetic neurons in the walls of those organs Cell bodies of sensory neurons in the abdomen are found in the dorsal root ganglia or the sensory ganglia of the vagus nerve Sympathetic innervation decreases the rate of peristalsis parasympathetic innervation increases the rate of peristalsis

Which of the following pairs of arteries will allow blood to bypass an occlusion of the celiac trunk

A Left gastric artery-right gastric artery

B Left gastroepiploic artery-right gastroepiploic artery

C Superior pancreaticoduodenal artery-inferior pancreaticoduodenal artery

D Splenic artery-common hepatic artery

E Left gastric artery - proper hepatic artery

Explanation The anastoOlosis of a branch of the celiac trunk and a branch of the superior mesenteric artery will

provide collateral circulation around an occlusion of the celiac trunk Each of the other choices pair

branches of the celiac trunk therefore these will not provide collateral flow around the obstruction of the

celiac trunk The left gastric splenic and common hepatic arteries are direct branches of the celiac trunk

The right gastric artery is a branch of the proper hepatic artery which is a branch of the common hepatic artery The left gastroepiploic artery is a branch of the splenic artery The right gastroepiploic artery is a

branch of the gastroduodenal artery whlch is a branch of the common hepatic artery

Which of the following organs has appendices epiploica The

A sigmoid colon

Bjejunum

C duodenum

D stomach E esophagus

Explanation Appendices epiploica are characteristic of the colon Appendices epiploica are subserosal accumulations

of fat None of the organs of the gastrointestinal tract has appendices epiploica except the colon

Page 20: Chirag's Abdomen Review

A is covered by all three layers of the spennatic fascia B passes medial to the inferior epi gastric artery

C passes medial to the lateral border of the rectus abdominis muscle

D passes posterior to the inguinal ligament E passes through the deep inguinal ring

The correct answer is B Direct inguinal hernias enter the inguinal canal by tearing through the posterior

wall of that structure The typical location for this type of hernia is through the inguinal triangle bounded

laterally by the inferior epigastric artery medially by the lateral border of the rectus abdominis and

inferiorly by the inguinal ligament Direct inguinal hernias pass medial to the inferior epigastric artery

whereas indirect inguinal hernias pass lateral to the inferior epigastric artery because the deep inguinal

ring is lateral to the artery Indirect inguinal hernias are covered by all three layers of the spermatic fascia (choice A) Direct inguinal hernias are covered by fewer than all three layers because the direct inguinal

hernia tears through one or more layers of fascia as it emerges though the abdominal wall The lateral

border of the rectus abdominis muscle (choice C) forms the medial border of the inguinal triangle All

inguinal hernias pass lateral to the rectus abdominis Femoral hernias pass posterior to the inguinal ligament (choice D) Inguinal hernias emerge through the superficial inguinal ring which is superior to the inguinal ligament Inguinal hernias that descend below the inguinal ligament pass anterior to the

ligament Indirect inguinal hernias pass through the deep inguinal ring (choice H) direct inguinal hernias

do not Both types of inguinal hernias pass through the superficial inguinal ring

During a gastric resection in a patient with stomach cancer a surgeon wants to remove the lesser

omentum because of tumor extension into it Which of the following structures lie in the free edge of the

l~~g omentum and consequently must be dissected out in order to be preserved

A Common bile duct cystic duct and hepatic artery 6

B Cystic duct hepatic artery and hepatic vein

e Hepatic vein and cystic duct

Portal vein common bile duct and hepatic artery

E Portal vein hepatic artery and hepatic vein

The correct answer is D The free edge of the lesser omentum contains three important structures the

common bile duct the hepatic artery and the portal vein Nei ther the cystic duct (choices A B and C) nor the hepatic vein (choices B C and E) lies in the free

edge of the lesser omentum

A 55-year-old male patient with chronic liver disease has portal hypertension To relieve the pressure in the portal system a porto-caval shunt is performed Which of the following veins may by anastomosed to

accomplish this porto-caval shunt A Left renal vein-left testicular veingt

B Right renal vein-right suprarenal vein I shy

e Splenic vein -left renal vein J

D Superior mesenteric vein-inferior mesenteric vein E Superior mesenteric vein-splenic vein

The correct answer is C The splenic vein drains directly into the portal vein The left renal vein drains

directly into the inferior vena cava Anastomosis of these veins would allow blood from the portal vein to

drain retrograde though the splenic vein into the renal vein and then into the inferior vena cava The left

renal vein (choice A) drains directly into the inferior vena cava The left testicular vein drains directly into

the left renal vein Thus these veins are already in communication and neither vein is part of the portal venous system The right renal vein (choice B) drains directly into the inferior vena cava The right

suprarenal vein also drains directly into the inferior vena cava Thus neither vein is part of the portal

venous system The superior mesenteric vein (choice D) drains directly into the portal vein The inferior

mesenteric vein drains into the splenic vein which then drains into the portal vein Thus neither vein is

part of the caval venous system The superior mesenteric vein (choice E) drains directly into the portal

vein The splenic vein also drains directly into the portal vein Thus neither vein is part of the caval

venous system

A 12 year old boy has fever vomiting and para-umbilical pain After examining the patient the doctor

makes an initial diagnosis of appendicitis Appendicular pain which is initially referred to the umbilicus goes to the dorsal root ganglion of

a TI b TI2 c L1 d T7

(e I TIO

A 59-year-old male undergoes a neurological examination which reveals that when the abdominal wall is

stroked the muscles of the abdominal wall of the side of the body stimulated failed to contract Other

neurological tests appeared normal The likely region affected includes

a CI - C5 spinal segments b C6 - TI c T2-TI ~T8-T12

e Ll- L5

The surgery done to relive portal hypertension is done by connecting two veins Which of the following veins would be suitable for connection

a Inferior vena cava and portal vein b Superior vena cava and portal vein c Splenic vein and right renal vein d Splenic vein and left renal vein e Superior mesenteric vein and Inferior vena cava

A mother brings her 3-week-old infant to the pediatric clinic reporting a new scrotal bulge that she found -~-

while changing a diaper yesterday The infant is afebrile Physical examination reveals a palpable mass in

the scrotum while in the standing position resolution of the mass in the supine position and no

transillumination of the scrotal sac What is the most likely diagnOSiS

a Cryptorchidism b Direct inguinal hernia c Hydrocele d Indirect inguinal hernia ~ e varicocele

The Vagal trunks enter the abdomen by passing through which of the following openings in the

diaphragm

a Right crus b Esophageal hiatus ~ c Vena caval hiatus d Aortic hiatus e Left crus

2 The anterior boundary of the epiploic foramen of Winslow is bounded by

a) First part of duodenum b) Lesser curvature of stomach c) Liver d) Hepato-duodenalligament v ~

3 The ilio-inguinal nerve is derived from

a TI2 ry b LI c L2 d L3 e L23

15 Surgically the structure used to suspend the kidney to the diaphragm is

a) Renal fascia b) True capsule c) Perinephric fat d) Paranephric fat

6 If there is portal obstruction because of carcinoma affecting the pancreas which of these of the

following signs would be present

a Caput medusae b Esophageal varices c Rectal varices c

d Pulmonary edema

7 In a sliding hernia the gastro-esophageal junction lies

a) At its normal position b) Below the normal position c) Above the normal position V d) None of the above

8 Which of the following structures is retroperi toneal

A transverse colon B spleen IJ2f6 C ileum D descending colon v r 1pound1111111

9 The renal angle is fonned lgtetween the 12th rib and ______ muscle

a Psoas major -middotshyb Erector spinae c Quadratus Iumborum d Diaphragm

10 The anterior structure at the hilum of the kidney is

a) Renal vein ~

b) Renal artery I middot~ I

c) Ureter d) Accessory renal artery

11 Because of origin of the muscle from the lateral one third of the inguinal ligament it

could not fonn the anterior wall of the inguinal ligament

a) External oblique b) Internal oblique c) Transversus abdominis_ d) Rectus abdominis

12 A large tumor mass impinges on the splenic artery and its branches as the artery pass out from below

the greater curvature of the stomach Branches o(which of the following arteries would most likely to

effected by the pressure on the splenic artery

a Left gastric b Left gastro-epipJoic c Right gastric d Right gastro-epipoloic e Short gastric_

13 A new born baby has projectile vomiting after each feeding It is determined that there is obstruction

of the digestive tract as a result of annular pancreas Annular pancreas is as a result of an abnormality in which of the following process

a Rotation of the dorsal pancreatic bud around the first part of duodenum b Rotation of the dorsal pancreatic bud around the second part of duodenum c Rotation of the dorsal pancreatic bud around the third part of duodenum d Rotation of the ventral pancreatic bud around the first part of duodenum y Rotation of the ventral pancreatic bud around the second part of duodenum

14 As the liver bud enters the ventral mesogastrium the region of the mesogastrium stretching from the

liver to the anterior abdominal wall is called

a Lesser Omentum b Greater Omentum ~ Falcifrom ligament d Lacunar ligament e Ligamentum teres of liver

16 A patient has absence of his 12th rib In such a patient if the doctor makes an incision to approach his

kidney mistaking the 11 th rib for the 12t he would end up injuring

Which of the following arteries is a direct branch of the gastroduodenal artery The

A right gastric artery

B left gastric artery

C inferior pancreaticoduodenal artery D left gastroepiploic artery

i E)right gastroepiploic artery --

E x pI a nation The right gastric artery is typically a branch of the proper hepatic artery The left gastric artery is a direct

branch of the celiac trunk The right and left gastric arteries anastomose along the lesser curvature of the

stomach The inferior pancreaticoduodenal artery is a branch of the superior mesenteric artery it

anastomoses with the superior pancreaticoduodenal in the head of the pancreas The left gastroepiploic

artery is a branch of the splenic artery it anastomoses with the right gastroepiploic artery along the greater

curvature of the stomach The right gastroepiploic artery is a branch of the gastroduodenal artery The

other branch of the gastroduodenal artery is the superior pancreaticoduodenal artery

Which of the following pairs of veins join together to form the portal vein The

A superior mesenteric vein and inferior mesenteric vein

B inferior mesenteric vein and splenic vein

C superior mesenteric vein and splenic vein

Ip)splenic vein and left gastric vein E superior mesenteric vein and left gastric vein

Explanation

The portal vein is formed behind the neck of the pancreas by the union of the superior mesenteric vein

and the splenic vein The inferior mesenteric vein drains into the splenic vein The left gastric vein drains

directly into the portal vein After the portal vein forms it enters the hepatoduodenalligament of the

lesser omentum to reach the liver The portal vein is the most posterior structure in the hepatoduodenal

ligament

At which of the following vertebral levels does the duodenum pass anterior to the aorta - _- shy

All ~

B L2 7~

CL3 I

~DL4

E L5

Explanation

The duodenum begins at the pyloric sphincter at the level of Ll The second (or descending) portion of

the duodenum is to the right of the aorta and extends inferiorly from the level of Ll to the level of L3 The third part of the duodenum crosses the aorta from the right side to the left side at the level of L3 The

fourth (ascending) portion of the duodenum extends from the level of LJ to the level of L2 The

duodenum ends at the duodenojejunal flexure The superior mesenteric artery passes anterior to the

duodenum as the duodenum passes anterior to the aorta The duodenum can be constricted at this level

In which of the following locations will perforation of the digestive tract result in the spilling of luminal

contents into the - lesser peritoneal sac

A Anterior wall of the second portion of the duodenum B Posterior wall of the second portion of the duodenum

C Anterior wall of the stomach

~Posterior wall of the stomach E Posterior wall of the transverse colon

Explanation

The posterior wall of the stomach is related to the lesser peritoneal sac The anterior wall of the stomach is related to the greater peritoneal sac The anterior wall of the second portion of the duodenum is related to the greater peritoneal sac The posterior wall of the second portion of the duodenum is related to the retroperitoneal space The posterior wall of the transverse colon is related to the greater peritoneal sac

The ureter lies against the anterior surface of which of the following muscles shyA Crus oftne diaphragm B Quadratus lumborum

0 Psoas major D Transversus abdominis

E Iliacus

Explanation The ureter exits the renal pelvis at about the level of vertebra L2 As it descends along the posterior abdominal wall it lies on the anterior surface of the psoas major The psoas major muscle arises from the bodies of the lower lumbar vertebrae The psoas major muscle is joined by the iliacus to fonn the

iliopsoas muscle The iliopsoas muscle then attaches to the lesser trochanter of the femur and is the major

flexor of the hip

As the right ureter passes the pelvic brim it lies against the anterior surface of which of the following

blood vessels

A Gonadal artery B Inferiorvena cava C Internal iliac artery

rJ- External Iliac artery

E Inferior mesenteric artery

Explanation

The ureter lies in the extraperitoneal space in the posterior abdominal wall Alter leaving the kidney it

passes inferiorly on the anterior surface of the psoas major muscle At the pelvic brim the ureter passes

into the pelvis At this point the common iliac artery is dividing into the external and iliac arteries The

ureter lies on the anterior surface of the external iliac artery immediately distal to the bifurcation This is a useful landmark for a surgeon to locate the ureter

When extravasated urine passes from the superficial perineal space into the anterior abdominal wall it is

found immediately deep to which of the following layers of the anterior abdominal wall

-ltScarpas fascia

B External oblique muscle

C Internal oblique muscle D Transversus abdominis muscle

E Transversalis fascia

Explanation

The superficial perineal space is bound by Colles fascia the fibrous portion of the superficial fascia This

layer of fascia is continuous with Scarpas fascia the fibrous portion of the superficial fascia of the anterior abdominal wall Therefore urine that is deep to Colles fascia will remain deep to Scarpa s fascia The urine will spread in the plane between Scarpas fascia and the external oblique layer

When a horseshoe kidney develops the ascent of the kidney is restricted by the A internal iliac artery B external Iliac artery

C common iliac artery

inferior mesenteric artery

E superior mesenteric artery

Explanation

A horseshoe kidney develops when the inferior poles of the to kidneys fuse together as they ascend into

the abdomen from the pelvis The first anterior midline vessel that is encountered by the horseshoe kidney

is the inferior mesenteric artery This artery prevents the kidney from continuing its ascent

The left testicular vein drains into which of the following veins

A Left internal iliac vein B Left common iliac vein

bflnferior vena cava D Left renal vein I

E Left internal pudendal vein

Explanation

The left testicular vein drains into the left renal vein The right testicular ~i~[~nsltjectlY into the

inferior vena cava This difference in venous drainage is believed to explain the greater incidence of

varicocele on the left side than on the right The venous drainage from the penis is to the internal vein

which then drains into the internal Iliac vein

The spinal nerve that provides cutaneous branches to the skin around the umbilicus is

A TS B TW-shy

C TI2

DL2 EtA

Explanation

The tenth intercostal nerve is the anterior ramus of the TIO spinal nerve After passing through the tenth

intercostal space the nerve continues forward in the anterolateral abdominal wall in the plane between

the internal oblique muscle and the transversus abdominis muscle In the abdominal wall the nerve innervates to the abdominal wall muscles as well as the skin and the parietal peritoneum The umbilicus is

a useful landmark for the region of distribution of the tenth thoracic nerve

The ligament of the vertebral column that resists its extension is the Aligamentum flavum

B supraspinous ligament

C posterior longitudinal ligament

D anterior longitudinal ligament

E interspinous ligament

Explanation

The ligaments of the vertebral column that resist flexion of the column include the supraspinous ligament

interspinous ligament ligamentum fiavum and posterior longitudinal ligament The ligament that resists

extension is the anterior longitudinal ligament This longitudinal ligament is very broad and strong It

covers the anterior and anterolateral surfaces of the vertebral bodies and the intervertebral disks In

addition to resisting extension the anterior longitudinal ligament provides reinforcement to the anterior

and anterolateral surfaces of the intervertebral disk The posterior longitudinal ligament is relatively

narrow and covers the posterior surface of the vertebral bodies and the intervertebral disks This ligament

reinforces the posterior surface of the disk The posterolateral surface of the disk is not reinforced and it

is through this region that herniation of the nucleus pulposus usually occurs

A patient presents with epigastric and right upper quadrant pain The pain is most intense 2-4 hours after

eating and is reduced by the ingestion of antacids The patient states that he has passed black tarry stools

(melena) within the last week Fiberoptic endoscopy reveals a yellowish crater surrounded by a rim of

erythema that is 3 cm distal to the pylorus Accordingly an ulcer has been identified in the patients

A fundus

B antrum

C duodenum

D jejunum

E ileum

A number of physiologic genetic and other factors increase the risk of gastric (and duodenal) peptic

ulcers The evidence that H pylori plays a principle role is compelling Smoking and caffeine are known to adversely affect the morbidity mortality and healing rates of peptic ulcers In general first-degree

relatives of peptic ulcer patients as well as males have a threefold to fourfold increased risk of developing this disorder Paradoxically in gastric ulcer disease acid secretion is not elevated It is possible that

excess secreted hydrogen ion is reabsorbed across the injured gastric mucosa In general a defect in gastric mucosal defense is the more important local physiologic

A patient presents with symptoms of duodenal obstruction caused by an annular pancreas Annular pancreas is caused by

A rotation of the dorsal pancreatic bud into the ventral mesentery B rotation of the ventral pancreatic bud into the dorsal mesentery

fJ failure of the major and minor pancreatic ducts to fuse ~ ~ cleavage of the ventral pancreatic bud and rotation of the two portions in opposite directions around -the duodenum E formation of one pancreatic bud instead of two

Explanation Normally the ventral pancreatic bud rotates around the gut tube to reach the dorsal pancreatic bud The two buds fuse to form a single pancreas and the distal portions of the two ducts fuse The ventral pancreatic bud forms the inferior portion of the head of the pancreas the uncinate process and the major pancreatic duct (of Wirsung) The dorsal pancreatic bud forms the superior part of the head the neck body and tail and the minor pancreatic duct (of Santorini) Annular pancreas is the result of the ventral pancreatic bud dividing into two portions before it rotates into the dorsal mesentery Each portion rotates in opposite directions to get to the dorsal mesentery thus encircling the duodenum The presence of annular pancreas can constrict the duodenum thus obstructing its lumen

In n _ phranlc----

Gon ~l ----_1 Lum bltano

~~--- CornmQ1t bull ac

+-~4--- lnlllirnaJ ilic

xtem iliac

OBJECTIVE - Identify the blood supply to each of the structures listed in the table on the previous page

Ill give you a head start

FOREGUT - Supplied bV Celiac Tru nk (T12)

Proper hepatic

GastiooUod 13Jafter

1nferlor pancreaticoduodenal artery

Common epatlc

Lett gas ric iiirtery

Spfen artery

shy Gastroepiphgtic artery

~ Superior mesenteric 8rtfry

~

1 Esophagus is a derivative of the foregut so its blood supply originates from the celiac trunk

(T12) The predominant blood supply to abdominal portion of the esophagus is the Esophageal

A (Branch of L Gastric) The venous drainage of the esophagus is particularly important because

it is 1 of 3 clinically relevant sites of Portal Caval anastamoses The Portal Esophageal Vein

meets the Caval Azygos System Persistent bleeding manifests as Esophageal Varices - a fata I

condition

2 The Stomach is also a derivative of the foregut has EXTENSIVE blood supply and is very high

yield on anatomy exams The lesser curvature is supplied superiorly by the L Gastric A (1 of 3

major branches ofthe Celiac trunk) and inferiorly by the R Gastric A ( a branch ofthe proper

Hepatic A) The greater curvature is supplied superiorly by the L Gastroepiploic A (a major

branch of the splenic A) and inferiorly by the R Gastroepiploic A

The Short Gastric arteries (branches of Splenic Artery) supply the fundus of the stomach and

are referred to as EIID ARTERIES because they have no collateral blood supply Therefore if the

splenic artery were occluded (ex - increased pressure in the ommental bursa) - there would be

ischemia to the fundus of the stomach Venous drainage of the stomach is extensive via various

veins lead ing to the portal system Posterior to the stomach the IMV joins the splenic V which

joins the SMV to form the PORTAL VEIN ADAMS

3 Duodenum blood supply has high clinical relevance because it is the junction of the foregut and

midgut and therefore is the site of anastamoses between branches ofthe Celiac Trunk (main

foregut artery) and the Superior Messenteric Artery (main midgut artery) The Proper hepatic

artery gives off the gastroduodenal artery which travels behind the 1st part of the duodenum

This point has high clin ical relevance because duodenal ulcers are very common and a posterior

rupture of the 1st part of the duodenum could rupture the gastroduodenal artery causing

traumatic abdominal bleeding The Gastroduodenal artery first gives off the R Gastroepiploic A

(mentioned above) and proceeds as the Superior pancreatico duodenal artery (supplies the

pancreas and duodenum) which anastamoses with the inferior pancreatico duodenal A (branch

of the SMA) This is the junction of foregut and midgut and occurs near the opening of the

bil iary system into the duodenum (ampula of vater) Portal venous drainage here is responsible

for delivering nutrients from digestion to the liver for metabolism Appreciate that the Superior

mesenteric artery (artery of the midgut) branches from the aorta at Ll travels posterior to the

pancreas than moves anteriorly (at the jxn of the pancreatic headbody) and comes over the

3rd4th part of the duodenum Tumor of the head of the pancreas can compress the SMA

4 Jiver blood supply is via the common hepatic artery (major branch of the cel iac trunk) The

common hepatiC becomes the proper hepatic gives off the R gastric A and the Gastroduodenal

A and then joins the common bile duct and the portal vein in the portal triad Clinical- if a

patient were bleeding from the hepatic A a surgeon can stick his fingers in the epiplOic foramen

and squeeze the free edge of the hepatoduodenalligament in order to stop bleeding to the

area Please note that the hepatic a branches into Rand L hepatic A The Right hepatic artery

gives off the cystic artery which supplies the gallbladder Afferent venous supply is via the

Portal vein which is bringing nutrient rich blood to the liver After metabolism takes place

venous blood leaves the liver through the hepatic veins into the IVC PLEASE UNDERSTAND THE

RELATIONSHIP OF THESE STRUCTURES - ADAMSNETIERSNH Etc

5 Pancreas - Head is supplied via the superior and inferior pancreaticoduodenal arteries

(mentioned above) The tail (situated towards the hilum of the spleen) is supplied via the

pancreatic branches of the splenic artery (END ARTERIES) This blood supply is very important

because the endocrine Alpha and Beta Cells from the pancreatic islets of lagerhans are located

towards the tail This is where Insulin and Glucagon is released to the blood

Now complete this for mid and hindgut structures Make sure to note clinically relevant arterial

anastomoses as well as portal caval anastomoses FYI Appendix blood supply SMA + IMA

anastamoses marginal artery Portalcaval rectal veins fhemmorhoids) and periumbilical caput

medusa are high yield THE BUTT THE GUT and THE CAPUT

Abdominal Development

Liver

Ij1f

II wall b

oh liN ~ VltJrti n be- bull

Pancreas

Secondary Retroperitonealization e I~tl r 1 a v-mtrai m ellter

Rotations of the Gut I i Ij (lIl1UtIJ f~ l r tilt

()l td 10 me l-ft and he v

--~--- -~ -~-~

i

I AolaijonjoI~guf I

STOMACH BED (IDENTIFY IN ADAMS)- the structures posterior to the ommental bursa which

support the stomach in the supine position

Abdomnal JQrUI

Splnic vein

OmQ-oul tv~ ) O(s(Jroa)

Lojt(r o m nturrt (hpJtodu o d~n31 Hid

Gadrl)SplerH (g3stroll~nal) IIgam~nt

hiad h~~atogrtricent IIQdmiddotcrt~)

Lt Dome of Diaphragm (why left Look this up in Adams)

Spleen (What is the blood supply)

Left Kidney (What is the blood supply - AND how is it different from the R kidney)

Suprarenal Gland (What is the Arterial AND Venous Blood supply - how are they different)

Pancreas (How does supply differ from Head to Tail What is the SMA Relationship)

Transverse Mesocolon

liver - ADAMSWET - Make sure you look at the liver in wet lab

Left triangular nl1am~nt

ComoaDj ligamnt

Erophg~1 impre$ioo

Hepatio veins

In1erior -ifena middotr3)Ia

Fibrous appendix o-t

live

impr~j on

Heprorendl p~rtion of Q)(Qllary ligament

Righllri~n9ul r 1I~met

(Common) bile quol

Gr)mmCtr~ hepatic dlJct

Ccentic duct

Duodenal impression

GaJdate p-fr)~S

Hepatic artgtrl prop-f iiiiila - Faloiform ligament

_ - shy Round ligamen liver

~--F-- CoJio imprgt-ssi-on

Prta heptis

Identify the lobes impressions and embryonic remnants associated with the liver

Caudate Lobe Quadrate Lobe Right Lobe Left Lobe Round ligament Falciform Ligament

Ligamentum Venosum (what is its fxn in embryonic life) Hepatic Veins (NOT PART OF THE

PORTAL TRIAD) IVC PORTAL TRIAD - Contents relationship cross section etc Know the

Galbladder relationship to the lobes of the liver

Biliary Duct System - Make sure you understand the sequence of these structures - BE ABLE TO

DRAW A FLOW CHART

TPVd i

t

I t

1 __ Cm-(r

patk GlJet

I

J

Clinical = JAUNDICE is caused by anything that prevents delivery of bile to intestine Tumor of the

head of the pancreas Stones etc Patient will have pale stools and yellowish colored mucus

membranes

Clinical- Any scenario that tells you the patient has BILLOUS VOMIT means that the obstruction to

the flow of digestive contents is after the Ampulla of Vater (Site of Entry of Billiary system to the

duodenum) - ie Duodenal Atresia

Spleen -located posterior to the mid axillary line between ribs 9 and 11 Make sure you know that

the 10th rib is the main axis of the spleen and this organ is susceptible to injury (stab wound errant

thoracoce ntesis etc)

The spleen is derived from mesodermal cells - NOT THE GUT TUBE

The spleen rests on the left colic flexure associates with the tail of the pancreas Know the

structures entering the Hilum of the spleen

Sh rt O~-t~ic 1 0(0 10 rtiltSPIric Iloa nt

(cut)

Peritoneum - similar concept to Pleura - think of a fist in a balloon

Visceral Peritoneum - Layer of balloon touching your fist

Parietal Peritoneum - Layer of balloon not touching your fist

Your fist represents the organ your wrist is the hilum and your arm contains the blood supply

entering the organ

Appreciate that there will never be organs in the peritoneal cavity - rather these organs invaginate

the cavity Kaplan videos

RULES OF NOMENCLATUREshy

1 Organ completely surrounded by peritoneum - peritoneal organ

2 Organ partially surrounded by peritoneum- Retroperitoneal

3 Peritoneum surrounding peritoneal organ is VISCERAL peritoneum

4 Peritoneum surrounding retroperitoneal organ is PARIETAL peritoneum

5 Peritoneum connecting visceral to parietal is called messentary 2 messentaries in the

gut Dorsal (to the gut tube) and ventral (to the gut tube) messentary

Aorta is in Retro peritoneal position - but blood must reach peritoneal position - vessels travel through

messentary All peritoneal organs will have blood supply reaching through messentary

-Mesentery is a 2 layer peritoneum with a neurovascular communication between body wall and organ

- Ligament connects one organ with another or to the abdominal wall (Ommentum = ligament)

lesser Ommentum (attach lesser curvature of stomach and duodenum to liver) =Hepatoduodenal

Ligament and Hepatogastric Ligament

Has a Superior and Inferior Recess (Accumulation of Fluid in Ascites)

Communicates with the greater sac through the epiplic foramen (what structures pass through

this foramen)

Boundaries - you must be able to visualize this

o Anterior - stomach

o Posterior - parietal peritoneum pancreas

o Superior - superior recess (bw diaphragm and coronary ligament)

o Inferior -Inferior recess (bw layers or greater momentum

Greater Ommentum (attach greater curvature of stomach) Gastrophrenic ligament Gastrosplenic

ligament gastrocolic ligament

The greater omentum is the largest peritoneal fold It consists of a double sheet of peritoneum folded on itself so that it is made up of four layers The two layers which descend from the greater curvature of the stomach and commencement of the duodenum pass in front of the small intestines sometimes as low down as the pelvis they then turn upon themselves and ascend again as far as the transverse colon where they separate and enclose that part of the intestine

ABDOMINAL PAIN

Parietal Peritoneum - supplied by same vasculature lymphatics and nerves supplying body wall it

lines and diaphragm Sensitive to pain pressure heat cold well localized

Visceral Peritoneum - supplied by same vasculature lymphatics and somatic nerve of organ it covers

Insensitive to touch heat cold and laceration - referred to dermatome of spinal ganglia providing

sensory fibers Where does appendicitis refer to

Foregut pain - epigastric area (ie - cholycystitis)

Midgut pain - periumbilical area (ie - appendicitis)

Hindgut Pain - suprapubic area (ie - diverticulitis)

Extra ImagesConcepts

ll~_____-

FalifCtrm ligament oind r~ud ligamet f Ilver

Blood from splenio gastriC and inferiof rne$e-rteri v~ins

Ca-I tributaries

Lett gastrio Ifein

Posterior superior pan~reatioodul)denal vaihS

Lott gamo-om~nlal (9aropip lomiddotic) -in

Poq_~ tjol imerl-9-r panCJertlcorllJod-nal veiopound --amp----I- - ~J Right grtr~-omntal

Anwrior interi (gartroepiploic) Jjn

pan euaii cod vl)denal veins middot Inf~Ji (t r mesentric vein

Miqdle (olic vein

Right cl)licvein Sigmoid and rectosigml)id (ei ns

IhH)Collc(~io

--- Mi~dl laquooLJl gtjrltgt

PoM ca vl1 illasto)moses -----shyampoptoageal 2 Paraumbilie-lt11 Inferi or Fectal vei ns

3 Recial 4 REuoperHonea1

Know how the Portal vein is formed I 4 sites of portal caval anastamoses and 1 clinical shunt

Col li t ltt-~ otTl~tI ~nj pc~ 1lt1 turJoG

Ltf 14i1 tImiddot~ artoftl9 on tj phtAt$

L-oftqf 4t t~r 1=laquoIran d 1 bull shy~p l ci rj o fOOOts

Nerves follow the arteries - appreciate the splanchnic nervous system I

Uet~ric branch of left ~nal art

Ureterie branch of righi renal artelY

Left Zld lumbar in and co mlTlunication to as)erdin9 lumbar l(~in Hi ~ht tEZ1~~t~ t3r j t itn ~ nJ l1t- rlnd lfe i r1

Inferior me5nteri~ artery

Notice that the right testicular vein drains directly into the IVC and the right testicular artery drains

directly into the aorta However the left testicular vein drains into the L renal vein at a right angleshy

reason left testicle is lower and more susceptible to varicocele (bag of worms)

Also notice that the left renal vein has a longer course because the IVC is on the right side whereas

the right renal artery has a longer course because the aorta is on the left side

Appreciate the anterior to posterior relationship of structures in the hilum of the kidney - VAP - Vein

Artery Renal Pelvis (Ureter)

11____ __ L_ L_ n VJ __ _ _ t_L I I_ _ L __ L_ I -pound1 bull LI_~-I ____

Posterior View of Head of Pancreas in ( of Duodenum

Celiao hunk

Co mmon ~L~jJth art~ry

GastNduQdonal artrf (partilly in phantn)

P1)Sterior $Up~Jior panCflaticuduodfmal art~r~t

(Co mm on) bile duct

middot~1t~~t-1l---~-~- Right gshomiddotomental (gastoe plp lolc) 3rte (phantomost)

Grener paocre atic art-ry

1n1~rjor pancr-iatlc artery

Jtrifll supejo r pal)oreailcento)dJodenal artr1 (phantom)

Anastomotlo branch

POostetlor bJanch of jo f~ri of pan-reatir(lduodensl drttnj

Anterio r branch of i flferior palcreati~)duodenal art~(phan1om)

Notice the extensive blood supply to the pancreas and duodenum via the branches of the celiac trunk

Notice collateral supply from SMA branches - makes sense bc this is the jxn of foregutmidgut

Identify the vessels in this arteriogram

Hiltid i)f N~ck oi B)dvof Tail 01 pa nereas pan cent~as P-nmiddot-reas panCtCas

I nferie v~na cava

jHept1iic p(lrlai v~in

Port1 tnd H~pti lt a ftH prol

Comm on) bll duct

Ouodtnum

~ft colic (sio)Atta~ hmtrlt jt~xJr-ofha~elSe

muo(IIQn

Right ~lIc (h~j)tic)

il~gtture

In1triol m~oten lIein (rttr op~ritoMdO

SlJp efl or mes~n~fiC amrV and lipln

KNOW YOUR NEIGHBORHOOD

Questions

vVhiJh structure supplied by a bnmdlof the cclia( artery is not derivcd from foregut LemCJUCrITI

(A) Head of the pancte-a5

CD) Pyloric duolenum

Cystkduct

( Liver hepatocyt~~

~F) Body of the spleen

An infant presents with an omrhaJucele at birth -hi oJ the [oHm illg applies to his cM1-dition

(A) It is 31so seen ill p4titnts with aganghonic megacolon

(11) ft reuirs from a fal1ure of resorption of theviteUine d let

(C) It results from herniation at the-site of regression of the right umbilk vein

DJ It is caustd by faihtrc of recanalization of the midgut part of the duodenum

~ It ill camioo by a failuIt vf the midgul to return to the abGQminal uity after herniashytion in-n the urnbilk s l stalk

Ot er than the spleen occlusion Cif the spit-Ilk artery at its odgin wm most likely affect die blood supply to jllch st cnud

(A) Jejunum

(B) Body of th pal1~lltas

(C) LeSStT Cllmiddotlaturc of tl )toma-ch

(D Duodenum dista to the entrance of the Ornmou bile duct

E Fundus of the stomach

A 38-yeu-old batL~er with a history of heartburn suddenly experiences excluciating pain in the (plgastric region of th~ abdomeu SurgCry is perf~rme immediard y upon admisshysion to the 1IlcrgCJliy tuomh~re i~ evidence uf a ruptured ulcer in the posterior waU of the stomach Vhere will a surgeon first fi nd the stomach contenlSf

A) Greater p4ritoneal sac

rB) Cul~de-s~c of Douglas (--

C Omental bursa ~

--D) Paracolic gutter

rEj Between -he panttal perimltum and the posterior body wal1

At birth an infant presents with a st()ma~ rb~tbas~njJled jfltotb~diaplfagru 1A1ltre is the defect thatresulied iiitJle heini~t()n shy~tsophagealbiatus

7 - rH-- Hiatus for the inferior vena cava

( Pleuroperitoneal membrane -(0) Septum transvcrsum

(E) Right Crlt~

An infant born with DOVv7l syndrome presents with bili()u~ vomiting Ahat congenital defect does the infant have

(A) Pyloric stenosis

(B) Meckel diverticulum C) Ornphaloce1e

(D) Gastroschisis

( ~ ) Duodenal atresia y A patient with cirrhosis of the liver presents with ~ bacalvaricestnlreased retrograde pressure in which veins caused the varices

(A) Paraumuilical

(B) Splenic

(ct AzygltJus

(15))G~trk ( (-F) Superior mesemeric

A htaltby 3-year~old male patient experiences a hernial sa protruding from the anterior abdominal wall about halfway between me anterior superior ilia spine and the pubk tuberde Pulsations of al1 artery are palpated medial to the protrusion site through the abdominal walL Which layer of the anterior abdominal wall will first be traversed by the

1hctma

fA) Rectus sheath (B) External oblique aponeurosis

(C) Inguinal ligament

lD) Transversalis fusda

(E) Cremasteric fa~cia

After 5urgi(aj ffpair of a hernia the patient tXperienccs mtmlgtness in the skin on the anteshyrior aspect of the S(Totum_ Vhaf nerve may have been lesioned during thehemiorrhaphy

(A) Femoral

(B) Obturator

(C) Ilioinguinal

(D) lliohypogastrk

(E) Pudendal

A 23~year-LJld female secretary il1 good health ~-uddcn1) doubles over with pain in the a ea of the 1JmbRicu$ Sbe feels vartn and ltneasy and has no appetite That night the pain seems to have mQved to the tower right abdominal regjol1 and she calls her family doctor who then arranges for an ambulance to pk-k her up and take her to the hospitaL Wh ell ntn~ perceived in the area of the urnbilirus most Hkely carried lhe pairfu I sensations into the eNS

tA) Vagus nerves I~

V B)

) Lessersplanchnk nerves

tC) Pudendal nerves

(D) lIiohpogastrk nerves

(E) Greater splam ic l erves

A CT reveals carcinoma in the bOod of the ancreas Vhich blood vessel trut ourses ----~- - -bull ------ --shy

immediately poftterior to the body ofthe pancreas is the m~t likely to be oompressed

(A) Splenk artery

(B) Abdominal aorta (C) Portal vein

(1) Splenic vein

(E) Renal vein

A patient has a penrln1l1ng uker of the posterior wall ot the br~l part ot the (lUooenmn llkh blood vessel is subject to erosion

(A) Common hepatic artery

(B) Gastroouodenal artery

(C) Proper hevatic artery

(D) Celiac artery

(E) Anterior inferior 11amrelltlcoduodcnal attery

Your patient has been diagnosed -ith a carcinoma locallted to the head and l~e(k of the pancreas Another clinical sign would be

A esophageal varices

(8) hemorrhoids

C) a caput medusa

(D) increased pra Teuro n th~ hepatic veins

(E) enlarged right supra lavkular lymph nodes

Wltkh of the foUowing structures develops in the ventral mesentery

(A) Spleen

(B) Jeiunum (C) Head of1ht pancreas (D) Transverse colon (E) Stomach

ti l Uw ~ littwin~ f( S-t lil oai Imdge ~ hi(h or tbt la~)d J truetur tgt liJ llntn nl) he hl p UC iJd [IIi ell

c o

A) drains Ie tht infCrior a La aI

R t middot~nfl0 ~ill to th~ lunlgtn of h i dtlndCrlllfH

(e) m t bull JiJattd on tl l J n T ~H

D ) sup Lc O VSlt I Hlid bhtu l 1 li - -I un oid

( ) U~tpli(t tr j middottUh~ 1 v(( b~nt rfK n1ilc~Zm

ANSWERS AND EXPLANATIONS

Answer E The spleen is t hlttnopodicand lymph organ demlted from mesoderm

Answ~ R Al1 tlmphalocele is caused by it failure of the nlidgut to return to the ahdomir nat cavity after herniation into the umbiliau Stalk Choices Aand D maybe seen in infants with Down syndrome choice D ~s the specific CBuse ofduudcnal JtiCSitt Choice C is (ile cause of gclstrosbisis and Choice B nsults iu a Meurolktldivertku1-tlB

Answer B The fundus ofthe stomach is suppHed by soort gastric brunches of the splenic altery The splenic artery supplies the body and tail of the pancreas part of the greater curvature of the sttmla(h and the spleen Te jejunum part of the head of the pancreas and tht~ duodenum distal to the entrance of the commOll bile duct are supplied by the superior mesenterk artery clll~l ~be less r ctlt1ature cmd the pylQric antrum are supplied by the right and lei gastric art(ries

AnSWftt C Tbeomental bursa or lesser ~ritoneaj sac lies direcdy posterior to the proxshyimal part of the duodeTtlm and the stomach and would be the first site where stomach contents ~Ott1d be fpoundluncL

Answer C A defect in a llleuropcritoneal membrane (uswlly the left) is the typical site of i1 cc-ngenitlI diilphragluatic hemia llere the membr4ne fails to dose ()pound( of the perishycCirdiopcritulleal canals

Answer E DuoJenal atresia and aganglionic megacoion are congwitaI defects S~Il in patients with Dowmiddotnsyndrome

Answer D RulaTgemt~llt of and retrograde flow in g~lstrk vel_ns in particlJl~r the kft gas~ tricveins dilates the capillary bed in rhe wall of the esophagus in (ases of porta yper~

tension Blood flow would increase in and dilampte tribntarkgts of the (lZygOUS vein on the other side of the capiUary bed but flow in this vein is in the typical direction t()ward the superior vena cava Paraumbiii(ltU vein eilgorgement contributes to a caput medusH Splenic ~nlargement might prc~nt with 5plcnonlegaly and balt-kflow in to tlu superior m~~ntclic vein occurs but is asymptomatic

Answer D The patient hagt an indirect inguinal hernia whi~h emerges from the antt-rior abdominal wall through the deep inguinltilling Theeep ring is a fault in the transv~rshysaUs fascia this I~yer wiIJ be penetrated first by the hernia

An~Wer C The ilioinguinal nenc which provides sens~llion to the lnedlal thigh ltmclanteshytior SClotunl pass~lt th rough the 5uperfh_ial inguinal ring ind $subject to inj i1T) becaus-e

it is in the operatitm Held of the erniorrhapny

Auswer B The leMHr splanchnic nerves are sympathdic nerVlts that carry viscera l sensashytlltgtrogt ftom illtllt1m~d ()J stietched gust (itinteitinal ~tructures (in this case the pprndix) into tnt eNS Lesser splanchnic ntTYcsarisc from thmiddot T9--T12 spinal cord segments lt1nd provide sympathetic innenation tD rnidgut siruc1ures whiCh include CLe app~JldD Viscera] Pain arising from affecLed Inidgut ampt 1C1ure is referred over the same dl- matorne~ of spinal segrnertts v-hich provide the sympathetic Innervation n this G1SC of appendicitis the invohen~n t of the ltire) of t e unlhHku indud s the T 10 dermatome

Answer B Of the five choices onty the dscending olon is retroperiton~al aldwould be a lik ~ ( choice to be seen immediately a(~jilcent to t11e posterior abdominal middotn~L

Amwen D The SpltftlC ~-ein ourses posterior to the body of the panneas m its way tt drain into the superior mCSfttltlri( vein

Answcr B TILt glstrodllolticnal artery 1 direct hIamh of the comrootl hepatic artery courses immediately pt))iwri() to the duodenum and is slbject to erosion

Answer B Carcinoma of th pan middott3S in the 1tilt1 may compreampgt the portltil vein at irs orishygill The poTtai vcin is fomled when the splenic vein jQiaswith tfie superior meStllt eric vein The inferiot mesenteric vein joins the ~plenjc vein just priOT to tlli~ point at which the splenic joins the superior Jlleit1ltcri( vein Increescd venous presslu in the inferior mesenteric vein is a cause of emo hoid~

Answer C The- velltral pancreas wilich forms most of the head of the p ~ncr as develops in the ventral mes(ntery as antutgrowth of the hepatic diverticulum Th~ hepatic divershyticulull induding the biIJary appa~atus develops in tbe ventral mesentery of the foregut

Answer~ A The superior mesenteric ~in joins with the spienkvein to form the hepatic portal vciu

Answer D The structure at gttlK is the proper hepatic artery~ whkh suppUesoxygenated b middotood to the liver

MAKE SURE YOU KNOW the diff bw Rectus Sheath above and below the arcuate line

ABOVE

Aponeurosis of xiiltmal obllque musclo

Extemll f)biquw musde

Reotln ilbdomlnls musole S~in

Internal 9bliquQ mY~QI

AponeUfOsi$ of hJH$V~~S Lir9a a lb lbdolTlin~ musolo Tri OJV6 rUi

atldomlnis mUS(loe

Sub cutanlilous tiue (tatty ye r)

BElOW

A POrl lJfosis 01 etemal oblique muscle

Aponeul~)sis 01 Internal oblique mU$cl~

Anteriol lay~ of r~ltdus st~ath EXttom1 oblique rnu$cll

Rectus Jbdominis muscle Intoernal Aponeurc-sis of tra~fersU$ oblique muscle-

at-domlnis muscentl ~ Skio

Tra nsvitSus abdomioLs ml)ZClt

TralSVersaHs fascia Medial umQil iegtt1 1i9Jment -and folj

Uldchus Peritoneum (ir median Umbilj~al Suboutane ous

Extraprftone 11ascia

Ymbilimiddot~1 fold)

preu9poundiea1 fascia

tissue (fatty 4nd m~mbr3n(iUS layers)

o Above the arcuate line (A horizontal line 13 of the distance bw the umbilicus and the

pubic symphysis) -10 Aponeurosis divides into an AntPost Laminae

o The Ant Laminae joins EO and Post Laminae joins Trans Abdominis = Ant and Post

RECTUS SHEATH respectively

o BElOW the arcuate line - all 3 aponeurosis join ANTERIOR to rectus muscle to meet its

counterpart in the midline (linea Alba)

o Take away Msg - The abdomen is devoid of a posterior rectus sheath below the

arcuate line and is therefore more vulnerable to herniasinjuries

Question - A physician makes a deep incision in the patients midline immediately superior to

the pubic symphysis which of the following layers is his knife least likely to pass

Rectus Abdominis External Oblique Ant Rectus Sheath Posterior Rectus Sheath All of the

Above

Answer - All of the above None of the other answer choices are midline structures -LINEA

ALBA

Linea Alba has very poor blood supply - doesnt heal well after surgery Therefore this is a

common site for incisional hernias

a Spleen b Transverse colon c Descending colon d Stomach e Pleura

17 Meckels diverticulum is normally found 2 feet proximal from the

a Pyloric sphincter b Lower esophageal sphincter c Ileo-cecal valve d Middle valve of Huston e Anal valve

18 Ulcer in the posterior wall of the first part of the duodenum would erode ___ artery and would cause bleeding

a Left gastric b Right gastric c Hepatic artery proper d Gastroduodenal artery e Middle colic artery

19 An inflamed appendix is identified by a surgeon on the operation table by noting

a The appendicies epiploicae b The convergence of tenia c The artery of Drummond d The mesocolon e The mesosalphinx

20 The nerve which emerges through the psoas major is

a Femoral b Ilio-inguinal c Ilio-hypogastric d Pudendal e Subcostal

21 The right gonadal vein drains into the

a Azygos b Hemiazygos c Inferior Vena Cava d Right renal vein e Left renal vein

22 The hepatocytes in the liver is derived from

a Ectoderm b Endoderm c Mesoderm

d Neural ectoderm

23 Abscess in the lumbar vertebrae due to tuberculosis would spread to the adjacent muscle which is

a Psoas Major b Iliacus c Quadratus lumborum d Tranversus Abdominis

24 The anterior wall of the inguinal canal is formed by

a External oblique and transverses abdominis b External oblique and fascia transversalis c Internal oblique and external oblique d Internal oblique and transverses abdominis e Fascia transversalis and peritoneum

Meckels diverticulum is a result of which of the following developmental abnormalities shy

A Failure of the vitelline duct to close

B Failure of the herniated intestinal loop to retract into the abdomen

C Failure of the urachus to close

D Failure of the midgut to rotate

E Failure of the hepatic duct to close

Explanation

Meckels diverticulum is a result of the persistence of the proximal part of the vitelline duct This

diverticulum is usually found about 2 feet proximal to the ileocecal junction and is usually about 2 inches

long It is present in about 2 of the popUlation It may be the site of ectopic pancreatic tissue or gastric

mucosa and may develop inflammatory processes and ulcerations Acute Meckels diverticulitis

simulates appendicitis

Which of the following veins carries blood from the esophagus to the portal vein The

A right gastric vein

B left gastric vein c splenic vein D azygos vein

E left gastroepiploic vein

Explanation

The left gastric vein a direct branch of the portal vein drains blood from the lesser curvature of the

stomach and the inferior portion of the esophagus Because branches of the portal vein do not have

valves blood can flow in a retrograde path when there is an obstruction to flow through the portal system or liveL Rlooci Cln then flow from the nortl] vein thr()1Ph the left PRstric vein to the esonhlPlIS lno

through venous communications within the submucosa of the esophagus to esophageal veins that drain

into the azygos vein The increase in blood flow through the esophageal submucosal veins results in esophageal varices

On the posterior wall of the abdomen the celiac ganglion A contains cell bodies of postganglionic parasympathetic neurons B is synapsed upon by neurons in the posterior vagal trunk C is synapsed upon by neurons in the greater splanchnic nerve D contains sensory cell bodies of lumbar spinal nerves E contains cell bodies of neurons that cause an increase in the rate of peristasis

Explanation The celiac ganglion is one of the preaortic ganglia of the sympathetic nervous system It contains cell bodies of postganglionic sympathetic neurons The sympathetic splanchnic nerves contain preganglionic sympathetic neurons that pass through the sympathetic chain without synapsing These splanchnic nerves go to the preaortic ganglia to synapse The greater splanchnic nerve contains preganglionic neurons from spinal cord segments T5-T9 This nerve synapses in the celiac ganglion The nerve fibers in the vagal trunks are preganglionic parasympathetic fibers that go to the walls of the organs that they will innervate and synapse on postganglionic parasympathetic neurons in the walls of those organs Cell bodies of sensory neurons in the abdomen are found in the dorsal root ganglia or the sensory ganglia of the vagus nerve Sympathetic innervation decreases the rate of peristalsis parasympathetic innervation increases the rate of peristalsis

Which of the following pairs of arteries will allow blood to bypass an occlusion of the celiac trunk

A Left gastric artery-right gastric artery

B Left gastroepiploic artery-right gastroepiploic artery

C Superior pancreaticoduodenal artery-inferior pancreaticoduodenal artery

D Splenic artery-common hepatic artery

E Left gastric artery - proper hepatic artery

Explanation The anastoOlosis of a branch of the celiac trunk and a branch of the superior mesenteric artery will

provide collateral circulation around an occlusion of the celiac trunk Each of the other choices pair

branches of the celiac trunk therefore these will not provide collateral flow around the obstruction of the

celiac trunk The left gastric splenic and common hepatic arteries are direct branches of the celiac trunk

The right gastric artery is a branch of the proper hepatic artery which is a branch of the common hepatic artery The left gastroepiploic artery is a branch of the splenic artery The right gastroepiploic artery is a

branch of the gastroduodenal artery whlch is a branch of the common hepatic artery

Which of the following organs has appendices epiploica The

A sigmoid colon

Bjejunum

C duodenum

D stomach E esophagus

Explanation Appendices epiploica are characteristic of the colon Appendices epiploica are subserosal accumulations

of fat None of the organs of the gastrointestinal tract has appendices epiploica except the colon

Page 21: Chirag's Abdomen Review

renal vein (choice A) drains directly into the inferior vena cava The left testicular vein drains directly into

the left renal vein Thus these veins are already in communication and neither vein is part of the portal venous system The right renal vein (choice B) drains directly into the inferior vena cava The right

suprarenal vein also drains directly into the inferior vena cava Thus neither vein is part of the portal

venous system The superior mesenteric vein (choice D) drains directly into the portal vein The inferior

mesenteric vein drains into the splenic vein which then drains into the portal vein Thus neither vein is

part of the caval venous system The superior mesenteric vein (choice E) drains directly into the portal

vein The splenic vein also drains directly into the portal vein Thus neither vein is part of the caval

venous system

A 12 year old boy has fever vomiting and para-umbilical pain After examining the patient the doctor

makes an initial diagnosis of appendicitis Appendicular pain which is initially referred to the umbilicus goes to the dorsal root ganglion of

a TI b TI2 c L1 d T7

(e I TIO

A 59-year-old male undergoes a neurological examination which reveals that when the abdominal wall is

stroked the muscles of the abdominal wall of the side of the body stimulated failed to contract Other

neurological tests appeared normal The likely region affected includes

a CI - C5 spinal segments b C6 - TI c T2-TI ~T8-T12

e Ll- L5

The surgery done to relive portal hypertension is done by connecting two veins Which of the following veins would be suitable for connection

a Inferior vena cava and portal vein b Superior vena cava and portal vein c Splenic vein and right renal vein d Splenic vein and left renal vein e Superior mesenteric vein and Inferior vena cava

A mother brings her 3-week-old infant to the pediatric clinic reporting a new scrotal bulge that she found -~-

while changing a diaper yesterday The infant is afebrile Physical examination reveals a palpable mass in

the scrotum while in the standing position resolution of the mass in the supine position and no

transillumination of the scrotal sac What is the most likely diagnOSiS

a Cryptorchidism b Direct inguinal hernia c Hydrocele d Indirect inguinal hernia ~ e varicocele

The Vagal trunks enter the abdomen by passing through which of the following openings in the

diaphragm

a Right crus b Esophageal hiatus ~ c Vena caval hiatus d Aortic hiatus e Left crus

2 The anterior boundary of the epiploic foramen of Winslow is bounded by

a) First part of duodenum b) Lesser curvature of stomach c) Liver d) Hepato-duodenalligament v ~

3 The ilio-inguinal nerve is derived from

a TI2 ry b LI c L2 d L3 e L23

15 Surgically the structure used to suspend the kidney to the diaphragm is

a) Renal fascia b) True capsule c) Perinephric fat d) Paranephric fat

6 If there is portal obstruction because of carcinoma affecting the pancreas which of these of the

following signs would be present

a Caput medusae b Esophageal varices c Rectal varices c

d Pulmonary edema

7 In a sliding hernia the gastro-esophageal junction lies

a) At its normal position b) Below the normal position c) Above the normal position V d) None of the above

8 Which of the following structures is retroperi toneal

A transverse colon B spleen IJ2f6 C ileum D descending colon v r 1pound1111111

9 The renal angle is fonned lgtetween the 12th rib and ______ muscle

a Psoas major -middotshyb Erector spinae c Quadratus Iumborum d Diaphragm

10 The anterior structure at the hilum of the kidney is

a) Renal vein ~

b) Renal artery I middot~ I

c) Ureter d) Accessory renal artery

11 Because of origin of the muscle from the lateral one third of the inguinal ligament it

could not fonn the anterior wall of the inguinal ligament

a) External oblique b) Internal oblique c) Transversus abdominis_ d) Rectus abdominis

12 A large tumor mass impinges on the splenic artery and its branches as the artery pass out from below

the greater curvature of the stomach Branches o(which of the following arteries would most likely to

effected by the pressure on the splenic artery

a Left gastric b Left gastro-epipJoic c Right gastric d Right gastro-epipoloic e Short gastric_

13 A new born baby has projectile vomiting after each feeding It is determined that there is obstruction

of the digestive tract as a result of annular pancreas Annular pancreas is as a result of an abnormality in which of the following process

a Rotation of the dorsal pancreatic bud around the first part of duodenum b Rotation of the dorsal pancreatic bud around the second part of duodenum c Rotation of the dorsal pancreatic bud around the third part of duodenum d Rotation of the ventral pancreatic bud around the first part of duodenum y Rotation of the ventral pancreatic bud around the second part of duodenum

14 As the liver bud enters the ventral mesogastrium the region of the mesogastrium stretching from the

liver to the anterior abdominal wall is called

a Lesser Omentum b Greater Omentum ~ Falcifrom ligament d Lacunar ligament e Ligamentum teres of liver

16 A patient has absence of his 12th rib In such a patient if the doctor makes an incision to approach his

kidney mistaking the 11 th rib for the 12t he would end up injuring

Which of the following arteries is a direct branch of the gastroduodenal artery The

A right gastric artery

B left gastric artery

C inferior pancreaticoduodenal artery D left gastroepiploic artery

i E)right gastroepiploic artery --

E x pI a nation The right gastric artery is typically a branch of the proper hepatic artery The left gastric artery is a direct

branch of the celiac trunk The right and left gastric arteries anastomose along the lesser curvature of the

stomach The inferior pancreaticoduodenal artery is a branch of the superior mesenteric artery it

anastomoses with the superior pancreaticoduodenal in the head of the pancreas The left gastroepiploic

artery is a branch of the splenic artery it anastomoses with the right gastroepiploic artery along the greater

curvature of the stomach The right gastroepiploic artery is a branch of the gastroduodenal artery The

other branch of the gastroduodenal artery is the superior pancreaticoduodenal artery

Which of the following pairs of veins join together to form the portal vein The

A superior mesenteric vein and inferior mesenteric vein

B inferior mesenteric vein and splenic vein

C superior mesenteric vein and splenic vein

Ip)splenic vein and left gastric vein E superior mesenteric vein and left gastric vein

Explanation

The portal vein is formed behind the neck of the pancreas by the union of the superior mesenteric vein

and the splenic vein The inferior mesenteric vein drains into the splenic vein The left gastric vein drains

directly into the portal vein After the portal vein forms it enters the hepatoduodenalligament of the

lesser omentum to reach the liver The portal vein is the most posterior structure in the hepatoduodenal

ligament

At which of the following vertebral levels does the duodenum pass anterior to the aorta - _- shy

All ~

B L2 7~

CL3 I

~DL4

E L5

Explanation

The duodenum begins at the pyloric sphincter at the level of Ll The second (or descending) portion of

the duodenum is to the right of the aorta and extends inferiorly from the level of Ll to the level of L3 The third part of the duodenum crosses the aorta from the right side to the left side at the level of L3 The

fourth (ascending) portion of the duodenum extends from the level of LJ to the level of L2 The

duodenum ends at the duodenojejunal flexure The superior mesenteric artery passes anterior to the

duodenum as the duodenum passes anterior to the aorta The duodenum can be constricted at this level

In which of the following locations will perforation of the digestive tract result in the spilling of luminal

contents into the - lesser peritoneal sac

A Anterior wall of the second portion of the duodenum B Posterior wall of the second portion of the duodenum

C Anterior wall of the stomach

~Posterior wall of the stomach E Posterior wall of the transverse colon

Explanation

The posterior wall of the stomach is related to the lesser peritoneal sac The anterior wall of the stomach is related to the greater peritoneal sac The anterior wall of the second portion of the duodenum is related to the greater peritoneal sac The posterior wall of the second portion of the duodenum is related to the retroperitoneal space The posterior wall of the transverse colon is related to the greater peritoneal sac

The ureter lies against the anterior surface of which of the following muscles shyA Crus oftne diaphragm B Quadratus lumborum

0 Psoas major D Transversus abdominis

E Iliacus

Explanation The ureter exits the renal pelvis at about the level of vertebra L2 As it descends along the posterior abdominal wall it lies on the anterior surface of the psoas major The psoas major muscle arises from the bodies of the lower lumbar vertebrae The psoas major muscle is joined by the iliacus to fonn the

iliopsoas muscle The iliopsoas muscle then attaches to the lesser trochanter of the femur and is the major

flexor of the hip

As the right ureter passes the pelvic brim it lies against the anterior surface of which of the following

blood vessels

A Gonadal artery B Inferiorvena cava C Internal iliac artery

rJ- External Iliac artery

E Inferior mesenteric artery

Explanation

The ureter lies in the extraperitoneal space in the posterior abdominal wall Alter leaving the kidney it

passes inferiorly on the anterior surface of the psoas major muscle At the pelvic brim the ureter passes

into the pelvis At this point the common iliac artery is dividing into the external and iliac arteries The

ureter lies on the anterior surface of the external iliac artery immediately distal to the bifurcation This is a useful landmark for a surgeon to locate the ureter

When extravasated urine passes from the superficial perineal space into the anterior abdominal wall it is

found immediately deep to which of the following layers of the anterior abdominal wall

-ltScarpas fascia

B External oblique muscle

C Internal oblique muscle D Transversus abdominis muscle

E Transversalis fascia

Explanation

The superficial perineal space is bound by Colles fascia the fibrous portion of the superficial fascia This

layer of fascia is continuous with Scarpas fascia the fibrous portion of the superficial fascia of the anterior abdominal wall Therefore urine that is deep to Colles fascia will remain deep to Scarpa s fascia The urine will spread in the plane between Scarpas fascia and the external oblique layer

When a horseshoe kidney develops the ascent of the kidney is restricted by the A internal iliac artery B external Iliac artery

C common iliac artery

inferior mesenteric artery

E superior mesenteric artery

Explanation

A horseshoe kidney develops when the inferior poles of the to kidneys fuse together as they ascend into

the abdomen from the pelvis The first anterior midline vessel that is encountered by the horseshoe kidney

is the inferior mesenteric artery This artery prevents the kidney from continuing its ascent

The left testicular vein drains into which of the following veins

A Left internal iliac vein B Left common iliac vein

bflnferior vena cava D Left renal vein I

E Left internal pudendal vein

Explanation

The left testicular vein drains into the left renal vein The right testicular ~i~[~nsltjectlY into the

inferior vena cava This difference in venous drainage is believed to explain the greater incidence of

varicocele on the left side than on the right The venous drainage from the penis is to the internal vein

which then drains into the internal Iliac vein

The spinal nerve that provides cutaneous branches to the skin around the umbilicus is

A TS B TW-shy

C TI2

DL2 EtA

Explanation

The tenth intercostal nerve is the anterior ramus of the TIO spinal nerve After passing through the tenth

intercostal space the nerve continues forward in the anterolateral abdominal wall in the plane between

the internal oblique muscle and the transversus abdominis muscle In the abdominal wall the nerve innervates to the abdominal wall muscles as well as the skin and the parietal peritoneum The umbilicus is

a useful landmark for the region of distribution of the tenth thoracic nerve

The ligament of the vertebral column that resists its extension is the Aligamentum flavum

B supraspinous ligament

C posterior longitudinal ligament

D anterior longitudinal ligament

E interspinous ligament

Explanation

The ligaments of the vertebral column that resist flexion of the column include the supraspinous ligament

interspinous ligament ligamentum fiavum and posterior longitudinal ligament The ligament that resists

extension is the anterior longitudinal ligament This longitudinal ligament is very broad and strong It

covers the anterior and anterolateral surfaces of the vertebral bodies and the intervertebral disks In

addition to resisting extension the anterior longitudinal ligament provides reinforcement to the anterior

and anterolateral surfaces of the intervertebral disk The posterior longitudinal ligament is relatively

narrow and covers the posterior surface of the vertebral bodies and the intervertebral disks This ligament

reinforces the posterior surface of the disk The posterolateral surface of the disk is not reinforced and it

is through this region that herniation of the nucleus pulposus usually occurs

A patient presents with epigastric and right upper quadrant pain The pain is most intense 2-4 hours after

eating and is reduced by the ingestion of antacids The patient states that he has passed black tarry stools

(melena) within the last week Fiberoptic endoscopy reveals a yellowish crater surrounded by a rim of

erythema that is 3 cm distal to the pylorus Accordingly an ulcer has been identified in the patients

A fundus

B antrum

C duodenum

D jejunum

E ileum

A number of physiologic genetic and other factors increase the risk of gastric (and duodenal) peptic

ulcers The evidence that H pylori plays a principle role is compelling Smoking and caffeine are known to adversely affect the morbidity mortality and healing rates of peptic ulcers In general first-degree

relatives of peptic ulcer patients as well as males have a threefold to fourfold increased risk of developing this disorder Paradoxically in gastric ulcer disease acid secretion is not elevated It is possible that

excess secreted hydrogen ion is reabsorbed across the injured gastric mucosa In general a defect in gastric mucosal defense is the more important local physiologic

A patient presents with symptoms of duodenal obstruction caused by an annular pancreas Annular pancreas is caused by

A rotation of the dorsal pancreatic bud into the ventral mesentery B rotation of the ventral pancreatic bud into the dorsal mesentery

fJ failure of the major and minor pancreatic ducts to fuse ~ ~ cleavage of the ventral pancreatic bud and rotation of the two portions in opposite directions around -the duodenum E formation of one pancreatic bud instead of two

Explanation Normally the ventral pancreatic bud rotates around the gut tube to reach the dorsal pancreatic bud The two buds fuse to form a single pancreas and the distal portions of the two ducts fuse The ventral pancreatic bud forms the inferior portion of the head of the pancreas the uncinate process and the major pancreatic duct (of Wirsung) The dorsal pancreatic bud forms the superior part of the head the neck body and tail and the minor pancreatic duct (of Santorini) Annular pancreas is the result of the ventral pancreatic bud dividing into two portions before it rotates into the dorsal mesentery Each portion rotates in opposite directions to get to the dorsal mesentery thus encircling the duodenum The presence of annular pancreas can constrict the duodenum thus obstructing its lumen

In n _ phranlc----

Gon ~l ----_1 Lum bltano

~~--- CornmQ1t bull ac

+-~4--- lnlllirnaJ ilic

xtem iliac

OBJECTIVE - Identify the blood supply to each of the structures listed in the table on the previous page

Ill give you a head start

FOREGUT - Supplied bV Celiac Tru nk (T12)

Proper hepatic

GastiooUod 13Jafter

1nferlor pancreaticoduodenal artery

Common epatlc

Lett gas ric iiirtery

Spfen artery

shy Gastroepiphgtic artery

~ Superior mesenteric 8rtfry

~

1 Esophagus is a derivative of the foregut so its blood supply originates from the celiac trunk

(T12) The predominant blood supply to abdominal portion of the esophagus is the Esophageal

A (Branch of L Gastric) The venous drainage of the esophagus is particularly important because

it is 1 of 3 clinically relevant sites of Portal Caval anastamoses The Portal Esophageal Vein

meets the Caval Azygos System Persistent bleeding manifests as Esophageal Varices - a fata I

condition

2 The Stomach is also a derivative of the foregut has EXTENSIVE blood supply and is very high

yield on anatomy exams The lesser curvature is supplied superiorly by the L Gastric A (1 of 3

major branches ofthe Celiac trunk) and inferiorly by the R Gastric A ( a branch ofthe proper

Hepatic A) The greater curvature is supplied superiorly by the L Gastroepiploic A (a major

branch of the splenic A) and inferiorly by the R Gastroepiploic A

The Short Gastric arteries (branches of Splenic Artery) supply the fundus of the stomach and

are referred to as EIID ARTERIES because they have no collateral blood supply Therefore if the

splenic artery were occluded (ex - increased pressure in the ommental bursa) - there would be

ischemia to the fundus of the stomach Venous drainage of the stomach is extensive via various

veins lead ing to the portal system Posterior to the stomach the IMV joins the splenic V which

joins the SMV to form the PORTAL VEIN ADAMS

3 Duodenum blood supply has high clinical relevance because it is the junction of the foregut and

midgut and therefore is the site of anastamoses between branches ofthe Celiac Trunk (main

foregut artery) and the Superior Messenteric Artery (main midgut artery) The Proper hepatic

artery gives off the gastroduodenal artery which travels behind the 1st part of the duodenum

This point has high clin ical relevance because duodenal ulcers are very common and a posterior

rupture of the 1st part of the duodenum could rupture the gastroduodenal artery causing

traumatic abdominal bleeding The Gastroduodenal artery first gives off the R Gastroepiploic A

(mentioned above) and proceeds as the Superior pancreatico duodenal artery (supplies the

pancreas and duodenum) which anastamoses with the inferior pancreatico duodenal A (branch

of the SMA) This is the junction of foregut and midgut and occurs near the opening of the

bil iary system into the duodenum (ampula of vater) Portal venous drainage here is responsible

for delivering nutrients from digestion to the liver for metabolism Appreciate that the Superior

mesenteric artery (artery of the midgut) branches from the aorta at Ll travels posterior to the

pancreas than moves anteriorly (at the jxn of the pancreatic headbody) and comes over the

3rd4th part of the duodenum Tumor of the head of the pancreas can compress the SMA

4 Jiver blood supply is via the common hepatic artery (major branch of the cel iac trunk) The

common hepatiC becomes the proper hepatic gives off the R gastric A and the Gastroduodenal

A and then joins the common bile duct and the portal vein in the portal triad Clinical- if a

patient were bleeding from the hepatic A a surgeon can stick his fingers in the epiplOic foramen

and squeeze the free edge of the hepatoduodenalligament in order to stop bleeding to the

area Please note that the hepatic a branches into Rand L hepatic A The Right hepatic artery

gives off the cystic artery which supplies the gallbladder Afferent venous supply is via the

Portal vein which is bringing nutrient rich blood to the liver After metabolism takes place

venous blood leaves the liver through the hepatic veins into the IVC PLEASE UNDERSTAND THE

RELATIONSHIP OF THESE STRUCTURES - ADAMSNETIERSNH Etc

5 Pancreas - Head is supplied via the superior and inferior pancreaticoduodenal arteries

(mentioned above) The tail (situated towards the hilum of the spleen) is supplied via the

pancreatic branches of the splenic artery (END ARTERIES) This blood supply is very important

because the endocrine Alpha and Beta Cells from the pancreatic islets of lagerhans are located

towards the tail This is where Insulin and Glucagon is released to the blood

Now complete this for mid and hindgut structures Make sure to note clinically relevant arterial

anastomoses as well as portal caval anastomoses FYI Appendix blood supply SMA + IMA

anastamoses marginal artery Portalcaval rectal veins fhemmorhoids) and periumbilical caput

medusa are high yield THE BUTT THE GUT and THE CAPUT

Abdominal Development

Liver

Ij1f

II wall b

oh liN ~ VltJrti n be- bull

Pancreas

Secondary Retroperitonealization e I~tl r 1 a v-mtrai m ellter

Rotations of the Gut I i Ij (lIl1UtIJ f~ l r tilt

()l td 10 me l-ft and he v

--~--- -~ -~-~

i

I AolaijonjoI~guf I

STOMACH BED (IDENTIFY IN ADAMS)- the structures posterior to the ommental bursa which

support the stomach in the supine position

Abdomnal JQrUI

Splnic vein

OmQ-oul tv~ ) O(s(Jroa)

Lojt(r o m nturrt (hpJtodu o d~n31 Hid

Gadrl)SplerH (g3stroll~nal) IIgam~nt

hiad h~~atogrtricent IIQdmiddotcrt~)

Lt Dome of Diaphragm (why left Look this up in Adams)

Spleen (What is the blood supply)

Left Kidney (What is the blood supply - AND how is it different from the R kidney)

Suprarenal Gland (What is the Arterial AND Venous Blood supply - how are they different)

Pancreas (How does supply differ from Head to Tail What is the SMA Relationship)

Transverse Mesocolon

liver - ADAMSWET - Make sure you look at the liver in wet lab

Left triangular nl1am~nt

ComoaDj ligamnt

Erophg~1 impre$ioo

Hepatio veins

In1erior -ifena middotr3)Ia

Fibrous appendix o-t

live

impr~j on

Heprorendl p~rtion of Q)(Qllary ligament

Righllri~n9ul r 1I~met

(Common) bile quol

Gr)mmCtr~ hepatic dlJct

Ccentic duct

Duodenal impression

GaJdate p-fr)~S

Hepatic artgtrl prop-f iiiiila - Faloiform ligament

_ - shy Round ligamen liver

~--F-- CoJio imprgt-ssi-on

Prta heptis

Identify the lobes impressions and embryonic remnants associated with the liver

Caudate Lobe Quadrate Lobe Right Lobe Left Lobe Round ligament Falciform Ligament

Ligamentum Venosum (what is its fxn in embryonic life) Hepatic Veins (NOT PART OF THE

PORTAL TRIAD) IVC PORTAL TRIAD - Contents relationship cross section etc Know the

Galbladder relationship to the lobes of the liver

Biliary Duct System - Make sure you understand the sequence of these structures - BE ABLE TO

DRAW A FLOW CHART

TPVd i

t

I t

1 __ Cm-(r

patk GlJet

I

J

Clinical = JAUNDICE is caused by anything that prevents delivery of bile to intestine Tumor of the

head of the pancreas Stones etc Patient will have pale stools and yellowish colored mucus

membranes

Clinical- Any scenario that tells you the patient has BILLOUS VOMIT means that the obstruction to

the flow of digestive contents is after the Ampulla of Vater (Site of Entry of Billiary system to the

duodenum) - ie Duodenal Atresia

Spleen -located posterior to the mid axillary line between ribs 9 and 11 Make sure you know that

the 10th rib is the main axis of the spleen and this organ is susceptible to injury (stab wound errant

thoracoce ntesis etc)

The spleen is derived from mesodermal cells - NOT THE GUT TUBE

The spleen rests on the left colic flexure associates with the tail of the pancreas Know the

structures entering the Hilum of the spleen

Sh rt O~-t~ic 1 0(0 10 rtiltSPIric Iloa nt

(cut)

Peritoneum - similar concept to Pleura - think of a fist in a balloon

Visceral Peritoneum - Layer of balloon touching your fist

Parietal Peritoneum - Layer of balloon not touching your fist

Your fist represents the organ your wrist is the hilum and your arm contains the blood supply

entering the organ

Appreciate that there will never be organs in the peritoneal cavity - rather these organs invaginate

the cavity Kaplan videos

RULES OF NOMENCLATUREshy

1 Organ completely surrounded by peritoneum - peritoneal organ

2 Organ partially surrounded by peritoneum- Retroperitoneal

3 Peritoneum surrounding peritoneal organ is VISCERAL peritoneum

4 Peritoneum surrounding retroperitoneal organ is PARIETAL peritoneum

5 Peritoneum connecting visceral to parietal is called messentary 2 messentaries in the

gut Dorsal (to the gut tube) and ventral (to the gut tube) messentary

Aorta is in Retro peritoneal position - but blood must reach peritoneal position - vessels travel through

messentary All peritoneal organs will have blood supply reaching through messentary

-Mesentery is a 2 layer peritoneum with a neurovascular communication between body wall and organ

- Ligament connects one organ with another or to the abdominal wall (Ommentum = ligament)

lesser Ommentum (attach lesser curvature of stomach and duodenum to liver) =Hepatoduodenal

Ligament and Hepatogastric Ligament

Has a Superior and Inferior Recess (Accumulation of Fluid in Ascites)

Communicates with the greater sac through the epiplic foramen (what structures pass through

this foramen)

Boundaries - you must be able to visualize this

o Anterior - stomach

o Posterior - parietal peritoneum pancreas

o Superior - superior recess (bw diaphragm and coronary ligament)

o Inferior -Inferior recess (bw layers or greater momentum

Greater Ommentum (attach greater curvature of stomach) Gastrophrenic ligament Gastrosplenic

ligament gastrocolic ligament

The greater omentum is the largest peritoneal fold It consists of a double sheet of peritoneum folded on itself so that it is made up of four layers The two layers which descend from the greater curvature of the stomach and commencement of the duodenum pass in front of the small intestines sometimes as low down as the pelvis they then turn upon themselves and ascend again as far as the transverse colon where they separate and enclose that part of the intestine

ABDOMINAL PAIN

Parietal Peritoneum - supplied by same vasculature lymphatics and nerves supplying body wall it

lines and diaphragm Sensitive to pain pressure heat cold well localized

Visceral Peritoneum - supplied by same vasculature lymphatics and somatic nerve of organ it covers

Insensitive to touch heat cold and laceration - referred to dermatome of spinal ganglia providing

sensory fibers Where does appendicitis refer to

Foregut pain - epigastric area (ie - cholycystitis)

Midgut pain - periumbilical area (ie - appendicitis)

Hindgut Pain - suprapubic area (ie - diverticulitis)

Extra ImagesConcepts

ll~_____-

FalifCtrm ligament oind r~ud ligamet f Ilver

Blood from splenio gastriC and inferiof rne$e-rteri v~ins

Ca-I tributaries

Lett gastrio Ifein

Posterior superior pan~reatioodul)denal vaihS

Lott gamo-om~nlal (9aropip lomiddotic) -in

Poq_~ tjol imerl-9-r panCJertlcorllJod-nal veiopound --amp----I- - ~J Right grtr~-omntal

Anwrior interi (gartroepiploic) Jjn

pan euaii cod vl)denal veins middot Inf~Ji (t r mesentric vein

Miqdle (olic vein

Right cl)licvein Sigmoid and rectosigml)id (ei ns

IhH)Collc(~io

--- Mi~dl laquooLJl gtjrltgt

PoM ca vl1 illasto)moses -----shyampoptoageal 2 Paraumbilie-lt11 Inferi or Fectal vei ns

3 Recial 4 REuoperHonea1

Know how the Portal vein is formed I 4 sites of portal caval anastamoses and 1 clinical shunt

Col li t ltt-~ otTl~tI ~nj pc~ 1lt1 turJoG

Ltf 14i1 tImiddot~ artoftl9 on tj phtAt$

L-oftqf 4t t~r 1=laquoIran d 1 bull shy~p l ci rj o fOOOts

Nerves follow the arteries - appreciate the splanchnic nervous system I

Uet~ric branch of left ~nal art

Ureterie branch of righi renal artelY

Left Zld lumbar in and co mlTlunication to as)erdin9 lumbar l(~in Hi ~ht tEZ1~~t~ t3r j t itn ~ nJ l1t- rlnd lfe i r1

Inferior me5nteri~ artery

Notice that the right testicular vein drains directly into the IVC and the right testicular artery drains

directly into the aorta However the left testicular vein drains into the L renal vein at a right angleshy

reason left testicle is lower and more susceptible to varicocele (bag of worms)

Also notice that the left renal vein has a longer course because the IVC is on the right side whereas

the right renal artery has a longer course because the aorta is on the left side

Appreciate the anterior to posterior relationship of structures in the hilum of the kidney - VAP - Vein

Artery Renal Pelvis (Ureter)

11____ __ L_ L_ n VJ __ _ _ t_L I I_ _ L __ L_ I -pound1 bull LI_~-I ____

Posterior View of Head of Pancreas in ( of Duodenum

Celiao hunk

Co mmon ~L~jJth art~ry

GastNduQdonal artrf (partilly in phantn)

P1)Sterior $Up~Jior panCflaticuduodfmal art~r~t

(Co mm on) bile duct

middot~1t~~t-1l---~-~- Right gshomiddotomental (gastoe plp lolc) 3rte (phantomost)

Grener paocre atic art-ry

1n1~rjor pancr-iatlc artery

Jtrifll supejo r pal)oreailcento)dJodenal artr1 (phantom)

Anastomotlo branch

POostetlor bJanch of jo f~ri of pan-reatir(lduodensl drttnj

Anterio r branch of i flferior palcreati~)duodenal art~(phan1om)

Notice the extensive blood supply to the pancreas and duodenum via the branches of the celiac trunk

Notice collateral supply from SMA branches - makes sense bc this is the jxn of foregutmidgut

Identify the vessels in this arteriogram

Hiltid i)f N~ck oi B)dvof Tail 01 pa nereas pan cent~as P-nmiddot-reas panCtCas

I nferie v~na cava

jHept1iic p(lrlai v~in

Port1 tnd H~pti lt a ftH prol

Comm on) bll duct

Ouodtnum

~ft colic (sio)Atta~ hmtrlt jt~xJr-ofha~elSe

muo(IIQn

Right ~lIc (h~j)tic)

il~gtture

In1triol m~oten lIein (rttr op~ritoMdO

SlJp efl or mes~n~fiC amrV and lipln

KNOW YOUR NEIGHBORHOOD

Questions

vVhiJh structure supplied by a bnmdlof the cclia( artery is not derivcd from foregut LemCJUCrITI

(A) Head of the pancte-a5

CD) Pyloric duolenum

Cystkduct

( Liver hepatocyt~~

~F) Body of the spleen

An infant presents with an omrhaJucele at birth -hi oJ the [oHm illg applies to his cM1-dition

(A) It is 31so seen ill p4titnts with aganghonic megacolon

(11) ft reuirs from a fal1ure of resorption of theviteUine d let

(C) It results from herniation at the-site of regression of the right umbilk vein

DJ It is caustd by faihtrc of recanalization of the midgut part of the duodenum

~ It ill camioo by a failuIt vf the midgul to return to the abGQminal uity after herniashytion in-n the urnbilk s l stalk

Ot er than the spleen occlusion Cif the spit-Ilk artery at its odgin wm most likely affect die blood supply to jllch st cnud

(A) Jejunum

(B) Body of th pal1~lltas

(C) LeSStT Cllmiddotlaturc of tl )toma-ch

(D Duodenum dista to the entrance of the Ornmou bile duct

E Fundus of the stomach

A 38-yeu-old batL~er with a history of heartburn suddenly experiences excluciating pain in the (plgastric region of th~ abdomeu SurgCry is perf~rme immediard y upon admisshysion to the 1IlcrgCJliy tuomh~re i~ evidence uf a ruptured ulcer in the posterior waU of the stomach Vhere will a surgeon first fi nd the stomach contenlSf

A) Greater p4ritoneal sac

rB) Cul~de-s~c of Douglas (--

C Omental bursa ~

--D) Paracolic gutter

rEj Between -he panttal perimltum and the posterior body wal1

At birth an infant presents with a st()ma~ rb~tbas~njJled jfltotb~diaplfagru 1A1ltre is the defect thatresulied iiitJle heini~t()n shy~tsophagealbiatus

7 - rH-- Hiatus for the inferior vena cava

( Pleuroperitoneal membrane -(0) Septum transvcrsum

(E) Right Crlt~

An infant born with DOVv7l syndrome presents with bili()u~ vomiting Ahat congenital defect does the infant have

(A) Pyloric stenosis

(B) Meckel diverticulum C) Ornphaloce1e

(D) Gastroschisis

( ~ ) Duodenal atresia y A patient with cirrhosis of the liver presents with ~ bacalvaricestnlreased retrograde pressure in which veins caused the varices

(A) Paraumuilical

(B) Splenic

(ct AzygltJus

(15))G~trk ( (-F) Superior mesemeric

A htaltby 3-year~old male patient experiences a hernial sa protruding from the anterior abdominal wall about halfway between me anterior superior ilia spine and the pubk tuberde Pulsations of al1 artery are palpated medial to the protrusion site through the abdominal walL Which layer of the anterior abdominal wall will first be traversed by the

1hctma

fA) Rectus sheath (B) External oblique aponeurosis

(C) Inguinal ligament

lD) Transversalis fusda

(E) Cremasteric fa~cia

After 5urgi(aj ffpair of a hernia the patient tXperienccs mtmlgtness in the skin on the anteshyrior aspect of the S(Totum_ Vhaf nerve may have been lesioned during thehemiorrhaphy

(A) Femoral

(B) Obturator

(C) Ilioinguinal

(D) lliohypogastrk

(E) Pudendal

A 23~year-LJld female secretary il1 good health ~-uddcn1) doubles over with pain in the a ea of the 1JmbRicu$ Sbe feels vartn and ltneasy and has no appetite That night the pain seems to have mQved to the tower right abdominal regjol1 and she calls her family doctor who then arranges for an ambulance to pk-k her up and take her to the hospitaL Wh ell ntn~ perceived in the area of the urnbilirus most Hkely carried lhe pairfu I sensations into the eNS

tA) Vagus nerves I~

V B)

) Lessersplanchnk nerves

tC) Pudendal nerves

(D) lIiohpogastrk nerves

(E) Greater splam ic l erves

A CT reveals carcinoma in the bOod of the ancreas Vhich blood vessel trut ourses ----~- - -bull ------ --shy

immediately poftterior to the body ofthe pancreas is the m~t likely to be oompressed

(A) Splenk artery

(B) Abdominal aorta (C) Portal vein

(1) Splenic vein

(E) Renal vein

A patient has a penrln1l1ng uker of the posterior wall ot the br~l part ot the (lUooenmn llkh blood vessel is subject to erosion

(A) Common hepatic artery

(B) Gastroouodenal artery

(C) Proper hevatic artery

(D) Celiac artery

(E) Anterior inferior 11amrelltlcoduodcnal attery

Your patient has been diagnosed -ith a carcinoma locallted to the head and l~e(k of the pancreas Another clinical sign would be

A esophageal varices

(8) hemorrhoids

C) a caput medusa

(D) increased pra Teuro n th~ hepatic veins

(E) enlarged right supra lavkular lymph nodes

Wltkh of the foUowing structures develops in the ventral mesentery

(A) Spleen

(B) Jeiunum (C) Head of1ht pancreas (D) Transverse colon (E) Stomach

ti l Uw ~ littwin~ f( S-t lil oai Imdge ~ hi(h or tbt la~)d J truetur tgt liJ llntn nl) he hl p UC iJd [IIi ell

c o

A) drains Ie tht infCrior a La aI

R t middot~nfl0 ~ill to th~ lunlgtn of h i dtlndCrlllfH

(e) m t bull JiJattd on tl l J n T ~H

D ) sup Lc O VSlt I Hlid bhtu l 1 li - -I un oid

( ) U~tpli(t tr j middottUh~ 1 v(( b~nt rfK n1ilc~Zm

ANSWERS AND EXPLANATIONS

Answer E The spleen is t hlttnopodicand lymph organ demlted from mesoderm

Answ~ R Al1 tlmphalocele is caused by it failure of the nlidgut to return to the ahdomir nat cavity after herniation into the umbiliau Stalk Choices Aand D maybe seen in infants with Down syndrome choice D ~s the specific CBuse ofduudcnal JtiCSitt Choice C is (ile cause of gclstrosbisis and Choice B nsults iu a Meurolktldivertku1-tlB

Answer B The fundus ofthe stomach is suppHed by soort gastric brunches of the splenic altery The splenic artery supplies the body and tail of the pancreas part of the greater curvature of the sttmla(h and the spleen Te jejunum part of the head of the pancreas and tht~ duodenum distal to the entrance of the commOll bile duct are supplied by the superior mesenterk artery clll~l ~be less r ctlt1ature cmd the pylQric antrum are supplied by the right and lei gastric art(ries

AnSWftt C Tbeomental bursa or lesser ~ritoneaj sac lies direcdy posterior to the proxshyimal part of the duodeTtlm and the stomach and would be the first site where stomach contents ~Ott1d be fpoundluncL

Answer C A defect in a llleuropcritoneal membrane (uswlly the left) is the typical site of i1 cc-ngenitlI diilphragluatic hemia llere the membr4ne fails to dose ()pound( of the perishycCirdiopcritulleal canals

Answer E DuoJenal atresia and aganglionic megacoion are congwitaI defects S~Il in patients with Dowmiddotnsyndrome

Answer D RulaTgemt~llt of and retrograde flow in g~lstrk vel_ns in particlJl~r the kft gas~ tricveins dilates the capillary bed in rhe wall of the esophagus in (ases of porta yper~

tension Blood flow would increase in and dilampte tribntarkgts of the (lZygOUS vein on the other side of the capiUary bed but flow in this vein is in the typical direction t()ward the superior vena cava Paraumbiii(ltU vein eilgorgement contributes to a caput medusH Splenic ~nlargement might prc~nt with 5plcnonlegaly and balt-kflow in to tlu superior m~~ntclic vein occurs but is asymptomatic

Answer D The patient hagt an indirect inguinal hernia whi~h emerges from the antt-rior abdominal wall through the deep inguinltilling Theeep ring is a fault in the transv~rshysaUs fascia this I~yer wiIJ be penetrated first by the hernia

An~Wer C The ilioinguinal nenc which provides sens~llion to the lnedlal thigh ltmclanteshytior SClotunl pass~lt th rough the 5uperfh_ial inguinal ring ind $subject to inj i1T) becaus-e

it is in the operatitm Held of the erniorrhapny

Auswer B The leMHr splanchnic nerves are sympathdic nerVlts that carry viscera l sensashytlltgtrogt ftom illtllt1m~d ()J stietched gust (itinteitinal ~tructures (in this case the pprndix) into tnt eNS Lesser splanchnic ntTYcsarisc from thmiddot T9--T12 spinal cord segments lt1nd provide sympathetic innenation tD rnidgut siruc1ures whiCh include CLe app~JldD Viscera] Pain arising from affecLed Inidgut ampt 1C1ure is referred over the same dl- matorne~ of spinal segrnertts v-hich provide the sympathetic Innervation n this G1SC of appendicitis the invohen~n t of the ltire) of t e unlhHku indud s the T 10 dermatome

Answer B Of the five choices onty the dscending olon is retroperiton~al aldwould be a lik ~ ( choice to be seen immediately a(~jilcent to t11e posterior abdominal middotn~L

Amwen D The SpltftlC ~-ein ourses posterior to the body of the panneas m its way tt drain into the superior mCSfttltlri( vein

Answcr B TILt glstrodllolticnal artery 1 direct hIamh of the comrootl hepatic artery courses immediately pt))iwri() to the duodenum and is slbject to erosion

Answer B Carcinoma of th pan middott3S in the 1tilt1 may compreampgt the portltil vein at irs orishygill The poTtai vcin is fomled when the splenic vein jQiaswith tfie superior meStllt eric vein The inferiot mesenteric vein joins the ~plenjc vein just priOT to tlli~ point at which the splenic joins the superior Jlleit1ltcri( vein Increescd venous presslu in the inferior mesenteric vein is a cause of emo hoid~

Answer C The- velltral pancreas wilich forms most of the head of the p ~ncr as develops in the ventral mes(ntery as antutgrowth of the hepatic diverticulum Th~ hepatic divershyticulull induding the biIJary appa~atus develops in tbe ventral mesentery of the foregut

Answer~ A The superior mesenteric ~in joins with the spienkvein to form the hepatic portal vciu

Answer D The structure at gttlK is the proper hepatic artery~ whkh suppUesoxygenated b middotood to the liver

MAKE SURE YOU KNOW the diff bw Rectus Sheath above and below the arcuate line

ABOVE

Aponeurosis of xiiltmal obllque musclo

Extemll f)biquw musde

Reotln ilbdomlnls musole S~in

Internal 9bliquQ mY~QI

AponeUfOsi$ of hJH$V~~S Lir9a a lb lbdolTlin~ musolo Tri OJV6 rUi

atldomlnis mUS(loe

Sub cutanlilous tiue (tatty ye r)

BElOW

A POrl lJfosis 01 etemal oblique muscle

Aponeul~)sis 01 Internal oblique mU$cl~

Anteriol lay~ of r~ltdus st~ath EXttom1 oblique rnu$cll

Rectus Jbdominis muscle Intoernal Aponeurc-sis of tra~fersU$ oblique muscle-

at-domlnis muscentl ~ Skio

Tra nsvitSus abdomioLs ml)ZClt

TralSVersaHs fascia Medial umQil iegtt1 1i9Jment -and folj

Uldchus Peritoneum (ir median Umbilj~al Suboutane ous

Extraprftone 11ascia

Ymbilimiddot~1 fold)

preu9poundiea1 fascia

tissue (fatty 4nd m~mbr3n(iUS layers)

o Above the arcuate line (A horizontal line 13 of the distance bw the umbilicus and the

pubic symphysis) -10 Aponeurosis divides into an AntPost Laminae

o The Ant Laminae joins EO and Post Laminae joins Trans Abdominis = Ant and Post

RECTUS SHEATH respectively

o BElOW the arcuate line - all 3 aponeurosis join ANTERIOR to rectus muscle to meet its

counterpart in the midline (linea Alba)

o Take away Msg - The abdomen is devoid of a posterior rectus sheath below the

arcuate line and is therefore more vulnerable to herniasinjuries

Question - A physician makes a deep incision in the patients midline immediately superior to

the pubic symphysis which of the following layers is his knife least likely to pass

Rectus Abdominis External Oblique Ant Rectus Sheath Posterior Rectus Sheath All of the

Above

Answer - All of the above None of the other answer choices are midline structures -LINEA

ALBA

Linea Alba has very poor blood supply - doesnt heal well after surgery Therefore this is a

common site for incisional hernias

a Spleen b Transverse colon c Descending colon d Stomach e Pleura

17 Meckels diverticulum is normally found 2 feet proximal from the

a Pyloric sphincter b Lower esophageal sphincter c Ileo-cecal valve d Middle valve of Huston e Anal valve

18 Ulcer in the posterior wall of the first part of the duodenum would erode ___ artery and would cause bleeding

a Left gastric b Right gastric c Hepatic artery proper d Gastroduodenal artery e Middle colic artery

19 An inflamed appendix is identified by a surgeon on the operation table by noting

a The appendicies epiploicae b The convergence of tenia c The artery of Drummond d The mesocolon e The mesosalphinx

20 The nerve which emerges through the psoas major is

a Femoral b Ilio-inguinal c Ilio-hypogastric d Pudendal e Subcostal

21 The right gonadal vein drains into the

a Azygos b Hemiazygos c Inferior Vena Cava d Right renal vein e Left renal vein

22 The hepatocytes in the liver is derived from

a Ectoderm b Endoderm c Mesoderm

d Neural ectoderm

23 Abscess in the lumbar vertebrae due to tuberculosis would spread to the adjacent muscle which is

a Psoas Major b Iliacus c Quadratus lumborum d Tranversus Abdominis

24 The anterior wall of the inguinal canal is formed by

a External oblique and transverses abdominis b External oblique and fascia transversalis c Internal oblique and external oblique d Internal oblique and transverses abdominis e Fascia transversalis and peritoneum

Meckels diverticulum is a result of which of the following developmental abnormalities shy

A Failure of the vitelline duct to close

B Failure of the herniated intestinal loop to retract into the abdomen

C Failure of the urachus to close

D Failure of the midgut to rotate

E Failure of the hepatic duct to close

Explanation

Meckels diverticulum is a result of the persistence of the proximal part of the vitelline duct This

diverticulum is usually found about 2 feet proximal to the ileocecal junction and is usually about 2 inches

long It is present in about 2 of the popUlation It may be the site of ectopic pancreatic tissue or gastric

mucosa and may develop inflammatory processes and ulcerations Acute Meckels diverticulitis

simulates appendicitis

Which of the following veins carries blood from the esophagus to the portal vein The

A right gastric vein

B left gastric vein c splenic vein D azygos vein

E left gastroepiploic vein

Explanation

The left gastric vein a direct branch of the portal vein drains blood from the lesser curvature of the

stomach and the inferior portion of the esophagus Because branches of the portal vein do not have

valves blood can flow in a retrograde path when there is an obstruction to flow through the portal system or liveL Rlooci Cln then flow from the nortl] vein thr()1Ph the left PRstric vein to the esonhlPlIS lno

through venous communications within the submucosa of the esophagus to esophageal veins that drain

into the azygos vein The increase in blood flow through the esophageal submucosal veins results in esophageal varices

On the posterior wall of the abdomen the celiac ganglion A contains cell bodies of postganglionic parasympathetic neurons B is synapsed upon by neurons in the posterior vagal trunk C is synapsed upon by neurons in the greater splanchnic nerve D contains sensory cell bodies of lumbar spinal nerves E contains cell bodies of neurons that cause an increase in the rate of peristasis

Explanation The celiac ganglion is one of the preaortic ganglia of the sympathetic nervous system It contains cell bodies of postganglionic sympathetic neurons The sympathetic splanchnic nerves contain preganglionic sympathetic neurons that pass through the sympathetic chain without synapsing These splanchnic nerves go to the preaortic ganglia to synapse The greater splanchnic nerve contains preganglionic neurons from spinal cord segments T5-T9 This nerve synapses in the celiac ganglion The nerve fibers in the vagal trunks are preganglionic parasympathetic fibers that go to the walls of the organs that they will innervate and synapse on postganglionic parasympathetic neurons in the walls of those organs Cell bodies of sensory neurons in the abdomen are found in the dorsal root ganglia or the sensory ganglia of the vagus nerve Sympathetic innervation decreases the rate of peristalsis parasympathetic innervation increases the rate of peristalsis

Which of the following pairs of arteries will allow blood to bypass an occlusion of the celiac trunk

A Left gastric artery-right gastric artery

B Left gastroepiploic artery-right gastroepiploic artery

C Superior pancreaticoduodenal artery-inferior pancreaticoduodenal artery

D Splenic artery-common hepatic artery

E Left gastric artery - proper hepatic artery

Explanation The anastoOlosis of a branch of the celiac trunk and a branch of the superior mesenteric artery will

provide collateral circulation around an occlusion of the celiac trunk Each of the other choices pair

branches of the celiac trunk therefore these will not provide collateral flow around the obstruction of the

celiac trunk The left gastric splenic and common hepatic arteries are direct branches of the celiac trunk

The right gastric artery is a branch of the proper hepatic artery which is a branch of the common hepatic artery The left gastroepiploic artery is a branch of the splenic artery The right gastroepiploic artery is a

branch of the gastroduodenal artery whlch is a branch of the common hepatic artery

Which of the following organs has appendices epiploica The

A sigmoid colon

Bjejunum

C duodenum

D stomach E esophagus

Explanation Appendices epiploica are characteristic of the colon Appendices epiploica are subserosal accumulations

of fat None of the organs of the gastrointestinal tract has appendices epiploica except the colon

Page 22: Chirag's Abdomen Review

The Vagal trunks enter the abdomen by passing through which of the following openings in the

diaphragm

a Right crus b Esophageal hiatus ~ c Vena caval hiatus d Aortic hiatus e Left crus

2 The anterior boundary of the epiploic foramen of Winslow is bounded by

a) First part of duodenum b) Lesser curvature of stomach c) Liver d) Hepato-duodenalligament v ~

3 The ilio-inguinal nerve is derived from

a TI2 ry b LI c L2 d L3 e L23

15 Surgically the structure used to suspend the kidney to the diaphragm is

a) Renal fascia b) True capsule c) Perinephric fat d) Paranephric fat

6 If there is portal obstruction because of carcinoma affecting the pancreas which of these of the

following signs would be present

a Caput medusae b Esophageal varices c Rectal varices c

d Pulmonary edema

7 In a sliding hernia the gastro-esophageal junction lies

a) At its normal position b) Below the normal position c) Above the normal position V d) None of the above

8 Which of the following structures is retroperi toneal

A transverse colon B spleen IJ2f6 C ileum D descending colon v r 1pound1111111

9 The renal angle is fonned lgtetween the 12th rib and ______ muscle

a Psoas major -middotshyb Erector spinae c Quadratus Iumborum d Diaphragm

10 The anterior structure at the hilum of the kidney is

a) Renal vein ~

b) Renal artery I middot~ I

c) Ureter d) Accessory renal artery

11 Because of origin of the muscle from the lateral one third of the inguinal ligament it

could not fonn the anterior wall of the inguinal ligament

a) External oblique b) Internal oblique c) Transversus abdominis_ d) Rectus abdominis

12 A large tumor mass impinges on the splenic artery and its branches as the artery pass out from below

the greater curvature of the stomach Branches o(which of the following arteries would most likely to

effected by the pressure on the splenic artery

a Left gastric b Left gastro-epipJoic c Right gastric d Right gastro-epipoloic e Short gastric_

13 A new born baby has projectile vomiting after each feeding It is determined that there is obstruction

of the digestive tract as a result of annular pancreas Annular pancreas is as a result of an abnormality in which of the following process

a Rotation of the dorsal pancreatic bud around the first part of duodenum b Rotation of the dorsal pancreatic bud around the second part of duodenum c Rotation of the dorsal pancreatic bud around the third part of duodenum d Rotation of the ventral pancreatic bud around the first part of duodenum y Rotation of the ventral pancreatic bud around the second part of duodenum

14 As the liver bud enters the ventral mesogastrium the region of the mesogastrium stretching from the

liver to the anterior abdominal wall is called

a Lesser Omentum b Greater Omentum ~ Falcifrom ligament d Lacunar ligament e Ligamentum teres of liver

16 A patient has absence of his 12th rib In such a patient if the doctor makes an incision to approach his

kidney mistaking the 11 th rib for the 12t he would end up injuring

Which of the following arteries is a direct branch of the gastroduodenal artery The

A right gastric artery

B left gastric artery

C inferior pancreaticoduodenal artery D left gastroepiploic artery

i E)right gastroepiploic artery --

E x pI a nation The right gastric artery is typically a branch of the proper hepatic artery The left gastric artery is a direct

branch of the celiac trunk The right and left gastric arteries anastomose along the lesser curvature of the

stomach The inferior pancreaticoduodenal artery is a branch of the superior mesenteric artery it

anastomoses with the superior pancreaticoduodenal in the head of the pancreas The left gastroepiploic

artery is a branch of the splenic artery it anastomoses with the right gastroepiploic artery along the greater

curvature of the stomach The right gastroepiploic artery is a branch of the gastroduodenal artery The

other branch of the gastroduodenal artery is the superior pancreaticoduodenal artery

Which of the following pairs of veins join together to form the portal vein The

A superior mesenteric vein and inferior mesenteric vein

B inferior mesenteric vein and splenic vein

C superior mesenteric vein and splenic vein

Ip)splenic vein and left gastric vein E superior mesenteric vein and left gastric vein

Explanation

The portal vein is formed behind the neck of the pancreas by the union of the superior mesenteric vein

and the splenic vein The inferior mesenteric vein drains into the splenic vein The left gastric vein drains

directly into the portal vein After the portal vein forms it enters the hepatoduodenalligament of the

lesser omentum to reach the liver The portal vein is the most posterior structure in the hepatoduodenal

ligament

At which of the following vertebral levels does the duodenum pass anterior to the aorta - _- shy

All ~

B L2 7~

CL3 I

~DL4

E L5

Explanation

The duodenum begins at the pyloric sphincter at the level of Ll The second (or descending) portion of

the duodenum is to the right of the aorta and extends inferiorly from the level of Ll to the level of L3 The third part of the duodenum crosses the aorta from the right side to the left side at the level of L3 The

fourth (ascending) portion of the duodenum extends from the level of LJ to the level of L2 The

duodenum ends at the duodenojejunal flexure The superior mesenteric artery passes anterior to the

duodenum as the duodenum passes anterior to the aorta The duodenum can be constricted at this level

In which of the following locations will perforation of the digestive tract result in the spilling of luminal

contents into the - lesser peritoneal sac

A Anterior wall of the second portion of the duodenum B Posterior wall of the second portion of the duodenum

C Anterior wall of the stomach

~Posterior wall of the stomach E Posterior wall of the transverse colon

Explanation

The posterior wall of the stomach is related to the lesser peritoneal sac The anterior wall of the stomach is related to the greater peritoneal sac The anterior wall of the second portion of the duodenum is related to the greater peritoneal sac The posterior wall of the second portion of the duodenum is related to the retroperitoneal space The posterior wall of the transverse colon is related to the greater peritoneal sac

The ureter lies against the anterior surface of which of the following muscles shyA Crus oftne diaphragm B Quadratus lumborum

0 Psoas major D Transversus abdominis

E Iliacus

Explanation The ureter exits the renal pelvis at about the level of vertebra L2 As it descends along the posterior abdominal wall it lies on the anterior surface of the psoas major The psoas major muscle arises from the bodies of the lower lumbar vertebrae The psoas major muscle is joined by the iliacus to fonn the

iliopsoas muscle The iliopsoas muscle then attaches to the lesser trochanter of the femur and is the major

flexor of the hip

As the right ureter passes the pelvic brim it lies against the anterior surface of which of the following

blood vessels

A Gonadal artery B Inferiorvena cava C Internal iliac artery

rJ- External Iliac artery

E Inferior mesenteric artery

Explanation

The ureter lies in the extraperitoneal space in the posterior abdominal wall Alter leaving the kidney it

passes inferiorly on the anterior surface of the psoas major muscle At the pelvic brim the ureter passes

into the pelvis At this point the common iliac artery is dividing into the external and iliac arteries The

ureter lies on the anterior surface of the external iliac artery immediately distal to the bifurcation This is a useful landmark for a surgeon to locate the ureter

When extravasated urine passes from the superficial perineal space into the anterior abdominal wall it is

found immediately deep to which of the following layers of the anterior abdominal wall

-ltScarpas fascia

B External oblique muscle

C Internal oblique muscle D Transversus abdominis muscle

E Transversalis fascia

Explanation

The superficial perineal space is bound by Colles fascia the fibrous portion of the superficial fascia This

layer of fascia is continuous with Scarpas fascia the fibrous portion of the superficial fascia of the anterior abdominal wall Therefore urine that is deep to Colles fascia will remain deep to Scarpa s fascia The urine will spread in the plane between Scarpas fascia and the external oblique layer

When a horseshoe kidney develops the ascent of the kidney is restricted by the A internal iliac artery B external Iliac artery

C common iliac artery

inferior mesenteric artery

E superior mesenteric artery

Explanation

A horseshoe kidney develops when the inferior poles of the to kidneys fuse together as they ascend into

the abdomen from the pelvis The first anterior midline vessel that is encountered by the horseshoe kidney

is the inferior mesenteric artery This artery prevents the kidney from continuing its ascent

The left testicular vein drains into which of the following veins

A Left internal iliac vein B Left common iliac vein

bflnferior vena cava D Left renal vein I

E Left internal pudendal vein

Explanation

The left testicular vein drains into the left renal vein The right testicular ~i~[~nsltjectlY into the

inferior vena cava This difference in venous drainage is believed to explain the greater incidence of

varicocele on the left side than on the right The venous drainage from the penis is to the internal vein

which then drains into the internal Iliac vein

The spinal nerve that provides cutaneous branches to the skin around the umbilicus is

A TS B TW-shy

C TI2

DL2 EtA

Explanation

The tenth intercostal nerve is the anterior ramus of the TIO spinal nerve After passing through the tenth

intercostal space the nerve continues forward in the anterolateral abdominal wall in the plane between

the internal oblique muscle and the transversus abdominis muscle In the abdominal wall the nerve innervates to the abdominal wall muscles as well as the skin and the parietal peritoneum The umbilicus is

a useful landmark for the region of distribution of the tenth thoracic nerve

The ligament of the vertebral column that resists its extension is the Aligamentum flavum

B supraspinous ligament

C posterior longitudinal ligament

D anterior longitudinal ligament

E interspinous ligament

Explanation

The ligaments of the vertebral column that resist flexion of the column include the supraspinous ligament

interspinous ligament ligamentum fiavum and posterior longitudinal ligament The ligament that resists

extension is the anterior longitudinal ligament This longitudinal ligament is very broad and strong It

covers the anterior and anterolateral surfaces of the vertebral bodies and the intervertebral disks In

addition to resisting extension the anterior longitudinal ligament provides reinforcement to the anterior

and anterolateral surfaces of the intervertebral disk The posterior longitudinal ligament is relatively

narrow and covers the posterior surface of the vertebral bodies and the intervertebral disks This ligament

reinforces the posterior surface of the disk The posterolateral surface of the disk is not reinforced and it

is through this region that herniation of the nucleus pulposus usually occurs

A patient presents with epigastric and right upper quadrant pain The pain is most intense 2-4 hours after

eating and is reduced by the ingestion of antacids The patient states that he has passed black tarry stools

(melena) within the last week Fiberoptic endoscopy reveals a yellowish crater surrounded by a rim of

erythema that is 3 cm distal to the pylorus Accordingly an ulcer has been identified in the patients

A fundus

B antrum

C duodenum

D jejunum

E ileum

A number of physiologic genetic and other factors increase the risk of gastric (and duodenal) peptic

ulcers The evidence that H pylori plays a principle role is compelling Smoking and caffeine are known to adversely affect the morbidity mortality and healing rates of peptic ulcers In general first-degree

relatives of peptic ulcer patients as well as males have a threefold to fourfold increased risk of developing this disorder Paradoxically in gastric ulcer disease acid secretion is not elevated It is possible that

excess secreted hydrogen ion is reabsorbed across the injured gastric mucosa In general a defect in gastric mucosal defense is the more important local physiologic

A patient presents with symptoms of duodenal obstruction caused by an annular pancreas Annular pancreas is caused by

A rotation of the dorsal pancreatic bud into the ventral mesentery B rotation of the ventral pancreatic bud into the dorsal mesentery

fJ failure of the major and minor pancreatic ducts to fuse ~ ~ cleavage of the ventral pancreatic bud and rotation of the two portions in opposite directions around -the duodenum E formation of one pancreatic bud instead of two

Explanation Normally the ventral pancreatic bud rotates around the gut tube to reach the dorsal pancreatic bud The two buds fuse to form a single pancreas and the distal portions of the two ducts fuse The ventral pancreatic bud forms the inferior portion of the head of the pancreas the uncinate process and the major pancreatic duct (of Wirsung) The dorsal pancreatic bud forms the superior part of the head the neck body and tail and the minor pancreatic duct (of Santorini) Annular pancreas is the result of the ventral pancreatic bud dividing into two portions before it rotates into the dorsal mesentery Each portion rotates in opposite directions to get to the dorsal mesentery thus encircling the duodenum The presence of annular pancreas can constrict the duodenum thus obstructing its lumen

In n _ phranlc----

Gon ~l ----_1 Lum bltano

~~--- CornmQ1t bull ac

+-~4--- lnlllirnaJ ilic

xtem iliac

OBJECTIVE - Identify the blood supply to each of the structures listed in the table on the previous page

Ill give you a head start

FOREGUT - Supplied bV Celiac Tru nk (T12)

Proper hepatic

GastiooUod 13Jafter

1nferlor pancreaticoduodenal artery

Common epatlc

Lett gas ric iiirtery

Spfen artery

shy Gastroepiphgtic artery

~ Superior mesenteric 8rtfry

~

1 Esophagus is a derivative of the foregut so its blood supply originates from the celiac trunk

(T12) The predominant blood supply to abdominal portion of the esophagus is the Esophageal

A (Branch of L Gastric) The venous drainage of the esophagus is particularly important because

it is 1 of 3 clinically relevant sites of Portal Caval anastamoses The Portal Esophageal Vein

meets the Caval Azygos System Persistent bleeding manifests as Esophageal Varices - a fata I

condition

2 The Stomach is also a derivative of the foregut has EXTENSIVE blood supply and is very high

yield on anatomy exams The lesser curvature is supplied superiorly by the L Gastric A (1 of 3

major branches ofthe Celiac trunk) and inferiorly by the R Gastric A ( a branch ofthe proper

Hepatic A) The greater curvature is supplied superiorly by the L Gastroepiploic A (a major

branch of the splenic A) and inferiorly by the R Gastroepiploic A

The Short Gastric arteries (branches of Splenic Artery) supply the fundus of the stomach and

are referred to as EIID ARTERIES because they have no collateral blood supply Therefore if the

splenic artery were occluded (ex - increased pressure in the ommental bursa) - there would be

ischemia to the fundus of the stomach Venous drainage of the stomach is extensive via various

veins lead ing to the portal system Posterior to the stomach the IMV joins the splenic V which

joins the SMV to form the PORTAL VEIN ADAMS

3 Duodenum blood supply has high clinical relevance because it is the junction of the foregut and

midgut and therefore is the site of anastamoses between branches ofthe Celiac Trunk (main

foregut artery) and the Superior Messenteric Artery (main midgut artery) The Proper hepatic

artery gives off the gastroduodenal artery which travels behind the 1st part of the duodenum

This point has high clin ical relevance because duodenal ulcers are very common and a posterior

rupture of the 1st part of the duodenum could rupture the gastroduodenal artery causing

traumatic abdominal bleeding The Gastroduodenal artery first gives off the R Gastroepiploic A

(mentioned above) and proceeds as the Superior pancreatico duodenal artery (supplies the

pancreas and duodenum) which anastamoses with the inferior pancreatico duodenal A (branch

of the SMA) This is the junction of foregut and midgut and occurs near the opening of the

bil iary system into the duodenum (ampula of vater) Portal venous drainage here is responsible

for delivering nutrients from digestion to the liver for metabolism Appreciate that the Superior

mesenteric artery (artery of the midgut) branches from the aorta at Ll travels posterior to the

pancreas than moves anteriorly (at the jxn of the pancreatic headbody) and comes over the

3rd4th part of the duodenum Tumor of the head of the pancreas can compress the SMA

4 Jiver blood supply is via the common hepatic artery (major branch of the cel iac trunk) The

common hepatiC becomes the proper hepatic gives off the R gastric A and the Gastroduodenal

A and then joins the common bile duct and the portal vein in the portal triad Clinical- if a

patient were bleeding from the hepatic A a surgeon can stick his fingers in the epiplOic foramen

and squeeze the free edge of the hepatoduodenalligament in order to stop bleeding to the

area Please note that the hepatic a branches into Rand L hepatic A The Right hepatic artery

gives off the cystic artery which supplies the gallbladder Afferent venous supply is via the

Portal vein which is bringing nutrient rich blood to the liver After metabolism takes place

venous blood leaves the liver through the hepatic veins into the IVC PLEASE UNDERSTAND THE

RELATIONSHIP OF THESE STRUCTURES - ADAMSNETIERSNH Etc

5 Pancreas - Head is supplied via the superior and inferior pancreaticoduodenal arteries

(mentioned above) The tail (situated towards the hilum of the spleen) is supplied via the

pancreatic branches of the splenic artery (END ARTERIES) This blood supply is very important

because the endocrine Alpha and Beta Cells from the pancreatic islets of lagerhans are located

towards the tail This is where Insulin and Glucagon is released to the blood

Now complete this for mid and hindgut structures Make sure to note clinically relevant arterial

anastomoses as well as portal caval anastomoses FYI Appendix blood supply SMA + IMA

anastamoses marginal artery Portalcaval rectal veins fhemmorhoids) and periumbilical caput

medusa are high yield THE BUTT THE GUT and THE CAPUT

Abdominal Development

Liver

Ij1f

II wall b

oh liN ~ VltJrti n be- bull

Pancreas

Secondary Retroperitonealization e I~tl r 1 a v-mtrai m ellter

Rotations of the Gut I i Ij (lIl1UtIJ f~ l r tilt

()l td 10 me l-ft and he v

--~--- -~ -~-~

i

I AolaijonjoI~guf I

STOMACH BED (IDENTIFY IN ADAMS)- the structures posterior to the ommental bursa which

support the stomach in the supine position

Abdomnal JQrUI

Splnic vein

OmQ-oul tv~ ) O(s(Jroa)

Lojt(r o m nturrt (hpJtodu o d~n31 Hid

Gadrl)SplerH (g3stroll~nal) IIgam~nt

hiad h~~atogrtricent IIQdmiddotcrt~)

Lt Dome of Diaphragm (why left Look this up in Adams)

Spleen (What is the blood supply)

Left Kidney (What is the blood supply - AND how is it different from the R kidney)

Suprarenal Gland (What is the Arterial AND Venous Blood supply - how are they different)

Pancreas (How does supply differ from Head to Tail What is the SMA Relationship)

Transverse Mesocolon

liver - ADAMSWET - Make sure you look at the liver in wet lab

Left triangular nl1am~nt

ComoaDj ligamnt

Erophg~1 impre$ioo

Hepatio veins

In1erior -ifena middotr3)Ia

Fibrous appendix o-t

live

impr~j on

Heprorendl p~rtion of Q)(Qllary ligament

Righllri~n9ul r 1I~met

(Common) bile quol

Gr)mmCtr~ hepatic dlJct

Ccentic duct

Duodenal impression

GaJdate p-fr)~S

Hepatic artgtrl prop-f iiiiila - Faloiform ligament

_ - shy Round ligamen liver

~--F-- CoJio imprgt-ssi-on

Prta heptis

Identify the lobes impressions and embryonic remnants associated with the liver

Caudate Lobe Quadrate Lobe Right Lobe Left Lobe Round ligament Falciform Ligament

Ligamentum Venosum (what is its fxn in embryonic life) Hepatic Veins (NOT PART OF THE

PORTAL TRIAD) IVC PORTAL TRIAD - Contents relationship cross section etc Know the

Galbladder relationship to the lobes of the liver

Biliary Duct System - Make sure you understand the sequence of these structures - BE ABLE TO

DRAW A FLOW CHART

TPVd i

t

I t

1 __ Cm-(r

patk GlJet

I

J

Clinical = JAUNDICE is caused by anything that prevents delivery of bile to intestine Tumor of the

head of the pancreas Stones etc Patient will have pale stools and yellowish colored mucus

membranes

Clinical- Any scenario that tells you the patient has BILLOUS VOMIT means that the obstruction to

the flow of digestive contents is after the Ampulla of Vater (Site of Entry of Billiary system to the

duodenum) - ie Duodenal Atresia

Spleen -located posterior to the mid axillary line between ribs 9 and 11 Make sure you know that

the 10th rib is the main axis of the spleen and this organ is susceptible to injury (stab wound errant

thoracoce ntesis etc)

The spleen is derived from mesodermal cells - NOT THE GUT TUBE

The spleen rests on the left colic flexure associates with the tail of the pancreas Know the

structures entering the Hilum of the spleen

Sh rt O~-t~ic 1 0(0 10 rtiltSPIric Iloa nt

(cut)

Peritoneum - similar concept to Pleura - think of a fist in a balloon

Visceral Peritoneum - Layer of balloon touching your fist

Parietal Peritoneum - Layer of balloon not touching your fist

Your fist represents the organ your wrist is the hilum and your arm contains the blood supply

entering the organ

Appreciate that there will never be organs in the peritoneal cavity - rather these organs invaginate

the cavity Kaplan videos

RULES OF NOMENCLATUREshy

1 Organ completely surrounded by peritoneum - peritoneal organ

2 Organ partially surrounded by peritoneum- Retroperitoneal

3 Peritoneum surrounding peritoneal organ is VISCERAL peritoneum

4 Peritoneum surrounding retroperitoneal organ is PARIETAL peritoneum

5 Peritoneum connecting visceral to parietal is called messentary 2 messentaries in the

gut Dorsal (to the gut tube) and ventral (to the gut tube) messentary

Aorta is in Retro peritoneal position - but blood must reach peritoneal position - vessels travel through

messentary All peritoneal organs will have blood supply reaching through messentary

-Mesentery is a 2 layer peritoneum with a neurovascular communication between body wall and organ

- Ligament connects one organ with another or to the abdominal wall (Ommentum = ligament)

lesser Ommentum (attach lesser curvature of stomach and duodenum to liver) =Hepatoduodenal

Ligament and Hepatogastric Ligament

Has a Superior and Inferior Recess (Accumulation of Fluid in Ascites)

Communicates with the greater sac through the epiplic foramen (what structures pass through

this foramen)

Boundaries - you must be able to visualize this

o Anterior - stomach

o Posterior - parietal peritoneum pancreas

o Superior - superior recess (bw diaphragm and coronary ligament)

o Inferior -Inferior recess (bw layers or greater momentum

Greater Ommentum (attach greater curvature of stomach) Gastrophrenic ligament Gastrosplenic

ligament gastrocolic ligament

The greater omentum is the largest peritoneal fold It consists of a double sheet of peritoneum folded on itself so that it is made up of four layers The two layers which descend from the greater curvature of the stomach and commencement of the duodenum pass in front of the small intestines sometimes as low down as the pelvis they then turn upon themselves and ascend again as far as the transverse colon where they separate and enclose that part of the intestine

ABDOMINAL PAIN

Parietal Peritoneum - supplied by same vasculature lymphatics and nerves supplying body wall it

lines and diaphragm Sensitive to pain pressure heat cold well localized

Visceral Peritoneum - supplied by same vasculature lymphatics and somatic nerve of organ it covers

Insensitive to touch heat cold and laceration - referred to dermatome of spinal ganglia providing

sensory fibers Where does appendicitis refer to

Foregut pain - epigastric area (ie - cholycystitis)

Midgut pain - periumbilical area (ie - appendicitis)

Hindgut Pain - suprapubic area (ie - diverticulitis)

Extra ImagesConcepts

ll~_____-

FalifCtrm ligament oind r~ud ligamet f Ilver

Blood from splenio gastriC and inferiof rne$e-rteri v~ins

Ca-I tributaries

Lett gastrio Ifein

Posterior superior pan~reatioodul)denal vaihS

Lott gamo-om~nlal (9aropip lomiddotic) -in

Poq_~ tjol imerl-9-r panCJertlcorllJod-nal veiopound --amp----I- - ~J Right grtr~-omntal

Anwrior interi (gartroepiploic) Jjn

pan euaii cod vl)denal veins middot Inf~Ji (t r mesentric vein

Miqdle (olic vein

Right cl)licvein Sigmoid and rectosigml)id (ei ns

IhH)Collc(~io

--- Mi~dl laquooLJl gtjrltgt

PoM ca vl1 illasto)moses -----shyampoptoageal 2 Paraumbilie-lt11 Inferi or Fectal vei ns

3 Recial 4 REuoperHonea1

Know how the Portal vein is formed I 4 sites of portal caval anastamoses and 1 clinical shunt

Col li t ltt-~ otTl~tI ~nj pc~ 1lt1 turJoG

Ltf 14i1 tImiddot~ artoftl9 on tj phtAt$

L-oftqf 4t t~r 1=laquoIran d 1 bull shy~p l ci rj o fOOOts

Nerves follow the arteries - appreciate the splanchnic nervous system I

Uet~ric branch of left ~nal art

Ureterie branch of righi renal artelY

Left Zld lumbar in and co mlTlunication to as)erdin9 lumbar l(~in Hi ~ht tEZ1~~t~ t3r j t itn ~ nJ l1t- rlnd lfe i r1

Inferior me5nteri~ artery

Notice that the right testicular vein drains directly into the IVC and the right testicular artery drains

directly into the aorta However the left testicular vein drains into the L renal vein at a right angleshy

reason left testicle is lower and more susceptible to varicocele (bag of worms)

Also notice that the left renal vein has a longer course because the IVC is on the right side whereas

the right renal artery has a longer course because the aorta is on the left side

Appreciate the anterior to posterior relationship of structures in the hilum of the kidney - VAP - Vein

Artery Renal Pelvis (Ureter)

11____ __ L_ L_ n VJ __ _ _ t_L I I_ _ L __ L_ I -pound1 bull LI_~-I ____

Posterior View of Head of Pancreas in ( of Duodenum

Celiao hunk

Co mmon ~L~jJth art~ry

GastNduQdonal artrf (partilly in phantn)

P1)Sterior $Up~Jior panCflaticuduodfmal art~r~t

(Co mm on) bile duct

middot~1t~~t-1l---~-~- Right gshomiddotomental (gastoe plp lolc) 3rte (phantomost)

Grener paocre atic art-ry

1n1~rjor pancr-iatlc artery

Jtrifll supejo r pal)oreailcento)dJodenal artr1 (phantom)

Anastomotlo branch

POostetlor bJanch of jo f~ri of pan-reatir(lduodensl drttnj

Anterio r branch of i flferior palcreati~)duodenal art~(phan1om)

Notice the extensive blood supply to the pancreas and duodenum via the branches of the celiac trunk

Notice collateral supply from SMA branches - makes sense bc this is the jxn of foregutmidgut

Identify the vessels in this arteriogram

Hiltid i)f N~ck oi B)dvof Tail 01 pa nereas pan cent~as P-nmiddot-reas panCtCas

I nferie v~na cava

jHept1iic p(lrlai v~in

Port1 tnd H~pti lt a ftH prol

Comm on) bll duct

Ouodtnum

~ft colic (sio)Atta~ hmtrlt jt~xJr-ofha~elSe

muo(IIQn

Right ~lIc (h~j)tic)

il~gtture

In1triol m~oten lIein (rttr op~ritoMdO

SlJp efl or mes~n~fiC amrV and lipln

KNOW YOUR NEIGHBORHOOD

Questions

vVhiJh structure supplied by a bnmdlof the cclia( artery is not derivcd from foregut LemCJUCrITI

(A) Head of the pancte-a5

CD) Pyloric duolenum

Cystkduct

( Liver hepatocyt~~

~F) Body of the spleen

An infant presents with an omrhaJucele at birth -hi oJ the [oHm illg applies to his cM1-dition

(A) It is 31so seen ill p4titnts with aganghonic megacolon

(11) ft reuirs from a fal1ure of resorption of theviteUine d let

(C) It results from herniation at the-site of regression of the right umbilk vein

DJ It is caustd by faihtrc of recanalization of the midgut part of the duodenum

~ It ill camioo by a failuIt vf the midgul to return to the abGQminal uity after herniashytion in-n the urnbilk s l stalk

Ot er than the spleen occlusion Cif the spit-Ilk artery at its odgin wm most likely affect die blood supply to jllch st cnud

(A) Jejunum

(B) Body of th pal1~lltas

(C) LeSStT Cllmiddotlaturc of tl )toma-ch

(D Duodenum dista to the entrance of the Ornmou bile duct

E Fundus of the stomach

A 38-yeu-old batL~er with a history of heartburn suddenly experiences excluciating pain in the (plgastric region of th~ abdomeu SurgCry is perf~rme immediard y upon admisshysion to the 1IlcrgCJliy tuomh~re i~ evidence uf a ruptured ulcer in the posterior waU of the stomach Vhere will a surgeon first fi nd the stomach contenlSf

A) Greater p4ritoneal sac

rB) Cul~de-s~c of Douglas (--

C Omental bursa ~

--D) Paracolic gutter

rEj Between -he panttal perimltum and the posterior body wal1

At birth an infant presents with a st()ma~ rb~tbas~njJled jfltotb~diaplfagru 1A1ltre is the defect thatresulied iiitJle heini~t()n shy~tsophagealbiatus

7 - rH-- Hiatus for the inferior vena cava

( Pleuroperitoneal membrane -(0) Septum transvcrsum

(E) Right Crlt~

An infant born with DOVv7l syndrome presents with bili()u~ vomiting Ahat congenital defect does the infant have

(A) Pyloric stenosis

(B) Meckel diverticulum C) Ornphaloce1e

(D) Gastroschisis

( ~ ) Duodenal atresia y A patient with cirrhosis of the liver presents with ~ bacalvaricestnlreased retrograde pressure in which veins caused the varices

(A) Paraumuilical

(B) Splenic

(ct AzygltJus

(15))G~trk ( (-F) Superior mesemeric

A htaltby 3-year~old male patient experiences a hernial sa protruding from the anterior abdominal wall about halfway between me anterior superior ilia spine and the pubk tuberde Pulsations of al1 artery are palpated medial to the protrusion site through the abdominal walL Which layer of the anterior abdominal wall will first be traversed by the

1hctma

fA) Rectus sheath (B) External oblique aponeurosis

(C) Inguinal ligament

lD) Transversalis fusda

(E) Cremasteric fa~cia

After 5urgi(aj ffpair of a hernia the patient tXperienccs mtmlgtness in the skin on the anteshyrior aspect of the S(Totum_ Vhaf nerve may have been lesioned during thehemiorrhaphy

(A) Femoral

(B) Obturator

(C) Ilioinguinal

(D) lliohypogastrk

(E) Pudendal

A 23~year-LJld female secretary il1 good health ~-uddcn1) doubles over with pain in the a ea of the 1JmbRicu$ Sbe feels vartn and ltneasy and has no appetite That night the pain seems to have mQved to the tower right abdominal regjol1 and she calls her family doctor who then arranges for an ambulance to pk-k her up and take her to the hospitaL Wh ell ntn~ perceived in the area of the urnbilirus most Hkely carried lhe pairfu I sensations into the eNS

tA) Vagus nerves I~

V B)

) Lessersplanchnk nerves

tC) Pudendal nerves

(D) lIiohpogastrk nerves

(E) Greater splam ic l erves

A CT reveals carcinoma in the bOod of the ancreas Vhich blood vessel trut ourses ----~- - -bull ------ --shy

immediately poftterior to the body ofthe pancreas is the m~t likely to be oompressed

(A) Splenk artery

(B) Abdominal aorta (C) Portal vein

(1) Splenic vein

(E) Renal vein

A patient has a penrln1l1ng uker of the posterior wall ot the br~l part ot the (lUooenmn llkh blood vessel is subject to erosion

(A) Common hepatic artery

(B) Gastroouodenal artery

(C) Proper hevatic artery

(D) Celiac artery

(E) Anterior inferior 11amrelltlcoduodcnal attery

Your patient has been diagnosed -ith a carcinoma locallted to the head and l~e(k of the pancreas Another clinical sign would be

A esophageal varices

(8) hemorrhoids

C) a caput medusa

(D) increased pra Teuro n th~ hepatic veins

(E) enlarged right supra lavkular lymph nodes

Wltkh of the foUowing structures develops in the ventral mesentery

(A) Spleen

(B) Jeiunum (C) Head of1ht pancreas (D) Transverse colon (E) Stomach

ti l Uw ~ littwin~ f( S-t lil oai Imdge ~ hi(h or tbt la~)d J truetur tgt liJ llntn nl) he hl p UC iJd [IIi ell

c o

A) drains Ie tht infCrior a La aI

R t middot~nfl0 ~ill to th~ lunlgtn of h i dtlndCrlllfH

(e) m t bull JiJattd on tl l J n T ~H

D ) sup Lc O VSlt I Hlid bhtu l 1 li - -I un oid

( ) U~tpli(t tr j middottUh~ 1 v(( b~nt rfK n1ilc~Zm

ANSWERS AND EXPLANATIONS

Answer E The spleen is t hlttnopodicand lymph organ demlted from mesoderm

Answ~ R Al1 tlmphalocele is caused by it failure of the nlidgut to return to the ahdomir nat cavity after herniation into the umbiliau Stalk Choices Aand D maybe seen in infants with Down syndrome choice D ~s the specific CBuse ofduudcnal JtiCSitt Choice C is (ile cause of gclstrosbisis and Choice B nsults iu a Meurolktldivertku1-tlB

Answer B The fundus ofthe stomach is suppHed by soort gastric brunches of the splenic altery The splenic artery supplies the body and tail of the pancreas part of the greater curvature of the sttmla(h and the spleen Te jejunum part of the head of the pancreas and tht~ duodenum distal to the entrance of the commOll bile duct are supplied by the superior mesenterk artery clll~l ~be less r ctlt1ature cmd the pylQric antrum are supplied by the right and lei gastric art(ries

AnSWftt C Tbeomental bursa or lesser ~ritoneaj sac lies direcdy posterior to the proxshyimal part of the duodeTtlm and the stomach and would be the first site where stomach contents ~Ott1d be fpoundluncL

Answer C A defect in a llleuropcritoneal membrane (uswlly the left) is the typical site of i1 cc-ngenitlI diilphragluatic hemia llere the membr4ne fails to dose ()pound( of the perishycCirdiopcritulleal canals

Answer E DuoJenal atresia and aganglionic megacoion are congwitaI defects S~Il in patients with Dowmiddotnsyndrome

Answer D RulaTgemt~llt of and retrograde flow in g~lstrk vel_ns in particlJl~r the kft gas~ tricveins dilates the capillary bed in rhe wall of the esophagus in (ases of porta yper~

tension Blood flow would increase in and dilampte tribntarkgts of the (lZygOUS vein on the other side of the capiUary bed but flow in this vein is in the typical direction t()ward the superior vena cava Paraumbiii(ltU vein eilgorgement contributes to a caput medusH Splenic ~nlargement might prc~nt with 5plcnonlegaly and balt-kflow in to tlu superior m~~ntclic vein occurs but is asymptomatic

Answer D The patient hagt an indirect inguinal hernia whi~h emerges from the antt-rior abdominal wall through the deep inguinltilling Theeep ring is a fault in the transv~rshysaUs fascia this I~yer wiIJ be penetrated first by the hernia

An~Wer C The ilioinguinal nenc which provides sens~llion to the lnedlal thigh ltmclanteshytior SClotunl pass~lt th rough the 5uperfh_ial inguinal ring ind $subject to inj i1T) becaus-e

it is in the operatitm Held of the erniorrhapny

Auswer B The leMHr splanchnic nerves are sympathdic nerVlts that carry viscera l sensashytlltgtrogt ftom illtllt1m~d ()J stietched gust (itinteitinal ~tructures (in this case the pprndix) into tnt eNS Lesser splanchnic ntTYcsarisc from thmiddot T9--T12 spinal cord segments lt1nd provide sympathetic innenation tD rnidgut siruc1ures whiCh include CLe app~JldD Viscera] Pain arising from affecLed Inidgut ampt 1C1ure is referred over the same dl- matorne~ of spinal segrnertts v-hich provide the sympathetic Innervation n this G1SC of appendicitis the invohen~n t of the ltire) of t e unlhHku indud s the T 10 dermatome

Answer B Of the five choices onty the dscending olon is retroperiton~al aldwould be a lik ~ ( choice to be seen immediately a(~jilcent to t11e posterior abdominal middotn~L

Amwen D The SpltftlC ~-ein ourses posterior to the body of the panneas m its way tt drain into the superior mCSfttltlri( vein

Answcr B TILt glstrodllolticnal artery 1 direct hIamh of the comrootl hepatic artery courses immediately pt))iwri() to the duodenum and is slbject to erosion

Answer B Carcinoma of th pan middott3S in the 1tilt1 may compreampgt the portltil vein at irs orishygill The poTtai vcin is fomled when the splenic vein jQiaswith tfie superior meStllt eric vein The inferiot mesenteric vein joins the ~plenjc vein just priOT to tlli~ point at which the splenic joins the superior Jlleit1ltcri( vein Increescd venous presslu in the inferior mesenteric vein is a cause of emo hoid~

Answer C The- velltral pancreas wilich forms most of the head of the p ~ncr as develops in the ventral mes(ntery as antutgrowth of the hepatic diverticulum Th~ hepatic divershyticulull induding the biIJary appa~atus develops in tbe ventral mesentery of the foregut

Answer~ A The superior mesenteric ~in joins with the spienkvein to form the hepatic portal vciu

Answer D The structure at gttlK is the proper hepatic artery~ whkh suppUesoxygenated b middotood to the liver

MAKE SURE YOU KNOW the diff bw Rectus Sheath above and below the arcuate line

ABOVE

Aponeurosis of xiiltmal obllque musclo

Extemll f)biquw musde

Reotln ilbdomlnls musole S~in

Internal 9bliquQ mY~QI

AponeUfOsi$ of hJH$V~~S Lir9a a lb lbdolTlin~ musolo Tri OJV6 rUi

atldomlnis mUS(loe

Sub cutanlilous tiue (tatty ye r)

BElOW

A POrl lJfosis 01 etemal oblique muscle

Aponeul~)sis 01 Internal oblique mU$cl~

Anteriol lay~ of r~ltdus st~ath EXttom1 oblique rnu$cll

Rectus Jbdominis muscle Intoernal Aponeurc-sis of tra~fersU$ oblique muscle-

at-domlnis muscentl ~ Skio

Tra nsvitSus abdomioLs ml)ZClt

TralSVersaHs fascia Medial umQil iegtt1 1i9Jment -and folj

Uldchus Peritoneum (ir median Umbilj~al Suboutane ous

Extraprftone 11ascia

Ymbilimiddot~1 fold)

preu9poundiea1 fascia

tissue (fatty 4nd m~mbr3n(iUS layers)

o Above the arcuate line (A horizontal line 13 of the distance bw the umbilicus and the

pubic symphysis) -10 Aponeurosis divides into an AntPost Laminae

o The Ant Laminae joins EO and Post Laminae joins Trans Abdominis = Ant and Post

RECTUS SHEATH respectively

o BElOW the arcuate line - all 3 aponeurosis join ANTERIOR to rectus muscle to meet its

counterpart in the midline (linea Alba)

o Take away Msg - The abdomen is devoid of a posterior rectus sheath below the

arcuate line and is therefore more vulnerable to herniasinjuries

Question - A physician makes a deep incision in the patients midline immediately superior to

the pubic symphysis which of the following layers is his knife least likely to pass

Rectus Abdominis External Oblique Ant Rectus Sheath Posterior Rectus Sheath All of the

Above

Answer - All of the above None of the other answer choices are midline structures -LINEA

ALBA

Linea Alba has very poor blood supply - doesnt heal well after surgery Therefore this is a

common site for incisional hernias

a Spleen b Transverse colon c Descending colon d Stomach e Pleura

17 Meckels diverticulum is normally found 2 feet proximal from the

a Pyloric sphincter b Lower esophageal sphincter c Ileo-cecal valve d Middle valve of Huston e Anal valve

18 Ulcer in the posterior wall of the first part of the duodenum would erode ___ artery and would cause bleeding

a Left gastric b Right gastric c Hepatic artery proper d Gastroduodenal artery e Middle colic artery

19 An inflamed appendix is identified by a surgeon on the operation table by noting

a The appendicies epiploicae b The convergence of tenia c The artery of Drummond d The mesocolon e The mesosalphinx

20 The nerve which emerges through the psoas major is

a Femoral b Ilio-inguinal c Ilio-hypogastric d Pudendal e Subcostal

21 The right gonadal vein drains into the

a Azygos b Hemiazygos c Inferior Vena Cava d Right renal vein e Left renal vein

22 The hepatocytes in the liver is derived from

a Ectoderm b Endoderm c Mesoderm

d Neural ectoderm

23 Abscess in the lumbar vertebrae due to tuberculosis would spread to the adjacent muscle which is

a Psoas Major b Iliacus c Quadratus lumborum d Tranversus Abdominis

24 The anterior wall of the inguinal canal is formed by

a External oblique and transverses abdominis b External oblique and fascia transversalis c Internal oblique and external oblique d Internal oblique and transverses abdominis e Fascia transversalis and peritoneum

Meckels diverticulum is a result of which of the following developmental abnormalities shy

A Failure of the vitelline duct to close

B Failure of the herniated intestinal loop to retract into the abdomen

C Failure of the urachus to close

D Failure of the midgut to rotate

E Failure of the hepatic duct to close

Explanation

Meckels diverticulum is a result of the persistence of the proximal part of the vitelline duct This

diverticulum is usually found about 2 feet proximal to the ileocecal junction and is usually about 2 inches

long It is present in about 2 of the popUlation It may be the site of ectopic pancreatic tissue or gastric

mucosa and may develop inflammatory processes and ulcerations Acute Meckels diverticulitis

simulates appendicitis

Which of the following veins carries blood from the esophagus to the portal vein The

A right gastric vein

B left gastric vein c splenic vein D azygos vein

E left gastroepiploic vein

Explanation

The left gastric vein a direct branch of the portal vein drains blood from the lesser curvature of the

stomach and the inferior portion of the esophagus Because branches of the portal vein do not have

valves blood can flow in a retrograde path when there is an obstruction to flow through the portal system or liveL Rlooci Cln then flow from the nortl] vein thr()1Ph the left PRstric vein to the esonhlPlIS lno

through venous communications within the submucosa of the esophagus to esophageal veins that drain

into the azygos vein The increase in blood flow through the esophageal submucosal veins results in esophageal varices

On the posterior wall of the abdomen the celiac ganglion A contains cell bodies of postganglionic parasympathetic neurons B is synapsed upon by neurons in the posterior vagal trunk C is synapsed upon by neurons in the greater splanchnic nerve D contains sensory cell bodies of lumbar spinal nerves E contains cell bodies of neurons that cause an increase in the rate of peristasis

Explanation The celiac ganglion is one of the preaortic ganglia of the sympathetic nervous system It contains cell bodies of postganglionic sympathetic neurons The sympathetic splanchnic nerves contain preganglionic sympathetic neurons that pass through the sympathetic chain without synapsing These splanchnic nerves go to the preaortic ganglia to synapse The greater splanchnic nerve contains preganglionic neurons from spinal cord segments T5-T9 This nerve synapses in the celiac ganglion The nerve fibers in the vagal trunks are preganglionic parasympathetic fibers that go to the walls of the organs that they will innervate and synapse on postganglionic parasympathetic neurons in the walls of those organs Cell bodies of sensory neurons in the abdomen are found in the dorsal root ganglia or the sensory ganglia of the vagus nerve Sympathetic innervation decreases the rate of peristalsis parasympathetic innervation increases the rate of peristalsis

Which of the following pairs of arteries will allow blood to bypass an occlusion of the celiac trunk

A Left gastric artery-right gastric artery

B Left gastroepiploic artery-right gastroepiploic artery

C Superior pancreaticoduodenal artery-inferior pancreaticoduodenal artery

D Splenic artery-common hepatic artery

E Left gastric artery - proper hepatic artery

Explanation The anastoOlosis of a branch of the celiac trunk and a branch of the superior mesenteric artery will

provide collateral circulation around an occlusion of the celiac trunk Each of the other choices pair

branches of the celiac trunk therefore these will not provide collateral flow around the obstruction of the

celiac trunk The left gastric splenic and common hepatic arteries are direct branches of the celiac trunk

The right gastric artery is a branch of the proper hepatic artery which is a branch of the common hepatic artery The left gastroepiploic artery is a branch of the splenic artery The right gastroepiploic artery is a

branch of the gastroduodenal artery whlch is a branch of the common hepatic artery

Which of the following organs has appendices epiploica The

A sigmoid colon

Bjejunum

C duodenum

D stomach E esophagus

Explanation Appendices epiploica are characteristic of the colon Appendices epiploica are subserosal accumulations

of fat None of the organs of the gastrointestinal tract has appendices epiploica except the colon

Page 23: Chirag's Abdomen Review

9 The renal angle is fonned lgtetween the 12th rib and ______ muscle

a Psoas major -middotshyb Erector spinae c Quadratus Iumborum d Diaphragm

10 The anterior structure at the hilum of the kidney is

a) Renal vein ~

b) Renal artery I middot~ I

c) Ureter d) Accessory renal artery

11 Because of origin of the muscle from the lateral one third of the inguinal ligament it

could not fonn the anterior wall of the inguinal ligament

a) External oblique b) Internal oblique c) Transversus abdominis_ d) Rectus abdominis

12 A large tumor mass impinges on the splenic artery and its branches as the artery pass out from below

the greater curvature of the stomach Branches o(which of the following arteries would most likely to

effected by the pressure on the splenic artery

a Left gastric b Left gastro-epipJoic c Right gastric d Right gastro-epipoloic e Short gastric_

13 A new born baby has projectile vomiting after each feeding It is determined that there is obstruction

of the digestive tract as a result of annular pancreas Annular pancreas is as a result of an abnormality in which of the following process

a Rotation of the dorsal pancreatic bud around the first part of duodenum b Rotation of the dorsal pancreatic bud around the second part of duodenum c Rotation of the dorsal pancreatic bud around the third part of duodenum d Rotation of the ventral pancreatic bud around the first part of duodenum y Rotation of the ventral pancreatic bud around the second part of duodenum

14 As the liver bud enters the ventral mesogastrium the region of the mesogastrium stretching from the

liver to the anterior abdominal wall is called

a Lesser Omentum b Greater Omentum ~ Falcifrom ligament d Lacunar ligament e Ligamentum teres of liver

16 A patient has absence of his 12th rib In such a patient if the doctor makes an incision to approach his

kidney mistaking the 11 th rib for the 12t he would end up injuring

Which of the following arteries is a direct branch of the gastroduodenal artery The

A right gastric artery

B left gastric artery

C inferior pancreaticoduodenal artery D left gastroepiploic artery

i E)right gastroepiploic artery --

E x pI a nation The right gastric artery is typically a branch of the proper hepatic artery The left gastric artery is a direct

branch of the celiac trunk The right and left gastric arteries anastomose along the lesser curvature of the

stomach The inferior pancreaticoduodenal artery is a branch of the superior mesenteric artery it

anastomoses with the superior pancreaticoduodenal in the head of the pancreas The left gastroepiploic

artery is a branch of the splenic artery it anastomoses with the right gastroepiploic artery along the greater

curvature of the stomach The right gastroepiploic artery is a branch of the gastroduodenal artery The

other branch of the gastroduodenal artery is the superior pancreaticoduodenal artery

Which of the following pairs of veins join together to form the portal vein The

A superior mesenteric vein and inferior mesenteric vein

B inferior mesenteric vein and splenic vein

C superior mesenteric vein and splenic vein

Ip)splenic vein and left gastric vein E superior mesenteric vein and left gastric vein

Explanation

The portal vein is formed behind the neck of the pancreas by the union of the superior mesenteric vein

and the splenic vein The inferior mesenteric vein drains into the splenic vein The left gastric vein drains

directly into the portal vein After the portal vein forms it enters the hepatoduodenalligament of the

lesser omentum to reach the liver The portal vein is the most posterior structure in the hepatoduodenal

ligament

At which of the following vertebral levels does the duodenum pass anterior to the aorta - _- shy

All ~

B L2 7~

CL3 I

~DL4

E L5

Explanation

The duodenum begins at the pyloric sphincter at the level of Ll The second (or descending) portion of

the duodenum is to the right of the aorta and extends inferiorly from the level of Ll to the level of L3 The third part of the duodenum crosses the aorta from the right side to the left side at the level of L3 The

fourth (ascending) portion of the duodenum extends from the level of LJ to the level of L2 The

duodenum ends at the duodenojejunal flexure The superior mesenteric artery passes anterior to the

duodenum as the duodenum passes anterior to the aorta The duodenum can be constricted at this level

In which of the following locations will perforation of the digestive tract result in the spilling of luminal

contents into the - lesser peritoneal sac

A Anterior wall of the second portion of the duodenum B Posterior wall of the second portion of the duodenum

C Anterior wall of the stomach

~Posterior wall of the stomach E Posterior wall of the transverse colon

Explanation

The posterior wall of the stomach is related to the lesser peritoneal sac The anterior wall of the stomach is related to the greater peritoneal sac The anterior wall of the second portion of the duodenum is related to the greater peritoneal sac The posterior wall of the second portion of the duodenum is related to the retroperitoneal space The posterior wall of the transverse colon is related to the greater peritoneal sac

The ureter lies against the anterior surface of which of the following muscles shyA Crus oftne diaphragm B Quadratus lumborum

0 Psoas major D Transversus abdominis

E Iliacus

Explanation The ureter exits the renal pelvis at about the level of vertebra L2 As it descends along the posterior abdominal wall it lies on the anterior surface of the psoas major The psoas major muscle arises from the bodies of the lower lumbar vertebrae The psoas major muscle is joined by the iliacus to fonn the

iliopsoas muscle The iliopsoas muscle then attaches to the lesser trochanter of the femur and is the major

flexor of the hip

As the right ureter passes the pelvic brim it lies against the anterior surface of which of the following

blood vessels

A Gonadal artery B Inferiorvena cava C Internal iliac artery

rJ- External Iliac artery

E Inferior mesenteric artery

Explanation

The ureter lies in the extraperitoneal space in the posterior abdominal wall Alter leaving the kidney it

passes inferiorly on the anterior surface of the psoas major muscle At the pelvic brim the ureter passes

into the pelvis At this point the common iliac artery is dividing into the external and iliac arteries The

ureter lies on the anterior surface of the external iliac artery immediately distal to the bifurcation This is a useful landmark for a surgeon to locate the ureter

When extravasated urine passes from the superficial perineal space into the anterior abdominal wall it is

found immediately deep to which of the following layers of the anterior abdominal wall

-ltScarpas fascia

B External oblique muscle

C Internal oblique muscle D Transversus abdominis muscle

E Transversalis fascia

Explanation

The superficial perineal space is bound by Colles fascia the fibrous portion of the superficial fascia This

layer of fascia is continuous with Scarpas fascia the fibrous portion of the superficial fascia of the anterior abdominal wall Therefore urine that is deep to Colles fascia will remain deep to Scarpa s fascia The urine will spread in the plane between Scarpas fascia and the external oblique layer

When a horseshoe kidney develops the ascent of the kidney is restricted by the A internal iliac artery B external Iliac artery

C common iliac artery

inferior mesenteric artery

E superior mesenteric artery

Explanation

A horseshoe kidney develops when the inferior poles of the to kidneys fuse together as they ascend into

the abdomen from the pelvis The first anterior midline vessel that is encountered by the horseshoe kidney

is the inferior mesenteric artery This artery prevents the kidney from continuing its ascent

The left testicular vein drains into which of the following veins

A Left internal iliac vein B Left common iliac vein

bflnferior vena cava D Left renal vein I

E Left internal pudendal vein

Explanation

The left testicular vein drains into the left renal vein The right testicular ~i~[~nsltjectlY into the

inferior vena cava This difference in venous drainage is believed to explain the greater incidence of

varicocele on the left side than on the right The venous drainage from the penis is to the internal vein

which then drains into the internal Iliac vein

The spinal nerve that provides cutaneous branches to the skin around the umbilicus is

A TS B TW-shy

C TI2

DL2 EtA

Explanation

The tenth intercostal nerve is the anterior ramus of the TIO spinal nerve After passing through the tenth

intercostal space the nerve continues forward in the anterolateral abdominal wall in the plane between

the internal oblique muscle and the transversus abdominis muscle In the abdominal wall the nerve innervates to the abdominal wall muscles as well as the skin and the parietal peritoneum The umbilicus is

a useful landmark for the region of distribution of the tenth thoracic nerve

The ligament of the vertebral column that resists its extension is the Aligamentum flavum

B supraspinous ligament

C posterior longitudinal ligament

D anterior longitudinal ligament

E interspinous ligament

Explanation

The ligaments of the vertebral column that resist flexion of the column include the supraspinous ligament

interspinous ligament ligamentum fiavum and posterior longitudinal ligament The ligament that resists

extension is the anterior longitudinal ligament This longitudinal ligament is very broad and strong It

covers the anterior and anterolateral surfaces of the vertebral bodies and the intervertebral disks In

addition to resisting extension the anterior longitudinal ligament provides reinforcement to the anterior

and anterolateral surfaces of the intervertebral disk The posterior longitudinal ligament is relatively

narrow and covers the posterior surface of the vertebral bodies and the intervertebral disks This ligament

reinforces the posterior surface of the disk The posterolateral surface of the disk is not reinforced and it

is through this region that herniation of the nucleus pulposus usually occurs

A patient presents with epigastric and right upper quadrant pain The pain is most intense 2-4 hours after

eating and is reduced by the ingestion of antacids The patient states that he has passed black tarry stools

(melena) within the last week Fiberoptic endoscopy reveals a yellowish crater surrounded by a rim of

erythema that is 3 cm distal to the pylorus Accordingly an ulcer has been identified in the patients

A fundus

B antrum

C duodenum

D jejunum

E ileum

A number of physiologic genetic and other factors increase the risk of gastric (and duodenal) peptic

ulcers The evidence that H pylori plays a principle role is compelling Smoking and caffeine are known to adversely affect the morbidity mortality and healing rates of peptic ulcers In general first-degree

relatives of peptic ulcer patients as well as males have a threefold to fourfold increased risk of developing this disorder Paradoxically in gastric ulcer disease acid secretion is not elevated It is possible that

excess secreted hydrogen ion is reabsorbed across the injured gastric mucosa In general a defect in gastric mucosal defense is the more important local physiologic

A patient presents with symptoms of duodenal obstruction caused by an annular pancreas Annular pancreas is caused by

A rotation of the dorsal pancreatic bud into the ventral mesentery B rotation of the ventral pancreatic bud into the dorsal mesentery

fJ failure of the major and minor pancreatic ducts to fuse ~ ~ cleavage of the ventral pancreatic bud and rotation of the two portions in opposite directions around -the duodenum E formation of one pancreatic bud instead of two

Explanation Normally the ventral pancreatic bud rotates around the gut tube to reach the dorsal pancreatic bud The two buds fuse to form a single pancreas and the distal portions of the two ducts fuse The ventral pancreatic bud forms the inferior portion of the head of the pancreas the uncinate process and the major pancreatic duct (of Wirsung) The dorsal pancreatic bud forms the superior part of the head the neck body and tail and the minor pancreatic duct (of Santorini) Annular pancreas is the result of the ventral pancreatic bud dividing into two portions before it rotates into the dorsal mesentery Each portion rotates in opposite directions to get to the dorsal mesentery thus encircling the duodenum The presence of annular pancreas can constrict the duodenum thus obstructing its lumen

In n _ phranlc----

Gon ~l ----_1 Lum bltano

~~--- CornmQ1t bull ac

+-~4--- lnlllirnaJ ilic

xtem iliac

OBJECTIVE - Identify the blood supply to each of the structures listed in the table on the previous page

Ill give you a head start

FOREGUT - Supplied bV Celiac Tru nk (T12)

Proper hepatic

GastiooUod 13Jafter

1nferlor pancreaticoduodenal artery

Common epatlc

Lett gas ric iiirtery

Spfen artery

shy Gastroepiphgtic artery

~ Superior mesenteric 8rtfry

~

1 Esophagus is a derivative of the foregut so its blood supply originates from the celiac trunk

(T12) The predominant blood supply to abdominal portion of the esophagus is the Esophageal

A (Branch of L Gastric) The venous drainage of the esophagus is particularly important because

it is 1 of 3 clinically relevant sites of Portal Caval anastamoses The Portal Esophageal Vein

meets the Caval Azygos System Persistent bleeding manifests as Esophageal Varices - a fata I

condition

2 The Stomach is also a derivative of the foregut has EXTENSIVE blood supply and is very high

yield on anatomy exams The lesser curvature is supplied superiorly by the L Gastric A (1 of 3

major branches ofthe Celiac trunk) and inferiorly by the R Gastric A ( a branch ofthe proper

Hepatic A) The greater curvature is supplied superiorly by the L Gastroepiploic A (a major

branch of the splenic A) and inferiorly by the R Gastroepiploic A

The Short Gastric arteries (branches of Splenic Artery) supply the fundus of the stomach and

are referred to as EIID ARTERIES because they have no collateral blood supply Therefore if the

splenic artery were occluded (ex - increased pressure in the ommental bursa) - there would be

ischemia to the fundus of the stomach Venous drainage of the stomach is extensive via various

veins lead ing to the portal system Posterior to the stomach the IMV joins the splenic V which

joins the SMV to form the PORTAL VEIN ADAMS

3 Duodenum blood supply has high clinical relevance because it is the junction of the foregut and

midgut and therefore is the site of anastamoses between branches ofthe Celiac Trunk (main

foregut artery) and the Superior Messenteric Artery (main midgut artery) The Proper hepatic

artery gives off the gastroduodenal artery which travels behind the 1st part of the duodenum

This point has high clin ical relevance because duodenal ulcers are very common and a posterior

rupture of the 1st part of the duodenum could rupture the gastroduodenal artery causing

traumatic abdominal bleeding The Gastroduodenal artery first gives off the R Gastroepiploic A

(mentioned above) and proceeds as the Superior pancreatico duodenal artery (supplies the

pancreas and duodenum) which anastamoses with the inferior pancreatico duodenal A (branch

of the SMA) This is the junction of foregut and midgut and occurs near the opening of the

bil iary system into the duodenum (ampula of vater) Portal venous drainage here is responsible

for delivering nutrients from digestion to the liver for metabolism Appreciate that the Superior

mesenteric artery (artery of the midgut) branches from the aorta at Ll travels posterior to the

pancreas than moves anteriorly (at the jxn of the pancreatic headbody) and comes over the

3rd4th part of the duodenum Tumor of the head of the pancreas can compress the SMA

4 Jiver blood supply is via the common hepatic artery (major branch of the cel iac trunk) The

common hepatiC becomes the proper hepatic gives off the R gastric A and the Gastroduodenal

A and then joins the common bile duct and the portal vein in the portal triad Clinical- if a

patient were bleeding from the hepatic A a surgeon can stick his fingers in the epiplOic foramen

and squeeze the free edge of the hepatoduodenalligament in order to stop bleeding to the

area Please note that the hepatic a branches into Rand L hepatic A The Right hepatic artery

gives off the cystic artery which supplies the gallbladder Afferent venous supply is via the

Portal vein which is bringing nutrient rich blood to the liver After metabolism takes place

venous blood leaves the liver through the hepatic veins into the IVC PLEASE UNDERSTAND THE

RELATIONSHIP OF THESE STRUCTURES - ADAMSNETIERSNH Etc

5 Pancreas - Head is supplied via the superior and inferior pancreaticoduodenal arteries

(mentioned above) The tail (situated towards the hilum of the spleen) is supplied via the

pancreatic branches of the splenic artery (END ARTERIES) This blood supply is very important

because the endocrine Alpha and Beta Cells from the pancreatic islets of lagerhans are located

towards the tail This is where Insulin and Glucagon is released to the blood

Now complete this for mid and hindgut structures Make sure to note clinically relevant arterial

anastomoses as well as portal caval anastomoses FYI Appendix blood supply SMA + IMA

anastamoses marginal artery Portalcaval rectal veins fhemmorhoids) and periumbilical caput

medusa are high yield THE BUTT THE GUT and THE CAPUT

Abdominal Development

Liver

Ij1f

II wall b

oh liN ~ VltJrti n be- bull

Pancreas

Secondary Retroperitonealization e I~tl r 1 a v-mtrai m ellter

Rotations of the Gut I i Ij (lIl1UtIJ f~ l r tilt

()l td 10 me l-ft and he v

--~--- -~ -~-~

i

I AolaijonjoI~guf I

STOMACH BED (IDENTIFY IN ADAMS)- the structures posterior to the ommental bursa which

support the stomach in the supine position

Abdomnal JQrUI

Splnic vein

OmQ-oul tv~ ) O(s(Jroa)

Lojt(r o m nturrt (hpJtodu o d~n31 Hid

Gadrl)SplerH (g3stroll~nal) IIgam~nt

hiad h~~atogrtricent IIQdmiddotcrt~)

Lt Dome of Diaphragm (why left Look this up in Adams)

Spleen (What is the blood supply)

Left Kidney (What is the blood supply - AND how is it different from the R kidney)

Suprarenal Gland (What is the Arterial AND Venous Blood supply - how are they different)

Pancreas (How does supply differ from Head to Tail What is the SMA Relationship)

Transverse Mesocolon

liver - ADAMSWET - Make sure you look at the liver in wet lab

Left triangular nl1am~nt

ComoaDj ligamnt

Erophg~1 impre$ioo

Hepatio veins

In1erior -ifena middotr3)Ia

Fibrous appendix o-t

live

impr~j on

Heprorendl p~rtion of Q)(Qllary ligament

Righllri~n9ul r 1I~met

(Common) bile quol

Gr)mmCtr~ hepatic dlJct

Ccentic duct

Duodenal impression

GaJdate p-fr)~S

Hepatic artgtrl prop-f iiiiila - Faloiform ligament

_ - shy Round ligamen liver

~--F-- CoJio imprgt-ssi-on

Prta heptis

Identify the lobes impressions and embryonic remnants associated with the liver

Caudate Lobe Quadrate Lobe Right Lobe Left Lobe Round ligament Falciform Ligament

Ligamentum Venosum (what is its fxn in embryonic life) Hepatic Veins (NOT PART OF THE

PORTAL TRIAD) IVC PORTAL TRIAD - Contents relationship cross section etc Know the

Galbladder relationship to the lobes of the liver

Biliary Duct System - Make sure you understand the sequence of these structures - BE ABLE TO

DRAW A FLOW CHART

TPVd i

t

I t

1 __ Cm-(r

patk GlJet

I

J

Clinical = JAUNDICE is caused by anything that prevents delivery of bile to intestine Tumor of the

head of the pancreas Stones etc Patient will have pale stools and yellowish colored mucus

membranes

Clinical- Any scenario that tells you the patient has BILLOUS VOMIT means that the obstruction to

the flow of digestive contents is after the Ampulla of Vater (Site of Entry of Billiary system to the

duodenum) - ie Duodenal Atresia

Spleen -located posterior to the mid axillary line between ribs 9 and 11 Make sure you know that

the 10th rib is the main axis of the spleen and this organ is susceptible to injury (stab wound errant

thoracoce ntesis etc)

The spleen is derived from mesodermal cells - NOT THE GUT TUBE

The spleen rests on the left colic flexure associates with the tail of the pancreas Know the

structures entering the Hilum of the spleen

Sh rt O~-t~ic 1 0(0 10 rtiltSPIric Iloa nt

(cut)

Peritoneum - similar concept to Pleura - think of a fist in a balloon

Visceral Peritoneum - Layer of balloon touching your fist

Parietal Peritoneum - Layer of balloon not touching your fist

Your fist represents the organ your wrist is the hilum and your arm contains the blood supply

entering the organ

Appreciate that there will never be organs in the peritoneal cavity - rather these organs invaginate

the cavity Kaplan videos

RULES OF NOMENCLATUREshy

1 Organ completely surrounded by peritoneum - peritoneal organ

2 Organ partially surrounded by peritoneum- Retroperitoneal

3 Peritoneum surrounding peritoneal organ is VISCERAL peritoneum

4 Peritoneum surrounding retroperitoneal organ is PARIETAL peritoneum

5 Peritoneum connecting visceral to parietal is called messentary 2 messentaries in the

gut Dorsal (to the gut tube) and ventral (to the gut tube) messentary

Aorta is in Retro peritoneal position - but blood must reach peritoneal position - vessels travel through

messentary All peritoneal organs will have blood supply reaching through messentary

-Mesentery is a 2 layer peritoneum with a neurovascular communication between body wall and organ

- Ligament connects one organ with another or to the abdominal wall (Ommentum = ligament)

lesser Ommentum (attach lesser curvature of stomach and duodenum to liver) =Hepatoduodenal

Ligament and Hepatogastric Ligament

Has a Superior and Inferior Recess (Accumulation of Fluid in Ascites)

Communicates with the greater sac through the epiplic foramen (what structures pass through

this foramen)

Boundaries - you must be able to visualize this

o Anterior - stomach

o Posterior - parietal peritoneum pancreas

o Superior - superior recess (bw diaphragm and coronary ligament)

o Inferior -Inferior recess (bw layers or greater momentum

Greater Ommentum (attach greater curvature of stomach) Gastrophrenic ligament Gastrosplenic

ligament gastrocolic ligament

The greater omentum is the largest peritoneal fold It consists of a double sheet of peritoneum folded on itself so that it is made up of four layers The two layers which descend from the greater curvature of the stomach and commencement of the duodenum pass in front of the small intestines sometimes as low down as the pelvis they then turn upon themselves and ascend again as far as the transverse colon where they separate and enclose that part of the intestine

ABDOMINAL PAIN

Parietal Peritoneum - supplied by same vasculature lymphatics and nerves supplying body wall it

lines and diaphragm Sensitive to pain pressure heat cold well localized

Visceral Peritoneum - supplied by same vasculature lymphatics and somatic nerve of organ it covers

Insensitive to touch heat cold and laceration - referred to dermatome of spinal ganglia providing

sensory fibers Where does appendicitis refer to

Foregut pain - epigastric area (ie - cholycystitis)

Midgut pain - periumbilical area (ie - appendicitis)

Hindgut Pain - suprapubic area (ie - diverticulitis)

Extra ImagesConcepts

ll~_____-

FalifCtrm ligament oind r~ud ligamet f Ilver

Blood from splenio gastriC and inferiof rne$e-rteri v~ins

Ca-I tributaries

Lett gastrio Ifein

Posterior superior pan~reatioodul)denal vaihS

Lott gamo-om~nlal (9aropip lomiddotic) -in

Poq_~ tjol imerl-9-r panCJertlcorllJod-nal veiopound --amp----I- - ~J Right grtr~-omntal

Anwrior interi (gartroepiploic) Jjn

pan euaii cod vl)denal veins middot Inf~Ji (t r mesentric vein

Miqdle (olic vein

Right cl)licvein Sigmoid and rectosigml)id (ei ns

IhH)Collc(~io

--- Mi~dl laquooLJl gtjrltgt

PoM ca vl1 illasto)moses -----shyampoptoageal 2 Paraumbilie-lt11 Inferi or Fectal vei ns

3 Recial 4 REuoperHonea1

Know how the Portal vein is formed I 4 sites of portal caval anastamoses and 1 clinical shunt

Col li t ltt-~ otTl~tI ~nj pc~ 1lt1 turJoG

Ltf 14i1 tImiddot~ artoftl9 on tj phtAt$

L-oftqf 4t t~r 1=laquoIran d 1 bull shy~p l ci rj o fOOOts

Nerves follow the arteries - appreciate the splanchnic nervous system I

Uet~ric branch of left ~nal art

Ureterie branch of righi renal artelY

Left Zld lumbar in and co mlTlunication to as)erdin9 lumbar l(~in Hi ~ht tEZ1~~t~ t3r j t itn ~ nJ l1t- rlnd lfe i r1

Inferior me5nteri~ artery

Notice that the right testicular vein drains directly into the IVC and the right testicular artery drains

directly into the aorta However the left testicular vein drains into the L renal vein at a right angleshy

reason left testicle is lower and more susceptible to varicocele (bag of worms)

Also notice that the left renal vein has a longer course because the IVC is on the right side whereas

the right renal artery has a longer course because the aorta is on the left side

Appreciate the anterior to posterior relationship of structures in the hilum of the kidney - VAP - Vein

Artery Renal Pelvis (Ureter)

11____ __ L_ L_ n VJ __ _ _ t_L I I_ _ L __ L_ I -pound1 bull LI_~-I ____

Posterior View of Head of Pancreas in ( of Duodenum

Celiao hunk

Co mmon ~L~jJth art~ry

GastNduQdonal artrf (partilly in phantn)

P1)Sterior $Up~Jior panCflaticuduodfmal art~r~t

(Co mm on) bile duct

middot~1t~~t-1l---~-~- Right gshomiddotomental (gastoe plp lolc) 3rte (phantomost)

Grener paocre atic art-ry

1n1~rjor pancr-iatlc artery

Jtrifll supejo r pal)oreailcento)dJodenal artr1 (phantom)

Anastomotlo branch

POostetlor bJanch of jo f~ri of pan-reatir(lduodensl drttnj

Anterio r branch of i flferior palcreati~)duodenal art~(phan1om)

Notice the extensive blood supply to the pancreas and duodenum via the branches of the celiac trunk

Notice collateral supply from SMA branches - makes sense bc this is the jxn of foregutmidgut

Identify the vessels in this arteriogram

Hiltid i)f N~ck oi B)dvof Tail 01 pa nereas pan cent~as P-nmiddot-reas panCtCas

I nferie v~na cava

jHept1iic p(lrlai v~in

Port1 tnd H~pti lt a ftH prol

Comm on) bll duct

Ouodtnum

~ft colic (sio)Atta~ hmtrlt jt~xJr-ofha~elSe

muo(IIQn

Right ~lIc (h~j)tic)

il~gtture

In1triol m~oten lIein (rttr op~ritoMdO

SlJp efl or mes~n~fiC amrV and lipln

KNOW YOUR NEIGHBORHOOD

Questions

vVhiJh structure supplied by a bnmdlof the cclia( artery is not derivcd from foregut LemCJUCrITI

(A) Head of the pancte-a5

CD) Pyloric duolenum

Cystkduct

( Liver hepatocyt~~

~F) Body of the spleen

An infant presents with an omrhaJucele at birth -hi oJ the [oHm illg applies to his cM1-dition

(A) It is 31so seen ill p4titnts with aganghonic megacolon

(11) ft reuirs from a fal1ure of resorption of theviteUine d let

(C) It results from herniation at the-site of regression of the right umbilk vein

DJ It is caustd by faihtrc of recanalization of the midgut part of the duodenum

~ It ill camioo by a failuIt vf the midgul to return to the abGQminal uity after herniashytion in-n the urnbilk s l stalk

Ot er than the spleen occlusion Cif the spit-Ilk artery at its odgin wm most likely affect die blood supply to jllch st cnud

(A) Jejunum

(B) Body of th pal1~lltas

(C) LeSStT Cllmiddotlaturc of tl )toma-ch

(D Duodenum dista to the entrance of the Ornmou bile duct

E Fundus of the stomach

A 38-yeu-old batL~er with a history of heartburn suddenly experiences excluciating pain in the (plgastric region of th~ abdomeu SurgCry is perf~rme immediard y upon admisshysion to the 1IlcrgCJliy tuomh~re i~ evidence uf a ruptured ulcer in the posterior waU of the stomach Vhere will a surgeon first fi nd the stomach contenlSf

A) Greater p4ritoneal sac

rB) Cul~de-s~c of Douglas (--

C Omental bursa ~

--D) Paracolic gutter

rEj Between -he panttal perimltum and the posterior body wal1

At birth an infant presents with a st()ma~ rb~tbas~njJled jfltotb~diaplfagru 1A1ltre is the defect thatresulied iiitJle heini~t()n shy~tsophagealbiatus

7 - rH-- Hiatus for the inferior vena cava

( Pleuroperitoneal membrane -(0) Septum transvcrsum

(E) Right Crlt~

An infant born with DOVv7l syndrome presents with bili()u~ vomiting Ahat congenital defect does the infant have

(A) Pyloric stenosis

(B) Meckel diverticulum C) Ornphaloce1e

(D) Gastroschisis

( ~ ) Duodenal atresia y A patient with cirrhosis of the liver presents with ~ bacalvaricestnlreased retrograde pressure in which veins caused the varices

(A) Paraumuilical

(B) Splenic

(ct AzygltJus

(15))G~trk ( (-F) Superior mesemeric

A htaltby 3-year~old male patient experiences a hernial sa protruding from the anterior abdominal wall about halfway between me anterior superior ilia spine and the pubk tuberde Pulsations of al1 artery are palpated medial to the protrusion site through the abdominal walL Which layer of the anterior abdominal wall will first be traversed by the

1hctma

fA) Rectus sheath (B) External oblique aponeurosis

(C) Inguinal ligament

lD) Transversalis fusda

(E) Cremasteric fa~cia

After 5urgi(aj ffpair of a hernia the patient tXperienccs mtmlgtness in the skin on the anteshyrior aspect of the S(Totum_ Vhaf nerve may have been lesioned during thehemiorrhaphy

(A) Femoral

(B) Obturator

(C) Ilioinguinal

(D) lliohypogastrk

(E) Pudendal

A 23~year-LJld female secretary il1 good health ~-uddcn1) doubles over with pain in the a ea of the 1JmbRicu$ Sbe feels vartn and ltneasy and has no appetite That night the pain seems to have mQved to the tower right abdominal regjol1 and she calls her family doctor who then arranges for an ambulance to pk-k her up and take her to the hospitaL Wh ell ntn~ perceived in the area of the urnbilirus most Hkely carried lhe pairfu I sensations into the eNS

tA) Vagus nerves I~

V B)

) Lessersplanchnk nerves

tC) Pudendal nerves

(D) lIiohpogastrk nerves

(E) Greater splam ic l erves

A CT reveals carcinoma in the bOod of the ancreas Vhich blood vessel trut ourses ----~- - -bull ------ --shy

immediately poftterior to the body ofthe pancreas is the m~t likely to be oompressed

(A) Splenk artery

(B) Abdominal aorta (C) Portal vein

(1) Splenic vein

(E) Renal vein

A patient has a penrln1l1ng uker of the posterior wall ot the br~l part ot the (lUooenmn llkh blood vessel is subject to erosion

(A) Common hepatic artery

(B) Gastroouodenal artery

(C) Proper hevatic artery

(D) Celiac artery

(E) Anterior inferior 11amrelltlcoduodcnal attery

Your patient has been diagnosed -ith a carcinoma locallted to the head and l~e(k of the pancreas Another clinical sign would be

A esophageal varices

(8) hemorrhoids

C) a caput medusa

(D) increased pra Teuro n th~ hepatic veins

(E) enlarged right supra lavkular lymph nodes

Wltkh of the foUowing structures develops in the ventral mesentery

(A) Spleen

(B) Jeiunum (C) Head of1ht pancreas (D) Transverse colon (E) Stomach

ti l Uw ~ littwin~ f( S-t lil oai Imdge ~ hi(h or tbt la~)d J truetur tgt liJ llntn nl) he hl p UC iJd [IIi ell

c o

A) drains Ie tht infCrior a La aI

R t middot~nfl0 ~ill to th~ lunlgtn of h i dtlndCrlllfH

(e) m t bull JiJattd on tl l J n T ~H

D ) sup Lc O VSlt I Hlid bhtu l 1 li - -I un oid

( ) U~tpli(t tr j middottUh~ 1 v(( b~nt rfK n1ilc~Zm

ANSWERS AND EXPLANATIONS

Answer E The spleen is t hlttnopodicand lymph organ demlted from mesoderm

Answ~ R Al1 tlmphalocele is caused by it failure of the nlidgut to return to the ahdomir nat cavity after herniation into the umbiliau Stalk Choices Aand D maybe seen in infants with Down syndrome choice D ~s the specific CBuse ofduudcnal JtiCSitt Choice C is (ile cause of gclstrosbisis and Choice B nsults iu a Meurolktldivertku1-tlB

Answer B The fundus ofthe stomach is suppHed by soort gastric brunches of the splenic altery The splenic artery supplies the body and tail of the pancreas part of the greater curvature of the sttmla(h and the spleen Te jejunum part of the head of the pancreas and tht~ duodenum distal to the entrance of the commOll bile duct are supplied by the superior mesenterk artery clll~l ~be less r ctlt1ature cmd the pylQric antrum are supplied by the right and lei gastric art(ries

AnSWftt C Tbeomental bursa or lesser ~ritoneaj sac lies direcdy posterior to the proxshyimal part of the duodeTtlm and the stomach and would be the first site where stomach contents ~Ott1d be fpoundluncL

Answer C A defect in a llleuropcritoneal membrane (uswlly the left) is the typical site of i1 cc-ngenitlI diilphragluatic hemia llere the membr4ne fails to dose ()pound( of the perishycCirdiopcritulleal canals

Answer E DuoJenal atresia and aganglionic megacoion are congwitaI defects S~Il in patients with Dowmiddotnsyndrome

Answer D RulaTgemt~llt of and retrograde flow in g~lstrk vel_ns in particlJl~r the kft gas~ tricveins dilates the capillary bed in rhe wall of the esophagus in (ases of porta yper~

tension Blood flow would increase in and dilampte tribntarkgts of the (lZygOUS vein on the other side of the capiUary bed but flow in this vein is in the typical direction t()ward the superior vena cava Paraumbiii(ltU vein eilgorgement contributes to a caput medusH Splenic ~nlargement might prc~nt with 5plcnonlegaly and balt-kflow in to tlu superior m~~ntclic vein occurs but is asymptomatic

Answer D The patient hagt an indirect inguinal hernia whi~h emerges from the antt-rior abdominal wall through the deep inguinltilling Theeep ring is a fault in the transv~rshysaUs fascia this I~yer wiIJ be penetrated first by the hernia

An~Wer C The ilioinguinal nenc which provides sens~llion to the lnedlal thigh ltmclanteshytior SClotunl pass~lt th rough the 5uperfh_ial inguinal ring ind $subject to inj i1T) becaus-e

it is in the operatitm Held of the erniorrhapny

Auswer B The leMHr splanchnic nerves are sympathdic nerVlts that carry viscera l sensashytlltgtrogt ftom illtllt1m~d ()J stietched gust (itinteitinal ~tructures (in this case the pprndix) into tnt eNS Lesser splanchnic ntTYcsarisc from thmiddot T9--T12 spinal cord segments lt1nd provide sympathetic innenation tD rnidgut siruc1ures whiCh include CLe app~JldD Viscera] Pain arising from affecLed Inidgut ampt 1C1ure is referred over the same dl- matorne~ of spinal segrnertts v-hich provide the sympathetic Innervation n this G1SC of appendicitis the invohen~n t of the ltire) of t e unlhHku indud s the T 10 dermatome

Answer B Of the five choices onty the dscending olon is retroperiton~al aldwould be a lik ~ ( choice to be seen immediately a(~jilcent to t11e posterior abdominal middotn~L

Amwen D The SpltftlC ~-ein ourses posterior to the body of the panneas m its way tt drain into the superior mCSfttltlri( vein

Answcr B TILt glstrodllolticnal artery 1 direct hIamh of the comrootl hepatic artery courses immediately pt))iwri() to the duodenum and is slbject to erosion

Answer B Carcinoma of th pan middott3S in the 1tilt1 may compreampgt the portltil vein at irs orishygill The poTtai vcin is fomled when the splenic vein jQiaswith tfie superior meStllt eric vein The inferiot mesenteric vein joins the ~plenjc vein just priOT to tlli~ point at which the splenic joins the superior Jlleit1ltcri( vein Increescd venous presslu in the inferior mesenteric vein is a cause of emo hoid~

Answer C The- velltral pancreas wilich forms most of the head of the p ~ncr as develops in the ventral mes(ntery as antutgrowth of the hepatic diverticulum Th~ hepatic divershyticulull induding the biIJary appa~atus develops in tbe ventral mesentery of the foregut

Answer~ A The superior mesenteric ~in joins with the spienkvein to form the hepatic portal vciu

Answer D The structure at gttlK is the proper hepatic artery~ whkh suppUesoxygenated b middotood to the liver

MAKE SURE YOU KNOW the diff bw Rectus Sheath above and below the arcuate line

ABOVE

Aponeurosis of xiiltmal obllque musclo

Extemll f)biquw musde

Reotln ilbdomlnls musole S~in

Internal 9bliquQ mY~QI

AponeUfOsi$ of hJH$V~~S Lir9a a lb lbdolTlin~ musolo Tri OJV6 rUi

atldomlnis mUS(loe

Sub cutanlilous tiue (tatty ye r)

BElOW

A POrl lJfosis 01 etemal oblique muscle

Aponeul~)sis 01 Internal oblique mU$cl~

Anteriol lay~ of r~ltdus st~ath EXttom1 oblique rnu$cll

Rectus Jbdominis muscle Intoernal Aponeurc-sis of tra~fersU$ oblique muscle-

at-domlnis muscentl ~ Skio

Tra nsvitSus abdomioLs ml)ZClt

TralSVersaHs fascia Medial umQil iegtt1 1i9Jment -and folj

Uldchus Peritoneum (ir median Umbilj~al Suboutane ous

Extraprftone 11ascia

Ymbilimiddot~1 fold)

preu9poundiea1 fascia

tissue (fatty 4nd m~mbr3n(iUS layers)

o Above the arcuate line (A horizontal line 13 of the distance bw the umbilicus and the

pubic symphysis) -10 Aponeurosis divides into an AntPost Laminae

o The Ant Laminae joins EO and Post Laminae joins Trans Abdominis = Ant and Post

RECTUS SHEATH respectively

o BElOW the arcuate line - all 3 aponeurosis join ANTERIOR to rectus muscle to meet its

counterpart in the midline (linea Alba)

o Take away Msg - The abdomen is devoid of a posterior rectus sheath below the

arcuate line and is therefore more vulnerable to herniasinjuries

Question - A physician makes a deep incision in the patients midline immediately superior to

the pubic symphysis which of the following layers is his knife least likely to pass

Rectus Abdominis External Oblique Ant Rectus Sheath Posterior Rectus Sheath All of the

Above

Answer - All of the above None of the other answer choices are midline structures -LINEA

ALBA

Linea Alba has very poor blood supply - doesnt heal well after surgery Therefore this is a

common site for incisional hernias

a Spleen b Transverse colon c Descending colon d Stomach e Pleura

17 Meckels diverticulum is normally found 2 feet proximal from the

a Pyloric sphincter b Lower esophageal sphincter c Ileo-cecal valve d Middle valve of Huston e Anal valve

18 Ulcer in the posterior wall of the first part of the duodenum would erode ___ artery and would cause bleeding

a Left gastric b Right gastric c Hepatic artery proper d Gastroduodenal artery e Middle colic artery

19 An inflamed appendix is identified by a surgeon on the operation table by noting

a The appendicies epiploicae b The convergence of tenia c The artery of Drummond d The mesocolon e The mesosalphinx

20 The nerve which emerges through the psoas major is

a Femoral b Ilio-inguinal c Ilio-hypogastric d Pudendal e Subcostal

21 The right gonadal vein drains into the

a Azygos b Hemiazygos c Inferior Vena Cava d Right renal vein e Left renal vein

22 The hepatocytes in the liver is derived from

a Ectoderm b Endoderm c Mesoderm

d Neural ectoderm

23 Abscess in the lumbar vertebrae due to tuberculosis would spread to the adjacent muscle which is

a Psoas Major b Iliacus c Quadratus lumborum d Tranversus Abdominis

24 The anterior wall of the inguinal canal is formed by

a External oblique and transverses abdominis b External oblique and fascia transversalis c Internal oblique and external oblique d Internal oblique and transverses abdominis e Fascia transversalis and peritoneum

Meckels diverticulum is a result of which of the following developmental abnormalities shy

A Failure of the vitelline duct to close

B Failure of the herniated intestinal loop to retract into the abdomen

C Failure of the urachus to close

D Failure of the midgut to rotate

E Failure of the hepatic duct to close

Explanation

Meckels diverticulum is a result of the persistence of the proximal part of the vitelline duct This

diverticulum is usually found about 2 feet proximal to the ileocecal junction and is usually about 2 inches

long It is present in about 2 of the popUlation It may be the site of ectopic pancreatic tissue or gastric

mucosa and may develop inflammatory processes and ulcerations Acute Meckels diverticulitis

simulates appendicitis

Which of the following veins carries blood from the esophagus to the portal vein The

A right gastric vein

B left gastric vein c splenic vein D azygos vein

E left gastroepiploic vein

Explanation

The left gastric vein a direct branch of the portal vein drains blood from the lesser curvature of the

stomach and the inferior portion of the esophagus Because branches of the portal vein do not have

valves blood can flow in a retrograde path when there is an obstruction to flow through the portal system or liveL Rlooci Cln then flow from the nortl] vein thr()1Ph the left PRstric vein to the esonhlPlIS lno

through venous communications within the submucosa of the esophagus to esophageal veins that drain

into the azygos vein The increase in blood flow through the esophageal submucosal veins results in esophageal varices

On the posterior wall of the abdomen the celiac ganglion A contains cell bodies of postganglionic parasympathetic neurons B is synapsed upon by neurons in the posterior vagal trunk C is synapsed upon by neurons in the greater splanchnic nerve D contains sensory cell bodies of lumbar spinal nerves E contains cell bodies of neurons that cause an increase in the rate of peristasis

Explanation The celiac ganglion is one of the preaortic ganglia of the sympathetic nervous system It contains cell bodies of postganglionic sympathetic neurons The sympathetic splanchnic nerves contain preganglionic sympathetic neurons that pass through the sympathetic chain without synapsing These splanchnic nerves go to the preaortic ganglia to synapse The greater splanchnic nerve contains preganglionic neurons from spinal cord segments T5-T9 This nerve synapses in the celiac ganglion The nerve fibers in the vagal trunks are preganglionic parasympathetic fibers that go to the walls of the organs that they will innervate and synapse on postganglionic parasympathetic neurons in the walls of those organs Cell bodies of sensory neurons in the abdomen are found in the dorsal root ganglia or the sensory ganglia of the vagus nerve Sympathetic innervation decreases the rate of peristalsis parasympathetic innervation increases the rate of peristalsis

Which of the following pairs of arteries will allow blood to bypass an occlusion of the celiac trunk

A Left gastric artery-right gastric artery

B Left gastroepiploic artery-right gastroepiploic artery

C Superior pancreaticoduodenal artery-inferior pancreaticoduodenal artery

D Splenic artery-common hepatic artery

E Left gastric artery - proper hepatic artery

Explanation The anastoOlosis of a branch of the celiac trunk and a branch of the superior mesenteric artery will

provide collateral circulation around an occlusion of the celiac trunk Each of the other choices pair

branches of the celiac trunk therefore these will not provide collateral flow around the obstruction of the

celiac trunk The left gastric splenic and common hepatic arteries are direct branches of the celiac trunk

The right gastric artery is a branch of the proper hepatic artery which is a branch of the common hepatic artery The left gastroepiploic artery is a branch of the splenic artery The right gastroepiploic artery is a

branch of the gastroduodenal artery whlch is a branch of the common hepatic artery

Which of the following organs has appendices epiploica The

A sigmoid colon

Bjejunum

C duodenum

D stomach E esophagus

Explanation Appendices epiploica are characteristic of the colon Appendices epiploica are subserosal accumulations

of fat None of the organs of the gastrointestinal tract has appendices epiploica except the colon

Page 24: Chirag's Abdomen Review

Which of the following arteries is a direct branch of the gastroduodenal artery The

A right gastric artery

B left gastric artery

C inferior pancreaticoduodenal artery D left gastroepiploic artery

i E)right gastroepiploic artery --

E x pI a nation The right gastric artery is typically a branch of the proper hepatic artery The left gastric artery is a direct

branch of the celiac trunk The right and left gastric arteries anastomose along the lesser curvature of the

stomach The inferior pancreaticoduodenal artery is a branch of the superior mesenteric artery it

anastomoses with the superior pancreaticoduodenal in the head of the pancreas The left gastroepiploic

artery is a branch of the splenic artery it anastomoses with the right gastroepiploic artery along the greater

curvature of the stomach The right gastroepiploic artery is a branch of the gastroduodenal artery The

other branch of the gastroduodenal artery is the superior pancreaticoduodenal artery

Which of the following pairs of veins join together to form the portal vein The

A superior mesenteric vein and inferior mesenteric vein

B inferior mesenteric vein and splenic vein

C superior mesenteric vein and splenic vein

Ip)splenic vein and left gastric vein E superior mesenteric vein and left gastric vein

Explanation

The portal vein is formed behind the neck of the pancreas by the union of the superior mesenteric vein

and the splenic vein The inferior mesenteric vein drains into the splenic vein The left gastric vein drains

directly into the portal vein After the portal vein forms it enters the hepatoduodenalligament of the

lesser omentum to reach the liver The portal vein is the most posterior structure in the hepatoduodenal

ligament

At which of the following vertebral levels does the duodenum pass anterior to the aorta - _- shy

All ~

B L2 7~

CL3 I

~DL4

E L5

Explanation

The duodenum begins at the pyloric sphincter at the level of Ll The second (or descending) portion of

the duodenum is to the right of the aorta and extends inferiorly from the level of Ll to the level of L3 The third part of the duodenum crosses the aorta from the right side to the left side at the level of L3 The

fourth (ascending) portion of the duodenum extends from the level of LJ to the level of L2 The

duodenum ends at the duodenojejunal flexure The superior mesenteric artery passes anterior to the

duodenum as the duodenum passes anterior to the aorta The duodenum can be constricted at this level

In which of the following locations will perforation of the digestive tract result in the spilling of luminal

contents into the - lesser peritoneal sac

A Anterior wall of the second portion of the duodenum B Posterior wall of the second portion of the duodenum

C Anterior wall of the stomach

~Posterior wall of the stomach E Posterior wall of the transverse colon

Explanation

The posterior wall of the stomach is related to the lesser peritoneal sac The anterior wall of the stomach is related to the greater peritoneal sac The anterior wall of the second portion of the duodenum is related to the greater peritoneal sac The posterior wall of the second portion of the duodenum is related to the retroperitoneal space The posterior wall of the transverse colon is related to the greater peritoneal sac

The ureter lies against the anterior surface of which of the following muscles shyA Crus oftne diaphragm B Quadratus lumborum

0 Psoas major D Transversus abdominis

E Iliacus

Explanation The ureter exits the renal pelvis at about the level of vertebra L2 As it descends along the posterior abdominal wall it lies on the anterior surface of the psoas major The psoas major muscle arises from the bodies of the lower lumbar vertebrae The psoas major muscle is joined by the iliacus to fonn the

iliopsoas muscle The iliopsoas muscle then attaches to the lesser trochanter of the femur and is the major

flexor of the hip

As the right ureter passes the pelvic brim it lies against the anterior surface of which of the following

blood vessels

A Gonadal artery B Inferiorvena cava C Internal iliac artery

rJ- External Iliac artery

E Inferior mesenteric artery

Explanation

The ureter lies in the extraperitoneal space in the posterior abdominal wall Alter leaving the kidney it

passes inferiorly on the anterior surface of the psoas major muscle At the pelvic brim the ureter passes

into the pelvis At this point the common iliac artery is dividing into the external and iliac arteries The

ureter lies on the anterior surface of the external iliac artery immediately distal to the bifurcation This is a useful landmark for a surgeon to locate the ureter

When extravasated urine passes from the superficial perineal space into the anterior abdominal wall it is

found immediately deep to which of the following layers of the anterior abdominal wall

-ltScarpas fascia

B External oblique muscle

C Internal oblique muscle D Transversus abdominis muscle

E Transversalis fascia

Explanation

The superficial perineal space is bound by Colles fascia the fibrous portion of the superficial fascia This

layer of fascia is continuous with Scarpas fascia the fibrous portion of the superficial fascia of the anterior abdominal wall Therefore urine that is deep to Colles fascia will remain deep to Scarpa s fascia The urine will spread in the plane between Scarpas fascia and the external oblique layer

When a horseshoe kidney develops the ascent of the kidney is restricted by the A internal iliac artery B external Iliac artery

C common iliac artery

inferior mesenteric artery

E superior mesenteric artery

Explanation

A horseshoe kidney develops when the inferior poles of the to kidneys fuse together as they ascend into

the abdomen from the pelvis The first anterior midline vessel that is encountered by the horseshoe kidney

is the inferior mesenteric artery This artery prevents the kidney from continuing its ascent

The left testicular vein drains into which of the following veins

A Left internal iliac vein B Left common iliac vein

bflnferior vena cava D Left renal vein I

E Left internal pudendal vein

Explanation

The left testicular vein drains into the left renal vein The right testicular ~i~[~nsltjectlY into the

inferior vena cava This difference in venous drainage is believed to explain the greater incidence of

varicocele on the left side than on the right The venous drainage from the penis is to the internal vein

which then drains into the internal Iliac vein

The spinal nerve that provides cutaneous branches to the skin around the umbilicus is

A TS B TW-shy

C TI2

DL2 EtA

Explanation

The tenth intercostal nerve is the anterior ramus of the TIO spinal nerve After passing through the tenth

intercostal space the nerve continues forward in the anterolateral abdominal wall in the plane between

the internal oblique muscle and the transversus abdominis muscle In the abdominal wall the nerve innervates to the abdominal wall muscles as well as the skin and the parietal peritoneum The umbilicus is

a useful landmark for the region of distribution of the tenth thoracic nerve

The ligament of the vertebral column that resists its extension is the Aligamentum flavum

B supraspinous ligament

C posterior longitudinal ligament

D anterior longitudinal ligament

E interspinous ligament

Explanation

The ligaments of the vertebral column that resist flexion of the column include the supraspinous ligament

interspinous ligament ligamentum fiavum and posterior longitudinal ligament The ligament that resists

extension is the anterior longitudinal ligament This longitudinal ligament is very broad and strong It

covers the anterior and anterolateral surfaces of the vertebral bodies and the intervertebral disks In

addition to resisting extension the anterior longitudinal ligament provides reinforcement to the anterior

and anterolateral surfaces of the intervertebral disk The posterior longitudinal ligament is relatively

narrow and covers the posterior surface of the vertebral bodies and the intervertebral disks This ligament

reinforces the posterior surface of the disk The posterolateral surface of the disk is not reinforced and it

is through this region that herniation of the nucleus pulposus usually occurs

A patient presents with epigastric and right upper quadrant pain The pain is most intense 2-4 hours after

eating and is reduced by the ingestion of antacids The patient states that he has passed black tarry stools

(melena) within the last week Fiberoptic endoscopy reveals a yellowish crater surrounded by a rim of

erythema that is 3 cm distal to the pylorus Accordingly an ulcer has been identified in the patients

A fundus

B antrum

C duodenum

D jejunum

E ileum

A number of physiologic genetic and other factors increase the risk of gastric (and duodenal) peptic

ulcers The evidence that H pylori plays a principle role is compelling Smoking and caffeine are known to adversely affect the morbidity mortality and healing rates of peptic ulcers In general first-degree

relatives of peptic ulcer patients as well as males have a threefold to fourfold increased risk of developing this disorder Paradoxically in gastric ulcer disease acid secretion is not elevated It is possible that

excess secreted hydrogen ion is reabsorbed across the injured gastric mucosa In general a defect in gastric mucosal defense is the more important local physiologic

A patient presents with symptoms of duodenal obstruction caused by an annular pancreas Annular pancreas is caused by

A rotation of the dorsal pancreatic bud into the ventral mesentery B rotation of the ventral pancreatic bud into the dorsal mesentery

fJ failure of the major and minor pancreatic ducts to fuse ~ ~ cleavage of the ventral pancreatic bud and rotation of the two portions in opposite directions around -the duodenum E formation of one pancreatic bud instead of two

Explanation Normally the ventral pancreatic bud rotates around the gut tube to reach the dorsal pancreatic bud The two buds fuse to form a single pancreas and the distal portions of the two ducts fuse The ventral pancreatic bud forms the inferior portion of the head of the pancreas the uncinate process and the major pancreatic duct (of Wirsung) The dorsal pancreatic bud forms the superior part of the head the neck body and tail and the minor pancreatic duct (of Santorini) Annular pancreas is the result of the ventral pancreatic bud dividing into two portions before it rotates into the dorsal mesentery Each portion rotates in opposite directions to get to the dorsal mesentery thus encircling the duodenum The presence of annular pancreas can constrict the duodenum thus obstructing its lumen

In n _ phranlc----

Gon ~l ----_1 Lum bltano

~~--- CornmQ1t bull ac

+-~4--- lnlllirnaJ ilic

xtem iliac

OBJECTIVE - Identify the blood supply to each of the structures listed in the table on the previous page

Ill give you a head start

FOREGUT - Supplied bV Celiac Tru nk (T12)

Proper hepatic

GastiooUod 13Jafter

1nferlor pancreaticoduodenal artery

Common epatlc

Lett gas ric iiirtery

Spfen artery

shy Gastroepiphgtic artery

~ Superior mesenteric 8rtfry

~

1 Esophagus is a derivative of the foregut so its blood supply originates from the celiac trunk

(T12) The predominant blood supply to abdominal portion of the esophagus is the Esophageal

A (Branch of L Gastric) The venous drainage of the esophagus is particularly important because

it is 1 of 3 clinically relevant sites of Portal Caval anastamoses The Portal Esophageal Vein

meets the Caval Azygos System Persistent bleeding manifests as Esophageal Varices - a fata I

condition

2 The Stomach is also a derivative of the foregut has EXTENSIVE blood supply and is very high

yield on anatomy exams The lesser curvature is supplied superiorly by the L Gastric A (1 of 3

major branches ofthe Celiac trunk) and inferiorly by the R Gastric A ( a branch ofthe proper

Hepatic A) The greater curvature is supplied superiorly by the L Gastroepiploic A (a major

branch of the splenic A) and inferiorly by the R Gastroepiploic A

The Short Gastric arteries (branches of Splenic Artery) supply the fundus of the stomach and

are referred to as EIID ARTERIES because they have no collateral blood supply Therefore if the

splenic artery were occluded (ex - increased pressure in the ommental bursa) - there would be

ischemia to the fundus of the stomach Venous drainage of the stomach is extensive via various

veins lead ing to the portal system Posterior to the stomach the IMV joins the splenic V which

joins the SMV to form the PORTAL VEIN ADAMS

3 Duodenum blood supply has high clinical relevance because it is the junction of the foregut and

midgut and therefore is the site of anastamoses between branches ofthe Celiac Trunk (main

foregut artery) and the Superior Messenteric Artery (main midgut artery) The Proper hepatic

artery gives off the gastroduodenal artery which travels behind the 1st part of the duodenum

This point has high clin ical relevance because duodenal ulcers are very common and a posterior

rupture of the 1st part of the duodenum could rupture the gastroduodenal artery causing

traumatic abdominal bleeding The Gastroduodenal artery first gives off the R Gastroepiploic A

(mentioned above) and proceeds as the Superior pancreatico duodenal artery (supplies the

pancreas and duodenum) which anastamoses with the inferior pancreatico duodenal A (branch

of the SMA) This is the junction of foregut and midgut and occurs near the opening of the

bil iary system into the duodenum (ampula of vater) Portal venous drainage here is responsible

for delivering nutrients from digestion to the liver for metabolism Appreciate that the Superior

mesenteric artery (artery of the midgut) branches from the aorta at Ll travels posterior to the

pancreas than moves anteriorly (at the jxn of the pancreatic headbody) and comes over the

3rd4th part of the duodenum Tumor of the head of the pancreas can compress the SMA

4 Jiver blood supply is via the common hepatic artery (major branch of the cel iac trunk) The

common hepatiC becomes the proper hepatic gives off the R gastric A and the Gastroduodenal

A and then joins the common bile duct and the portal vein in the portal triad Clinical- if a

patient were bleeding from the hepatic A a surgeon can stick his fingers in the epiplOic foramen

and squeeze the free edge of the hepatoduodenalligament in order to stop bleeding to the

area Please note that the hepatic a branches into Rand L hepatic A The Right hepatic artery

gives off the cystic artery which supplies the gallbladder Afferent venous supply is via the

Portal vein which is bringing nutrient rich blood to the liver After metabolism takes place

venous blood leaves the liver through the hepatic veins into the IVC PLEASE UNDERSTAND THE

RELATIONSHIP OF THESE STRUCTURES - ADAMSNETIERSNH Etc

5 Pancreas - Head is supplied via the superior and inferior pancreaticoduodenal arteries

(mentioned above) The tail (situated towards the hilum of the spleen) is supplied via the

pancreatic branches of the splenic artery (END ARTERIES) This blood supply is very important

because the endocrine Alpha and Beta Cells from the pancreatic islets of lagerhans are located

towards the tail This is where Insulin and Glucagon is released to the blood

Now complete this for mid and hindgut structures Make sure to note clinically relevant arterial

anastomoses as well as portal caval anastomoses FYI Appendix blood supply SMA + IMA

anastamoses marginal artery Portalcaval rectal veins fhemmorhoids) and periumbilical caput

medusa are high yield THE BUTT THE GUT and THE CAPUT

Abdominal Development

Liver

Ij1f

II wall b

oh liN ~ VltJrti n be- bull

Pancreas

Secondary Retroperitonealization e I~tl r 1 a v-mtrai m ellter

Rotations of the Gut I i Ij (lIl1UtIJ f~ l r tilt

()l td 10 me l-ft and he v

--~--- -~ -~-~

i

I AolaijonjoI~guf I

STOMACH BED (IDENTIFY IN ADAMS)- the structures posterior to the ommental bursa which

support the stomach in the supine position

Abdomnal JQrUI

Splnic vein

OmQ-oul tv~ ) O(s(Jroa)

Lojt(r o m nturrt (hpJtodu o d~n31 Hid

Gadrl)SplerH (g3stroll~nal) IIgam~nt

hiad h~~atogrtricent IIQdmiddotcrt~)

Lt Dome of Diaphragm (why left Look this up in Adams)

Spleen (What is the blood supply)

Left Kidney (What is the blood supply - AND how is it different from the R kidney)

Suprarenal Gland (What is the Arterial AND Venous Blood supply - how are they different)

Pancreas (How does supply differ from Head to Tail What is the SMA Relationship)

Transverse Mesocolon

liver - ADAMSWET - Make sure you look at the liver in wet lab

Left triangular nl1am~nt

ComoaDj ligamnt

Erophg~1 impre$ioo

Hepatio veins

In1erior -ifena middotr3)Ia

Fibrous appendix o-t

live

impr~j on

Heprorendl p~rtion of Q)(Qllary ligament

Righllri~n9ul r 1I~met

(Common) bile quol

Gr)mmCtr~ hepatic dlJct

Ccentic duct

Duodenal impression

GaJdate p-fr)~S

Hepatic artgtrl prop-f iiiiila - Faloiform ligament

_ - shy Round ligamen liver

~--F-- CoJio imprgt-ssi-on

Prta heptis

Identify the lobes impressions and embryonic remnants associated with the liver

Caudate Lobe Quadrate Lobe Right Lobe Left Lobe Round ligament Falciform Ligament

Ligamentum Venosum (what is its fxn in embryonic life) Hepatic Veins (NOT PART OF THE

PORTAL TRIAD) IVC PORTAL TRIAD - Contents relationship cross section etc Know the

Galbladder relationship to the lobes of the liver

Biliary Duct System - Make sure you understand the sequence of these structures - BE ABLE TO

DRAW A FLOW CHART

TPVd i

t

I t

1 __ Cm-(r

patk GlJet

I

J

Clinical = JAUNDICE is caused by anything that prevents delivery of bile to intestine Tumor of the

head of the pancreas Stones etc Patient will have pale stools and yellowish colored mucus

membranes

Clinical- Any scenario that tells you the patient has BILLOUS VOMIT means that the obstruction to

the flow of digestive contents is after the Ampulla of Vater (Site of Entry of Billiary system to the

duodenum) - ie Duodenal Atresia

Spleen -located posterior to the mid axillary line between ribs 9 and 11 Make sure you know that

the 10th rib is the main axis of the spleen and this organ is susceptible to injury (stab wound errant

thoracoce ntesis etc)

The spleen is derived from mesodermal cells - NOT THE GUT TUBE

The spleen rests on the left colic flexure associates with the tail of the pancreas Know the

structures entering the Hilum of the spleen

Sh rt O~-t~ic 1 0(0 10 rtiltSPIric Iloa nt

(cut)

Peritoneum - similar concept to Pleura - think of a fist in a balloon

Visceral Peritoneum - Layer of balloon touching your fist

Parietal Peritoneum - Layer of balloon not touching your fist

Your fist represents the organ your wrist is the hilum and your arm contains the blood supply

entering the organ

Appreciate that there will never be organs in the peritoneal cavity - rather these organs invaginate

the cavity Kaplan videos

RULES OF NOMENCLATUREshy

1 Organ completely surrounded by peritoneum - peritoneal organ

2 Organ partially surrounded by peritoneum- Retroperitoneal

3 Peritoneum surrounding peritoneal organ is VISCERAL peritoneum

4 Peritoneum surrounding retroperitoneal organ is PARIETAL peritoneum

5 Peritoneum connecting visceral to parietal is called messentary 2 messentaries in the

gut Dorsal (to the gut tube) and ventral (to the gut tube) messentary

Aorta is in Retro peritoneal position - but blood must reach peritoneal position - vessels travel through

messentary All peritoneal organs will have blood supply reaching through messentary

-Mesentery is a 2 layer peritoneum with a neurovascular communication between body wall and organ

- Ligament connects one organ with another or to the abdominal wall (Ommentum = ligament)

lesser Ommentum (attach lesser curvature of stomach and duodenum to liver) =Hepatoduodenal

Ligament and Hepatogastric Ligament

Has a Superior and Inferior Recess (Accumulation of Fluid in Ascites)

Communicates with the greater sac through the epiplic foramen (what structures pass through

this foramen)

Boundaries - you must be able to visualize this

o Anterior - stomach

o Posterior - parietal peritoneum pancreas

o Superior - superior recess (bw diaphragm and coronary ligament)

o Inferior -Inferior recess (bw layers or greater momentum

Greater Ommentum (attach greater curvature of stomach) Gastrophrenic ligament Gastrosplenic

ligament gastrocolic ligament

The greater omentum is the largest peritoneal fold It consists of a double sheet of peritoneum folded on itself so that it is made up of four layers The two layers which descend from the greater curvature of the stomach and commencement of the duodenum pass in front of the small intestines sometimes as low down as the pelvis they then turn upon themselves and ascend again as far as the transverse colon where they separate and enclose that part of the intestine

ABDOMINAL PAIN

Parietal Peritoneum - supplied by same vasculature lymphatics and nerves supplying body wall it

lines and diaphragm Sensitive to pain pressure heat cold well localized

Visceral Peritoneum - supplied by same vasculature lymphatics and somatic nerve of organ it covers

Insensitive to touch heat cold and laceration - referred to dermatome of spinal ganglia providing

sensory fibers Where does appendicitis refer to

Foregut pain - epigastric area (ie - cholycystitis)

Midgut pain - periumbilical area (ie - appendicitis)

Hindgut Pain - suprapubic area (ie - diverticulitis)

Extra ImagesConcepts

ll~_____-

FalifCtrm ligament oind r~ud ligamet f Ilver

Blood from splenio gastriC and inferiof rne$e-rteri v~ins

Ca-I tributaries

Lett gastrio Ifein

Posterior superior pan~reatioodul)denal vaihS

Lott gamo-om~nlal (9aropip lomiddotic) -in

Poq_~ tjol imerl-9-r panCJertlcorllJod-nal veiopound --amp----I- - ~J Right grtr~-omntal

Anwrior interi (gartroepiploic) Jjn

pan euaii cod vl)denal veins middot Inf~Ji (t r mesentric vein

Miqdle (olic vein

Right cl)licvein Sigmoid and rectosigml)id (ei ns

IhH)Collc(~io

--- Mi~dl laquooLJl gtjrltgt

PoM ca vl1 illasto)moses -----shyampoptoageal 2 Paraumbilie-lt11 Inferi or Fectal vei ns

3 Recial 4 REuoperHonea1

Know how the Portal vein is formed I 4 sites of portal caval anastamoses and 1 clinical shunt

Col li t ltt-~ otTl~tI ~nj pc~ 1lt1 turJoG

Ltf 14i1 tImiddot~ artoftl9 on tj phtAt$

L-oftqf 4t t~r 1=laquoIran d 1 bull shy~p l ci rj o fOOOts

Nerves follow the arteries - appreciate the splanchnic nervous system I

Uet~ric branch of left ~nal art

Ureterie branch of righi renal artelY

Left Zld lumbar in and co mlTlunication to as)erdin9 lumbar l(~in Hi ~ht tEZ1~~t~ t3r j t itn ~ nJ l1t- rlnd lfe i r1

Inferior me5nteri~ artery

Notice that the right testicular vein drains directly into the IVC and the right testicular artery drains

directly into the aorta However the left testicular vein drains into the L renal vein at a right angleshy

reason left testicle is lower and more susceptible to varicocele (bag of worms)

Also notice that the left renal vein has a longer course because the IVC is on the right side whereas

the right renal artery has a longer course because the aorta is on the left side

Appreciate the anterior to posterior relationship of structures in the hilum of the kidney - VAP - Vein

Artery Renal Pelvis (Ureter)

11____ __ L_ L_ n VJ __ _ _ t_L I I_ _ L __ L_ I -pound1 bull LI_~-I ____

Posterior View of Head of Pancreas in ( of Duodenum

Celiao hunk

Co mmon ~L~jJth art~ry

GastNduQdonal artrf (partilly in phantn)

P1)Sterior $Up~Jior panCflaticuduodfmal art~r~t

(Co mm on) bile duct

middot~1t~~t-1l---~-~- Right gshomiddotomental (gastoe plp lolc) 3rte (phantomost)

Grener paocre atic art-ry

1n1~rjor pancr-iatlc artery

Jtrifll supejo r pal)oreailcento)dJodenal artr1 (phantom)

Anastomotlo branch

POostetlor bJanch of jo f~ri of pan-reatir(lduodensl drttnj

Anterio r branch of i flferior palcreati~)duodenal art~(phan1om)

Notice the extensive blood supply to the pancreas and duodenum via the branches of the celiac trunk

Notice collateral supply from SMA branches - makes sense bc this is the jxn of foregutmidgut

Identify the vessels in this arteriogram

Hiltid i)f N~ck oi B)dvof Tail 01 pa nereas pan cent~as P-nmiddot-reas panCtCas

I nferie v~na cava

jHept1iic p(lrlai v~in

Port1 tnd H~pti lt a ftH prol

Comm on) bll duct

Ouodtnum

~ft colic (sio)Atta~ hmtrlt jt~xJr-ofha~elSe

muo(IIQn

Right ~lIc (h~j)tic)

il~gtture

In1triol m~oten lIein (rttr op~ritoMdO

SlJp efl or mes~n~fiC amrV and lipln

KNOW YOUR NEIGHBORHOOD

Questions

vVhiJh structure supplied by a bnmdlof the cclia( artery is not derivcd from foregut LemCJUCrITI

(A) Head of the pancte-a5

CD) Pyloric duolenum

Cystkduct

( Liver hepatocyt~~

~F) Body of the spleen

An infant presents with an omrhaJucele at birth -hi oJ the [oHm illg applies to his cM1-dition

(A) It is 31so seen ill p4titnts with aganghonic megacolon

(11) ft reuirs from a fal1ure of resorption of theviteUine d let

(C) It results from herniation at the-site of regression of the right umbilk vein

DJ It is caustd by faihtrc of recanalization of the midgut part of the duodenum

~ It ill camioo by a failuIt vf the midgul to return to the abGQminal uity after herniashytion in-n the urnbilk s l stalk

Ot er than the spleen occlusion Cif the spit-Ilk artery at its odgin wm most likely affect die blood supply to jllch st cnud

(A) Jejunum

(B) Body of th pal1~lltas

(C) LeSStT Cllmiddotlaturc of tl )toma-ch

(D Duodenum dista to the entrance of the Ornmou bile duct

E Fundus of the stomach

A 38-yeu-old batL~er with a history of heartburn suddenly experiences excluciating pain in the (plgastric region of th~ abdomeu SurgCry is perf~rme immediard y upon admisshysion to the 1IlcrgCJliy tuomh~re i~ evidence uf a ruptured ulcer in the posterior waU of the stomach Vhere will a surgeon first fi nd the stomach contenlSf

A) Greater p4ritoneal sac

rB) Cul~de-s~c of Douglas (--

C Omental bursa ~

--D) Paracolic gutter

rEj Between -he panttal perimltum and the posterior body wal1

At birth an infant presents with a st()ma~ rb~tbas~njJled jfltotb~diaplfagru 1A1ltre is the defect thatresulied iiitJle heini~t()n shy~tsophagealbiatus

7 - rH-- Hiatus for the inferior vena cava

( Pleuroperitoneal membrane -(0) Septum transvcrsum

(E) Right Crlt~

An infant born with DOVv7l syndrome presents with bili()u~ vomiting Ahat congenital defect does the infant have

(A) Pyloric stenosis

(B) Meckel diverticulum C) Ornphaloce1e

(D) Gastroschisis

( ~ ) Duodenal atresia y A patient with cirrhosis of the liver presents with ~ bacalvaricestnlreased retrograde pressure in which veins caused the varices

(A) Paraumuilical

(B) Splenic

(ct AzygltJus

(15))G~trk ( (-F) Superior mesemeric

A htaltby 3-year~old male patient experiences a hernial sa protruding from the anterior abdominal wall about halfway between me anterior superior ilia spine and the pubk tuberde Pulsations of al1 artery are palpated medial to the protrusion site through the abdominal walL Which layer of the anterior abdominal wall will first be traversed by the

1hctma

fA) Rectus sheath (B) External oblique aponeurosis

(C) Inguinal ligament

lD) Transversalis fusda

(E) Cremasteric fa~cia

After 5urgi(aj ffpair of a hernia the patient tXperienccs mtmlgtness in the skin on the anteshyrior aspect of the S(Totum_ Vhaf nerve may have been lesioned during thehemiorrhaphy

(A) Femoral

(B) Obturator

(C) Ilioinguinal

(D) lliohypogastrk

(E) Pudendal

A 23~year-LJld female secretary il1 good health ~-uddcn1) doubles over with pain in the a ea of the 1JmbRicu$ Sbe feels vartn and ltneasy and has no appetite That night the pain seems to have mQved to the tower right abdominal regjol1 and she calls her family doctor who then arranges for an ambulance to pk-k her up and take her to the hospitaL Wh ell ntn~ perceived in the area of the urnbilirus most Hkely carried lhe pairfu I sensations into the eNS

tA) Vagus nerves I~

V B)

) Lessersplanchnk nerves

tC) Pudendal nerves

(D) lIiohpogastrk nerves

(E) Greater splam ic l erves

A CT reveals carcinoma in the bOod of the ancreas Vhich blood vessel trut ourses ----~- - -bull ------ --shy

immediately poftterior to the body ofthe pancreas is the m~t likely to be oompressed

(A) Splenk artery

(B) Abdominal aorta (C) Portal vein

(1) Splenic vein

(E) Renal vein

A patient has a penrln1l1ng uker of the posterior wall ot the br~l part ot the (lUooenmn llkh blood vessel is subject to erosion

(A) Common hepatic artery

(B) Gastroouodenal artery

(C) Proper hevatic artery

(D) Celiac artery

(E) Anterior inferior 11amrelltlcoduodcnal attery

Your patient has been diagnosed -ith a carcinoma locallted to the head and l~e(k of the pancreas Another clinical sign would be

A esophageal varices

(8) hemorrhoids

C) a caput medusa

(D) increased pra Teuro n th~ hepatic veins

(E) enlarged right supra lavkular lymph nodes

Wltkh of the foUowing structures develops in the ventral mesentery

(A) Spleen

(B) Jeiunum (C) Head of1ht pancreas (D) Transverse colon (E) Stomach

ti l Uw ~ littwin~ f( S-t lil oai Imdge ~ hi(h or tbt la~)d J truetur tgt liJ llntn nl) he hl p UC iJd [IIi ell

c o

A) drains Ie tht infCrior a La aI

R t middot~nfl0 ~ill to th~ lunlgtn of h i dtlndCrlllfH

(e) m t bull JiJattd on tl l J n T ~H

D ) sup Lc O VSlt I Hlid bhtu l 1 li - -I un oid

( ) U~tpli(t tr j middottUh~ 1 v(( b~nt rfK n1ilc~Zm

ANSWERS AND EXPLANATIONS

Answer E The spleen is t hlttnopodicand lymph organ demlted from mesoderm

Answ~ R Al1 tlmphalocele is caused by it failure of the nlidgut to return to the ahdomir nat cavity after herniation into the umbiliau Stalk Choices Aand D maybe seen in infants with Down syndrome choice D ~s the specific CBuse ofduudcnal JtiCSitt Choice C is (ile cause of gclstrosbisis and Choice B nsults iu a Meurolktldivertku1-tlB

Answer B The fundus ofthe stomach is suppHed by soort gastric brunches of the splenic altery The splenic artery supplies the body and tail of the pancreas part of the greater curvature of the sttmla(h and the spleen Te jejunum part of the head of the pancreas and tht~ duodenum distal to the entrance of the commOll bile duct are supplied by the superior mesenterk artery clll~l ~be less r ctlt1ature cmd the pylQric antrum are supplied by the right and lei gastric art(ries

AnSWftt C Tbeomental bursa or lesser ~ritoneaj sac lies direcdy posterior to the proxshyimal part of the duodeTtlm and the stomach and would be the first site where stomach contents ~Ott1d be fpoundluncL

Answer C A defect in a llleuropcritoneal membrane (uswlly the left) is the typical site of i1 cc-ngenitlI diilphragluatic hemia llere the membr4ne fails to dose ()pound( of the perishycCirdiopcritulleal canals

Answer E DuoJenal atresia and aganglionic megacoion are congwitaI defects S~Il in patients with Dowmiddotnsyndrome

Answer D RulaTgemt~llt of and retrograde flow in g~lstrk vel_ns in particlJl~r the kft gas~ tricveins dilates the capillary bed in rhe wall of the esophagus in (ases of porta yper~

tension Blood flow would increase in and dilampte tribntarkgts of the (lZygOUS vein on the other side of the capiUary bed but flow in this vein is in the typical direction t()ward the superior vena cava Paraumbiii(ltU vein eilgorgement contributes to a caput medusH Splenic ~nlargement might prc~nt with 5plcnonlegaly and balt-kflow in to tlu superior m~~ntclic vein occurs but is asymptomatic

Answer D The patient hagt an indirect inguinal hernia whi~h emerges from the antt-rior abdominal wall through the deep inguinltilling Theeep ring is a fault in the transv~rshysaUs fascia this I~yer wiIJ be penetrated first by the hernia

An~Wer C The ilioinguinal nenc which provides sens~llion to the lnedlal thigh ltmclanteshytior SClotunl pass~lt th rough the 5uperfh_ial inguinal ring ind $subject to inj i1T) becaus-e

it is in the operatitm Held of the erniorrhapny

Auswer B The leMHr splanchnic nerves are sympathdic nerVlts that carry viscera l sensashytlltgtrogt ftom illtllt1m~d ()J stietched gust (itinteitinal ~tructures (in this case the pprndix) into tnt eNS Lesser splanchnic ntTYcsarisc from thmiddot T9--T12 spinal cord segments lt1nd provide sympathetic innenation tD rnidgut siruc1ures whiCh include CLe app~JldD Viscera] Pain arising from affecLed Inidgut ampt 1C1ure is referred over the same dl- matorne~ of spinal segrnertts v-hich provide the sympathetic Innervation n this G1SC of appendicitis the invohen~n t of the ltire) of t e unlhHku indud s the T 10 dermatome

Answer B Of the five choices onty the dscending olon is retroperiton~al aldwould be a lik ~ ( choice to be seen immediately a(~jilcent to t11e posterior abdominal middotn~L

Amwen D The SpltftlC ~-ein ourses posterior to the body of the panneas m its way tt drain into the superior mCSfttltlri( vein

Answcr B TILt glstrodllolticnal artery 1 direct hIamh of the comrootl hepatic artery courses immediately pt))iwri() to the duodenum and is slbject to erosion

Answer B Carcinoma of th pan middott3S in the 1tilt1 may compreampgt the portltil vein at irs orishygill The poTtai vcin is fomled when the splenic vein jQiaswith tfie superior meStllt eric vein The inferiot mesenteric vein joins the ~plenjc vein just priOT to tlli~ point at which the splenic joins the superior Jlleit1ltcri( vein Increescd venous presslu in the inferior mesenteric vein is a cause of emo hoid~

Answer C The- velltral pancreas wilich forms most of the head of the p ~ncr as develops in the ventral mes(ntery as antutgrowth of the hepatic diverticulum Th~ hepatic divershyticulull induding the biIJary appa~atus develops in tbe ventral mesentery of the foregut

Answer~ A The superior mesenteric ~in joins with the spienkvein to form the hepatic portal vciu

Answer D The structure at gttlK is the proper hepatic artery~ whkh suppUesoxygenated b middotood to the liver

MAKE SURE YOU KNOW the diff bw Rectus Sheath above and below the arcuate line

ABOVE

Aponeurosis of xiiltmal obllque musclo

Extemll f)biquw musde

Reotln ilbdomlnls musole S~in

Internal 9bliquQ mY~QI

AponeUfOsi$ of hJH$V~~S Lir9a a lb lbdolTlin~ musolo Tri OJV6 rUi

atldomlnis mUS(loe

Sub cutanlilous tiue (tatty ye r)

BElOW

A POrl lJfosis 01 etemal oblique muscle

Aponeul~)sis 01 Internal oblique mU$cl~

Anteriol lay~ of r~ltdus st~ath EXttom1 oblique rnu$cll

Rectus Jbdominis muscle Intoernal Aponeurc-sis of tra~fersU$ oblique muscle-

at-domlnis muscentl ~ Skio

Tra nsvitSus abdomioLs ml)ZClt

TralSVersaHs fascia Medial umQil iegtt1 1i9Jment -and folj

Uldchus Peritoneum (ir median Umbilj~al Suboutane ous

Extraprftone 11ascia

Ymbilimiddot~1 fold)

preu9poundiea1 fascia

tissue (fatty 4nd m~mbr3n(iUS layers)

o Above the arcuate line (A horizontal line 13 of the distance bw the umbilicus and the

pubic symphysis) -10 Aponeurosis divides into an AntPost Laminae

o The Ant Laminae joins EO and Post Laminae joins Trans Abdominis = Ant and Post

RECTUS SHEATH respectively

o BElOW the arcuate line - all 3 aponeurosis join ANTERIOR to rectus muscle to meet its

counterpart in the midline (linea Alba)

o Take away Msg - The abdomen is devoid of a posterior rectus sheath below the

arcuate line and is therefore more vulnerable to herniasinjuries

Question - A physician makes a deep incision in the patients midline immediately superior to

the pubic symphysis which of the following layers is his knife least likely to pass

Rectus Abdominis External Oblique Ant Rectus Sheath Posterior Rectus Sheath All of the

Above

Answer - All of the above None of the other answer choices are midline structures -LINEA

ALBA

Linea Alba has very poor blood supply - doesnt heal well after surgery Therefore this is a

common site for incisional hernias

a Spleen b Transverse colon c Descending colon d Stomach e Pleura

17 Meckels diverticulum is normally found 2 feet proximal from the

a Pyloric sphincter b Lower esophageal sphincter c Ileo-cecal valve d Middle valve of Huston e Anal valve

18 Ulcer in the posterior wall of the first part of the duodenum would erode ___ artery and would cause bleeding

a Left gastric b Right gastric c Hepatic artery proper d Gastroduodenal artery e Middle colic artery

19 An inflamed appendix is identified by a surgeon on the operation table by noting

a The appendicies epiploicae b The convergence of tenia c The artery of Drummond d The mesocolon e The mesosalphinx

20 The nerve which emerges through the psoas major is

a Femoral b Ilio-inguinal c Ilio-hypogastric d Pudendal e Subcostal

21 The right gonadal vein drains into the

a Azygos b Hemiazygos c Inferior Vena Cava d Right renal vein e Left renal vein

22 The hepatocytes in the liver is derived from

a Ectoderm b Endoderm c Mesoderm

d Neural ectoderm

23 Abscess in the lumbar vertebrae due to tuberculosis would spread to the adjacent muscle which is

a Psoas Major b Iliacus c Quadratus lumborum d Tranversus Abdominis

24 The anterior wall of the inguinal canal is formed by

a External oblique and transverses abdominis b External oblique and fascia transversalis c Internal oblique and external oblique d Internal oblique and transverses abdominis e Fascia transversalis and peritoneum

Meckels diverticulum is a result of which of the following developmental abnormalities shy

A Failure of the vitelline duct to close

B Failure of the herniated intestinal loop to retract into the abdomen

C Failure of the urachus to close

D Failure of the midgut to rotate

E Failure of the hepatic duct to close

Explanation

Meckels diverticulum is a result of the persistence of the proximal part of the vitelline duct This

diverticulum is usually found about 2 feet proximal to the ileocecal junction and is usually about 2 inches

long It is present in about 2 of the popUlation It may be the site of ectopic pancreatic tissue or gastric

mucosa and may develop inflammatory processes and ulcerations Acute Meckels diverticulitis

simulates appendicitis

Which of the following veins carries blood from the esophagus to the portal vein The

A right gastric vein

B left gastric vein c splenic vein D azygos vein

E left gastroepiploic vein

Explanation

The left gastric vein a direct branch of the portal vein drains blood from the lesser curvature of the

stomach and the inferior portion of the esophagus Because branches of the portal vein do not have

valves blood can flow in a retrograde path when there is an obstruction to flow through the portal system or liveL Rlooci Cln then flow from the nortl] vein thr()1Ph the left PRstric vein to the esonhlPlIS lno

through venous communications within the submucosa of the esophagus to esophageal veins that drain

into the azygos vein The increase in blood flow through the esophageal submucosal veins results in esophageal varices

On the posterior wall of the abdomen the celiac ganglion A contains cell bodies of postganglionic parasympathetic neurons B is synapsed upon by neurons in the posterior vagal trunk C is synapsed upon by neurons in the greater splanchnic nerve D contains sensory cell bodies of lumbar spinal nerves E contains cell bodies of neurons that cause an increase in the rate of peristasis

Explanation The celiac ganglion is one of the preaortic ganglia of the sympathetic nervous system It contains cell bodies of postganglionic sympathetic neurons The sympathetic splanchnic nerves contain preganglionic sympathetic neurons that pass through the sympathetic chain without synapsing These splanchnic nerves go to the preaortic ganglia to synapse The greater splanchnic nerve contains preganglionic neurons from spinal cord segments T5-T9 This nerve synapses in the celiac ganglion The nerve fibers in the vagal trunks are preganglionic parasympathetic fibers that go to the walls of the organs that they will innervate and synapse on postganglionic parasympathetic neurons in the walls of those organs Cell bodies of sensory neurons in the abdomen are found in the dorsal root ganglia or the sensory ganglia of the vagus nerve Sympathetic innervation decreases the rate of peristalsis parasympathetic innervation increases the rate of peristalsis

Which of the following pairs of arteries will allow blood to bypass an occlusion of the celiac trunk

A Left gastric artery-right gastric artery

B Left gastroepiploic artery-right gastroepiploic artery

C Superior pancreaticoduodenal artery-inferior pancreaticoduodenal artery

D Splenic artery-common hepatic artery

E Left gastric artery - proper hepatic artery

Explanation The anastoOlosis of a branch of the celiac trunk and a branch of the superior mesenteric artery will

provide collateral circulation around an occlusion of the celiac trunk Each of the other choices pair

branches of the celiac trunk therefore these will not provide collateral flow around the obstruction of the

celiac trunk The left gastric splenic and common hepatic arteries are direct branches of the celiac trunk

The right gastric artery is a branch of the proper hepatic artery which is a branch of the common hepatic artery The left gastroepiploic artery is a branch of the splenic artery The right gastroepiploic artery is a

branch of the gastroduodenal artery whlch is a branch of the common hepatic artery

Which of the following organs has appendices epiploica The

A sigmoid colon

Bjejunum

C duodenum

D stomach E esophagus

Explanation Appendices epiploica are characteristic of the colon Appendices epiploica are subserosal accumulations

of fat None of the organs of the gastrointestinal tract has appendices epiploica except the colon

Page 25: Chirag's Abdomen Review

In which of the following locations will perforation of the digestive tract result in the spilling of luminal

contents into the - lesser peritoneal sac

A Anterior wall of the second portion of the duodenum B Posterior wall of the second portion of the duodenum

C Anterior wall of the stomach

~Posterior wall of the stomach E Posterior wall of the transverse colon

Explanation

The posterior wall of the stomach is related to the lesser peritoneal sac The anterior wall of the stomach is related to the greater peritoneal sac The anterior wall of the second portion of the duodenum is related to the greater peritoneal sac The posterior wall of the second portion of the duodenum is related to the retroperitoneal space The posterior wall of the transverse colon is related to the greater peritoneal sac

The ureter lies against the anterior surface of which of the following muscles shyA Crus oftne diaphragm B Quadratus lumborum

0 Psoas major D Transversus abdominis

E Iliacus

Explanation The ureter exits the renal pelvis at about the level of vertebra L2 As it descends along the posterior abdominal wall it lies on the anterior surface of the psoas major The psoas major muscle arises from the bodies of the lower lumbar vertebrae The psoas major muscle is joined by the iliacus to fonn the

iliopsoas muscle The iliopsoas muscle then attaches to the lesser trochanter of the femur and is the major

flexor of the hip

As the right ureter passes the pelvic brim it lies against the anterior surface of which of the following

blood vessels

A Gonadal artery B Inferiorvena cava C Internal iliac artery

rJ- External Iliac artery

E Inferior mesenteric artery

Explanation

The ureter lies in the extraperitoneal space in the posterior abdominal wall Alter leaving the kidney it

passes inferiorly on the anterior surface of the psoas major muscle At the pelvic brim the ureter passes

into the pelvis At this point the common iliac artery is dividing into the external and iliac arteries The

ureter lies on the anterior surface of the external iliac artery immediately distal to the bifurcation This is a useful landmark for a surgeon to locate the ureter

When extravasated urine passes from the superficial perineal space into the anterior abdominal wall it is

found immediately deep to which of the following layers of the anterior abdominal wall

-ltScarpas fascia

B External oblique muscle

C Internal oblique muscle D Transversus abdominis muscle

E Transversalis fascia

Explanation

The superficial perineal space is bound by Colles fascia the fibrous portion of the superficial fascia This

layer of fascia is continuous with Scarpas fascia the fibrous portion of the superficial fascia of the anterior abdominal wall Therefore urine that is deep to Colles fascia will remain deep to Scarpa s fascia The urine will spread in the plane between Scarpas fascia and the external oblique layer

When a horseshoe kidney develops the ascent of the kidney is restricted by the A internal iliac artery B external Iliac artery

C common iliac artery

inferior mesenteric artery

E superior mesenteric artery

Explanation

A horseshoe kidney develops when the inferior poles of the to kidneys fuse together as they ascend into

the abdomen from the pelvis The first anterior midline vessel that is encountered by the horseshoe kidney

is the inferior mesenteric artery This artery prevents the kidney from continuing its ascent

The left testicular vein drains into which of the following veins

A Left internal iliac vein B Left common iliac vein

bflnferior vena cava D Left renal vein I

E Left internal pudendal vein

Explanation

The left testicular vein drains into the left renal vein The right testicular ~i~[~nsltjectlY into the

inferior vena cava This difference in venous drainage is believed to explain the greater incidence of

varicocele on the left side than on the right The venous drainage from the penis is to the internal vein

which then drains into the internal Iliac vein

The spinal nerve that provides cutaneous branches to the skin around the umbilicus is

A TS B TW-shy

C TI2

DL2 EtA

Explanation

The tenth intercostal nerve is the anterior ramus of the TIO spinal nerve After passing through the tenth

intercostal space the nerve continues forward in the anterolateral abdominal wall in the plane between

the internal oblique muscle and the transversus abdominis muscle In the abdominal wall the nerve innervates to the abdominal wall muscles as well as the skin and the parietal peritoneum The umbilicus is

a useful landmark for the region of distribution of the tenth thoracic nerve

The ligament of the vertebral column that resists its extension is the Aligamentum flavum

B supraspinous ligament

C posterior longitudinal ligament

D anterior longitudinal ligament

E interspinous ligament

Explanation

The ligaments of the vertebral column that resist flexion of the column include the supraspinous ligament

interspinous ligament ligamentum fiavum and posterior longitudinal ligament The ligament that resists

extension is the anterior longitudinal ligament This longitudinal ligament is very broad and strong It

covers the anterior and anterolateral surfaces of the vertebral bodies and the intervertebral disks In

addition to resisting extension the anterior longitudinal ligament provides reinforcement to the anterior

and anterolateral surfaces of the intervertebral disk The posterior longitudinal ligament is relatively

narrow and covers the posterior surface of the vertebral bodies and the intervertebral disks This ligament

reinforces the posterior surface of the disk The posterolateral surface of the disk is not reinforced and it

is through this region that herniation of the nucleus pulposus usually occurs

A patient presents with epigastric and right upper quadrant pain The pain is most intense 2-4 hours after

eating and is reduced by the ingestion of antacids The patient states that he has passed black tarry stools

(melena) within the last week Fiberoptic endoscopy reveals a yellowish crater surrounded by a rim of

erythema that is 3 cm distal to the pylorus Accordingly an ulcer has been identified in the patients

A fundus

B antrum

C duodenum

D jejunum

E ileum

A number of physiologic genetic and other factors increase the risk of gastric (and duodenal) peptic

ulcers The evidence that H pylori plays a principle role is compelling Smoking and caffeine are known to adversely affect the morbidity mortality and healing rates of peptic ulcers In general first-degree

relatives of peptic ulcer patients as well as males have a threefold to fourfold increased risk of developing this disorder Paradoxically in gastric ulcer disease acid secretion is not elevated It is possible that

excess secreted hydrogen ion is reabsorbed across the injured gastric mucosa In general a defect in gastric mucosal defense is the more important local physiologic

A patient presents with symptoms of duodenal obstruction caused by an annular pancreas Annular pancreas is caused by

A rotation of the dorsal pancreatic bud into the ventral mesentery B rotation of the ventral pancreatic bud into the dorsal mesentery

fJ failure of the major and minor pancreatic ducts to fuse ~ ~ cleavage of the ventral pancreatic bud and rotation of the two portions in opposite directions around -the duodenum E formation of one pancreatic bud instead of two

Explanation Normally the ventral pancreatic bud rotates around the gut tube to reach the dorsal pancreatic bud The two buds fuse to form a single pancreas and the distal portions of the two ducts fuse The ventral pancreatic bud forms the inferior portion of the head of the pancreas the uncinate process and the major pancreatic duct (of Wirsung) The dorsal pancreatic bud forms the superior part of the head the neck body and tail and the minor pancreatic duct (of Santorini) Annular pancreas is the result of the ventral pancreatic bud dividing into two portions before it rotates into the dorsal mesentery Each portion rotates in opposite directions to get to the dorsal mesentery thus encircling the duodenum The presence of annular pancreas can constrict the duodenum thus obstructing its lumen

In n _ phranlc----

Gon ~l ----_1 Lum bltano

~~--- CornmQ1t bull ac

+-~4--- lnlllirnaJ ilic

xtem iliac

OBJECTIVE - Identify the blood supply to each of the structures listed in the table on the previous page

Ill give you a head start

FOREGUT - Supplied bV Celiac Tru nk (T12)

Proper hepatic

GastiooUod 13Jafter

1nferlor pancreaticoduodenal artery

Common epatlc

Lett gas ric iiirtery

Spfen artery

shy Gastroepiphgtic artery

~ Superior mesenteric 8rtfry

~

1 Esophagus is a derivative of the foregut so its blood supply originates from the celiac trunk

(T12) The predominant blood supply to abdominal portion of the esophagus is the Esophageal

A (Branch of L Gastric) The venous drainage of the esophagus is particularly important because

it is 1 of 3 clinically relevant sites of Portal Caval anastamoses The Portal Esophageal Vein

meets the Caval Azygos System Persistent bleeding manifests as Esophageal Varices - a fata I

condition

2 The Stomach is also a derivative of the foregut has EXTENSIVE blood supply and is very high

yield on anatomy exams The lesser curvature is supplied superiorly by the L Gastric A (1 of 3

major branches ofthe Celiac trunk) and inferiorly by the R Gastric A ( a branch ofthe proper

Hepatic A) The greater curvature is supplied superiorly by the L Gastroepiploic A (a major

branch of the splenic A) and inferiorly by the R Gastroepiploic A

The Short Gastric arteries (branches of Splenic Artery) supply the fundus of the stomach and

are referred to as EIID ARTERIES because they have no collateral blood supply Therefore if the

splenic artery were occluded (ex - increased pressure in the ommental bursa) - there would be

ischemia to the fundus of the stomach Venous drainage of the stomach is extensive via various

veins lead ing to the portal system Posterior to the stomach the IMV joins the splenic V which

joins the SMV to form the PORTAL VEIN ADAMS

3 Duodenum blood supply has high clinical relevance because it is the junction of the foregut and

midgut and therefore is the site of anastamoses between branches ofthe Celiac Trunk (main

foregut artery) and the Superior Messenteric Artery (main midgut artery) The Proper hepatic

artery gives off the gastroduodenal artery which travels behind the 1st part of the duodenum

This point has high clin ical relevance because duodenal ulcers are very common and a posterior

rupture of the 1st part of the duodenum could rupture the gastroduodenal artery causing

traumatic abdominal bleeding The Gastroduodenal artery first gives off the R Gastroepiploic A

(mentioned above) and proceeds as the Superior pancreatico duodenal artery (supplies the

pancreas and duodenum) which anastamoses with the inferior pancreatico duodenal A (branch

of the SMA) This is the junction of foregut and midgut and occurs near the opening of the

bil iary system into the duodenum (ampula of vater) Portal venous drainage here is responsible

for delivering nutrients from digestion to the liver for metabolism Appreciate that the Superior

mesenteric artery (artery of the midgut) branches from the aorta at Ll travels posterior to the

pancreas than moves anteriorly (at the jxn of the pancreatic headbody) and comes over the

3rd4th part of the duodenum Tumor of the head of the pancreas can compress the SMA

4 Jiver blood supply is via the common hepatic artery (major branch of the cel iac trunk) The

common hepatiC becomes the proper hepatic gives off the R gastric A and the Gastroduodenal

A and then joins the common bile duct and the portal vein in the portal triad Clinical- if a

patient were bleeding from the hepatic A a surgeon can stick his fingers in the epiplOic foramen

and squeeze the free edge of the hepatoduodenalligament in order to stop bleeding to the

area Please note that the hepatic a branches into Rand L hepatic A The Right hepatic artery

gives off the cystic artery which supplies the gallbladder Afferent venous supply is via the

Portal vein which is bringing nutrient rich blood to the liver After metabolism takes place

venous blood leaves the liver through the hepatic veins into the IVC PLEASE UNDERSTAND THE

RELATIONSHIP OF THESE STRUCTURES - ADAMSNETIERSNH Etc

5 Pancreas - Head is supplied via the superior and inferior pancreaticoduodenal arteries

(mentioned above) The tail (situated towards the hilum of the spleen) is supplied via the

pancreatic branches of the splenic artery (END ARTERIES) This blood supply is very important

because the endocrine Alpha and Beta Cells from the pancreatic islets of lagerhans are located

towards the tail This is where Insulin and Glucagon is released to the blood

Now complete this for mid and hindgut structures Make sure to note clinically relevant arterial

anastomoses as well as portal caval anastomoses FYI Appendix blood supply SMA + IMA

anastamoses marginal artery Portalcaval rectal veins fhemmorhoids) and periumbilical caput

medusa are high yield THE BUTT THE GUT and THE CAPUT

Abdominal Development

Liver

Ij1f

II wall b

oh liN ~ VltJrti n be- bull

Pancreas

Secondary Retroperitonealization e I~tl r 1 a v-mtrai m ellter

Rotations of the Gut I i Ij (lIl1UtIJ f~ l r tilt

()l td 10 me l-ft and he v

--~--- -~ -~-~

i

I AolaijonjoI~guf I

STOMACH BED (IDENTIFY IN ADAMS)- the structures posterior to the ommental bursa which

support the stomach in the supine position

Abdomnal JQrUI

Splnic vein

OmQ-oul tv~ ) O(s(Jroa)

Lojt(r o m nturrt (hpJtodu o d~n31 Hid

Gadrl)SplerH (g3stroll~nal) IIgam~nt

hiad h~~atogrtricent IIQdmiddotcrt~)

Lt Dome of Diaphragm (why left Look this up in Adams)

Spleen (What is the blood supply)

Left Kidney (What is the blood supply - AND how is it different from the R kidney)

Suprarenal Gland (What is the Arterial AND Venous Blood supply - how are they different)

Pancreas (How does supply differ from Head to Tail What is the SMA Relationship)

Transverse Mesocolon

liver - ADAMSWET - Make sure you look at the liver in wet lab

Left triangular nl1am~nt

ComoaDj ligamnt

Erophg~1 impre$ioo

Hepatio veins

In1erior -ifena middotr3)Ia

Fibrous appendix o-t

live

impr~j on

Heprorendl p~rtion of Q)(Qllary ligament

Righllri~n9ul r 1I~met

(Common) bile quol

Gr)mmCtr~ hepatic dlJct

Ccentic duct

Duodenal impression

GaJdate p-fr)~S

Hepatic artgtrl prop-f iiiiila - Faloiform ligament

_ - shy Round ligamen liver

~--F-- CoJio imprgt-ssi-on

Prta heptis

Identify the lobes impressions and embryonic remnants associated with the liver

Caudate Lobe Quadrate Lobe Right Lobe Left Lobe Round ligament Falciform Ligament

Ligamentum Venosum (what is its fxn in embryonic life) Hepatic Veins (NOT PART OF THE

PORTAL TRIAD) IVC PORTAL TRIAD - Contents relationship cross section etc Know the

Galbladder relationship to the lobes of the liver

Biliary Duct System - Make sure you understand the sequence of these structures - BE ABLE TO

DRAW A FLOW CHART

TPVd i

t

I t

1 __ Cm-(r

patk GlJet

I

J

Clinical = JAUNDICE is caused by anything that prevents delivery of bile to intestine Tumor of the

head of the pancreas Stones etc Patient will have pale stools and yellowish colored mucus

membranes

Clinical- Any scenario that tells you the patient has BILLOUS VOMIT means that the obstruction to

the flow of digestive contents is after the Ampulla of Vater (Site of Entry of Billiary system to the

duodenum) - ie Duodenal Atresia

Spleen -located posterior to the mid axillary line between ribs 9 and 11 Make sure you know that

the 10th rib is the main axis of the spleen and this organ is susceptible to injury (stab wound errant

thoracoce ntesis etc)

The spleen is derived from mesodermal cells - NOT THE GUT TUBE

The spleen rests on the left colic flexure associates with the tail of the pancreas Know the

structures entering the Hilum of the spleen

Sh rt O~-t~ic 1 0(0 10 rtiltSPIric Iloa nt

(cut)

Peritoneum - similar concept to Pleura - think of a fist in a balloon

Visceral Peritoneum - Layer of balloon touching your fist

Parietal Peritoneum - Layer of balloon not touching your fist

Your fist represents the organ your wrist is the hilum and your arm contains the blood supply

entering the organ

Appreciate that there will never be organs in the peritoneal cavity - rather these organs invaginate

the cavity Kaplan videos

RULES OF NOMENCLATUREshy

1 Organ completely surrounded by peritoneum - peritoneal organ

2 Organ partially surrounded by peritoneum- Retroperitoneal

3 Peritoneum surrounding peritoneal organ is VISCERAL peritoneum

4 Peritoneum surrounding retroperitoneal organ is PARIETAL peritoneum

5 Peritoneum connecting visceral to parietal is called messentary 2 messentaries in the

gut Dorsal (to the gut tube) and ventral (to the gut tube) messentary

Aorta is in Retro peritoneal position - but blood must reach peritoneal position - vessels travel through

messentary All peritoneal organs will have blood supply reaching through messentary

-Mesentery is a 2 layer peritoneum with a neurovascular communication between body wall and organ

- Ligament connects one organ with another or to the abdominal wall (Ommentum = ligament)

lesser Ommentum (attach lesser curvature of stomach and duodenum to liver) =Hepatoduodenal

Ligament and Hepatogastric Ligament

Has a Superior and Inferior Recess (Accumulation of Fluid in Ascites)

Communicates with the greater sac through the epiplic foramen (what structures pass through

this foramen)

Boundaries - you must be able to visualize this

o Anterior - stomach

o Posterior - parietal peritoneum pancreas

o Superior - superior recess (bw diaphragm and coronary ligament)

o Inferior -Inferior recess (bw layers or greater momentum

Greater Ommentum (attach greater curvature of stomach) Gastrophrenic ligament Gastrosplenic

ligament gastrocolic ligament

The greater omentum is the largest peritoneal fold It consists of a double sheet of peritoneum folded on itself so that it is made up of four layers The two layers which descend from the greater curvature of the stomach and commencement of the duodenum pass in front of the small intestines sometimes as low down as the pelvis they then turn upon themselves and ascend again as far as the transverse colon where they separate and enclose that part of the intestine

ABDOMINAL PAIN

Parietal Peritoneum - supplied by same vasculature lymphatics and nerves supplying body wall it

lines and diaphragm Sensitive to pain pressure heat cold well localized

Visceral Peritoneum - supplied by same vasculature lymphatics and somatic nerve of organ it covers

Insensitive to touch heat cold and laceration - referred to dermatome of spinal ganglia providing

sensory fibers Where does appendicitis refer to

Foregut pain - epigastric area (ie - cholycystitis)

Midgut pain - periumbilical area (ie - appendicitis)

Hindgut Pain - suprapubic area (ie - diverticulitis)

Extra ImagesConcepts

ll~_____-

FalifCtrm ligament oind r~ud ligamet f Ilver

Blood from splenio gastriC and inferiof rne$e-rteri v~ins

Ca-I tributaries

Lett gastrio Ifein

Posterior superior pan~reatioodul)denal vaihS

Lott gamo-om~nlal (9aropip lomiddotic) -in

Poq_~ tjol imerl-9-r panCJertlcorllJod-nal veiopound --amp----I- - ~J Right grtr~-omntal

Anwrior interi (gartroepiploic) Jjn

pan euaii cod vl)denal veins middot Inf~Ji (t r mesentric vein

Miqdle (olic vein

Right cl)licvein Sigmoid and rectosigml)id (ei ns

IhH)Collc(~io

--- Mi~dl laquooLJl gtjrltgt

PoM ca vl1 illasto)moses -----shyampoptoageal 2 Paraumbilie-lt11 Inferi or Fectal vei ns

3 Recial 4 REuoperHonea1

Know how the Portal vein is formed I 4 sites of portal caval anastamoses and 1 clinical shunt

Col li t ltt-~ otTl~tI ~nj pc~ 1lt1 turJoG

Ltf 14i1 tImiddot~ artoftl9 on tj phtAt$

L-oftqf 4t t~r 1=laquoIran d 1 bull shy~p l ci rj o fOOOts

Nerves follow the arteries - appreciate the splanchnic nervous system I

Uet~ric branch of left ~nal art

Ureterie branch of righi renal artelY

Left Zld lumbar in and co mlTlunication to as)erdin9 lumbar l(~in Hi ~ht tEZ1~~t~ t3r j t itn ~ nJ l1t- rlnd lfe i r1

Inferior me5nteri~ artery

Notice that the right testicular vein drains directly into the IVC and the right testicular artery drains

directly into the aorta However the left testicular vein drains into the L renal vein at a right angleshy

reason left testicle is lower and more susceptible to varicocele (bag of worms)

Also notice that the left renal vein has a longer course because the IVC is on the right side whereas

the right renal artery has a longer course because the aorta is on the left side

Appreciate the anterior to posterior relationship of structures in the hilum of the kidney - VAP - Vein

Artery Renal Pelvis (Ureter)

11____ __ L_ L_ n VJ __ _ _ t_L I I_ _ L __ L_ I -pound1 bull LI_~-I ____

Posterior View of Head of Pancreas in ( of Duodenum

Celiao hunk

Co mmon ~L~jJth art~ry

GastNduQdonal artrf (partilly in phantn)

P1)Sterior $Up~Jior panCflaticuduodfmal art~r~t

(Co mm on) bile duct

middot~1t~~t-1l---~-~- Right gshomiddotomental (gastoe plp lolc) 3rte (phantomost)

Grener paocre atic art-ry

1n1~rjor pancr-iatlc artery

Jtrifll supejo r pal)oreailcento)dJodenal artr1 (phantom)

Anastomotlo branch

POostetlor bJanch of jo f~ri of pan-reatir(lduodensl drttnj

Anterio r branch of i flferior palcreati~)duodenal art~(phan1om)

Notice the extensive blood supply to the pancreas and duodenum via the branches of the celiac trunk

Notice collateral supply from SMA branches - makes sense bc this is the jxn of foregutmidgut

Identify the vessels in this arteriogram

Hiltid i)f N~ck oi B)dvof Tail 01 pa nereas pan cent~as P-nmiddot-reas panCtCas

I nferie v~na cava

jHept1iic p(lrlai v~in

Port1 tnd H~pti lt a ftH prol

Comm on) bll duct

Ouodtnum

~ft colic (sio)Atta~ hmtrlt jt~xJr-ofha~elSe

muo(IIQn

Right ~lIc (h~j)tic)

il~gtture

In1triol m~oten lIein (rttr op~ritoMdO

SlJp efl or mes~n~fiC amrV and lipln

KNOW YOUR NEIGHBORHOOD

Questions

vVhiJh structure supplied by a bnmdlof the cclia( artery is not derivcd from foregut LemCJUCrITI

(A) Head of the pancte-a5

CD) Pyloric duolenum

Cystkduct

( Liver hepatocyt~~

~F) Body of the spleen

An infant presents with an omrhaJucele at birth -hi oJ the [oHm illg applies to his cM1-dition

(A) It is 31so seen ill p4titnts with aganghonic megacolon

(11) ft reuirs from a fal1ure of resorption of theviteUine d let

(C) It results from herniation at the-site of regression of the right umbilk vein

DJ It is caustd by faihtrc of recanalization of the midgut part of the duodenum

~ It ill camioo by a failuIt vf the midgul to return to the abGQminal uity after herniashytion in-n the urnbilk s l stalk

Ot er than the spleen occlusion Cif the spit-Ilk artery at its odgin wm most likely affect die blood supply to jllch st cnud

(A) Jejunum

(B) Body of th pal1~lltas

(C) LeSStT Cllmiddotlaturc of tl )toma-ch

(D Duodenum dista to the entrance of the Ornmou bile duct

E Fundus of the stomach

A 38-yeu-old batL~er with a history of heartburn suddenly experiences excluciating pain in the (plgastric region of th~ abdomeu SurgCry is perf~rme immediard y upon admisshysion to the 1IlcrgCJliy tuomh~re i~ evidence uf a ruptured ulcer in the posterior waU of the stomach Vhere will a surgeon first fi nd the stomach contenlSf

A) Greater p4ritoneal sac

rB) Cul~de-s~c of Douglas (--

C Omental bursa ~

--D) Paracolic gutter

rEj Between -he panttal perimltum and the posterior body wal1

At birth an infant presents with a st()ma~ rb~tbas~njJled jfltotb~diaplfagru 1A1ltre is the defect thatresulied iiitJle heini~t()n shy~tsophagealbiatus

7 - rH-- Hiatus for the inferior vena cava

( Pleuroperitoneal membrane -(0) Septum transvcrsum

(E) Right Crlt~

An infant born with DOVv7l syndrome presents with bili()u~ vomiting Ahat congenital defect does the infant have

(A) Pyloric stenosis

(B) Meckel diverticulum C) Ornphaloce1e

(D) Gastroschisis

( ~ ) Duodenal atresia y A patient with cirrhosis of the liver presents with ~ bacalvaricestnlreased retrograde pressure in which veins caused the varices

(A) Paraumuilical

(B) Splenic

(ct AzygltJus

(15))G~trk ( (-F) Superior mesemeric

A htaltby 3-year~old male patient experiences a hernial sa protruding from the anterior abdominal wall about halfway between me anterior superior ilia spine and the pubk tuberde Pulsations of al1 artery are palpated medial to the protrusion site through the abdominal walL Which layer of the anterior abdominal wall will first be traversed by the

1hctma

fA) Rectus sheath (B) External oblique aponeurosis

(C) Inguinal ligament

lD) Transversalis fusda

(E) Cremasteric fa~cia

After 5urgi(aj ffpair of a hernia the patient tXperienccs mtmlgtness in the skin on the anteshyrior aspect of the S(Totum_ Vhaf nerve may have been lesioned during thehemiorrhaphy

(A) Femoral

(B) Obturator

(C) Ilioinguinal

(D) lliohypogastrk

(E) Pudendal

A 23~year-LJld female secretary il1 good health ~-uddcn1) doubles over with pain in the a ea of the 1JmbRicu$ Sbe feels vartn and ltneasy and has no appetite That night the pain seems to have mQved to the tower right abdominal regjol1 and she calls her family doctor who then arranges for an ambulance to pk-k her up and take her to the hospitaL Wh ell ntn~ perceived in the area of the urnbilirus most Hkely carried lhe pairfu I sensations into the eNS

tA) Vagus nerves I~

V B)

) Lessersplanchnk nerves

tC) Pudendal nerves

(D) lIiohpogastrk nerves

(E) Greater splam ic l erves

A CT reveals carcinoma in the bOod of the ancreas Vhich blood vessel trut ourses ----~- - -bull ------ --shy

immediately poftterior to the body ofthe pancreas is the m~t likely to be oompressed

(A) Splenk artery

(B) Abdominal aorta (C) Portal vein

(1) Splenic vein

(E) Renal vein

A patient has a penrln1l1ng uker of the posterior wall ot the br~l part ot the (lUooenmn llkh blood vessel is subject to erosion

(A) Common hepatic artery

(B) Gastroouodenal artery

(C) Proper hevatic artery

(D) Celiac artery

(E) Anterior inferior 11amrelltlcoduodcnal attery

Your patient has been diagnosed -ith a carcinoma locallted to the head and l~e(k of the pancreas Another clinical sign would be

A esophageal varices

(8) hemorrhoids

C) a caput medusa

(D) increased pra Teuro n th~ hepatic veins

(E) enlarged right supra lavkular lymph nodes

Wltkh of the foUowing structures develops in the ventral mesentery

(A) Spleen

(B) Jeiunum (C) Head of1ht pancreas (D) Transverse colon (E) Stomach

ti l Uw ~ littwin~ f( S-t lil oai Imdge ~ hi(h or tbt la~)d J truetur tgt liJ llntn nl) he hl p UC iJd [IIi ell

c o

A) drains Ie tht infCrior a La aI

R t middot~nfl0 ~ill to th~ lunlgtn of h i dtlndCrlllfH

(e) m t bull JiJattd on tl l J n T ~H

D ) sup Lc O VSlt I Hlid bhtu l 1 li - -I un oid

( ) U~tpli(t tr j middottUh~ 1 v(( b~nt rfK n1ilc~Zm

ANSWERS AND EXPLANATIONS

Answer E The spleen is t hlttnopodicand lymph organ demlted from mesoderm

Answ~ R Al1 tlmphalocele is caused by it failure of the nlidgut to return to the ahdomir nat cavity after herniation into the umbiliau Stalk Choices Aand D maybe seen in infants with Down syndrome choice D ~s the specific CBuse ofduudcnal JtiCSitt Choice C is (ile cause of gclstrosbisis and Choice B nsults iu a Meurolktldivertku1-tlB

Answer B The fundus ofthe stomach is suppHed by soort gastric brunches of the splenic altery The splenic artery supplies the body and tail of the pancreas part of the greater curvature of the sttmla(h and the spleen Te jejunum part of the head of the pancreas and tht~ duodenum distal to the entrance of the commOll bile duct are supplied by the superior mesenterk artery clll~l ~be less r ctlt1ature cmd the pylQric antrum are supplied by the right and lei gastric art(ries

AnSWftt C Tbeomental bursa or lesser ~ritoneaj sac lies direcdy posterior to the proxshyimal part of the duodeTtlm and the stomach and would be the first site where stomach contents ~Ott1d be fpoundluncL

Answer C A defect in a llleuropcritoneal membrane (uswlly the left) is the typical site of i1 cc-ngenitlI diilphragluatic hemia llere the membr4ne fails to dose ()pound( of the perishycCirdiopcritulleal canals

Answer E DuoJenal atresia and aganglionic megacoion are congwitaI defects S~Il in patients with Dowmiddotnsyndrome

Answer D RulaTgemt~llt of and retrograde flow in g~lstrk vel_ns in particlJl~r the kft gas~ tricveins dilates the capillary bed in rhe wall of the esophagus in (ases of porta yper~

tension Blood flow would increase in and dilampte tribntarkgts of the (lZygOUS vein on the other side of the capiUary bed but flow in this vein is in the typical direction t()ward the superior vena cava Paraumbiii(ltU vein eilgorgement contributes to a caput medusH Splenic ~nlargement might prc~nt with 5plcnonlegaly and balt-kflow in to tlu superior m~~ntclic vein occurs but is asymptomatic

Answer D The patient hagt an indirect inguinal hernia whi~h emerges from the antt-rior abdominal wall through the deep inguinltilling Theeep ring is a fault in the transv~rshysaUs fascia this I~yer wiIJ be penetrated first by the hernia

An~Wer C The ilioinguinal nenc which provides sens~llion to the lnedlal thigh ltmclanteshytior SClotunl pass~lt th rough the 5uperfh_ial inguinal ring ind $subject to inj i1T) becaus-e

it is in the operatitm Held of the erniorrhapny

Auswer B The leMHr splanchnic nerves are sympathdic nerVlts that carry viscera l sensashytlltgtrogt ftom illtllt1m~d ()J stietched gust (itinteitinal ~tructures (in this case the pprndix) into tnt eNS Lesser splanchnic ntTYcsarisc from thmiddot T9--T12 spinal cord segments lt1nd provide sympathetic innenation tD rnidgut siruc1ures whiCh include CLe app~JldD Viscera] Pain arising from affecLed Inidgut ampt 1C1ure is referred over the same dl- matorne~ of spinal segrnertts v-hich provide the sympathetic Innervation n this G1SC of appendicitis the invohen~n t of the ltire) of t e unlhHku indud s the T 10 dermatome

Answer B Of the five choices onty the dscending olon is retroperiton~al aldwould be a lik ~ ( choice to be seen immediately a(~jilcent to t11e posterior abdominal middotn~L

Amwen D The SpltftlC ~-ein ourses posterior to the body of the panneas m its way tt drain into the superior mCSfttltlri( vein

Answcr B TILt glstrodllolticnal artery 1 direct hIamh of the comrootl hepatic artery courses immediately pt))iwri() to the duodenum and is slbject to erosion

Answer B Carcinoma of th pan middott3S in the 1tilt1 may compreampgt the portltil vein at irs orishygill The poTtai vcin is fomled when the splenic vein jQiaswith tfie superior meStllt eric vein The inferiot mesenteric vein joins the ~plenjc vein just priOT to tlli~ point at which the splenic joins the superior Jlleit1ltcri( vein Increescd venous presslu in the inferior mesenteric vein is a cause of emo hoid~

Answer C The- velltral pancreas wilich forms most of the head of the p ~ncr as develops in the ventral mes(ntery as antutgrowth of the hepatic diverticulum Th~ hepatic divershyticulull induding the biIJary appa~atus develops in tbe ventral mesentery of the foregut

Answer~ A The superior mesenteric ~in joins with the spienkvein to form the hepatic portal vciu

Answer D The structure at gttlK is the proper hepatic artery~ whkh suppUesoxygenated b middotood to the liver

MAKE SURE YOU KNOW the diff bw Rectus Sheath above and below the arcuate line

ABOVE

Aponeurosis of xiiltmal obllque musclo

Extemll f)biquw musde

Reotln ilbdomlnls musole S~in

Internal 9bliquQ mY~QI

AponeUfOsi$ of hJH$V~~S Lir9a a lb lbdolTlin~ musolo Tri OJV6 rUi

atldomlnis mUS(loe

Sub cutanlilous tiue (tatty ye r)

BElOW

A POrl lJfosis 01 etemal oblique muscle

Aponeul~)sis 01 Internal oblique mU$cl~

Anteriol lay~ of r~ltdus st~ath EXttom1 oblique rnu$cll

Rectus Jbdominis muscle Intoernal Aponeurc-sis of tra~fersU$ oblique muscle-

at-domlnis muscentl ~ Skio

Tra nsvitSus abdomioLs ml)ZClt

TralSVersaHs fascia Medial umQil iegtt1 1i9Jment -and folj

Uldchus Peritoneum (ir median Umbilj~al Suboutane ous

Extraprftone 11ascia

Ymbilimiddot~1 fold)

preu9poundiea1 fascia

tissue (fatty 4nd m~mbr3n(iUS layers)

o Above the arcuate line (A horizontal line 13 of the distance bw the umbilicus and the

pubic symphysis) -10 Aponeurosis divides into an AntPost Laminae

o The Ant Laminae joins EO and Post Laminae joins Trans Abdominis = Ant and Post

RECTUS SHEATH respectively

o BElOW the arcuate line - all 3 aponeurosis join ANTERIOR to rectus muscle to meet its

counterpart in the midline (linea Alba)

o Take away Msg - The abdomen is devoid of a posterior rectus sheath below the

arcuate line and is therefore more vulnerable to herniasinjuries

Question - A physician makes a deep incision in the patients midline immediately superior to

the pubic symphysis which of the following layers is his knife least likely to pass

Rectus Abdominis External Oblique Ant Rectus Sheath Posterior Rectus Sheath All of the

Above

Answer - All of the above None of the other answer choices are midline structures -LINEA

ALBA

Linea Alba has very poor blood supply - doesnt heal well after surgery Therefore this is a

common site for incisional hernias

a Spleen b Transverse colon c Descending colon d Stomach e Pleura

17 Meckels diverticulum is normally found 2 feet proximal from the

a Pyloric sphincter b Lower esophageal sphincter c Ileo-cecal valve d Middle valve of Huston e Anal valve

18 Ulcer in the posterior wall of the first part of the duodenum would erode ___ artery and would cause bleeding

a Left gastric b Right gastric c Hepatic artery proper d Gastroduodenal artery e Middle colic artery

19 An inflamed appendix is identified by a surgeon on the operation table by noting

a The appendicies epiploicae b The convergence of tenia c The artery of Drummond d The mesocolon e The mesosalphinx

20 The nerve which emerges through the psoas major is

a Femoral b Ilio-inguinal c Ilio-hypogastric d Pudendal e Subcostal

21 The right gonadal vein drains into the

a Azygos b Hemiazygos c Inferior Vena Cava d Right renal vein e Left renal vein

22 The hepatocytes in the liver is derived from

a Ectoderm b Endoderm c Mesoderm

d Neural ectoderm

23 Abscess in the lumbar vertebrae due to tuberculosis would spread to the adjacent muscle which is

a Psoas Major b Iliacus c Quadratus lumborum d Tranversus Abdominis

24 The anterior wall of the inguinal canal is formed by

a External oblique and transverses abdominis b External oblique and fascia transversalis c Internal oblique and external oblique d Internal oblique and transverses abdominis e Fascia transversalis and peritoneum

Meckels diverticulum is a result of which of the following developmental abnormalities shy

A Failure of the vitelline duct to close

B Failure of the herniated intestinal loop to retract into the abdomen

C Failure of the urachus to close

D Failure of the midgut to rotate

E Failure of the hepatic duct to close

Explanation

Meckels diverticulum is a result of the persistence of the proximal part of the vitelline duct This

diverticulum is usually found about 2 feet proximal to the ileocecal junction and is usually about 2 inches

long It is present in about 2 of the popUlation It may be the site of ectopic pancreatic tissue or gastric

mucosa and may develop inflammatory processes and ulcerations Acute Meckels diverticulitis

simulates appendicitis

Which of the following veins carries blood from the esophagus to the portal vein The

A right gastric vein

B left gastric vein c splenic vein D azygos vein

E left gastroepiploic vein

Explanation

The left gastric vein a direct branch of the portal vein drains blood from the lesser curvature of the

stomach and the inferior portion of the esophagus Because branches of the portal vein do not have

valves blood can flow in a retrograde path when there is an obstruction to flow through the portal system or liveL Rlooci Cln then flow from the nortl] vein thr()1Ph the left PRstric vein to the esonhlPlIS lno

through venous communications within the submucosa of the esophagus to esophageal veins that drain

into the azygos vein The increase in blood flow through the esophageal submucosal veins results in esophageal varices

On the posterior wall of the abdomen the celiac ganglion A contains cell bodies of postganglionic parasympathetic neurons B is synapsed upon by neurons in the posterior vagal trunk C is synapsed upon by neurons in the greater splanchnic nerve D contains sensory cell bodies of lumbar spinal nerves E contains cell bodies of neurons that cause an increase in the rate of peristasis

Explanation The celiac ganglion is one of the preaortic ganglia of the sympathetic nervous system It contains cell bodies of postganglionic sympathetic neurons The sympathetic splanchnic nerves contain preganglionic sympathetic neurons that pass through the sympathetic chain without synapsing These splanchnic nerves go to the preaortic ganglia to synapse The greater splanchnic nerve contains preganglionic neurons from spinal cord segments T5-T9 This nerve synapses in the celiac ganglion The nerve fibers in the vagal trunks are preganglionic parasympathetic fibers that go to the walls of the organs that they will innervate and synapse on postganglionic parasympathetic neurons in the walls of those organs Cell bodies of sensory neurons in the abdomen are found in the dorsal root ganglia or the sensory ganglia of the vagus nerve Sympathetic innervation decreases the rate of peristalsis parasympathetic innervation increases the rate of peristalsis

Which of the following pairs of arteries will allow blood to bypass an occlusion of the celiac trunk

A Left gastric artery-right gastric artery

B Left gastroepiploic artery-right gastroepiploic artery

C Superior pancreaticoduodenal artery-inferior pancreaticoduodenal artery

D Splenic artery-common hepatic artery

E Left gastric artery - proper hepatic artery

Explanation The anastoOlosis of a branch of the celiac trunk and a branch of the superior mesenteric artery will

provide collateral circulation around an occlusion of the celiac trunk Each of the other choices pair

branches of the celiac trunk therefore these will not provide collateral flow around the obstruction of the

celiac trunk The left gastric splenic and common hepatic arteries are direct branches of the celiac trunk

The right gastric artery is a branch of the proper hepatic artery which is a branch of the common hepatic artery The left gastroepiploic artery is a branch of the splenic artery The right gastroepiploic artery is a

branch of the gastroduodenal artery whlch is a branch of the common hepatic artery

Which of the following organs has appendices epiploica The

A sigmoid colon

Bjejunum

C duodenum

D stomach E esophagus

Explanation Appendices epiploica are characteristic of the colon Appendices epiploica are subserosal accumulations

of fat None of the organs of the gastrointestinal tract has appendices epiploica except the colon

Page 26: Chirag's Abdomen Review

When extravasated urine passes from the superficial perineal space into the anterior abdominal wall it is

found immediately deep to which of the following layers of the anterior abdominal wall

-ltScarpas fascia

B External oblique muscle

C Internal oblique muscle D Transversus abdominis muscle

E Transversalis fascia

Explanation

The superficial perineal space is bound by Colles fascia the fibrous portion of the superficial fascia This

layer of fascia is continuous with Scarpas fascia the fibrous portion of the superficial fascia of the anterior abdominal wall Therefore urine that is deep to Colles fascia will remain deep to Scarpa s fascia The urine will spread in the plane between Scarpas fascia and the external oblique layer

When a horseshoe kidney develops the ascent of the kidney is restricted by the A internal iliac artery B external Iliac artery

C common iliac artery

inferior mesenteric artery

E superior mesenteric artery

Explanation

A horseshoe kidney develops when the inferior poles of the to kidneys fuse together as they ascend into

the abdomen from the pelvis The first anterior midline vessel that is encountered by the horseshoe kidney

is the inferior mesenteric artery This artery prevents the kidney from continuing its ascent

The left testicular vein drains into which of the following veins

A Left internal iliac vein B Left common iliac vein

bflnferior vena cava D Left renal vein I

E Left internal pudendal vein

Explanation

The left testicular vein drains into the left renal vein The right testicular ~i~[~nsltjectlY into the

inferior vena cava This difference in venous drainage is believed to explain the greater incidence of

varicocele on the left side than on the right The venous drainage from the penis is to the internal vein

which then drains into the internal Iliac vein

The spinal nerve that provides cutaneous branches to the skin around the umbilicus is

A TS B TW-shy

C TI2

DL2 EtA

Explanation

The tenth intercostal nerve is the anterior ramus of the TIO spinal nerve After passing through the tenth

intercostal space the nerve continues forward in the anterolateral abdominal wall in the plane between

the internal oblique muscle and the transversus abdominis muscle In the abdominal wall the nerve innervates to the abdominal wall muscles as well as the skin and the parietal peritoneum The umbilicus is

a useful landmark for the region of distribution of the tenth thoracic nerve

The ligament of the vertebral column that resists its extension is the Aligamentum flavum

B supraspinous ligament

C posterior longitudinal ligament

D anterior longitudinal ligament

E interspinous ligament

Explanation

The ligaments of the vertebral column that resist flexion of the column include the supraspinous ligament

interspinous ligament ligamentum fiavum and posterior longitudinal ligament The ligament that resists

extension is the anterior longitudinal ligament This longitudinal ligament is very broad and strong It

covers the anterior and anterolateral surfaces of the vertebral bodies and the intervertebral disks In

addition to resisting extension the anterior longitudinal ligament provides reinforcement to the anterior

and anterolateral surfaces of the intervertebral disk The posterior longitudinal ligament is relatively

narrow and covers the posterior surface of the vertebral bodies and the intervertebral disks This ligament

reinforces the posterior surface of the disk The posterolateral surface of the disk is not reinforced and it

is through this region that herniation of the nucleus pulposus usually occurs

A patient presents with epigastric and right upper quadrant pain The pain is most intense 2-4 hours after

eating and is reduced by the ingestion of antacids The patient states that he has passed black tarry stools

(melena) within the last week Fiberoptic endoscopy reveals a yellowish crater surrounded by a rim of

erythema that is 3 cm distal to the pylorus Accordingly an ulcer has been identified in the patients

A fundus

B antrum

C duodenum

D jejunum

E ileum

A number of physiologic genetic and other factors increase the risk of gastric (and duodenal) peptic

ulcers The evidence that H pylori plays a principle role is compelling Smoking and caffeine are known to adversely affect the morbidity mortality and healing rates of peptic ulcers In general first-degree

relatives of peptic ulcer patients as well as males have a threefold to fourfold increased risk of developing this disorder Paradoxically in gastric ulcer disease acid secretion is not elevated It is possible that

excess secreted hydrogen ion is reabsorbed across the injured gastric mucosa In general a defect in gastric mucosal defense is the more important local physiologic

A patient presents with symptoms of duodenal obstruction caused by an annular pancreas Annular pancreas is caused by

A rotation of the dorsal pancreatic bud into the ventral mesentery B rotation of the ventral pancreatic bud into the dorsal mesentery

fJ failure of the major and minor pancreatic ducts to fuse ~ ~ cleavage of the ventral pancreatic bud and rotation of the two portions in opposite directions around -the duodenum E formation of one pancreatic bud instead of two

Explanation Normally the ventral pancreatic bud rotates around the gut tube to reach the dorsal pancreatic bud The two buds fuse to form a single pancreas and the distal portions of the two ducts fuse The ventral pancreatic bud forms the inferior portion of the head of the pancreas the uncinate process and the major pancreatic duct (of Wirsung) The dorsal pancreatic bud forms the superior part of the head the neck body and tail and the minor pancreatic duct (of Santorini) Annular pancreas is the result of the ventral pancreatic bud dividing into two portions before it rotates into the dorsal mesentery Each portion rotates in opposite directions to get to the dorsal mesentery thus encircling the duodenum The presence of annular pancreas can constrict the duodenum thus obstructing its lumen

In n _ phranlc----

Gon ~l ----_1 Lum bltano

~~--- CornmQ1t bull ac

+-~4--- lnlllirnaJ ilic

xtem iliac

OBJECTIVE - Identify the blood supply to each of the structures listed in the table on the previous page

Ill give you a head start

FOREGUT - Supplied bV Celiac Tru nk (T12)

Proper hepatic

GastiooUod 13Jafter

1nferlor pancreaticoduodenal artery

Common epatlc

Lett gas ric iiirtery

Spfen artery

shy Gastroepiphgtic artery

~ Superior mesenteric 8rtfry

~

1 Esophagus is a derivative of the foregut so its blood supply originates from the celiac trunk

(T12) The predominant blood supply to abdominal portion of the esophagus is the Esophageal

A (Branch of L Gastric) The venous drainage of the esophagus is particularly important because

it is 1 of 3 clinically relevant sites of Portal Caval anastamoses The Portal Esophageal Vein

meets the Caval Azygos System Persistent bleeding manifests as Esophageal Varices - a fata I

condition

2 The Stomach is also a derivative of the foregut has EXTENSIVE blood supply and is very high

yield on anatomy exams The lesser curvature is supplied superiorly by the L Gastric A (1 of 3

major branches ofthe Celiac trunk) and inferiorly by the R Gastric A ( a branch ofthe proper

Hepatic A) The greater curvature is supplied superiorly by the L Gastroepiploic A (a major

branch of the splenic A) and inferiorly by the R Gastroepiploic A

The Short Gastric arteries (branches of Splenic Artery) supply the fundus of the stomach and

are referred to as EIID ARTERIES because they have no collateral blood supply Therefore if the

splenic artery were occluded (ex - increased pressure in the ommental bursa) - there would be

ischemia to the fundus of the stomach Venous drainage of the stomach is extensive via various

veins lead ing to the portal system Posterior to the stomach the IMV joins the splenic V which

joins the SMV to form the PORTAL VEIN ADAMS

3 Duodenum blood supply has high clinical relevance because it is the junction of the foregut and

midgut and therefore is the site of anastamoses between branches ofthe Celiac Trunk (main

foregut artery) and the Superior Messenteric Artery (main midgut artery) The Proper hepatic

artery gives off the gastroduodenal artery which travels behind the 1st part of the duodenum

This point has high clin ical relevance because duodenal ulcers are very common and a posterior

rupture of the 1st part of the duodenum could rupture the gastroduodenal artery causing

traumatic abdominal bleeding The Gastroduodenal artery first gives off the R Gastroepiploic A

(mentioned above) and proceeds as the Superior pancreatico duodenal artery (supplies the

pancreas and duodenum) which anastamoses with the inferior pancreatico duodenal A (branch

of the SMA) This is the junction of foregut and midgut and occurs near the opening of the

bil iary system into the duodenum (ampula of vater) Portal venous drainage here is responsible

for delivering nutrients from digestion to the liver for metabolism Appreciate that the Superior

mesenteric artery (artery of the midgut) branches from the aorta at Ll travels posterior to the

pancreas than moves anteriorly (at the jxn of the pancreatic headbody) and comes over the

3rd4th part of the duodenum Tumor of the head of the pancreas can compress the SMA

4 Jiver blood supply is via the common hepatic artery (major branch of the cel iac trunk) The

common hepatiC becomes the proper hepatic gives off the R gastric A and the Gastroduodenal

A and then joins the common bile duct and the portal vein in the portal triad Clinical- if a

patient were bleeding from the hepatic A a surgeon can stick his fingers in the epiplOic foramen

and squeeze the free edge of the hepatoduodenalligament in order to stop bleeding to the

area Please note that the hepatic a branches into Rand L hepatic A The Right hepatic artery

gives off the cystic artery which supplies the gallbladder Afferent venous supply is via the

Portal vein which is bringing nutrient rich blood to the liver After metabolism takes place

venous blood leaves the liver through the hepatic veins into the IVC PLEASE UNDERSTAND THE

RELATIONSHIP OF THESE STRUCTURES - ADAMSNETIERSNH Etc

5 Pancreas - Head is supplied via the superior and inferior pancreaticoduodenal arteries

(mentioned above) The tail (situated towards the hilum of the spleen) is supplied via the

pancreatic branches of the splenic artery (END ARTERIES) This blood supply is very important

because the endocrine Alpha and Beta Cells from the pancreatic islets of lagerhans are located

towards the tail This is where Insulin and Glucagon is released to the blood

Now complete this for mid and hindgut structures Make sure to note clinically relevant arterial

anastomoses as well as portal caval anastomoses FYI Appendix blood supply SMA + IMA

anastamoses marginal artery Portalcaval rectal veins fhemmorhoids) and periumbilical caput

medusa are high yield THE BUTT THE GUT and THE CAPUT

Abdominal Development

Liver

Ij1f

II wall b

oh liN ~ VltJrti n be- bull

Pancreas

Secondary Retroperitonealization e I~tl r 1 a v-mtrai m ellter

Rotations of the Gut I i Ij (lIl1UtIJ f~ l r tilt

()l td 10 me l-ft and he v

--~--- -~ -~-~

i

I AolaijonjoI~guf I

STOMACH BED (IDENTIFY IN ADAMS)- the structures posterior to the ommental bursa which

support the stomach in the supine position

Abdomnal JQrUI

Splnic vein

OmQ-oul tv~ ) O(s(Jroa)

Lojt(r o m nturrt (hpJtodu o d~n31 Hid

Gadrl)SplerH (g3stroll~nal) IIgam~nt

hiad h~~atogrtricent IIQdmiddotcrt~)

Lt Dome of Diaphragm (why left Look this up in Adams)

Spleen (What is the blood supply)

Left Kidney (What is the blood supply - AND how is it different from the R kidney)

Suprarenal Gland (What is the Arterial AND Venous Blood supply - how are they different)

Pancreas (How does supply differ from Head to Tail What is the SMA Relationship)

Transverse Mesocolon

liver - ADAMSWET - Make sure you look at the liver in wet lab

Left triangular nl1am~nt

ComoaDj ligamnt

Erophg~1 impre$ioo

Hepatio veins

In1erior -ifena middotr3)Ia

Fibrous appendix o-t

live

impr~j on

Heprorendl p~rtion of Q)(Qllary ligament

Righllri~n9ul r 1I~met

(Common) bile quol

Gr)mmCtr~ hepatic dlJct

Ccentic duct

Duodenal impression

GaJdate p-fr)~S

Hepatic artgtrl prop-f iiiiila - Faloiform ligament

_ - shy Round ligamen liver

~--F-- CoJio imprgt-ssi-on

Prta heptis

Identify the lobes impressions and embryonic remnants associated with the liver

Caudate Lobe Quadrate Lobe Right Lobe Left Lobe Round ligament Falciform Ligament

Ligamentum Venosum (what is its fxn in embryonic life) Hepatic Veins (NOT PART OF THE

PORTAL TRIAD) IVC PORTAL TRIAD - Contents relationship cross section etc Know the

Galbladder relationship to the lobes of the liver

Biliary Duct System - Make sure you understand the sequence of these structures - BE ABLE TO

DRAW A FLOW CHART

TPVd i

t

I t

1 __ Cm-(r

patk GlJet

I

J

Clinical = JAUNDICE is caused by anything that prevents delivery of bile to intestine Tumor of the

head of the pancreas Stones etc Patient will have pale stools and yellowish colored mucus

membranes

Clinical- Any scenario that tells you the patient has BILLOUS VOMIT means that the obstruction to

the flow of digestive contents is after the Ampulla of Vater (Site of Entry of Billiary system to the

duodenum) - ie Duodenal Atresia

Spleen -located posterior to the mid axillary line between ribs 9 and 11 Make sure you know that

the 10th rib is the main axis of the spleen and this organ is susceptible to injury (stab wound errant

thoracoce ntesis etc)

The spleen is derived from mesodermal cells - NOT THE GUT TUBE

The spleen rests on the left colic flexure associates with the tail of the pancreas Know the

structures entering the Hilum of the spleen

Sh rt O~-t~ic 1 0(0 10 rtiltSPIric Iloa nt

(cut)

Peritoneum - similar concept to Pleura - think of a fist in a balloon

Visceral Peritoneum - Layer of balloon touching your fist

Parietal Peritoneum - Layer of balloon not touching your fist

Your fist represents the organ your wrist is the hilum and your arm contains the blood supply

entering the organ

Appreciate that there will never be organs in the peritoneal cavity - rather these organs invaginate

the cavity Kaplan videos

RULES OF NOMENCLATUREshy

1 Organ completely surrounded by peritoneum - peritoneal organ

2 Organ partially surrounded by peritoneum- Retroperitoneal

3 Peritoneum surrounding peritoneal organ is VISCERAL peritoneum

4 Peritoneum surrounding retroperitoneal organ is PARIETAL peritoneum

5 Peritoneum connecting visceral to parietal is called messentary 2 messentaries in the

gut Dorsal (to the gut tube) and ventral (to the gut tube) messentary

Aorta is in Retro peritoneal position - but blood must reach peritoneal position - vessels travel through

messentary All peritoneal organs will have blood supply reaching through messentary

-Mesentery is a 2 layer peritoneum with a neurovascular communication between body wall and organ

- Ligament connects one organ with another or to the abdominal wall (Ommentum = ligament)

lesser Ommentum (attach lesser curvature of stomach and duodenum to liver) =Hepatoduodenal

Ligament and Hepatogastric Ligament

Has a Superior and Inferior Recess (Accumulation of Fluid in Ascites)

Communicates with the greater sac through the epiplic foramen (what structures pass through

this foramen)

Boundaries - you must be able to visualize this

o Anterior - stomach

o Posterior - parietal peritoneum pancreas

o Superior - superior recess (bw diaphragm and coronary ligament)

o Inferior -Inferior recess (bw layers or greater momentum

Greater Ommentum (attach greater curvature of stomach) Gastrophrenic ligament Gastrosplenic

ligament gastrocolic ligament

The greater omentum is the largest peritoneal fold It consists of a double sheet of peritoneum folded on itself so that it is made up of four layers The two layers which descend from the greater curvature of the stomach and commencement of the duodenum pass in front of the small intestines sometimes as low down as the pelvis they then turn upon themselves and ascend again as far as the transverse colon where they separate and enclose that part of the intestine

ABDOMINAL PAIN

Parietal Peritoneum - supplied by same vasculature lymphatics and nerves supplying body wall it

lines and diaphragm Sensitive to pain pressure heat cold well localized

Visceral Peritoneum - supplied by same vasculature lymphatics and somatic nerve of organ it covers

Insensitive to touch heat cold and laceration - referred to dermatome of spinal ganglia providing

sensory fibers Where does appendicitis refer to

Foregut pain - epigastric area (ie - cholycystitis)

Midgut pain - periumbilical area (ie - appendicitis)

Hindgut Pain - suprapubic area (ie - diverticulitis)

Extra ImagesConcepts

ll~_____-

FalifCtrm ligament oind r~ud ligamet f Ilver

Blood from splenio gastriC and inferiof rne$e-rteri v~ins

Ca-I tributaries

Lett gastrio Ifein

Posterior superior pan~reatioodul)denal vaihS

Lott gamo-om~nlal (9aropip lomiddotic) -in

Poq_~ tjol imerl-9-r panCJertlcorllJod-nal veiopound --amp----I- - ~J Right grtr~-omntal

Anwrior interi (gartroepiploic) Jjn

pan euaii cod vl)denal veins middot Inf~Ji (t r mesentric vein

Miqdle (olic vein

Right cl)licvein Sigmoid and rectosigml)id (ei ns

IhH)Collc(~io

--- Mi~dl laquooLJl gtjrltgt

PoM ca vl1 illasto)moses -----shyampoptoageal 2 Paraumbilie-lt11 Inferi or Fectal vei ns

3 Recial 4 REuoperHonea1

Know how the Portal vein is formed I 4 sites of portal caval anastamoses and 1 clinical shunt

Col li t ltt-~ otTl~tI ~nj pc~ 1lt1 turJoG

Ltf 14i1 tImiddot~ artoftl9 on tj phtAt$

L-oftqf 4t t~r 1=laquoIran d 1 bull shy~p l ci rj o fOOOts

Nerves follow the arteries - appreciate the splanchnic nervous system I

Uet~ric branch of left ~nal art

Ureterie branch of righi renal artelY

Left Zld lumbar in and co mlTlunication to as)erdin9 lumbar l(~in Hi ~ht tEZ1~~t~ t3r j t itn ~ nJ l1t- rlnd lfe i r1

Inferior me5nteri~ artery

Notice that the right testicular vein drains directly into the IVC and the right testicular artery drains

directly into the aorta However the left testicular vein drains into the L renal vein at a right angleshy

reason left testicle is lower and more susceptible to varicocele (bag of worms)

Also notice that the left renal vein has a longer course because the IVC is on the right side whereas

the right renal artery has a longer course because the aorta is on the left side

Appreciate the anterior to posterior relationship of structures in the hilum of the kidney - VAP - Vein

Artery Renal Pelvis (Ureter)

11____ __ L_ L_ n VJ __ _ _ t_L I I_ _ L __ L_ I -pound1 bull LI_~-I ____

Posterior View of Head of Pancreas in ( of Duodenum

Celiao hunk

Co mmon ~L~jJth art~ry

GastNduQdonal artrf (partilly in phantn)

P1)Sterior $Up~Jior panCflaticuduodfmal art~r~t

(Co mm on) bile duct

middot~1t~~t-1l---~-~- Right gshomiddotomental (gastoe plp lolc) 3rte (phantomost)

Grener paocre atic art-ry

1n1~rjor pancr-iatlc artery

Jtrifll supejo r pal)oreailcento)dJodenal artr1 (phantom)

Anastomotlo branch

POostetlor bJanch of jo f~ri of pan-reatir(lduodensl drttnj

Anterio r branch of i flferior palcreati~)duodenal art~(phan1om)

Notice the extensive blood supply to the pancreas and duodenum via the branches of the celiac trunk

Notice collateral supply from SMA branches - makes sense bc this is the jxn of foregutmidgut

Identify the vessels in this arteriogram

Hiltid i)f N~ck oi B)dvof Tail 01 pa nereas pan cent~as P-nmiddot-reas panCtCas

I nferie v~na cava

jHept1iic p(lrlai v~in

Port1 tnd H~pti lt a ftH prol

Comm on) bll duct

Ouodtnum

~ft colic (sio)Atta~ hmtrlt jt~xJr-ofha~elSe

muo(IIQn

Right ~lIc (h~j)tic)

il~gtture

In1triol m~oten lIein (rttr op~ritoMdO

SlJp efl or mes~n~fiC amrV and lipln

KNOW YOUR NEIGHBORHOOD

Questions

vVhiJh structure supplied by a bnmdlof the cclia( artery is not derivcd from foregut LemCJUCrITI

(A) Head of the pancte-a5

CD) Pyloric duolenum

Cystkduct

( Liver hepatocyt~~

~F) Body of the spleen

An infant presents with an omrhaJucele at birth -hi oJ the [oHm illg applies to his cM1-dition

(A) It is 31so seen ill p4titnts with aganghonic megacolon

(11) ft reuirs from a fal1ure of resorption of theviteUine d let

(C) It results from herniation at the-site of regression of the right umbilk vein

DJ It is caustd by faihtrc of recanalization of the midgut part of the duodenum

~ It ill camioo by a failuIt vf the midgul to return to the abGQminal uity after herniashytion in-n the urnbilk s l stalk

Ot er than the spleen occlusion Cif the spit-Ilk artery at its odgin wm most likely affect die blood supply to jllch st cnud

(A) Jejunum

(B) Body of th pal1~lltas

(C) LeSStT Cllmiddotlaturc of tl )toma-ch

(D Duodenum dista to the entrance of the Ornmou bile duct

E Fundus of the stomach

A 38-yeu-old batL~er with a history of heartburn suddenly experiences excluciating pain in the (plgastric region of th~ abdomeu SurgCry is perf~rme immediard y upon admisshysion to the 1IlcrgCJliy tuomh~re i~ evidence uf a ruptured ulcer in the posterior waU of the stomach Vhere will a surgeon first fi nd the stomach contenlSf

A) Greater p4ritoneal sac

rB) Cul~de-s~c of Douglas (--

C Omental bursa ~

--D) Paracolic gutter

rEj Between -he panttal perimltum and the posterior body wal1

At birth an infant presents with a st()ma~ rb~tbas~njJled jfltotb~diaplfagru 1A1ltre is the defect thatresulied iiitJle heini~t()n shy~tsophagealbiatus

7 - rH-- Hiatus for the inferior vena cava

( Pleuroperitoneal membrane -(0) Septum transvcrsum

(E) Right Crlt~

An infant born with DOVv7l syndrome presents with bili()u~ vomiting Ahat congenital defect does the infant have

(A) Pyloric stenosis

(B) Meckel diverticulum C) Ornphaloce1e

(D) Gastroschisis

( ~ ) Duodenal atresia y A patient with cirrhosis of the liver presents with ~ bacalvaricestnlreased retrograde pressure in which veins caused the varices

(A) Paraumuilical

(B) Splenic

(ct AzygltJus

(15))G~trk ( (-F) Superior mesemeric

A htaltby 3-year~old male patient experiences a hernial sa protruding from the anterior abdominal wall about halfway between me anterior superior ilia spine and the pubk tuberde Pulsations of al1 artery are palpated medial to the protrusion site through the abdominal walL Which layer of the anterior abdominal wall will first be traversed by the

1hctma

fA) Rectus sheath (B) External oblique aponeurosis

(C) Inguinal ligament

lD) Transversalis fusda

(E) Cremasteric fa~cia

After 5urgi(aj ffpair of a hernia the patient tXperienccs mtmlgtness in the skin on the anteshyrior aspect of the S(Totum_ Vhaf nerve may have been lesioned during thehemiorrhaphy

(A) Femoral

(B) Obturator

(C) Ilioinguinal

(D) lliohypogastrk

(E) Pudendal

A 23~year-LJld female secretary il1 good health ~-uddcn1) doubles over with pain in the a ea of the 1JmbRicu$ Sbe feels vartn and ltneasy and has no appetite That night the pain seems to have mQved to the tower right abdominal regjol1 and she calls her family doctor who then arranges for an ambulance to pk-k her up and take her to the hospitaL Wh ell ntn~ perceived in the area of the urnbilirus most Hkely carried lhe pairfu I sensations into the eNS

tA) Vagus nerves I~

V B)

) Lessersplanchnk nerves

tC) Pudendal nerves

(D) lIiohpogastrk nerves

(E) Greater splam ic l erves

A CT reveals carcinoma in the bOod of the ancreas Vhich blood vessel trut ourses ----~- - -bull ------ --shy

immediately poftterior to the body ofthe pancreas is the m~t likely to be oompressed

(A) Splenk artery

(B) Abdominal aorta (C) Portal vein

(1) Splenic vein

(E) Renal vein

A patient has a penrln1l1ng uker of the posterior wall ot the br~l part ot the (lUooenmn llkh blood vessel is subject to erosion

(A) Common hepatic artery

(B) Gastroouodenal artery

(C) Proper hevatic artery

(D) Celiac artery

(E) Anterior inferior 11amrelltlcoduodcnal attery

Your patient has been diagnosed -ith a carcinoma locallted to the head and l~e(k of the pancreas Another clinical sign would be

A esophageal varices

(8) hemorrhoids

C) a caput medusa

(D) increased pra Teuro n th~ hepatic veins

(E) enlarged right supra lavkular lymph nodes

Wltkh of the foUowing structures develops in the ventral mesentery

(A) Spleen

(B) Jeiunum (C) Head of1ht pancreas (D) Transverse colon (E) Stomach

ti l Uw ~ littwin~ f( S-t lil oai Imdge ~ hi(h or tbt la~)d J truetur tgt liJ llntn nl) he hl p UC iJd [IIi ell

c o

A) drains Ie tht infCrior a La aI

R t middot~nfl0 ~ill to th~ lunlgtn of h i dtlndCrlllfH

(e) m t bull JiJattd on tl l J n T ~H

D ) sup Lc O VSlt I Hlid bhtu l 1 li - -I un oid

( ) U~tpli(t tr j middottUh~ 1 v(( b~nt rfK n1ilc~Zm

ANSWERS AND EXPLANATIONS

Answer E The spleen is t hlttnopodicand lymph organ demlted from mesoderm

Answ~ R Al1 tlmphalocele is caused by it failure of the nlidgut to return to the ahdomir nat cavity after herniation into the umbiliau Stalk Choices Aand D maybe seen in infants with Down syndrome choice D ~s the specific CBuse ofduudcnal JtiCSitt Choice C is (ile cause of gclstrosbisis and Choice B nsults iu a Meurolktldivertku1-tlB

Answer B The fundus ofthe stomach is suppHed by soort gastric brunches of the splenic altery The splenic artery supplies the body and tail of the pancreas part of the greater curvature of the sttmla(h and the spleen Te jejunum part of the head of the pancreas and tht~ duodenum distal to the entrance of the commOll bile duct are supplied by the superior mesenterk artery clll~l ~be less r ctlt1ature cmd the pylQric antrum are supplied by the right and lei gastric art(ries

AnSWftt C Tbeomental bursa or lesser ~ritoneaj sac lies direcdy posterior to the proxshyimal part of the duodeTtlm and the stomach and would be the first site where stomach contents ~Ott1d be fpoundluncL

Answer C A defect in a llleuropcritoneal membrane (uswlly the left) is the typical site of i1 cc-ngenitlI diilphragluatic hemia llere the membr4ne fails to dose ()pound( of the perishycCirdiopcritulleal canals

Answer E DuoJenal atresia and aganglionic megacoion are congwitaI defects S~Il in patients with Dowmiddotnsyndrome

Answer D RulaTgemt~llt of and retrograde flow in g~lstrk vel_ns in particlJl~r the kft gas~ tricveins dilates the capillary bed in rhe wall of the esophagus in (ases of porta yper~

tension Blood flow would increase in and dilampte tribntarkgts of the (lZygOUS vein on the other side of the capiUary bed but flow in this vein is in the typical direction t()ward the superior vena cava Paraumbiii(ltU vein eilgorgement contributes to a caput medusH Splenic ~nlargement might prc~nt with 5plcnonlegaly and balt-kflow in to tlu superior m~~ntclic vein occurs but is asymptomatic

Answer D The patient hagt an indirect inguinal hernia whi~h emerges from the antt-rior abdominal wall through the deep inguinltilling Theeep ring is a fault in the transv~rshysaUs fascia this I~yer wiIJ be penetrated first by the hernia

An~Wer C The ilioinguinal nenc which provides sens~llion to the lnedlal thigh ltmclanteshytior SClotunl pass~lt th rough the 5uperfh_ial inguinal ring ind $subject to inj i1T) becaus-e

it is in the operatitm Held of the erniorrhapny

Auswer B The leMHr splanchnic nerves are sympathdic nerVlts that carry viscera l sensashytlltgtrogt ftom illtllt1m~d ()J stietched gust (itinteitinal ~tructures (in this case the pprndix) into tnt eNS Lesser splanchnic ntTYcsarisc from thmiddot T9--T12 spinal cord segments lt1nd provide sympathetic innenation tD rnidgut siruc1ures whiCh include CLe app~JldD Viscera] Pain arising from affecLed Inidgut ampt 1C1ure is referred over the same dl- matorne~ of spinal segrnertts v-hich provide the sympathetic Innervation n this G1SC of appendicitis the invohen~n t of the ltire) of t e unlhHku indud s the T 10 dermatome

Answer B Of the five choices onty the dscending olon is retroperiton~al aldwould be a lik ~ ( choice to be seen immediately a(~jilcent to t11e posterior abdominal middotn~L

Amwen D The SpltftlC ~-ein ourses posterior to the body of the panneas m its way tt drain into the superior mCSfttltlri( vein

Answcr B TILt glstrodllolticnal artery 1 direct hIamh of the comrootl hepatic artery courses immediately pt))iwri() to the duodenum and is slbject to erosion

Answer B Carcinoma of th pan middott3S in the 1tilt1 may compreampgt the portltil vein at irs orishygill The poTtai vcin is fomled when the splenic vein jQiaswith tfie superior meStllt eric vein The inferiot mesenteric vein joins the ~plenjc vein just priOT to tlli~ point at which the splenic joins the superior Jlleit1ltcri( vein Increescd venous presslu in the inferior mesenteric vein is a cause of emo hoid~

Answer C The- velltral pancreas wilich forms most of the head of the p ~ncr as develops in the ventral mes(ntery as antutgrowth of the hepatic diverticulum Th~ hepatic divershyticulull induding the biIJary appa~atus develops in tbe ventral mesentery of the foregut

Answer~ A The superior mesenteric ~in joins with the spienkvein to form the hepatic portal vciu

Answer D The structure at gttlK is the proper hepatic artery~ whkh suppUesoxygenated b middotood to the liver

MAKE SURE YOU KNOW the diff bw Rectus Sheath above and below the arcuate line

ABOVE

Aponeurosis of xiiltmal obllque musclo

Extemll f)biquw musde

Reotln ilbdomlnls musole S~in

Internal 9bliquQ mY~QI

AponeUfOsi$ of hJH$V~~S Lir9a a lb lbdolTlin~ musolo Tri OJV6 rUi

atldomlnis mUS(loe

Sub cutanlilous tiue (tatty ye r)

BElOW

A POrl lJfosis 01 etemal oblique muscle

Aponeul~)sis 01 Internal oblique mU$cl~

Anteriol lay~ of r~ltdus st~ath EXttom1 oblique rnu$cll

Rectus Jbdominis muscle Intoernal Aponeurc-sis of tra~fersU$ oblique muscle-

at-domlnis muscentl ~ Skio

Tra nsvitSus abdomioLs ml)ZClt

TralSVersaHs fascia Medial umQil iegtt1 1i9Jment -and folj

Uldchus Peritoneum (ir median Umbilj~al Suboutane ous

Extraprftone 11ascia

Ymbilimiddot~1 fold)

preu9poundiea1 fascia

tissue (fatty 4nd m~mbr3n(iUS layers)

o Above the arcuate line (A horizontal line 13 of the distance bw the umbilicus and the

pubic symphysis) -10 Aponeurosis divides into an AntPost Laminae

o The Ant Laminae joins EO and Post Laminae joins Trans Abdominis = Ant and Post

RECTUS SHEATH respectively

o BElOW the arcuate line - all 3 aponeurosis join ANTERIOR to rectus muscle to meet its

counterpart in the midline (linea Alba)

o Take away Msg - The abdomen is devoid of a posterior rectus sheath below the

arcuate line and is therefore more vulnerable to herniasinjuries

Question - A physician makes a deep incision in the patients midline immediately superior to

the pubic symphysis which of the following layers is his knife least likely to pass

Rectus Abdominis External Oblique Ant Rectus Sheath Posterior Rectus Sheath All of the

Above

Answer - All of the above None of the other answer choices are midline structures -LINEA

ALBA

Linea Alba has very poor blood supply - doesnt heal well after surgery Therefore this is a

common site for incisional hernias

a Spleen b Transverse colon c Descending colon d Stomach e Pleura

17 Meckels diverticulum is normally found 2 feet proximal from the

a Pyloric sphincter b Lower esophageal sphincter c Ileo-cecal valve d Middle valve of Huston e Anal valve

18 Ulcer in the posterior wall of the first part of the duodenum would erode ___ artery and would cause bleeding

a Left gastric b Right gastric c Hepatic artery proper d Gastroduodenal artery e Middle colic artery

19 An inflamed appendix is identified by a surgeon on the operation table by noting

a The appendicies epiploicae b The convergence of tenia c The artery of Drummond d The mesocolon e The mesosalphinx

20 The nerve which emerges through the psoas major is

a Femoral b Ilio-inguinal c Ilio-hypogastric d Pudendal e Subcostal

21 The right gonadal vein drains into the

a Azygos b Hemiazygos c Inferior Vena Cava d Right renal vein e Left renal vein

22 The hepatocytes in the liver is derived from

a Ectoderm b Endoderm c Mesoderm

d Neural ectoderm

23 Abscess in the lumbar vertebrae due to tuberculosis would spread to the adjacent muscle which is

a Psoas Major b Iliacus c Quadratus lumborum d Tranversus Abdominis

24 The anterior wall of the inguinal canal is formed by

a External oblique and transverses abdominis b External oblique and fascia transversalis c Internal oblique and external oblique d Internal oblique and transverses abdominis e Fascia transversalis and peritoneum

Meckels diverticulum is a result of which of the following developmental abnormalities shy

A Failure of the vitelline duct to close

B Failure of the herniated intestinal loop to retract into the abdomen

C Failure of the urachus to close

D Failure of the midgut to rotate

E Failure of the hepatic duct to close

Explanation

Meckels diverticulum is a result of the persistence of the proximal part of the vitelline duct This

diverticulum is usually found about 2 feet proximal to the ileocecal junction and is usually about 2 inches

long It is present in about 2 of the popUlation It may be the site of ectopic pancreatic tissue or gastric

mucosa and may develop inflammatory processes and ulcerations Acute Meckels diverticulitis

simulates appendicitis

Which of the following veins carries blood from the esophagus to the portal vein The

A right gastric vein

B left gastric vein c splenic vein D azygos vein

E left gastroepiploic vein

Explanation

The left gastric vein a direct branch of the portal vein drains blood from the lesser curvature of the

stomach and the inferior portion of the esophagus Because branches of the portal vein do not have

valves blood can flow in a retrograde path when there is an obstruction to flow through the portal system or liveL Rlooci Cln then flow from the nortl] vein thr()1Ph the left PRstric vein to the esonhlPlIS lno

through venous communications within the submucosa of the esophagus to esophageal veins that drain

into the azygos vein The increase in blood flow through the esophageal submucosal veins results in esophageal varices

On the posterior wall of the abdomen the celiac ganglion A contains cell bodies of postganglionic parasympathetic neurons B is synapsed upon by neurons in the posterior vagal trunk C is synapsed upon by neurons in the greater splanchnic nerve D contains sensory cell bodies of lumbar spinal nerves E contains cell bodies of neurons that cause an increase in the rate of peristasis

Explanation The celiac ganglion is one of the preaortic ganglia of the sympathetic nervous system It contains cell bodies of postganglionic sympathetic neurons The sympathetic splanchnic nerves contain preganglionic sympathetic neurons that pass through the sympathetic chain without synapsing These splanchnic nerves go to the preaortic ganglia to synapse The greater splanchnic nerve contains preganglionic neurons from spinal cord segments T5-T9 This nerve synapses in the celiac ganglion The nerve fibers in the vagal trunks are preganglionic parasympathetic fibers that go to the walls of the organs that they will innervate and synapse on postganglionic parasympathetic neurons in the walls of those organs Cell bodies of sensory neurons in the abdomen are found in the dorsal root ganglia or the sensory ganglia of the vagus nerve Sympathetic innervation decreases the rate of peristalsis parasympathetic innervation increases the rate of peristalsis

Which of the following pairs of arteries will allow blood to bypass an occlusion of the celiac trunk

A Left gastric artery-right gastric artery

B Left gastroepiploic artery-right gastroepiploic artery

C Superior pancreaticoduodenal artery-inferior pancreaticoduodenal artery

D Splenic artery-common hepatic artery

E Left gastric artery - proper hepatic artery

Explanation The anastoOlosis of a branch of the celiac trunk and a branch of the superior mesenteric artery will

provide collateral circulation around an occlusion of the celiac trunk Each of the other choices pair

branches of the celiac trunk therefore these will not provide collateral flow around the obstruction of the

celiac trunk The left gastric splenic and common hepatic arteries are direct branches of the celiac trunk

The right gastric artery is a branch of the proper hepatic artery which is a branch of the common hepatic artery The left gastroepiploic artery is a branch of the splenic artery The right gastroepiploic artery is a

branch of the gastroduodenal artery whlch is a branch of the common hepatic artery

Which of the following organs has appendices epiploica The

A sigmoid colon

Bjejunum

C duodenum

D stomach E esophagus

Explanation Appendices epiploica are characteristic of the colon Appendices epiploica are subserosal accumulations

of fat None of the organs of the gastrointestinal tract has appendices epiploica except the colon

Page 27: Chirag's Abdomen Review

The spinal nerve that provides cutaneous branches to the skin around the umbilicus is

A TS B TW-shy

C TI2

DL2 EtA

Explanation

The tenth intercostal nerve is the anterior ramus of the TIO spinal nerve After passing through the tenth

intercostal space the nerve continues forward in the anterolateral abdominal wall in the plane between

the internal oblique muscle and the transversus abdominis muscle In the abdominal wall the nerve innervates to the abdominal wall muscles as well as the skin and the parietal peritoneum The umbilicus is

a useful landmark for the region of distribution of the tenth thoracic nerve

The ligament of the vertebral column that resists its extension is the Aligamentum flavum

B supraspinous ligament

C posterior longitudinal ligament

D anterior longitudinal ligament

E interspinous ligament

Explanation

The ligaments of the vertebral column that resist flexion of the column include the supraspinous ligament

interspinous ligament ligamentum fiavum and posterior longitudinal ligament The ligament that resists

extension is the anterior longitudinal ligament This longitudinal ligament is very broad and strong It

covers the anterior and anterolateral surfaces of the vertebral bodies and the intervertebral disks In

addition to resisting extension the anterior longitudinal ligament provides reinforcement to the anterior

and anterolateral surfaces of the intervertebral disk The posterior longitudinal ligament is relatively

narrow and covers the posterior surface of the vertebral bodies and the intervertebral disks This ligament

reinforces the posterior surface of the disk The posterolateral surface of the disk is not reinforced and it

is through this region that herniation of the nucleus pulposus usually occurs

A patient presents with epigastric and right upper quadrant pain The pain is most intense 2-4 hours after

eating and is reduced by the ingestion of antacids The patient states that he has passed black tarry stools

(melena) within the last week Fiberoptic endoscopy reveals a yellowish crater surrounded by a rim of

erythema that is 3 cm distal to the pylorus Accordingly an ulcer has been identified in the patients

A fundus

B antrum

C duodenum

D jejunum

E ileum

A number of physiologic genetic and other factors increase the risk of gastric (and duodenal) peptic

ulcers The evidence that H pylori plays a principle role is compelling Smoking and caffeine are known to adversely affect the morbidity mortality and healing rates of peptic ulcers In general first-degree

relatives of peptic ulcer patients as well as males have a threefold to fourfold increased risk of developing this disorder Paradoxically in gastric ulcer disease acid secretion is not elevated It is possible that

excess secreted hydrogen ion is reabsorbed across the injured gastric mucosa In general a defect in gastric mucosal defense is the more important local physiologic

A patient presents with symptoms of duodenal obstruction caused by an annular pancreas Annular pancreas is caused by

A rotation of the dorsal pancreatic bud into the ventral mesentery B rotation of the ventral pancreatic bud into the dorsal mesentery

fJ failure of the major and minor pancreatic ducts to fuse ~ ~ cleavage of the ventral pancreatic bud and rotation of the two portions in opposite directions around -the duodenum E formation of one pancreatic bud instead of two

Explanation Normally the ventral pancreatic bud rotates around the gut tube to reach the dorsal pancreatic bud The two buds fuse to form a single pancreas and the distal portions of the two ducts fuse The ventral pancreatic bud forms the inferior portion of the head of the pancreas the uncinate process and the major pancreatic duct (of Wirsung) The dorsal pancreatic bud forms the superior part of the head the neck body and tail and the minor pancreatic duct (of Santorini) Annular pancreas is the result of the ventral pancreatic bud dividing into two portions before it rotates into the dorsal mesentery Each portion rotates in opposite directions to get to the dorsal mesentery thus encircling the duodenum The presence of annular pancreas can constrict the duodenum thus obstructing its lumen

In n _ phranlc----

Gon ~l ----_1 Lum bltano

~~--- CornmQ1t bull ac

+-~4--- lnlllirnaJ ilic

xtem iliac

OBJECTIVE - Identify the blood supply to each of the structures listed in the table on the previous page

Ill give you a head start

FOREGUT - Supplied bV Celiac Tru nk (T12)

Proper hepatic

GastiooUod 13Jafter

1nferlor pancreaticoduodenal artery

Common epatlc

Lett gas ric iiirtery

Spfen artery

shy Gastroepiphgtic artery

~ Superior mesenteric 8rtfry

~

1 Esophagus is a derivative of the foregut so its blood supply originates from the celiac trunk

(T12) The predominant blood supply to abdominal portion of the esophagus is the Esophageal

A (Branch of L Gastric) The venous drainage of the esophagus is particularly important because

it is 1 of 3 clinically relevant sites of Portal Caval anastamoses The Portal Esophageal Vein

meets the Caval Azygos System Persistent bleeding manifests as Esophageal Varices - a fata I

condition

2 The Stomach is also a derivative of the foregut has EXTENSIVE blood supply and is very high

yield on anatomy exams The lesser curvature is supplied superiorly by the L Gastric A (1 of 3

major branches ofthe Celiac trunk) and inferiorly by the R Gastric A ( a branch ofthe proper

Hepatic A) The greater curvature is supplied superiorly by the L Gastroepiploic A (a major

branch of the splenic A) and inferiorly by the R Gastroepiploic A

The Short Gastric arteries (branches of Splenic Artery) supply the fundus of the stomach and

are referred to as EIID ARTERIES because they have no collateral blood supply Therefore if the

splenic artery were occluded (ex - increased pressure in the ommental bursa) - there would be

ischemia to the fundus of the stomach Venous drainage of the stomach is extensive via various

veins lead ing to the portal system Posterior to the stomach the IMV joins the splenic V which

joins the SMV to form the PORTAL VEIN ADAMS

3 Duodenum blood supply has high clinical relevance because it is the junction of the foregut and

midgut and therefore is the site of anastamoses between branches ofthe Celiac Trunk (main

foregut artery) and the Superior Messenteric Artery (main midgut artery) The Proper hepatic

artery gives off the gastroduodenal artery which travels behind the 1st part of the duodenum

This point has high clin ical relevance because duodenal ulcers are very common and a posterior

rupture of the 1st part of the duodenum could rupture the gastroduodenal artery causing

traumatic abdominal bleeding The Gastroduodenal artery first gives off the R Gastroepiploic A

(mentioned above) and proceeds as the Superior pancreatico duodenal artery (supplies the

pancreas and duodenum) which anastamoses with the inferior pancreatico duodenal A (branch

of the SMA) This is the junction of foregut and midgut and occurs near the opening of the

bil iary system into the duodenum (ampula of vater) Portal venous drainage here is responsible

for delivering nutrients from digestion to the liver for metabolism Appreciate that the Superior

mesenteric artery (artery of the midgut) branches from the aorta at Ll travels posterior to the

pancreas than moves anteriorly (at the jxn of the pancreatic headbody) and comes over the

3rd4th part of the duodenum Tumor of the head of the pancreas can compress the SMA

4 Jiver blood supply is via the common hepatic artery (major branch of the cel iac trunk) The

common hepatiC becomes the proper hepatic gives off the R gastric A and the Gastroduodenal

A and then joins the common bile duct and the portal vein in the portal triad Clinical- if a

patient were bleeding from the hepatic A a surgeon can stick his fingers in the epiplOic foramen

and squeeze the free edge of the hepatoduodenalligament in order to stop bleeding to the

area Please note that the hepatic a branches into Rand L hepatic A The Right hepatic artery

gives off the cystic artery which supplies the gallbladder Afferent venous supply is via the

Portal vein which is bringing nutrient rich blood to the liver After metabolism takes place

venous blood leaves the liver through the hepatic veins into the IVC PLEASE UNDERSTAND THE

RELATIONSHIP OF THESE STRUCTURES - ADAMSNETIERSNH Etc

5 Pancreas - Head is supplied via the superior and inferior pancreaticoduodenal arteries

(mentioned above) The tail (situated towards the hilum of the spleen) is supplied via the

pancreatic branches of the splenic artery (END ARTERIES) This blood supply is very important

because the endocrine Alpha and Beta Cells from the pancreatic islets of lagerhans are located

towards the tail This is where Insulin and Glucagon is released to the blood

Now complete this for mid and hindgut structures Make sure to note clinically relevant arterial

anastomoses as well as portal caval anastomoses FYI Appendix blood supply SMA + IMA

anastamoses marginal artery Portalcaval rectal veins fhemmorhoids) and periumbilical caput

medusa are high yield THE BUTT THE GUT and THE CAPUT

Abdominal Development

Liver

Ij1f

II wall b

oh liN ~ VltJrti n be- bull

Pancreas

Secondary Retroperitonealization e I~tl r 1 a v-mtrai m ellter

Rotations of the Gut I i Ij (lIl1UtIJ f~ l r tilt

()l td 10 me l-ft and he v

--~--- -~ -~-~

i

I AolaijonjoI~guf I

STOMACH BED (IDENTIFY IN ADAMS)- the structures posterior to the ommental bursa which

support the stomach in the supine position

Abdomnal JQrUI

Splnic vein

OmQ-oul tv~ ) O(s(Jroa)

Lojt(r o m nturrt (hpJtodu o d~n31 Hid

Gadrl)SplerH (g3stroll~nal) IIgam~nt

hiad h~~atogrtricent IIQdmiddotcrt~)

Lt Dome of Diaphragm (why left Look this up in Adams)

Spleen (What is the blood supply)

Left Kidney (What is the blood supply - AND how is it different from the R kidney)

Suprarenal Gland (What is the Arterial AND Venous Blood supply - how are they different)

Pancreas (How does supply differ from Head to Tail What is the SMA Relationship)

Transverse Mesocolon

liver - ADAMSWET - Make sure you look at the liver in wet lab

Left triangular nl1am~nt

ComoaDj ligamnt

Erophg~1 impre$ioo

Hepatio veins

In1erior -ifena middotr3)Ia

Fibrous appendix o-t

live

impr~j on

Heprorendl p~rtion of Q)(Qllary ligament

Righllri~n9ul r 1I~met

(Common) bile quol

Gr)mmCtr~ hepatic dlJct

Ccentic duct

Duodenal impression

GaJdate p-fr)~S

Hepatic artgtrl prop-f iiiiila - Faloiform ligament

_ - shy Round ligamen liver

~--F-- CoJio imprgt-ssi-on

Prta heptis

Identify the lobes impressions and embryonic remnants associated with the liver

Caudate Lobe Quadrate Lobe Right Lobe Left Lobe Round ligament Falciform Ligament

Ligamentum Venosum (what is its fxn in embryonic life) Hepatic Veins (NOT PART OF THE

PORTAL TRIAD) IVC PORTAL TRIAD - Contents relationship cross section etc Know the

Galbladder relationship to the lobes of the liver

Biliary Duct System - Make sure you understand the sequence of these structures - BE ABLE TO

DRAW A FLOW CHART

TPVd i

t

I t

1 __ Cm-(r

patk GlJet

I

J

Clinical = JAUNDICE is caused by anything that prevents delivery of bile to intestine Tumor of the

head of the pancreas Stones etc Patient will have pale stools and yellowish colored mucus

membranes

Clinical- Any scenario that tells you the patient has BILLOUS VOMIT means that the obstruction to

the flow of digestive contents is after the Ampulla of Vater (Site of Entry of Billiary system to the

duodenum) - ie Duodenal Atresia

Spleen -located posterior to the mid axillary line between ribs 9 and 11 Make sure you know that

the 10th rib is the main axis of the spleen and this organ is susceptible to injury (stab wound errant

thoracoce ntesis etc)

The spleen is derived from mesodermal cells - NOT THE GUT TUBE

The spleen rests on the left colic flexure associates with the tail of the pancreas Know the

structures entering the Hilum of the spleen

Sh rt O~-t~ic 1 0(0 10 rtiltSPIric Iloa nt

(cut)

Peritoneum - similar concept to Pleura - think of a fist in a balloon

Visceral Peritoneum - Layer of balloon touching your fist

Parietal Peritoneum - Layer of balloon not touching your fist

Your fist represents the organ your wrist is the hilum and your arm contains the blood supply

entering the organ

Appreciate that there will never be organs in the peritoneal cavity - rather these organs invaginate

the cavity Kaplan videos

RULES OF NOMENCLATUREshy

1 Organ completely surrounded by peritoneum - peritoneal organ

2 Organ partially surrounded by peritoneum- Retroperitoneal

3 Peritoneum surrounding peritoneal organ is VISCERAL peritoneum

4 Peritoneum surrounding retroperitoneal organ is PARIETAL peritoneum

5 Peritoneum connecting visceral to parietal is called messentary 2 messentaries in the

gut Dorsal (to the gut tube) and ventral (to the gut tube) messentary

Aorta is in Retro peritoneal position - but blood must reach peritoneal position - vessels travel through

messentary All peritoneal organs will have blood supply reaching through messentary

-Mesentery is a 2 layer peritoneum with a neurovascular communication between body wall and organ

- Ligament connects one organ with another or to the abdominal wall (Ommentum = ligament)

lesser Ommentum (attach lesser curvature of stomach and duodenum to liver) =Hepatoduodenal

Ligament and Hepatogastric Ligament

Has a Superior and Inferior Recess (Accumulation of Fluid in Ascites)

Communicates with the greater sac through the epiplic foramen (what structures pass through

this foramen)

Boundaries - you must be able to visualize this

o Anterior - stomach

o Posterior - parietal peritoneum pancreas

o Superior - superior recess (bw diaphragm and coronary ligament)

o Inferior -Inferior recess (bw layers or greater momentum

Greater Ommentum (attach greater curvature of stomach) Gastrophrenic ligament Gastrosplenic

ligament gastrocolic ligament

The greater omentum is the largest peritoneal fold It consists of a double sheet of peritoneum folded on itself so that it is made up of four layers The two layers which descend from the greater curvature of the stomach and commencement of the duodenum pass in front of the small intestines sometimes as low down as the pelvis they then turn upon themselves and ascend again as far as the transverse colon where they separate and enclose that part of the intestine

ABDOMINAL PAIN

Parietal Peritoneum - supplied by same vasculature lymphatics and nerves supplying body wall it

lines and diaphragm Sensitive to pain pressure heat cold well localized

Visceral Peritoneum - supplied by same vasculature lymphatics and somatic nerve of organ it covers

Insensitive to touch heat cold and laceration - referred to dermatome of spinal ganglia providing

sensory fibers Where does appendicitis refer to

Foregut pain - epigastric area (ie - cholycystitis)

Midgut pain - periumbilical area (ie - appendicitis)

Hindgut Pain - suprapubic area (ie - diverticulitis)

Extra ImagesConcepts

ll~_____-

FalifCtrm ligament oind r~ud ligamet f Ilver

Blood from splenio gastriC and inferiof rne$e-rteri v~ins

Ca-I tributaries

Lett gastrio Ifein

Posterior superior pan~reatioodul)denal vaihS

Lott gamo-om~nlal (9aropip lomiddotic) -in

Poq_~ tjol imerl-9-r panCJertlcorllJod-nal veiopound --amp----I- - ~J Right grtr~-omntal

Anwrior interi (gartroepiploic) Jjn

pan euaii cod vl)denal veins middot Inf~Ji (t r mesentric vein

Miqdle (olic vein

Right cl)licvein Sigmoid and rectosigml)id (ei ns

IhH)Collc(~io

--- Mi~dl laquooLJl gtjrltgt

PoM ca vl1 illasto)moses -----shyampoptoageal 2 Paraumbilie-lt11 Inferi or Fectal vei ns

3 Recial 4 REuoperHonea1

Know how the Portal vein is formed I 4 sites of portal caval anastamoses and 1 clinical shunt

Col li t ltt-~ otTl~tI ~nj pc~ 1lt1 turJoG

Ltf 14i1 tImiddot~ artoftl9 on tj phtAt$

L-oftqf 4t t~r 1=laquoIran d 1 bull shy~p l ci rj o fOOOts

Nerves follow the arteries - appreciate the splanchnic nervous system I

Uet~ric branch of left ~nal art

Ureterie branch of righi renal artelY

Left Zld lumbar in and co mlTlunication to as)erdin9 lumbar l(~in Hi ~ht tEZ1~~t~ t3r j t itn ~ nJ l1t- rlnd lfe i r1

Inferior me5nteri~ artery

Notice that the right testicular vein drains directly into the IVC and the right testicular artery drains

directly into the aorta However the left testicular vein drains into the L renal vein at a right angleshy

reason left testicle is lower and more susceptible to varicocele (bag of worms)

Also notice that the left renal vein has a longer course because the IVC is on the right side whereas

the right renal artery has a longer course because the aorta is on the left side

Appreciate the anterior to posterior relationship of structures in the hilum of the kidney - VAP - Vein

Artery Renal Pelvis (Ureter)

11____ __ L_ L_ n VJ __ _ _ t_L I I_ _ L __ L_ I -pound1 bull LI_~-I ____

Posterior View of Head of Pancreas in ( of Duodenum

Celiao hunk

Co mmon ~L~jJth art~ry

GastNduQdonal artrf (partilly in phantn)

P1)Sterior $Up~Jior panCflaticuduodfmal art~r~t

(Co mm on) bile duct

middot~1t~~t-1l---~-~- Right gshomiddotomental (gastoe plp lolc) 3rte (phantomost)

Grener paocre atic art-ry

1n1~rjor pancr-iatlc artery

Jtrifll supejo r pal)oreailcento)dJodenal artr1 (phantom)

Anastomotlo branch

POostetlor bJanch of jo f~ri of pan-reatir(lduodensl drttnj

Anterio r branch of i flferior palcreati~)duodenal art~(phan1om)

Notice the extensive blood supply to the pancreas and duodenum via the branches of the celiac trunk

Notice collateral supply from SMA branches - makes sense bc this is the jxn of foregutmidgut

Identify the vessels in this arteriogram

Hiltid i)f N~ck oi B)dvof Tail 01 pa nereas pan cent~as P-nmiddot-reas panCtCas

I nferie v~na cava

jHept1iic p(lrlai v~in

Port1 tnd H~pti lt a ftH prol

Comm on) bll duct

Ouodtnum

~ft colic (sio)Atta~ hmtrlt jt~xJr-ofha~elSe

muo(IIQn

Right ~lIc (h~j)tic)

il~gtture

In1triol m~oten lIein (rttr op~ritoMdO

SlJp efl or mes~n~fiC amrV and lipln

KNOW YOUR NEIGHBORHOOD

Questions

vVhiJh structure supplied by a bnmdlof the cclia( artery is not derivcd from foregut LemCJUCrITI

(A) Head of the pancte-a5

CD) Pyloric duolenum

Cystkduct

( Liver hepatocyt~~

~F) Body of the spleen

An infant presents with an omrhaJucele at birth -hi oJ the [oHm illg applies to his cM1-dition

(A) It is 31so seen ill p4titnts with aganghonic megacolon

(11) ft reuirs from a fal1ure of resorption of theviteUine d let

(C) It results from herniation at the-site of regression of the right umbilk vein

DJ It is caustd by faihtrc of recanalization of the midgut part of the duodenum

~ It ill camioo by a failuIt vf the midgul to return to the abGQminal uity after herniashytion in-n the urnbilk s l stalk

Ot er than the spleen occlusion Cif the spit-Ilk artery at its odgin wm most likely affect die blood supply to jllch st cnud

(A) Jejunum

(B) Body of th pal1~lltas

(C) LeSStT Cllmiddotlaturc of tl )toma-ch

(D Duodenum dista to the entrance of the Ornmou bile duct

E Fundus of the stomach

A 38-yeu-old batL~er with a history of heartburn suddenly experiences excluciating pain in the (plgastric region of th~ abdomeu SurgCry is perf~rme immediard y upon admisshysion to the 1IlcrgCJliy tuomh~re i~ evidence uf a ruptured ulcer in the posterior waU of the stomach Vhere will a surgeon first fi nd the stomach contenlSf

A) Greater p4ritoneal sac

rB) Cul~de-s~c of Douglas (--

C Omental bursa ~

--D) Paracolic gutter

rEj Between -he panttal perimltum and the posterior body wal1

At birth an infant presents with a st()ma~ rb~tbas~njJled jfltotb~diaplfagru 1A1ltre is the defect thatresulied iiitJle heini~t()n shy~tsophagealbiatus

7 - rH-- Hiatus for the inferior vena cava

( Pleuroperitoneal membrane -(0) Septum transvcrsum

(E) Right Crlt~

An infant born with DOVv7l syndrome presents with bili()u~ vomiting Ahat congenital defect does the infant have

(A) Pyloric stenosis

(B) Meckel diverticulum C) Ornphaloce1e

(D) Gastroschisis

( ~ ) Duodenal atresia y A patient with cirrhosis of the liver presents with ~ bacalvaricestnlreased retrograde pressure in which veins caused the varices

(A) Paraumuilical

(B) Splenic

(ct AzygltJus

(15))G~trk ( (-F) Superior mesemeric

A htaltby 3-year~old male patient experiences a hernial sa protruding from the anterior abdominal wall about halfway between me anterior superior ilia spine and the pubk tuberde Pulsations of al1 artery are palpated medial to the protrusion site through the abdominal walL Which layer of the anterior abdominal wall will first be traversed by the

1hctma

fA) Rectus sheath (B) External oblique aponeurosis

(C) Inguinal ligament

lD) Transversalis fusda

(E) Cremasteric fa~cia

After 5urgi(aj ffpair of a hernia the patient tXperienccs mtmlgtness in the skin on the anteshyrior aspect of the S(Totum_ Vhaf nerve may have been lesioned during thehemiorrhaphy

(A) Femoral

(B) Obturator

(C) Ilioinguinal

(D) lliohypogastrk

(E) Pudendal

A 23~year-LJld female secretary il1 good health ~-uddcn1) doubles over with pain in the a ea of the 1JmbRicu$ Sbe feels vartn and ltneasy and has no appetite That night the pain seems to have mQved to the tower right abdominal regjol1 and she calls her family doctor who then arranges for an ambulance to pk-k her up and take her to the hospitaL Wh ell ntn~ perceived in the area of the urnbilirus most Hkely carried lhe pairfu I sensations into the eNS

tA) Vagus nerves I~

V B)

) Lessersplanchnk nerves

tC) Pudendal nerves

(D) lIiohpogastrk nerves

(E) Greater splam ic l erves

A CT reveals carcinoma in the bOod of the ancreas Vhich blood vessel trut ourses ----~- - -bull ------ --shy

immediately poftterior to the body ofthe pancreas is the m~t likely to be oompressed

(A) Splenk artery

(B) Abdominal aorta (C) Portal vein

(1) Splenic vein

(E) Renal vein

A patient has a penrln1l1ng uker of the posterior wall ot the br~l part ot the (lUooenmn llkh blood vessel is subject to erosion

(A) Common hepatic artery

(B) Gastroouodenal artery

(C) Proper hevatic artery

(D) Celiac artery

(E) Anterior inferior 11amrelltlcoduodcnal attery

Your patient has been diagnosed -ith a carcinoma locallted to the head and l~e(k of the pancreas Another clinical sign would be

A esophageal varices

(8) hemorrhoids

C) a caput medusa

(D) increased pra Teuro n th~ hepatic veins

(E) enlarged right supra lavkular lymph nodes

Wltkh of the foUowing structures develops in the ventral mesentery

(A) Spleen

(B) Jeiunum (C) Head of1ht pancreas (D) Transverse colon (E) Stomach

ti l Uw ~ littwin~ f( S-t lil oai Imdge ~ hi(h or tbt la~)d J truetur tgt liJ llntn nl) he hl p UC iJd [IIi ell

c o

A) drains Ie tht infCrior a La aI

R t middot~nfl0 ~ill to th~ lunlgtn of h i dtlndCrlllfH

(e) m t bull JiJattd on tl l J n T ~H

D ) sup Lc O VSlt I Hlid bhtu l 1 li - -I un oid

( ) U~tpli(t tr j middottUh~ 1 v(( b~nt rfK n1ilc~Zm

ANSWERS AND EXPLANATIONS

Answer E The spleen is t hlttnopodicand lymph organ demlted from mesoderm

Answ~ R Al1 tlmphalocele is caused by it failure of the nlidgut to return to the ahdomir nat cavity after herniation into the umbiliau Stalk Choices Aand D maybe seen in infants with Down syndrome choice D ~s the specific CBuse ofduudcnal JtiCSitt Choice C is (ile cause of gclstrosbisis and Choice B nsults iu a Meurolktldivertku1-tlB

Answer B The fundus ofthe stomach is suppHed by soort gastric brunches of the splenic altery The splenic artery supplies the body and tail of the pancreas part of the greater curvature of the sttmla(h and the spleen Te jejunum part of the head of the pancreas and tht~ duodenum distal to the entrance of the commOll bile duct are supplied by the superior mesenterk artery clll~l ~be less r ctlt1ature cmd the pylQric antrum are supplied by the right and lei gastric art(ries

AnSWftt C Tbeomental bursa or lesser ~ritoneaj sac lies direcdy posterior to the proxshyimal part of the duodeTtlm and the stomach and would be the first site where stomach contents ~Ott1d be fpoundluncL

Answer C A defect in a llleuropcritoneal membrane (uswlly the left) is the typical site of i1 cc-ngenitlI diilphragluatic hemia llere the membr4ne fails to dose ()pound( of the perishycCirdiopcritulleal canals

Answer E DuoJenal atresia and aganglionic megacoion are congwitaI defects S~Il in patients with Dowmiddotnsyndrome

Answer D RulaTgemt~llt of and retrograde flow in g~lstrk vel_ns in particlJl~r the kft gas~ tricveins dilates the capillary bed in rhe wall of the esophagus in (ases of porta yper~

tension Blood flow would increase in and dilampte tribntarkgts of the (lZygOUS vein on the other side of the capiUary bed but flow in this vein is in the typical direction t()ward the superior vena cava Paraumbiii(ltU vein eilgorgement contributes to a caput medusH Splenic ~nlargement might prc~nt with 5plcnonlegaly and balt-kflow in to tlu superior m~~ntclic vein occurs but is asymptomatic

Answer D The patient hagt an indirect inguinal hernia whi~h emerges from the antt-rior abdominal wall through the deep inguinltilling Theeep ring is a fault in the transv~rshysaUs fascia this I~yer wiIJ be penetrated first by the hernia

An~Wer C The ilioinguinal nenc which provides sens~llion to the lnedlal thigh ltmclanteshytior SClotunl pass~lt th rough the 5uperfh_ial inguinal ring ind $subject to inj i1T) becaus-e

it is in the operatitm Held of the erniorrhapny

Auswer B The leMHr splanchnic nerves are sympathdic nerVlts that carry viscera l sensashytlltgtrogt ftom illtllt1m~d ()J stietched gust (itinteitinal ~tructures (in this case the pprndix) into tnt eNS Lesser splanchnic ntTYcsarisc from thmiddot T9--T12 spinal cord segments lt1nd provide sympathetic innenation tD rnidgut siruc1ures whiCh include CLe app~JldD Viscera] Pain arising from affecLed Inidgut ampt 1C1ure is referred over the same dl- matorne~ of spinal segrnertts v-hich provide the sympathetic Innervation n this G1SC of appendicitis the invohen~n t of the ltire) of t e unlhHku indud s the T 10 dermatome

Answer B Of the five choices onty the dscending olon is retroperiton~al aldwould be a lik ~ ( choice to be seen immediately a(~jilcent to t11e posterior abdominal middotn~L

Amwen D The SpltftlC ~-ein ourses posterior to the body of the panneas m its way tt drain into the superior mCSfttltlri( vein

Answcr B TILt glstrodllolticnal artery 1 direct hIamh of the comrootl hepatic artery courses immediately pt))iwri() to the duodenum and is slbject to erosion

Answer B Carcinoma of th pan middott3S in the 1tilt1 may compreampgt the portltil vein at irs orishygill The poTtai vcin is fomled when the splenic vein jQiaswith tfie superior meStllt eric vein The inferiot mesenteric vein joins the ~plenjc vein just priOT to tlli~ point at which the splenic joins the superior Jlleit1ltcri( vein Increescd venous presslu in the inferior mesenteric vein is a cause of emo hoid~

Answer C The- velltral pancreas wilich forms most of the head of the p ~ncr as develops in the ventral mes(ntery as antutgrowth of the hepatic diverticulum Th~ hepatic divershyticulull induding the biIJary appa~atus develops in tbe ventral mesentery of the foregut

Answer~ A The superior mesenteric ~in joins with the spienkvein to form the hepatic portal vciu

Answer D The structure at gttlK is the proper hepatic artery~ whkh suppUesoxygenated b middotood to the liver

MAKE SURE YOU KNOW the diff bw Rectus Sheath above and below the arcuate line

ABOVE

Aponeurosis of xiiltmal obllque musclo

Extemll f)biquw musde

Reotln ilbdomlnls musole S~in

Internal 9bliquQ mY~QI

AponeUfOsi$ of hJH$V~~S Lir9a a lb lbdolTlin~ musolo Tri OJV6 rUi

atldomlnis mUS(loe

Sub cutanlilous tiue (tatty ye r)

BElOW

A POrl lJfosis 01 etemal oblique muscle

Aponeul~)sis 01 Internal oblique mU$cl~

Anteriol lay~ of r~ltdus st~ath EXttom1 oblique rnu$cll

Rectus Jbdominis muscle Intoernal Aponeurc-sis of tra~fersU$ oblique muscle-

at-domlnis muscentl ~ Skio

Tra nsvitSus abdomioLs ml)ZClt

TralSVersaHs fascia Medial umQil iegtt1 1i9Jment -and folj

Uldchus Peritoneum (ir median Umbilj~al Suboutane ous

Extraprftone 11ascia

Ymbilimiddot~1 fold)

preu9poundiea1 fascia

tissue (fatty 4nd m~mbr3n(iUS layers)

o Above the arcuate line (A horizontal line 13 of the distance bw the umbilicus and the

pubic symphysis) -10 Aponeurosis divides into an AntPost Laminae

o The Ant Laminae joins EO and Post Laminae joins Trans Abdominis = Ant and Post

RECTUS SHEATH respectively

o BElOW the arcuate line - all 3 aponeurosis join ANTERIOR to rectus muscle to meet its

counterpart in the midline (linea Alba)

o Take away Msg - The abdomen is devoid of a posterior rectus sheath below the

arcuate line and is therefore more vulnerable to herniasinjuries

Question - A physician makes a deep incision in the patients midline immediately superior to

the pubic symphysis which of the following layers is his knife least likely to pass

Rectus Abdominis External Oblique Ant Rectus Sheath Posterior Rectus Sheath All of the

Above

Answer - All of the above None of the other answer choices are midline structures -LINEA

ALBA

Linea Alba has very poor blood supply - doesnt heal well after surgery Therefore this is a

common site for incisional hernias

a Spleen b Transverse colon c Descending colon d Stomach e Pleura

17 Meckels diverticulum is normally found 2 feet proximal from the

a Pyloric sphincter b Lower esophageal sphincter c Ileo-cecal valve d Middle valve of Huston e Anal valve

18 Ulcer in the posterior wall of the first part of the duodenum would erode ___ artery and would cause bleeding

a Left gastric b Right gastric c Hepatic artery proper d Gastroduodenal artery e Middle colic artery

19 An inflamed appendix is identified by a surgeon on the operation table by noting

a The appendicies epiploicae b The convergence of tenia c The artery of Drummond d The mesocolon e The mesosalphinx

20 The nerve which emerges through the psoas major is

a Femoral b Ilio-inguinal c Ilio-hypogastric d Pudendal e Subcostal

21 The right gonadal vein drains into the

a Azygos b Hemiazygos c Inferior Vena Cava d Right renal vein e Left renal vein

22 The hepatocytes in the liver is derived from

a Ectoderm b Endoderm c Mesoderm

d Neural ectoderm

23 Abscess in the lumbar vertebrae due to tuberculosis would spread to the adjacent muscle which is

a Psoas Major b Iliacus c Quadratus lumborum d Tranversus Abdominis

24 The anterior wall of the inguinal canal is formed by

a External oblique and transverses abdominis b External oblique and fascia transversalis c Internal oblique and external oblique d Internal oblique and transverses abdominis e Fascia transversalis and peritoneum

Meckels diverticulum is a result of which of the following developmental abnormalities shy

A Failure of the vitelline duct to close

B Failure of the herniated intestinal loop to retract into the abdomen

C Failure of the urachus to close

D Failure of the midgut to rotate

E Failure of the hepatic duct to close

Explanation

Meckels diverticulum is a result of the persistence of the proximal part of the vitelline duct This

diverticulum is usually found about 2 feet proximal to the ileocecal junction and is usually about 2 inches

long It is present in about 2 of the popUlation It may be the site of ectopic pancreatic tissue or gastric

mucosa and may develop inflammatory processes and ulcerations Acute Meckels diverticulitis

simulates appendicitis

Which of the following veins carries blood from the esophagus to the portal vein The

A right gastric vein

B left gastric vein c splenic vein D azygos vein

E left gastroepiploic vein

Explanation

The left gastric vein a direct branch of the portal vein drains blood from the lesser curvature of the

stomach and the inferior portion of the esophagus Because branches of the portal vein do not have

valves blood can flow in a retrograde path when there is an obstruction to flow through the portal system or liveL Rlooci Cln then flow from the nortl] vein thr()1Ph the left PRstric vein to the esonhlPlIS lno

through venous communications within the submucosa of the esophagus to esophageal veins that drain

into the azygos vein The increase in blood flow through the esophageal submucosal veins results in esophageal varices

On the posterior wall of the abdomen the celiac ganglion A contains cell bodies of postganglionic parasympathetic neurons B is synapsed upon by neurons in the posterior vagal trunk C is synapsed upon by neurons in the greater splanchnic nerve D contains sensory cell bodies of lumbar spinal nerves E contains cell bodies of neurons that cause an increase in the rate of peristasis

Explanation The celiac ganglion is one of the preaortic ganglia of the sympathetic nervous system It contains cell bodies of postganglionic sympathetic neurons The sympathetic splanchnic nerves contain preganglionic sympathetic neurons that pass through the sympathetic chain without synapsing These splanchnic nerves go to the preaortic ganglia to synapse The greater splanchnic nerve contains preganglionic neurons from spinal cord segments T5-T9 This nerve synapses in the celiac ganglion The nerve fibers in the vagal trunks are preganglionic parasympathetic fibers that go to the walls of the organs that they will innervate and synapse on postganglionic parasympathetic neurons in the walls of those organs Cell bodies of sensory neurons in the abdomen are found in the dorsal root ganglia or the sensory ganglia of the vagus nerve Sympathetic innervation decreases the rate of peristalsis parasympathetic innervation increases the rate of peristalsis

Which of the following pairs of arteries will allow blood to bypass an occlusion of the celiac trunk

A Left gastric artery-right gastric artery

B Left gastroepiploic artery-right gastroepiploic artery

C Superior pancreaticoduodenal artery-inferior pancreaticoduodenal artery

D Splenic artery-common hepatic artery

E Left gastric artery - proper hepatic artery

Explanation The anastoOlosis of a branch of the celiac trunk and a branch of the superior mesenteric artery will

provide collateral circulation around an occlusion of the celiac trunk Each of the other choices pair

branches of the celiac trunk therefore these will not provide collateral flow around the obstruction of the

celiac trunk The left gastric splenic and common hepatic arteries are direct branches of the celiac trunk

The right gastric artery is a branch of the proper hepatic artery which is a branch of the common hepatic artery The left gastroepiploic artery is a branch of the splenic artery The right gastroepiploic artery is a

branch of the gastroduodenal artery whlch is a branch of the common hepatic artery

Which of the following organs has appendices epiploica The

A sigmoid colon

Bjejunum

C duodenum

D stomach E esophagus

Explanation Appendices epiploica are characteristic of the colon Appendices epiploica are subserosal accumulations

of fat None of the organs of the gastrointestinal tract has appendices epiploica except the colon

Page 28: Chirag's Abdomen Review

excess secreted hydrogen ion is reabsorbed across the injured gastric mucosa In general a defect in gastric mucosal defense is the more important local physiologic

A patient presents with symptoms of duodenal obstruction caused by an annular pancreas Annular pancreas is caused by

A rotation of the dorsal pancreatic bud into the ventral mesentery B rotation of the ventral pancreatic bud into the dorsal mesentery

fJ failure of the major and minor pancreatic ducts to fuse ~ ~ cleavage of the ventral pancreatic bud and rotation of the two portions in opposite directions around -the duodenum E formation of one pancreatic bud instead of two

Explanation Normally the ventral pancreatic bud rotates around the gut tube to reach the dorsal pancreatic bud The two buds fuse to form a single pancreas and the distal portions of the two ducts fuse The ventral pancreatic bud forms the inferior portion of the head of the pancreas the uncinate process and the major pancreatic duct (of Wirsung) The dorsal pancreatic bud forms the superior part of the head the neck body and tail and the minor pancreatic duct (of Santorini) Annular pancreas is the result of the ventral pancreatic bud dividing into two portions before it rotates into the dorsal mesentery Each portion rotates in opposite directions to get to the dorsal mesentery thus encircling the duodenum The presence of annular pancreas can constrict the duodenum thus obstructing its lumen

In n _ phranlc----

Gon ~l ----_1 Lum bltano

~~--- CornmQ1t bull ac

+-~4--- lnlllirnaJ ilic

xtem iliac

OBJECTIVE - Identify the blood supply to each of the structures listed in the table on the previous page

Ill give you a head start

FOREGUT - Supplied bV Celiac Tru nk (T12)

Proper hepatic

GastiooUod 13Jafter

1nferlor pancreaticoduodenal artery

Common epatlc

Lett gas ric iiirtery

Spfen artery

shy Gastroepiphgtic artery

~ Superior mesenteric 8rtfry

~

1 Esophagus is a derivative of the foregut so its blood supply originates from the celiac trunk

(T12) The predominant blood supply to abdominal portion of the esophagus is the Esophageal

A (Branch of L Gastric) The venous drainage of the esophagus is particularly important because

it is 1 of 3 clinically relevant sites of Portal Caval anastamoses The Portal Esophageal Vein

meets the Caval Azygos System Persistent bleeding manifests as Esophageal Varices - a fata I

condition

2 The Stomach is also a derivative of the foregut has EXTENSIVE blood supply and is very high

yield on anatomy exams The lesser curvature is supplied superiorly by the L Gastric A (1 of 3

major branches ofthe Celiac trunk) and inferiorly by the R Gastric A ( a branch ofthe proper

Hepatic A) The greater curvature is supplied superiorly by the L Gastroepiploic A (a major

branch of the splenic A) and inferiorly by the R Gastroepiploic A

The Short Gastric arteries (branches of Splenic Artery) supply the fundus of the stomach and

are referred to as EIID ARTERIES because they have no collateral blood supply Therefore if the

splenic artery were occluded (ex - increased pressure in the ommental bursa) - there would be

ischemia to the fundus of the stomach Venous drainage of the stomach is extensive via various

veins lead ing to the portal system Posterior to the stomach the IMV joins the splenic V which

joins the SMV to form the PORTAL VEIN ADAMS

3 Duodenum blood supply has high clinical relevance because it is the junction of the foregut and

midgut and therefore is the site of anastamoses between branches ofthe Celiac Trunk (main

foregut artery) and the Superior Messenteric Artery (main midgut artery) The Proper hepatic

artery gives off the gastroduodenal artery which travels behind the 1st part of the duodenum

This point has high clin ical relevance because duodenal ulcers are very common and a posterior

rupture of the 1st part of the duodenum could rupture the gastroduodenal artery causing

traumatic abdominal bleeding The Gastroduodenal artery first gives off the R Gastroepiploic A

(mentioned above) and proceeds as the Superior pancreatico duodenal artery (supplies the

pancreas and duodenum) which anastamoses with the inferior pancreatico duodenal A (branch

of the SMA) This is the junction of foregut and midgut and occurs near the opening of the

bil iary system into the duodenum (ampula of vater) Portal venous drainage here is responsible

for delivering nutrients from digestion to the liver for metabolism Appreciate that the Superior

mesenteric artery (artery of the midgut) branches from the aorta at Ll travels posterior to the

pancreas than moves anteriorly (at the jxn of the pancreatic headbody) and comes over the

3rd4th part of the duodenum Tumor of the head of the pancreas can compress the SMA

4 Jiver blood supply is via the common hepatic artery (major branch of the cel iac trunk) The

common hepatiC becomes the proper hepatic gives off the R gastric A and the Gastroduodenal

A and then joins the common bile duct and the portal vein in the portal triad Clinical- if a

patient were bleeding from the hepatic A a surgeon can stick his fingers in the epiplOic foramen

and squeeze the free edge of the hepatoduodenalligament in order to stop bleeding to the

area Please note that the hepatic a branches into Rand L hepatic A The Right hepatic artery

gives off the cystic artery which supplies the gallbladder Afferent venous supply is via the

Portal vein which is bringing nutrient rich blood to the liver After metabolism takes place

venous blood leaves the liver through the hepatic veins into the IVC PLEASE UNDERSTAND THE

RELATIONSHIP OF THESE STRUCTURES - ADAMSNETIERSNH Etc

5 Pancreas - Head is supplied via the superior and inferior pancreaticoduodenal arteries

(mentioned above) The tail (situated towards the hilum of the spleen) is supplied via the

pancreatic branches of the splenic artery (END ARTERIES) This blood supply is very important

because the endocrine Alpha and Beta Cells from the pancreatic islets of lagerhans are located

towards the tail This is where Insulin and Glucagon is released to the blood

Now complete this for mid and hindgut structures Make sure to note clinically relevant arterial

anastomoses as well as portal caval anastomoses FYI Appendix blood supply SMA + IMA

anastamoses marginal artery Portalcaval rectal veins fhemmorhoids) and periumbilical caput

medusa are high yield THE BUTT THE GUT and THE CAPUT

Abdominal Development

Liver

Ij1f

II wall b

oh liN ~ VltJrti n be- bull

Pancreas

Secondary Retroperitonealization e I~tl r 1 a v-mtrai m ellter

Rotations of the Gut I i Ij (lIl1UtIJ f~ l r tilt

()l td 10 me l-ft and he v

--~--- -~ -~-~

i

I AolaijonjoI~guf I

STOMACH BED (IDENTIFY IN ADAMS)- the structures posterior to the ommental bursa which

support the stomach in the supine position

Abdomnal JQrUI

Splnic vein

OmQ-oul tv~ ) O(s(Jroa)

Lojt(r o m nturrt (hpJtodu o d~n31 Hid

Gadrl)SplerH (g3stroll~nal) IIgam~nt

hiad h~~atogrtricent IIQdmiddotcrt~)

Lt Dome of Diaphragm (why left Look this up in Adams)

Spleen (What is the blood supply)

Left Kidney (What is the blood supply - AND how is it different from the R kidney)

Suprarenal Gland (What is the Arterial AND Venous Blood supply - how are they different)

Pancreas (How does supply differ from Head to Tail What is the SMA Relationship)

Transverse Mesocolon

liver - ADAMSWET - Make sure you look at the liver in wet lab

Left triangular nl1am~nt

ComoaDj ligamnt

Erophg~1 impre$ioo

Hepatio veins

In1erior -ifena middotr3)Ia

Fibrous appendix o-t

live

impr~j on

Heprorendl p~rtion of Q)(Qllary ligament

Righllri~n9ul r 1I~met

(Common) bile quol

Gr)mmCtr~ hepatic dlJct

Ccentic duct

Duodenal impression

GaJdate p-fr)~S

Hepatic artgtrl prop-f iiiiila - Faloiform ligament

_ - shy Round ligamen liver

~--F-- CoJio imprgt-ssi-on

Prta heptis

Identify the lobes impressions and embryonic remnants associated with the liver

Caudate Lobe Quadrate Lobe Right Lobe Left Lobe Round ligament Falciform Ligament

Ligamentum Venosum (what is its fxn in embryonic life) Hepatic Veins (NOT PART OF THE

PORTAL TRIAD) IVC PORTAL TRIAD - Contents relationship cross section etc Know the

Galbladder relationship to the lobes of the liver

Biliary Duct System - Make sure you understand the sequence of these structures - BE ABLE TO

DRAW A FLOW CHART

TPVd i

t

I t

1 __ Cm-(r

patk GlJet

I

J

Clinical = JAUNDICE is caused by anything that prevents delivery of bile to intestine Tumor of the

head of the pancreas Stones etc Patient will have pale stools and yellowish colored mucus

membranes

Clinical- Any scenario that tells you the patient has BILLOUS VOMIT means that the obstruction to

the flow of digestive contents is after the Ampulla of Vater (Site of Entry of Billiary system to the

duodenum) - ie Duodenal Atresia

Spleen -located posterior to the mid axillary line between ribs 9 and 11 Make sure you know that

the 10th rib is the main axis of the spleen and this organ is susceptible to injury (stab wound errant

thoracoce ntesis etc)

The spleen is derived from mesodermal cells - NOT THE GUT TUBE

The spleen rests on the left colic flexure associates with the tail of the pancreas Know the

structures entering the Hilum of the spleen

Sh rt O~-t~ic 1 0(0 10 rtiltSPIric Iloa nt

(cut)

Peritoneum - similar concept to Pleura - think of a fist in a balloon

Visceral Peritoneum - Layer of balloon touching your fist

Parietal Peritoneum - Layer of balloon not touching your fist

Your fist represents the organ your wrist is the hilum and your arm contains the blood supply

entering the organ

Appreciate that there will never be organs in the peritoneal cavity - rather these organs invaginate

the cavity Kaplan videos

RULES OF NOMENCLATUREshy

1 Organ completely surrounded by peritoneum - peritoneal organ

2 Organ partially surrounded by peritoneum- Retroperitoneal

3 Peritoneum surrounding peritoneal organ is VISCERAL peritoneum

4 Peritoneum surrounding retroperitoneal organ is PARIETAL peritoneum

5 Peritoneum connecting visceral to parietal is called messentary 2 messentaries in the

gut Dorsal (to the gut tube) and ventral (to the gut tube) messentary

Aorta is in Retro peritoneal position - but blood must reach peritoneal position - vessels travel through

messentary All peritoneal organs will have blood supply reaching through messentary

-Mesentery is a 2 layer peritoneum with a neurovascular communication between body wall and organ

- Ligament connects one organ with another or to the abdominal wall (Ommentum = ligament)

lesser Ommentum (attach lesser curvature of stomach and duodenum to liver) =Hepatoduodenal

Ligament and Hepatogastric Ligament

Has a Superior and Inferior Recess (Accumulation of Fluid in Ascites)

Communicates with the greater sac through the epiplic foramen (what structures pass through

this foramen)

Boundaries - you must be able to visualize this

o Anterior - stomach

o Posterior - parietal peritoneum pancreas

o Superior - superior recess (bw diaphragm and coronary ligament)

o Inferior -Inferior recess (bw layers or greater momentum

Greater Ommentum (attach greater curvature of stomach) Gastrophrenic ligament Gastrosplenic

ligament gastrocolic ligament

The greater omentum is the largest peritoneal fold It consists of a double sheet of peritoneum folded on itself so that it is made up of four layers The two layers which descend from the greater curvature of the stomach and commencement of the duodenum pass in front of the small intestines sometimes as low down as the pelvis they then turn upon themselves and ascend again as far as the transverse colon where they separate and enclose that part of the intestine

ABDOMINAL PAIN

Parietal Peritoneum - supplied by same vasculature lymphatics and nerves supplying body wall it

lines and diaphragm Sensitive to pain pressure heat cold well localized

Visceral Peritoneum - supplied by same vasculature lymphatics and somatic nerve of organ it covers

Insensitive to touch heat cold and laceration - referred to dermatome of spinal ganglia providing

sensory fibers Where does appendicitis refer to

Foregut pain - epigastric area (ie - cholycystitis)

Midgut pain - periumbilical area (ie - appendicitis)

Hindgut Pain - suprapubic area (ie - diverticulitis)

Extra ImagesConcepts

ll~_____-

FalifCtrm ligament oind r~ud ligamet f Ilver

Blood from splenio gastriC and inferiof rne$e-rteri v~ins

Ca-I tributaries

Lett gastrio Ifein

Posterior superior pan~reatioodul)denal vaihS

Lott gamo-om~nlal (9aropip lomiddotic) -in

Poq_~ tjol imerl-9-r panCJertlcorllJod-nal veiopound --amp----I- - ~J Right grtr~-omntal

Anwrior interi (gartroepiploic) Jjn

pan euaii cod vl)denal veins middot Inf~Ji (t r mesentric vein

Miqdle (olic vein

Right cl)licvein Sigmoid and rectosigml)id (ei ns

IhH)Collc(~io

--- Mi~dl laquooLJl gtjrltgt

PoM ca vl1 illasto)moses -----shyampoptoageal 2 Paraumbilie-lt11 Inferi or Fectal vei ns

3 Recial 4 REuoperHonea1

Know how the Portal vein is formed I 4 sites of portal caval anastamoses and 1 clinical shunt

Col li t ltt-~ otTl~tI ~nj pc~ 1lt1 turJoG

Ltf 14i1 tImiddot~ artoftl9 on tj phtAt$

L-oftqf 4t t~r 1=laquoIran d 1 bull shy~p l ci rj o fOOOts

Nerves follow the arteries - appreciate the splanchnic nervous system I

Uet~ric branch of left ~nal art

Ureterie branch of righi renal artelY

Left Zld lumbar in and co mlTlunication to as)erdin9 lumbar l(~in Hi ~ht tEZ1~~t~ t3r j t itn ~ nJ l1t- rlnd lfe i r1

Inferior me5nteri~ artery

Notice that the right testicular vein drains directly into the IVC and the right testicular artery drains

directly into the aorta However the left testicular vein drains into the L renal vein at a right angleshy

reason left testicle is lower and more susceptible to varicocele (bag of worms)

Also notice that the left renal vein has a longer course because the IVC is on the right side whereas

the right renal artery has a longer course because the aorta is on the left side

Appreciate the anterior to posterior relationship of structures in the hilum of the kidney - VAP - Vein

Artery Renal Pelvis (Ureter)

11____ __ L_ L_ n VJ __ _ _ t_L I I_ _ L __ L_ I -pound1 bull LI_~-I ____

Posterior View of Head of Pancreas in ( of Duodenum

Celiao hunk

Co mmon ~L~jJth art~ry

GastNduQdonal artrf (partilly in phantn)

P1)Sterior $Up~Jior panCflaticuduodfmal art~r~t

(Co mm on) bile duct

middot~1t~~t-1l---~-~- Right gshomiddotomental (gastoe plp lolc) 3rte (phantomost)

Grener paocre atic art-ry

1n1~rjor pancr-iatlc artery

Jtrifll supejo r pal)oreailcento)dJodenal artr1 (phantom)

Anastomotlo branch

POostetlor bJanch of jo f~ri of pan-reatir(lduodensl drttnj

Anterio r branch of i flferior palcreati~)duodenal art~(phan1om)

Notice the extensive blood supply to the pancreas and duodenum via the branches of the celiac trunk

Notice collateral supply from SMA branches - makes sense bc this is the jxn of foregutmidgut

Identify the vessels in this arteriogram

Hiltid i)f N~ck oi B)dvof Tail 01 pa nereas pan cent~as P-nmiddot-reas panCtCas

I nferie v~na cava

jHept1iic p(lrlai v~in

Port1 tnd H~pti lt a ftH prol

Comm on) bll duct

Ouodtnum

~ft colic (sio)Atta~ hmtrlt jt~xJr-ofha~elSe

muo(IIQn

Right ~lIc (h~j)tic)

il~gtture

In1triol m~oten lIein (rttr op~ritoMdO

SlJp efl or mes~n~fiC amrV and lipln

KNOW YOUR NEIGHBORHOOD

Questions

vVhiJh structure supplied by a bnmdlof the cclia( artery is not derivcd from foregut LemCJUCrITI

(A) Head of the pancte-a5

CD) Pyloric duolenum

Cystkduct

( Liver hepatocyt~~

~F) Body of the spleen

An infant presents with an omrhaJucele at birth -hi oJ the [oHm illg applies to his cM1-dition

(A) It is 31so seen ill p4titnts with aganghonic megacolon

(11) ft reuirs from a fal1ure of resorption of theviteUine d let

(C) It results from herniation at the-site of regression of the right umbilk vein

DJ It is caustd by faihtrc of recanalization of the midgut part of the duodenum

~ It ill camioo by a failuIt vf the midgul to return to the abGQminal uity after herniashytion in-n the urnbilk s l stalk

Ot er than the spleen occlusion Cif the spit-Ilk artery at its odgin wm most likely affect die blood supply to jllch st cnud

(A) Jejunum

(B) Body of th pal1~lltas

(C) LeSStT Cllmiddotlaturc of tl )toma-ch

(D Duodenum dista to the entrance of the Ornmou bile duct

E Fundus of the stomach

A 38-yeu-old batL~er with a history of heartburn suddenly experiences excluciating pain in the (plgastric region of th~ abdomeu SurgCry is perf~rme immediard y upon admisshysion to the 1IlcrgCJliy tuomh~re i~ evidence uf a ruptured ulcer in the posterior waU of the stomach Vhere will a surgeon first fi nd the stomach contenlSf

A) Greater p4ritoneal sac

rB) Cul~de-s~c of Douglas (--

C Omental bursa ~

--D) Paracolic gutter

rEj Between -he panttal perimltum and the posterior body wal1

At birth an infant presents with a st()ma~ rb~tbas~njJled jfltotb~diaplfagru 1A1ltre is the defect thatresulied iiitJle heini~t()n shy~tsophagealbiatus

7 - rH-- Hiatus for the inferior vena cava

( Pleuroperitoneal membrane -(0) Septum transvcrsum

(E) Right Crlt~

An infant born with DOVv7l syndrome presents with bili()u~ vomiting Ahat congenital defect does the infant have

(A) Pyloric stenosis

(B) Meckel diverticulum C) Ornphaloce1e

(D) Gastroschisis

( ~ ) Duodenal atresia y A patient with cirrhosis of the liver presents with ~ bacalvaricestnlreased retrograde pressure in which veins caused the varices

(A) Paraumuilical

(B) Splenic

(ct AzygltJus

(15))G~trk ( (-F) Superior mesemeric

A htaltby 3-year~old male patient experiences a hernial sa protruding from the anterior abdominal wall about halfway between me anterior superior ilia spine and the pubk tuberde Pulsations of al1 artery are palpated medial to the protrusion site through the abdominal walL Which layer of the anterior abdominal wall will first be traversed by the

1hctma

fA) Rectus sheath (B) External oblique aponeurosis

(C) Inguinal ligament

lD) Transversalis fusda

(E) Cremasteric fa~cia

After 5urgi(aj ffpair of a hernia the patient tXperienccs mtmlgtness in the skin on the anteshyrior aspect of the S(Totum_ Vhaf nerve may have been lesioned during thehemiorrhaphy

(A) Femoral

(B) Obturator

(C) Ilioinguinal

(D) lliohypogastrk

(E) Pudendal

A 23~year-LJld female secretary il1 good health ~-uddcn1) doubles over with pain in the a ea of the 1JmbRicu$ Sbe feels vartn and ltneasy and has no appetite That night the pain seems to have mQved to the tower right abdominal regjol1 and she calls her family doctor who then arranges for an ambulance to pk-k her up and take her to the hospitaL Wh ell ntn~ perceived in the area of the urnbilirus most Hkely carried lhe pairfu I sensations into the eNS

tA) Vagus nerves I~

V B)

) Lessersplanchnk nerves

tC) Pudendal nerves

(D) lIiohpogastrk nerves

(E) Greater splam ic l erves

A CT reveals carcinoma in the bOod of the ancreas Vhich blood vessel trut ourses ----~- - -bull ------ --shy

immediately poftterior to the body ofthe pancreas is the m~t likely to be oompressed

(A) Splenk artery

(B) Abdominal aorta (C) Portal vein

(1) Splenic vein

(E) Renal vein

A patient has a penrln1l1ng uker of the posterior wall ot the br~l part ot the (lUooenmn llkh blood vessel is subject to erosion

(A) Common hepatic artery

(B) Gastroouodenal artery

(C) Proper hevatic artery

(D) Celiac artery

(E) Anterior inferior 11amrelltlcoduodcnal attery

Your patient has been diagnosed -ith a carcinoma locallted to the head and l~e(k of the pancreas Another clinical sign would be

A esophageal varices

(8) hemorrhoids

C) a caput medusa

(D) increased pra Teuro n th~ hepatic veins

(E) enlarged right supra lavkular lymph nodes

Wltkh of the foUowing structures develops in the ventral mesentery

(A) Spleen

(B) Jeiunum (C) Head of1ht pancreas (D) Transverse colon (E) Stomach

ti l Uw ~ littwin~ f( S-t lil oai Imdge ~ hi(h or tbt la~)d J truetur tgt liJ llntn nl) he hl p UC iJd [IIi ell

c o

A) drains Ie tht infCrior a La aI

R t middot~nfl0 ~ill to th~ lunlgtn of h i dtlndCrlllfH

(e) m t bull JiJattd on tl l J n T ~H

D ) sup Lc O VSlt I Hlid bhtu l 1 li - -I un oid

( ) U~tpli(t tr j middottUh~ 1 v(( b~nt rfK n1ilc~Zm

ANSWERS AND EXPLANATIONS

Answer E The spleen is t hlttnopodicand lymph organ demlted from mesoderm

Answ~ R Al1 tlmphalocele is caused by it failure of the nlidgut to return to the ahdomir nat cavity after herniation into the umbiliau Stalk Choices Aand D maybe seen in infants with Down syndrome choice D ~s the specific CBuse ofduudcnal JtiCSitt Choice C is (ile cause of gclstrosbisis and Choice B nsults iu a Meurolktldivertku1-tlB

Answer B The fundus ofthe stomach is suppHed by soort gastric brunches of the splenic altery The splenic artery supplies the body and tail of the pancreas part of the greater curvature of the sttmla(h and the spleen Te jejunum part of the head of the pancreas and tht~ duodenum distal to the entrance of the commOll bile duct are supplied by the superior mesenterk artery clll~l ~be less r ctlt1ature cmd the pylQric antrum are supplied by the right and lei gastric art(ries

AnSWftt C Tbeomental bursa or lesser ~ritoneaj sac lies direcdy posterior to the proxshyimal part of the duodeTtlm and the stomach and would be the first site where stomach contents ~Ott1d be fpoundluncL

Answer C A defect in a llleuropcritoneal membrane (uswlly the left) is the typical site of i1 cc-ngenitlI diilphragluatic hemia llere the membr4ne fails to dose ()pound( of the perishycCirdiopcritulleal canals

Answer E DuoJenal atresia and aganglionic megacoion are congwitaI defects S~Il in patients with Dowmiddotnsyndrome

Answer D RulaTgemt~llt of and retrograde flow in g~lstrk vel_ns in particlJl~r the kft gas~ tricveins dilates the capillary bed in rhe wall of the esophagus in (ases of porta yper~

tension Blood flow would increase in and dilampte tribntarkgts of the (lZygOUS vein on the other side of the capiUary bed but flow in this vein is in the typical direction t()ward the superior vena cava Paraumbiii(ltU vein eilgorgement contributes to a caput medusH Splenic ~nlargement might prc~nt with 5plcnonlegaly and balt-kflow in to tlu superior m~~ntclic vein occurs but is asymptomatic

Answer D The patient hagt an indirect inguinal hernia whi~h emerges from the antt-rior abdominal wall through the deep inguinltilling Theeep ring is a fault in the transv~rshysaUs fascia this I~yer wiIJ be penetrated first by the hernia

An~Wer C The ilioinguinal nenc which provides sens~llion to the lnedlal thigh ltmclanteshytior SClotunl pass~lt th rough the 5uperfh_ial inguinal ring ind $subject to inj i1T) becaus-e

it is in the operatitm Held of the erniorrhapny

Auswer B The leMHr splanchnic nerves are sympathdic nerVlts that carry viscera l sensashytlltgtrogt ftom illtllt1m~d ()J stietched gust (itinteitinal ~tructures (in this case the pprndix) into tnt eNS Lesser splanchnic ntTYcsarisc from thmiddot T9--T12 spinal cord segments lt1nd provide sympathetic innenation tD rnidgut siruc1ures whiCh include CLe app~JldD Viscera] Pain arising from affecLed Inidgut ampt 1C1ure is referred over the same dl- matorne~ of spinal segrnertts v-hich provide the sympathetic Innervation n this G1SC of appendicitis the invohen~n t of the ltire) of t e unlhHku indud s the T 10 dermatome

Answer B Of the five choices onty the dscending olon is retroperiton~al aldwould be a lik ~ ( choice to be seen immediately a(~jilcent to t11e posterior abdominal middotn~L

Amwen D The SpltftlC ~-ein ourses posterior to the body of the panneas m its way tt drain into the superior mCSfttltlri( vein

Answcr B TILt glstrodllolticnal artery 1 direct hIamh of the comrootl hepatic artery courses immediately pt))iwri() to the duodenum and is slbject to erosion

Answer B Carcinoma of th pan middott3S in the 1tilt1 may compreampgt the portltil vein at irs orishygill The poTtai vcin is fomled when the splenic vein jQiaswith tfie superior meStllt eric vein The inferiot mesenteric vein joins the ~plenjc vein just priOT to tlli~ point at which the splenic joins the superior Jlleit1ltcri( vein Increescd venous presslu in the inferior mesenteric vein is a cause of emo hoid~

Answer C The- velltral pancreas wilich forms most of the head of the p ~ncr as develops in the ventral mes(ntery as antutgrowth of the hepatic diverticulum Th~ hepatic divershyticulull induding the biIJary appa~atus develops in tbe ventral mesentery of the foregut

Answer~ A The superior mesenteric ~in joins with the spienkvein to form the hepatic portal vciu

Answer D The structure at gttlK is the proper hepatic artery~ whkh suppUesoxygenated b middotood to the liver

MAKE SURE YOU KNOW the diff bw Rectus Sheath above and below the arcuate line

ABOVE

Aponeurosis of xiiltmal obllque musclo

Extemll f)biquw musde

Reotln ilbdomlnls musole S~in

Internal 9bliquQ mY~QI

AponeUfOsi$ of hJH$V~~S Lir9a a lb lbdolTlin~ musolo Tri OJV6 rUi

atldomlnis mUS(loe

Sub cutanlilous tiue (tatty ye r)

BElOW

A POrl lJfosis 01 etemal oblique muscle

Aponeul~)sis 01 Internal oblique mU$cl~

Anteriol lay~ of r~ltdus st~ath EXttom1 oblique rnu$cll

Rectus Jbdominis muscle Intoernal Aponeurc-sis of tra~fersU$ oblique muscle-

at-domlnis muscentl ~ Skio

Tra nsvitSus abdomioLs ml)ZClt

TralSVersaHs fascia Medial umQil iegtt1 1i9Jment -and folj

Uldchus Peritoneum (ir median Umbilj~al Suboutane ous

Extraprftone 11ascia

Ymbilimiddot~1 fold)

preu9poundiea1 fascia

tissue (fatty 4nd m~mbr3n(iUS layers)

o Above the arcuate line (A horizontal line 13 of the distance bw the umbilicus and the

pubic symphysis) -10 Aponeurosis divides into an AntPost Laminae

o The Ant Laminae joins EO and Post Laminae joins Trans Abdominis = Ant and Post

RECTUS SHEATH respectively

o BElOW the arcuate line - all 3 aponeurosis join ANTERIOR to rectus muscle to meet its

counterpart in the midline (linea Alba)

o Take away Msg - The abdomen is devoid of a posterior rectus sheath below the

arcuate line and is therefore more vulnerable to herniasinjuries

Question - A physician makes a deep incision in the patients midline immediately superior to

the pubic symphysis which of the following layers is his knife least likely to pass

Rectus Abdominis External Oblique Ant Rectus Sheath Posterior Rectus Sheath All of the

Above

Answer - All of the above None of the other answer choices are midline structures -LINEA

ALBA

Linea Alba has very poor blood supply - doesnt heal well after surgery Therefore this is a

common site for incisional hernias

a Spleen b Transverse colon c Descending colon d Stomach e Pleura

17 Meckels diverticulum is normally found 2 feet proximal from the

a Pyloric sphincter b Lower esophageal sphincter c Ileo-cecal valve d Middle valve of Huston e Anal valve

18 Ulcer in the posterior wall of the first part of the duodenum would erode ___ artery and would cause bleeding

a Left gastric b Right gastric c Hepatic artery proper d Gastroduodenal artery e Middle colic artery

19 An inflamed appendix is identified by a surgeon on the operation table by noting

a The appendicies epiploicae b The convergence of tenia c The artery of Drummond d The mesocolon e The mesosalphinx

20 The nerve which emerges through the psoas major is

a Femoral b Ilio-inguinal c Ilio-hypogastric d Pudendal e Subcostal

21 The right gonadal vein drains into the

a Azygos b Hemiazygos c Inferior Vena Cava d Right renal vein e Left renal vein

22 The hepatocytes in the liver is derived from

a Ectoderm b Endoderm c Mesoderm

d Neural ectoderm

23 Abscess in the lumbar vertebrae due to tuberculosis would spread to the adjacent muscle which is

a Psoas Major b Iliacus c Quadratus lumborum d Tranversus Abdominis

24 The anterior wall of the inguinal canal is formed by

a External oblique and transverses abdominis b External oblique and fascia transversalis c Internal oblique and external oblique d Internal oblique and transverses abdominis e Fascia transversalis and peritoneum

Meckels diverticulum is a result of which of the following developmental abnormalities shy

A Failure of the vitelline duct to close

B Failure of the herniated intestinal loop to retract into the abdomen

C Failure of the urachus to close

D Failure of the midgut to rotate

E Failure of the hepatic duct to close

Explanation

Meckels diverticulum is a result of the persistence of the proximal part of the vitelline duct This

diverticulum is usually found about 2 feet proximal to the ileocecal junction and is usually about 2 inches

long It is present in about 2 of the popUlation It may be the site of ectopic pancreatic tissue or gastric

mucosa and may develop inflammatory processes and ulcerations Acute Meckels diverticulitis

simulates appendicitis

Which of the following veins carries blood from the esophagus to the portal vein The

A right gastric vein

B left gastric vein c splenic vein D azygos vein

E left gastroepiploic vein

Explanation

The left gastric vein a direct branch of the portal vein drains blood from the lesser curvature of the

stomach and the inferior portion of the esophagus Because branches of the portal vein do not have

valves blood can flow in a retrograde path when there is an obstruction to flow through the portal system or liveL Rlooci Cln then flow from the nortl] vein thr()1Ph the left PRstric vein to the esonhlPlIS lno

through venous communications within the submucosa of the esophagus to esophageal veins that drain

into the azygos vein The increase in blood flow through the esophageal submucosal veins results in esophageal varices

On the posterior wall of the abdomen the celiac ganglion A contains cell bodies of postganglionic parasympathetic neurons B is synapsed upon by neurons in the posterior vagal trunk C is synapsed upon by neurons in the greater splanchnic nerve D contains sensory cell bodies of lumbar spinal nerves E contains cell bodies of neurons that cause an increase in the rate of peristasis

Explanation The celiac ganglion is one of the preaortic ganglia of the sympathetic nervous system It contains cell bodies of postganglionic sympathetic neurons The sympathetic splanchnic nerves contain preganglionic sympathetic neurons that pass through the sympathetic chain without synapsing These splanchnic nerves go to the preaortic ganglia to synapse The greater splanchnic nerve contains preganglionic neurons from spinal cord segments T5-T9 This nerve synapses in the celiac ganglion The nerve fibers in the vagal trunks are preganglionic parasympathetic fibers that go to the walls of the organs that they will innervate and synapse on postganglionic parasympathetic neurons in the walls of those organs Cell bodies of sensory neurons in the abdomen are found in the dorsal root ganglia or the sensory ganglia of the vagus nerve Sympathetic innervation decreases the rate of peristalsis parasympathetic innervation increases the rate of peristalsis

Which of the following pairs of arteries will allow blood to bypass an occlusion of the celiac trunk

A Left gastric artery-right gastric artery

B Left gastroepiploic artery-right gastroepiploic artery

C Superior pancreaticoduodenal artery-inferior pancreaticoduodenal artery

D Splenic artery-common hepatic artery

E Left gastric artery - proper hepatic artery

Explanation The anastoOlosis of a branch of the celiac trunk and a branch of the superior mesenteric artery will

provide collateral circulation around an occlusion of the celiac trunk Each of the other choices pair

branches of the celiac trunk therefore these will not provide collateral flow around the obstruction of the

celiac trunk The left gastric splenic and common hepatic arteries are direct branches of the celiac trunk

The right gastric artery is a branch of the proper hepatic artery which is a branch of the common hepatic artery The left gastroepiploic artery is a branch of the splenic artery The right gastroepiploic artery is a

branch of the gastroduodenal artery whlch is a branch of the common hepatic artery

Which of the following organs has appendices epiploica The

A sigmoid colon

Bjejunum

C duodenum

D stomach E esophagus

Explanation Appendices epiploica are characteristic of the colon Appendices epiploica are subserosal accumulations

of fat None of the organs of the gastrointestinal tract has appendices epiploica except the colon

Page 29: Chirag's Abdomen Review

In n _ phranlc----

Gon ~l ----_1 Lum bltano

~~--- CornmQ1t bull ac

+-~4--- lnlllirnaJ ilic

xtem iliac

OBJECTIVE - Identify the blood supply to each of the structures listed in the table on the previous page

Ill give you a head start

FOREGUT - Supplied bV Celiac Tru nk (T12)

Proper hepatic

GastiooUod 13Jafter

1nferlor pancreaticoduodenal artery

Common epatlc

Lett gas ric iiirtery

Spfen artery

shy Gastroepiphgtic artery

~ Superior mesenteric 8rtfry

~

1 Esophagus is a derivative of the foregut so its blood supply originates from the celiac trunk

(T12) The predominant blood supply to abdominal portion of the esophagus is the Esophageal

A (Branch of L Gastric) The venous drainage of the esophagus is particularly important because

it is 1 of 3 clinically relevant sites of Portal Caval anastamoses The Portal Esophageal Vein

meets the Caval Azygos System Persistent bleeding manifests as Esophageal Varices - a fata I

condition

2 The Stomach is also a derivative of the foregut has EXTENSIVE blood supply and is very high

yield on anatomy exams The lesser curvature is supplied superiorly by the L Gastric A (1 of 3

major branches ofthe Celiac trunk) and inferiorly by the R Gastric A ( a branch ofthe proper

Hepatic A) The greater curvature is supplied superiorly by the L Gastroepiploic A (a major

branch of the splenic A) and inferiorly by the R Gastroepiploic A

The Short Gastric arteries (branches of Splenic Artery) supply the fundus of the stomach and

are referred to as EIID ARTERIES because they have no collateral blood supply Therefore if the

splenic artery were occluded (ex - increased pressure in the ommental bursa) - there would be

ischemia to the fundus of the stomach Venous drainage of the stomach is extensive via various

veins lead ing to the portal system Posterior to the stomach the IMV joins the splenic V which

joins the SMV to form the PORTAL VEIN ADAMS

3 Duodenum blood supply has high clinical relevance because it is the junction of the foregut and

midgut and therefore is the site of anastamoses between branches ofthe Celiac Trunk (main

foregut artery) and the Superior Messenteric Artery (main midgut artery) The Proper hepatic

artery gives off the gastroduodenal artery which travels behind the 1st part of the duodenum

This point has high clin ical relevance because duodenal ulcers are very common and a posterior

rupture of the 1st part of the duodenum could rupture the gastroduodenal artery causing

traumatic abdominal bleeding The Gastroduodenal artery first gives off the R Gastroepiploic A

(mentioned above) and proceeds as the Superior pancreatico duodenal artery (supplies the

pancreas and duodenum) which anastamoses with the inferior pancreatico duodenal A (branch

of the SMA) This is the junction of foregut and midgut and occurs near the opening of the

bil iary system into the duodenum (ampula of vater) Portal venous drainage here is responsible

for delivering nutrients from digestion to the liver for metabolism Appreciate that the Superior

mesenteric artery (artery of the midgut) branches from the aorta at Ll travels posterior to the

pancreas than moves anteriorly (at the jxn of the pancreatic headbody) and comes over the

3rd4th part of the duodenum Tumor of the head of the pancreas can compress the SMA

4 Jiver blood supply is via the common hepatic artery (major branch of the cel iac trunk) The

common hepatiC becomes the proper hepatic gives off the R gastric A and the Gastroduodenal

A and then joins the common bile duct and the portal vein in the portal triad Clinical- if a

patient were bleeding from the hepatic A a surgeon can stick his fingers in the epiplOic foramen

and squeeze the free edge of the hepatoduodenalligament in order to stop bleeding to the

area Please note that the hepatic a branches into Rand L hepatic A The Right hepatic artery

gives off the cystic artery which supplies the gallbladder Afferent venous supply is via the

Portal vein which is bringing nutrient rich blood to the liver After metabolism takes place

venous blood leaves the liver through the hepatic veins into the IVC PLEASE UNDERSTAND THE

RELATIONSHIP OF THESE STRUCTURES - ADAMSNETIERSNH Etc

5 Pancreas - Head is supplied via the superior and inferior pancreaticoduodenal arteries

(mentioned above) The tail (situated towards the hilum of the spleen) is supplied via the

pancreatic branches of the splenic artery (END ARTERIES) This blood supply is very important

because the endocrine Alpha and Beta Cells from the pancreatic islets of lagerhans are located

towards the tail This is where Insulin and Glucagon is released to the blood

Now complete this for mid and hindgut structures Make sure to note clinically relevant arterial

anastomoses as well as portal caval anastomoses FYI Appendix blood supply SMA + IMA

anastamoses marginal artery Portalcaval rectal veins fhemmorhoids) and periumbilical caput

medusa are high yield THE BUTT THE GUT and THE CAPUT

Abdominal Development

Liver

Ij1f

II wall b

oh liN ~ VltJrti n be- bull

Pancreas

Secondary Retroperitonealization e I~tl r 1 a v-mtrai m ellter

Rotations of the Gut I i Ij (lIl1UtIJ f~ l r tilt

()l td 10 me l-ft and he v

--~--- -~ -~-~

i

I AolaijonjoI~guf I

STOMACH BED (IDENTIFY IN ADAMS)- the structures posterior to the ommental bursa which

support the stomach in the supine position

Abdomnal JQrUI

Splnic vein

OmQ-oul tv~ ) O(s(Jroa)

Lojt(r o m nturrt (hpJtodu o d~n31 Hid

Gadrl)SplerH (g3stroll~nal) IIgam~nt

hiad h~~atogrtricent IIQdmiddotcrt~)

Lt Dome of Diaphragm (why left Look this up in Adams)

Spleen (What is the blood supply)

Left Kidney (What is the blood supply - AND how is it different from the R kidney)

Suprarenal Gland (What is the Arterial AND Venous Blood supply - how are they different)

Pancreas (How does supply differ from Head to Tail What is the SMA Relationship)

Transverse Mesocolon

liver - ADAMSWET - Make sure you look at the liver in wet lab

Left triangular nl1am~nt

ComoaDj ligamnt

Erophg~1 impre$ioo

Hepatio veins

In1erior -ifena middotr3)Ia

Fibrous appendix o-t

live

impr~j on

Heprorendl p~rtion of Q)(Qllary ligament

Righllri~n9ul r 1I~met

(Common) bile quol

Gr)mmCtr~ hepatic dlJct

Ccentic duct

Duodenal impression

GaJdate p-fr)~S

Hepatic artgtrl prop-f iiiiila - Faloiform ligament

_ - shy Round ligamen liver

~--F-- CoJio imprgt-ssi-on

Prta heptis

Identify the lobes impressions and embryonic remnants associated with the liver

Caudate Lobe Quadrate Lobe Right Lobe Left Lobe Round ligament Falciform Ligament

Ligamentum Venosum (what is its fxn in embryonic life) Hepatic Veins (NOT PART OF THE

PORTAL TRIAD) IVC PORTAL TRIAD - Contents relationship cross section etc Know the

Galbladder relationship to the lobes of the liver

Biliary Duct System - Make sure you understand the sequence of these structures - BE ABLE TO

DRAW A FLOW CHART

TPVd i

t

I t

1 __ Cm-(r

patk GlJet

I

J

Clinical = JAUNDICE is caused by anything that prevents delivery of bile to intestine Tumor of the

head of the pancreas Stones etc Patient will have pale stools and yellowish colored mucus

membranes

Clinical- Any scenario that tells you the patient has BILLOUS VOMIT means that the obstruction to

the flow of digestive contents is after the Ampulla of Vater (Site of Entry of Billiary system to the

duodenum) - ie Duodenal Atresia

Spleen -located posterior to the mid axillary line between ribs 9 and 11 Make sure you know that

the 10th rib is the main axis of the spleen and this organ is susceptible to injury (stab wound errant

thoracoce ntesis etc)

The spleen is derived from mesodermal cells - NOT THE GUT TUBE

The spleen rests on the left colic flexure associates with the tail of the pancreas Know the

structures entering the Hilum of the spleen

Sh rt O~-t~ic 1 0(0 10 rtiltSPIric Iloa nt

(cut)

Peritoneum - similar concept to Pleura - think of a fist in a balloon

Visceral Peritoneum - Layer of balloon touching your fist

Parietal Peritoneum - Layer of balloon not touching your fist

Your fist represents the organ your wrist is the hilum and your arm contains the blood supply

entering the organ

Appreciate that there will never be organs in the peritoneal cavity - rather these organs invaginate

the cavity Kaplan videos

RULES OF NOMENCLATUREshy

1 Organ completely surrounded by peritoneum - peritoneal organ

2 Organ partially surrounded by peritoneum- Retroperitoneal

3 Peritoneum surrounding peritoneal organ is VISCERAL peritoneum

4 Peritoneum surrounding retroperitoneal organ is PARIETAL peritoneum

5 Peritoneum connecting visceral to parietal is called messentary 2 messentaries in the

gut Dorsal (to the gut tube) and ventral (to the gut tube) messentary

Aorta is in Retro peritoneal position - but blood must reach peritoneal position - vessels travel through

messentary All peritoneal organs will have blood supply reaching through messentary

-Mesentery is a 2 layer peritoneum with a neurovascular communication between body wall and organ

- Ligament connects one organ with another or to the abdominal wall (Ommentum = ligament)

lesser Ommentum (attach lesser curvature of stomach and duodenum to liver) =Hepatoduodenal

Ligament and Hepatogastric Ligament

Has a Superior and Inferior Recess (Accumulation of Fluid in Ascites)

Communicates with the greater sac through the epiplic foramen (what structures pass through

this foramen)

Boundaries - you must be able to visualize this

o Anterior - stomach

o Posterior - parietal peritoneum pancreas

o Superior - superior recess (bw diaphragm and coronary ligament)

o Inferior -Inferior recess (bw layers or greater momentum

Greater Ommentum (attach greater curvature of stomach) Gastrophrenic ligament Gastrosplenic

ligament gastrocolic ligament

The greater omentum is the largest peritoneal fold It consists of a double sheet of peritoneum folded on itself so that it is made up of four layers The two layers which descend from the greater curvature of the stomach and commencement of the duodenum pass in front of the small intestines sometimes as low down as the pelvis they then turn upon themselves and ascend again as far as the transverse colon where they separate and enclose that part of the intestine

ABDOMINAL PAIN

Parietal Peritoneum - supplied by same vasculature lymphatics and nerves supplying body wall it

lines and diaphragm Sensitive to pain pressure heat cold well localized

Visceral Peritoneum - supplied by same vasculature lymphatics and somatic nerve of organ it covers

Insensitive to touch heat cold and laceration - referred to dermatome of spinal ganglia providing

sensory fibers Where does appendicitis refer to

Foregut pain - epigastric area (ie - cholycystitis)

Midgut pain - periumbilical area (ie - appendicitis)

Hindgut Pain - suprapubic area (ie - diverticulitis)

Extra ImagesConcepts

ll~_____-

FalifCtrm ligament oind r~ud ligamet f Ilver

Blood from splenio gastriC and inferiof rne$e-rteri v~ins

Ca-I tributaries

Lett gastrio Ifein

Posterior superior pan~reatioodul)denal vaihS

Lott gamo-om~nlal (9aropip lomiddotic) -in

Poq_~ tjol imerl-9-r panCJertlcorllJod-nal veiopound --amp----I- - ~J Right grtr~-omntal

Anwrior interi (gartroepiploic) Jjn

pan euaii cod vl)denal veins middot Inf~Ji (t r mesentric vein

Miqdle (olic vein

Right cl)licvein Sigmoid and rectosigml)id (ei ns

IhH)Collc(~io

--- Mi~dl laquooLJl gtjrltgt

PoM ca vl1 illasto)moses -----shyampoptoageal 2 Paraumbilie-lt11 Inferi or Fectal vei ns

3 Recial 4 REuoperHonea1

Know how the Portal vein is formed I 4 sites of portal caval anastamoses and 1 clinical shunt

Col li t ltt-~ otTl~tI ~nj pc~ 1lt1 turJoG

Ltf 14i1 tImiddot~ artoftl9 on tj phtAt$

L-oftqf 4t t~r 1=laquoIran d 1 bull shy~p l ci rj o fOOOts

Nerves follow the arteries - appreciate the splanchnic nervous system I

Uet~ric branch of left ~nal art

Ureterie branch of righi renal artelY

Left Zld lumbar in and co mlTlunication to as)erdin9 lumbar l(~in Hi ~ht tEZ1~~t~ t3r j t itn ~ nJ l1t- rlnd lfe i r1

Inferior me5nteri~ artery

Notice that the right testicular vein drains directly into the IVC and the right testicular artery drains

directly into the aorta However the left testicular vein drains into the L renal vein at a right angleshy

reason left testicle is lower and more susceptible to varicocele (bag of worms)

Also notice that the left renal vein has a longer course because the IVC is on the right side whereas

the right renal artery has a longer course because the aorta is on the left side

Appreciate the anterior to posterior relationship of structures in the hilum of the kidney - VAP - Vein

Artery Renal Pelvis (Ureter)

11____ __ L_ L_ n VJ __ _ _ t_L I I_ _ L __ L_ I -pound1 bull LI_~-I ____

Posterior View of Head of Pancreas in ( of Duodenum

Celiao hunk

Co mmon ~L~jJth art~ry

GastNduQdonal artrf (partilly in phantn)

P1)Sterior $Up~Jior panCflaticuduodfmal art~r~t

(Co mm on) bile duct

middot~1t~~t-1l---~-~- Right gshomiddotomental (gastoe plp lolc) 3rte (phantomost)

Grener paocre atic art-ry

1n1~rjor pancr-iatlc artery

Jtrifll supejo r pal)oreailcento)dJodenal artr1 (phantom)

Anastomotlo branch

POostetlor bJanch of jo f~ri of pan-reatir(lduodensl drttnj

Anterio r branch of i flferior palcreati~)duodenal art~(phan1om)

Notice the extensive blood supply to the pancreas and duodenum via the branches of the celiac trunk

Notice collateral supply from SMA branches - makes sense bc this is the jxn of foregutmidgut

Identify the vessels in this arteriogram

Hiltid i)f N~ck oi B)dvof Tail 01 pa nereas pan cent~as P-nmiddot-reas panCtCas

I nferie v~na cava

jHept1iic p(lrlai v~in

Port1 tnd H~pti lt a ftH prol

Comm on) bll duct

Ouodtnum

~ft colic (sio)Atta~ hmtrlt jt~xJr-ofha~elSe

muo(IIQn

Right ~lIc (h~j)tic)

il~gtture

In1triol m~oten lIein (rttr op~ritoMdO

SlJp efl or mes~n~fiC amrV and lipln

KNOW YOUR NEIGHBORHOOD

Questions

vVhiJh structure supplied by a bnmdlof the cclia( artery is not derivcd from foregut LemCJUCrITI

(A) Head of the pancte-a5

CD) Pyloric duolenum

Cystkduct

( Liver hepatocyt~~

~F) Body of the spleen

An infant presents with an omrhaJucele at birth -hi oJ the [oHm illg applies to his cM1-dition

(A) It is 31so seen ill p4titnts with aganghonic megacolon

(11) ft reuirs from a fal1ure of resorption of theviteUine d let

(C) It results from herniation at the-site of regression of the right umbilk vein

DJ It is caustd by faihtrc of recanalization of the midgut part of the duodenum

~ It ill camioo by a failuIt vf the midgul to return to the abGQminal uity after herniashytion in-n the urnbilk s l stalk

Ot er than the spleen occlusion Cif the spit-Ilk artery at its odgin wm most likely affect die blood supply to jllch st cnud

(A) Jejunum

(B) Body of th pal1~lltas

(C) LeSStT Cllmiddotlaturc of tl )toma-ch

(D Duodenum dista to the entrance of the Ornmou bile duct

E Fundus of the stomach

A 38-yeu-old batL~er with a history of heartburn suddenly experiences excluciating pain in the (plgastric region of th~ abdomeu SurgCry is perf~rme immediard y upon admisshysion to the 1IlcrgCJliy tuomh~re i~ evidence uf a ruptured ulcer in the posterior waU of the stomach Vhere will a surgeon first fi nd the stomach contenlSf

A) Greater p4ritoneal sac

rB) Cul~de-s~c of Douglas (--

C Omental bursa ~

--D) Paracolic gutter

rEj Between -he panttal perimltum and the posterior body wal1

At birth an infant presents with a st()ma~ rb~tbas~njJled jfltotb~diaplfagru 1A1ltre is the defect thatresulied iiitJle heini~t()n shy~tsophagealbiatus

7 - rH-- Hiatus for the inferior vena cava

( Pleuroperitoneal membrane -(0) Septum transvcrsum

(E) Right Crlt~

An infant born with DOVv7l syndrome presents with bili()u~ vomiting Ahat congenital defect does the infant have

(A) Pyloric stenosis

(B) Meckel diverticulum C) Ornphaloce1e

(D) Gastroschisis

( ~ ) Duodenal atresia y A patient with cirrhosis of the liver presents with ~ bacalvaricestnlreased retrograde pressure in which veins caused the varices

(A) Paraumuilical

(B) Splenic

(ct AzygltJus

(15))G~trk ( (-F) Superior mesemeric

A htaltby 3-year~old male patient experiences a hernial sa protruding from the anterior abdominal wall about halfway between me anterior superior ilia spine and the pubk tuberde Pulsations of al1 artery are palpated medial to the protrusion site through the abdominal walL Which layer of the anterior abdominal wall will first be traversed by the

1hctma

fA) Rectus sheath (B) External oblique aponeurosis

(C) Inguinal ligament

lD) Transversalis fusda

(E) Cremasteric fa~cia

After 5urgi(aj ffpair of a hernia the patient tXperienccs mtmlgtness in the skin on the anteshyrior aspect of the S(Totum_ Vhaf nerve may have been lesioned during thehemiorrhaphy

(A) Femoral

(B) Obturator

(C) Ilioinguinal

(D) lliohypogastrk

(E) Pudendal

A 23~year-LJld female secretary il1 good health ~-uddcn1) doubles over with pain in the a ea of the 1JmbRicu$ Sbe feels vartn and ltneasy and has no appetite That night the pain seems to have mQved to the tower right abdominal regjol1 and she calls her family doctor who then arranges for an ambulance to pk-k her up and take her to the hospitaL Wh ell ntn~ perceived in the area of the urnbilirus most Hkely carried lhe pairfu I sensations into the eNS

tA) Vagus nerves I~

V B)

) Lessersplanchnk nerves

tC) Pudendal nerves

(D) lIiohpogastrk nerves

(E) Greater splam ic l erves

A CT reveals carcinoma in the bOod of the ancreas Vhich blood vessel trut ourses ----~- - -bull ------ --shy

immediately poftterior to the body ofthe pancreas is the m~t likely to be oompressed

(A) Splenk artery

(B) Abdominal aorta (C) Portal vein

(1) Splenic vein

(E) Renal vein

A patient has a penrln1l1ng uker of the posterior wall ot the br~l part ot the (lUooenmn llkh blood vessel is subject to erosion

(A) Common hepatic artery

(B) Gastroouodenal artery

(C) Proper hevatic artery

(D) Celiac artery

(E) Anterior inferior 11amrelltlcoduodcnal attery

Your patient has been diagnosed -ith a carcinoma locallted to the head and l~e(k of the pancreas Another clinical sign would be

A esophageal varices

(8) hemorrhoids

C) a caput medusa

(D) increased pra Teuro n th~ hepatic veins

(E) enlarged right supra lavkular lymph nodes

Wltkh of the foUowing structures develops in the ventral mesentery

(A) Spleen

(B) Jeiunum (C) Head of1ht pancreas (D) Transverse colon (E) Stomach

ti l Uw ~ littwin~ f( S-t lil oai Imdge ~ hi(h or tbt la~)d J truetur tgt liJ llntn nl) he hl p UC iJd [IIi ell

c o

A) drains Ie tht infCrior a La aI

R t middot~nfl0 ~ill to th~ lunlgtn of h i dtlndCrlllfH

(e) m t bull JiJattd on tl l J n T ~H

D ) sup Lc O VSlt I Hlid bhtu l 1 li - -I un oid

( ) U~tpli(t tr j middottUh~ 1 v(( b~nt rfK n1ilc~Zm

ANSWERS AND EXPLANATIONS

Answer E The spleen is t hlttnopodicand lymph organ demlted from mesoderm

Answ~ R Al1 tlmphalocele is caused by it failure of the nlidgut to return to the ahdomir nat cavity after herniation into the umbiliau Stalk Choices Aand D maybe seen in infants with Down syndrome choice D ~s the specific CBuse ofduudcnal JtiCSitt Choice C is (ile cause of gclstrosbisis and Choice B nsults iu a Meurolktldivertku1-tlB

Answer B The fundus ofthe stomach is suppHed by soort gastric brunches of the splenic altery The splenic artery supplies the body and tail of the pancreas part of the greater curvature of the sttmla(h and the spleen Te jejunum part of the head of the pancreas and tht~ duodenum distal to the entrance of the commOll bile duct are supplied by the superior mesenterk artery clll~l ~be less r ctlt1ature cmd the pylQric antrum are supplied by the right and lei gastric art(ries

AnSWftt C Tbeomental bursa or lesser ~ritoneaj sac lies direcdy posterior to the proxshyimal part of the duodeTtlm and the stomach and would be the first site where stomach contents ~Ott1d be fpoundluncL

Answer C A defect in a llleuropcritoneal membrane (uswlly the left) is the typical site of i1 cc-ngenitlI diilphragluatic hemia llere the membr4ne fails to dose ()pound( of the perishycCirdiopcritulleal canals

Answer E DuoJenal atresia and aganglionic megacoion are congwitaI defects S~Il in patients with Dowmiddotnsyndrome

Answer D RulaTgemt~llt of and retrograde flow in g~lstrk vel_ns in particlJl~r the kft gas~ tricveins dilates the capillary bed in rhe wall of the esophagus in (ases of porta yper~

tension Blood flow would increase in and dilampte tribntarkgts of the (lZygOUS vein on the other side of the capiUary bed but flow in this vein is in the typical direction t()ward the superior vena cava Paraumbiii(ltU vein eilgorgement contributes to a caput medusH Splenic ~nlargement might prc~nt with 5plcnonlegaly and balt-kflow in to tlu superior m~~ntclic vein occurs but is asymptomatic

Answer D The patient hagt an indirect inguinal hernia whi~h emerges from the antt-rior abdominal wall through the deep inguinltilling Theeep ring is a fault in the transv~rshysaUs fascia this I~yer wiIJ be penetrated first by the hernia

An~Wer C The ilioinguinal nenc which provides sens~llion to the lnedlal thigh ltmclanteshytior SClotunl pass~lt th rough the 5uperfh_ial inguinal ring ind $subject to inj i1T) becaus-e

it is in the operatitm Held of the erniorrhapny

Auswer B The leMHr splanchnic nerves are sympathdic nerVlts that carry viscera l sensashytlltgtrogt ftom illtllt1m~d ()J stietched gust (itinteitinal ~tructures (in this case the pprndix) into tnt eNS Lesser splanchnic ntTYcsarisc from thmiddot T9--T12 spinal cord segments lt1nd provide sympathetic innenation tD rnidgut siruc1ures whiCh include CLe app~JldD Viscera] Pain arising from affecLed Inidgut ampt 1C1ure is referred over the same dl- matorne~ of spinal segrnertts v-hich provide the sympathetic Innervation n this G1SC of appendicitis the invohen~n t of the ltire) of t e unlhHku indud s the T 10 dermatome

Answer B Of the five choices onty the dscending olon is retroperiton~al aldwould be a lik ~ ( choice to be seen immediately a(~jilcent to t11e posterior abdominal middotn~L

Amwen D The SpltftlC ~-ein ourses posterior to the body of the panneas m its way tt drain into the superior mCSfttltlri( vein

Answcr B TILt glstrodllolticnal artery 1 direct hIamh of the comrootl hepatic artery courses immediately pt))iwri() to the duodenum and is slbject to erosion

Answer B Carcinoma of th pan middott3S in the 1tilt1 may compreampgt the portltil vein at irs orishygill The poTtai vcin is fomled when the splenic vein jQiaswith tfie superior meStllt eric vein The inferiot mesenteric vein joins the ~plenjc vein just priOT to tlli~ point at which the splenic joins the superior Jlleit1ltcri( vein Increescd venous presslu in the inferior mesenteric vein is a cause of emo hoid~

Answer C The- velltral pancreas wilich forms most of the head of the p ~ncr as develops in the ventral mes(ntery as antutgrowth of the hepatic diverticulum Th~ hepatic divershyticulull induding the biIJary appa~atus develops in tbe ventral mesentery of the foregut

Answer~ A The superior mesenteric ~in joins with the spienkvein to form the hepatic portal vciu

Answer D The structure at gttlK is the proper hepatic artery~ whkh suppUesoxygenated b middotood to the liver

MAKE SURE YOU KNOW the diff bw Rectus Sheath above and below the arcuate line

ABOVE

Aponeurosis of xiiltmal obllque musclo

Extemll f)biquw musde

Reotln ilbdomlnls musole S~in

Internal 9bliquQ mY~QI

AponeUfOsi$ of hJH$V~~S Lir9a a lb lbdolTlin~ musolo Tri OJV6 rUi

atldomlnis mUS(loe

Sub cutanlilous tiue (tatty ye r)

BElOW

A POrl lJfosis 01 etemal oblique muscle

Aponeul~)sis 01 Internal oblique mU$cl~

Anteriol lay~ of r~ltdus st~ath EXttom1 oblique rnu$cll

Rectus Jbdominis muscle Intoernal Aponeurc-sis of tra~fersU$ oblique muscle-

at-domlnis muscentl ~ Skio

Tra nsvitSus abdomioLs ml)ZClt

TralSVersaHs fascia Medial umQil iegtt1 1i9Jment -and folj

Uldchus Peritoneum (ir median Umbilj~al Suboutane ous

Extraprftone 11ascia

Ymbilimiddot~1 fold)

preu9poundiea1 fascia

tissue (fatty 4nd m~mbr3n(iUS layers)

o Above the arcuate line (A horizontal line 13 of the distance bw the umbilicus and the

pubic symphysis) -10 Aponeurosis divides into an AntPost Laminae

o The Ant Laminae joins EO and Post Laminae joins Trans Abdominis = Ant and Post

RECTUS SHEATH respectively

o BElOW the arcuate line - all 3 aponeurosis join ANTERIOR to rectus muscle to meet its

counterpart in the midline (linea Alba)

o Take away Msg - The abdomen is devoid of a posterior rectus sheath below the

arcuate line and is therefore more vulnerable to herniasinjuries

Question - A physician makes a deep incision in the patients midline immediately superior to

the pubic symphysis which of the following layers is his knife least likely to pass

Rectus Abdominis External Oblique Ant Rectus Sheath Posterior Rectus Sheath All of the

Above

Answer - All of the above None of the other answer choices are midline structures -LINEA

ALBA

Linea Alba has very poor blood supply - doesnt heal well after surgery Therefore this is a

common site for incisional hernias

a Spleen b Transverse colon c Descending colon d Stomach e Pleura

17 Meckels diverticulum is normally found 2 feet proximal from the

a Pyloric sphincter b Lower esophageal sphincter c Ileo-cecal valve d Middle valve of Huston e Anal valve

18 Ulcer in the posterior wall of the first part of the duodenum would erode ___ artery and would cause bleeding

a Left gastric b Right gastric c Hepatic artery proper d Gastroduodenal artery e Middle colic artery

19 An inflamed appendix is identified by a surgeon on the operation table by noting

a The appendicies epiploicae b The convergence of tenia c The artery of Drummond d The mesocolon e The mesosalphinx

20 The nerve which emerges through the psoas major is

a Femoral b Ilio-inguinal c Ilio-hypogastric d Pudendal e Subcostal

21 The right gonadal vein drains into the

a Azygos b Hemiazygos c Inferior Vena Cava d Right renal vein e Left renal vein

22 The hepatocytes in the liver is derived from

a Ectoderm b Endoderm c Mesoderm

d Neural ectoderm

23 Abscess in the lumbar vertebrae due to tuberculosis would spread to the adjacent muscle which is

a Psoas Major b Iliacus c Quadratus lumborum d Tranversus Abdominis

24 The anterior wall of the inguinal canal is formed by

a External oblique and transverses abdominis b External oblique and fascia transversalis c Internal oblique and external oblique d Internal oblique and transverses abdominis e Fascia transversalis and peritoneum

Meckels diverticulum is a result of which of the following developmental abnormalities shy

A Failure of the vitelline duct to close

B Failure of the herniated intestinal loop to retract into the abdomen

C Failure of the urachus to close

D Failure of the midgut to rotate

E Failure of the hepatic duct to close

Explanation

Meckels diverticulum is a result of the persistence of the proximal part of the vitelline duct This

diverticulum is usually found about 2 feet proximal to the ileocecal junction and is usually about 2 inches

long It is present in about 2 of the popUlation It may be the site of ectopic pancreatic tissue or gastric

mucosa and may develop inflammatory processes and ulcerations Acute Meckels diverticulitis

simulates appendicitis

Which of the following veins carries blood from the esophagus to the portal vein The

A right gastric vein

B left gastric vein c splenic vein D azygos vein

E left gastroepiploic vein

Explanation

The left gastric vein a direct branch of the portal vein drains blood from the lesser curvature of the

stomach and the inferior portion of the esophagus Because branches of the portal vein do not have

valves blood can flow in a retrograde path when there is an obstruction to flow through the portal system or liveL Rlooci Cln then flow from the nortl] vein thr()1Ph the left PRstric vein to the esonhlPlIS lno

through venous communications within the submucosa of the esophagus to esophageal veins that drain

into the azygos vein The increase in blood flow through the esophageal submucosal veins results in esophageal varices

On the posterior wall of the abdomen the celiac ganglion A contains cell bodies of postganglionic parasympathetic neurons B is synapsed upon by neurons in the posterior vagal trunk C is synapsed upon by neurons in the greater splanchnic nerve D contains sensory cell bodies of lumbar spinal nerves E contains cell bodies of neurons that cause an increase in the rate of peristasis

Explanation The celiac ganglion is one of the preaortic ganglia of the sympathetic nervous system It contains cell bodies of postganglionic sympathetic neurons The sympathetic splanchnic nerves contain preganglionic sympathetic neurons that pass through the sympathetic chain without synapsing These splanchnic nerves go to the preaortic ganglia to synapse The greater splanchnic nerve contains preganglionic neurons from spinal cord segments T5-T9 This nerve synapses in the celiac ganglion The nerve fibers in the vagal trunks are preganglionic parasympathetic fibers that go to the walls of the organs that they will innervate and synapse on postganglionic parasympathetic neurons in the walls of those organs Cell bodies of sensory neurons in the abdomen are found in the dorsal root ganglia or the sensory ganglia of the vagus nerve Sympathetic innervation decreases the rate of peristalsis parasympathetic innervation increases the rate of peristalsis

Which of the following pairs of arteries will allow blood to bypass an occlusion of the celiac trunk

A Left gastric artery-right gastric artery

B Left gastroepiploic artery-right gastroepiploic artery

C Superior pancreaticoduodenal artery-inferior pancreaticoduodenal artery

D Splenic artery-common hepatic artery

E Left gastric artery - proper hepatic artery

Explanation The anastoOlosis of a branch of the celiac trunk and a branch of the superior mesenteric artery will

provide collateral circulation around an occlusion of the celiac trunk Each of the other choices pair

branches of the celiac trunk therefore these will not provide collateral flow around the obstruction of the

celiac trunk The left gastric splenic and common hepatic arteries are direct branches of the celiac trunk

The right gastric artery is a branch of the proper hepatic artery which is a branch of the common hepatic artery The left gastroepiploic artery is a branch of the splenic artery The right gastroepiploic artery is a

branch of the gastroduodenal artery whlch is a branch of the common hepatic artery

Which of the following organs has appendices epiploica The

A sigmoid colon

Bjejunum

C duodenum

D stomach E esophagus

Explanation Appendices epiploica are characteristic of the colon Appendices epiploica are subserosal accumulations

of fat None of the organs of the gastrointestinal tract has appendices epiploica except the colon

Page 30: Chirag's Abdomen Review

~

1 Esophagus is a derivative of the foregut so its blood supply originates from the celiac trunk

(T12) The predominant blood supply to abdominal portion of the esophagus is the Esophageal

A (Branch of L Gastric) The venous drainage of the esophagus is particularly important because

it is 1 of 3 clinically relevant sites of Portal Caval anastamoses The Portal Esophageal Vein

meets the Caval Azygos System Persistent bleeding manifests as Esophageal Varices - a fata I

condition

2 The Stomach is also a derivative of the foregut has EXTENSIVE blood supply and is very high

yield on anatomy exams The lesser curvature is supplied superiorly by the L Gastric A (1 of 3

major branches ofthe Celiac trunk) and inferiorly by the R Gastric A ( a branch ofthe proper

Hepatic A) The greater curvature is supplied superiorly by the L Gastroepiploic A (a major

branch of the splenic A) and inferiorly by the R Gastroepiploic A

The Short Gastric arteries (branches of Splenic Artery) supply the fundus of the stomach and

are referred to as EIID ARTERIES because they have no collateral blood supply Therefore if the

splenic artery were occluded (ex - increased pressure in the ommental bursa) - there would be

ischemia to the fundus of the stomach Venous drainage of the stomach is extensive via various

veins lead ing to the portal system Posterior to the stomach the IMV joins the splenic V which

joins the SMV to form the PORTAL VEIN ADAMS

3 Duodenum blood supply has high clinical relevance because it is the junction of the foregut and

midgut and therefore is the site of anastamoses between branches ofthe Celiac Trunk (main

foregut artery) and the Superior Messenteric Artery (main midgut artery) The Proper hepatic

artery gives off the gastroduodenal artery which travels behind the 1st part of the duodenum

This point has high clin ical relevance because duodenal ulcers are very common and a posterior

rupture of the 1st part of the duodenum could rupture the gastroduodenal artery causing

traumatic abdominal bleeding The Gastroduodenal artery first gives off the R Gastroepiploic A

(mentioned above) and proceeds as the Superior pancreatico duodenal artery (supplies the

pancreas and duodenum) which anastamoses with the inferior pancreatico duodenal A (branch

of the SMA) This is the junction of foregut and midgut and occurs near the opening of the

bil iary system into the duodenum (ampula of vater) Portal venous drainage here is responsible

for delivering nutrients from digestion to the liver for metabolism Appreciate that the Superior

mesenteric artery (artery of the midgut) branches from the aorta at Ll travels posterior to the

pancreas than moves anteriorly (at the jxn of the pancreatic headbody) and comes over the

3rd4th part of the duodenum Tumor of the head of the pancreas can compress the SMA

4 Jiver blood supply is via the common hepatic artery (major branch of the cel iac trunk) The

common hepatiC becomes the proper hepatic gives off the R gastric A and the Gastroduodenal

A and then joins the common bile duct and the portal vein in the portal triad Clinical- if a

patient were bleeding from the hepatic A a surgeon can stick his fingers in the epiplOic foramen

and squeeze the free edge of the hepatoduodenalligament in order to stop bleeding to the

area Please note that the hepatic a branches into Rand L hepatic A The Right hepatic artery

gives off the cystic artery which supplies the gallbladder Afferent venous supply is via the

Portal vein which is bringing nutrient rich blood to the liver After metabolism takes place

venous blood leaves the liver through the hepatic veins into the IVC PLEASE UNDERSTAND THE

RELATIONSHIP OF THESE STRUCTURES - ADAMSNETIERSNH Etc

5 Pancreas - Head is supplied via the superior and inferior pancreaticoduodenal arteries

(mentioned above) The tail (situated towards the hilum of the spleen) is supplied via the

pancreatic branches of the splenic artery (END ARTERIES) This blood supply is very important

because the endocrine Alpha and Beta Cells from the pancreatic islets of lagerhans are located

towards the tail This is where Insulin and Glucagon is released to the blood

Now complete this for mid and hindgut structures Make sure to note clinically relevant arterial

anastomoses as well as portal caval anastomoses FYI Appendix blood supply SMA + IMA

anastamoses marginal artery Portalcaval rectal veins fhemmorhoids) and periumbilical caput

medusa are high yield THE BUTT THE GUT and THE CAPUT

Abdominal Development

Liver

Ij1f

II wall b

oh liN ~ VltJrti n be- bull

Pancreas

Secondary Retroperitonealization e I~tl r 1 a v-mtrai m ellter

Rotations of the Gut I i Ij (lIl1UtIJ f~ l r tilt

()l td 10 me l-ft and he v

--~--- -~ -~-~

i

I AolaijonjoI~guf I

STOMACH BED (IDENTIFY IN ADAMS)- the structures posterior to the ommental bursa which

support the stomach in the supine position

Abdomnal JQrUI

Splnic vein

OmQ-oul tv~ ) O(s(Jroa)

Lojt(r o m nturrt (hpJtodu o d~n31 Hid

Gadrl)SplerH (g3stroll~nal) IIgam~nt

hiad h~~atogrtricent IIQdmiddotcrt~)

Lt Dome of Diaphragm (why left Look this up in Adams)

Spleen (What is the blood supply)

Left Kidney (What is the blood supply - AND how is it different from the R kidney)

Suprarenal Gland (What is the Arterial AND Venous Blood supply - how are they different)

Pancreas (How does supply differ from Head to Tail What is the SMA Relationship)

Transverse Mesocolon

liver - ADAMSWET - Make sure you look at the liver in wet lab

Left triangular nl1am~nt

ComoaDj ligamnt

Erophg~1 impre$ioo

Hepatio veins

In1erior -ifena middotr3)Ia

Fibrous appendix o-t

live

impr~j on

Heprorendl p~rtion of Q)(Qllary ligament

Righllri~n9ul r 1I~met

(Common) bile quol

Gr)mmCtr~ hepatic dlJct

Ccentic duct

Duodenal impression

GaJdate p-fr)~S

Hepatic artgtrl prop-f iiiiila - Faloiform ligament

_ - shy Round ligamen liver

~--F-- CoJio imprgt-ssi-on

Prta heptis

Identify the lobes impressions and embryonic remnants associated with the liver

Caudate Lobe Quadrate Lobe Right Lobe Left Lobe Round ligament Falciform Ligament

Ligamentum Venosum (what is its fxn in embryonic life) Hepatic Veins (NOT PART OF THE

PORTAL TRIAD) IVC PORTAL TRIAD - Contents relationship cross section etc Know the

Galbladder relationship to the lobes of the liver

Biliary Duct System - Make sure you understand the sequence of these structures - BE ABLE TO

DRAW A FLOW CHART

TPVd i

t

I t

1 __ Cm-(r

patk GlJet

I

J

Clinical = JAUNDICE is caused by anything that prevents delivery of bile to intestine Tumor of the

head of the pancreas Stones etc Patient will have pale stools and yellowish colored mucus

membranes

Clinical- Any scenario that tells you the patient has BILLOUS VOMIT means that the obstruction to

the flow of digestive contents is after the Ampulla of Vater (Site of Entry of Billiary system to the

duodenum) - ie Duodenal Atresia

Spleen -located posterior to the mid axillary line between ribs 9 and 11 Make sure you know that

the 10th rib is the main axis of the spleen and this organ is susceptible to injury (stab wound errant

thoracoce ntesis etc)

The spleen is derived from mesodermal cells - NOT THE GUT TUBE

The spleen rests on the left colic flexure associates with the tail of the pancreas Know the

structures entering the Hilum of the spleen

Sh rt O~-t~ic 1 0(0 10 rtiltSPIric Iloa nt

(cut)

Peritoneum - similar concept to Pleura - think of a fist in a balloon

Visceral Peritoneum - Layer of balloon touching your fist

Parietal Peritoneum - Layer of balloon not touching your fist

Your fist represents the organ your wrist is the hilum and your arm contains the blood supply

entering the organ

Appreciate that there will never be organs in the peritoneal cavity - rather these organs invaginate

the cavity Kaplan videos

RULES OF NOMENCLATUREshy

1 Organ completely surrounded by peritoneum - peritoneal organ

2 Organ partially surrounded by peritoneum- Retroperitoneal

3 Peritoneum surrounding peritoneal organ is VISCERAL peritoneum

4 Peritoneum surrounding retroperitoneal organ is PARIETAL peritoneum

5 Peritoneum connecting visceral to parietal is called messentary 2 messentaries in the

gut Dorsal (to the gut tube) and ventral (to the gut tube) messentary

Aorta is in Retro peritoneal position - but blood must reach peritoneal position - vessels travel through

messentary All peritoneal organs will have blood supply reaching through messentary

-Mesentery is a 2 layer peritoneum with a neurovascular communication between body wall and organ

- Ligament connects one organ with another or to the abdominal wall (Ommentum = ligament)

lesser Ommentum (attach lesser curvature of stomach and duodenum to liver) =Hepatoduodenal

Ligament and Hepatogastric Ligament

Has a Superior and Inferior Recess (Accumulation of Fluid in Ascites)

Communicates with the greater sac through the epiplic foramen (what structures pass through

this foramen)

Boundaries - you must be able to visualize this

o Anterior - stomach

o Posterior - parietal peritoneum pancreas

o Superior - superior recess (bw diaphragm and coronary ligament)

o Inferior -Inferior recess (bw layers or greater momentum

Greater Ommentum (attach greater curvature of stomach) Gastrophrenic ligament Gastrosplenic

ligament gastrocolic ligament

The greater omentum is the largest peritoneal fold It consists of a double sheet of peritoneum folded on itself so that it is made up of four layers The two layers which descend from the greater curvature of the stomach and commencement of the duodenum pass in front of the small intestines sometimes as low down as the pelvis they then turn upon themselves and ascend again as far as the transverse colon where they separate and enclose that part of the intestine

ABDOMINAL PAIN

Parietal Peritoneum - supplied by same vasculature lymphatics and nerves supplying body wall it

lines and diaphragm Sensitive to pain pressure heat cold well localized

Visceral Peritoneum - supplied by same vasculature lymphatics and somatic nerve of organ it covers

Insensitive to touch heat cold and laceration - referred to dermatome of spinal ganglia providing

sensory fibers Where does appendicitis refer to

Foregut pain - epigastric area (ie - cholycystitis)

Midgut pain - periumbilical area (ie - appendicitis)

Hindgut Pain - suprapubic area (ie - diverticulitis)

Extra ImagesConcepts

ll~_____-

FalifCtrm ligament oind r~ud ligamet f Ilver

Blood from splenio gastriC and inferiof rne$e-rteri v~ins

Ca-I tributaries

Lett gastrio Ifein

Posterior superior pan~reatioodul)denal vaihS

Lott gamo-om~nlal (9aropip lomiddotic) -in

Poq_~ tjol imerl-9-r panCJertlcorllJod-nal veiopound --amp----I- - ~J Right grtr~-omntal

Anwrior interi (gartroepiploic) Jjn

pan euaii cod vl)denal veins middot Inf~Ji (t r mesentric vein

Miqdle (olic vein

Right cl)licvein Sigmoid and rectosigml)id (ei ns

IhH)Collc(~io

--- Mi~dl laquooLJl gtjrltgt

PoM ca vl1 illasto)moses -----shyampoptoageal 2 Paraumbilie-lt11 Inferi or Fectal vei ns

3 Recial 4 REuoperHonea1

Know how the Portal vein is formed I 4 sites of portal caval anastamoses and 1 clinical shunt

Col li t ltt-~ otTl~tI ~nj pc~ 1lt1 turJoG

Ltf 14i1 tImiddot~ artoftl9 on tj phtAt$

L-oftqf 4t t~r 1=laquoIran d 1 bull shy~p l ci rj o fOOOts

Nerves follow the arteries - appreciate the splanchnic nervous system I

Uet~ric branch of left ~nal art

Ureterie branch of righi renal artelY

Left Zld lumbar in and co mlTlunication to as)erdin9 lumbar l(~in Hi ~ht tEZ1~~t~ t3r j t itn ~ nJ l1t- rlnd lfe i r1

Inferior me5nteri~ artery

Notice that the right testicular vein drains directly into the IVC and the right testicular artery drains

directly into the aorta However the left testicular vein drains into the L renal vein at a right angleshy

reason left testicle is lower and more susceptible to varicocele (bag of worms)

Also notice that the left renal vein has a longer course because the IVC is on the right side whereas

the right renal artery has a longer course because the aorta is on the left side

Appreciate the anterior to posterior relationship of structures in the hilum of the kidney - VAP - Vein

Artery Renal Pelvis (Ureter)

11____ __ L_ L_ n VJ __ _ _ t_L I I_ _ L __ L_ I -pound1 bull LI_~-I ____

Posterior View of Head of Pancreas in ( of Duodenum

Celiao hunk

Co mmon ~L~jJth art~ry

GastNduQdonal artrf (partilly in phantn)

P1)Sterior $Up~Jior panCflaticuduodfmal art~r~t

(Co mm on) bile duct

middot~1t~~t-1l---~-~- Right gshomiddotomental (gastoe plp lolc) 3rte (phantomost)

Grener paocre atic art-ry

1n1~rjor pancr-iatlc artery

Jtrifll supejo r pal)oreailcento)dJodenal artr1 (phantom)

Anastomotlo branch

POostetlor bJanch of jo f~ri of pan-reatir(lduodensl drttnj

Anterio r branch of i flferior palcreati~)duodenal art~(phan1om)

Notice the extensive blood supply to the pancreas and duodenum via the branches of the celiac trunk

Notice collateral supply from SMA branches - makes sense bc this is the jxn of foregutmidgut

Identify the vessels in this arteriogram

Hiltid i)f N~ck oi B)dvof Tail 01 pa nereas pan cent~as P-nmiddot-reas panCtCas

I nferie v~na cava

jHept1iic p(lrlai v~in

Port1 tnd H~pti lt a ftH prol

Comm on) bll duct

Ouodtnum

~ft colic (sio)Atta~ hmtrlt jt~xJr-ofha~elSe

muo(IIQn

Right ~lIc (h~j)tic)

il~gtture

In1triol m~oten lIein (rttr op~ritoMdO

SlJp efl or mes~n~fiC amrV and lipln

KNOW YOUR NEIGHBORHOOD

Questions

vVhiJh structure supplied by a bnmdlof the cclia( artery is not derivcd from foregut LemCJUCrITI

(A) Head of the pancte-a5

CD) Pyloric duolenum

Cystkduct

( Liver hepatocyt~~

~F) Body of the spleen

An infant presents with an omrhaJucele at birth -hi oJ the [oHm illg applies to his cM1-dition

(A) It is 31so seen ill p4titnts with aganghonic megacolon

(11) ft reuirs from a fal1ure of resorption of theviteUine d let

(C) It results from herniation at the-site of regression of the right umbilk vein

DJ It is caustd by faihtrc of recanalization of the midgut part of the duodenum

~ It ill camioo by a failuIt vf the midgul to return to the abGQminal uity after herniashytion in-n the urnbilk s l stalk

Ot er than the spleen occlusion Cif the spit-Ilk artery at its odgin wm most likely affect die blood supply to jllch st cnud

(A) Jejunum

(B) Body of th pal1~lltas

(C) LeSStT Cllmiddotlaturc of tl )toma-ch

(D Duodenum dista to the entrance of the Ornmou bile duct

E Fundus of the stomach

A 38-yeu-old batL~er with a history of heartburn suddenly experiences excluciating pain in the (plgastric region of th~ abdomeu SurgCry is perf~rme immediard y upon admisshysion to the 1IlcrgCJliy tuomh~re i~ evidence uf a ruptured ulcer in the posterior waU of the stomach Vhere will a surgeon first fi nd the stomach contenlSf

A) Greater p4ritoneal sac

rB) Cul~de-s~c of Douglas (--

C Omental bursa ~

--D) Paracolic gutter

rEj Between -he panttal perimltum and the posterior body wal1

At birth an infant presents with a st()ma~ rb~tbas~njJled jfltotb~diaplfagru 1A1ltre is the defect thatresulied iiitJle heini~t()n shy~tsophagealbiatus

7 - rH-- Hiatus for the inferior vena cava

( Pleuroperitoneal membrane -(0) Septum transvcrsum

(E) Right Crlt~

An infant born with DOVv7l syndrome presents with bili()u~ vomiting Ahat congenital defect does the infant have

(A) Pyloric stenosis

(B) Meckel diverticulum C) Ornphaloce1e

(D) Gastroschisis

( ~ ) Duodenal atresia y A patient with cirrhosis of the liver presents with ~ bacalvaricestnlreased retrograde pressure in which veins caused the varices

(A) Paraumuilical

(B) Splenic

(ct AzygltJus

(15))G~trk ( (-F) Superior mesemeric

A htaltby 3-year~old male patient experiences a hernial sa protruding from the anterior abdominal wall about halfway between me anterior superior ilia spine and the pubk tuberde Pulsations of al1 artery are palpated medial to the protrusion site through the abdominal walL Which layer of the anterior abdominal wall will first be traversed by the

1hctma

fA) Rectus sheath (B) External oblique aponeurosis

(C) Inguinal ligament

lD) Transversalis fusda

(E) Cremasteric fa~cia

After 5urgi(aj ffpair of a hernia the patient tXperienccs mtmlgtness in the skin on the anteshyrior aspect of the S(Totum_ Vhaf nerve may have been lesioned during thehemiorrhaphy

(A) Femoral

(B) Obturator

(C) Ilioinguinal

(D) lliohypogastrk

(E) Pudendal

A 23~year-LJld female secretary il1 good health ~-uddcn1) doubles over with pain in the a ea of the 1JmbRicu$ Sbe feels vartn and ltneasy and has no appetite That night the pain seems to have mQved to the tower right abdominal regjol1 and she calls her family doctor who then arranges for an ambulance to pk-k her up and take her to the hospitaL Wh ell ntn~ perceived in the area of the urnbilirus most Hkely carried lhe pairfu I sensations into the eNS

tA) Vagus nerves I~

V B)

) Lessersplanchnk nerves

tC) Pudendal nerves

(D) lIiohpogastrk nerves

(E) Greater splam ic l erves

A CT reveals carcinoma in the bOod of the ancreas Vhich blood vessel trut ourses ----~- - -bull ------ --shy

immediately poftterior to the body ofthe pancreas is the m~t likely to be oompressed

(A) Splenk artery

(B) Abdominal aorta (C) Portal vein

(1) Splenic vein

(E) Renal vein

A patient has a penrln1l1ng uker of the posterior wall ot the br~l part ot the (lUooenmn llkh blood vessel is subject to erosion

(A) Common hepatic artery

(B) Gastroouodenal artery

(C) Proper hevatic artery

(D) Celiac artery

(E) Anterior inferior 11amrelltlcoduodcnal attery

Your patient has been diagnosed -ith a carcinoma locallted to the head and l~e(k of the pancreas Another clinical sign would be

A esophageal varices

(8) hemorrhoids

C) a caput medusa

(D) increased pra Teuro n th~ hepatic veins

(E) enlarged right supra lavkular lymph nodes

Wltkh of the foUowing structures develops in the ventral mesentery

(A) Spleen

(B) Jeiunum (C) Head of1ht pancreas (D) Transverse colon (E) Stomach

ti l Uw ~ littwin~ f( S-t lil oai Imdge ~ hi(h or tbt la~)d J truetur tgt liJ llntn nl) he hl p UC iJd [IIi ell

c o

A) drains Ie tht infCrior a La aI

R t middot~nfl0 ~ill to th~ lunlgtn of h i dtlndCrlllfH

(e) m t bull JiJattd on tl l J n T ~H

D ) sup Lc O VSlt I Hlid bhtu l 1 li - -I un oid

( ) U~tpli(t tr j middottUh~ 1 v(( b~nt rfK n1ilc~Zm

ANSWERS AND EXPLANATIONS

Answer E The spleen is t hlttnopodicand lymph organ demlted from mesoderm

Answ~ R Al1 tlmphalocele is caused by it failure of the nlidgut to return to the ahdomir nat cavity after herniation into the umbiliau Stalk Choices Aand D maybe seen in infants with Down syndrome choice D ~s the specific CBuse ofduudcnal JtiCSitt Choice C is (ile cause of gclstrosbisis and Choice B nsults iu a Meurolktldivertku1-tlB

Answer B The fundus ofthe stomach is suppHed by soort gastric brunches of the splenic altery The splenic artery supplies the body and tail of the pancreas part of the greater curvature of the sttmla(h and the spleen Te jejunum part of the head of the pancreas and tht~ duodenum distal to the entrance of the commOll bile duct are supplied by the superior mesenterk artery clll~l ~be less r ctlt1ature cmd the pylQric antrum are supplied by the right and lei gastric art(ries

AnSWftt C Tbeomental bursa or lesser ~ritoneaj sac lies direcdy posterior to the proxshyimal part of the duodeTtlm and the stomach and would be the first site where stomach contents ~Ott1d be fpoundluncL

Answer C A defect in a llleuropcritoneal membrane (uswlly the left) is the typical site of i1 cc-ngenitlI diilphragluatic hemia llere the membr4ne fails to dose ()pound( of the perishycCirdiopcritulleal canals

Answer E DuoJenal atresia and aganglionic megacoion are congwitaI defects S~Il in patients with Dowmiddotnsyndrome

Answer D RulaTgemt~llt of and retrograde flow in g~lstrk vel_ns in particlJl~r the kft gas~ tricveins dilates the capillary bed in rhe wall of the esophagus in (ases of porta yper~

tension Blood flow would increase in and dilampte tribntarkgts of the (lZygOUS vein on the other side of the capiUary bed but flow in this vein is in the typical direction t()ward the superior vena cava Paraumbiii(ltU vein eilgorgement contributes to a caput medusH Splenic ~nlargement might prc~nt with 5plcnonlegaly and balt-kflow in to tlu superior m~~ntclic vein occurs but is asymptomatic

Answer D The patient hagt an indirect inguinal hernia whi~h emerges from the antt-rior abdominal wall through the deep inguinltilling Theeep ring is a fault in the transv~rshysaUs fascia this I~yer wiIJ be penetrated first by the hernia

An~Wer C The ilioinguinal nenc which provides sens~llion to the lnedlal thigh ltmclanteshytior SClotunl pass~lt th rough the 5uperfh_ial inguinal ring ind $subject to inj i1T) becaus-e

it is in the operatitm Held of the erniorrhapny

Auswer B The leMHr splanchnic nerves are sympathdic nerVlts that carry viscera l sensashytlltgtrogt ftom illtllt1m~d ()J stietched gust (itinteitinal ~tructures (in this case the pprndix) into tnt eNS Lesser splanchnic ntTYcsarisc from thmiddot T9--T12 spinal cord segments lt1nd provide sympathetic innenation tD rnidgut siruc1ures whiCh include CLe app~JldD Viscera] Pain arising from affecLed Inidgut ampt 1C1ure is referred over the same dl- matorne~ of spinal segrnertts v-hich provide the sympathetic Innervation n this G1SC of appendicitis the invohen~n t of the ltire) of t e unlhHku indud s the T 10 dermatome

Answer B Of the five choices onty the dscending olon is retroperiton~al aldwould be a lik ~ ( choice to be seen immediately a(~jilcent to t11e posterior abdominal middotn~L

Amwen D The SpltftlC ~-ein ourses posterior to the body of the panneas m its way tt drain into the superior mCSfttltlri( vein

Answcr B TILt glstrodllolticnal artery 1 direct hIamh of the comrootl hepatic artery courses immediately pt))iwri() to the duodenum and is slbject to erosion

Answer B Carcinoma of th pan middott3S in the 1tilt1 may compreampgt the portltil vein at irs orishygill The poTtai vcin is fomled when the splenic vein jQiaswith tfie superior meStllt eric vein The inferiot mesenteric vein joins the ~plenjc vein just priOT to tlli~ point at which the splenic joins the superior Jlleit1ltcri( vein Increescd venous presslu in the inferior mesenteric vein is a cause of emo hoid~

Answer C The- velltral pancreas wilich forms most of the head of the p ~ncr as develops in the ventral mes(ntery as antutgrowth of the hepatic diverticulum Th~ hepatic divershyticulull induding the biIJary appa~atus develops in tbe ventral mesentery of the foregut

Answer~ A The superior mesenteric ~in joins with the spienkvein to form the hepatic portal vciu

Answer D The structure at gttlK is the proper hepatic artery~ whkh suppUesoxygenated b middotood to the liver

MAKE SURE YOU KNOW the diff bw Rectus Sheath above and below the arcuate line

ABOVE

Aponeurosis of xiiltmal obllque musclo

Extemll f)biquw musde

Reotln ilbdomlnls musole S~in

Internal 9bliquQ mY~QI

AponeUfOsi$ of hJH$V~~S Lir9a a lb lbdolTlin~ musolo Tri OJV6 rUi

atldomlnis mUS(loe

Sub cutanlilous tiue (tatty ye r)

BElOW

A POrl lJfosis 01 etemal oblique muscle

Aponeul~)sis 01 Internal oblique mU$cl~

Anteriol lay~ of r~ltdus st~ath EXttom1 oblique rnu$cll

Rectus Jbdominis muscle Intoernal Aponeurc-sis of tra~fersU$ oblique muscle-

at-domlnis muscentl ~ Skio

Tra nsvitSus abdomioLs ml)ZClt

TralSVersaHs fascia Medial umQil iegtt1 1i9Jment -and folj

Uldchus Peritoneum (ir median Umbilj~al Suboutane ous

Extraprftone 11ascia

Ymbilimiddot~1 fold)

preu9poundiea1 fascia

tissue (fatty 4nd m~mbr3n(iUS layers)

o Above the arcuate line (A horizontal line 13 of the distance bw the umbilicus and the

pubic symphysis) -10 Aponeurosis divides into an AntPost Laminae

o The Ant Laminae joins EO and Post Laminae joins Trans Abdominis = Ant and Post

RECTUS SHEATH respectively

o BElOW the arcuate line - all 3 aponeurosis join ANTERIOR to rectus muscle to meet its

counterpart in the midline (linea Alba)

o Take away Msg - The abdomen is devoid of a posterior rectus sheath below the

arcuate line and is therefore more vulnerable to herniasinjuries

Question - A physician makes a deep incision in the patients midline immediately superior to

the pubic symphysis which of the following layers is his knife least likely to pass

Rectus Abdominis External Oblique Ant Rectus Sheath Posterior Rectus Sheath All of the

Above

Answer - All of the above None of the other answer choices are midline structures -LINEA

ALBA

Linea Alba has very poor blood supply - doesnt heal well after surgery Therefore this is a

common site for incisional hernias

a Spleen b Transverse colon c Descending colon d Stomach e Pleura

17 Meckels diverticulum is normally found 2 feet proximal from the

a Pyloric sphincter b Lower esophageal sphincter c Ileo-cecal valve d Middle valve of Huston e Anal valve

18 Ulcer in the posterior wall of the first part of the duodenum would erode ___ artery and would cause bleeding

a Left gastric b Right gastric c Hepatic artery proper d Gastroduodenal artery e Middle colic artery

19 An inflamed appendix is identified by a surgeon on the operation table by noting

a The appendicies epiploicae b The convergence of tenia c The artery of Drummond d The mesocolon e The mesosalphinx

20 The nerve which emerges through the psoas major is

a Femoral b Ilio-inguinal c Ilio-hypogastric d Pudendal e Subcostal

21 The right gonadal vein drains into the

a Azygos b Hemiazygos c Inferior Vena Cava d Right renal vein e Left renal vein

22 The hepatocytes in the liver is derived from

a Ectoderm b Endoderm c Mesoderm

d Neural ectoderm

23 Abscess in the lumbar vertebrae due to tuberculosis would spread to the adjacent muscle which is

a Psoas Major b Iliacus c Quadratus lumborum d Tranversus Abdominis

24 The anterior wall of the inguinal canal is formed by

a External oblique and transverses abdominis b External oblique and fascia transversalis c Internal oblique and external oblique d Internal oblique and transverses abdominis e Fascia transversalis and peritoneum

Meckels diverticulum is a result of which of the following developmental abnormalities shy

A Failure of the vitelline duct to close

B Failure of the herniated intestinal loop to retract into the abdomen

C Failure of the urachus to close

D Failure of the midgut to rotate

E Failure of the hepatic duct to close

Explanation

Meckels diverticulum is a result of the persistence of the proximal part of the vitelline duct This

diverticulum is usually found about 2 feet proximal to the ileocecal junction and is usually about 2 inches

long It is present in about 2 of the popUlation It may be the site of ectopic pancreatic tissue or gastric

mucosa and may develop inflammatory processes and ulcerations Acute Meckels diverticulitis

simulates appendicitis

Which of the following veins carries blood from the esophagus to the portal vein The

A right gastric vein

B left gastric vein c splenic vein D azygos vein

E left gastroepiploic vein

Explanation

The left gastric vein a direct branch of the portal vein drains blood from the lesser curvature of the

stomach and the inferior portion of the esophagus Because branches of the portal vein do not have

valves blood can flow in a retrograde path when there is an obstruction to flow through the portal system or liveL Rlooci Cln then flow from the nortl] vein thr()1Ph the left PRstric vein to the esonhlPlIS lno

through venous communications within the submucosa of the esophagus to esophageal veins that drain

into the azygos vein The increase in blood flow through the esophageal submucosal veins results in esophageal varices

On the posterior wall of the abdomen the celiac ganglion A contains cell bodies of postganglionic parasympathetic neurons B is synapsed upon by neurons in the posterior vagal trunk C is synapsed upon by neurons in the greater splanchnic nerve D contains sensory cell bodies of lumbar spinal nerves E contains cell bodies of neurons that cause an increase in the rate of peristasis

Explanation The celiac ganglion is one of the preaortic ganglia of the sympathetic nervous system It contains cell bodies of postganglionic sympathetic neurons The sympathetic splanchnic nerves contain preganglionic sympathetic neurons that pass through the sympathetic chain without synapsing These splanchnic nerves go to the preaortic ganglia to synapse The greater splanchnic nerve contains preganglionic neurons from spinal cord segments T5-T9 This nerve synapses in the celiac ganglion The nerve fibers in the vagal trunks are preganglionic parasympathetic fibers that go to the walls of the organs that they will innervate and synapse on postganglionic parasympathetic neurons in the walls of those organs Cell bodies of sensory neurons in the abdomen are found in the dorsal root ganglia or the sensory ganglia of the vagus nerve Sympathetic innervation decreases the rate of peristalsis parasympathetic innervation increases the rate of peristalsis

Which of the following pairs of arteries will allow blood to bypass an occlusion of the celiac trunk

A Left gastric artery-right gastric artery

B Left gastroepiploic artery-right gastroepiploic artery

C Superior pancreaticoduodenal artery-inferior pancreaticoduodenal artery

D Splenic artery-common hepatic artery

E Left gastric artery - proper hepatic artery

Explanation The anastoOlosis of a branch of the celiac trunk and a branch of the superior mesenteric artery will

provide collateral circulation around an occlusion of the celiac trunk Each of the other choices pair

branches of the celiac trunk therefore these will not provide collateral flow around the obstruction of the

celiac trunk The left gastric splenic and common hepatic arteries are direct branches of the celiac trunk

The right gastric artery is a branch of the proper hepatic artery which is a branch of the common hepatic artery The left gastroepiploic artery is a branch of the splenic artery The right gastroepiploic artery is a

branch of the gastroduodenal artery whlch is a branch of the common hepatic artery

Which of the following organs has appendices epiploica The

A sigmoid colon

Bjejunum

C duodenum

D stomach E esophagus

Explanation Appendices epiploica are characteristic of the colon Appendices epiploica are subserosal accumulations

of fat None of the organs of the gastrointestinal tract has appendices epiploica except the colon

Page 31: Chirag's Abdomen Review

Now complete this for mid and hindgut structures Make sure to note clinically relevant arterial

anastomoses as well as portal caval anastomoses FYI Appendix blood supply SMA + IMA

anastamoses marginal artery Portalcaval rectal veins fhemmorhoids) and periumbilical caput

medusa are high yield THE BUTT THE GUT and THE CAPUT

Abdominal Development

Liver

Ij1f

II wall b

oh liN ~ VltJrti n be- bull

Pancreas

Secondary Retroperitonealization e I~tl r 1 a v-mtrai m ellter

Rotations of the Gut I i Ij (lIl1UtIJ f~ l r tilt

()l td 10 me l-ft and he v

--~--- -~ -~-~

i

I AolaijonjoI~guf I

STOMACH BED (IDENTIFY IN ADAMS)- the structures posterior to the ommental bursa which

support the stomach in the supine position

Abdomnal JQrUI

Splnic vein

OmQ-oul tv~ ) O(s(Jroa)

Lojt(r o m nturrt (hpJtodu o d~n31 Hid

Gadrl)SplerH (g3stroll~nal) IIgam~nt

hiad h~~atogrtricent IIQdmiddotcrt~)

Lt Dome of Diaphragm (why left Look this up in Adams)

Spleen (What is the blood supply)

Left Kidney (What is the blood supply - AND how is it different from the R kidney)

Suprarenal Gland (What is the Arterial AND Venous Blood supply - how are they different)

Pancreas (How does supply differ from Head to Tail What is the SMA Relationship)

Transverse Mesocolon

liver - ADAMSWET - Make sure you look at the liver in wet lab

Left triangular nl1am~nt

ComoaDj ligamnt

Erophg~1 impre$ioo

Hepatio veins

In1erior -ifena middotr3)Ia

Fibrous appendix o-t

live

impr~j on

Heprorendl p~rtion of Q)(Qllary ligament

Righllri~n9ul r 1I~met

(Common) bile quol

Gr)mmCtr~ hepatic dlJct

Ccentic duct

Duodenal impression

GaJdate p-fr)~S

Hepatic artgtrl prop-f iiiiila - Faloiform ligament

_ - shy Round ligamen liver

~--F-- CoJio imprgt-ssi-on

Prta heptis

Identify the lobes impressions and embryonic remnants associated with the liver

Caudate Lobe Quadrate Lobe Right Lobe Left Lobe Round ligament Falciform Ligament

Ligamentum Venosum (what is its fxn in embryonic life) Hepatic Veins (NOT PART OF THE

PORTAL TRIAD) IVC PORTAL TRIAD - Contents relationship cross section etc Know the

Galbladder relationship to the lobes of the liver

Biliary Duct System - Make sure you understand the sequence of these structures - BE ABLE TO

DRAW A FLOW CHART

TPVd i

t

I t

1 __ Cm-(r

patk GlJet

I

J

Clinical = JAUNDICE is caused by anything that prevents delivery of bile to intestine Tumor of the

head of the pancreas Stones etc Patient will have pale stools and yellowish colored mucus

membranes

Clinical- Any scenario that tells you the patient has BILLOUS VOMIT means that the obstruction to

the flow of digestive contents is after the Ampulla of Vater (Site of Entry of Billiary system to the

duodenum) - ie Duodenal Atresia

Spleen -located posterior to the mid axillary line between ribs 9 and 11 Make sure you know that

the 10th rib is the main axis of the spleen and this organ is susceptible to injury (stab wound errant

thoracoce ntesis etc)

The spleen is derived from mesodermal cells - NOT THE GUT TUBE

The spleen rests on the left colic flexure associates with the tail of the pancreas Know the

structures entering the Hilum of the spleen

Sh rt O~-t~ic 1 0(0 10 rtiltSPIric Iloa nt

(cut)

Peritoneum - similar concept to Pleura - think of a fist in a balloon

Visceral Peritoneum - Layer of balloon touching your fist

Parietal Peritoneum - Layer of balloon not touching your fist

Your fist represents the organ your wrist is the hilum and your arm contains the blood supply

entering the organ

Appreciate that there will never be organs in the peritoneal cavity - rather these organs invaginate

the cavity Kaplan videos

RULES OF NOMENCLATUREshy

1 Organ completely surrounded by peritoneum - peritoneal organ

2 Organ partially surrounded by peritoneum- Retroperitoneal

3 Peritoneum surrounding peritoneal organ is VISCERAL peritoneum

4 Peritoneum surrounding retroperitoneal organ is PARIETAL peritoneum

5 Peritoneum connecting visceral to parietal is called messentary 2 messentaries in the

gut Dorsal (to the gut tube) and ventral (to the gut tube) messentary

Aorta is in Retro peritoneal position - but blood must reach peritoneal position - vessels travel through

messentary All peritoneal organs will have blood supply reaching through messentary

-Mesentery is a 2 layer peritoneum with a neurovascular communication between body wall and organ

- Ligament connects one organ with another or to the abdominal wall (Ommentum = ligament)

lesser Ommentum (attach lesser curvature of stomach and duodenum to liver) =Hepatoduodenal

Ligament and Hepatogastric Ligament

Has a Superior and Inferior Recess (Accumulation of Fluid in Ascites)

Communicates with the greater sac through the epiplic foramen (what structures pass through

this foramen)

Boundaries - you must be able to visualize this

o Anterior - stomach

o Posterior - parietal peritoneum pancreas

o Superior - superior recess (bw diaphragm and coronary ligament)

o Inferior -Inferior recess (bw layers or greater momentum

Greater Ommentum (attach greater curvature of stomach) Gastrophrenic ligament Gastrosplenic

ligament gastrocolic ligament

The greater omentum is the largest peritoneal fold It consists of a double sheet of peritoneum folded on itself so that it is made up of four layers The two layers which descend from the greater curvature of the stomach and commencement of the duodenum pass in front of the small intestines sometimes as low down as the pelvis they then turn upon themselves and ascend again as far as the transverse colon where they separate and enclose that part of the intestine

ABDOMINAL PAIN

Parietal Peritoneum - supplied by same vasculature lymphatics and nerves supplying body wall it

lines and diaphragm Sensitive to pain pressure heat cold well localized

Visceral Peritoneum - supplied by same vasculature lymphatics and somatic nerve of organ it covers

Insensitive to touch heat cold and laceration - referred to dermatome of spinal ganglia providing

sensory fibers Where does appendicitis refer to

Foregut pain - epigastric area (ie - cholycystitis)

Midgut pain - periumbilical area (ie - appendicitis)

Hindgut Pain - suprapubic area (ie - diverticulitis)

Extra ImagesConcepts

ll~_____-

FalifCtrm ligament oind r~ud ligamet f Ilver

Blood from splenio gastriC and inferiof rne$e-rteri v~ins

Ca-I tributaries

Lett gastrio Ifein

Posterior superior pan~reatioodul)denal vaihS

Lott gamo-om~nlal (9aropip lomiddotic) -in

Poq_~ tjol imerl-9-r panCJertlcorllJod-nal veiopound --amp----I- - ~J Right grtr~-omntal

Anwrior interi (gartroepiploic) Jjn

pan euaii cod vl)denal veins middot Inf~Ji (t r mesentric vein

Miqdle (olic vein

Right cl)licvein Sigmoid and rectosigml)id (ei ns

IhH)Collc(~io

--- Mi~dl laquooLJl gtjrltgt

PoM ca vl1 illasto)moses -----shyampoptoageal 2 Paraumbilie-lt11 Inferi or Fectal vei ns

3 Recial 4 REuoperHonea1

Know how the Portal vein is formed I 4 sites of portal caval anastamoses and 1 clinical shunt

Col li t ltt-~ otTl~tI ~nj pc~ 1lt1 turJoG

Ltf 14i1 tImiddot~ artoftl9 on tj phtAt$

L-oftqf 4t t~r 1=laquoIran d 1 bull shy~p l ci rj o fOOOts

Nerves follow the arteries - appreciate the splanchnic nervous system I

Uet~ric branch of left ~nal art

Ureterie branch of righi renal artelY

Left Zld lumbar in and co mlTlunication to as)erdin9 lumbar l(~in Hi ~ht tEZ1~~t~ t3r j t itn ~ nJ l1t- rlnd lfe i r1

Inferior me5nteri~ artery

Notice that the right testicular vein drains directly into the IVC and the right testicular artery drains

directly into the aorta However the left testicular vein drains into the L renal vein at a right angleshy

reason left testicle is lower and more susceptible to varicocele (bag of worms)

Also notice that the left renal vein has a longer course because the IVC is on the right side whereas

the right renal artery has a longer course because the aorta is on the left side

Appreciate the anterior to posterior relationship of structures in the hilum of the kidney - VAP - Vein

Artery Renal Pelvis (Ureter)

11____ __ L_ L_ n VJ __ _ _ t_L I I_ _ L __ L_ I -pound1 bull LI_~-I ____

Posterior View of Head of Pancreas in ( of Duodenum

Celiao hunk

Co mmon ~L~jJth art~ry

GastNduQdonal artrf (partilly in phantn)

P1)Sterior $Up~Jior panCflaticuduodfmal art~r~t

(Co mm on) bile duct

middot~1t~~t-1l---~-~- Right gshomiddotomental (gastoe plp lolc) 3rte (phantomost)

Grener paocre atic art-ry

1n1~rjor pancr-iatlc artery

Jtrifll supejo r pal)oreailcento)dJodenal artr1 (phantom)

Anastomotlo branch

POostetlor bJanch of jo f~ri of pan-reatir(lduodensl drttnj

Anterio r branch of i flferior palcreati~)duodenal art~(phan1om)

Notice the extensive blood supply to the pancreas and duodenum via the branches of the celiac trunk

Notice collateral supply from SMA branches - makes sense bc this is the jxn of foregutmidgut

Identify the vessels in this arteriogram

Hiltid i)f N~ck oi B)dvof Tail 01 pa nereas pan cent~as P-nmiddot-reas panCtCas

I nferie v~na cava

jHept1iic p(lrlai v~in

Port1 tnd H~pti lt a ftH prol

Comm on) bll duct

Ouodtnum

~ft colic (sio)Atta~ hmtrlt jt~xJr-ofha~elSe

muo(IIQn

Right ~lIc (h~j)tic)

il~gtture

In1triol m~oten lIein (rttr op~ritoMdO

SlJp efl or mes~n~fiC amrV and lipln

KNOW YOUR NEIGHBORHOOD

Questions

vVhiJh structure supplied by a bnmdlof the cclia( artery is not derivcd from foregut LemCJUCrITI

(A) Head of the pancte-a5

CD) Pyloric duolenum

Cystkduct

( Liver hepatocyt~~

~F) Body of the spleen

An infant presents with an omrhaJucele at birth -hi oJ the [oHm illg applies to his cM1-dition

(A) It is 31so seen ill p4titnts with aganghonic megacolon

(11) ft reuirs from a fal1ure of resorption of theviteUine d let

(C) It results from herniation at the-site of regression of the right umbilk vein

DJ It is caustd by faihtrc of recanalization of the midgut part of the duodenum

~ It ill camioo by a failuIt vf the midgul to return to the abGQminal uity after herniashytion in-n the urnbilk s l stalk

Ot er than the spleen occlusion Cif the spit-Ilk artery at its odgin wm most likely affect die blood supply to jllch st cnud

(A) Jejunum

(B) Body of th pal1~lltas

(C) LeSStT Cllmiddotlaturc of tl )toma-ch

(D Duodenum dista to the entrance of the Ornmou bile duct

E Fundus of the stomach

A 38-yeu-old batL~er with a history of heartburn suddenly experiences excluciating pain in the (plgastric region of th~ abdomeu SurgCry is perf~rme immediard y upon admisshysion to the 1IlcrgCJliy tuomh~re i~ evidence uf a ruptured ulcer in the posterior waU of the stomach Vhere will a surgeon first fi nd the stomach contenlSf

A) Greater p4ritoneal sac

rB) Cul~de-s~c of Douglas (--

C Omental bursa ~

--D) Paracolic gutter

rEj Between -he panttal perimltum and the posterior body wal1

At birth an infant presents with a st()ma~ rb~tbas~njJled jfltotb~diaplfagru 1A1ltre is the defect thatresulied iiitJle heini~t()n shy~tsophagealbiatus

7 - rH-- Hiatus for the inferior vena cava

( Pleuroperitoneal membrane -(0) Septum transvcrsum

(E) Right Crlt~

An infant born with DOVv7l syndrome presents with bili()u~ vomiting Ahat congenital defect does the infant have

(A) Pyloric stenosis

(B) Meckel diverticulum C) Ornphaloce1e

(D) Gastroschisis

( ~ ) Duodenal atresia y A patient with cirrhosis of the liver presents with ~ bacalvaricestnlreased retrograde pressure in which veins caused the varices

(A) Paraumuilical

(B) Splenic

(ct AzygltJus

(15))G~trk ( (-F) Superior mesemeric

A htaltby 3-year~old male patient experiences a hernial sa protruding from the anterior abdominal wall about halfway between me anterior superior ilia spine and the pubk tuberde Pulsations of al1 artery are palpated medial to the protrusion site through the abdominal walL Which layer of the anterior abdominal wall will first be traversed by the

1hctma

fA) Rectus sheath (B) External oblique aponeurosis

(C) Inguinal ligament

lD) Transversalis fusda

(E) Cremasteric fa~cia

After 5urgi(aj ffpair of a hernia the patient tXperienccs mtmlgtness in the skin on the anteshyrior aspect of the S(Totum_ Vhaf nerve may have been lesioned during thehemiorrhaphy

(A) Femoral

(B) Obturator

(C) Ilioinguinal

(D) lliohypogastrk

(E) Pudendal

A 23~year-LJld female secretary il1 good health ~-uddcn1) doubles over with pain in the a ea of the 1JmbRicu$ Sbe feels vartn and ltneasy and has no appetite That night the pain seems to have mQved to the tower right abdominal regjol1 and she calls her family doctor who then arranges for an ambulance to pk-k her up and take her to the hospitaL Wh ell ntn~ perceived in the area of the urnbilirus most Hkely carried lhe pairfu I sensations into the eNS

tA) Vagus nerves I~

V B)

) Lessersplanchnk nerves

tC) Pudendal nerves

(D) lIiohpogastrk nerves

(E) Greater splam ic l erves

A CT reveals carcinoma in the bOod of the ancreas Vhich blood vessel trut ourses ----~- - -bull ------ --shy

immediately poftterior to the body ofthe pancreas is the m~t likely to be oompressed

(A) Splenk artery

(B) Abdominal aorta (C) Portal vein

(1) Splenic vein

(E) Renal vein

A patient has a penrln1l1ng uker of the posterior wall ot the br~l part ot the (lUooenmn llkh blood vessel is subject to erosion

(A) Common hepatic artery

(B) Gastroouodenal artery

(C) Proper hevatic artery

(D) Celiac artery

(E) Anterior inferior 11amrelltlcoduodcnal attery

Your patient has been diagnosed -ith a carcinoma locallted to the head and l~e(k of the pancreas Another clinical sign would be

A esophageal varices

(8) hemorrhoids

C) a caput medusa

(D) increased pra Teuro n th~ hepatic veins

(E) enlarged right supra lavkular lymph nodes

Wltkh of the foUowing structures develops in the ventral mesentery

(A) Spleen

(B) Jeiunum (C) Head of1ht pancreas (D) Transverse colon (E) Stomach

ti l Uw ~ littwin~ f( S-t lil oai Imdge ~ hi(h or tbt la~)d J truetur tgt liJ llntn nl) he hl p UC iJd [IIi ell

c o

A) drains Ie tht infCrior a La aI

R t middot~nfl0 ~ill to th~ lunlgtn of h i dtlndCrlllfH

(e) m t bull JiJattd on tl l J n T ~H

D ) sup Lc O VSlt I Hlid bhtu l 1 li - -I un oid

( ) U~tpli(t tr j middottUh~ 1 v(( b~nt rfK n1ilc~Zm

ANSWERS AND EXPLANATIONS

Answer E The spleen is t hlttnopodicand lymph organ demlted from mesoderm

Answ~ R Al1 tlmphalocele is caused by it failure of the nlidgut to return to the ahdomir nat cavity after herniation into the umbiliau Stalk Choices Aand D maybe seen in infants with Down syndrome choice D ~s the specific CBuse ofduudcnal JtiCSitt Choice C is (ile cause of gclstrosbisis and Choice B nsults iu a Meurolktldivertku1-tlB

Answer B The fundus ofthe stomach is suppHed by soort gastric brunches of the splenic altery The splenic artery supplies the body and tail of the pancreas part of the greater curvature of the sttmla(h and the spleen Te jejunum part of the head of the pancreas and tht~ duodenum distal to the entrance of the commOll bile duct are supplied by the superior mesenterk artery clll~l ~be less r ctlt1ature cmd the pylQric antrum are supplied by the right and lei gastric art(ries

AnSWftt C Tbeomental bursa or lesser ~ritoneaj sac lies direcdy posterior to the proxshyimal part of the duodeTtlm and the stomach and would be the first site where stomach contents ~Ott1d be fpoundluncL

Answer C A defect in a llleuropcritoneal membrane (uswlly the left) is the typical site of i1 cc-ngenitlI diilphragluatic hemia llere the membr4ne fails to dose ()pound( of the perishycCirdiopcritulleal canals

Answer E DuoJenal atresia and aganglionic megacoion are congwitaI defects S~Il in patients with Dowmiddotnsyndrome

Answer D RulaTgemt~llt of and retrograde flow in g~lstrk vel_ns in particlJl~r the kft gas~ tricveins dilates the capillary bed in rhe wall of the esophagus in (ases of porta yper~

tension Blood flow would increase in and dilampte tribntarkgts of the (lZygOUS vein on the other side of the capiUary bed but flow in this vein is in the typical direction t()ward the superior vena cava Paraumbiii(ltU vein eilgorgement contributes to a caput medusH Splenic ~nlargement might prc~nt with 5plcnonlegaly and balt-kflow in to tlu superior m~~ntclic vein occurs but is asymptomatic

Answer D The patient hagt an indirect inguinal hernia whi~h emerges from the antt-rior abdominal wall through the deep inguinltilling Theeep ring is a fault in the transv~rshysaUs fascia this I~yer wiIJ be penetrated first by the hernia

An~Wer C The ilioinguinal nenc which provides sens~llion to the lnedlal thigh ltmclanteshytior SClotunl pass~lt th rough the 5uperfh_ial inguinal ring ind $subject to inj i1T) becaus-e

it is in the operatitm Held of the erniorrhapny

Auswer B The leMHr splanchnic nerves are sympathdic nerVlts that carry viscera l sensashytlltgtrogt ftom illtllt1m~d ()J stietched gust (itinteitinal ~tructures (in this case the pprndix) into tnt eNS Lesser splanchnic ntTYcsarisc from thmiddot T9--T12 spinal cord segments lt1nd provide sympathetic innenation tD rnidgut siruc1ures whiCh include CLe app~JldD Viscera] Pain arising from affecLed Inidgut ampt 1C1ure is referred over the same dl- matorne~ of spinal segrnertts v-hich provide the sympathetic Innervation n this G1SC of appendicitis the invohen~n t of the ltire) of t e unlhHku indud s the T 10 dermatome

Answer B Of the five choices onty the dscending olon is retroperiton~al aldwould be a lik ~ ( choice to be seen immediately a(~jilcent to t11e posterior abdominal middotn~L

Amwen D The SpltftlC ~-ein ourses posterior to the body of the panneas m its way tt drain into the superior mCSfttltlri( vein

Answcr B TILt glstrodllolticnal artery 1 direct hIamh of the comrootl hepatic artery courses immediately pt))iwri() to the duodenum and is slbject to erosion

Answer B Carcinoma of th pan middott3S in the 1tilt1 may compreampgt the portltil vein at irs orishygill The poTtai vcin is fomled when the splenic vein jQiaswith tfie superior meStllt eric vein The inferiot mesenteric vein joins the ~plenjc vein just priOT to tlli~ point at which the splenic joins the superior Jlleit1ltcri( vein Increescd venous presslu in the inferior mesenteric vein is a cause of emo hoid~

Answer C The- velltral pancreas wilich forms most of the head of the p ~ncr as develops in the ventral mes(ntery as antutgrowth of the hepatic diverticulum Th~ hepatic divershyticulull induding the biIJary appa~atus develops in tbe ventral mesentery of the foregut

Answer~ A The superior mesenteric ~in joins with the spienkvein to form the hepatic portal vciu

Answer D The structure at gttlK is the proper hepatic artery~ whkh suppUesoxygenated b middotood to the liver

MAKE SURE YOU KNOW the diff bw Rectus Sheath above and below the arcuate line

ABOVE

Aponeurosis of xiiltmal obllque musclo

Extemll f)biquw musde

Reotln ilbdomlnls musole S~in

Internal 9bliquQ mY~QI

AponeUfOsi$ of hJH$V~~S Lir9a a lb lbdolTlin~ musolo Tri OJV6 rUi

atldomlnis mUS(loe

Sub cutanlilous tiue (tatty ye r)

BElOW

A POrl lJfosis 01 etemal oblique muscle

Aponeul~)sis 01 Internal oblique mU$cl~

Anteriol lay~ of r~ltdus st~ath EXttom1 oblique rnu$cll

Rectus Jbdominis muscle Intoernal Aponeurc-sis of tra~fersU$ oblique muscle-

at-domlnis muscentl ~ Skio

Tra nsvitSus abdomioLs ml)ZClt

TralSVersaHs fascia Medial umQil iegtt1 1i9Jment -and folj

Uldchus Peritoneum (ir median Umbilj~al Suboutane ous

Extraprftone 11ascia

Ymbilimiddot~1 fold)

preu9poundiea1 fascia

tissue (fatty 4nd m~mbr3n(iUS layers)

o Above the arcuate line (A horizontal line 13 of the distance bw the umbilicus and the

pubic symphysis) -10 Aponeurosis divides into an AntPost Laminae

o The Ant Laminae joins EO and Post Laminae joins Trans Abdominis = Ant and Post

RECTUS SHEATH respectively

o BElOW the arcuate line - all 3 aponeurosis join ANTERIOR to rectus muscle to meet its

counterpart in the midline (linea Alba)

o Take away Msg - The abdomen is devoid of a posterior rectus sheath below the

arcuate line and is therefore more vulnerable to herniasinjuries

Question - A physician makes a deep incision in the patients midline immediately superior to

the pubic symphysis which of the following layers is his knife least likely to pass

Rectus Abdominis External Oblique Ant Rectus Sheath Posterior Rectus Sheath All of the

Above

Answer - All of the above None of the other answer choices are midline structures -LINEA

ALBA

Linea Alba has very poor blood supply - doesnt heal well after surgery Therefore this is a

common site for incisional hernias

a Spleen b Transverse colon c Descending colon d Stomach e Pleura

17 Meckels diverticulum is normally found 2 feet proximal from the

a Pyloric sphincter b Lower esophageal sphincter c Ileo-cecal valve d Middle valve of Huston e Anal valve

18 Ulcer in the posterior wall of the first part of the duodenum would erode ___ artery and would cause bleeding

a Left gastric b Right gastric c Hepatic artery proper d Gastroduodenal artery e Middle colic artery

19 An inflamed appendix is identified by a surgeon on the operation table by noting

a The appendicies epiploicae b The convergence of tenia c The artery of Drummond d The mesocolon e The mesosalphinx

20 The nerve which emerges through the psoas major is

a Femoral b Ilio-inguinal c Ilio-hypogastric d Pudendal e Subcostal

21 The right gonadal vein drains into the

a Azygos b Hemiazygos c Inferior Vena Cava d Right renal vein e Left renal vein

22 The hepatocytes in the liver is derived from

a Ectoderm b Endoderm c Mesoderm

d Neural ectoderm

23 Abscess in the lumbar vertebrae due to tuberculosis would spread to the adjacent muscle which is

a Psoas Major b Iliacus c Quadratus lumborum d Tranversus Abdominis

24 The anterior wall of the inguinal canal is formed by

a External oblique and transverses abdominis b External oblique and fascia transversalis c Internal oblique and external oblique d Internal oblique and transverses abdominis e Fascia transversalis and peritoneum

Meckels diverticulum is a result of which of the following developmental abnormalities shy

A Failure of the vitelline duct to close

B Failure of the herniated intestinal loop to retract into the abdomen

C Failure of the urachus to close

D Failure of the midgut to rotate

E Failure of the hepatic duct to close

Explanation

Meckels diverticulum is a result of the persistence of the proximal part of the vitelline duct This

diverticulum is usually found about 2 feet proximal to the ileocecal junction and is usually about 2 inches

long It is present in about 2 of the popUlation It may be the site of ectopic pancreatic tissue or gastric

mucosa and may develop inflammatory processes and ulcerations Acute Meckels diverticulitis

simulates appendicitis

Which of the following veins carries blood from the esophagus to the portal vein The

A right gastric vein

B left gastric vein c splenic vein D azygos vein

E left gastroepiploic vein

Explanation

The left gastric vein a direct branch of the portal vein drains blood from the lesser curvature of the

stomach and the inferior portion of the esophagus Because branches of the portal vein do not have

valves blood can flow in a retrograde path when there is an obstruction to flow through the portal system or liveL Rlooci Cln then flow from the nortl] vein thr()1Ph the left PRstric vein to the esonhlPlIS lno

through venous communications within the submucosa of the esophagus to esophageal veins that drain

into the azygos vein The increase in blood flow through the esophageal submucosal veins results in esophageal varices

On the posterior wall of the abdomen the celiac ganglion A contains cell bodies of postganglionic parasympathetic neurons B is synapsed upon by neurons in the posterior vagal trunk C is synapsed upon by neurons in the greater splanchnic nerve D contains sensory cell bodies of lumbar spinal nerves E contains cell bodies of neurons that cause an increase in the rate of peristasis

Explanation The celiac ganglion is one of the preaortic ganglia of the sympathetic nervous system It contains cell bodies of postganglionic sympathetic neurons The sympathetic splanchnic nerves contain preganglionic sympathetic neurons that pass through the sympathetic chain without synapsing These splanchnic nerves go to the preaortic ganglia to synapse The greater splanchnic nerve contains preganglionic neurons from spinal cord segments T5-T9 This nerve synapses in the celiac ganglion The nerve fibers in the vagal trunks are preganglionic parasympathetic fibers that go to the walls of the organs that they will innervate and synapse on postganglionic parasympathetic neurons in the walls of those organs Cell bodies of sensory neurons in the abdomen are found in the dorsal root ganglia or the sensory ganglia of the vagus nerve Sympathetic innervation decreases the rate of peristalsis parasympathetic innervation increases the rate of peristalsis

Which of the following pairs of arteries will allow blood to bypass an occlusion of the celiac trunk

A Left gastric artery-right gastric artery

B Left gastroepiploic artery-right gastroepiploic artery

C Superior pancreaticoduodenal artery-inferior pancreaticoduodenal artery

D Splenic artery-common hepatic artery

E Left gastric artery - proper hepatic artery

Explanation The anastoOlosis of a branch of the celiac trunk and a branch of the superior mesenteric artery will

provide collateral circulation around an occlusion of the celiac trunk Each of the other choices pair

branches of the celiac trunk therefore these will not provide collateral flow around the obstruction of the

celiac trunk The left gastric splenic and common hepatic arteries are direct branches of the celiac trunk

The right gastric artery is a branch of the proper hepatic artery which is a branch of the common hepatic artery The left gastroepiploic artery is a branch of the splenic artery The right gastroepiploic artery is a

branch of the gastroduodenal artery whlch is a branch of the common hepatic artery

Which of the following organs has appendices epiploica The

A sigmoid colon

Bjejunum

C duodenum

D stomach E esophagus

Explanation Appendices epiploica are characteristic of the colon Appendices epiploica are subserosal accumulations

of fat None of the organs of the gastrointestinal tract has appendices epiploica except the colon

Page 32: Chirag's Abdomen Review

--~--- -~ -~-~

i

I AolaijonjoI~guf I

STOMACH BED (IDENTIFY IN ADAMS)- the structures posterior to the ommental bursa which

support the stomach in the supine position

Abdomnal JQrUI

Splnic vein

OmQ-oul tv~ ) O(s(Jroa)

Lojt(r o m nturrt (hpJtodu o d~n31 Hid

Gadrl)SplerH (g3stroll~nal) IIgam~nt

hiad h~~atogrtricent IIQdmiddotcrt~)

Lt Dome of Diaphragm (why left Look this up in Adams)

Spleen (What is the blood supply)

Left Kidney (What is the blood supply - AND how is it different from the R kidney)

Suprarenal Gland (What is the Arterial AND Venous Blood supply - how are they different)

Pancreas (How does supply differ from Head to Tail What is the SMA Relationship)

Transverse Mesocolon

liver - ADAMSWET - Make sure you look at the liver in wet lab

Left triangular nl1am~nt

ComoaDj ligamnt

Erophg~1 impre$ioo

Hepatio veins

In1erior -ifena middotr3)Ia

Fibrous appendix o-t

live

impr~j on

Heprorendl p~rtion of Q)(Qllary ligament

Righllri~n9ul r 1I~met

(Common) bile quol

Gr)mmCtr~ hepatic dlJct

Ccentic duct

Duodenal impression

GaJdate p-fr)~S

Hepatic artgtrl prop-f iiiiila - Faloiform ligament

_ - shy Round ligamen liver

~--F-- CoJio imprgt-ssi-on

Prta heptis

Identify the lobes impressions and embryonic remnants associated with the liver

Caudate Lobe Quadrate Lobe Right Lobe Left Lobe Round ligament Falciform Ligament

Ligamentum Venosum (what is its fxn in embryonic life) Hepatic Veins (NOT PART OF THE

PORTAL TRIAD) IVC PORTAL TRIAD - Contents relationship cross section etc Know the

Galbladder relationship to the lobes of the liver

Biliary Duct System - Make sure you understand the sequence of these structures - BE ABLE TO

DRAW A FLOW CHART

TPVd i

t

I t

1 __ Cm-(r

patk GlJet

I

J

Clinical = JAUNDICE is caused by anything that prevents delivery of bile to intestine Tumor of the

head of the pancreas Stones etc Patient will have pale stools and yellowish colored mucus

membranes

Clinical- Any scenario that tells you the patient has BILLOUS VOMIT means that the obstruction to

the flow of digestive contents is after the Ampulla of Vater (Site of Entry of Billiary system to the

duodenum) - ie Duodenal Atresia

Spleen -located posterior to the mid axillary line between ribs 9 and 11 Make sure you know that

the 10th rib is the main axis of the spleen and this organ is susceptible to injury (stab wound errant

thoracoce ntesis etc)

The spleen is derived from mesodermal cells - NOT THE GUT TUBE

The spleen rests on the left colic flexure associates with the tail of the pancreas Know the

structures entering the Hilum of the spleen

Sh rt O~-t~ic 1 0(0 10 rtiltSPIric Iloa nt

(cut)

Peritoneum - similar concept to Pleura - think of a fist in a balloon

Visceral Peritoneum - Layer of balloon touching your fist

Parietal Peritoneum - Layer of balloon not touching your fist

Your fist represents the organ your wrist is the hilum and your arm contains the blood supply

entering the organ

Appreciate that there will never be organs in the peritoneal cavity - rather these organs invaginate

the cavity Kaplan videos

RULES OF NOMENCLATUREshy

1 Organ completely surrounded by peritoneum - peritoneal organ

2 Organ partially surrounded by peritoneum- Retroperitoneal

3 Peritoneum surrounding peritoneal organ is VISCERAL peritoneum

4 Peritoneum surrounding retroperitoneal organ is PARIETAL peritoneum

5 Peritoneum connecting visceral to parietal is called messentary 2 messentaries in the

gut Dorsal (to the gut tube) and ventral (to the gut tube) messentary

Aorta is in Retro peritoneal position - but blood must reach peritoneal position - vessels travel through

messentary All peritoneal organs will have blood supply reaching through messentary

-Mesentery is a 2 layer peritoneum with a neurovascular communication between body wall and organ

- Ligament connects one organ with another or to the abdominal wall (Ommentum = ligament)

lesser Ommentum (attach lesser curvature of stomach and duodenum to liver) =Hepatoduodenal

Ligament and Hepatogastric Ligament

Has a Superior and Inferior Recess (Accumulation of Fluid in Ascites)

Communicates with the greater sac through the epiplic foramen (what structures pass through

this foramen)

Boundaries - you must be able to visualize this

o Anterior - stomach

o Posterior - parietal peritoneum pancreas

o Superior - superior recess (bw diaphragm and coronary ligament)

o Inferior -Inferior recess (bw layers or greater momentum

Greater Ommentum (attach greater curvature of stomach) Gastrophrenic ligament Gastrosplenic

ligament gastrocolic ligament

The greater omentum is the largest peritoneal fold It consists of a double sheet of peritoneum folded on itself so that it is made up of four layers The two layers which descend from the greater curvature of the stomach and commencement of the duodenum pass in front of the small intestines sometimes as low down as the pelvis they then turn upon themselves and ascend again as far as the transverse colon where they separate and enclose that part of the intestine

ABDOMINAL PAIN

Parietal Peritoneum - supplied by same vasculature lymphatics and nerves supplying body wall it

lines and diaphragm Sensitive to pain pressure heat cold well localized

Visceral Peritoneum - supplied by same vasculature lymphatics and somatic nerve of organ it covers

Insensitive to touch heat cold and laceration - referred to dermatome of spinal ganglia providing

sensory fibers Where does appendicitis refer to

Foregut pain - epigastric area (ie - cholycystitis)

Midgut pain - periumbilical area (ie - appendicitis)

Hindgut Pain - suprapubic area (ie - diverticulitis)

Extra ImagesConcepts

ll~_____-

FalifCtrm ligament oind r~ud ligamet f Ilver

Blood from splenio gastriC and inferiof rne$e-rteri v~ins

Ca-I tributaries

Lett gastrio Ifein

Posterior superior pan~reatioodul)denal vaihS

Lott gamo-om~nlal (9aropip lomiddotic) -in

Poq_~ tjol imerl-9-r panCJertlcorllJod-nal veiopound --amp----I- - ~J Right grtr~-omntal

Anwrior interi (gartroepiploic) Jjn

pan euaii cod vl)denal veins middot Inf~Ji (t r mesentric vein

Miqdle (olic vein

Right cl)licvein Sigmoid and rectosigml)id (ei ns

IhH)Collc(~io

--- Mi~dl laquooLJl gtjrltgt

PoM ca vl1 illasto)moses -----shyampoptoageal 2 Paraumbilie-lt11 Inferi or Fectal vei ns

3 Recial 4 REuoperHonea1

Know how the Portal vein is formed I 4 sites of portal caval anastamoses and 1 clinical shunt

Col li t ltt-~ otTl~tI ~nj pc~ 1lt1 turJoG

Ltf 14i1 tImiddot~ artoftl9 on tj phtAt$

L-oftqf 4t t~r 1=laquoIran d 1 bull shy~p l ci rj o fOOOts

Nerves follow the arteries - appreciate the splanchnic nervous system I

Uet~ric branch of left ~nal art

Ureterie branch of righi renal artelY

Left Zld lumbar in and co mlTlunication to as)erdin9 lumbar l(~in Hi ~ht tEZ1~~t~ t3r j t itn ~ nJ l1t- rlnd lfe i r1

Inferior me5nteri~ artery

Notice that the right testicular vein drains directly into the IVC and the right testicular artery drains

directly into the aorta However the left testicular vein drains into the L renal vein at a right angleshy

reason left testicle is lower and more susceptible to varicocele (bag of worms)

Also notice that the left renal vein has a longer course because the IVC is on the right side whereas

the right renal artery has a longer course because the aorta is on the left side

Appreciate the anterior to posterior relationship of structures in the hilum of the kidney - VAP - Vein

Artery Renal Pelvis (Ureter)

11____ __ L_ L_ n VJ __ _ _ t_L I I_ _ L __ L_ I -pound1 bull LI_~-I ____

Posterior View of Head of Pancreas in ( of Duodenum

Celiao hunk

Co mmon ~L~jJth art~ry

GastNduQdonal artrf (partilly in phantn)

P1)Sterior $Up~Jior panCflaticuduodfmal art~r~t

(Co mm on) bile duct

middot~1t~~t-1l---~-~- Right gshomiddotomental (gastoe plp lolc) 3rte (phantomost)

Grener paocre atic art-ry

1n1~rjor pancr-iatlc artery

Jtrifll supejo r pal)oreailcento)dJodenal artr1 (phantom)

Anastomotlo branch

POostetlor bJanch of jo f~ri of pan-reatir(lduodensl drttnj

Anterio r branch of i flferior palcreati~)duodenal art~(phan1om)

Notice the extensive blood supply to the pancreas and duodenum via the branches of the celiac trunk

Notice collateral supply from SMA branches - makes sense bc this is the jxn of foregutmidgut

Identify the vessels in this arteriogram

Hiltid i)f N~ck oi B)dvof Tail 01 pa nereas pan cent~as P-nmiddot-reas panCtCas

I nferie v~na cava

jHept1iic p(lrlai v~in

Port1 tnd H~pti lt a ftH prol

Comm on) bll duct

Ouodtnum

~ft colic (sio)Atta~ hmtrlt jt~xJr-ofha~elSe

muo(IIQn

Right ~lIc (h~j)tic)

il~gtture

In1triol m~oten lIein (rttr op~ritoMdO

SlJp efl or mes~n~fiC amrV and lipln

KNOW YOUR NEIGHBORHOOD

Questions

vVhiJh structure supplied by a bnmdlof the cclia( artery is not derivcd from foregut LemCJUCrITI

(A) Head of the pancte-a5

CD) Pyloric duolenum

Cystkduct

( Liver hepatocyt~~

~F) Body of the spleen

An infant presents with an omrhaJucele at birth -hi oJ the [oHm illg applies to his cM1-dition

(A) It is 31so seen ill p4titnts with aganghonic megacolon

(11) ft reuirs from a fal1ure of resorption of theviteUine d let

(C) It results from herniation at the-site of regression of the right umbilk vein

DJ It is caustd by faihtrc of recanalization of the midgut part of the duodenum

~ It ill camioo by a failuIt vf the midgul to return to the abGQminal uity after herniashytion in-n the urnbilk s l stalk

Ot er than the spleen occlusion Cif the spit-Ilk artery at its odgin wm most likely affect die blood supply to jllch st cnud

(A) Jejunum

(B) Body of th pal1~lltas

(C) LeSStT Cllmiddotlaturc of tl )toma-ch

(D Duodenum dista to the entrance of the Ornmou bile duct

E Fundus of the stomach

A 38-yeu-old batL~er with a history of heartburn suddenly experiences excluciating pain in the (plgastric region of th~ abdomeu SurgCry is perf~rme immediard y upon admisshysion to the 1IlcrgCJliy tuomh~re i~ evidence uf a ruptured ulcer in the posterior waU of the stomach Vhere will a surgeon first fi nd the stomach contenlSf

A) Greater p4ritoneal sac

rB) Cul~de-s~c of Douglas (--

C Omental bursa ~

--D) Paracolic gutter

rEj Between -he panttal perimltum and the posterior body wal1

At birth an infant presents with a st()ma~ rb~tbas~njJled jfltotb~diaplfagru 1A1ltre is the defect thatresulied iiitJle heini~t()n shy~tsophagealbiatus

7 - rH-- Hiatus for the inferior vena cava

( Pleuroperitoneal membrane -(0) Septum transvcrsum

(E) Right Crlt~

An infant born with DOVv7l syndrome presents with bili()u~ vomiting Ahat congenital defect does the infant have

(A) Pyloric stenosis

(B) Meckel diverticulum C) Ornphaloce1e

(D) Gastroschisis

( ~ ) Duodenal atresia y A patient with cirrhosis of the liver presents with ~ bacalvaricestnlreased retrograde pressure in which veins caused the varices

(A) Paraumuilical

(B) Splenic

(ct AzygltJus

(15))G~trk ( (-F) Superior mesemeric

A htaltby 3-year~old male patient experiences a hernial sa protruding from the anterior abdominal wall about halfway between me anterior superior ilia spine and the pubk tuberde Pulsations of al1 artery are palpated medial to the protrusion site through the abdominal walL Which layer of the anterior abdominal wall will first be traversed by the

1hctma

fA) Rectus sheath (B) External oblique aponeurosis

(C) Inguinal ligament

lD) Transversalis fusda

(E) Cremasteric fa~cia

After 5urgi(aj ffpair of a hernia the patient tXperienccs mtmlgtness in the skin on the anteshyrior aspect of the S(Totum_ Vhaf nerve may have been lesioned during thehemiorrhaphy

(A) Femoral

(B) Obturator

(C) Ilioinguinal

(D) lliohypogastrk

(E) Pudendal

A 23~year-LJld female secretary il1 good health ~-uddcn1) doubles over with pain in the a ea of the 1JmbRicu$ Sbe feels vartn and ltneasy and has no appetite That night the pain seems to have mQved to the tower right abdominal regjol1 and she calls her family doctor who then arranges for an ambulance to pk-k her up and take her to the hospitaL Wh ell ntn~ perceived in the area of the urnbilirus most Hkely carried lhe pairfu I sensations into the eNS

tA) Vagus nerves I~

V B)

) Lessersplanchnk nerves

tC) Pudendal nerves

(D) lIiohpogastrk nerves

(E) Greater splam ic l erves

A CT reveals carcinoma in the bOod of the ancreas Vhich blood vessel trut ourses ----~- - -bull ------ --shy

immediately poftterior to the body ofthe pancreas is the m~t likely to be oompressed

(A) Splenk artery

(B) Abdominal aorta (C) Portal vein

(1) Splenic vein

(E) Renal vein

A patient has a penrln1l1ng uker of the posterior wall ot the br~l part ot the (lUooenmn llkh blood vessel is subject to erosion

(A) Common hepatic artery

(B) Gastroouodenal artery

(C) Proper hevatic artery

(D) Celiac artery

(E) Anterior inferior 11amrelltlcoduodcnal attery

Your patient has been diagnosed -ith a carcinoma locallted to the head and l~e(k of the pancreas Another clinical sign would be

A esophageal varices

(8) hemorrhoids

C) a caput medusa

(D) increased pra Teuro n th~ hepatic veins

(E) enlarged right supra lavkular lymph nodes

Wltkh of the foUowing structures develops in the ventral mesentery

(A) Spleen

(B) Jeiunum (C) Head of1ht pancreas (D) Transverse colon (E) Stomach

ti l Uw ~ littwin~ f( S-t lil oai Imdge ~ hi(h or tbt la~)d J truetur tgt liJ llntn nl) he hl p UC iJd [IIi ell

c o

A) drains Ie tht infCrior a La aI

R t middot~nfl0 ~ill to th~ lunlgtn of h i dtlndCrlllfH

(e) m t bull JiJattd on tl l J n T ~H

D ) sup Lc O VSlt I Hlid bhtu l 1 li - -I un oid

( ) U~tpli(t tr j middottUh~ 1 v(( b~nt rfK n1ilc~Zm

ANSWERS AND EXPLANATIONS

Answer E The spleen is t hlttnopodicand lymph organ demlted from mesoderm

Answ~ R Al1 tlmphalocele is caused by it failure of the nlidgut to return to the ahdomir nat cavity after herniation into the umbiliau Stalk Choices Aand D maybe seen in infants with Down syndrome choice D ~s the specific CBuse ofduudcnal JtiCSitt Choice C is (ile cause of gclstrosbisis and Choice B nsults iu a Meurolktldivertku1-tlB

Answer B The fundus ofthe stomach is suppHed by soort gastric brunches of the splenic altery The splenic artery supplies the body and tail of the pancreas part of the greater curvature of the sttmla(h and the spleen Te jejunum part of the head of the pancreas and tht~ duodenum distal to the entrance of the commOll bile duct are supplied by the superior mesenterk artery clll~l ~be less r ctlt1ature cmd the pylQric antrum are supplied by the right and lei gastric art(ries

AnSWftt C Tbeomental bursa or lesser ~ritoneaj sac lies direcdy posterior to the proxshyimal part of the duodeTtlm and the stomach and would be the first site where stomach contents ~Ott1d be fpoundluncL

Answer C A defect in a llleuropcritoneal membrane (uswlly the left) is the typical site of i1 cc-ngenitlI diilphragluatic hemia llere the membr4ne fails to dose ()pound( of the perishycCirdiopcritulleal canals

Answer E DuoJenal atresia and aganglionic megacoion are congwitaI defects S~Il in patients with Dowmiddotnsyndrome

Answer D RulaTgemt~llt of and retrograde flow in g~lstrk vel_ns in particlJl~r the kft gas~ tricveins dilates the capillary bed in rhe wall of the esophagus in (ases of porta yper~

tension Blood flow would increase in and dilampte tribntarkgts of the (lZygOUS vein on the other side of the capiUary bed but flow in this vein is in the typical direction t()ward the superior vena cava Paraumbiii(ltU vein eilgorgement contributes to a caput medusH Splenic ~nlargement might prc~nt with 5plcnonlegaly and balt-kflow in to tlu superior m~~ntclic vein occurs but is asymptomatic

Answer D The patient hagt an indirect inguinal hernia whi~h emerges from the antt-rior abdominal wall through the deep inguinltilling Theeep ring is a fault in the transv~rshysaUs fascia this I~yer wiIJ be penetrated first by the hernia

An~Wer C The ilioinguinal nenc which provides sens~llion to the lnedlal thigh ltmclanteshytior SClotunl pass~lt th rough the 5uperfh_ial inguinal ring ind $subject to inj i1T) becaus-e

it is in the operatitm Held of the erniorrhapny

Auswer B The leMHr splanchnic nerves are sympathdic nerVlts that carry viscera l sensashytlltgtrogt ftom illtllt1m~d ()J stietched gust (itinteitinal ~tructures (in this case the pprndix) into tnt eNS Lesser splanchnic ntTYcsarisc from thmiddot T9--T12 spinal cord segments lt1nd provide sympathetic innenation tD rnidgut siruc1ures whiCh include CLe app~JldD Viscera] Pain arising from affecLed Inidgut ampt 1C1ure is referred over the same dl- matorne~ of spinal segrnertts v-hich provide the sympathetic Innervation n this G1SC of appendicitis the invohen~n t of the ltire) of t e unlhHku indud s the T 10 dermatome

Answer B Of the five choices onty the dscending olon is retroperiton~al aldwould be a lik ~ ( choice to be seen immediately a(~jilcent to t11e posterior abdominal middotn~L

Amwen D The SpltftlC ~-ein ourses posterior to the body of the panneas m its way tt drain into the superior mCSfttltlri( vein

Answcr B TILt glstrodllolticnal artery 1 direct hIamh of the comrootl hepatic artery courses immediately pt))iwri() to the duodenum and is slbject to erosion

Answer B Carcinoma of th pan middott3S in the 1tilt1 may compreampgt the portltil vein at irs orishygill The poTtai vcin is fomled when the splenic vein jQiaswith tfie superior meStllt eric vein The inferiot mesenteric vein joins the ~plenjc vein just priOT to tlli~ point at which the splenic joins the superior Jlleit1ltcri( vein Increescd venous presslu in the inferior mesenteric vein is a cause of emo hoid~

Answer C The- velltral pancreas wilich forms most of the head of the p ~ncr as develops in the ventral mes(ntery as antutgrowth of the hepatic diverticulum Th~ hepatic divershyticulull induding the biIJary appa~atus develops in tbe ventral mesentery of the foregut

Answer~ A The superior mesenteric ~in joins with the spienkvein to form the hepatic portal vciu

Answer D The structure at gttlK is the proper hepatic artery~ whkh suppUesoxygenated b middotood to the liver

MAKE SURE YOU KNOW the diff bw Rectus Sheath above and below the arcuate line

ABOVE

Aponeurosis of xiiltmal obllque musclo

Extemll f)biquw musde

Reotln ilbdomlnls musole S~in

Internal 9bliquQ mY~QI

AponeUfOsi$ of hJH$V~~S Lir9a a lb lbdolTlin~ musolo Tri OJV6 rUi

atldomlnis mUS(loe

Sub cutanlilous tiue (tatty ye r)

BElOW

A POrl lJfosis 01 etemal oblique muscle

Aponeul~)sis 01 Internal oblique mU$cl~

Anteriol lay~ of r~ltdus st~ath EXttom1 oblique rnu$cll

Rectus Jbdominis muscle Intoernal Aponeurc-sis of tra~fersU$ oblique muscle-

at-domlnis muscentl ~ Skio

Tra nsvitSus abdomioLs ml)ZClt

TralSVersaHs fascia Medial umQil iegtt1 1i9Jment -and folj

Uldchus Peritoneum (ir median Umbilj~al Suboutane ous

Extraprftone 11ascia

Ymbilimiddot~1 fold)

preu9poundiea1 fascia

tissue (fatty 4nd m~mbr3n(iUS layers)

o Above the arcuate line (A horizontal line 13 of the distance bw the umbilicus and the

pubic symphysis) -10 Aponeurosis divides into an AntPost Laminae

o The Ant Laminae joins EO and Post Laminae joins Trans Abdominis = Ant and Post

RECTUS SHEATH respectively

o BElOW the arcuate line - all 3 aponeurosis join ANTERIOR to rectus muscle to meet its

counterpart in the midline (linea Alba)

o Take away Msg - The abdomen is devoid of a posterior rectus sheath below the

arcuate line and is therefore more vulnerable to herniasinjuries

Question - A physician makes a deep incision in the patients midline immediately superior to

the pubic symphysis which of the following layers is his knife least likely to pass

Rectus Abdominis External Oblique Ant Rectus Sheath Posterior Rectus Sheath All of the

Above

Answer - All of the above None of the other answer choices are midline structures -LINEA

ALBA

Linea Alba has very poor blood supply - doesnt heal well after surgery Therefore this is a

common site for incisional hernias

a Spleen b Transverse colon c Descending colon d Stomach e Pleura

17 Meckels diverticulum is normally found 2 feet proximal from the

a Pyloric sphincter b Lower esophageal sphincter c Ileo-cecal valve d Middle valve of Huston e Anal valve

18 Ulcer in the posterior wall of the first part of the duodenum would erode ___ artery and would cause bleeding

a Left gastric b Right gastric c Hepatic artery proper d Gastroduodenal artery e Middle colic artery

19 An inflamed appendix is identified by a surgeon on the operation table by noting

a The appendicies epiploicae b The convergence of tenia c The artery of Drummond d The mesocolon e The mesosalphinx

20 The nerve which emerges through the psoas major is

a Femoral b Ilio-inguinal c Ilio-hypogastric d Pudendal e Subcostal

21 The right gonadal vein drains into the

a Azygos b Hemiazygos c Inferior Vena Cava d Right renal vein e Left renal vein

22 The hepatocytes in the liver is derived from

a Ectoderm b Endoderm c Mesoderm

d Neural ectoderm

23 Abscess in the lumbar vertebrae due to tuberculosis would spread to the adjacent muscle which is

a Psoas Major b Iliacus c Quadratus lumborum d Tranversus Abdominis

24 The anterior wall of the inguinal canal is formed by

a External oblique and transverses abdominis b External oblique and fascia transversalis c Internal oblique and external oblique d Internal oblique and transverses abdominis e Fascia transversalis and peritoneum

Meckels diverticulum is a result of which of the following developmental abnormalities shy

A Failure of the vitelline duct to close

B Failure of the herniated intestinal loop to retract into the abdomen

C Failure of the urachus to close

D Failure of the midgut to rotate

E Failure of the hepatic duct to close

Explanation

Meckels diverticulum is a result of the persistence of the proximal part of the vitelline duct This

diverticulum is usually found about 2 feet proximal to the ileocecal junction and is usually about 2 inches

long It is present in about 2 of the popUlation It may be the site of ectopic pancreatic tissue or gastric

mucosa and may develop inflammatory processes and ulcerations Acute Meckels diverticulitis

simulates appendicitis

Which of the following veins carries blood from the esophagus to the portal vein The

A right gastric vein

B left gastric vein c splenic vein D azygos vein

E left gastroepiploic vein

Explanation

The left gastric vein a direct branch of the portal vein drains blood from the lesser curvature of the

stomach and the inferior portion of the esophagus Because branches of the portal vein do not have

valves blood can flow in a retrograde path when there is an obstruction to flow through the portal system or liveL Rlooci Cln then flow from the nortl] vein thr()1Ph the left PRstric vein to the esonhlPlIS lno

through venous communications within the submucosa of the esophagus to esophageal veins that drain

into the azygos vein The increase in blood flow through the esophageal submucosal veins results in esophageal varices

On the posterior wall of the abdomen the celiac ganglion A contains cell bodies of postganglionic parasympathetic neurons B is synapsed upon by neurons in the posterior vagal trunk C is synapsed upon by neurons in the greater splanchnic nerve D contains sensory cell bodies of lumbar spinal nerves E contains cell bodies of neurons that cause an increase in the rate of peristasis

Explanation The celiac ganglion is one of the preaortic ganglia of the sympathetic nervous system It contains cell bodies of postganglionic sympathetic neurons The sympathetic splanchnic nerves contain preganglionic sympathetic neurons that pass through the sympathetic chain without synapsing These splanchnic nerves go to the preaortic ganglia to synapse The greater splanchnic nerve contains preganglionic neurons from spinal cord segments T5-T9 This nerve synapses in the celiac ganglion The nerve fibers in the vagal trunks are preganglionic parasympathetic fibers that go to the walls of the organs that they will innervate and synapse on postganglionic parasympathetic neurons in the walls of those organs Cell bodies of sensory neurons in the abdomen are found in the dorsal root ganglia or the sensory ganglia of the vagus nerve Sympathetic innervation decreases the rate of peristalsis parasympathetic innervation increases the rate of peristalsis

Which of the following pairs of arteries will allow blood to bypass an occlusion of the celiac trunk

A Left gastric artery-right gastric artery

B Left gastroepiploic artery-right gastroepiploic artery

C Superior pancreaticoduodenal artery-inferior pancreaticoduodenal artery

D Splenic artery-common hepatic artery

E Left gastric artery - proper hepatic artery

Explanation The anastoOlosis of a branch of the celiac trunk and a branch of the superior mesenteric artery will

provide collateral circulation around an occlusion of the celiac trunk Each of the other choices pair

branches of the celiac trunk therefore these will not provide collateral flow around the obstruction of the

celiac trunk The left gastric splenic and common hepatic arteries are direct branches of the celiac trunk

The right gastric artery is a branch of the proper hepatic artery which is a branch of the common hepatic artery The left gastroepiploic artery is a branch of the splenic artery The right gastroepiploic artery is a

branch of the gastroduodenal artery whlch is a branch of the common hepatic artery

Which of the following organs has appendices epiploica The

A sigmoid colon

Bjejunum

C duodenum

D stomach E esophagus

Explanation Appendices epiploica are characteristic of the colon Appendices epiploica are subserosal accumulations

of fat None of the organs of the gastrointestinal tract has appendices epiploica except the colon

Page 33: Chirag's Abdomen Review

Lt Dome of Diaphragm (why left Look this up in Adams)

Spleen (What is the blood supply)

Left Kidney (What is the blood supply - AND how is it different from the R kidney)

Suprarenal Gland (What is the Arterial AND Venous Blood supply - how are they different)

Pancreas (How does supply differ from Head to Tail What is the SMA Relationship)

Transverse Mesocolon

liver - ADAMSWET - Make sure you look at the liver in wet lab

Left triangular nl1am~nt

ComoaDj ligamnt

Erophg~1 impre$ioo

Hepatio veins

In1erior -ifena middotr3)Ia

Fibrous appendix o-t

live

impr~j on

Heprorendl p~rtion of Q)(Qllary ligament

Righllri~n9ul r 1I~met

(Common) bile quol

Gr)mmCtr~ hepatic dlJct

Ccentic duct

Duodenal impression

GaJdate p-fr)~S

Hepatic artgtrl prop-f iiiiila - Faloiform ligament

_ - shy Round ligamen liver

~--F-- CoJio imprgt-ssi-on

Prta heptis

Identify the lobes impressions and embryonic remnants associated with the liver

Caudate Lobe Quadrate Lobe Right Lobe Left Lobe Round ligament Falciform Ligament

Ligamentum Venosum (what is its fxn in embryonic life) Hepatic Veins (NOT PART OF THE

PORTAL TRIAD) IVC PORTAL TRIAD - Contents relationship cross section etc Know the

Galbladder relationship to the lobes of the liver

Biliary Duct System - Make sure you understand the sequence of these structures - BE ABLE TO

DRAW A FLOW CHART

TPVd i

t

I t

1 __ Cm-(r

patk GlJet

I

J

Clinical = JAUNDICE is caused by anything that prevents delivery of bile to intestine Tumor of the

head of the pancreas Stones etc Patient will have pale stools and yellowish colored mucus

membranes

Clinical- Any scenario that tells you the patient has BILLOUS VOMIT means that the obstruction to

the flow of digestive contents is after the Ampulla of Vater (Site of Entry of Billiary system to the

duodenum) - ie Duodenal Atresia

Spleen -located posterior to the mid axillary line between ribs 9 and 11 Make sure you know that

the 10th rib is the main axis of the spleen and this organ is susceptible to injury (stab wound errant

thoracoce ntesis etc)

The spleen is derived from mesodermal cells - NOT THE GUT TUBE

The spleen rests on the left colic flexure associates with the tail of the pancreas Know the

structures entering the Hilum of the spleen

Sh rt O~-t~ic 1 0(0 10 rtiltSPIric Iloa nt

(cut)

Peritoneum - similar concept to Pleura - think of a fist in a balloon

Visceral Peritoneum - Layer of balloon touching your fist

Parietal Peritoneum - Layer of balloon not touching your fist

Your fist represents the organ your wrist is the hilum and your arm contains the blood supply

entering the organ

Appreciate that there will never be organs in the peritoneal cavity - rather these organs invaginate

the cavity Kaplan videos

RULES OF NOMENCLATUREshy

1 Organ completely surrounded by peritoneum - peritoneal organ

2 Organ partially surrounded by peritoneum- Retroperitoneal

3 Peritoneum surrounding peritoneal organ is VISCERAL peritoneum

4 Peritoneum surrounding retroperitoneal organ is PARIETAL peritoneum

5 Peritoneum connecting visceral to parietal is called messentary 2 messentaries in the

gut Dorsal (to the gut tube) and ventral (to the gut tube) messentary

Aorta is in Retro peritoneal position - but blood must reach peritoneal position - vessels travel through

messentary All peritoneal organs will have blood supply reaching through messentary

-Mesentery is a 2 layer peritoneum with a neurovascular communication between body wall and organ

- Ligament connects one organ with another or to the abdominal wall (Ommentum = ligament)

lesser Ommentum (attach lesser curvature of stomach and duodenum to liver) =Hepatoduodenal

Ligament and Hepatogastric Ligament

Has a Superior and Inferior Recess (Accumulation of Fluid in Ascites)

Communicates with the greater sac through the epiplic foramen (what structures pass through

this foramen)

Boundaries - you must be able to visualize this

o Anterior - stomach

o Posterior - parietal peritoneum pancreas

o Superior - superior recess (bw diaphragm and coronary ligament)

o Inferior -Inferior recess (bw layers or greater momentum

Greater Ommentum (attach greater curvature of stomach) Gastrophrenic ligament Gastrosplenic

ligament gastrocolic ligament

The greater omentum is the largest peritoneal fold It consists of a double sheet of peritoneum folded on itself so that it is made up of four layers The two layers which descend from the greater curvature of the stomach and commencement of the duodenum pass in front of the small intestines sometimes as low down as the pelvis they then turn upon themselves and ascend again as far as the transverse colon where they separate and enclose that part of the intestine

ABDOMINAL PAIN

Parietal Peritoneum - supplied by same vasculature lymphatics and nerves supplying body wall it

lines and diaphragm Sensitive to pain pressure heat cold well localized

Visceral Peritoneum - supplied by same vasculature lymphatics and somatic nerve of organ it covers

Insensitive to touch heat cold and laceration - referred to dermatome of spinal ganglia providing

sensory fibers Where does appendicitis refer to

Foregut pain - epigastric area (ie - cholycystitis)

Midgut pain - periumbilical area (ie - appendicitis)

Hindgut Pain - suprapubic area (ie - diverticulitis)

Extra ImagesConcepts

ll~_____-

FalifCtrm ligament oind r~ud ligamet f Ilver

Blood from splenio gastriC and inferiof rne$e-rteri v~ins

Ca-I tributaries

Lett gastrio Ifein

Posterior superior pan~reatioodul)denal vaihS

Lott gamo-om~nlal (9aropip lomiddotic) -in

Poq_~ tjol imerl-9-r panCJertlcorllJod-nal veiopound --amp----I- - ~J Right grtr~-omntal

Anwrior interi (gartroepiploic) Jjn

pan euaii cod vl)denal veins middot Inf~Ji (t r mesentric vein

Miqdle (olic vein

Right cl)licvein Sigmoid and rectosigml)id (ei ns

IhH)Collc(~io

--- Mi~dl laquooLJl gtjrltgt

PoM ca vl1 illasto)moses -----shyampoptoageal 2 Paraumbilie-lt11 Inferi or Fectal vei ns

3 Recial 4 REuoperHonea1

Know how the Portal vein is formed I 4 sites of portal caval anastamoses and 1 clinical shunt

Col li t ltt-~ otTl~tI ~nj pc~ 1lt1 turJoG

Ltf 14i1 tImiddot~ artoftl9 on tj phtAt$

L-oftqf 4t t~r 1=laquoIran d 1 bull shy~p l ci rj o fOOOts

Nerves follow the arteries - appreciate the splanchnic nervous system I

Uet~ric branch of left ~nal art

Ureterie branch of righi renal artelY

Left Zld lumbar in and co mlTlunication to as)erdin9 lumbar l(~in Hi ~ht tEZ1~~t~ t3r j t itn ~ nJ l1t- rlnd lfe i r1

Inferior me5nteri~ artery

Notice that the right testicular vein drains directly into the IVC and the right testicular artery drains

directly into the aorta However the left testicular vein drains into the L renal vein at a right angleshy

reason left testicle is lower and more susceptible to varicocele (bag of worms)

Also notice that the left renal vein has a longer course because the IVC is on the right side whereas

the right renal artery has a longer course because the aorta is on the left side

Appreciate the anterior to posterior relationship of structures in the hilum of the kidney - VAP - Vein

Artery Renal Pelvis (Ureter)

11____ __ L_ L_ n VJ __ _ _ t_L I I_ _ L __ L_ I -pound1 bull LI_~-I ____

Posterior View of Head of Pancreas in ( of Duodenum

Celiao hunk

Co mmon ~L~jJth art~ry

GastNduQdonal artrf (partilly in phantn)

P1)Sterior $Up~Jior panCflaticuduodfmal art~r~t

(Co mm on) bile duct

middot~1t~~t-1l---~-~- Right gshomiddotomental (gastoe plp lolc) 3rte (phantomost)

Grener paocre atic art-ry

1n1~rjor pancr-iatlc artery

Jtrifll supejo r pal)oreailcento)dJodenal artr1 (phantom)

Anastomotlo branch

POostetlor bJanch of jo f~ri of pan-reatir(lduodensl drttnj

Anterio r branch of i flferior palcreati~)duodenal art~(phan1om)

Notice the extensive blood supply to the pancreas and duodenum via the branches of the celiac trunk

Notice collateral supply from SMA branches - makes sense bc this is the jxn of foregutmidgut

Identify the vessels in this arteriogram

Hiltid i)f N~ck oi B)dvof Tail 01 pa nereas pan cent~as P-nmiddot-reas panCtCas

I nferie v~na cava

jHept1iic p(lrlai v~in

Port1 tnd H~pti lt a ftH prol

Comm on) bll duct

Ouodtnum

~ft colic (sio)Atta~ hmtrlt jt~xJr-ofha~elSe

muo(IIQn

Right ~lIc (h~j)tic)

il~gtture

In1triol m~oten lIein (rttr op~ritoMdO

SlJp efl or mes~n~fiC amrV and lipln

KNOW YOUR NEIGHBORHOOD

Questions

vVhiJh structure supplied by a bnmdlof the cclia( artery is not derivcd from foregut LemCJUCrITI

(A) Head of the pancte-a5

CD) Pyloric duolenum

Cystkduct

( Liver hepatocyt~~

~F) Body of the spleen

An infant presents with an omrhaJucele at birth -hi oJ the [oHm illg applies to his cM1-dition

(A) It is 31so seen ill p4titnts with aganghonic megacolon

(11) ft reuirs from a fal1ure of resorption of theviteUine d let

(C) It results from herniation at the-site of regression of the right umbilk vein

DJ It is caustd by faihtrc of recanalization of the midgut part of the duodenum

~ It ill camioo by a failuIt vf the midgul to return to the abGQminal uity after herniashytion in-n the urnbilk s l stalk

Ot er than the spleen occlusion Cif the spit-Ilk artery at its odgin wm most likely affect die blood supply to jllch st cnud

(A) Jejunum

(B) Body of th pal1~lltas

(C) LeSStT Cllmiddotlaturc of tl )toma-ch

(D Duodenum dista to the entrance of the Ornmou bile duct

E Fundus of the stomach

A 38-yeu-old batL~er with a history of heartburn suddenly experiences excluciating pain in the (plgastric region of th~ abdomeu SurgCry is perf~rme immediard y upon admisshysion to the 1IlcrgCJliy tuomh~re i~ evidence uf a ruptured ulcer in the posterior waU of the stomach Vhere will a surgeon first fi nd the stomach contenlSf

A) Greater p4ritoneal sac

rB) Cul~de-s~c of Douglas (--

C Omental bursa ~

--D) Paracolic gutter

rEj Between -he panttal perimltum and the posterior body wal1

At birth an infant presents with a st()ma~ rb~tbas~njJled jfltotb~diaplfagru 1A1ltre is the defect thatresulied iiitJle heini~t()n shy~tsophagealbiatus

7 - rH-- Hiatus for the inferior vena cava

( Pleuroperitoneal membrane -(0) Septum transvcrsum

(E) Right Crlt~

An infant born with DOVv7l syndrome presents with bili()u~ vomiting Ahat congenital defect does the infant have

(A) Pyloric stenosis

(B) Meckel diverticulum C) Ornphaloce1e

(D) Gastroschisis

( ~ ) Duodenal atresia y A patient with cirrhosis of the liver presents with ~ bacalvaricestnlreased retrograde pressure in which veins caused the varices

(A) Paraumuilical

(B) Splenic

(ct AzygltJus

(15))G~trk ( (-F) Superior mesemeric

A htaltby 3-year~old male patient experiences a hernial sa protruding from the anterior abdominal wall about halfway between me anterior superior ilia spine and the pubk tuberde Pulsations of al1 artery are palpated medial to the protrusion site through the abdominal walL Which layer of the anterior abdominal wall will first be traversed by the

1hctma

fA) Rectus sheath (B) External oblique aponeurosis

(C) Inguinal ligament

lD) Transversalis fusda

(E) Cremasteric fa~cia

After 5urgi(aj ffpair of a hernia the patient tXperienccs mtmlgtness in the skin on the anteshyrior aspect of the S(Totum_ Vhaf nerve may have been lesioned during thehemiorrhaphy

(A) Femoral

(B) Obturator

(C) Ilioinguinal

(D) lliohypogastrk

(E) Pudendal

A 23~year-LJld female secretary il1 good health ~-uddcn1) doubles over with pain in the a ea of the 1JmbRicu$ Sbe feels vartn and ltneasy and has no appetite That night the pain seems to have mQved to the tower right abdominal regjol1 and she calls her family doctor who then arranges for an ambulance to pk-k her up and take her to the hospitaL Wh ell ntn~ perceived in the area of the urnbilirus most Hkely carried lhe pairfu I sensations into the eNS

tA) Vagus nerves I~

V B)

) Lessersplanchnk nerves

tC) Pudendal nerves

(D) lIiohpogastrk nerves

(E) Greater splam ic l erves

A CT reveals carcinoma in the bOod of the ancreas Vhich blood vessel trut ourses ----~- - -bull ------ --shy

immediately poftterior to the body ofthe pancreas is the m~t likely to be oompressed

(A) Splenk artery

(B) Abdominal aorta (C) Portal vein

(1) Splenic vein

(E) Renal vein

A patient has a penrln1l1ng uker of the posterior wall ot the br~l part ot the (lUooenmn llkh blood vessel is subject to erosion

(A) Common hepatic artery

(B) Gastroouodenal artery

(C) Proper hevatic artery

(D) Celiac artery

(E) Anterior inferior 11amrelltlcoduodcnal attery

Your patient has been diagnosed -ith a carcinoma locallted to the head and l~e(k of the pancreas Another clinical sign would be

A esophageal varices

(8) hemorrhoids

C) a caput medusa

(D) increased pra Teuro n th~ hepatic veins

(E) enlarged right supra lavkular lymph nodes

Wltkh of the foUowing structures develops in the ventral mesentery

(A) Spleen

(B) Jeiunum (C) Head of1ht pancreas (D) Transverse colon (E) Stomach

ti l Uw ~ littwin~ f( S-t lil oai Imdge ~ hi(h or tbt la~)d J truetur tgt liJ llntn nl) he hl p UC iJd [IIi ell

c o

A) drains Ie tht infCrior a La aI

R t middot~nfl0 ~ill to th~ lunlgtn of h i dtlndCrlllfH

(e) m t bull JiJattd on tl l J n T ~H

D ) sup Lc O VSlt I Hlid bhtu l 1 li - -I un oid

( ) U~tpli(t tr j middottUh~ 1 v(( b~nt rfK n1ilc~Zm

ANSWERS AND EXPLANATIONS

Answer E The spleen is t hlttnopodicand lymph organ demlted from mesoderm

Answ~ R Al1 tlmphalocele is caused by it failure of the nlidgut to return to the ahdomir nat cavity after herniation into the umbiliau Stalk Choices Aand D maybe seen in infants with Down syndrome choice D ~s the specific CBuse ofduudcnal JtiCSitt Choice C is (ile cause of gclstrosbisis and Choice B nsults iu a Meurolktldivertku1-tlB

Answer B The fundus ofthe stomach is suppHed by soort gastric brunches of the splenic altery The splenic artery supplies the body and tail of the pancreas part of the greater curvature of the sttmla(h and the spleen Te jejunum part of the head of the pancreas and tht~ duodenum distal to the entrance of the commOll bile duct are supplied by the superior mesenterk artery clll~l ~be less r ctlt1ature cmd the pylQric antrum are supplied by the right and lei gastric art(ries

AnSWftt C Tbeomental bursa or lesser ~ritoneaj sac lies direcdy posterior to the proxshyimal part of the duodeTtlm and the stomach and would be the first site where stomach contents ~Ott1d be fpoundluncL

Answer C A defect in a llleuropcritoneal membrane (uswlly the left) is the typical site of i1 cc-ngenitlI diilphragluatic hemia llere the membr4ne fails to dose ()pound( of the perishycCirdiopcritulleal canals

Answer E DuoJenal atresia and aganglionic megacoion are congwitaI defects S~Il in patients with Dowmiddotnsyndrome

Answer D RulaTgemt~llt of and retrograde flow in g~lstrk vel_ns in particlJl~r the kft gas~ tricveins dilates the capillary bed in rhe wall of the esophagus in (ases of porta yper~

tension Blood flow would increase in and dilampte tribntarkgts of the (lZygOUS vein on the other side of the capiUary bed but flow in this vein is in the typical direction t()ward the superior vena cava Paraumbiii(ltU vein eilgorgement contributes to a caput medusH Splenic ~nlargement might prc~nt with 5plcnonlegaly and balt-kflow in to tlu superior m~~ntclic vein occurs but is asymptomatic

Answer D The patient hagt an indirect inguinal hernia whi~h emerges from the antt-rior abdominal wall through the deep inguinltilling Theeep ring is a fault in the transv~rshysaUs fascia this I~yer wiIJ be penetrated first by the hernia

An~Wer C The ilioinguinal nenc which provides sens~llion to the lnedlal thigh ltmclanteshytior SClotunl pass~lt th rough the 5uperfh_ial inguinal ring ind $subject to inj i1T) becaus-e

it is in the operatitm Held of the erniorrhapny

Auswer B The leMHr splanchnic nerves are sympathdic nerVlts that carry viscera l sensashytlltgtrogt ftom illtllt1m~d ()J stietched gust (itinteitinal ~tructures (in this case the pprndix) into tnt eNS Lesser splanchnic ntTYcsarisc from thmiddot T9--T12 spinal cord segments lt1nd provide sympathetic innenation tD rnidgut siruc1ures whiCh include CLe app~JldD Viscera] Pain arising from affecLed Inidgut ampt 1C1ure is referred over the same dl- matorne~ of spinal segrnertts v-hich provide the sympathetic Innervation n this G1SC of appendicitis the invohen~n t of the ltire) of t e unlhHku indud s the T 10 dermatome

Answer B Of the five choices onty the dscending olon is retroperiton~al aldwould be a lik ~ ( choice to be seen immediately a(~jilcent to t11e posterior abdominal middotn~L

Amwen D The SpltftlC ~-ein ourses posterior to the body of the panneas m its way tt drain into the superior mCSfttltlri( vein

Answcr B TILt glstrodllolticnal artery 1 direct hIamh of the comrootl hepatic artery courses immediately pt))iwri() to the duodenum and is slbject to erosion

Answer B Carcinoma of th pan middott3S in the 1tilt1 may compreampgt the portltil vein at irs orishygill The poTtai vcin is fomled when the splenic vein jQiaswith tfie superior meStllt eric vein The inferiot mesenteric vein joins the ~plenjc vein just priOT to tlli~ point at which the splenic joins the superior Jlleit1ltcri( vein Increescd venous presslu in the inferior mesenteric vein is a cause of emo hoid~

Answer C The- velltral pancreas wilich forms most of the head of the p ~ncr as develops in the ventral mes(ntery as antutgrowth of the hepatic diverticulum Th~ hepatic divershyticulull induding the biIJary appa~atus develops in tbe ventral mesentery of the foregut

Answer~ A The superior mesenteric ~in joins with the spienkvein to form the hepatic portal vciu

Answer D The structure at gttlK is the proper hepatic artery~ whkh suppUesoxygenated b middotood to the liver

MAKE SURE YOU KNOW the diff bw Rectus Sheath above and below the arcuate line

ABOVE

Aponeurosis of xiiltmal obllque musclo

Extemll f)biquw musde

Reotln ilbdomlnls musole S~in

Internal 9bliquQ mY~QI

AponeUfOsi$ of hJH$V~~S Lir9a a lb lbdolTlin~ musolo Tri OJV6 rUi

atldomlnis mUS(loe

Sub cutanlilous tiue (tatty ye r)

BElOW

A POrl lJfosis 01 etemal oblique muscle

Aponeul~)sis 01 Internal oblique mU$cl~

Anteriol lay~ of r~ltdus st~ath EXttom1 oblique rnu$cll

Rectus Jbdominis muscle Intoernal Aponeurc-sis of tra~fersU$ oblique muscle-

at-domlnis muscentl ~ Skio

Tra nsvitSus abdomioLs ml)ZClt

TralSVersaHs fascia Medial umQil iegtt1 1i9Jment -and folj

Uldchus Peritoneum (ir median Umbilj~al Suboutane ous

Extraprftone 11ascia

Ymbilimiddot~1 fold)

preu9poundiea1 fascia

tissue (fatty 4nd m~mbr3n(iUS layers)

o Above the arcuate line (A horizontal line 13 of the distance bw the umbilicus and the

pubic symphysis) -10 Aponeurosis divides into an AntPost Laminae

o The Ant Laminae joins EO and Post Laminae joins Trans Abdominis = Ant and Post

RECTUS SHEATH respectively

o BElOW the arcuate line - all 3 aponeurosis join ANTERIOR to rectus muscle to meet its

counterpart in the midline (linea Alba)

o Take away Msg - The abdomen is devoid of a posterior rectus sheath below the

arcuate line and is therefore more vulnerable to herniasinjuries

Question - A physician makes a deep incision in the patients midline immediately superior to

the pubic symphysis which of the following layers is his knife least likely to pass

Rectus Abdominis External Oblique Ant Rectus Sheath Posterior Rectus Sheath All of the

Above

Answer - All of the above None of the other answer choices are midline structures -LINEA

ALBA

Linea Alba has very poor blood supply - doesnt heal well after surgery Therefore this is a

common site for incisional hernias

a Spleen b Transverse colon c Descending colon d Stomach e Pleura

17 Meckels diverticulum is normally found 2 feet proximal from the

a Pyloric sphincter b Lower esophageal sphincter c Ileo-cecal valve d Middle valve of Huston e Anal valve

18 Ulcer in the posterior wall of the first part of the duodenum would erode ___ artery and would cause bleeding

a Left gastric b Right gastric c Hepatic artery proper d Gastroduodenal artery e Middle colic artery

19 An inflamed appendix is identified by a surgeon on the operation table by noting

a The appendicies epiploicae b The convergence of tenia c The artery of Drummond d The mesocolon e The mesosalphinx

20 The nerve which emerges through the psoas major is

a Femoral b Ilio-inguinal c Ilio-hypogastric d Pudendal e Subcostal

21 The right gonadal vein drains into the

a Azygos b Hemiazygos c Inferior Vena Cava d Right renal vein e Left renal vein

22 The hepatocytes in the liver is derived from

a Ectoderm b Endoderm c Mesoderm

d Neural ectoderm

23 Abscess in the lumbar vertebrae due to tuberculosis would spread to the adjacent muscle which is

a Psoas Major b Iliacus c Quadratus lumborum d Tranversus Abdominis

24 The anterior wall of the inguinal canal is formed by

a External oblique and transverses abdominis b External oblique and fascia transversalis c Internal oblique and external oblique d Internal oblique and transverses abdominis e Fascia transversalis and peritoneum

Meckels diverticulum is a result of which of the following developmental abnormalities shy

A Failure of the vitelline duct to close

B Failure of the herniated intestinal loop to retract into the abdomen

C Failure of the urachus to close

D Failure of the midgut to rotate

E Failure of the hepatic duct to close

Explanation

Meckels diverticulum is a result of the persistence of the proximal part of the vitelline duct This

diverticulum is usually found about 2 feet proximal to the ileocecal junction and is usually about 2 inches

long It is present in about 2 of the popUlation It may be the site of ectopic pancreatic tissue or gastric

mucosa and may develop inflammatory processes and ulcerations Acute Meckels diverticulitis

simulates appendicitis

Which of the following veins carries blood from the esophagus to the portal vein The

A right gastric vein

B left gastric vein c splenic vein D azygos vein

E left gastroepiploic vein

Explanation

The left gastric vein a direct branch of the portal vein drains blood from the lesser curvature of the

stomach and the inferior portion of the esophagus Because branches of the portal vein do not have

valves blood can flow in a retrograde path when there is an obstruction to flow through the portal system or liveL Rlooci Cln then flow from the nortl] vein thr()1Ph the left PRstric vein to the esonhlPlIS lno

through venous communications within the submucosa of the esophagus to esophageal veins that drain

into the azygos vein The increase in blood flow through the esophageal submucosal veins results in esophageal varices

On the posterior wall of the abdomen the celiac ganglion A contains cell bodies of postganglionic parasympathetic neurons B is synapsed upon by neurons in the posterior vagal trunk C is synapsed upon by neurons in the greater splanchnic nerve D contains sensory cell bodies of lumbar spinal nerves E contains cell bodies of neurons that cause an increase in the rate of peristasis

Explanation The celiac ganglion is one of the preaortic ganglia of the sympathetic nervous system It contains cell bodies of postganglionic sympathetic neurons The sympathetic splanchnic nerves contain preganglionic sympathetic neurons that pass through the sympathetic chain without synapsing These splanchnic nerves go to the preaortic ganglia to synapse The greater splanchnic nerve contains preganglionic neurons from spinal cord segments T5-T9 This nerve synapses in the celiac ganglion The nerve fibers in the vagal trunks are preganglionic parasympathetic fibers that go to the walls of the organs that they will innervate and synapse on postganglionic parasympathetic neurons in the walls of those organs Cell bodies of sensory neurons in the abdomen are found in the dorsal root ganglia or the sensory ganglia of the vagus nerve Sympathetic innervation decreases the rate of peristalsis parasympathetic innervation increases the rate of peristalsis

Which of the following pairs of arteries will allow blood to bypass an occlusion of the celiac trunk

A Left gastric artery-right gastric artery

B Left gastroepiploic artery-right gastroepiploic artery

C Superior pancreaticoduodenal artery-inferior pancreaticoduodenal artery

D Splenic artery-common hepatic artery

E Left gastric artery - proper hepatic artery

Explanation The anastoOlosis of a branch of the celiac trunk and a branch of the superior mesenteric artery will

provide collateral circulation around an occlusion of the celiac trunk Each of the other choices pair

branches of the celiac trunk therefore these will not provide collateral flow around the obstruction of the

celiac trunk The left gastric splenic and common hepatic arteries are direct branches of the celiac trunk

The right gastric artery is a branch of the proper hepatic artery which is a branch of the common hepatic artery The left gastroepiploic artery is a branch of the splenic artery The right gastroepiploic artery is a

branch of the gastroduodenal artery whlch is a branch of the common hepatic artery

Which of the following organs has appendices epiploica The

A sigmoid colon

Bjejunum

C duodenum

D stomach E esophagus

Explanation Appendices epiploica are characteristic of the colon Appendices epiploica are subserosal accumulations

of fat None of the organs of the gastrointestinal tract has appendices epiploica except the colon

Page 34: Chirag's Abdomen Review

Biliary Duct System - Make sure you understand the sequence of these structures - BE ABLE TO

DRAW A FLOW CHART

TPVd i

t

I t

1 __ Cm-(r

patk GlJet

I

J

Clinical = JAUNDICE is caused by anything that prevents delivery of bile to intestine Tumor of the

head of the pancreas Stones etc Patient will have pale stools and yellowish colored mucus

membranes

Clinical- Any scenario that tells you the patient has BILLOUS VOMIT means that the obstruction to

the flow of digestive contents is after the Ampulla of Vater (Site of Entry of Billiary system to the

duodenum) - ie Duodenal Atresia

Spleen -located posterior to the mid axillary line between ribs 9 and 11 Make sure you know that

the 10th rib is the main axis of the spleen and this organ is susceptible to injury (stab wound errant

thoracoce ntesis etc)

The spleen is derived from mesodermal cells - NOT THE GUT TUBE

The spleen rests on the left colic flexure associates with the tail of the pancreas Know the

structures entering the Hilum of the spleen

Sh rt O~-t~ic 1 0(0 10 rtiltSPIric Iloa nt

(cut)

Peritoneum - similar concept to Pleura - think of a fist in a balloon

Visceral Peritoneum - Layer of balloon touching your fist

Parietal Peritoneum - Layer of balloon not touching your fist

Your fist represents the organ your wrist is the hilum and your arm contains the blood supply

entering the organ

Appreciate that there will never be organs in the peritoneal cavity - rather these organs invaginate

the cavity Kaplan videos

RULES OF NOMENCLATUREshy

1 Organ completely surrounded by peritoneum - peritoneal organ

2 Organ partially surrounded by peritoneum- Retroperitoneal

3 Peritoneum surrounding peritoneal organ is VISCERAL peritoneum

4 Peritoneum surrounding retroperitoneal organ is PARIETAL peritoneum

5 Peritoneum connecting visceral to parietal is called messentary 2 messentaries in the

gut Dorsal (to the gut tube) and ventral (to the gut tube) messentary

Aorta is in Retro peritoneal position - but blood must reach peritoneal position - vessels travel through

messentary All peritoneal organs will have blood supply reaching through messentary

-Mesentery is a 2 layer peritoneum with a neurovascular communication between body wall and organ

- Ligament connects one organ with another or to the abdominal wall (Ommentum = ligament)

lesser Ommentum (attach lesser curvature of stomach and duodenum to liver) =Hepatoduodenal

Ligament and Hepatogastric Ligament

Has a Superior and Inferior Recess (Accumulation of Fluid in Ascites)

Communicates with the greater sac through the epiplic foramen (what structures pass through

this foramen)

Boundaries - you must be able to visualize this

o Anterior - stomach

o Posterior - parietal peritoneum pancreas

o Superior - superior recess (bw diaphragm and coronary ligament)

o Inferior -Inferior recess (bw layers or greater momentum

Greater Ommentum (attach greater curvature of stomach) Gastrophrenic ligament Gastrosplenic

ligament gastrocolic ligament

The greater omentum is the largest peritoneal fold It consists of a double sheet of peritoneum folded on itself so that it is made up of four layers The two layers which descend from the greater curvature of the stomach and commencement of the duodenum pass in front of the small intestines sometimes as low down as the pelvis they then turn upon themselves and ascend again as far as the transverse colon where they separate and enclose that part of the intestine

ABDOMINAL PAIN

Parietal Peritoneum - supplied by same vasculature lymphatics and nerves supplying body wall it

lines and diaphragm Sensitive to pain pressure heat cold well localized

Visceral Peritoneum - supplied by same vasculature lymphatics and somatic nerve of organ it covers

Insensitive to touch heat cold and laceration - referred to dermatome of spinal ganglia providing

sensory fibers Where does appendicitis refer to

Foregut pain - epigastric area (ie - cholycystitis)

Midgut pain - periumbilical area (ie - appendicitis)

Hindgut Pain - suprapubic area (ie - diverticulitis)

Extra ImagesConcepts

ll~_____-

FalifCtrm ligament oind r~ud ligamet f Ilver

Blood from splenio gastriC and inferiof rne$e-rteri v~ins

Ca-I tributaries

Lett gastrio Ifein

Posterior superior pan~reatioodul)denal vaihS

Lott gamo-om~nlal (9aropip lomiddotic) -in

Poq_~ tjol imerl-9-r panCJertlcorllJod-nal veiopound --amp----I- - ~J Right grtr~-omntal

Anwrior interi (gartroepiploic) Jjn

pan euaii cod vl)denal veins middot Inf~Ji (t r mesentric vein

Miqdle (olic vein

Right cl)licvein Sigmoid and rectosigml)id (ei ns

IhH)Collc(~io

--- Mi~dl laquooLJl gtjrltgt

PoM ca vl1 illasto)moses -----shyampoptoageal 2 Paraumbilie-lt11 Inferi or Fectal vei ns

3 Recial 4 REuoperHonea1

Know how the Portal vein is formed I 4 sites of portal caval anastamoses and 1 clinical shunt

Col li t ltt-~ otTl~tI ~nj pc~ 1lt1 turJoG

Ltf 14i1 tImiddot~ artoftl9 on tj phtAt$

L-oftqf 4t t~r 1=laquoIran d 1 bull shy~p l ci rj o fOOOts

Nerves follow the arteries - appreciate the splanchnic nervous system I

Uet~ric branch of left ~nal art

Ureterie branch of righi renal artelY

Left Zld lumbar in and co mlTlunication to as)erdin9 lumbar l(~in Hi ~ht tEZ1~~t~ t3r j t itn ~ nJ l1t- rlnd lfe i r1

Inferior me5nteri~ artery

Notice that the right testicular vein drains directly into the IVC and the right testicular artery drains

directly into the aorta However the left testicular vein drains into the L renal vein at a right angleshy

reason left testicle is lower and more susceptible to varicocele (bag of worms)

Also notice that the left renal vein has a longer course because the IVC is on the right side whereas

the right renal artery has a longer course because the aorta is on the left side

Appreciate the anterior to posterior relationship of structures in the hilum of the kidney - VAP - Vein

Artery Renal Pelvis (Ureter)

11____ __ L_ L_ n VJ __ _ _ t_L I I_ _ L __ L_ I -pound1 bull LI_~-I ____

Posterior View of Head of Pancreas in ( of Duodenum

Celiao hunk

Co mmon ~L~jJth art~ry

GastNduQdonal artrf (partilly in phantn)

P1)Sterior $Up~Jior panCflaticuduodfmal art~r~t

(Co mm on) bile duct

middot~1t~~t-1l---~-~- Right gshomiddotomental (gastoe plp lolc) 3rte (phantomost)

Grener paocre atic art-ry

1n1~rjor pancr-iatlc artery

Jtrifll supejo r pal)oreailcento)dJodenal artr1 (phantom)

Anastomotlo branch

POostetlor bJanch of jo f~ri of pan-reatir(lduodensl drttnj

Anterio r branch of i flferior palcreati~)duodenal art~(phan1om)

Notice the extensive blood supply to the pancreas and duodenum via the branches of the celiac trunk

Notice collateral supply from SMA branches - makes sense bc this is the jxn of foregutmidgut

Identify the vessels in this arteriogram

Hiltid i)f N~ck oi B)dvof Tail 01 pa nereas pan cent~as P-nmiddot-reas panCtCas

I nferie v~na cava

jHept1iic p(lrlai v~in

Port1 tnd H~pti lt a ftH prol

Comm on) bll duct

Ouodtnum

~ft colic (sio)Atta~ hmtrlt jt~xJr-ofha~elSe

muo(IIQn

Right ~lIc (h~j)tic)

il~gtture

In1triol m~oten lIein (rttr op~ritoMdO

SlJp efl or mes~n~fiC amrV and lipln

KNOW YOUR NEIGHBORHOOD

Questions

vVhiJh structure supplied by a bnmdlof the cclia( artery is not derivcd from foregut LemCJUCrITI

(A) Head of the pancte-a5

CD) Pyloric duolenum

Cystkduct

( Liver hepatocyt~~

~F) Body of the spleen

An infant presents with an omrhaJucele at birth -hi oJ the [oHm illg applies to his cM1-dition

(A) It is 31so seen ill p4titnts with aganghonic megacolon

(11) ft reuirs from a fal1ure of resorption of theviteUine d let

(C) It results from herniation at the-site of regression of the right umbilk vein

DJ It is caustd by faihtrc of recanalization of the midgut part of the duodenum

~ It ill camioo by a failuIt vf the midgul to return to the abGQminal uity after herniashytion in-n the urnbilk s l stalk

Ot er than the spleen occlusion Cif the spit-Ilk artery at its odgin wm most likely affect die blood supply to jllch st cnud

(A) Jejunum

(B) Body of th pal1~lltas

(C) LeSStT Cllmiddotlaturc of tl )toma-ch

(D Duodenum dista to the entrance of the Ornmou bile duct

E Fundus of the stomach

A 38-yeu-old batL~er with a history of heartburn suddenly experiences excluciating pain in the (plgastric region of th~ abdomeu SurgCry is perf~rme immediard y upon admisshysion to the 1IlcrgCJliy tuomh~re i~ evidence uf a ruptured ulcer in the posterior waU of the stomach Vhere will a surgeon first fi nd the stomach contenlSf

A) Greater p4ritoneal sac

rB) Cul~de-s~c of Douglas (--

C Omental bursa ~

--D) Paracolic gutter

rEj Between -he panttal perimltum and the posterior body wal1

At birth an infant presents with a st()ma~ rb~tbas~njJled jfltotb~diaplfagru 1A1ltre is the defect thatresulied iiitJle heini~t()n shy~tsophagealbiatus

7 - rH-- Hiatus for the inferior vena cava

( Pleuroperitoneal membrane -(0) Septum transvcrsum

(E) Right Crlt~

An infant born with DOVv7l syndrome presents with bili()u~ vomiting Ahat congenital defect does the infant have

(A) Pyloric stenosis

(B) Meckel diverticulum C) Ornphaloce1e

(D) Gastroschisis

( ~ ) Duodenal atresia y A patient with cirrhosis of the liver presents with ~ bacalvaricestnlreased retrograde pressure in which veins caused the varices

(A) Paraumuilical

(B) Splenic

(ct AzygltJus

(15))G~trk ( (-F) Superior mesemeric

A htaltby 3-year~old male patient experiences a hernial sa protruding from the anterior abdominal wall about halfway between me anterior superior ilia spine and the pubk tuberde Pulsations of al1 artery are palpated medial to the protrusion site through the abdominal walL Which layer of the anterior abdominal wall will first be traversed by the

1hctma

fA) Rectus sheath (B) External oblique aponeurosis

(C) Inguinal ligament

lD) Transversalis fusda

(E) Cremasteric fa~cia

After 5urgi(aj ffpair of a hernia the patient tXperienccs mtmlgtness in the skin on the anteshyrior aspect of the S(Totum_ Vhaf nerve may have been lesioned during thehemiorrhaphy

(A) Femoral

(B) Obturator

(C) Ilioinguinal

(D) lliohypogastrk

(E) Pudendal

A 23~year-LJld female secretary il1 good health ~-uddcn1) doubles over with pain in the a ea of the 1JmbRicu$ Sbe feels vartn and ltneasy and has no appetite That night the pain seems to have mQved to the tower right abdominal regjol1 and she calls her family doctor who then arranges for an ambulance to pk-k her up and take her to the hospitaL Wh ell ntn~ perceived in the area of the urnbilirus most Hkely carried lhe pairfu I sensations into the eNS

tA) Vagus nerves I~

V B)

) Lessersplanchnk nerves

tC) Pudendal nerves

(D) lIiohpogastrk nerves

(E) Greater splam ic l erves

A CT reveals carcinoma in the bOod of the ancreas Vhich blood vessel trut ourses ----~- - -bull ------ --shy

immediately poftterior to the body ofthe pancreas is the m~t likely to be oompressed

(A) Splenk artery

(B) Abdominal aorta (C) Portal vein

(1) Splenic vein

(E) Renal vein

A patient has a penrln1l1ng uker of the posterior wall ot the br~l part ot the (lUooenmn llkh blood vessel is subject to erosion

(A) Common hepatic artery

(B) Gastroouodenal artery

(C) Proper hevatic artery

(D) Celiac artery

(E) Anterior inferior 11amrelltlcoduodcnal attery

Your patient has been diagnosed -ith a carcinoma locallted to the head and l~e(k of the pancreas Another clinical sign would be

A esophageal varices

(8) hemorrhoids

C) a caput medusa

(D) increased pra Teuro n th~ hepatic veins

(E) enlarged right supra lavkular lymph nodes

Wltkh of the foUowing structures develops in the ventral mesentery

(A) Spleen

(B) Jeiunum (C) Head of1ht pancreas (D) Transverse colon (E) Stomach

ti l Uw ~ littwin~ f( S-t lil oai Imdge ~ hi(h or tbt la~)d J truetur tgt liJ llntn nl) he hl p UC iJd [IIi ell

c o

A) drains Ie tht infCrior a La aI

R t middot~nfl0 ~ill to th~ lunlgtn of h i dtlndCrlllfH

(e) m t bull JiJattd on tl l J n T ~H

D ) sup Lc O VSlt I Hlid bhtu l 1 li - -I un oid

( ) U~tpli(t tr j middottUh~ 1 v(( b~nt rfK n1ilc~Zm

ANSWERS AND EXPLANATIONS

Answer E The spleen is t hlttnopodicand lymph organ demlted from mesoderm

Answ~ R Al1 tlmphalocele is caused by it failure of the nlidgut to return to the ahdomir nat cavity after herniation into the umbiliau Stalk Choices Aand D maybe seen in infants with Down syndrome choice D ~s the specific CBuse ofduudcnal JtiCSitt Choice C is (ile cause of gclstrosbisis and Choice B nsults iu a Meurolktldivertku1-tlB

Answer B The fundus ofthe stomach is suppHed by soort gastric brunches of the splenic altery The splenic artery supplies the body and tail of the pancreas part of the greater curvature of the sttmla(h and the spleen Te jejunum part of the head of the pancreas and tht~ duodenum distal to the entrance of the commOll bile duct are supplied by the superior mesenterk artery clll~l ~be less r ctlt1ature cmd the pylQric antrum are supplied by the right and lei gastric art(ries

AnSWftt C Tbeomental bursa or lesser ~ritoneaj sac lies direcdy posterior to the proxshyimal part of the duodeTtlm and the stomach and would be the first site where stomach contents ~Ott1d be fpoundluncL

Answer C A defect in a llleuropcritoneal membrane (uswlly the left) is the typical site of i1 cc-ngenitlI diilphragluatic hemia llere the membr4ne fails to dose ()pound( of the perishycCirdiopcritulleal canals

Answer E DuoJenal atresia and aganglionic megacoion are congwitaI defects S~Il in patients with Dowmiddotnsyndrome

Answer D RulaTgemt~llt of and retrograde flow in g~lstrk vel_ns in particlJl~r the kft gas~ tricveins dilates the capillary bed in rhe wall of the esophagus in (ases of porta yper~

tension Blood flow would increase in and dilampte tribntarkgts of the (lZygOUS vein on the other side of the capiUary bed but flow in this vein is in the typical direction t()ward the superior vena cava Paraumbiii(ltU vein eilgorgement contributes to a caput medusH Splenic ~nlargement might prc~nt with 5plcnonlegaly and balt-kflow in to tlu superior m~~ntclic vein occurs but is asymptomatic

Answer D The patient hagt an indirect inguinal hernia whi~h emerges from the antt-rior abdominal wall through the deep inguinltilling Theeep ring is a fault in the transv~rshysaUs fascia this I~yer wiIJ be penetrated first by the hernia

An~Wer C The ilioinguinal nenc which provides sens~llion to the lnedlal thigh ltmclanteshytior SClotunl pass~lt th rough the 5uperfh_ial inguinal ring ind $subject to inj i1T) becaus-e

it is in the operatitm Held of the erniorrhapny

Auswer B The leMHr splanchnic nerves are sympathdic nerVlts that carry viscera l sensashytlltgtrogt ftom illtllt1m~d ()J stietched gust (itinteitinal ~tructures (in this case the pprndix) into tnt eNS Lesser splanchnic ntTYcsarisc from thmiddot T9--T12 spinal cord segments lt1nd provide sympathetic innenation tD rnidgut siruc1ures whiCh include CLe app~JldD Viscera] Pain arising from affecLed Inidgut ampt 1C1ure is referred over the same dl- matorne~ of spinal segrnertts v-hich provide the sympathetic Innervation n this G1SC of appendicitis the invohen~n t of the ltire) of t e unlhHku indud s the T 10 dermatome

Answer B Of the five choices onty the dscending olon is retroperiton~al aldwould be a lik ~ ( choice to be seen immediately a(~jilcent to t11e posterior abdominal middotn~L

Amwen D The SpltftlC ~-ein ourses posterior to the body of the panneas m its way tt drain into the superior mCSfttltlri( vein

Answcr B TILt glstrodllolticnal artery 1 direct hIamh of the comrootl hepatic artery courses immediately pt))iwri() to the duodenum and is slbject to erosion

Answer B Carcinoma of th pan middott3S in the 1tilt1 may compreampgt the portltil vein at irs orishygill The poTtai vcin is fomled when the splenic vein jQiaswith tfie superior meStllt eric vein The inferiot mesenteric vein joins the ~plenjc vein just priOT to tlli~ point at which the splenic joins the superior Jlleit1ltcri( vein Increescd venous presslu in the inferior mesenteric vein is a cause of emo hoid~

Answer C The- velltral pancreas wilich forms most of the head of the p ~ncr as develops in the ventral mes(ntery as antutgrowth of the hepatic diverticulum Th~ hepatic divershyticulull induding the biIJary appa~atus develops in tbe ventral mesentery of the foregut

Answer~ A The superior mesenteric ~in joins with the spienkvein to form the hepatic portal vciu

Answer D The structure at gttlK is the proper hepatic artery~ whkh suppUesoxygenated b middotood to the liver

MAKE SURE YOU KNOW the diff bw Rectus Sheath above and below the arcuate line

ABOVE

Aponeurosis of xiiltmal obllque musclo

Extemll f)biquw musde

Reotln ilbdomlnls musole S~in

Internal 9bliquQ mY~QI

AponeUfOsi$ of hJH$V~~S Lir9a a lb lbdolTlin~ musolo Tri OJV6 rUi

atldomlnis mUS(loe

Sub cutanlilous tiue (tatty ye r)

BElOW

A POrl lJfosis 01 etemal oblique muscle

Aponeul~)sis 01 Internal oblique mU$cl~

Anteriol lay~ of r~ltdus st~ath EXttom1 oblique rnu$cll

Rectus Jbdominis muscle Intoernal Aponeurc-sis of tra~fersU$ oblique muscle-

at-domlnis muscentl ~ Skio

Tra nsvitSus abdomioLs ml)ZClt

TralSVersaHs fascia Medial umQil iegtt1 1i9Jment -and folj

Uldchus Peritoneum (ir median Umbilj~al Suboutane ous

Extraprftone 11ascia

Ymbilimiddot~1 fold)

preu9poundiea1 fascia

tissue (fatty 4nd m~mbr3n(iUS layers)

o Above the arcuate line (A horizontal line 13 of the distance bw the umbilicus and the

pubic symphysis) -10 Aponeurosis divides into an AntPost Laminae

o The Ant Laminae joins EO and Post Laminae joins Trans Abdominis = Ant and Post

RECTUS SHEATH respectively

o BElOW the arcuate line - all 3 aponeurosis join ANTERIOR to rectus muscle to meet its

counterpart in the midline (linea Alba)

o Take away Msg - The abdomen is devoid of a posterior rectus sheath below the

arcuate line and is therefore more vulnerable to herniasinjuries

Question - A physician makes a deep incision in the patients midline immediately superior to

the pubic symphysis which of the following layers is his knife least likely to pass

Rectus Abdominis External Oblique Ant Rectus Sheath Posterior Rectus Sheath All of the

Above

Answer - All of the above None of the other answer choices are midline structures -LINEA

ALBA

Linea Alba has very poor blood supply - doesnt heal well after surgery Therefore this is a

common site for incisional hernias

a Spleen b Transverse colon c Descending colon d Stomach e Pleura

17 Meckels diverticulum is normally found 2 feet proximal from the

a Pyloric sphincter b Lower esophageal sphincter c Ileo-cecal valve d Middle valve of Huston e Anal valve

18 Ulcer in the posterior wall of the first part of the duodenum would erode ___ artery and would cause bleeding

a Left gastric b Right gastric c Hepatic artery proper d Gastroduodenal artery e Middle colic artery

19 An inflamed appendix is identified by a surgeon on the operation table by noting

a The appendicies epiploicae b The convergence of tenia c The artery of Drummond d The mesocolon e The mesosalphinx

20 The nerve which emerges through the psoas major is

a Femoral b Ilio-inguinal c Ilio-hypogastric d Pudendal e Subcostal

21 The right gonadal vein drains into the

a Azygos b Hemiazygos c Inferior Vena Cava d Right renal vein e Left renal vein

22 The hepatocytes in the liver is derived from

a Ectoderm b Endoderm c Mesoderm

d Neural ectoderm

23 Abscess in the lumbar vertebrae due to tuberculosis would spread to the adjacent muscle which is

a Psoas Major b Iliacus c Quadratus lumborum d Tranversus Abdominis

24 The anterior wall of the inguinal canal is formed by

a External oblique and transverses abdominis b External oblique and fascia transversalis c Internal oblique and external oblique d Internal oblique and transverses abdominis e Fascia transversalis and peritoneum

Meckels diverticulum is a result of which of the following developmental abnormalities shy

A Failure of the vitelline duct to close

B Failure of the herniated intestinal loop to retract into the abdomen

C Failure of the urachus to close

D Failure of the midgut to rotate

E Failure of the hepatic duct to close

Explanation

Meckels diverticulum is a result of the persistence of the proximal part of the vitelline duct This

diverticulum is usually found about 2 feet proximal to the ileocecal junction and is usually about 2 inches

long It is present in about 2 of the popUlation It may be the site of ectopic pancreatic tissue or gastric

mucosa and may develop inflammatory processes and ulcerations Acute Meckels diverticulitis

simulates appendicitis

Which of the following veins carries blood from the esophagus to the portal vein The

A right gastric vein

B left gastric vein c splenic vein D azygos vein

E left gastroepiploic vein

Explanation

The left gastric vein a direct branch of the portal vein drains blood from the lesser curvature of the

stomach and the inferior portion of the esophagus Because branches of the portal vein do not have

valves blood can flow in a retrograde path when there is an obstruction to flow through the portal system or liveL Rlooci Cln then flow from the nortl] vein thr()1Ph the left PRstric vein to the esonhlPlIS lno

through venous communications within the submucosa of the esophagus to esophageal veins that drain

into the azygos vein The increase in blood flow through the esophageal submucosal veins results in esophageal varices

On the posterior wall of the abdomen the celiac ganglion A contains cell bodies of postganglionic parasympathetic neurons B is synapsed upon by neurons in the posterior vagal trunk C is synapsed upon by neurons in the greater splanchnic nerve D contains sensory cell bodies of lumbar spinal nerves E contains cell bodies of neurons that cause an increase in the rate of peristasis

Explanation The celiac ganglion is one of the preaortic ganglia of the sympathetic nervous system It contains cell bodies of postganglionic sympathetic neurons The sympathetic splanchnic nerves contain preganglionic sympathetic neurons that pass through the sympathetic chain without synapsing These splanchnic nerves go to the preaortic ganglia to synapse The greater splanchnic nerve contains preganglionic neurons from spinal cord segments T5-T9 This nerve synapses in the celiac ganglion The nerve fibers in the vagal trunks are preganglionic parasympathetic fibers that go to the walls of the organs that they will innervate and synapse on postganglionic parasympathetic neurons in the walls of those organs Cell bodies of sensory neurons in the abdomen are found in the dorsal root ganglia or the sensory ganglia of the vagus nerve Sympathetic innervation decreases the rate of peristalsis parasympathetic innervation increases the rate of peristalsis

Which of the following pairs of arteries will allow blood to bypass an occlusion of the celiac trunk

A Left gastric artery-right gastric artery

B Left gastroepiploic artery-right gastroepiploic artery

C Superior pancreaticoduodenal artery-inferior pancreaticoduodenal artery

D Splenic artery-common hepatic artery

E Left gastric artery - proper hepatic artery

Explanation The anastoOlosis of a branch of the celiac trunk and a branch of the superior mesenteric artery will

provide collateral circulation around an occlusion of the celiac trunk Each of the other choices pair

branches of the celiac trunk therefore these will not provide collateral flow around the obstruction of the

celiac trunk The left gastric splenic and common hepatic arteries are direct branches of the celiac trunk

The right gastric artery is a branch of the proper hepatic artery which is a branch of the common hepatic artery The left gastroepiploic artery is a branch of the splenic artery The right gastroepiploic artery is a

branch of the gastroduodenal artery whlch is a branch of the common hepatic artery

Which of the following organs has appendices epiploica The

A sigmoid colon

Bjejunum

C duodenum

D stomach E esophagus

Explanation Appendices epiploica are characteristic of the colon Appendices epiploica are subserosal accumulations

of fat None of the organs of the gastrointestinal tract has appendices epiploica except the colon

Page 35: Chirag's Abdomen Review

Peritoneum - similar concept to Pleura - think of a fist in a balloon

Visceral Peritoneum - Layer of balloon touching your fist

Parietal Peritoneum - Layer of balloon not touching your fist

Your fist represents the organ your wrist is the hilum and your arm contains the blood supply

entering the organ

Appreciate that there will never be organs in the peritoneal cavity - rather these organs invaginate

the cavity Kaplan videos

RULES OF NOMENCLATUREshy

1 Organ completely surrounded by peritoneum - peritoneal organ

2 Organ partially surrounded by peritoneum- Retroperitoneal

3 Peritoneum surrounding peritoneal organ is VISCERAL peritoneum

4 Peritoneum surrounding retroperitoneal organ is PARIETAL peritoneum

5 Peritoneum connecting visceral to parietal is called messentary 2 messentaries in the

gut Dorsal (to the gut tube) and ventral (to the gut tube) messentary

Aorta is in Retro peritoneal position - but blood must reach peritoneal position - vessels travel through

messentary All peritoneal organs will have blood supply reaching through messentary

-Mesentery is a 2 layer peritoneum with a neurovascular communication between body wall and organ

- Ligament connects one organ with another or to the abdominal wall (Ommentum = ligament)

lesser Ommentum (attach lesser curvature of stomach and duodenum to liver) =Hepatoduodenal

Ligament and Hepatogastric Ligament

Has a Superior and Inferior Recess (Accumulation of Fluid in Ascites)

Communicates with the greater sac through the epiplic foramen (what structures pass through

this foramen)

Boundaries - you must be able to visualize this

o Anterior - stomach

o Posterior - parietal peritoneum pancreas

o Superior - superior recess (bw diaphragm and coronary ligament)

o Inferior -Inferior recess (bw layers or greater momentum

Greater Ommentum (attach greater curvature of stomach) Gastrophrenic ligament Gastrosplenic

ligament gastrocolic ligament

The greater omentum is the largest peritoneal fold It consists of a double sheet of peritoneum folded on itself so that it is made up of four layers The two layers which descend from the greater curvature of the stomach and commencement of the duodenum pass in front of the small intestines sometimes as low down as the pelvis they then turn upon themselves and ascend again as far as the transverse colon where they separate and enclose that part of the intestine

ABDOMINAL PAIN

Parietal Peritoneum - supplied by same vasculature lymphatics and nerves supplying body wall it

lines and diaphragm Sensitive to pain pressure heat cold well localized

Visceral Peritoneum - supplied by same vasculature lymphatics and somatic nerve of organ it covers

Insensitive to touch heat cold and laceration - referred to dermatome of spinal ganglia providing

sensory fibers Where does appendicitis refer to

Foregut pain - epigastric area (ie - cholycystitis)

Midgut pain - periumbilical area (ie - appendicitis)

Hindgut Pain - suprapubic area (ie - diverticulitis)

Extra ImagesConcepts

ll~_____-

FalifCtrm ligament oind r~ud ligamet f Ilver

Blood from splenio gastriC and inferiof rne$e-rteri v~ins

Ca-I tributaries

Lett gastrio Ifein

Posterior superior pan~reatioodul)denal vaihS

Lott gamo-om~nlal (9aropip lomiddotic) -in

Poq_~ tjol imerl-9-r panCJertlcorllJod-nal veiopound --amp----I- - ~J Right grtr~-omntal

Anwrior interi (gartroepiploic) Jjn

pan euaii cod vl)denal veins middot Inf~Ji (t r mesentric vein

Miqdle (olic vein

Right cl)licvein Sigmoid and rectosigml)id (ei ns

IhH)Collc(~io

--- Mi~dl laquooLJl gtjrltgt

PoM ca vl1 illasto)moses -----shyampoptoageal 2 Paraumbilie-lt11 Inferi or Fectal vei ns

3 Recial 4 REuoperHonea1

Know how the Portal vein is formed I 4 sites of portal caval anastamoses and 1 clinical shunt

Col li t ltt-~ otTl~tI ~nj pc~ 1lt1 turJoG

Ltf 14i1 tImiddot~ artoftl9 on tj phtAt$

L-oftqf 4t t~r 1=laquoIran d 1 bull shy~p l ci rj o fOOOts

Nerves follow the arteries - appreciate the splanchnic nervous system I

Uet~ric branch of left ~nal art

Ureterie branch of righi renal artelY

Left Zld lumbar in and co mlTlunication to as)erdin9 lumbar l(~in Hi ~ht tEZ1~~t~ t3r j t itn ~ nJ l1t- rlnd lfe i r1

Inferior me5nteri~ artery

Notice that the right testicular vein drains directly into the IVC and the right testicular artery drains

directly into the aorta However the left testicular vein drains into the L renal vein at a right angleshy

reason left testicle is lower and more susceptible to varicocele (bag of worms)

Also notice that the left renal vein has a longer course because the IVC is on the right side whereas

the right renal artery has a longer course because the aorta is on the left side

Appreciate the anterior to posterior relationship of structures in the hilum of the kidney - VAP - Vein

Artery Renal Pelvis (Ureter)

11____ __ L_ L_ n VJ __ _ _ t_L I I_ _ L __ L_ I -pound1 bull LI_~-I ____

Posterior View of Head of Pancreas in ( of Duodenum

Celiao hunk

Co mmon ~L~jJth art~ry

GastNduQdonal artrf (partilly in phantn)

P1)Sterior $Up~Jior panCflaticuduodfmal art~r~t

(Co mm on) bile duct

middot~1t~~t-1l---~-~- Right gshomiddotomental (gastoe plp lolc) 3rte (phantomost)

Grener paocre atic art-ry

1n1~rjor pancr-iatlc artery

Jtrifll supejo r pal)oreailcento)dJodenal artr1 (phantom)

Anastomotlo branch

POostetlor bJanch of jo f~ri of pan-reatir(lduodensl drttnj

Anterio r branch of i flferior palcreati~)duodenal art~(phan1om)

Notice the extensive blood supply to the pancreas and duodenum via the branches of the celiac trunk

Notice collateral supply from SMA branches - makes sense bc this is the jxn of foregutmidgut

Identify the vessels in this arteriogram

Hiltid i)f N~ck oi B)dvof Tail 01 pa nereas pan cent~as P-nmiddot-reas panCtCas

I nferie v~na cava

jHept1iic p(lrlai v~in

Port1 tnd H~pti lt a ftH prol

Comm on) bll duct

Ouodtnum

~ft colic (sio)Atta~ hmtrlt jt~xJr-ofha~elSe

muo(IIQn

Right ~lIc (h~j)tic)

il~gtture

In1triol m~oten lIein (rttr op~ritoMdO

SlJp efl or mes~n~fiC amrV and lipln

KNOW YOUR NEIGHBORHOOD

Questions

vVhiJh structure supplied by a bnmdlof the cclia( artery is not derivcd from foregut LemCJUCrITI

(A) Head of the pancte-a5

CD) Pyloric duolenum

Cystkduct

( Liver hepatocyt~~

~F) Body of the spleen

An infant presents with an omrhaJucele at birth -hi oJ the [oHm illg applies to his cM1-dition

(A) It is 31so seen ill p4titnts with aganghonic megacolon

(11) ft reuirs from a fal1ure of resorption of theviteUine d let

(C) It results from herniation at the-site of regression of the right umbilk vein

DJ It is caustd by faihtrc of recanalization of the midgut part of the duodenum

~ It ill camioo by a failuIt vf the midgul to return to the abGQminal uity after herniashytion in-n the urnbilk s l stalk

Ot er than the spleen occlusion Cif the spit-Ilk artery at its odgin wm most likely affect die blood supply to jllch st cnud

(A) Jejunum

(B) Body of th pal1~lltas

(C) LeSStT Cllmiddotlaturc of tl )toma-ch

(D Duodenum dista to the entrance of the Ornmou bile duct

E Fundus of the stomach

A 38-yeu-old batL~er with a history of heartburn suddenly experiences excluciating pain in the (plgastric region of th~ abdomeu SurgCry is perf~rme immediard y upon admisshysion to the 1IlcrgCJliy tuomh~re i~ evidence uf a ruptured ulcer in the posterior waU of the stomach Vhere will a surgeon first fi nd the stomach contenlSf

A) Greater p4ritoneal sac

rB) Cul~de-s~c of Douglas (--

C Omental bursa ~

--D) Paracolic gutter

rEj Between -he panttal perimltum and the posterior body wal1

At birth an infant presents with a st()ma~ rb~tbas~njJled jfltotb~diaplfagru 1A1ltre is the defect thatresulied iiitJle heini~t()n shy~tsophagealbiatus

7 - rH-- Hiatus for the inferior vena cava

( Pleuroperitoneal membrane -(0) Septum transvcrsum

(E) Right Crlt~

An infant born with DOVv7l syndrome presents with bili()u~ vomiting Ahat congenital defect does the infant have

(A) Pyloric stenosis

(B) Meckel diverticulum C) Ornphaloce1e

(D) Gastroschisis

( ~ ) Duodenal atresia y A patient with cirrhosis of the liver presents with ~ bacalvaricestnlreased retrograde pressure in which veins caused the varices

(A) Paraumuilical

(B) Splenic

(ct AzygltJus

(15))G~trk ( (-F) Superior mesemeric

A htaltby 3-year~old male patient experiences a hernial sa protruding from the anterior abdominal wall about halfway between me anterior superior ilia spine and the pubk tuberde Pulsations of al1 artery are palpated medial to the protrusion site through the abdominal walL Which layer of the anterior abdominal wall will first be traversed by the

1hctma

fA) Rectus sheath (B) External oblique aponeurosis

(C) Inguinal ligament

lD) Transversalis fusda

(E) Cremasteric fa~cia

After 5urgi(aj ffpair of a hernia the patient tXperienccs mtmlgtness in the skin on the anteshyrior aspect of the S(Totum_ Vhaf nerve may have been lesioned during thehemiorrhaphy

(A) Femoral

(B) Obturator

(C) Ilioinguinal

(D) lliohypogastrk

(E) Pudendal

A 23~year-LJld female secretary il1 good health ~-uddcn1) doubles over with pain in the a ea of the 1JmbRicu$ Sbe feels vartn and ltneasy and has no appetite That night the pain seems to have mQved to the tower right abdominal regjol1 and she calls her family doctor who then arranges for an ambulance to pk-k her up and take her to the hospitaL Wh ell ntn~ perceived in the area of the urnbilirus most Hkely carried lhe pairfu I sensations into the eNS

tA) Vagus nerves I~

V B)

) Lessersplanchnk nerves

tC) Pudendal nerves

(D) lIiohpogastrk nerves

(E) Greater splam ic l erves

A CT reveals carcinoma in the bOod of the ancreas Vhich blood vessel trut ourses ----~- - -bull ------ --shy

immediately poftterior to the body ofthe pancreas is the m~t likely to be oompressed

(A) Splenk artery

(B) Abdominal aorta (C) Portal vein

(1) Splenic vein

(E) Renal vein

A patient has a penrln1l1ng uker of the posterior wall ot the br~l part ot the (lUooenmn llkh blood vessel is subject to erosion

(A) Common hepatic artery

(B) Gastroouodenal artery

(C) Proper hevatic artery

(D) Celiac artery

(E) Anterior inferior 11amrelltlcoduodcnal attery

Your patient has been diagnosed -ith a carcinoma locallted to the head and l~e(k of the pancreas Another clinical sign would be

A esophageal varices

(8) hemorrhoids

C) a caput medusa

(D) increased pra Teuro n th~ hepatic veins

(E) enlarged right supra lavkular lymph nodes

Wltkh of the foUowing structures develops in the ventral mesentery

(A) Spleen

(B) Jeiunum (C) Head of1ht pancreas (D) Transverse colon (E) Stomach

ti l Uw ~ littwin~ f( S-t lil oai Imdge ~ hi(h or tbt la~)d J truetur tgt liJ llntn nl) he hl p UC iJd [IIi ell

c o

A) drains Ie tht infCrior a La aI

R t middot~nfl0 ~ill to th~ lunlgtn of h i dtlndCrlllfH

(e) m t bull JiJattd on tl l J n T ~H

D ) sup Lc O VSlt I Hlid bhtu l 1 li - -I un oid

( ) U~tpli(t tr j middottUh~ 1 v(( b~nt rfK n1ilc~Zm

ANSWERS AND EXPLANATIONS

Answer E The spleen is t hlttnopodicand lymph organ demlted from mesoderm

Answ~ R Al1 tlmphalocele is caused by it failure of the nlidgut to return to the ahdomir nat cavity after herniation into the umbiliau Stalk Choices Aand D maybe seen in infants with Down syndrome choice D ~s the specific CBuse ofduudcnal JtiCSitt Choice C is (ile cause of gclstrosbisis and Choice B nsults iu a Meurolktldivertku1-tlB

Answer B The fundus ofthe stomach is suppHed by soort gastric brunches of the splenic altery The splenic artery supplies the body and tail of the pancreas part of the greater curvature of the sttmla(h and the spleen Te jejunum part of the head of the pancreas and tht~ duodenum distal to the entrance of the commOll bile duct are supplied by the superior mesenterk artery clll~l ~be less r ctlt1ature cmd the pylQric antrum are supplied by the right and lei gastric art(ries

AnSWftt C Tbeomental bursa or lesser ~ritoneaj sac lies direcdy posterior to the proxshyimal part of the duodeTtlm and the stomach and would be the first site where stomach contents ~Ott1d be fpoundluncL

Answer C A defect in a llleuropcritoneal membrane (uswlly the left) is the typical site of i1 cc-ngenitlI diilphragluatic hemia llere the membr4ne fails to dose ()pound( of the perishycCirdiopcritulleal canals

Answer E DuoJenal atresia and aganglionic megacoion are congwitaI defects S~Il in patients with Dowmiddotnsyndrome

Answer D RulaTgemt~llt of and retrograde flow in g~lstrk vel_ns in particlJl~r the kft gas~ tricveins dilates the capillary bed in rhe wall of the esophagus in (ases of porta yper~

tension Blood flow would increase in and dilampte tribntarkgts of the (lZygOUS vein on the other side of the capiUary bed but flow in this vein is in the typical direction t()ward the superior vena cava Paraumbiii(ltU vein eilgorgement contributes to a caput medusH Splenic ~nlargement might prc~nt with 5plcnonlegaly and balt-kflow in to tlu superior m~~ntclic vein occurs but is asymptomatic

Answer D The patient hagt an indirect inguinal hernia whi~h emerges from the antt-rior abdominal wall through the deep inguinltilling Theeep ring is a fault in the transv~rshysaUs fascia this I~yer wiIJ be penetrated first by the hernia

An~Wer C The ilioinguinal nenc which provides sens~llion to the lnedlal thigh ltmclanteshytior SClotunl pass~lt th rough the 5uperfh_ial inguinal ring ind $subject to inj i1T) becaus-e

it is in the operatitm Held of the erniorrhapny

Auswer B The leMHr splanchnic nerves are sympathdic nerVlts that carry viscera l sensashytlltgtrogt ftom illtllt1m~d ()J stietched gust (itinteitinal ~tructures (in this case the pprndix) into tnt eNS Lesser splanchnic ntTYcsarisc from thmiddot T9--T12 spinal cord segments lt1nd provide sympathetic innenation tD rnidgut siruc1ures whiCh include CLe app~JldD Viscera] Pain arising from affecLed Inidgut ampt 1C1ure is referred over the same dl- matorne~ of spinal segrnertts v-hich provide the sympathetic Innervation n this G1SC of appendicitis the invohen~n t of the ltire) of t e unlhHku indud s the T 10 dermatome

Answer B Of the five choices onty the dscending olon is retroperiton~al aldwould be a lik ~ ( choice to be seen immediately a(~jilcent to t11e posterior abdominal middotn~L

Amwen D The SpltftlC ~-ein ourses posterior to the body of the panneas m its way tt drain into the superior mCSfttltlri( vein

Answcr B TILt glstrodllolticnal artery 1 direct hIamh of the comrootl hepatic artery courses immediately pt))iwri() to the duodenum and is slbject to erosion

Answer B Carcinoma of th pan middott3S in the 1tilt1 may compreampgt the portltil vein at irs orishygill The poTtai vcin is fomled when the splenic vein jQiaswith tfie superior meStllt eric vein The inferiot mesenteric vein joins the ~plenjc vein just priOT to tlli~ point at which the splenic joins the superior Jlleit1ltcri( vein Increescd venous presslu in the inferior mesenteric vein is a cause of emo hoid~

Answer C The- velltral pancreas wilich forms most of the head of the p ~ncr as develops in the ventral mes(ntery as antutgrowth of the hepatic diverticulum Th~ hepatic divershyticulull induding the biIJary appa~atus develops in tbe ventral mesentery of the foregut

Answer~ A The superior mesenteric ~in joins with the spienkvein to form the hepatic portal vciu

Answer D The structure at gttlK is the proper hepatic artery~ whkh suppUesoxygenated b middotood to the liver

MAKE SURE YOU KNOW the diff bw Rectus Sheath above and below the arcuate line

ABOVE

Aponeurosis of xiiltmal obllque musclo

Extemll f)biquw musde

Reotln ilbdomlnls musole S~in

Internal 9bliquQ mY~QI

AponeUfOsi$ of hJH$V~~S Lir9a a lb lbdolTlin~ musolo Tri OJV6 rUi

atldomlnis mUS(loe

Sub cutanlilous tiue (tatty ye r)

BElOW

A POrl lJfosis 01 etemal oblique muscle

Aponeul~)sis 01 Internal oblique mU$cl~

Anteriol lay~ of r~ltdus st~ath EXttom1 oblique rnu$cll

Rectus Jbdominis muscle Intoernal Aponeurc-sis of tra~fersU$ oblique muscle-

at-domlnis muscentl ~ Skio

Tra nsvitSus abdomioLs ml)ZClt

TralSVersaHs fascia Medial umQil iegtt1 1i9Jment -and folj

Uldchus Peritoneum (ir median Umbilj~al Suboutane ous

Extraprftone 11ascia

Ymbilimiddot~1 fold)

preu9poundiea1 fascia

tissue (fatty 4nd m~mbr3n(iUS layers)

o Above the arcuate line (A horizontal line 13 of the distance bw the umbilicus and the

pubic symphysis) -10 Aponeurosis divides into an AntPost Laminae

o The Ant Laminae joins EO and Post Laminae joins Trans Abdominis = Ant and Post

RECTUS SHEATH respectively

o BElOW the arcuate line - all 3 aponeurosis join ANTERIOR to rectus muscle to meet its

counterpart in the midline (linea Alba)

o Take away Msg - The abdomen is devoid of a posterior rectus sheath below the

arcuate line and is therefore more vulnerable to herniasinjuries

Question - A physician makes a deep incision in the patients midline immediately superior to

the pubic symphysis which of the following layers is his knife least likely to pass

Rectus Abdominis External Oblique Ant Rectus Sheath Posterior Rectus Sheath All of the

Above

Answer - All of the above None of the other answer choices are midline structures -LINEA

ALBA

Linea Alba has very poor blood supply - doesnt heal well after surgery Therefore this is a

common site for incisional hernias

a Spleen b Transverse colon c Descending colon d Stomach e Pleura

17 Meckels diverticulum is normally found 2 feet proximal from the

a Pyloric sphincter b Lower esophageal sphincter c Ileo-cecal valve d Middle valve of Huston e Anal valve

18 Ulcer in the posterior wall of the first part of the duodenum would erode ___ artery and would cause bleeding

a Left gastric b Right gastric c Hepatic artery proper d Gastroduodenal artery e Middle colic artery

19 An inflamed appendix is identified by a surgeon on the operation table by noting

a The appendicies epiploicae b The convergence of tenia c The artery of Drummond d The mesocolon e The mesosalphinx

20 The nerve which emerges through the psoas major is

a Femoral b Ilio-inguinal c Ilio-hypogastric d Pudendal e Subcostal

21 The right gonadal vein drains into the

a Azygos b Hemiazygos c Inferior Vena Cava d Right renal vein e Left renal vein

22 The hepatocytes in the liver is derived from

a Ectoderm b Endoderm c Mesoderm

d Neural ectoderm

23 Abscess in the lumbar vertebrae due to tuberculosis would spread to the adjacent muscle which is

a Psoas Major b Iliacus c Quadratus lumborum d Tranversus Abdominis

24 The anterior wall of the inguinal canal is formed by

a External oblique and transverses abdominis b External oblique and fascia transversalis c Internal oblique and external oblique d Internal oblique and transverses abdominis e Fascia transversalis and peritoneum

Meckels diverticulum is a result of which of the following developmental abnormalities shy

A Failure of the vitelline duct to close

B Failure of the herniated intestinal loop to retract into the abdomen

C Failure of the urachus to close

D Failure of the midgut to rotate

E Failure of the hepatic duct to close

Explanation

Meckels diverticulum is a result of the persistence of the proximal part of the vitelline duct This

diverticulum is usually found about 2 feet proximal to the ileocecal junction and is usually about 2 inches

long It is present in about 2 of the popUlation It may be the site of ectopic pancreatic tissue or gastric

mucosa and may develop inflammatory processes and ulcerations Acute Meckels diverticulitis

simulates appendicitis

Which of the following veins carries blood from the esophagus to the portal vein The

A right gastric vein

B left gastric vein c splenic vein D azygos vein

E left gastroepiploic vein

Explanation

The left gastric vein a direct branch of the portal vein drains blood from the lesser curvature of the

stomach and the inferior portion of the esophagus Because branches of the portal vein do not have

valves blood can flow in a retrograde path when there is an obstruction to flow through the portal system or liveL Rlooci Cln then flow from the nortl] vein thr()1Ph the left PRstric vein to the esonhlPlIS lno

through venous communications within the submucosa of the esophagus to esophageal veins that drain

into the azygos vein The increase in blood flow through the esophageal submucosal veins results in esophageal varices

On the posterior wall of the abdomen the celiac ganglion A contains cell bodies of postganglionic parasympathetic neurons B is synapsed upon by neurons in the posterior vagal trunk C is synapsed upon by neurons in the greater splanchnic nerve D contains sensory cell bodies of lumbar spinal nerves E contains cell bodies of neurons that cause an increase in the rate of peristasis

Explanation The celiac ganglion is one of the preaortic ganglia of the sympathetic nervous system It contains cell bodies of postganglionic sympathetic neurons The sympathetic splanchnic nerves contain preganglionic sympathetic neurons that pass through the sympathetic chain without synapsing These splanchnic nerves go to the preaortic ganglia to synapse The greater splanchnic nerve contains preganglionic neurons from spinal cord segments T5-T9 This nerve synapses in the celiac ganglion The nerve fibers in the vagal trunks are preganglionic parasympathetic fibers that go to the walls of the organs that they will innervate and synapse on postganglionic parasympathetic neurons in the walls of those organs Cell bodies of sensory neurons in the abdomen are found in the dorsal root ganglia or the sensory ganglia of the vagus nerve Sympathetic innervation decreases the rate of peristalsis parasympathetic innervation increases the rate of peristalsis

Which of the following pairs of arteries will allow blood to bypass an occlusion of the celiac trunk

A Left gastric artery-right gastric artery

B Left gastroepiploic artery-right gastroepiploic artery

C Superior pancreaticoduodenal artery-inferior pancreaticoduodenal artery

D Splenic artery-common hepatic artery

E Left gastric artery - proper hepatic artery

Explanation The anastoOlosis of a branch of the celiac trunk and a branch of the superior mesenteric artery will

provide collateral circulation around an occlusion of the celiac trunk Each of the other choices pair

branches of the celiac trunk therefore these will not provide collateral flow around the obstruction of the

celiac trunk The left gastric splenic and common hepatic arteries are direct branches of the celiac trunk

The right gastric artery is a branch of the proper hepatic artery which is a branch of the common hepatic artery The left gastroepiploic artery is a branch of the splenic artery The right gastroepiploic artery is a

branch of the gastroduodenal artery whlch is a branch of the common hepatic artery

Which of the following organs has appendices epiploica The

A sigmoid colon

Bjejunum

C duodenum

D stomach E esophagus

Explanation Appendices epiploica are characteristic of the colon Appendices epiploica are subserosal accumulations

of fat None of the organs of the gastrointestinal tract has appendices epiploica except the colon

Page 36: Chirag's Abdomen Review

Greater Ommentum (attach greater curvature of stomach) Gastrophrenic ligament Gastrosplenic

ligament gastrocolic ligament

The greater omentum is the largest peritoneal fold It consists of a double sheet of peritoneum folded on itself so that it is made up of four layers The two layers which descend from the greater curvature of the stomach and commencement of the duodenum pass in front of the small intestines sometimes as low down as the pelvis they then turn upon themselves and ascend again as far as the transverse colon where they separate and enclose that part of the intestine

ABDOMINAL PAIN

Parietal Peritoneum - supplied by same vasculature lymphatics and nerves supplying body wall it

lines and diaphragm Sensitive to pain pressure heat cold well localized

Visceral Peritoneum - supplied by same vasculature lymphatics and somatic nerve of organ it covers

Insensitive to touch heat cold and laceration - referred to dermatome of spinal ganglia providing

sensory fibers Where does appendicitis refer to

Foregut pain - epigastric area (ie - cholycystitis)

Midgut pain - periumbilical area (ie - appendicitis)

Hindgut Pain - suprapubic area (ie - diverticulitis)

Extra ImagesConcepts

ll~_____-

FalifCtrm ligament oind r~ud ligamet f Ilver

Blood from splenio gastriC and inferiof rne$e-rteri v~ins

Ca-I tributaries

Lett gastrio Ifein

Posterior superior pan~reatioodul)denal vaihS

Lott gamo-om~nlal (9aropip lomiddotic) -in

Poq_~ tjol imerl-9-r panCJertlcorllJod-nal veiopound --amp----I- - ~J Right grtr~-omntal

Anwrior interi (gartroepiploic) Jjn

pan euaii cod vl)denal veins middot Inf~Ji (t r mesentric vein

Miqdle (olic vein

Right cl)licvein Sigmoid and rectosigml)id (ei ns

IhH)Collc(~io

--- Mi~dl laquooLJl gtjrltgt

PoM ca vl1 illasto)moses -----shyampoptoageal 2 Paraumbilie-lt11 Inferi or Fectal vei ns

3 Recial 4 REuoperHonea1

Know how the Portal vein is formed I 4 sites of portal caval anastamoses and 1 clinical shunt

Col li t ltt-~ otTl~tI ~nj pc~ 1lt1 turJoG

Ltf 14i1 tImiddot~ artoftl9 on tj phtAt$

L-oftqf 4t t~r 1=laquoIran d 1 bull shy~p l ci rj o fOOOts

Nerves follow the arteries - appreciate the splanchnic nervous system I

Uet~ric branch of left ~nal art

Ureterie branch of righi renal artelY

Left Zld lumbar in and co mlTlunication to as)erdin9 lumbar l(~in Hi ~ht tEZ1~~t~ t3r j t itn ~ nJ l1t- rlnd lfe i r1

Inferior me5nteri~ artery

Notice that the right testicular vein drains directly into the IVC and the right testicular artery drains

directly into the aorta However the left testicular vein drains into the L renal vein at a right angleshy

reason left testicle is lower and more susceptible to varicocele (bag of worms)

Also notice that the left renal vein has a longer course because the IVC is on the right side whereas

the right renal artery has a longer course because the aorta is on the left side

Appreciate the anterior to posterior relationship of structures in the hilum of the kidney - VAP - Vein

Artery Renal Pelvis (Ureter)

11____ __ L_ L_ n VJ __ _ _ t_L I I_ _ L __ L_ I -pound1 bull LI_~-I ____

Posterior View of Head of Pancreas in ( of Duodenum

Celiao hunk

Co mmon ~L~jJth art~ry

GastNduQdonal artrf (partilly in phantn)

P1)Sterior $Up~Jior panCflaticuduodfmal art~r~t

(Co mm on) bile duct

middot~1t~~t-1l---~-~- Right gshomiddotomental (gastoe plp lolc) 3rte (phantomost)

Grener paocre atic art-ry

1n1~rjor pancr-iatlc artery

Jtrifll supejo r pal)oreailcento)dJodenal artr1 (phantom)

Anastomotlo branch

POostetlor bJanch of jo f~ri of pan-reatir(lduodensl drttnj

Anterio r branch of i flferior palcreati~)duodenal art~(phan1om)

Notice the extensive blood supply to the pancreas and duodenum via the branches of the celiac trunk

Notice collateral supply from SMA branches - makes sense bc this is the jxn of foregutmidgut

Identify the vessels in this arteriogram

Hiltid i)f N~ck oi B)dvof Tail 01 pa nereas pan cent~as P-nmiddot-reas panCtCas

I nferie v~na cava

jHept1iic p(lrlai v~in

Port1 tnd H~pti lt a ftH prol

Comm on) bll duct

Ouodtnum

~ft colic (sio)Atta~ hmtrlt jt~xJr-ofha~elSe

muo(IIQn

Right ~lIc (h~j)tic)

il~gtture

In1triol m~oten lIein (rttr op~ritoMdO

SlJp efl or mes~n~fiC amrV and lipln

KNOW YOUR NEIGHBORHOOD

Questions

vVhiJh structure supplied by a bnmdlof the cclia( artery is not derivcd from foregut LemCJUCrITI

(A) Head of the pancte-a5

CD) Pyloric duolenum

Cystkduct

( Liver hepatocyt~~

~F) Body of the spleen

An infant presents with an omrhaJucele at birth -hi oJ the [oHm illg applies to his cM1-dition

(A) It is 31so seen ill p4titnts with aganghonic megacolon

(11) ft reuirs from a fal1ure of resorption of theviteUine d let

(C) It results from herniation at the-site of regression of the right umbilk vein

DJ It is caustd by faihtrc of recanalization of the midgut part of the duodenum

~ It ill camioo by a failuIt vf the midgul to return to the abGQminal uity after herniashytion in-n the urnbilk s l stalk

Ot er than the spleen occlusion Cif the spit-Ilk artery at its odgin wm most likely affect die blood supply to jllch st cnud

(A) Jejunum

(B) Body of th pal1~lltas

(C) LeSStT Cllmiddotlaturc of tl )toma-ch

(D Duodenum dista to the entrance of the Ornmou bile duct

E Fundus of the stomach

A 38-yeu-old batL~er with a history of heartburn suddenly experiences excluciating pain in the (plgastric region of th~ abdomeu SurgCry is perf~rme immediard y upon admisshysion to the 1IlcrgCJliy tuomh~re i~ evidence uf a ruptured ulcer in the posterior waU of the stomach Vhere will a surgeon first fi nd the stomach contenlSf

A) Greater p4ritoneal sac

rB) Cul~de-s~c of Douglas (--

C Omental bursa ~

--D) Paracolic gutter

rEj Between -he panttal perimltum and the posterior body wal1

At birth an infant presents with a st()ma~ rb~tbas~njJled jfltotb~diaplfagru 1A1ltre is the defect thatresulied iiitJle heini~t()n shy~tsophagealbiatus

7 - rH-- Hiatus for the inferior vena cava

( Pleuroperitoneal membrane -(0) Septum transvcrsum

(E) Right Crlt~

An infant born with DOVv7l syndrome presents with bili()u~ vomiting Ahat congenital defect does the infant have

(A) Pyloric stenosis

(B) Meckel diverticulum C) Ornphaloce1e

(D) Gastroschisis

( ~ ) Duodenal atresia y A patient with cirrhosis of the liver presents with ~ bacalvaricestnlreased retrograde pressure in which veins caused the varices

(A) Paraumuilical

(B) Splenic

(ct AzygltJus

(15))G~trk ( (-F) Superior mesemeric

A htaltby 3-year~old male patient experiences a hernial sa protruding from the anterior abdominal wall about halfway between me anterior superior ilia spine and the pubk tuberde Pulsations of al1 artery are palpated medial to the protrusion site through the abdominal walL Which layer of the anterior abdominal wall will first be traversed by the

1hctma

fA) Rectus sheath (B) External oblique aponeurosis

(C) Inguinal ligament

lD) Transversalis fusda

(E) Cremasteric fa~cia

After 5urgi(aj ffpair of a hernia the patient tXperienccs mtmlgtness in the skin on the anteshyrior aspect of the S(Totum_ Vhaf nerve may have been lesioned during thehemiorrhaphy

(A) Femoral

(B) Obturator

(C) Ilioinguinal

(D) lliohypogastrk

(E) Pudendal

A 23~year-LJld female secretary il1 good health ~-uddcn1) doubles over with pain in the a ea of the 1JmbRicu$ Sbe feels vartn and ltneasy and has no appetite That night the pain seems to have mQved to the tower right abdominal regjol1 and she calls her family doctor who then arranges for an ambulance to pk-k her up and take her to the hospitaL Wh ell ntn~ perceived in the area of the urnbilirus most Hkely carried lhe pairfu I sensations into the eNS

tA) Vagus nerves I~

V B)

) Lessersplanchnk nerves

tC) Pudendal nerves

(D) lIiohpogastrk nerves

(E) Greater splam ic l erves

A CT reveals carcinoma in the bOod of the ancreas Vhich blood vessel trut ourses ----~- - -bull ------ --shy

immediately poftterior to the body ofthe pancreas is the m~t likely to be oompressed

(A) Splenk artery

(B) Abdominal aorta (C) Portal vein

(1) Splenic vein

(E) Renal vein

A patient has a penrln1l1ng uker of the posterior wall ot the br~l part ot the (lUooenmn llkh blood vessel is subject to erosion

(A) Common hepatic artery

(B) Gastroouodenal artery

(C) Proper hevatic artery

(D) Celiac artery

(E) Anterior inferior 11amrelltlcoduodcnal attery

Your patient has been diagnosed -ith a carcinoma locallted to the head and l~e(k of the pancreas Another clinical sign would be

A esophageal varices

(8) hemorrhoids

C) a caput medusa

(D) increased pra Teuro n th~ hepatic veins

(E) enlarged right supra lavkular lymph nodes

Wltkh of the foUowing structures develops in the ventral mesentery

(A) Spleen

(B) Jeiunum (C) Head of1ht pancreas (D) Transverse colon (E) Stomach

ti l Uw ~ littwin~ f( S-t lil oai Imdge ~ hi(h or tbt la~)d J truetur tgt liJ llntn nl) he hl p UC iJd [IIi ell

c o

A) drains Ie tht infCrior a La aI

R t middot~nfl0 ~ill to th~ lunlgtn of h i dtlndCrlllfH

(e) m t bull JiJattd on tl l J n T ~H

D ) sup Lc O VSlt I Hlid bhtu l 1 li - -I un oid

( ) U~tpli(t tr j middottUh~ 1 v(( b~nt rfK n1ilc~Zm

ANSWERS AND EXPLANATIONS

Answer E The spleen is t hlttnopodicand lymph organ demlted from mesoderm

Answ~ R Al1 tlmphalocele is caused by it failure of the nlidgut to return to the ahdomir nat cavity after herniation into the umbiliau Stalk Choices Aand D maybe seen in infants with Down syndrome choice D ~s the specific CBuse ofduudcnal JtiCSitt Choice C is (ile cause of gclstrosbisis and Choice B nsults iu a Meurolktldivertku1-tlB

Answer B The fundus ofthe stomach is suppHed by soort gastric brunches of the splenic altery The splenic artery supplies the body and tail of the pancreas part of the greater curvature of the sttmla(h and the spleen Te jejunum part of the head of the pancreas and tht~ duodenum distal to the entrance of the commOll bile duct are supplied by the superior mesenterk artery clll~l ~be less r ctlt1ature cmd the pylQric antrum are supplied by the right and lei gastric art(ries

AnSWftt C Tbeomental bursa or lesser ~ritoneaj sac lies direcdy posterior to the proxshyimal part of the duodeTtlm and the stomach and would be the first site where stomach contents ~Ott1d be fpoundluncL

Answer C A defect in a llleuropcritoneal membrane (uswlly the left) is the typical site of i1 cc-ngenitlI diilphragluatic hemia llere the membr4ne fails to dose ()pound( of the perishycCirdiopcritulleal canals

Answer E DuoJenal atresia and aganglionic megacoion are congwitaI defects S~Il in patients with Dowmiddotnsyndrome

Answer D RulaTgemt~llt of and retrograde flow in g~lstrk vel_ns in particlJl~r the kft gas~ tricveins dilates the capillary bed in rhe wall of the esophagus in (ases of porta yper~

tension Blood flow would increase in and dilampte tribntarkgts of the (lZygOUS vein on the other side of the capiUary bed but flow in this vein is in the typical direction t()ward the superior vena cava Paraumbiii(ltU vein eilgorgement contributes to a caput medusH Splenic ~nlargement might prc~nt with 5plcnonlegaly and balt-kflow in to tlu superior m~~ntclic vein occurs but is asymptomatic

Answer D The patient hagt an indirect inguinal hernia whi~h emerges from the antt-rior abdominal wall through the deep inguinltilling Theeep ring is a fault in the transv~rshysaUs fascia this I~yer wiIJ be penetrated first by the hernia

An~Wer C The ilioinguinal nenc which provides sens~llion to the lnedlal thigh ltmclanteshytior SClotunl pass~lt th rough the 5uperfh_ial inguinal ring ind $subject to inj i1T) becaus-e

it is in the operatitm Held of the erniorrhapny

Auswer B The leMHr splanchnic nerves are sympathdic nerVlts that carry viscera l sensashytlltgtrogt ftom illtllt1m~d ()J stietched gust (itinteitinal ~tructures (in this case the pprndix) into tnt eNS Lesser splanchnic ntTYcsarisc from thmiddot T9--T12 spinal cord segments lt1nd provide sympathetic innenation tD rnidgut siruc1ures whiCh include CLe app~JldD Viscera] Pain arising from affecLed Inidgut ampt 1C1ure is referred over the same dl- matorne~ of spinal segrnertts v-hich provide the sympathetic Innervation n this G1SC of appendicitis the invohen~n t of the ltire) of t e unlhHku indud s the T 10 dermatome

Answer B Of the five choices onty the dscending olon is retroperiton~al aldwould be a lik ~ ( choice to be seen immediately a(~jilcent to t11e posterior abdominal middotn~L

Amwen D The SpltftlC ~-ein ourses posterior to the body of the panneas m its way tt drain into the superior mCSfttltlri( vein

Answcr B TILt glstrodllolticnal artery 1 direct hIamh of the comrootl hepatic artery courses immediately pt))iwri() to the duodenum and is slbject to erosion

Answer B Carcinoma of th pan middott3S in the 1tilt1 may compreampgt the portltil vein at irs orishygill The poTtai vcin is fomled when the splenic vein jQiaswith tfie superior meStllt eric vein The inferiot mesenteric vein joins the ~plenjc vein just priOT to tlli~ point at which the splenic joins the superior Jlleit1ltcri( vein Increescd venous presslu in the inferior mesenteric vein is a cause of emo hoid~

Answer C The- velltral pancreas wilich forms most of the head of the p ~ncr as develops in the ventral mes(ntery as antutgrowth of the hepatic diverticulum Th~ hepatic divershyticulull induding the biIJary appa~atus develops in tbe ventral mesentery of the foregut

Answer~ A The superior mesenteric ~in joins with the spienkvein to form the hepatic portal vciu

Answer D The structure at gttlK is the proper hepatic artery~ whkh suppUesoxygenated b middotood to the liver

MAKE SURE YOU KNOW the diff bw Rectus Sheath above and below the arcuate line

ABOVE

Aponeurosis of xiiltmal obllque musclo

Extemll f)biquw musde

Reotln ilbdomlnls musole S~in

Internal 9bliquQ mY~QI

AponeUfOsi$ of hJH$V~~S Lir9a a lb lbdolTlin~ musolo Tri OJV6 rUi

atldomlnis mUS(loe

Sub cutanlilous tiue (tatty ye r)

BElOW

A POrl lJfosis 01 etemal oblique muscle

Aponeul~)sis 01 Internal oblique mU$cl~

Anteriol lay~ of r~ltdus st~ath EXttom1 oblique rnu$cll

Rectus Jbdominis muscle Intoernal Aponeurc-sis of tra~fersU$ oblique muscle-

at-domlnis muscentl ~ Skio

Tra nsvitSus abdomioLs ml)ZClt

TralSVersaHs fascia Medial umQil iegtt1 1i9Jment -and folj

Uldchus Peritoneum (ir median Umbilj~al Suboutane ous

Extraprftone 11ascia

Ymbilimiddot~1 fold)

preu9poundiea1 fascia

tissue (fatty 4nd m~mbr3n(iUS layers)

o Above the arcuate line (A horizontal line 13 of the distance bw the umbilicus and the

pubic symphysis) -10 Aponeurosis divides into an AntPost Laminae

o The Ant Laminae joins EO and Post Laminae joins Trans Abdominis = Ant and Post

RECTUS SHEATH respectively

o BElOW the arcuate line - all 3 aponeurosis join ANTERIOR to rectus muscle to meet its

counterpart in the midline (linea Alba)

o Take away Msg - The abdomen is devoid of a posterior rectus sheath below the

arcuate line and is therefore more vulnerable to herniasinjuries

Question - A physician makes a deep incision in the patients midline immediately superior to

the pubic symphysis which of the following layers is his knife least likely to pass

Rectus Abdominis External Oblique Ant Rectus Sheath Posterior Rectus Sheath All of the

Above

Answer - All of the above None of the other answer choices are midline structures -LINEA

ALBA

Linea Alba has very poor blood supply - doesnt heal well after surgery Therefore this is a

common site for incisional hernias

a Spleen b Transverse colon c Descending colon d Stomach e Pleura

17 Meckels diverticulum is normally found 2 feet proximal from the

a Pyloric sphincter b Lower esophageal sphincter c Ileo-cecal valve d Middle valve of Huston e Anal valve

18 Ulcer in the posterior wall of the first part of the duodenum would erode ___ artery and would cause bleeding

a Left gastric b Right gastric c Hepatic artery proper d Gastroduodenal artery e Middle colic artery

19 An inflamed appendix is identified by a surgeon on the operation table by noting

a The appendicies epiploicae b The convergence of tenia c The artery of Drummond d The mesocolon e The mesosalphinx

20 The nerve which emerges through the psoas major is

a Femoral b Ilio-inguinal c Ilio-hypogastric d Pudendal e Subcostal

21 The right gonadal vein drains into the

a Azygos b Hemiazygos c Inferior Vena Cava d Right renal vein e Left renal vein

22 The hepatocytes in the liver is derived from

a Ectoderm b Endoderm c Mesoderm

d Neural ectoderm

23 Abscess in the lumbar vertebrae due to tuberculosis would spread to the adjacent muscle which is

a Psoas Major b Iliacus c Quadratus lumborum d Tranversus Abdominis

24 The anterior wall of the inguinal canal is formed by

a External oblique and transverses abdominis b External oblique and fascia transversalis c Internal oblique and external oblique d Internal oblique and transverses abdominis e Fascia transversalis and peritoneum

Meckels diverticulum is a result of which of the following developmental abnormalities shy

A Failure of the vitelline duct to close

B Failure of the herniated intestinal loop to retract into the abdomen

C Failure of the urachus to close

D Failure of the midgut to rotate

E Failure of the hepatic duct to close

Explanation

Meckels diverticulum is a result of the persistence of the proximal part of the vitelline duct This

diverticulum is usually found about 2 feet proximal to the ileocecal junction and is usually about 2 inches

long It is present in about 2 of the popUlation It may be the site of ectopic pancreatic tissue or gastric

mucosa and may develop inflammatory processes and ulcerations Acute Meckels diverticulitis

simulates appendicitis

Which of the following veins carries blood from the esophagus to the portal vein The

A right gastric vein

B left gastric vein c splenic vein D azygos vein

E left gastroepiploic vein

Explanation

The left gastric vein a direct branch of the portal vein drains blood from the lesser curvature of the

stomach and the inferior portion of the esophagus Because branches of the portal vein do not have

valves blood can flow in a retrograde path when there is an obstruction to flow through the portal system or liveL Rlooci Cln then flow from the nortl] vein thr()1Ph the left PRstric vein to the esonhlPlIS lno

through venous communications within the submucosa of the esophagus to esophageal veins that drain

into the azygos vein The increase in blood flow through the esophageal submucosal veins results in esophageal varices

On the posterior wall of the abdomen the celiac ganglion A contains cell bodies of postganglionic parasympathetic neurons B is synapsed upon by neurons in the posterior vagal trunk C is synapsed upon by neurons in the greater splanchnic nerve D contains sensory cell bodies of lumbar spinal nerves E contains cell bodies of neurons that cause an increase in the rate of peristasis

Explanation The celiac ganglion is one of the preaortic ganglia of the sympathetic nervous system It contains cell bodies of postganglionic sympathetic neurons The sympathetic splanchnic nerves contain preganglionic sympathetic neurons that pass through the sympathetic chain without synapsing These splanchnic nerves go to the preaortic ganglia to synapse The greater splanchnic nerve contains preganglionic neurons from spinal cord segments T5-T9 This nerve synapses in the celiac ganglion The nerve fibers in the vagal trunks are preganglionic parasympathetic fibers that go to the walls of the organs that they will innervate and synapse on postganglionic parasympathetic neurons in the walls of those organs Cell bodies of sensory neurons in the abdomen are found in the dorsal root ganglia or the sensory ganglia of the vagus nerve Sympathetic innervation decreases the rate of peristalsis parasympathetic innervation increases the rate of peristalsis

Which of the following pairs of arteries will allow blood to bypass an occlusion of the celiac trunk

A Left gastric artery-right gastric artery

B Left gastroepiploic artery-right gastroepiploic artery

C Superior pancreaticoduodenal artery-inferior pancreaticoduodenal artery

D Splenic artery-common hepatic artery

E Left gastric artery - proper hepatic artery

Explanation The anastoOlosis of a branch of the celiac trunk and a branch of the superior mesenteric artery will

provide collateral circulation around an occlusion of the celiac trunk Each of the other choices pair

branches of the celiac trunk therefore these will not provide collateral flow around the obstruction of the

celiac trunk The left gastric splenic and common hepatic arteries are direct branches of the celiac trunk

The right gastric artery is a branch of the proper hepatic artery which is a branch of the common hepatic artery The left gastroepiploic artery is a branch of the splenic artery The right gastroepiploic artery is a

branch of the gastroduodenal artery whlch is a branch of the common hepatic artery

Which of the following organs has appendices epiploica The

A sigmoid colon

Bjejunum

C duodenum

D stomach E esophagus

Explanation Appendices epiploica are characteristic of the colon Appendices epiploica are subserosal accumulations

of fat None of the organs of the gastrointestinal tract has appendices epiploica except the colon

Page 37: Chirag's Abdomen Review

Extra ImagesConcepts

ll~_____-

FalifCtrm ligament oind r~ud ligamet f Ilver

Blood from splenio gastriC and inferiof rne$e-rteri v~ins

Ca-I tributaries

Lett gastrio Ifein

Posterior superior pan~reatioodul)denal vaihS

Lott gamo-om~nlal (9aropip lomiddotic) -in

Poq_~ tjol imerl-9-r panCJertlcorllJod-nal veiopound --amp----I- - ~J Right grtr~-omntal

Anwrior interi (gartroepiploic) Jjn

pan euaii cod vl)denal veins middot Inf~Ji (t r mesentric vein

Miqdle (olic vein

Right cl)licvein Sigmoid and rectosigml)id (ei ns

IhH)Collc(~io

--- Mi~dl laquooLJl gtjrltgt

PoM ca vl1 illasto)moses -----shyampoptoageal 2 Paraumbilie-lt11 Inferi or Fectal vei ns

3 Recial 4 REuoperHonea1

Know how the Portal vein is formed I 4 sites of portal caval anastamoses and 1 clinical shunt

Col li t ltt-~ otTl~tI ~nj pc~ 1lt1 turJoG

Ltf 14i1 tImiddot~ artoftl9 on tj phtAt$

L-oftqf 4t t~r 1=laquoIran d 1 bull shy~p l ci rj o fOOOts

Nerves follow the arteries - appreciate the splanchnic nervous system I

Uet~ric branch of left ~nal art

Ureterie branch of righi renal artelY

Left Zld lumbar in and co mlTlunication to as)erdin9 lumbar l(~in Hi ~ht tEZ1~~t~ t3r j t itn ~ nJ l1t- rlnd lfe i r1

Inferior me5nteri~ artery

Notice that the right testicular vein drains directly into the IVC and the right testicular artery drains

directly into the aorta However the left testicular vein drains into the L renal vein at a right angleshy

reason left testicle is lower and more susceptible to varicocele (bag of worms)

Also notice that the left renal vein has a longer course because the IVC is on the right side whereas

the right renal artery has a longer course because the aorta is on the left side

Appreciate the anterior to posterior relationship of structures in the hilum of the kidney - VAP - Vein

Artery Renal Pelvis (Ureter)

11____ __ L_ L_ n VJ __ _ _ t_L I I_ _ L __ L_ I -pound1 bull LI_~-I ____

Posterior View of Head of Pancreas in ( of Duodenum

Celiao hunk

Co mmon ~L~jJth art~ry

GastNduQdonal artrf (partilly in phantn)

P1)Sterior $Up~Jior panCflaticuduodfmal art~r~t

(Co mm on) bile duct

middot~1t~~t-1l---~-~- Right gshomiddotomental (gastoe plp lolc) 3rte (phantomost)

Grener paocre atic art-ry

1n1~rjor pancr-iatlc artery

Jtrifll supejo r pal)oreailcento)dJodenal artr1 (phantom)

Anastomotlo branch

POostetlor bJanch of jo f~ri of pan-reatir(lduodensl drttnj

Anterio r branch of i flferior palcreati~)duodenal art~(phan1om)

Notice the extensive blood supply to the pancreas and duodenum via the branches of the celiac trunk

Notice collateral supply from SMA branches - makes sense bc this is the jxn of foregutmidgut

Identify the vessels in this arteriogram

Hiltid i)f N~ck oi B)dvof Tail 01 pa nereas pan cent~as P-nmiddot-reas panCtCas

I nferie v~na cava

jHept1iic p(lrlai v~in

Port1 tnd H~pti lt a ftH prol

Comm on) bll duct

Ouodtnum

~ft colic (sio)Atta~ hmtrlt jt~xJr-ofha~elSe

muo(IIQn

Right ~lIc (h~j)tic)

il~gtture

In1triol m~oten lIein (rttr op~ritoMdO

SlJp efl or mes~n~fiC amrV and lipln

KNOW YOUR NEIGHBORHOOD

Questions

vVhiJh structure supplied by a bnmdlof the cclia( artery is not derivcd from foregut LemCJUCrITI

(A) Head of the pancte-a5

CD) Pyloric duolenum

Cystkduct

( Liver hepatocyt~~

~F) Body of the spleen

An infant presents with an omrhaJucele at birth -hi oJ the [oHm illg applies to his cM1-dition

(A) It is 31so seen ill p4titnts with aganghonic megacolon

(11) ft reuirs from a fal1ure of resorption of theviteUine d let

(C) It results from herniation at the-site of regression of the right umbilk vein

DJ It is caustd by faihtrc of recanalization of the midgut part of the duodenum

~ It ill camioo by a failuIt vf the midgul to return to the abGQminal uity after herniashytion in-n the urnbilk s l stalk

Ot er than the spleen occlusion Cif the spit-Ilk artery at its odgin wm most likely affect die blood supply to jllch st cnud

(A) Jejunum

(B) Body of th pal1~lltas

(C) LeSStT Cllmiddotlaturc of tl )toma-ch

(D Duodenum dista to the entrance of the Ornmou bile duct

E Fundus of the stomach

A 38-yeu-old batL~er with a history of heartburn suddenly experiences excluciating pain in the (plgastric region of th~ abdomeu SurgCry is perf~rme immediard y upon admisshysion to the 1IlcrgCJliy tuomh~re i~ evidence uf a ruptured ulcer in the posterior waU of the stomach Vhere will a surgeon first fi nd the stomach contenlSf

A) Greater p4ritoneal sac

rB) Cul~de-s~c of Douglas (--

C Omental bursa ~

--D) Paracolic gutter

rEj Between -he panttal perimltum and the posterior body wal1

At birth an infant presents with a st()ma~ rb~tbas~njJled jfltotb~diaplfagru 1A1ltre is the defect thatresulied iiitJle heini~t()n shy~tsophagealbiatus

7 - rH-- Hiatus for the inferior vena cava

( Pleuroperitoneal membrane -(0) Septum transvcrsum

(E) Right Crlt~

An infant born with DOVv7l syndrome presents with bili()u~ vomiting Ahat congenital defect does the infant have

(A) Pyloric stenosis

(B) Meckel diverticulum C) Ornphaloce1e

(D) Gastroschisis

( ~ ) Duodenal atresia y A patient with cirrhosis of the liver presents with ~ bacalvaricestnlreased retrograde pressure in which veins caused the varices

(A) Paraumuilical

(B) Splenic

(ct AzygltJus

(15))G~trk ( (-F) Superior mesemeric

A htaltby 3-year~old male patient experiences a hernial sa protruding from the anterior abdominal wall about halfway between me anterior superior ilia spine and the pubk tuberde Pulsations of al1 artery are palpated medial to the protrusion site through the abdominal walL Which layer of the anterior abdominal wall will first be traversed by the

1hctma

fA) Rectus sheath (B) External oblique aponeurosis

(C) Inguinal ligament

lD) Transversalis fusda

(E) Cremasteric fa~cia

After 5urgi(aj ffpair of a hernia the patient tXperienccs mtmlgtness in the skin on the anteshyrior aspect of the S(Totum_ Vhaf nerve may have been lesioned during thehemiorrhaphy

(A) Femoral

(B) Obturator

(C) Ilioinguinal

(D) lliohypogastrk

(E) Pudendal

A 23~year-LJld female secretary il1 good health ~-uddcn1) doubles over with pain in the a ea of the 1JmbRicu$ Sbe feels vartn and ltneasy and has no appetite That night the pain seems to have mQved to the tower right abdominal regjol1 and she calls her family doctor who then arranges for an ambulance to pk-k her up and take her to the hospitaL Wh ell ntn~ perceived in the area of the urnbilirus most Hkely carried lhe pairfu I sensations into the eNS

tA) Vagus nerves I~

V B)

) Lessersplanchnk nerves

tC) Pudendal nerves

(D) lIiohpogastrk nerves

(E) Greater splam ic l erves

A CT reveals carcinoma in the bOod of the ancreas Vhich blood vessel trut ourses ----~- - -bull ------ --shy

immediately poftterior to the body ofthe pancreas is the m~t likely to be oompressed

(A) Splenk artery

(B) Abdominal aorta (C) Portal vein

(1) Splenic vein

(E) Renal vein

A patient has a penrln1l1ng uker of the posterior wall ot the br~l part ot the (lUooenmn llkh blood vessel is subject to erosion

(A) Common hepatic artery

(B) Gastroouodenal artery

(C) Proper hevatic artery

(D) Celiac artery

(E) Anterior inferior 11amrelltlcoduodcnal attery

Your patient has been diagnosed -ith a carcinoma locallted to the head and l~e(k of the pancreas Another clinical sign would be

A esophageal varices

(8) hemorrhoids

C) a caput medusa

(D) increased pra Teuro n th~ hepatic veins

(E) enlarged right supra lavkular lymph nodes

Wltkh of the foUowing structures develops in the ventral mesentery

(A) Spleen

(B) Jeiunum (C) Head of1ht pancreas (D) Transverse colon (E) Stomach

ti l Uw ~ littwin~ f( S-t lil oai Imdge ~ hi(h or tbt la~)d J truetur tgt liJ llntn nl) he hl p UC iJd [IIi ell

c o

A) drains Ie tht infCrior a La aI

R t middot~nfl0 ~ill to th~ lunlgtn of h i dtlndCrlllfH

(e) m t bull JiJattd on tl l J n T ~H

D ) sup Lc O VSlt I Hlid bhtu l 1 li - -I un oid

( ) U~tpli(t tr j middottUh~ 1 v(( b~nt rfK n1ilc~Zm

ANSWERS AND EXPLANATIONS

Answer E The spleen is t hlttnopodicand lymph organ demlted from mesoderm

Answ~ R Al1 tlmphalocele is caused by it failure of the nlidgut to return to the ahdomir nat cavity after herniation into the umbiliau Stalk Choices Aand D maybe seen in infants with Down syndrome choice D ~s the specific CBuse ofduudcnal JtiCSitt Choice C is (ile cause of gclstrosbisis and Choice B nsults iu a Meurolktldivertku1-tlB

Answer B The fundus ofthe stomach is suppHed by soort gastric brunches of the splenic altery The splenic artery supplies the body and tail of the pancreas part of the greater curvature of the sttmla(h and the spleen Te jejunum part of the head of the pancreas and tht~ duodenum distal to the entrance of the commOll bile duct are supplied by the superior mesenterk artery clll~l ~be less r ctlt1ature cmd the pylQric antrum are supplied by the right and lei gastric art(ries

AnSWftt C Tbeomental bursa or lesser ~ritoneaj sac lies direcdy posterior to the proxshyimal part of the duodeTtlm and the stomach and would be the first site where stomach contents ~Ott1d be fpoundluncL

Answer C A defect in a llleuropcritoneal membrane (uswlly the left) is the typical site of i1 cc-ngenitlI diilphragluatic hemia llere the membr4ne fails to dose ()pound( of the perishycCirdiopcritulleal canals

Answer E DuoJenal atresia and aganglionic megacoion are congwitaI defects S~Il in patients with Dowmiddotnsyndrome

Answer D RulaTgemt~llt of and retrograde flow in g~lstrk vel_ns in particlJl~r the kft gas~ tricveins dilates the capillary bed in rhe wall of the esophagus in (ases of porta yper~

tension Blood flow would increase in and dilampte tribntarkgts of the (lZygOUS vein on the other side of the capiUary bed but flow in this vein is in the typical direction t()ward the superior vena cava Paraumbiii(ltU vein eilgorgement contributes to a caput medusH Splenic ~nlargement might prc~nt with 5plcnonlegaly and balt-kflow in to tlu superior m~~ntclic vein occurs but is asymptomatic

Answer D The patient hagt an indirect inguinal hernia whi~h emerges from the antt-rior abdominal wall through the deep inguinltilling Theeep ring is a fault in the transv~rshysaUs fascia this I~yer wiIJ be penetrated first by the hernia

An~Wer C The ilioinguinal nenc which provides sens~llion to the lnedlal thigh ltmclanteshytior SClotunl pass~lt th rough the 5uperfh_ial inguinal ring ind $subject to inj i1T) becaus-e

it is in the operatitm Held of the erniorrhapny

Auswer B The leMHr splanchnic nerves are sympathdic nerVlts that carry viscera l sensashytlltgtrogt ftom illtllt1m~d ()J stietched gust (itinteitinal ~tructures (in this case the pprndix) into tnt eNS Lesser splanchnic ntTYcsarisc from thmiddot T9--T12 spinal cord segments lt1nd provide sympathetic innenation tD rnidgut siruc1ures whiCh include CLe app~JldD Viscera] Pain arising from affecLed Inidgut ampt 1C1ure is referred over the same dl- matorne~ of spinal segrnertts v-hich provide the sympathetic Innervation n this G1SC of appendicitis the invohen~n t of the ltire) of t e unlhHku indud s the T 10 dermatome

Answer B Of the five choices onty the dscending olon is retroperiton~al aldwould be a lik ~ ( choice to be seen immediately a(~jilcent to t11e posterior abdominal middotn~L

Amwen D The SpltftlC ~-ein ourses posterior to the body of the panneas m its way tt drain into the superior mCSfttltlri( vein

Answcr B TILt glstrodllolticnal artery 1 direct hIamh of the comrootl hepatic artery courses immediately pt))iwri() to the duodenum and is slbject to erosion

Answer B Carcinoma of th pan middott3S in the 1tilt1 may compreampgt the portltil vein at irs orishygill The poTtai vcin is fomled when the splenic vein jQiaswith tfie superior meStllt eric vein The inferiot mesenteric vein joins the ~plenjc vein just priOT to tlli~ point at which the splenic joins the superior Jlleit1ltcri( vein Increescd venous presslu in the inferior mesenteric vein is a cause of emo hoid~

Answer C The- velltral pancreas wilich forms most of the head of the p ~ncr as develops in the ventral mes(ntery as antutgrowth of the hepatic diverticulum Th~ hepatic divershyticulull induding the biIJary appa~atus develops in tbe ventral mesentery of the foregut

Answer~ A The superior mesenteric ~in joins with the spienkvein to form the hepatic portal vciu

Answer D The structure at gttlK is the proper hepatic artery~ whkh suppUesoxygenated b middotood to the liver

MAKE SURE YOU KNOW the diff bw Rectus Sheath above and below the arcuate line

ABOVE

Aponeurosis of xiiltmal obllque musclo

Extemll f)biquw musde

Reotln ilbdomlnls musole S~in

Internal 9bliquQ mY~QI

AponeUfOsi$ of hJH$V~~S Lir9a a lb lbdolTlin~ musolo Tri OJV6 rUi

atldomlnis mUS(loe

Sub cutanlilous tiue (tatty ye r)

BElOW

A POrl lJfosis 01 etemal oblique muscle

Aponeul~)sis 01 Internal oblique mU$cl~

Anteriol lay~ of r~ltdus st~ath EXttom1 oblique rnu$cll

Rectus Jbdominis muscle Intoernal Aponeurc-sis of tra~fersU$ oblique muscle-

at-domlnis muscentl ~ Skio

Tra nsvitSus abdomioLs ml)ZClt

TralSVersaHs fascia Medial umQil iegtt1 1i9Jment -and folj

Uldchus Peritoneum (ir median Umbilj~al Suboutane ous

Extraprftone 11ascia

Ymbilimiddot~1 fold)

preu9poundiea1 fascia

tissue (fatty 4nd m~mbr3n(iUS layers)

o Above the arcuate line (A horizontal line 13 of the distance bw the umbilicus and the

pubic symphysis) -10 Aponeurosis divides into an AntPost Laminae

o The Ant Laminae joins EO and Post Laminae joins Trans Abdominis = Ant and Post

RECTUS SHEATH respectively

o BElOW the arcuate line - all 3 aponeurosis join ANTERIOR to rectus muscle to meet its

counterpart in the midline (linea Alba)

o Take away Msg - The abdomen is devoid of a posterior rectus sheath below the

arcuate line and is therefore more vulnerable to herniasinjuries

Question - A physician makes a deep incision in the patients midline immediately superior to

the pubic symphysis which of the following layers is his knife least likely to pass

Rectus Abdominis External Oblique Ant Rectus Sheath Posterior Rectus Sheath All of the

Above

Answer - All of the above None of the other answer choices are midline structures -LINEA

ALBA

Linea Alba has very poor blood supply - doesnt heal well after surgery Therefore this is a

common site for incisional hernias

a Spleen b Transverse colon c Descending colon d Stomach e Pleura

17 Meckels diverticulum is normally found 2 feet proximal from the

a Pyloric sphincter b Lower esophageal sphincter c Ileo-cecal valve d Middle valve of Huston e Anal valve

18 Ulcer in the posterior wall of the first part of the duodenum would erode ___ artery and would cause bleeding

a Left gastric b Right gastric c Hepatic artery proper d Gastroduodenal artery e Middle colic artery

19 An inflamed appendix is identified by a surgeon on the operation table by noting

a The appendicies epiploicae b The convergence of tenia c The artery of Drummond d The mesocolon e The mesosalphinx

20 The nerve which emerges through the psoas major is

a Femoral b Ilio-inguinal c Ilio-hypogastric d Pudendal e Subcostal

21 The right gonadal vein drains into the

a Azygos b Hemiazygos c Inferior Vena Cava d Right renal vein e Left renal vein

22 The hepatocytes in the liver is derived from

a Ectoderm b Endoderm c Mesoderm

d Neural ectoderm

23 Abscess in the lumbar vertebrae due to tuberculosis would spread to the adjacent muscle which is

a Psoas Major b Iliacus c Quadratus lumborum d Tranversus Abdominis

24 The anterior wall of the inguinal canal is formed by

a External oblique and transverses abdominis b External oblique and fascia transversalis c Internal oblique and external oblique d Internal oblique and transverses abdominis e Fascia transversalis and peritoneum

Meckels diverticulum is a result of which of the following developmental abnormalities shy

A Failure of the vitelline duct to close

B Failure of the herniated intestinal loop to retract into the abdomen

C Failure of the urachus to close

D Failure of the midgut to rotate

E Failure of the hepatic duct to close

Explanation

Meckels diverticulum is a result of the persistence of the proximal part of the vitelline duct This

diverticulum is usually found about 2 feet proximal to the ileocecal junction and is usually about 2 inches

long It is present in about 2 of the popUlation It may be the site of ectopic pancreatic tissue or gastric

mucosa and may develop inflammatory processes and ulcerations Acute Meckels diverticulitis

simulates appendicitis

Which of the following veins carries blood from the esophagus to the portal vein The

A right gastric vein

B left gastric vein c splenic vein D azygos vein

E left gastroepiploic vein

Explanation

The left gastric vein a direct branch of the portal vein drains blood from the lesser curvature of the

stomach and the inferior portion of the esophagus Because branches of the portal vein do not have

valves blood can flow in a retrograde path when there is an obstruction to flow through the portal system or liveL Rlooci Cln then flow from the nortl] vein thr()1Ph the left PRstric vein to the esonhlPlIS lno

through venous communications within the submucosa of the esophagus to esophageal veins that drain

into the azygos vein The increase in blood flow through the esophageal submucosal veins results in esophageal varices

On the posterior wall of the abdomen the celiac ganglion A contains cell bodies of postganglionic parasympathetic neurons B is synapsed upon by neurons in the posterior vagal trunk C is synapsed upon by neurons in the greater splanchnic nerve D contains sensory cell bodies of lumbar spinal nerves E contains cell bodies of neurons that cause an increase in the rate of peristasis

Explanation The celiac ganglion is one of the preaortic ganglia of the sympathetic nervous system It contains cell bodies of postganglionic sympathetic neurons The sympathetic splanchnic nerves contain preganglionic sympathetic neurons that pass through the sympathetic chain without synapsing These splanchnic nerves go to the preaortic ganglia to synapse The greater splanchnic nerve contains preganglionic neurons from spinal cord segments T5-T9 This nerve synapses in the celiac ganglion The nerve fibers in the vagal trunks are preganglionic parasympathetic fibers that go to the walls of the organs that they will innervate and synapse on postganglionic parasympathetic neurons in the walls of those organs Cell bodies of sensory neurons in the abdomen are found in the dorsal root ganglia or the sensory ganglia of the vagus nerve Sympathetic innervation decreases the rate of peristalsis parasympathetic innervation increases the rate of peristalsis

Which of the following pairs of arteries will allow blood to bypass an occlusion of the celiac trunk

A Left gastric artery-right gastric artery

B Left gastroepiploic artery-right gastroepiploic artery

C Superior pancreaticoduodenal artery-inferior pancreaticoduodenal artery

D Splenic artery-common hepatic artery

E Left gastric artery - proper hepatic artery

Explanation The anastoOlosis of a branch of the celiac trunk and a branch of the superior mesenteric artery will

provide collateral circulation around an occlusion of the celiac trunk Each of the other choices pair

branches of the celiac trunk therefore these will not provide collateral flow around the obstruction of the

celiac trunk The left gastric splenic and common hepatic arteries are direct branches of the celiac trunk

The right gastric artery is a branch of the proper hepatic artery which is a branch of the common hepatic artery The left gastroepiploic artery is a branch of the splenic artery The right gastroepiploic artery is a

branch of the gastroduodenal artery whlch is a branch of the common hepatic artery

Which of the following organs has appendices epiploica The

A sigmoid colon

Bjejunum

C duodenum

D stomach E esophagus

Explanation Appendices epiploica are characteristic of the colon Appendices epiploica are subserosal accumulations

of fat None of the organs of the gastrointestinal tract has appendices epiploica except the colon

Page 38: Chirag's Abdomen Review

Col li t ltt-~ otTl~tI ~nj pc~ 1lt1 turJoG

Ltf 14i1 tImiddot~ artoftl9 on tj phtAt$

L-oftqf 4t t~r 1=laquoIran d 1 bull shy~p l ci rj o fOOOts

Nerves follow the arteries - appreciate the splanchnic nervous system I

Uet~ric branch of left ~nal art

Ureterie branch of righi renal artelY

Left Zld lumbar in and co mlTlunication to as)erdin9 lumbar l(~in Hi ~ht tEZ1~~t~ t3r j t itn ~ nJ l1t- rlnd lfe i r1

Inferior me5nteri~ artery

Notice that the right testicular vein drains directly into the IVC and the right testicular artery drains

directly into the aorta However the left testicular vein drains into the L renal vein at a right angleshy

reason left testicle is lower and more susceptible to varicocele (bag of worms)

Also notice that the left renal vein has a longer course because the IVC is on the right side whereas

the right renal artery has a longer course because the aorta is on the left side

Appreciate the anterior to posterior relationship of structures in the hilum of the kidney - VAP - Vein

Artery Renal Pelvis (Ureter)

11____ __ L_ L_ n VJ __ _ _ t_L I I_ _ L __ L_ I -pound1 bull LI_~-I ____

Posterior View of Head of Pancreas in ( of Duodenum

Celiao hunk

Co mmon ~L~jJth art~ry

GastNduQdonal artrf (partilly in phantn)

P1)Sterior $Up~Jior panCflaticuduodfmal art~r~t

(Co mm on) bile duct

middot~1t~~t-1l---~-~- Right gshomiddotomental (gastoe plp lolc) 3rte (phantomost)

Grener paocre atic art-ry

1n1~rjor pancr-iatlc artery

Jtrifll supejo r pal)oreailcento)dJodenal artr1 (phantom)

Anastomotlo branch

POostetlor bJanch of jo f~ri of pan-reatir(lduodensl drttnj

Anterio r branch of i flferior palcreati~)duodenal art~(phan1om)

Notice the extensive blood supply to the pancreas and duodenum via the branches of the celiac trunk

Notice collateral supply from SMA branches - makes sense bc this is the jxn of foregutmidgut

Identify the vessels in this arteriogram

Hiltid i)f N~ck oi B)dvof Tail 01 pa nereas pan cent~as P-nmiddot-reas panCtCas

I nferie v~na cava

jHept1iic p(lrlai v~in

Port1 tnd H~pti lt a ftH prol

Comm on) bll duct

Ouodtnum

~ft colic (sio)Atta~ hmtrlt jt~xJr-ofha~elSe

muo(IIQn

Right ~lIc (h~j)tic)

il~gtture

In1triol m~oten lIein (rttr op~ritoMdO

SlJp efl or mes~n~fiC amrV and lipln

KNOW YOUR NEIGHBORHOOD

Questions

vVhiJh structure supplied by a bnmdlof the cclia( artery is not derivcd from foregut LemCJUCrITI

(A) Head of the pancte-a5

CD) Pyloric duolenum

Cystkduct

( Liver hepatocyt~~

~F) Body of the spleen

An infant presents with an omrhaJucele at birth -hi oJ the [oHm illg applies to his cM1-dition

(A) It is 31so seen ill p4titnts with aganghonic megacolon

(11) ft reuirs from a fal1ure of resorption of theviteUine d let

(C) It results from herniation at the-site of regression of the right umbilk vein

DJ It is caustd by faihtrc of recanalization of the midgut part of the duodenum

~ It ill camioo by a failuIt vf the midgul to return to the abGQminal uity after herniashytion in-n the urnbilk s l stalk

Ot er than the spleen occlusion Cif the spit-Ilk artery at its odgin wm most likely affect die blood supply to jllch st cnud

(A) Jejunum

(B) Body of th pal1~lltas

(C) LeSStT Cllmiddotlaturc of tl )toma-ch

(D Duodenum dista to the entrance of the Ornmou bile duct

E Fundus of the stomach

A 38-yeu-old batL~er with a history of heartburn suddenly experiences excluciating pain in the (plgastric region of th~ abdomeu SurgCry is perf~rme immediard y upon admisshysion to the 1IlcrgCJliy tuomh~re i~ evidence uf a ruptured ulcer in the posterior waU of the stomach Vhere will a surgeon first fi nd the stomach contenlSf

A) Greater p4ritoneal sac

rB) Cul~de-s~c of Douglas (--

C Omental bursa ~

--D) Paracolic gutter

rEj Between -he panttal perimltum and the posterior body wal1

At birth an infant presents with a st()ma~ rb~tbas~njJled jfltotb~diaplfagru 1A1ltre is the defect thatresulied iiitJle heini~t()n shy~tsophagealbiatus

7 - rH-- Hiatus for the inferior vena cava

( Pleuroperitoneal membrane -(0) Septum transvcrsum

(E) Right Crlt~

An infant born with DOVv7l syndrome presents with bili()u~ vomiting Ahat congenital defect does the infant have

(A) Pyloric stenosis

(B) Meckel diverticulum C) Ornphaloce1e

(D) Gastroschisis

( ~ ) Duodenal atresia y A patient with cirrhosis of the liver presents with ~ bacalvaricestnlreased retrograde pressure in which veins caused the varices

(A) Paraumuilical

(B) Splenic

(ct AzygltJus

(15))G~trk ( (-F) Superior mesemeric

A htaltby 3-year~old male patient experiences a hernial sa protruding from the anterior abdominal wall about halfway between me anterior superior ilia spine and the pubk tuberde Pulsations of al1 artery are palpated medial to the protrusion site through the abdominal walL Which layer of the anterior abdominal wall will first be traversed by the

1hctma

fA) Rectus sheath (B) External oblique aponeurosis

(C) Inguinal ligament

lD) Transversalis fusda

(E) Cremasteric fa~cia

After 5urgi(aj ffpair of a hernia the patient tXperienccs mtmlgtness in the skin on the anteshyrior aspect of the S(Totum_ Vhaf nerve may have been lesioned during thehemiorrhaphy

(A) Femoral

(B) Obturator

(C) Ilioinguinal

(D) lliohypogastrk

(E) Pudendal

A 23~year-LJld female secretary il1 good health ~-uddcn1) doubles over with pain in the a ea of the 1JmbRicu$ Sbe feels vartn and ltneasy and has no appetite That night the pain seems to have mQved to the tower right abdominal regjol1 and she calls her family doctor who then arranges for an ambulance to pk-k her up and take her to the hospitaL Wh ell ntn~ perceived in the area of the urnbilirus most Hkely carried lhe pairfu I sensations into the eNS

tA) Vagus nerves I~

V B)

) Lessersplanchnk nerves

tC) Pudendal nerves

(D) lIiohpogastrk nerves

(E) Greater splam ic l erves

A CT reveals carcinoma in the bOod of the ancreas Vhich blood vessel trut ourses ----~- - -bull ------ --shy

immediately poftterior to the body ofthe pancreas is the m~t likely to be oompressed

(A) Splenk artery

(B) Abdominal aorta (C) Portal vein

(1) Splenic vein

(E) Renal vein

A patient has a penrln1l1ng uker of the posterior wall ot the br~l part ot the (lUooenmn llkh blood vessel is subject to erosion

(A) Common hepatic artery

(B) Gastroouodenal artery

(C) Proper hevatic artery

(D) Celiac artery

(E) Anterior inferior 11amrelltlcoduodcnal attery

Your patient has been diagnosed -ith a carcinoma locallted to the head and l~e(k of the pancreas Another clinical sign would be

A esophageal varices

(8) hemorrhoids

C) a caput medusa

(D) increased pra Teuro n th~ hepatic veins

(E) enlarged right supra lavkular lymph nodes

Wltkh of the foUowing structures develops in the ventral mesentery

(A) Spleen

(B) Jeiunum (C) Head of1ht pancreas (D) Transverse colon (E) Stomach

ti l Uw ~ littwin~ f( S-t lil oai Imdge ~ hi(h or tbt la~)d J truetur tgt liJ llntn nl) he hl p UC iJd [IIi ell

c o

A) drains Ie tht infCrior a La aI

R t middot~nfl0 ~ill to th~ lunlgtn of h i dtlndCrlllfH

(e) m t bull JiJattd on tl l J n T ~H

D ) sup Lc O VSlt I Hlid bhtu l 1 li - -I un oid

( ) U~tpli(t tr j middottUh~ 1 v(( b~nt rfK n1ilc~Zm

ANSWERS AND EXPLANATIONS

Answer E The spleen is t hlttnopodicand lymph organ demlted from mesoderm

Answ~ R Al1 tlmphalocele is caused by it failure of the nlidgut to return to the ahdomir nat cavity after herniation into the umbiliau Stalk Choices Aand D maybe seen in infants with Down syndrome choice D ~s the specific CBuse ofduudcnal JtiCSitt Choice C is (ile cause of gclstrosbisis and Choice B nsults iu a Meurolktldivertku1-tlB

Answer B The fundus ofthe stomach is suppHed by soort gastric brunches of the splenic altery The splenic artery supplies the body and tail of the pancreas part of the greater curvature of the sttmla(h and the spleen Te jejunum part of the head of the pancreas and tht~ duodenum distal to the entrance of the commOll bile duct are supplied by the superior mesenterk artery clll~l ~be less r ctlt1ature cmd the pylQric antrum are supplied by the right and lei gastric art(ries

AnSWftt C Tbeomental bursa or lesser ~ritoneaj sac lies direcdy posterior to the proxshyimal part of the duodeTtlm and the stomach and would be the first site where stomach contents ~Ott1d be fpoundluncL

Answer C A defect in a llleuropcritoneal membrane (uswlly the left) is the typical site of i1 cc-ngenitlI diilphragluatic hemia llere the membr4ne fails to dose ()pound( of the perishycCirdiopcritulleal canals

Answer E DuoJenal atresia and aganglionic megacoion are congwitaI defects S~Il in patients with Dowmiddotnsyndrome

Answer D RulaTgemt~llt of and retrograde flow in g~lstrk vel_ns in particlJl~r the kft gas~ tricveins dilates the capillary bed in rhe wall of the esophagus in (ases of porta yper~

tension Blood flow would increase in and dilampte tribntarkgts of the (lZygOUS vein on the other side of the capiUary bed but flow in this vein is in the typical direction t()ward the superior vena cava Paraumbiii(ltU vein eilgorgement contributes to a caput medusH Splenic ~nlargement might prc~nt with 5plcnonlegaly and balt-kflow in to tlu superior m~~ntclic vein occurs but is asymptomatic

Answer D The patient hagt an indirect inguinal hernia whi~h emerges from the antt-rior abdominal wall through the deep inguinltilling Theeep ring is a fault in the transv~rshysaUs fascia this I~yer wiIJ be penetrated first by the hernia

An~Wer C The ilioinguinal nenc which provides sens~llion to the lnedlal thigh ltmclanteshytior SClotunl pass~lt th rough the 5uperfh_ial inguinal ring ind $subject to inj i1T) becaus-e

it is in the operatitm Held of the erniorrhapny

Auswer B The leMHr splanchnic nerves are sympathdic nerVlts that carry viscera l sensashytlltgtrogt ftom illtllt1m~d ()J stietched gust (itinteitinal ~tructures (in this case the pprndix) into tnt eNS Lesser splanchnic ntTYcsarisc from thmiddot T9--T12 spinal cord segments lt1nd provide sympathetic innenation tD rnidgut siruc1ures whiCh include CLe app~JldD Viscera] Pain arising from affecLed Inidgut ampt 1C1ure is referred over the same dl- matorne~ of spinal segrnertts v-hich provide the sympathetic Innervation n this G1SC of appendicitis the invohen~n t of the ltire) of t e unlhHku indud s the T 10 dermatome

Answer B Of the five choices onty the dscending olon is retroperiton~al aldwould be a lik ~ ( choice to be seen immediately a(~jilcent to t11e posterior abdominal middotn~L

Amwen D The SpltftlC ~-ein ourses posterior to the body of the panneas m its way tt drain into the superior mCSfttltlri( vein

Answcr B TILt glstrodllolticnal artery 1 direct hIamh of the comrootl hepatic artery courses immediately pt))iwri() to the duodenum and is slbject to erosion

Answer B Carcinoma of th pan middott3S in the 1tilt1 may compreampgt the portltil vein at irs orishygill The poTtai vcin is fomled when the splenic vein jQiaswith tfie superior meStllt eric vein The inferiot mesenteric vein joins the ~plenjc vein just priOT to tlli~ point at which the splenic joins the superior Jlleit1ltcri( vein Increescd venous presslu in the inferior mesenteric vein is a cause of emo hoid~

Answer C The- velltral pancreas wilich forms most of the head of the p ~ncr as develops in the ventral mes(ntery as antutgrowth of the hepatic diverticulum Th~ hepatic divershyticulull induding the biIJary appa~atus develops in tbe ventral mesentery of the foregut

Answer~ A The superior mesenteric ~in joins with the spienkvein to form the hepatic portal vciu

Answer D The structure at gttlK is the proper hepatic artery~ whkh suppUesoxygenated b middotood to the liver

MAKE SURE YOU KNOW the diff bw Rectus Sheath above and below the arcuate line

ABOVE

Aponeurosis of xiiltmal obllque musclo

Extemll f)biquw musde

Reotln ilbdomlnls musole S~in

Internal 9bliquQ mY~QI

AponeUfOsi$ of hJH$V~~S Lir9a a lb lbdolTlin~ musolo Tri OJV6 rUi

atldomlnis mUS(loe

Sub cutanlilous tiue (tatty ye r)

BElOW

A POrl lJfosis 01 etemal oblique muscle

Aponeul~)sis 01 Internal oblique mU$cl~

Anteriol lay~ of r~ltdus st~ath EXttom1 oblique rnu$cll

Rectus Jbdominis muscle Intoernal Aponeurc-sis of tra~fersU$ oblique muscle-

at-domlnis muscentl ~ Skio

Tra nsvitSus abdomioLs ml)ZClt

TralSVersaHs fascia Medial umQil iegtt1 1i9Jment -and folj

Uldchus Peritoneum (ir median Umbilj~al Suboutane ous

Extraprftone 11ascia

Ymbilimiddot~1 fold)

preu9poundiea1 fascia

tissue (fatty 4nd m~mbr3n(iUS layers)

o Above the arcuate line (A horizontal line 13 of the distance bw the umbilicus and the

pubic symphysis) -10 Aponeurosis divides into an AntPost Laminae

o The Ant Laminae joins EO and Post Laminae joins Trans Abdominis = Ant and Post

RECTUS SHEATH respectively

o BElOW the arcuate line - all 3 aponeurosis join ANTERIOR to rectus muscle to meet its

counterpart in the midline (linea Alba)

o Take away Msg - The abdomen is devoid of a posterior rectus sheath below the

arcuate line and is therefore more vulnerable to herniasinjuries

Question - A physician makes a deep incision in the patients midline immediately superior to

the pubic symphysis which of the following layers is his knife least likely to pass

Rectus Abdominis External Oblique Ant Rectus Sheath Posterior Rectus Sheath All of the

Above

Answer - All of the above None of the other answer choices are midline structures -LINEA

ALBA

Linea Alba has very poor blood supply - doesnt heal well after surgery Therefore this is a

common site for incisional hernias

a Spleen b Transverse colon c Descending colon d Stomach e Pleura

17 Meckels diverticulum is normally found 2 feet proximal from the

a Pyloric sphincter b Lower esophageal sphincter c Ileo-cecal valve d Middle valve of Huston e Anal valve

18 Ulcer in the posterior wall of the first part of the duodenum would erode ___ artery and would cause bleeding

a Left gastric b Right gastric c Hepatic artery proper d Gastroduodenal artery e Middle colic artery

19 An inflamed appendix is identified by a surgeon on the operation table by noting

a The appendicies epiploicae b The convergence of tenia c The artery of Drummond d The mesocolon e The mesosalphinx

20 The nerve which emerges through the psoas major is

a Femoral b Ilio-inguinal c Ilio-hypogastric d Pudendal e Subcostal

21 The right gonadal vein drains into the

a Azygos b Hemiazygos c Inferior Vena Cava d Right renal vein e Left renal vein

22 The hepatocytes in the liver is derived from

a Ectoderm b Endoderm c Mesoderm

d Neural ectoderm

23 Abscess in the lumbar vertebrae due to tuberculosis would spread to the adjacent muscle which is

a Psoas Major b Iliacus c Quadratus lumborum d Tranversus Abdominis

24 The anterior wall of the inguinal canal is formed by

a External oblique and transverses abdominis b External oblique and fascia transversalis c Internal oblique and external oblique d Internal oblique and transverses abdominis e Fascia transversalis and peritoneum

Meckels diverticulum is a result of which of the following developmental abnormalities shy

A Failure of the vitelline duct to close

B Failure of the herniated intestinal loop to retract into the abdomen

C Failure of the urachus to close

D Failure of the midgut to rotate

E Failure of the hepatic duct to close

Explanation

Meckels diverticulum is a result of the persistence of the proximal part of the vitelline duct This

diverticulum is usually found about 2 feet proximal to the ileocecal junction and is usually about 2 inches

long It is present in about 2 of the popUlation It may be the site of ectopic pancreatic tissue or gastric

mucosa and may develop inflammatory processes and ulcerations Acute Meckels diverticulitis

simulates appendicitis

Which of the following veins carries blood from the esophagus to the portal vein The

A right gastric vein

B left gastric vein c splenic vein D azygos vein

E left gastroepiploic vein

Explanation

The left gastric vein a direct branch of the portal vein drains blood from the lesser curvature of the

stomach and the inferior portion of the esophagus Because branches of the portal vein do not have

valves blood can flow in a retrograde path when there is an obstruction to flow through the portal system or liveL Rlooci Cln then flow from the nortl] vein thr()1Ph the left PRstric vein to the esonhlPlIS lno

through venous communications within the submucosa of the esophagus to esophageal veins that drain

into the azygos vein The increase in blood flow through the esophageal submucosal veins results in esophageal varices

On the posterior wall of the abdomen the celiac ganglion A contains cell bodies of postganglionic parasympathetic neurons B is synapsed upon by neurons in the posterior vagal trunk C is synapsed upon by neurons in the greater splanchnic nerve D contains sensory cell bodies of lumbar spinal nerves E contains cell bodies of neurons that cause an increase in the rate of peristasis

Explanation The celiac ganglion is one of the preaortic ganglia of the sympathetic nervous system It contains cell bodies of postganglionic sympathetic neurons The sympathetic splanchnic nerves contain preganglionic sympathetic neurons that pass through the sympathetic chain without synapsing These splanchnic nerves go to the preaortic ganglia to synapse The greater splanchnic nerve contains preganglionic neurons from spinal cord segments T5-T9 This nerve synapses in the celiac ganglion The nerve fibers in the vagal trunks are preganglionic parasympathetic fibers that go to the walls of the organs that they will innervate and synapse on postganglionic parasympathetic neurons in the walls of those organs Cell bodies of sensory neurons in the abdomen are found in the dorsal root ganglia or the sensory ganglia of the vagus nerve Sympathetic innervation decreases the rate of peristalsis parasympathetic innervation increases the rate of peristalsis

Which of the following pairs of arteries will allow blood to bypass an occlusion of the celiac trunk

A Left gastric artery-right gastric artery

B Left gastroepiploic artery-right gastroepiploic artery

C Superior pancreaticoduodenal artery-inferior pancreaticoduodenal artery

D Splenic artery-common hepatic artery

E Left gastric artery - proper hepatic artery

Explanation The anastoOlosis of a branch of the celiac trunk and a branch of the superior mesenteric artery will

provide collateral circulation around an occlusion of the celiac trunk Each of the other choices pair

branches of the celiac trunk therefore these will not provide collateral flow around the obstruction of the

celiac trunk The left gastric splenic and common hepatic arteries are direct branches of the celiac trunk

The right gastric artery is a branch of the proper hepatic artery which is a branch of the common hepatic artery The left gastroepiploic artery is a branch of the splenic artery The right gastroepiploic artery is a

branch of the gastroduodenal artery whlch is a branch of the common hepatic artery

Which of the following organs has appendices epiploica The

A sigmoid colon

Bjejunum

C duodenum

D stomach E esophagus

Explanation Appendices epiploica are characteristic of the colon Appendices epiploica are subserosal accumulations

of fat None of the organs of the gastrointestinal tract has appendices epiploica except the colon

Page 39: Chirag's Abdomen Review

Posterior View of Head of Pancreas in ( of Duodenum

Celiao hunk

Co mmon ~L~jJth art~ry

GastNduQdonal artrf (partilly in phantn)

P1)Sterior $Up~Jior panCflaticuduodfmal art~r~t

(Co mm on) bile duct

middot~1t~~t-1l---~-~- Right gshomiddotomental (gastoe plp lolc) 3rte (phantomost)

Grener paocre atic art-ry

1n1~rjor pancr-iatlc artery

Jtrifll supejo r pal)oreailcento)dJodenal artr1 (phantom)

Anastomotlo branch

POostetlor bJanch of jo f~ri of pan-reatir(lduodensl drttnj

Anterio r branch of i flferior palcreati~)duodenal art~(phan1om)

Notice the extensive blood supply to the pancreas and duodenum via the branches of the celiac trunk

Notice collateral supply from SMA branches - makes sense bc this is the jxn of foregutmidgut

Identify the vessels in this arteriogram

Hiltid i)f N~ck oi B)dvof Tail 01 pa nereas pan cent~as P-nmiddot-reas panCtCas

I nferie v~na cava

jHept1iic p(lrlai v~in

Port1 tnd H~pti lt a ftH prol

Comm on) bll duct

Ouodtnum

~ft colic (sio)Atta~ hmtrlt jt~xJr-ofha~elSe

muo(IIQn

Right ~lIc (h~j)tic)

il~gtture

In1triol m~oten lIein (rttr op~ritoMdO

SlJp efl or mes~n~fiC amrV and lipln

KNOW YOUR NEIGHBORHOOD

Questions

vVhiJh structure supplied by a bnmdlof the cclia( artery is not derivcd from foregut LemCJUCrITI

(A) Head of the pancte-a5

CD) Pyloric duolenum

Cystkduct

( Liver hepatocyt~~

~F) Body of the spleen

An infant presents with an omrhaJucele at birth -hi oJ the [oHm illg applies to his cM1-dition

(A) It is 31so seen ill p4titnts with aganghonic megacolon

(11) ft reuirs from a fal1ure of resorption of theviteUine d let

(C) It results from herniation at the-site of regression of the right umbilk vein

DJ It is caustd by faihtrc of recanalization of the midgut part of the duodenum

~ It ill camioo by a failuIt vf the midgul to return to the abGQminal uity after herniashytion in-n the urnbilk s l stalk

Ot er than the spleen occlusion Cif the spit-Ilk artery at its odgin wm most likely affect die blood supply to jllch st cnud

(A) Jejunum

(B) Body of th pal1~lltas

(C) LeSStT Cllmiddotlaturc of tl )toma-ch

(D Duodenum dista to the entrance of the Ornmou bile duct

E Fundus of the stomach

A 38-yeu-old batL~er with a history of heartburn suddenly experiences excluciating pain in the (plgastric region of th~ abdomeu SurgCry is perf~rme immediard y upon admisshysion to the 1IlcrgCJliy tuomh~re i~ evidence uf a ruptured ulcer in the posterior waU of the stomach Vhere will a surgeon first fi nd the stomach contenlSf

A) Greater p4ritoneal sac

rB) Cul~de-s~c of Douglas (--

C Omental bursa ~

--D) Paracolic gutter

rEj Between -he panttal perimltum and the posterior body wal1

At birth an infant presents with a st()ma~ rb~tbas~njJled jfltotb~diaplfagru 1A1ltre is the defect thatresulied iiitJle heini~t()n shy~tsophagealbiatus

7 - rH-- Hiatus for the inferior vena cava

( Pleuroperitoneal membrane -(0) Septum transvcrsum

(E) Right Crlt~

An infant born with DOVv7l syndrome presents with bili()u~ vomiting Ahat congenital defect does the infant have

(A) Pyloric stenosis

(B) Meckel diverticulum C) Ornphaloce1e

(D) Gastroschisis

( ~ ) Duodenal atresia y A patient with cirrhosis of the liver presents with ~ bacalvaricestnlreased retrograde pressure in which veins caused the varices

(A) Paraumuilical

(B) Splenic

(ct AzygltJus

(15))G~trk ( (-F) Superior mesemeric

A htaltby 3-year~old male patient experiences a hernial sa protruding from the anterior abdominal wall about halfway between me anterior superior ilia spine and the pubk tuberde Pulsations of al1 artery are palpated medial to the protrusion site through the abdominal walL Which layer of the anterior abdominal wall will first be traversed by the

1hctma

fA) Rectus sheath (B) External oblique aponeurosis

(C) Inguinal ligament

lD) Transversalis fusda

(E) Cremasteric fa~cia

After 5urgi(aj ffpair of a hernia the patient tXperienccs mtmlgtness in the skin on the anteshyrior aspect of the S(Totum_ Vhaf nerve may have been lesioned during thehemiorrhaphy

(A) Femoral

(B) Obturator

(C) Ilioinguinal

(D) lliohypogastrk

(E) Pudendal

A 23~year-LJld female secretary il1 good health ~-uddcn1) doubles over with pain in the a ea of the 1JmbRicu$ Sbe feels vartn and ltneasy and has no appetite That night the pain seems to have mQved to the tower right abdominal regjol1 and she calls her family doctor who then arranges for an ambulance to pk-k her up and take her to the hospitaL Wh ell ntn~ perceived in the area of the urnbilirus most Hkely carried lhe pairfu I sensations into the eNS

tA) Vagus nerves I~

V B)

) Lessersplanchnk nerves

tC) Pudendal nerves

(D) lIiohpogastrk nerves

(E) Greater splam ic l erves

A CT reveals carcinoma in the bOod of the ancreas Vhich blood vessel trut ourses ----~- - -bull ------ --shy

immediately poftterior to the body ofthe pancreas is the m~t likely to be oompressed

(A) Splenk artery

(B) Abdominal aorta (C) Portal vein

(1) Splenic vein

(E) Renal vein

A patient has a penrln1l1ng uker of the posterior wall ot the br~l part ot the (lUooenmn llkh blood vessel is subject to erosion

(A) Common hepatic artery

(B) Gastroouodenal artery

(C) Proper hevatic artery

(D) Celiac artery

(E) Anterior inferior 11amrelltlcoduodcnal attery

Your patient has been diagnosed -ith a carcinoma locallted to the head and l~e(k of the pancreas Another clinical sign would be

A esophageal varices

(8) hemorrhoids

C) a caput medusa

(D) increased pra Teuro n th~ hepatic veins

(E) enlarged right supra lavkular lymph nodes

Wltkh of the foUowing structures develops in the ventral mesentery

(A) Spleen

(B) Jeiunum (C) Head of1ht pancreas (D) Transverse colon (E) Stomach

ti l Uw ~ littwin~ f( S-t lil oai Imdge ~ hi(h or tbt la~)d J truetur tgt liJ llntn nl) he hl p UC iJd [IIi ell

c o

A) drains Ie tht infCrior a La aI

R t middot~nfl0 ~ill to th~ lunlgtn of h i dtlndCrlllfH

(e) m t bull JiJattd on tl l J n T ~H

D ) sup Lc O VSlt I Hlid bhtu l 1 li - -I un oid

( ) U~tpli(t tr j middottUh~ 1 v(( b~nt rfK n1ilc~Zm

ANSWERS AND EXPLANATIONS

Answer E The spleen is t hlttnopodicand lymph organ demlted from mesoderm

Answ~ R Al1 tlmphalocele is caused by it failure of the nlidgut to return to the ahdomir nat cavity after herniation into the umbiliau Stalk Choices Aand D maybe seen in infants with Down syndrome choice D ~s the specific CBuse ofduudcnal JtiCSitt Choice C is (ile cause of gclstrosbisis and Choice B nsults iu a Meurolktldivertku1-tlB

Answer B The fundus ofthe stomach is suppHed by soort gastric brunches of the splenic altery The splenic artery supplies the body and tail of the pancreas part of the greater curvature of the sttmla(h and the spleen Te jejunum part of the head of the pancreas and tht~ duodenum distal to the entrance of the commOll bile duct are supplied by the superior mesenterk artery clll~l ~be less r ctlt1ature cmd the pylQric antrum are supplied by the right and lei gastric art(ries

AnSWftt C Tbeomental bursa or lesser ~ritoneaj sac lies direcdy posterior to the proxshyimal part of the duodeTtlm and the stomach and would be the first site where stomach contents ~Ott1d be fpoundluncL

Answer C A defect in a llleuropcritoneal membrane (uswlly the left) is the typical site of i1 cc-ngenitlI diilphragluatic hemia llere the membr4ne fails to dose ()pound( of the perishycCirdiopcritulleal canals

Answer E DuoJenal atresia and aganglionic megacoion are congwitaI defects S~Il in patients with Dowmiddotnsyndrome

Answer D RulaTgemt~llt of and retrograde flow in g~lstrk vel_ns in particlJl~r the kft gas~ tricveins dilates the capillary bed in rhe wall of the esophagus in (ases of porta yper~

tension Blood flow would increase in and dilampte tribntarkgts of the (lZygOUS vein on the other side of the capiUary bed but flow in this vein is in the typical direction t()ward the superior vena cava Paraumbiii(ltU vein eilgorgement contributes to a caput medusH Splenic ~nlargement might prc~nt with 5plcnonlegaly and balt-kflow in to tlu superior m~~ntclic vein occurs but is asymptomatic

Answer D The patient hagt an indirect inguinal hernia whi~h emerges from the antt-rior abdominal wall through the deep inguinltilling Theeep ring is a fault in the transv~rshysaUs fascia this I~yer wiIJ be penetrated first by the hernia

An~Wer C The ilioinguinal nenc which provides sens~llion to the lnedlal thigh ltmclanteshytior SClotunl pass~lt th rough the 5uperfh_ial inguinal ring ind $subject to inj i1T) becaus-e

it is in the operatitm Held of the erniorrhapny

Auswer B The leMHr splanchnic nerves are sympathdic nerVlts that carry viscera l sensashytlltgtrogt ftom illtllt1m~d ()J stietched gust (itinteitinal ~tructures (in this case the pprndix) into tnt eNS Lesser splanchnic ntTYcsarisc from thmiddot T9--T12 spinal cord segments lt1nd provide sympathetic innenation tD rnidgut siruc1ures whiCh include CLe app~JldD Viscera] Pain arising from affecLed Inidgut ampt 1C1ure is referred over the same dl- matorne~ of spinal segrnertts v-hich provide the sympathetic Innervation n this G1SC of appendicitis the invohen~n t of the ltire) of t e unlhHku indud s the T 10 dermatome

Answer B Of the five choices onty the dscending olon is retroperiton~al aldwould be a lik ~ ( choice to be seen immediately a(~jilcent to t11e posterior abdominal middotn~L

Amwen D The SpltftlC ~-ein ourses posterior to the body of the panneas m its way tt drain into the superior mCSfttltlri( vein

Answcr B TILt glstrodllolticnal artery 1 direct hIamh of the comrootl hepatic artery courses immediately pt))iwri() to the duodenum and is slbject to erosion

Answer B Carcinoma of th pan middott3S in the 1tilt1 may compreampgt the portltil vein at irs orishygill The poTtai vcin is fomled when the splenic vein jQiaswith tfie superior meStllt eric vein The inferiot mesenteric vein joins the ~plenjc vein just priOT to tlli~ point at which the splenic joins the superior Jlleit1ltcri( vein Increescd venous presslu in the inferior mesenteric vein is a cause of emo hoid~

Answer C The- velltral pancreas wilich forms most of the head of the p ~ncr as develops in the ventral mes(ntery as antutgrowth of the hepatic diverticulum Th~ hepatic divershyticulull induding the biIJary appa~atus develops in tbe ventral mesentery of the foregut

Answer~ A The superior mesenteric ~in joins with the spienkvein to form the hepatic portal vciu

Answer D The structure at gttlK is the proper hepatic artery~ whkh suppUesoxygenated b middotood to the liver

MAKE SURE YOU KNOW the diff bw Rectus Sheath above and below the arcuate line

ABOVE

Aponeurosis of xiiltmal obllque musclo

Extemll f)biquw musde

Reotln ilbdomlnls musole S~in

Internal 9bliquQ mY~QI

AponeUfOsi$ of hJH$V~~S Lir9a a lb lbdolTlin~ musolo Tri OJV6 rUi

atldomlnis mUS(loe

Sub cutanlilous tiue (tatty ye r)

BElOW

A POrl lJfosis 01 etemal oblique muscle

Aponeul~)sis 01 Internal oblique mU$cl~

Anteriol lay~ of r~ltdus st~ath EXttom1 oblique rnu$cll

Rectus Jbdominis muscle Intoernal Aponeurc-sis of tra~fersU$ oblique muscle-

at-domlnis muscentl ~ Skio

Tra nsvitSus abdomioLs ml)ZClt

TralSVersaHs fascia Medial umQil iegtt1 1i9Jment -and folj

Uldchus Peritoneum (ir median Umbilj~al Suboutane ous

Extraprftone 11ascia

Ymbilimiddot~1 fold)

preu9poundiea1 fascia

tissue (fatty 4nd m~mbr3n(iUS layers)

o Above the arcuate line (A horizontal line 13 of the distance bw the umbilicus and the

pubic symphysis) -10 Aponeurosis divides into an AntPost Laminae

o The Ant Laminae joins EO and Post Laminae joins Trans Abdominis = Ant and Post

RECTUS SHEATH respectively

o BElOW the arcuate line - all 3 aponeurosis join ANTERIOR to rectus muscle to meet its

counterpart in the midline (linea Alba)

o Take away Msg - The abdomen is devoid of a posterior rectus sheath below the

arcuate line and is therefore more vulnerable to herniasinjuries

Question - A physician makes a deep incision in the patients midline immediately superior to

the pubic symphysis which of the following layers is his knife least likely to pass

Rectus Abdominis External Oblique Ant Rectus Sheath Posterior Rectus Sheath All of the

Above

Answer - All of the above None of the other answer choices are midline structures -LINEA

ALBA

Linea Alba has very poor blood supply - doesnt heal well after surgery Therefore this is a

common site for incisional hernias

a Spleen b Transverse colon c Descending colon d Stomach e Pleura

17 Meckels diverticulum is normally found 2 feet proximal from the

a Pyloric sphincter b Lower esophageal sphincter c Ileo-cecal valve d Middle valve of Huston e Anal valve

18 Ulcer in the posterior wall of the first part of the duodenum would erode ___ artery and would cause bleeding

a Left gastric b Right gastric c Hepatic artery proper d Gastroduodenal artery e Middle colic artery

19 An inflamed appendix is identified by a surgeon on the operation table by noting

a The appendicies epiploicae b The convergence of tenia c The artery of Drummond d The mesocolon e The mesosalphinx

20 The nerve which emerges through the psoas major is

a Femoral b Ilio-inguinal c Ilio-hypogastric d Pudendal e Subcostal

21 The right gonadal vein drains into the

a Azygos b Hemiazygos c Inferior Vena Cava d Right renal vein e Left renal vein

22 The hepatocytes in the liver is derived from

a Ectoderm b Endoderm c Mesoderm

d Neural ectoderm

23 Abscess in the lumbar vertebrae due to tuberculosis would spread to the adjacent muscle which is

a Psoas Major b Iliacus c Quadratus lumborum d Tranversus Abdominis

24 The anterior wall of the inguinal canal is formed by

a External oblique and transverses abdominis b External oblique and fascia transversalis c Internal oblique and external oblique d Internal oblique and transverses abdominis e Fascia transversalis and peritoneum

Meckels diverticulum is a result of which of the following developmental abnormalities shy

A Failure of the vitelline duct to close

B Failure of the herniated intestinal loop to retract into the abdomen

C Failure of the urachus to close

D Failure of the midgut to rotate

E Failure of the hepatic duct to close

Explanation

Meckels diverticulum is a result of the persistence of the proximal part of the vitelline duct This

diverticulum is usually found about 2 feet proximal to the ileocecal junction and is usually about 2 inches

long It is present in about 2 of the popUlation It may be the site of ectopic pancreatic tissue or gastric

mucosa and may develop inflammatory processes and ulcerations Acute Meckels diverticulitis

simulates appendicitis

Which of the following veins carries blood from the esophagus to the portal vein The

A right gastric vein

B left gastric vein c splenic vein D azygos vein

E left gastroepiploic vein

Explanation

The left gastric vein a direct branch of the portal vein drains blood from the lesser curvature of the

stomach and the inferior portion of the esophagus Because branches of the portal vein do not have

valves blood can flow in a retrograde path when there is an obstruction to flow through the portal system or liveL Rlooci Cln then flow from the nortl] vein thr()1Ph the left PRstric vein to the esonhlPlIS lno

through venous communications within the submucosa of the esophagus to esophageal veins that drain

into the azygos vein The increase in blood flow through the esophageal submucosal veins results in esophageal varices

On the posterior wall of the abdomen the celiac ganglion A contains cell bodies of postganglionic parasympathetic neurons B is synapsed upon by neurons in the posterior vagal trunk C is synapsed upon by neurons in the greater splanchnic nerve D contains sensory cell bodies of lumbar spinal nerves E contains cell bodies of neurons that cause an increase in the rate of peristasis

Explanation The celiac ganglion is one of the preaortic ganglia of the sympathetic nervous system It contains cell bodies of postganglionic sympathetic neurons The sympathetic splanchnic nerves contain preganglionic sympathetic neurons that pass through the sympathetic chain without synapsing These splanchnic nerves go to the preaortic ganglia to synapse The greater splanchnic nerve contains preganglionic neurons from spinal cord segments T5-T9 This nerve synapses in the celiac ganglion The nerve fibers in the vagal trunks are preganglionic parasympathetic fibers that go to the walls of the organs that they will innervate and synapse on postganglionic parasympathetic neurons in the walls of those organs Cell bodies of sensory neurons in the abdomen are found in the dorsal root ganglia or the sensory ganglia of the vagus nerve Sympathetic innervation decreases the rate of peristalsis parasympathetic innervation increases the rate of peristalsis

Which of the following pairs of arteries will allow blood to bypass an occlusion of the celiac trunk

A Left gastric artery-right gastric artery

B Left gastroepiploic artery-right gastroepiploic artery

C Superior pancreaticoduodenal artery-inferior pancreaticoduodenal artery

D Splenic artery-common hepatic artery

E Left gastric artery - proper hepatic artery

Explanation The anastoOlosis of a branch of the celiac trunk and a branch of the superior mesenteric artery will

provide collateral circulation around an occlusion of the celiac trunk Each of the other choices pair

branches of the celiac trunk therefore these will not provide collateral flow around the obstruction of the

celiac trunk The left gastric splenic and common hepatic arteries are direct branches of the celiac trunk

The right gastric artery is a branch of the proper hepatic artery which is a branch of the common hepatic artery The left gastroepiploic artery is a branch of the splenic artery The right gastroepiploic artery is a

branch of the gastroduodenal artery whlch is a branch of the common hepatic artery

Which of the following organs has appendices epiploica The

A sigmoid colon

Bjejunum

C duodenum

D stomach E esophagus

Explanation Appendices epiploica are characteristic of the colon Appendices epiploica are subserosal accumulations

of fat None of the organs of the gastrointestinal tract has appendices epiploica except the colon

Page 40: Chirag's Abdomen Review

Hiltid i)f N~ck oi B)dvof Tail 01 pa nereas pan cent~as P-nmiddot-reas panCtCas

I nferie v~na cava

jHept1iic p(lrlai v~in

Port1 tnd H~pti lt a ftH prol

Comm on) bll duct

Ouodtnum

~ft colic (sio)Atta~ hmtrlt jt~xJr-ofha~elSe

muo(IIQn

Right ~lIc (h~j)tic)

il~gtture

In1triol m~oten lIein (rttr op~ritoMdO

SlJp efl or mes~n~fiC amrV and lipln

KNOW YOUR NEIGHBORHOOD

Questions

vVhiJh structure supplied by a bnmdlof the cclia( artery is not derivcd from foregut LemCJUCrITI

(A) Head of the pancte-a5

CD) Pyloric duolenum

Cystkduct

( Liver hepatocyt~~

~F) Body of the spleen

An infant presents with an omrhaJucele at birth -hi oJ the [oHm illg applies to his cM1-dition

(A) It is 31so seen ill p4titnts with aganghonic megacolon

(11) ft reuirs from a fal1ure of resorption of theviteUine d let

(C) It results from herniation at the-site of regression of the right umbilk vein

DJ It is caustd by faihtrc of recanalization of the midgut part of the duodenum

~ It ill camioo by a failuIt vf the midgul to return to the abGQminal uity after herniashytion in-n the urnbilk s l stalk

Ot er than the spleen occlusion Cif the spit-Ilk artery at its odgin wm most likely affect die blood supply to jllch st cnud

(A) Jejunum

(B) Body of th pal1~lltas

(C) LeSStT Cllmiddotlaturc of tl )toma-ch

(D Duodenum dista to the entrance of the Ornmou bile duct

E Fundus of the stomach

A 38-yeu-old batL~er with a history of heartburn suddenly experiences excluciating pain in the (plgastric region of th~ abdomeu SurgCry is perf~rme immediard y upon admisshysion to the 1IlcrgCJliy tuomh~re i~ evidence uf a ruptured ulcer in the posterior waU of the stomach Vhere will a surgeon first fi nd the stomach contenlSf

A) Greater p4ritoneal sac

rB) Cul~de-s~c of Douglas (--

C Omental bursa ~

--D) Paracolic gutter

rEj Between -he panttal perimltum and the posterior body wal1

At birth an infant presents with a st()ma~ rb~tbas~njJled jfltotb~diaplfagru 1A1ltre is the defect thatresulied iiitJle heini~t()n shy~tsophagealbiatus

7 - rH-- Hiatus for the inferior vena cava

( Pleuroperitoneal membrane -(0) Septum transvcrsum

(E) Right Crlt~

An infant born with DOVv7l syndrome presents with bili()u~ vomiting Ahat congenital defect does the infant have

(A) Pyloric stenosis

(B) Meckel diverticulum C) Ornphaloce1e

(D) Gastroschisis

( ~ ) Duodenal atresia y A patient with cirrhosis of the liver presents with ~ bacalvaricestnlreased retrograde pressure in which veins caused the varices

(A) Paraumuilical

(B) Splenic

(ct AzygltJus

(15))G~trk ( (-F) Superior mesemeric

A htaltby 3-year~old male patient experiences a hernial sa protruding from the anterior abdominal wall about halfway between me anterior superior ilia spine and the pubk tuberde Pulsations of al1 artery are palpated medial to the protrusion site through the abdominal walL Which layer of the anterior abdominal wall will first be traversed by the

1hctma

fA) Rectus sheath (B) External oblique aponeurosis

(C) Inguinal ligament

lD) Transversalis fusda

(E) Cremasteric fa~cia

After 5urgi(aj ffpair of a hernia the patient tXperienccs mtmlgtness in the skin on the anteshyrior aspect of the S(Totum_ Vhaf nerve may have been lesioned during thehemiorrhaphy

(A) Femoral

(B) Obturator

(C) Ilioinguinal

(D) lliohypogastrk

(E) Pudendal

A 23~year-LJld female secretary il1 good health ~-uddcn1) doubles over with pain in the a ea of the 1JmbRicu$ Sbe feels vartn and ltneasy and has no appetite That night the pain seems to have mQved to the tower right abdominal regjol1 and she calls her family doctor who then arranges for an ambulance to pk-k her up and take her to the hospitaL Wh ell ntn~ perceived in the area of the urnbilirus most Hkely carried lhe pairfu I sensations into the eNS

tA) Vagus nerves I~

V B)

) Lessersplanchnk nerves

tC) Pudendal nerves

(D) lIiohpogastrk nerves

(E) Greater splam ic l erves

A CT reveals carcinoma in the bOod of the ancreas Vhich blood vessel trut ourses ----~- - -bull ------ --shy

immediately poftterior to the body ofthe pancreas is the m~t likely to be oompressed

(A) Splenk artery

(B) Abdominal aorta (C) Portal vein

(1) Splenic vein

(E) Renal vein

A patient has a penrln1l1ng uker of the posterior wall ot the br~l part ot the (lUooenmn llkh blood vessel is subject to erosion

(A) Common hepatic artery

(B) Gastroouodenal artery

(C) Proper hevatic artery

(D) Celiac artery

(E) Anterior inferior 11amrelltlcoduodcnal attery

Your patient has been diagnosed -ith a carcinoma locallted to the head and l~e(k of the pancreas Another clinical sign would be

A esophageal varices

(8) hemorrhoids

C) a caput medusa

(D) increased pra Teuro n th~ hepatic veins

(E) enlarged right supra lavkular lymph nodes

Wltkh of the foUowing structures develops in the ventral mesentery

(A) Spleen

(B) Jeiunum (C) Head of1ht pancreas (D) Transverse colon (E) Stomach

ti l Uw ~ littwin~ f( S-t lil oai Imdge ~ hi(h or tbt la~)d J truetur tgt liJ llntn nl) he hl p UC iJd [IIi ell

c o

A) drains Ie tht infCrior a La aI

R t middot~nfl0 ~ill to th~ lunlgtn of h i dtlndCrlllfH

(e) m t bull JiJattd on tl l J n T ~H

D ) sup Lc O VSlt I Hlid bhtu l 1 li - -I un oid

( ) U~tpli(t tr j middottUh~ 1 v(( b~nt rfK n1ilc~Zm

ANSWERS AND EXPLANATIONS

Answer E The spleen is t hlttnopodicand lymph organ demlted from mesoderm

Answ~ R Al1 tlmphalocele is caused by it failure of the nlidgut to return to the ahdomir nat cavity after herniation into the umbiliau Stalk Choices Aand D maybe seen in infants with Down syndrome choice D ~s the specific CBuse ofduudcnal JtiCSitt Choice C is (ile cause of gclstrosbisis and Choice B nsults iu a Meurolktldivertku1-tlB

Answer B The fundus ofthe stomach is suppHed by soort gastric brunches of the splenic altery The splenic artery supplies the body and tail of the pancreas part of the greater curvature of the sttmla(h and the spleen Te jejunum part of the head of the pancreas and tht~ duodenum distal to the entrance of the commOll bile duct are supplied by the superior mesenterk artery clll~l ~be less r ctlt1ature cmd the pylQric antrum are supplied by the right and lei gastric art(ries

AnSWftt C Tbeomental bursa or lesser ~ritoneaj sac lies direcdy posterior to the proxshyimal part of the duodeTtlm and the stomach and would be the first site where stomach contents ~Ott1d be fpoundluncL

Answer C A defect in a llleuropcritoneal membrane (uswlly the left) is the typical site of i1 cc-ngenitlI diilphragluatic hemia llere the membr4ne fails to dose ()pound( of the perishycCirdiopcritulleal canals

Answer E DuoJenal atresia and aganglionic megacoion are congwitaI defects S~Il in patients with Dowmiddotnsyndrome

Answer D RulaTgemt~llt of and retrograde flow in g~lstrk vel_ns in particlJl~r the kft gas~ tricveins dilates the capillary bed in rhe wall of the esophagus in (ases of porta yper~

tension Blood flow would increase in and dilampte tribntarkgts of the (lZygOUS vein on the other side of the capiUary bed but flow in this vein is in the typical direction t()ward the superior vena cava Paraumbiii(ltU vein eilgorgement contributes to a caput medusH Splenic ~nlargement might prc~nt with 5plcnonlegaly and balt-kflow in to tlu superior m~~ntclic vein occurs but is asymptomatic

Answer D The patient hagt an indirect inguinal hernia whi~h emerges from the antt-rior abdominal wall through the deep inguinltilling Theeep ring is a fault in the transv~rshysaUs fascia this I~yer wiIJ be penetrated first by the hernia

An~Wer C The ilioinguinal nenc which provides sens~llion to the lnedlal thigh ltmclanteshytior SClotunl pass~lt th rough the 5uperfh_ial inguinal ring ind $subject to inj i1T) becaus-e

it is in the operatitm Held of the erniorrhapny

Auswer B The leMHr splanchnic nerves are sympathdic nerVlts that carry viscera l sensashytlltgtrogt ftom illtllt1m~d ()J stietched gust (itinteitinal ~tructures (in this case the pprndix) into tnt eNS Lesser splanchnic ntTYcsarisc from thmiddot T9--T12 spinal cord segments lt1nd provide sympathetic innenation tD rnidgut siruc1ures whiCh include CLe app~JldD Viscera] Pain arising from affecLed Inidgut ampt 1C1ure is referred over the same dl- matorne~ of spinal segrnertts v-hich provide the sympathetic Innervation n this G1SC of appendicitis the invohen~n t of the ltire) of t e unlhHku indud s the T 10 dermatome

Answer B Of the five choices onty the dscending olon is retroperiton~al aldwould be a lik ~ ( choice to be seen immediately a(~jilcent to t11e posterior abdominal middotn~L

Amwen D The SpltftlC ~-ein ourses posterior to the body of the panneas m its way tt drain into the superior mCSfttltlri( vein

Answcr B TILt glstrodllolticnal artery 1 direct hIamh of the comrootl hepatic artery courses immediately pt))iwri() to the duodenum and is slbject to erosion

Answer B Carcinoma of th pan middott3S in the 1tilt1 may compreampgt the portltil vein at irs orishygill The poTtai vcin is fomled when the splenic vein jQiaswith tfie superior meStllt eric vein The inferiot mesenteric vein joins the ~plenjc vein just priOT to tlli~ point at which the splenic joins the superior Jlleit1ltcri( vein Increescd venous presslu in the inferior mesenteric vein is a cause of emo hoid~

Answer C The- velltral pancreas wilich forms most of the head of the p ~ncr as develops in the ventral mes(ntery as antutgrowth of the hepatic diverticulum Th~ hepatic divershyticulull induding the biIJary appa~atus develops in tbe ventral mesentery of the foregut

Answer~ A The superior mesenteric ~in joins with the spienkvein to form the hepatic portal vciu

Answer D The structure at gttlK is the proper hepatic artery~ whkh suppUesoxygenated b middotood to the liver

MAKE SURE YOU KNOW the diff bw Rectus Sheath above and below the arcuate line

ABOVE

Aponeurosis of xiiltmal obllque musclo

Extemll f)biquw musde

Reotln ilbdomlnls musole S~in

Internal 9bliquQ mY~QI

AponeUfOsi$ of hJH$V~~S Lir9a a lb lbdolTlin~ musolo Tri OJV6 rUi

atldomlnis mUS(loe

Sub cutanlilous tiue (tatty ye r)

BElOW

A POrl lJfosis 01 etemal oblique muscle

Aponeul~)sis 01 Internal oblique mU$cl~

Anteriol lay~ of r~ltdus st~ath EXttom1 oblique rnu$cll

Rectus Jbdominis muscle Intoernal Aponeurc-sis of tra~fersU$ oblique muscle-

at-domlnis muscentl ~ Skio

Tra nsvitSus abdomioLs ml)ZClt

TralSVersaHs fascia Medial umQil iegtt1 1i9Jment -and folj

Uldchus Peritoneum (ir median Umbilj~al Suboutane ous

Extraprftone 11ascia

Ymbilimiddot~1 fold)

preu9poundiea1 fascia

tissue (fatty 4nd m~mbr3n(iUS layers)

o Above the arcuate line (A horizontal line 13 of the distance bw the umbilicus and the

pubic symphysis) -10 Aponeurosis divides into an AntPost Laminae

o The Ant Laminae joins EO and Post Laminae joins Trans Abdominis = Ant and Post

RECTUS SHEATH respectively

o BElOW the arcuate line - all 3 aponeurosis join ANTERIOR to rectus muscle to meet its

counterpart in the midline (linea Alba)

o Take away Msg - The abdomen is devoid of a posterior rectus sheath below the

arcuate line and is therefore more vulnerable to herniasinjuries

Question - A physician makes a deep incision in the patients midline immediately superior to

the pubic symphysis which of the following layers is his knife least likely to pass

Rectus Abdominis External Oblique Ant Rectus Sheath Posterior Rectus Sheath All of the

Above

Answer - All of the above None of the other answer choices are midline structures -LINEA

ALBA

Linea Alba has very poor blood supply - doesnt heal well after surgery Therefore this is a

common site for incisional hernias

a Spleen b Transverse colon c Descending colon d Stomach e Pleura

17 Meckels diverticulum is normally found 2 feet proximal from the

a Pyloric sphincter b Lower esophageal sphincter c Ileo-cecal valve d Middle valve of Huston e Anal valve

18 Ulcer in the posterior wall of the first part of the duodenum would erode ___ artery and would cause bleeding

a Left gastric b Right gastric c Hepatic artery proper d Gastroduodenal artery e Middle colic artery

19 An inflamed appendix is identified by a surgeon on the operation table by noting

a The appendicies epiploicae b The convergence of tenia c The artery of Drummond d The mesocolon e The mesosalphinx

20 The nerve which emerges through the psoas major is

a Femoral b Ilio-inguinal c Ilio-hypogastric d Pudendal e Subcostal

21 The right gonadal vein drains into the

a Azygos b Hemiazygos c Inferior Vena Cava d Right renal vein e Left renal vein

22 The hepatocytes in the liver is derived from

a Ectoderm b Endoderm c Mesoderm

d Neural ectoderm

23 Abscess in the lumbar vertebrae due to tuberculosis would spread to the adjacent muscle which is

a Psoas Major b Iliacus c Quadratus lumborum d Tranversus Abdominis

24 The anterior wall of the inguinal canal is formed by

a External oblique and transverses abdominis b External oblique and fascia transversalis c Internal oblique and external oblique d Internal oblique and transverses abdominis e Fascia transversalis and peritoneum

Meckels diverticulum is a result of which of the following developmental abnormalities shy

A Failure of the vitelline duct to close

B Failure of the herniated intestinal loop to retract into the abdomen

C Failure of the urachus to close

D Failure of the midgut to rotate

E Failure of the hepatic duct to close

Explanation

Meckels diverticulum is a result of the persistence of the proximal part of the vitelline duct This

diverticulum is usually found about 2 feet proximal to the ileocecal junction and is usually about 2 inches

long It is present in about 2 of the popUlation It may be the site of ectopic pancreatic tissue or gastric

mucosa and may develop inflammatory processes and ulcerations Acute Meckels diverticulitis

simulates appendicitis

Which of the following veins carries blood from the esophagus to the portal vein The

A right gastric vein

B left gastric vein c splenic vein D azygos vein

E left gastroepiploic vein

Explanation

The left gastric vein a direct branch of the portal vein drains blood from the lesser curvature of the

stomach and the inferior portion of the esophagus Because branches of the portal vein do not have

valves blood can flow in a retrograde path when there is an obstruction to flow through the portal system or liveL Rlooci Cln then flow from the nortl] vein thr()1Ph the left PRstric vein to the esonhlPlIS lno

through venous communications within the submucosa of the esophagus to esophageal veins that drain

into the azygos vein The increase in blood flow through the esophageal submucosal veins results in esophageal varices

On the posterior wall of the abdomen the celiac ganglion A contains cell bodies of postganglionic parasympathetic neurons B is synapsed upon by neurons in the posterior vagal trunk C is synapsed upon by neurons in the greater splanchnic nerve D contains sensory cell bodies of lumbar spinal nerves E contains cell bodies of neurons that cause an increase in the rate of peristasis

Explanation The celiac ganglion is one of the preaortic ganglia of the sympathetic nervous system It contains cell bodies of postganglionic sympathetic neurons The sympathetic splanchnic nerves contain preganglionic sympathetic neurons that pass through the sympathetic chain without synapsing These splanchnic nerves go to the preaortic ganglia to synapse The greater splanchnic nerve contains preganglionic neurons from spinal cord segments T5-T9 This nerve synapses in the celiac ganglion The nerve fibers in the vagal trunks are preganglionic parasympathetic fibers that go to the walls of the organs that they will innervate and synapse on postganglionic parasympathetic neurons in the walls of those organs Cell bodies of sensory neurons in the abdomen are found in the dorsal root ganglia or the sensory ganglia of the vagus nerve Sympathetic innervation decreases the rate of peristalsis parasympathetic innervation increases the rate of peristalsis

Which of the following pairs of arteries will allow blood to bypass an occlusion of the celiac trunk

A Left gastric artery-right gastric artery

B Left gastroepiploic artery-right gastroepiploic artery

C Superior pancreaticoduodenal artery-inferior pancreaticoduodenal artery

D Splenic artery-common hepatic artery

E Left gastric artery - proper hepatic artery

Explanation The anastoOlosis of a branch of the celiac trunk and a branch of the superior mesenteric artery will

provide collateral circulation around an occlusion of the celiac trunk Each of the other choices pair

branches of the celiac trunk therefore these will not provide collateral flow around the obstruction of the

celiac trunk The left gastric splenic and common hepatic arteries are direct branches of the celiac trunk

The right gastric artery is a branch of the proper hepatic artery which is a branch of the common hepatic artery The left gastroepiploic artery is a branch of the splenic artery The right gastroepiploic artery is a

branch of the gastroduodenal artery whlch is a branch of the common hepatic artery

Which of the following organs has appendices epiploica The

A sigmoid colon

Bjejunum

C duodenum

D stomach E esophagus

Explanation Appendices epiploica are characteristic of the colon Appendices epiploica are subserosal accumulations

of fat None of the organs of the gastrointestinal tract has appendices epiploica except the colon

Page 41: Chirag's Abdomen Review

Ot er than the spleen occlusion Cif the spit-Ilk artery at its odgin wm most likely affect die blood supply to jllch st cnud

(A) Jejunum

(B) Body of th pal1~lltas

(C) LeSStT Cllmiddotlaturc of tl )toma-ch

(D Duodenum dista to the entrance of the Ornmou bile duct

E Fundus of the stomach

A 38-yeu-old batL~er with a history of heartburn suddenly experiences excluciating pain in the (plgastric region of th~ abdomeu SurgCry is perf~rme immediard y upon admisshysion to the 1IlcrgCJliy tuomh~re i~ evidence uf a ruptured ulcer in the posterior waU of the stomach Vhere will a surgeon first fi nd the stomach contenlSf

A) Greater p4ritoneal sac

rB) Cul~de-s~c of Douglas (--

C Omental bursa ~

--D) Paracolic gutter

rEj Between -he panttal perimltum and the posterior body wal1

At birth an infant presents with a st()ma~ rb~tbas~njJled jfltotb~diaplfagru 1A1ltre is the defect thatresulied iiitJle heini~t()n shy~tsophagealbiatus

7 - rH-- Hiatus for the inferior vena cava

( Pleuroperitoneal membrane -(0) Septum transvcrsum

(E) Right Crlt~

An infant born with DOVv7l syndrome presents with bili()u~ vomiting Ahat congenital defect does the infant have

(A) Pyloric stenosis

(B) Meckel diverticulum C) Ornphaloce1e

(D) Gastroschisis

( ~ ) Duodenal atresia y A patient with cirrhosis of the liver presents with ~ bacalvaricestnlreased retrograde pressure in which veins caused the varices

(A) Paraumuilical

(B) Splenic

(ct AzygltJus

(15))G~trk ( (-F) Superior mesemeric

A htaltby 3-year~old male patient experiences a hernial sa protruding from the anterior abdominal wall about halfway between me anterior superior ilia spine and the pubk tuberde Pulsations of al1 artery are palpated medial to the protrusion site through the abdominal walL Which layer of the anterior abdominal wall will first be traversed by the

1hctma

fA) Rectus sheath (B) External oblique aponeurosis

(C) Inguinal ligament

lD) Transversalis fusda

(E) Cremasteric fa~cia

After 5urgi(aj ffpair of a hernia the patient tXperienccs mtmlgtness in the skin on the anteshyrior aspect of the S(Totum_ Vhaf nerve may have been lesioned during thehemiorrhaphy

(A) Femoral

(B) Obturator

(C) Ilioinguinal

(D) lliohypogastrk

(E) Pudendal

A 23~year-LJld female secretary il1 good health ~-uddcn1) doubles over with pain in the a ea of the 1JmbRicu$ Sbe feels vartn and ltneasy and has no appetite That night the pain seems to have mQved to the tower right abdominal regjol1 and she calls her family doctor who then arranges for an ambulance to pk-k her up and take her to the hospitaL Wh ell ntn~ perceived in the area of the urnbilirus most Hkely carried lhe pairfu I sensations into the eNS

tA) Vagus nerves I~

V B)

) Lessersplanchnk nerves

tC) Pudendal nerves

(D) lIiohpogastrk nerves

(E) Greater splam ic l erves

A CT reveals carcinoma in the bOod of the ancreas Vhich blood vessel trut ourses ----~- - -bull ------ --shy

immediately poftterior to the body ofthe pancreas is the m~t likely to be oompressed

(A) Splenk artery

(B) Abdominal aorta (C) Portal vein

(1) Splenic vein

(E) Renal vein

A patient has a penrln1l1ng uker of the posterior wall ot the br~l part ot the (lUooenmn llkh blood vessel is subject to erosion

(A) Common hepatic artery

(B) Gastroouodenal artery

(C) Proper hevatic artery

(D) Celiac artery

(E) Anterior inferior 11amrelltlcoduodcnal attery

Your patient has been diagnosed -ith a carcinoma locallted to the head and l~e(k of the pancreas Another clinical sign would be

A esophageal varices

(8) hemorrhoids

C) a caput medusa

(D) increased pra Teuro n th~ hepatic veins

(E) enlarged right supra lavkular lymph nodes

Wltkh of the foUowing structures develops in the ventral mesentery

(A) Spleen

(B) Jeiunum (C) Head of1ht pancreas (D) Transverse colon (E) Stomach

ti l Uw ~ littwin~ f( S-t lil oai Imdge ~ hi(h or tbt la~)d J truetur tgt liJ llntn nl) he hl p UC iJd [IIi ell

c o

A) drains Ie tht infCrior a La aI

R t middot~nfl0 ~ill to th~ lunlgtn of h i dtlndCrlllfH

(e) m t bull JiJattd on tl l J n T ~H

D ) sup Lc O VSlt I Hlid bhtu l 1 li - -I un oid

( ) U~tpli(t tr j middottUh~ 1 v(( b~nt rfK n1ilc~Zm

ANSWERS AND EXPLANATIONS

Answer E The spleen is t hlttnopodicand lymph organ demlted from mesoderm

Answ~ R Al1 tlmphalocele is caused by it failure of the nlidgut to return to the ahdomir nat cavity after herniation into the umbiliau Stalk Choices Aand D maybe seen in infants with Down syndrome choice D ~s the specific CBuse ofduudcnal JtiCSitt Choice C is (ile cause of gclstrosbisis and Choice B nsults iu a Meurolktldivertku1-tlB

Answer B The fundus ofthe stomach is suppHed by soort gastric brunches of the splenic altery The splenic artery supplies the body and tail of the pancreas part of the greater curvature of the sttmla(h and the spleen Te jejunum part of the head of the pancreas and tht~ duodenum distal to the entrance of the commOll bile duct are supplied by the superior mesenterk artery clll~l ~be less r ctlt1ature cmd the pylQric antrum are supplied by the right and lei gastric art(ries

AnSWftt C Tbeomental bursa or lesser ~ritoneaj sac lies direcdy posterior to the proxshyimal part of the duodeTtlm and the stomach and would be the first site where stomach contents ~Ott1d be fpoundluncL

Answer C A defect in a llleuropcritoneal membrane (uswlly the left) is the typical site of i1 cc-ngenitlI diilphragluatic hemia llere the membr4ne fails to dose ()pound( of the perishycCirdiopcritulleal canals

Answer E DuoJenal atresia and aganglionic megacoion are congwitaI defects S~Il in patients with Dowmiddotnsyndrome

Answer D RulaTgemt~llt of and retrograde flow in g~lstrk vel_ns in particlJl~r the kft gas~ tricveins dilates the capillary bed in rhe wall of the esophagus in (ases of porta yper~

tension Blood flow would increase in and dilampte tribntarkgts of the (lZygOUS vein on the other side of the capiUary bed but flow in this vein is in the typical direction t()ward the superior vena cava Paraumbiii(ltU vein eilgorgement contributes to a caput medusH Splenic ~nlargement might prc~nt with 5plcnonlegaly and balt-kflow in to tlu superior m~~ntclic vein occurs but is asymptomatic

Answer D The patient hagt an indirect inguinal hernia whi~h emerges from the antt-rior abdominal wall through the deep inguinltilling Theeep ring is a fault in the transv~rshysaUs fascia this I~yer wiIJ be penetrated first by the hernia

An~Wer C The ilioinguinal nenc which provides sens~llion to the lnedlal thigh ltmclanteshytior SClotunl pass~lt th rough the 5uperfh_ial inguinal ring ind $subject to inj i1T) becaus-e

it is in the operatitm Held of the erniorrhapny

Auswer B The leMHr splanchnic nerves are sympathdic nerVlts that carry viscera l sensashytlltgtrogt ftom illtllt1m~d ()J stietched gust (itinteitinal ~tructures (in this case the pprndix) into tnt eNS Lesser splanchnic ntTYcsarisc from thmiddot T9--T12 spinal cord segments lt1nd provide sympathetic innenation tD rnidgut siruc1ures whiCh include CLe app~JldD Viscera] Pain arising from affecLed Inidgut ampt 1C1ure is referred over the same dl- matorne~ of spinal segrnertts v-hich provide the sympathetic Innervation n this G1SC of appendicitis the invohen~n t of the ltire) of t e unlhHku indud s the T 10 dermatome

Answer B Of the five choices onty the dscending olon is retroperiton~al aldwould be a lik ~ ( choice to be seen immediately a(~jilcent to t11e posterior abdominal middotn~L

Amwen D The SpltftlC ~-ein ourses posterior to the body of the panneas m its way tt drain into the superior mCSfttltlri( vein

Answcr B TILt glstrodllolticnal artery 1 direct hIamh of the comrootl hepatic artery courses immediately pt))iwri() to the duodenum and is slbject to erosion

Answer B Carcinoma of th pan middott3S in the 1tilt1 may compreampgt the portltil vein at irs orishygill The poTtai vcin is fomled when the splenic vein jQiaswith tfie superior meStllt eric vein The inferiot mesenteric vein joins the ~plenjc vein just priOT to tlli~ point at which the splenic joins the superior Jlleit1ltcri( vein Increescd venous presslu in the inferior mesenteric vein is a cause of emo hoid~

Answer C The- velltral pancreas wilich forms most of the head of the p ~ncr as develops in the ventral mes(ntery as antutgrowth of the hepatic diverticulum Th~ hepatic divershyticulull induding the biIJary appa~atus develops in tbe ventral mesentery of the foregut

Answer~ A The superior mesenteric ~in joins with the spienkvein to form the hepatic portal vciu

Answer D The structure at gttlK is the proper hepatic artery~ whkh suppUesoxygenated b middotood to the liver

MAKE SURE YOU KNOW the diff bw Rectus Sheath above and below the arcuate line

ABOVE

Aponeurosis of xiiltmal obllque musclo

Extemll f)biquw musde

Reotln ilbdomlnls musole S~in

Internal 9bliquQ mY~QI

AponeUfOsi$ of hJH$V~~S Lir9a a lb lbdolTlin~ musolo Tri OJV6 rUi

atldomlnis mUS(loe

Sub cutanlilous tiue (tatty ye r)

BElOW

A POrl lJfosis 01 etemal oblique muscle

Aponeul~)sis 01 Internal oblique mU$cl~

Anteriol lay~ of r~ltdus st~ath EXttom1 oblique rnu$cll

Rectus Jbdominis muscle Intoernal Aponeurc-sis of tra~fersU$ oblique muscle-

at-domlnis muscentl ~ Skio

Tra nsvitSus abdomioLs ml)ZClt

TralSVersaHs fascia Medial umQil iegtt1 1i9Jment -and folj

Uldchus Peritoneum (ir median Umbilj~al Suboutane ous

Extraprftone 11ascia

Ymbilimiddot~1 fold)

preu9poundiea1 fascia

tissue (fatty 4nd m~mbr3n(iUS layers)

o Above the arcuate line (A horizontal line 13 of the distance bw the umbilicus and the

pubic symphysis) -10 Aponeurosis divides into an AntPost Laminae

o The Ant Laminae joins EO and Post Laminae joins Trans Abdominis = Ant and Post

RECTUS SHEATH respectively

o BElOW the arcuate line - all 3 aponeurosis join ANTERIOR to rectus muscle to meet its

counterpart in the midline (linea Alba)

o Take away Msg - The abdomen is devoid of a posterior rectus sheath below the

arcuate line and is therefore more vulnerable to herniasinjuries

Question - A physician makes a deep incision in the patients midline immediately superior to

the pubic symphysis which of the following layers is his knife least likely to pass

Rectus Abdominis External Oblique Ant Rectus Sheath Posterior Rectus Sheath All of the

Above

Answer - All of the above None of the other answer choices are midline structures -LINEA

ALBA

Linea Alba has very poor blood supply - doesnt heal well after surgery Therefore this is a

common site for incisional hernias

a Spleen b Transverse colon c Descending colon d Stomach e Pleura

17 Meckels diverticulum is normally found 2 feet proximal from the

a Pyloric sphincter b Lower esophageal sphincter c Ileo-cecal valve d Middle valve of Huston e Anal valve

18 Ulcer in the posterior wall of the first part of the duodenum would erode ___ artery and would cause bleeding

a Left gastric b Right gastric c Hepatic artery proper d Gastroduodenal artery e Middle colic artery

19 An inflamed appendix is identified by a surgeon on the operation table by noting

a The appendicies epiploicae b The convergence of tenia c The artery of Drummond d The mesocolon e The mesosalphinx

20 The nerve which emerges through the psoas major is

a Femoral b Ilio-inguinal c Ilio-hypogastric d Pudendal e Subcostal

21 The right gonadal vein drains into the

a Azygos b Hemiazygos c Inferior Vena Cava d Right renal vein e Left renal vein

22 The hepatocytes in the liver is derived from

a Ectoderm b Endoderm c Mesoderm

d Neural ectoderm

23 Abscess in the lumbar vertebrae due to tuberculosis would spread to the adjacent muscle which is

a Psoas Major b Iliacus c Quadratus lumborum d Tranversus Abdominis

24 The anterior wall of the inguinal canal is formed by

a External oblique and transverses abdominis b External oblique and fascia transversalis c Internal oblique and external oblique d Internal oblique and transverses abdominis e Fascia transversalis and peritoneum

Meckels diverticulum is a result of which of the following developmental abnormalities shy

A Failure of the vitelline duct to close

B Failure of the herniated intestinal loop to retract into the abdomen

C Failure of the urachus to close

D Failure of the midgut to rotate

E Failure of the hepatic duct to close

Explanation

Meckels diverticulum is a result of the persistence of the proximal part of the vitelline duct This

diverticulum is usually found about 2 feet proximal to the ileocecal junction and is usually about 2 inches

long It is present in about 2 of the popUlation It may be the site of ectopic pancreatic tissue or gastric

mucosa and may develop inflammatory processes and ulcerations Acute Meckels diverticulitis

simulates appendicitis

Which of the following veins carries blood from the esophagus to the portal vein The

A right gastric vein

B left gastric vein c splenic vein D azygos vein

E left gastroepiploic vein

Explanation

The left gastric vein a direct branch of the portal vein drains blood from the lesser curvature of the

stomach and the inferior portion of the esophagus Because branches of the portal vein do not have

valves blood can flow in a retrograde path when there is an obstruction to flow through the portal system or liveL Rlooci Cln then flow from the nortl] vein thr()1Ph the left PRstric vein to the esonhlPlIS lno

through venous communications within the submucosa of the esophagus to esophageal veins that drain

into the azygos vein The increase in blood flow through the esophageal submucosal veins results in esophageal varices

On the posterior wall of the abdomen the celiac ganglion A contains cell bodies of postganglionic parasympathetic neurons B is synapsed upon by neurons in the posterior vagal trunk C is synapsed upon by neurons in the greater splanchnic nerve D contains sensory cell bodies of lumbar spinal nerves E contains cell bodies of neurons that cause an increase in the rate of peristasis

Explanation The celiac ganglion is one of the preaortic ganglia of the sympathetic nervous system It contains cell bodies of postganglionic sympathetic neurons The sympathetic splanchnic nerves contain preganglionic sympathetic neurons that pass through the sympathetic chain without synapsing These splanchnic nerves go to the preaortic ganglia to synapse The greater splanchnic nerve contains preganglionic neurons from spinal cord segments T5-T9 This nerve synapses in the celiac ganglion The nerve fibers in the vagal trunks are preganglionic parasympathetic fibers that go to the walls of the organs that they will innervate and synapse on postganglionic parasympathetic neurons in the walls of those organs Cell bodies of sensory neurons in the abdomen are found in the dorsal root ganglia or the sensory ganglia of the vagus nerve Sympathetic innervation decreases the rate of peristalsis parasympathetic innervation increases the rate of peristalsis

Which of the following pairs of arteries will allow blood to bypass an occlusion of the celiac trunk

A Left gastric artery-right gastric artery

B Left gastroepiploic artery-right gastroepiploic artery

C Superior pancreaticoduodenal artery-inferior pancreaticoduodenal artery

D Splenic artery-common hepatic artery

E Left gastric artery - proper hepatic artery

Explanation The anastoOlosis of a branch of the celiac trunk and a branch of the superior mesenteric artery will

provide collateral circulation around an occlusion of the celiac trunk Each of the other choices pair

branches of the celiac trunk therefore these will not provide collateral flow around the obstruction of the

celiac trunk The left gastric splenic and common hepatic arteries are direct branches of the celiac trunk

The right gastric artery is a branch of the proper hepatic artery which is a branch of the common hepatic artery The left gastroepiploic artery is a branch of the splenic artery The right gastroepiploic artery is a

branch of the gastroduodenal artery whlch is a branch of the common hepatic artery

Which of the following organs has appendices epiploica The

A sigmoid colon

Bjejunum

C duodenum

D stomach E esophagus

Explanation Appendices epiploica are characteristic of the colon Appendices epiploica are subserosal accumulations

of fat None of the organs of the gastrointestinal tract has appendices epiploica except the colon

Page 42: Chirag's Abdomen Review

A htaltby 3-year~old male patient experiences a hernial sa protruding from the anterior abdominal wall about halfway between me anterior superior ilia spine and the pubk tuberde Pulsations of al1 artery are palpated medial to the protrusion site through the abdominal walL Which layer of the anterior abdominal wall will first be traversed by the

1hctma

fA) Rectus sheath (B) External oblique aponeurosis

(C) Inguinal ligament

lD) Transversalis fusda

(E) Cremasteric fa~cia

After 5urgi(aj ffpair of a hernia the patient tXperienccs mtmlgtness in the skin on the anteshyrior aspect of the S(Totum_ Vhaf nerve may have been lesioned during thehemiorrhaphy

(A) Femoral

(B) Obturator

(C) Ilioinguinal

(D) lliohypogastrk

(E) Pudendal

A 23~year-LJld female secretary il1 good health ~-uddcn1) doubles over with pain in the a ea of the 1JmbRicu$ Sbe feels vartn and ltneasy and has no appetite That night the pain seems to have mQved to the tower right abdominal regjol1 and she calls her family doctor who then arranges for an ambulance to pk-k her up and take her to the hospitaL Wh ell ntn~ perceived in the area of the urnbilirus most Hkely carried lhe pairfu I sensations into the eNS

tA) Vagus nerves I~

V B)

) Lessersplanchnk nerves

tC) Pudendal nerves

(D) lIiohpogastrk nerves

(E) Greater splam ic l erves

A CT reveals carcinoma in the bOod of the ancreas Vhich blood vessel trut ourses ----~- - -bull ------ --shy

immediately poftterior to the body ofthe pancreas is the m~t likely to be oompressed

(A) Splenk artery

(B) Abdominal aorta (C) Portal vein

(1) Splenic vein

(E) Renal vein

A patient has a penrln1l1ng uker of the posterior wall ot the br~l part ot the (lUooenmn llkh blood vessel is subject to erosion

(A) Common hepatic artery

(B) Gastroouodenal artery

(C) Proper hevatic artery

(D) Celiac artery

(E) Anterior inferior 11amrelltlcoduodcnal attery

Your patient has been diagnosed -ith a carcinoma locallted to the head and l~e(k of the pancreas Another clinical sign would be

A esophageal varices

(8) hemorrhoids

C) a caput medusa

(D) increased pra Teuro n th~ hepatic veins

(E) enlarged right supra lavkular lymph nodes

Wltkh of the foUowing structures develops in the ventral mesentery

(A) Spleen

(B) Jeiunum (C) Head of1ht pancreas (D) Transverse colon (E) Stomach

ti l Uw ~ littwin~ f( S-t lil oai Imdge ~ hi(h or tbt la~)d J truetur tgt liJ llntn nl) he hl p UC iJd [IIi ell

c o

A) drains Ie tht infCrior a La aI

R t middot~nfl0 ~ill to th~ lunlgtn of h i dtlndCrlllfH

(e) m t bull JiJattd on tl l J n T ~H

D ) sup Lc O VSlt I Hlid bhtu l 1 li - -I un oid

( ) U~tpli(t tr j middottUh~ 1 v(( b~nt rfK n1ilc~Zm

ANSWERS AND EXPLANATIONS

Answer E The spleen is t hlttnopodicand lymph organ demlted from mesoderm

Answ~ R Al1 tlmphalocele is caused by it failure of the nlidgut to return to the ahdomir nat cavity after herniation into the umbiliau Stalk Choices Aand D maybe seen in infants with Down syndrome choice D ~s the specific CBuse ofduudcnal JtiCSitt Choice C is (ile cause of gclstrosbisis and Choice B nsults iu a Meurolktldivertku1-tlB

Answer B The fundus ofthe stomach is suppHed by soort gastric brunches of the splenic altery The splenic artery supplies the body and tail of the pancreas part of the greater curvature of the sttmla(h and the spleen Te jejunum part of the head of the pancreas and tht~ duodenum distal to the entrance of the commOll bile duct are supplied by the superior mesenterk artery clll~l ~be less r ctlt1ature cmd the pylQric antrum are supplied by the right and lei gastric art(ries

AnSWftt C Tbeomental bursa or lesser ~ritoneaj sac lies direcdy posterior to the proxshyimal part of the duodeTtlm and the stomach and would be the first site where stomach contents ~Ott1d be fpoundluncL

Answer C A defect in a llleuropcritoneal membrane (uswlly the left) is the typical site of i1 cc-ngenitlI diilphragluatic hemia llere the membr4ne fails to dose ()pound( of the perishycCirdiopcritulleal canals

Answer E DuoJenal atresia and aganglionic megacoion are congwitaI defects S~Il in patients with Dowmiddotnsyndrome

Answer D RulaTgemt~llt of and retrograde flow in g~lstrk vel_ns in particlJl~r the kft gas~ tricveins dilates the capillary bed in rhe wall of the esophagus in (ases of porta yper~

tension Blood flow would increase in and dilampte tribntarkgts of the (lZygOUS vein on the other side of the capiUary bed but flow in this vein is in the typical direction t()ward the superior vena cava Paraumbiii(ltU vein eilgorgement contributes to a caput medusH Splenic ~nlargement might prc~nt with 5plcnonlegaly and balt-kflow in to tlu superior m~~ntclic vein occurs but is asymptomatic

Answer D The patient hagt an indirect inguinal hernia whi~h emerges from the antt-rior abdominal wall through the deep inguinltilling Theeep ring is a fault in the transv~rshysaUs fascia this I~yer wiIJ be penetrated first by the hernia

An~Wer C The ilioinguinal nenc which provides sens~llion to the lnedlal thigh ltmclanteshytior SClotunl pass~lt th rough the 5uperfh_ial inguinal ring ind $subject to inj i1T) becaus-e

it is in the operatitm Held of the erniorrhapny

Auswer B The leMHr splanchnic nerves are sympathdic nerVlts that carry viscera l sensashytlltgtrogt ftom illtllt1m~d ()J stietched gust (itinteitinal ~tructures (in this case the pprndix) into tnt eNS Lesser splanchnic ntTYcsarisc from thmiddot T9--T12 spinal cord segments lt1nd provide sympathetic innenation tD rnidgut siruc1ures whiCh include CLe app~JldD Viscera] Pain arising from affecLed Inidgut ampt 1C1ure is referred over the same dl- matorne~ of spinal segrnertts v-hich provide the sympathetic Innervation n this G1SC of appendicitis the invohen~n t of the ltire) of t e unlhHku indud s the T 10 dermatome

Answer B Of the five choices onty the dscending olon is retroperiton~al aldwould be a lik ~ ( choice to be seen immediately a(~jilcent to t11e posterior abdominal middotn~L

Amwen D The SpltftlC ~-ein ourses posterior to the body of the panneas m its way tt drain into the superior mCSfttltlri( vein

Answcr B TILt glstrodllolticnal artery 1 direct hIamh of the comrootl hepatic artery courses immediately pt))iwri() to the duodenum and is slbject to erosion

Answer B Carcinoma of th pan middott3S in the 1tilt1 may compreampgt the portltil vein at irs orishygill The poTtai vcin is fomled when the splenic vein jQiaswith tfie superior meStllt eric vein The inferiot mesenteric vein joins the ~plenjc vein just priOT to tlli~ point at which the splenic joins the superior Jlleit1ltcri( vein Increescd venous presslu in the inferior mesenteric vein is a cause of emo hoid~

Answer C The- velltral pancreas wilich forms most of the head of the p ~ncr as develops in the ventral mes(ntery as antutgrowth of the hepatic diverticulum Th~ hepatic divershyticulull induding the biIJary appa~atus develops in tbe ventral mesentery of the foregut

Answer~ A The superior mesenteric ~in joins with the spienkvein to form the hepatic portal vciu

Answer D The structure at gttlK is the proper hepatic artery~ whkh suppUesoxygenated b middotood to the liver

MAKE SURE YOU KNOW the diff bw Rectus Sheath above and below the arcuate line

ABOVE

Aponeurosis of xiiltmal obllque musclo

Extemll f)biquw musde

Reotln ilbdomlnls musole S~in

Internal 9bliquQ mY~QI

AponeUfOsi$ of hJH$V~~S Lir9a a lb lbdolTlin~ musolo Tri OJV6 rUi

atldomlnis mUS(loe

Sub cutanlilous tiue (tatty ye r)

BElOW

A POrl lJfosis 01 etemal oblique muscle

Aponeul~)sis 01 Internal oblique mU$cl~

Anteriol lay~ of r~ltdus st~ath EXttom1 oblique rnu$cll

Rectus Jbdominis muscle Intoernal Aponeurc-sis of tra~fersU$ oblique muscle-

at-domlnis muscentl ~ Skio

Tra nsvitSus abdomioLs ml)ZClt

TralSVersaHs fascia Medial umQil iegtt1 1i9Jment -and folj

Uldchus Peritoneum (ir median Umbilj~al Suboutane ous

Extraprftone 11ascia

Ymbilimiddot~1 fold)

preu9poundiea1 fascia

tissue (fatty 4nd m~mbr3n(iUS layers)

o Above the arcuate line (A horizontal line 13 of the distance bw the umbilicus and the

pubic symphysis) -10 Aponeurosis divides into an AntPost Laminae

o The Ant Laminae joins EO and Post Laminae joins Trans Abdominis = Ant and Post

RECTUS SHEATH respectively

o BElOW the arcuate line - all 3 aponeurosis join ANTERIOR to rectus muscle to meet its

counterpart in the midline (linea Alba)

o Take away Msg - The abdomen is devoid of a posterior rectus sheath below the

arcuate line and is therefore more vulnerable to herniasinjuries

Question - A physician makes a deep incision in the patients midline immediately superior to

the pubic symphysis which of the following layers is his knife least likely to pass

Rectus Abdominis External Oblique Ant Rectus Sheath Posterior Rectus Sheath All of the

Above

Answer - All of the above None of the other answer choices are midline structures -LINEA

ALBA

Linea Alba has very poor blood supply - doesnt heal well after surgery Therefore this is a

common site for incisional hernias

a Spleen b Transverse colon c Descending colon d Stomach e Pleura

17 Meckels diverticulum is normally found 2 feet proximal from the

a Pyloric sphincter b Lower esophageal sphincter c Ileo-cecal valve d Middle valve of Huston e Anal valve

18 Ulcer in the posterior wall of the first part of the duodenum would erode ___ artery and would cause bleeding

a Left gastric b Right gastric c Hepatic artery proper d Gastroduodenal artery e Middle colic artery

19 An inflamed appendix is identified by a surgeon on the operation table by noting

a The appendicies epiploicae b The convergence of tenia c The artery of Drummond d The mesocolon e The mesosalphinx

20 The nerve which emerges through the psoas major is

a Femoral b Ilio-inguinal c Ilio-hypogastric d Pudendal e Subcostal

21 The right gonadal vein drains into the

a Azygos b Hemiazygos c Inferior Vena Cava d Right renal vein e Left renal vein

22 The hepatocytes in the liver is derived from

a Ectoderm b Endoderm c Mesoderm

d Neural ectoderm

23 Abscess in the lumbar vertebrae due to tuberculosis would spread to the adjacent muscle which is

a Psoas Major b Iliacus c Quadratus lumborum d Tranversus Abdominis

24 The anterior wall of the inguinal canal is formed by

a External oblique and transverses abdominis b External oblique and fascia transversalis c Internal oblique and external oblique d Internal oblique and transverses abdominis e Fascia transversalis and peritoneum

Meckels diverticulum is a result of which of the following developmental abnormalities shy

A Failure of the vitelline duct to close

B Failure of the herniated intestinal loop to retract into the abdomen

C Failure of the urachus to close

D Failure of the midgut to rotate

E Failure of the hepatic duct to close

Explanation

Meckels diverticulum is a result of the persistence of the proximal part of the vitelline duct This

diverticulum is usually found about 2 feet proximal to the ileocecal junction and is usually about 2 inches

long It is present in about 2 of the popUlation It may be the site of ectopic pancreatic tissue or gastric

mucosa and may develop inflammatory processes and ulcerations Acute Meckels diverticulitis

simulates appendicitis

Which of the following veins carries blood from the esophagus to the portal vein The

A right gastric vein

B left gastric vein c splenic vein D azygos vein

E left gastroepiploic vein

Explanation

The left gastric vein a direct branch of the portal vein drains blood from the lesser curvature of the

stomach and the inferior portion of the esophagus Because branches of the portal vein do not have

valves blood can flow in a retrograde path when there is an obstruction to flow through the portal system or liveL Rlooci Cln then flow from the nortl] vein thr()1Ph the left PRstric vein to the esonhlPlIS lno

through venous communications within the submucosa of the esophagus to esophageal veins that drain

into the azygos vein The increase in blood flow through the esophageal submucosal veins results in esophageal varices

On the posterior wall of the abdomen the celiac ganglion A contains cell bodies of postganglionic parasympathetic neurons B is synapsed upon by neurons in the posterior vagal trunk C is synapsed upon by neurons in the greater splanchnic nerve D contains sensory cell bodies of lumbar spinal nerves E contains cell bodies of neurons that cause an increase in the rate of peristasis

Explanation The celiac ganglion is one of the preaortic ganglia of the sympathetic nervous system It contains cell bodies of postganglionic sympathetic neurons The sympathetic splanchnic nerves contain preganglionic sympathetic neurons that pass through the sympathetic chain without synapsing These splanchnic nerves go to the preaortic ganglia to synapse The greater splanchnic nerve contains preganglionic neurons from spinal cord segments T5-T9 This nerve synapses in the celiac ganglion The nerve fibers in the vagal trunks are preganglionic parasympathetic fibers that go to the walls of the organs that they will innervate and synapse on postganglionic parasympathetic neurons in the walls of those organs Cell bodies of sensory neurons in the abdomen are found in the dorsal root ganglia or the sensory ganglia of the vagus nerve Sympathetic innervation decreases the rate of peristalsis parasympathetic innervation increases the rate of peristalsis

Which of the following pairs of arteries will allow blood to bypass an occlusion of the celiac trunk

A Left gastric artery-right gastric artery

B Left gastroepiploic artery-right gastroepiploic artery

C Superior pancreaticoduodenal artery-inferior pancreaticoduodenal artery

D Splenic artery-common hepatic artery

E Left gastric artery - proper hepatic artery

Explanation The anastoOlosis of a branch of the celiac trunk and a branch of the superior mesenteric artery will

provide collateral circulation around an occlusion of the celiac trunk Each of the other choices pair

branches of the celiac trunk therefore these will not provide collateral flow around the obstruction of the

celiac trunk The left gastric splenic and common hepatic arteries are direct branches of the celiac trunk

The right gastric artery is a branch of the proper hepatic artery which is a branch of the common hepatic artery The left gastroepiploic artery is a branch of the splenic artery The right gastroepiploic artery is a

branch of the gastroduodenal artery whlch is a branch of the common hepatic artery

Which of the following organs has appendices epiploica The

A sigmoid colon

Bjejunum

C duodenum

D stomach E esophagus

Explanation Appendices epiploica are characteristic of the colon Appendices epiploica are subserosal accumulations

of fat None of the organs of the gastrointestinal tract has appendices epiploica except the colon

Page 43: Chirag's Abdomen Review

A patient has a penrln1l1ng uker of the posterior wall ot the br~l part ot the (lUooenmn llkh blood vessel is subject to erosion

(A) Common hepatic artery

(B) Gastroouodenal artery

(C) Proper hevatic artery

(D) Celiac artery

(E) Anterior inferior 11amrelltlcoduodcnal attery

Your patient has been diagnosed -ith a carcinoma locallted to the head and l~e(k of the pancreas Another clinical sign would be

A esophageal varices

(8) hemorrhoids

C) a caput medusa

(D) increased pra Teuro n th~ hepatic veins

(E) enlarged right supra lavkular lymph nodes

Wltkh of the foUowing structures develops in the ventral mesentery

(A) Spleen

(B) Jeiunum (C) Head of1ht pancreas (D) Transverse colon (E) Stomach

ti l Uw ~ littwin~ f( S-t lil oai Imdge ~ hi(h or tbt la~)d J truetur tgt liJ llntn nl) he hl p UC iJd [IIi ell

c o

A) drains Ie tht infCrior a La aI

R t middot~nfl0 ~ill to th~ lunlgtn of h i dtlndCrlllfH

(e) m t bull JiJattd on tl l J n T ~H

D ) sup Lc O VSlt I Hlid bhtu l 1 li - -I un oid

( ) U~tpli(t tr j middottUh~ 1 v(( b~nt rfK n1ilc~Zm

ANSWERS AND EXPLANATIONS

Answer E The spleen is t hlttnopodicand lymph organ demlted from mesoderm

Answ~ R Al1 tlmphalocele is caused by it failure of the nlidgut to return to the ahdomir nat cavity after herniation into the umbiliau Stalk Choices Aand D maybe seen in infants with Down syndrome choice D ~s the specific CBuse ofduudcnal JtiCSitt Choice C is (ile cause of gclstrosbisis and Choice B nsults iu a Meurolktldivertku1-tlB

Answer B The fundus ofthe stomach is suppHed by soort gastric brunches of the splenic altery The splenic artery supplies the body and tail of the pancreas part of the greater curvature of the sttmla(h and the spleen Te jejunum part of the head of the pancreas and tht~ duodenum distal to the entrance of the commOll bile duct are supplied by the superior mesenterk artery clll~l ~be less r ctlt1ature cmd the pylQric antrum are supplied by the right and lei gastric art(ries

AnSWftt C Tbeomental bursa or lesser ~ritoneaj sac lies direcdy posterior to the proxshyimal part of the duodeTtlm and the stomach and would be the first site where stomach contents ~Ott1d be fpoundluncL

Answer C A defect in a llleuropcritoneal membrane (uswlly the left) is the typical site of i1 cc-ngenitlI diilphragluatic hemia llere the membr4ne fails to dose ()pound( of the perishycCirdiopcritulleal canals

Answer E DuoJenal atresia and aganglionic megacoion are congwitaI defects S~Il in patients with Dowmiddotnsyndrome

Answer D RulaTgemt~llt of and retrograde flow in g~lstrk vel_ns in particlJl~r the kft gas~ tricveins dilates the capillary bed in rhe wall of the esophagus in (ases of porta yper~

tension Blood flow would increase in and dilampte tribntarkgts of the (lZygOUS vein on the other side of the capiUary bed but flow in this vein is in the typical direction t()ward the superior vena cava Paraumbiii(ltU vein eilgorgement contributes to a caput medusH Splenic ~nlargement might prc~nt with 5plcnonlegaly and balt-kflow in to tlu superior m~~ntclic vein occurs but is asymptomatic

Answer D The patient hagt an indirect inguinal hernia whi~h emerges from the antt-rior abdominal wall through the deep inguinltilling Theeep ring is a fault in the transv~rshysaUs fascia this I~yer wiIJ be penetrated first by the hernia

An~Wer C The ilioinguinal nenc which provides sens~llion to the lnedlal thigh ltmclanteshytior SClotunl pass~lt th rough the 5uperfh_ial inguinal ring ind $subject to inj i1T) becaus-e

it is in the operatitm Held of the erniorrhapny

Auswer B The leMHr splanchnic nerves are sympathdic nerVlts that carry viscera l sensashytlltgtrogt ftom illtllt1m~d ()J stietched gust (itinteitinal ~tructures (in this case the pprndix) into tnt eNS Lesser splanchnic ntTYcsarisc from thmiddot T9--T12 spinal cord segments lt1nd provide sympathetic innenation tD rnidgut siruc1ures whiCh include CLe app~JldD Viscera] Pain arising from affecLed Inidgut ampt 1C1ure is referred over the same dl- matorne~ of spinal segrnertts v-hich provide the sympathetic Innervation n this G1SC of appendicitis the invohen~n t of the ltire) of t e unlhHku indud s the T 10 dermatome

Answer B Of the five choices onty the dscending olon is retroperiton~al aldwould be a lik ~ ( choice to be seen immediately a(~jilcent to t11e posterior abdominal middotn~L

Amwen D The SpltftlC ~-ein ourses posterior to the body of the panneas m its way tt drain into the superior mCSfttltlri( vein

Answcr B TILt glstrodllolticnal artery 1 direct hIamh of the comrootl hepatic artery courses immediately pt))iwri() to the duodenum and is slbject to erosion

Answer B Carcinoma of th pan middott3S in the 1tilt1 may compreampgt the portltil vein at irs orishygill The poTtai vcin is fomled when the splenic vein jQiaswith tfie superior meStllt eric vein The inferiot mesenteric vein joins the ~plenjc vein just priOT to tlli~ point at which the splenic joins the superior Jlleit1ltcri( vein Increescd venous presslu in the inferior mesenteric vein is a cause of emo hoid~

Answer C The- velltral pancreas wilich forms most of the head of the p ~ncr as develops in the ventral mes(ntery as antutgrowth of the hepatic diverticulum Th~ hepatic divershyticulull induding the biIJary appa~atus develops in tbe ventral mesentery of the foregut

Answer~ A The superior mesenteric ~in joins with the spienkvein to form the hepatic portal vciu

Answer D The structure at gttlK is the proper hepatic artery~ whkh suppUesoxygenated b middotood to the liver

MAKE SURE YOU KNOW the diff bw Rectus Sheath above and below the arcuate line

ABOVE

Aponeurosis of xiiltmal obllque musclo

Extemll f)biquw musde

Reotln ilbdomlnls musole S~in

Internal 9bliquQ mY~QI

AponeUfOsi$ of hJH$V~~S Lir9a a lb lbdolTlin~ musolo Tri OJV6 rUi

atldomlnis mUS(loe

Sub cutanlilous tiue (tatty ye r)

BElOW

A POrl lJfosis 01 etemal oblique muscle

Aponeul~)sis 01 Internal oblique mU$cl~

Anteriol lay~ of r~ltdus st~ath EXttom1 oblique rnu$cll

Rectus Jbdominis muscle Intoernal Aponeurc-sis of tra~fersU$ oblique muscle-

at-domlnis muscentl ~ Skio

Tra nsvitSus abdomioLs ml)ZClt

TralSVersaHs fascia Medial umQil iegtt1 1i9Jment -and folj

Uldchus Peritoneum (ir median Umbilj~al Suboutane ous

Extraprftone 11ascia

Ymbilimiddot~1 fold)

preu9poundiea1 fascia

tissue (fatty 4nd m~mbr3n(iUS layers)

o Above the arcuate line (A horizontal line 13 of the distance bw the umbilicus and the

pubic symphysis) -10 Aponeurosis divides into an AntPost Laminae

o The Ant Laminae joins EO and Post Laminae joins Trans Abdominis = Ant and Post

RECTUS SHEATH respectively

o BElOW the arcuate line - all 3 aponeurosis join ANTERIOR to rectus muscle to meet its

counterpart in the midline (linea Alba)

o Take away Msg - The abdomen is devoid of a posterior rectus sheath below the

arcuate line and is therefore more vulnerable to herniasinjuries

Question - A physician makes a deep incision in the patients midline immediately superior to

the pubic symphysis which of the following layers is his knife least likely to pass

Rectus Abdominis External Oblique Ant Rectus Sheath Posterior Rectus Sheath All of the

Above

Answer - All of the above None of the other answer choices are midline structures -LINEA

ALBA

Linea Alba has very poor blood supply - doesnt heal well after surgery Therefore this is a

common site for incisional hernias

a Spleen b Transverse colon c Descending colon d Stomach e Pleura

17 Meckels diverticulum is normally found 2 feet proximal from the

a Pyloric sphincter b Lower esophageal sphincter c Ileo-cecal valve d Middle valve of Huston e Anal valve

18 Ulcer in the posterior wall of the first part of the duodenum would erode ___ artery and would cause bleeding

a Left gastric b Right gastric c Hepatic artery proper d Gastroduodenal artery e Middle colic artery

19 An inflamed appendix is identified by a surgeon on the operation table by noting

a The appendicies epiploicae b The convergence of tenia c The artery of Drummond d The mesocolon e The mesosalphinx

20 The nerve which emerges through the psoas major is

a Femoral b Ilio-inguinal c Ilio-hypogastric d Pudendal e Subcostal

21 The right gonadal vein drains into the

a Azygos b Hemiazygos c Inferior Vena Cava d Right renal vein e Left renal vein

22 The hepatocytes in the liver is derived from

a Ectoderm b Endoderm c Mesoderm

d Neural ectoderm

23 Abscess in the lumbar vertebrae due to tuberculosis would spread to the adjacent muscle which is

a Psoas Major b Iliacus c Quadratus lumborum d Tranversus Abdominis

24 The anterior wall of the inguinal canal is formed by

a External oblique and transverses abdominis b External oblique and fascia transversalis c Internal oblique and external oblique d Internal oblique and transverses abdominis e Fascia transversalis and peritoneum

Meckels diverticulum is a result of which of the following developmental abnormalities shy

A Failure of the vitelline duct to close

B Failure of the herniated intestinal loop to retract into the abdomen

C Failure of the urachus to close

D Failure of the midgut to rotate

E Failure of the hepatic duct to close

Explanation

Meckels diverticulum is a result of the persistence of the proximal part of the vitelline duct This

diverticulum is usually found about 2 feet proximal to the ileocecal junction and is usually about 2 inches

long It is present in about 2 of the popUlation It may be the site of ectopic pancreatic tissue or gastric

mucosa and may develop inflammatory processes and ulcerations Acute Meckels diverticulitis

simulates appendicitis

Which of the following veins carries blood from the esophagus to the portal vein The

A right gastric vein

B left gastric vein c splenic vein D azygos vein

E left gastroepiploic vein

Explanation

The left gastric vein a direct branch of the portal vein drains blood from the lesser curvature of the

stomach and the inferior portion of the esophagus Because branches of the portal vein do not have

valves blood can flow in a retrograde path when there is an obstruction to flow through the portal system or liveL Rlooci Cln then flow from the nortl] vein thr()1Ph the left PRstric vein to the esonhlPlIS lno

through venous communications within the submucosa of the esophagus to esophageal veins that drain

into the azygos vein The increase in blood flow through the esophageal submucosal veins results in esophageal varices

On the posterior wall of the abdomen the celiac ganglion A contains cell bodies of postganglionic parasympathetic neurons B is synapsed upon by neurons in the posterior vagal trunk C is synapsed upon by neurons in the greater splanchnic nerve D contains sensory cell bodies of lumbar spinal nerves E contains cell bodies of neurons that cause an increase in the rate of peristasis

Explanation The celiac ganglion is one of the preaortic ganglia of the sympathetic nervous system It contains cell bodies of postganglionic sympathetic neurons The sympathetic splanchnic nerves contain preganglionic sympathetic neurons that pass through the sympathetic chain without synapsing These splanchnic nerves go to the preaortic ganglia to synapse The greater splanchnic nerve contains preganglionic neurons from spinal cord segments T5-T9 This nerve synapses in the celiac ganglion The nerve fibers in the vagal trunks are preganglionic parasympathetic fibers that go to the walls of the organs that they will innervate and synapse on postganglionic parasympathetic neurons in the walls of those organs Cell bodies of sensory neurons in the abdomen are found in the dorsal root ganglia or the sensory ganglia of the vagus nerve Sympathetic innervation decreases the rate of peristalsis parasympathetic innervation increases the rate of peristalsis

Which of the following pairs of arteries will allow blood to bypass an occlusion of the celiac trunk

A Left gastric artery-right gastric artery

B Left gastroepiploic artery-right gastroepiploic artery

C Superior pancreaticoduodenal artery-inferior pancreaticoduodenal artery

D Splenic artery-common hepatic artery

E Left gastric artery - proper hepatic artery

Explanation The anastoOlosis of a branch of the celiac trunk and a branch of the superior mesenteric artery will

provide collateral circulation around an occlusion of the celiac trunk Each of the other choices pair

branches of the celiac trunk therefore these will not provide collateral flow around the obstruction of the

celiac trunk The left gastric splenic and common hepatic arteries are direct branches of the celiac trunk

The right gastric artery is a branch of the proper hepatic artery which is a branch of the common hepatic artery The left gastroepiploic artery is a branch of the splenic artery The right gastroepiploic artery is a

branch of the gastroduodenal artery whlch is a branch of the common hepatic artery

Which of the following organs has appendices epiploica The

A sigmoid colon

Bjejunum

C duodenum

D stomach E esophagus

Explanation Appendices epiploica are characteristic of the colon Appendices epiploica are subserosal accumulations

of fat None of the organs of the gastrointestinal tract has appendices epiploica except the colon

Page 44: Chirag's Abdomen Review

A) drains Ie tht infCrior a La aI

R t middot~nfl0 ~ill to th~ lunlgtn of h i dtlndCrlllfH

(e) m t bull JiJattd on tl l J n T ~H

D ) sup Lc O VSlt I Hlid bhtu l 1 li - -I un oid

( ) U~tpli(t tr j middottUh~ 1 v(( b~nt rfK n1ilc~Zm

ANSWERS AND EXPLANATIONS

Answer E The spleen is t hlttnopodicand lymph organ demlted from mesoderm

Answ~ R Al1 tlmphalocele is caused by it failure of the nlidgut to return to the ahdomir nat cavity after herniation into the umbiliau Stalk Choices Aand D maybe seen in infants with Down syndrome choice D ~s the specific CBuse ofduudcnal JtiCSitt Choice C is (ile cause of gclstrosbisis and Choice B nsults iu a Meurolktldivertku1-tlB

Answer B The fundus ofthe stomach is suppHed by soort gastric brunches of the splenic altery The splenic artery supplies the body and tail of the pancreas part of the greater curvature of the sttmla(h and the spleen Te jejunum part of the head of the pancreas and tht~ duodenum distal to the entrance of the commOll bile duct are supplied by the superior mesenterk artery clll~l ~be less r ctlt1ature cmd the pylQric antrum are supplied by the right and lei gastric art(ries

AnSWftt C Tbeomental bursa or lesser ~ritoneaj sac lies direcdy posterior to the proxshyimal part of the duodeTtlm and the stomach and would be the first site where stomach contents ~Ott1d be fpoundluncL

Answer C A defect in a llleuropcritoneal membrane (uswlly the left) is the typical site of i1 cc-ngenitlI diilphragluatic hemia llere the membr4ne fails to dose ()pound( of the perishycCirdiopcritulleal canals

Answer E DuoJenal atresia and aganglionic megacoion are congwitaI defects S~Il in patients with Dowmiddotnsyndrome

Answer D RulaTgemt~llt of and retrograde flow in g~lstrk vel_ns in particlJl~r the kft gas~ tricveins dilates the capillary bed in rhe wall of the esophagus in (ases of porta yper~

tension Blood flow would increase in and dilampte tribntarkgts of the (lZygOUS vein on the other side of the capiUary bed but flow in this vein is in the typical direction t()ward the superior vena cava Paraumbiii(ltU vein eilgorgement contributes to a caput medusH Splenic ~nlargement might prc~nt with 5plcnonlegaly and balt-kflow in to tlu superior m~~ntclic vein occurs but is asymptomatic

Answer D The patient hagt an indirect inguinal hernia whi~h emerges from the antt-rior abdominal wall through the deep inguinltilling Theeep ring is a fault in the transv~rshysaUs fascia this I~yer wiIJ be penetrated first by the hernia

An~Wer C The ilioinguinal nenc which provides sens~llion to the lnedlal thigh ltmclanteshytior SClotunl pass~lt th rough the 5uperfh_ial inguinal ring ind $subject to inj i1T) becaus-e

it is in the operatitm Held of the erniorrhapny

Auswer B The leMHr splanchnic nerves are sympathdic nerVlts that carry viscera l sensashytlltgtrogt ftom illtllt1m~d ()J stietched gust (itinteitinal ~tructures (in this case the pprndix) into tnt eNS Lesser splanchnic ntTYcsarisc from thmiddot T9--T12 spinal cord segments lt1nd provide sympathetic innenation tD rnidgut siruc1ures whiCh include CLe app~JldD Viscera] Pain arising from affecLed Inidgut ampt 1C1ure is referred over the same dl- matorne~ of spinal segrnertts v-hich provide the sympathetic Innervation n this G1SC of appendicitis the invohen~n t of the ltire) of t e unlhHku indud s the T 10 dermatome

Answer B Of the five choices onty the dscending olon is retroperiton~al aldwould be a lik ~ ( choice to be seen immediately a(~jilcent to t11e posterior abdominal middotn~L

Amwen D The SpltftlC ~-ein ourses posterior to the body of the panneas m its way tt drain into the superior mCSfttltlri( vein

Answcr B TILt glstrodllolticnal artery 1 direct hIamh of the comrootl hepatic artery courses immediately pt))iwri() to the duodenum and is slbject to erosion

Answer B Carcinoma of th pan middott3S in the 1tilt1 may compreampgt the portltil vein at irs orishygill The poTtai vcin is fomled when the splenic vein jQiaswith tfie superior meStllt eric vein The inferiot mesenteric vein joins the ~plenjc vein just priOT to tlli~ point at which the splenic joins the superior Jlleit1ltcri( vein Increescd venous presslu in the inferior mesenteric vein is a cause of emo hoid~

Answer C The- velltral pancreas wilich forms most of the head of the p ~ncr as develops in the ventral mes(ntery as antutgrowth of the hepatic diverticulum Th~ hepatic divershyticulull induding the biIJary appa~atus develops in tbe ventral mesentery of the foregut

Answer~ A The superior mesenteric ~in joins with the spienkvein to form the hepatic portal vciu

Answer D The structure at gttlK is the proper hepatic artery~ whkh suppUesoxygenated b middotood to the liver

MAKE SURE YOU KNOW the diff bw Rectus Sheath above and below the arcuate line

ABOVE

Aponeurosis of xiiltmal obllque musclo

Extemll f)biquw musde

Reotln ilbdomlnls musole S~in

Internal 9bliquQ mY~QI

AponeUfOsi$ of hJH$V~~S Lir9a a lb lbdolTlin~ musolo Tri OJV6 rUi

atldomlnis mUS(loe

Sub cutanlilous tiue (tatty ye r)

BElOW

A POrl lJfosis 01 etemal oblique muscle

Aponeul~)sis 01 Internal oblique mU$cl~

Anteriol lay~ of r~ltdus st~ath EXttom1 oblique rnu$cll

Rectus Jbdominis muscle Intoernal Aponeurc-sis of tra~fersU$ oblique muscle-

at-domlnis muscentl ~ Skio

Tra nsvitSus abdomioLs ml)ZClt

TralSVersaHs fascia Medial umQil iegtt1 1i9Jment -and folj

Uldchus Peritoneum (ir median Umbilj~al Suboutane ous

Extraprftone 11ascia

Ymbilimiddot~1 fold)

preu9poundiea1 fascia

tissue (fatty 4nd m~mbr3n(iUS layers)

o Above the arcuate line (A horizontal line 13 of the distance bw the umbilicus and the

pubic symphysis) -10 Aponeurosis divides into an AntPost Laminae

o The Ant Laminae joins EO and Post Laminae joins Trans Abdominis = Ant and Post

RECTUS SHEATH respectively

o BElOW the arcuate line - all 3 aponeurosis join ANTERIOR to rectus muscle to meet its

counterpart in the midline (linea Alba)

o Take away Msg - The abdomen is devoid of a posterior rectus sheath below the

arcuate line and is therefore more vulnerable to herniasinjuries

Question - A physician makes a deep incision in the patients midline immediately superior to

the pubic symphysis which of the following layers is his knife least likely to pass

Rectus Abdominis External Oblique Ant Rectus Sheath Posterior Rectus Sheath All of the

Above

Answer - All of the above None of the other answer choices are midline structures -LINEA

ALBA

Linea Alba has very poor blood supply - doesnt heal well after surgery Therefore this is a

common site for incisional hernias

a Spleen b Transverse colon c Descending colon d Stomach e Pleura

17 Meckels diverticulum is normally found 2 feet proximal from the

a Pyloric sphincter b Lower esophageal sphincter c Ileo-cecal valve d Middle valve of Huston e Anal valve

18 Ulcer in the posterior wall of the first part of the duodenum would erode ___ artery and would cause bleeding

a Left gastric b Right gastric c Hepatic artery proper d Gastroduodenal artery e Middle colic artery

19 An inflamed appendix is identified by a surgeon on the operation table by noting

a The appendicies epiploicae b The convergence of tenia c The artery of Drummond d The mesocolon e The mesosalphinx

20 The nerve which emerges through the psoas major is

a Femoral b Ilio-inguinal c Ilio-hypogastric d Pudendal e Subcostal

21 The right gonadal vein drains into the

a Azygos b Hemiazygos c Inferior Vena Cava d Right renal vein e Left renal vein

22 The hepatocytes in the liver is derived from

a Ectoderm b Endoderm c Mesoderm

d Neural ectoderm

23 Abscess in the lumbar vertebrae due to tuberculosis would spread to the adjacent muscle which is

a Psoas Major b Iliacus c Quadratus lumborum d Tranversus Abdominis

24 The anterior wall of the inguinal canal is formed by

a External oblique and transverses abdominis b External oblique and fascia transversalis c Internal oblique and external oblique d Internal oblique and transverses abdominis e Fascia transversalis and peritoneum

Meckels diverticulum is a result of which of the following developmental abnormalities shy

A Failure of the vitelline duct to close

B Failure of the herniated intestinal loop to retract into the abdomen

C Failure of the urachus to close

D Failure of the midgut to rotate

E Failure of the hepatic duct to close

Explanation

Meckels diverticulum is a result of the persistence of the proximal part of the vitelline duct This

diverticulum is usually found about 2 feet proximal to the ileocecal junction and is usually about 2 inches

long It is present in about 2 of the popUlation It may be the site of ectopic pancreatic tissue or gastric

mucosa and may develop inflammatory processes and ulcerations Acute Meckels diverticulitis

simulates appendicitis

Which of the following veins carries blood from the esophagus to the portal vein The

A right gastric vein

B left gastric vein c splenic vein D azygos vein

E left gastroepiploic vein

Explanation

The left gastric vein a direct branch of the portal vein drains blood from the lesser curvature of the

stomach and the inferior portion of the esophagus Because branches of the portal vein do not have

valves blood can flow in a retrograde path when there is an obstruction to flow through the portal system or liveL Rlooci Cln then flow from the nortl] vein thr()1Ph the left PRstric vein to the esonhlPlIS lno

through venous communications within the submucosa of the esophagus to esophageal veins that drain

into the azygos vein The increase in blood flow through the esophageal submucosal veins results in esophageal varices

On the posterior wall of the abdomen the celiac ganglion A contains cell bodies of postganglionic parasympathetic neurons B is synapsed upon by neurons in the posterior vagal trunk C is synapsed upon by neurons in the greater splanchnic nerve D contains sensory cell bodies of lumbar spinal nerves E contains cell bodies of neurons that cause an increase in the rate of peristasis

Explanation The celiac ganglion is one of the preaortic ganglia of the sympathetic nervous system It contains cell bodies of postganglionic sympathetic neurons The sympathetic splanchnic nerves contain preganglionic sympathetic neurons that pass through the sympathetic chain without synapsing These splanchnic nerves go to the preaortic ganglia to synapse The greater splanchnic nerve contains preganglionic neurons from spinal cord segments T5-T9 This nerve synapses in the celiac ganglion The nerve fibers in the vagal trunks are preganglionic parasympathetic fibers that go to the walls of the organs that they will innervate and synapse on postganglionic parasympathetic neurons in the walls of those organs Cell bodies of sensory neurons in the abdomen are found in the dorsal root ganglia or the sensory ganglia of the vagus nerve Sympathetic innervation decreases the rate of peristalsis parasympathetic innervation increases the rate of peristalsis

Which of the following pairs of arteries will allow blood to bypass an occlusion of the celiac trunk

A Left gastric artery-right gastric artery

B Left gastroepiploic artery-right gastroepiploic artery

C Superior pancreaticoduodenal artery-inferior pancreaticoduodenal artery

D Splenic artery-common hepatic artery

E Left gastric artery - proper hepatic artery

Explanation The anastoOlosis of a branch of the celiac trunk and a branch of the superior mesenteric artery will

provide collateral circulation around an occlusion of the celiac trunk Each of the other choices pair

branches of the celiac trunk therefore these will not provide collateral flow around the obstruction of the

celiac trunk The left gastric splenic and common hepatic arteries are direct branches of the celiac trunk

The right gastric artery is a branch of the proper hepatic artery which is a branch of the common hepatic artery The left gastroepiploic artery is a branch of the splenic artery The right gastroepiploic artery is a

branch of the gastroduodenal artery whlch is a branch of the common hepatic artery

Which of the following organs has appendices epiploica The

A sigmoid colon

Bjejunum

C duodenum

D stomach E esophagus

Explanation Appendices epiploica are characteristic of the colon Appendices epiploica are subserosal accumulations

of fat None of the organs of the gastrointestinal tract has appendices epiploica except the colon

Page 45: Chirag's Abdomen Review

ANSWERS AND EXPLANATIONS

Answer E The spleen is t hlttnopodicand lymph organ demlted from mesoderm

Answ~ R Al1 tlmphalocele is caused by it failure of the nlidgut to return to the ahdomir nat cavity after herniation into the umbiliau Stalk Choices Aand D maybe seen in infants with Down syndrome choice D ~s the specific CBuse ofduudcnal JtiCSitt Choice C is (ile cause of gclstrosbisis and Choice B nsults iu a Meurolktldivertku1-tlB

Answer B The fundus ofthe stomach is suppHed by soort gastric brunches of the splenic altery The splenic artery supplies the body and tail of the pancreas part of the greater curvature of the sttmla(h and the spleen Te jejunum part of the head of the pancreas and tht~ duodenum distal to the entrance of the commOll bile duct are supplied by the superior mesenterk artery clll~l ~be less r ctlt1ature cmd the pylQric antrum are supplied by the right and lei gastric art(ries

AnSWftt C Tbeomental bursa or lesser ~ritoneaj sac lies direcdy posterior to the proxshyimal part of the duodeTtlm and the stomach and would be the first site where stomach contents ~Ott1d be fpoundluncL

Answer C A defect in a llleuropcritoneal membrane (uswlly the left) is the typical site of i1 cc-ngenitlI diilphragluatic hemia llere the membr4ne fails to dose ()pound( of the perishycCirdiopcritulleal canals

Answer E DuoJenal atresia and aganglionic megacoion are congwitaI defects S~Il in patients with Dowmiddotnsyndrome

Answer D RulaTgemt~llt of and retrograde flow in g~lstrk vel_ns in particlJl~r the kft gas~ tricveins dilates the capillary bed in rhe wall of the esophagus in (ases of porta yper~

tension Blood flow would increase in and dilampte tribntarkgts of the (lZygOUS vein on the other side of the capiUary bed but flow in this vein is in the typical direction t()ward the superior vena cava Paraumbiii(ltU vein eilgorgement contributes to a caput medusH Splenic ~nlargement might prc~nt with 5plcnonlegaly and balt-kflow in to tlu superior m~~ntclic vein occurs but is asymptomatic

Answer D The patient hagt an indirect inguinal hernia whi~h emerges from the antt-rior abdominal wall through the deep inguinltilling Theeep ring is a fault in the transv~rshysaUs fascia this I~yer wiIJ be penetrated first by the hernia

An~Wer C The ilioinguinal nenc which provides sens~llion to the lnedlal thigh ltmclanteshytior SClotunl pass~lt th rough the 5uperfh_ial inguinal ring ind $subject to inj i1T) becaus-e

it is in the operatitm Held of the erniorrhapny

Auswer B The leMHr splanchnic nerves are sympathdic nerVlts that carry viscera l sensashytlltgtrogt ftom illtllt1m~d ()J stietched gust (itinteitinal ~tructures (in this case the pprndix) into tnt eNS Lesser splanchnic ntTYcsarisc from thmiddot T9--T12 spinal cord segments lt1nd provide sympathetic innenation tD rnidgut siruc1ures whiCh include CLe app~JldD Viscera] Pain arising from affecLed Inidgut ampt 1C1ure is referred over the same dl- matorne~ of spinal segrnertts v-hich provide the sympathetic Innervation n this G1SC of appendicitis the invohen~n t of the ltire) of t e unlhHku indud s the T 10 dermatome

Answer B Of the five choices onty the dscending olon is retroperiton~al aldwould be a lik ~ ( choice to be seen immediately a(~jilcent to t11e posterior abdominal middotn~L

Amwen D The SpltftlC ~-ein ourses posterior to the body of the panneas m its way tt drain into the superior mCSfttltlri( vein

Answcr B TILt glstrodllolticnal artery 1 direct hIamh of the comrootl hepatic artery courses immediately pt))iwri() to the duodenum and is slbject to erosion

Answer B Carcinoma of th pan middott3S in the 1tilt1 may compreampgt the portltil vein at irs orishygill The poTtai vcin is fomled when the splenic vein jQiaswith tfie superior meStllt eric vein The inferiot mesenteric vein joins the ~plenjc vein just priOT to tlli~ point at which the splenic joins the superior Jlleit1ltcri( vein Increescd venous presslu in the inferior mesenteric vein is a cause of emo hoid~

Answer C The- velltral pancreas wilich forms most of the head of the p ~ncr as develops in the ventral mes(ntery as antutgrowth of the hepatic diverticulum Th~ hepatic divershyticulull induding the biIJary appa~atus develops in tbe ventral mesentery of the foregut

Answer~ A The superior mesenteric ~in joins with the spienkvein to form the hepatic portal vciu

Answer D The structure at gttlK is the proper hepatic artery~ whkh suppUesoxygenated b middotood to the liver

MAKE SURE YOU KNOW the diff bw Rectus Sheath above and below the arcuate line

ABOVE

Aponeurosis of xiiltmal obllque musclo

Extemll f)biquw musde

Reotln ilbdomlnls musole S~in

Internal 9bliquQ mY~QI

AponeUfOsi$ of hJH$V~~S Lir9a a lb lbdolTlin~ musolo Tri OJV6 rUi

atldomlnis mUS(loe

Sub cutanlilous tiue (tatty ye r)

BElOW

A POrl lJfosis 01 etemal oblique muscle

Aponeul~)sis 01 Internal oblique mU$cl~

Anteriol lay~ of r~ltdus st~ath EXttom1 oblique rnu$cll

Rectus Jbdominis muscle Intoernal Aponeurc-sis of tra~fersU$ oblique muscle-

at-domlnis muscentl ~ Skio

Tra nsvitSus abdomioLs ml)ZClt

TralSVersaHs fascia Medial umQil iegtt1 1i9Jment -and folj

Uldchus Peritoneum (ir median Umbilj~al Suboutane ous

Extraprftone 11ascia

Ymbilimiddot~1 fold)

preu9poundiea1 fascia

tissue (fatty 4nd m~mbr3n(iUS layers)

o Above the arcuate line (A horizontal line 13 of the distance bw the umbilicus and the

pubic symphysis) -10 Aponeurosis divides into an AntPost Laminae

o The Ant Laminae joins EO and Post Laminae joins Trans Abdominis = Ant and Post

RECTUS SHEATH respectively

o BElOW the arcuate line - all 3 aponeurosis join ANTERIOR to rectus muscle to meet its

counterpart in the midline (linea Alba)

o Take away Msg - The abdomen is devoid of a posterior rectus sheath below the

arcuate line and is therefore more vulnerable to herniasinjuries

Question - A physician makes a deep incision in the patients midline immediately superior to

the pubic symphysis which of the following layers is his knife least likely to pass

Rectus Abdominis External Oblique Ant Rectus Sheath Posterior Rectus Sheath All of the

Above

Answer - All of the above None of the other answer choices are midline structures -LINEA

ALBA

Linea Alba has very poor blood supply - doesnt heal well after surgery Therefore this is a

common site for incisional hernias

a Spleen b Transverse colon c Descending colon d Stomach e Pleura

17 Meckels diverticulum is normally found 2 feet proximal from the

a Pyloric sphincter b Lower esophageal sphincter c Ileo-cecal valve d Middle valve of Huston e Anal valve

18 Ulcer in the posterior wall of the first part of the duodenum would erode ___ artery and would cause bleeding

a Left gastric b Right gastric c Hepatic artery proper d Gastroduodenal artery e Middle colic artery

19 An inflamed appendix is identified by a surgeon on the operation table by noting

a The appendicies epiploicae b The convergence of tenia c The artery of Drummond d The mesocolon e The mesosalphinx

20 The nerve which emerges through the psoas major is

a Femoral b Ilio-inguinal c Ilio-hypogastric d Pudendal e Subcostal

21 The right gonadal vein drains into the

a Azygos b Hemiazygos c Inferior Vena Cava d Right renal vein e Left renal vein

22 The hepatocytes in the liver is derived from

a Ectoderm b Endoderm c Mesoderm

d Neural ectoderm

23 Abscess in the lumbar vertebrae due to tuberculosis would spread to the adjacent muscle which is

a Psoas Major b Iliacus c Quadratus lumborum d Tranversus Abdominis

24 The anterior wall of the inguinal canal is formed by

a External oblique and transverses abdominis b External oblique and fascia transversalis c Internal oblique and external oblique d Internal oblique and transverses abdominis e Fascia transversalis and peritoneum

Meckels diverticulum is a result of which of the following developmental abnormalities shy

A Failure of the vitelline duct to close

B Failure of the herniated intestinal loop to retract into the abdomen

C Failure of the urachus to close

D Failure of the midgut to rotate

E Failure of the hepatic duct to close

Explanation

Meckels diverticulum is a result of the persistence of the proximal part of the vitelline duct This

diverticulum is usually found about 2 feet proximal to the ileocecal junction and is usually about 2 inches

long It is present in about 2 of the popUlation It may be the site of ectopic pancreatic tissue or gastric

mucosa and may develop inflammatory processes and ulcerations Acute Meckels diverticulitis

simulates appendicitis

Which of the following veins carries blood from the esophagus to the portal vein The

A right gastric vein

B left gastric vein c splenic vein D azygos vein

E left gastroepiploic vein

Explanation

The left gastric vein a direct branch of the portal vein drains blood from the lesser curvature of the

stomach and the inferior portion of the esophagus Because branches of the portal vein do not have

valves blood can flow in a retrograde path when there is an obstruction to flow through the portal system or liveL Rlooci Cln then flow from the nortl] vein thr()1Ph the left PRstric vein to the esonhlPlIS lno

through venous communications within the submucosa of the esophagus to esophageal veins that drain

into the azygos vein The increase in blood flow through the esophageal submucosal veins results in esophageal varices

On the posterior wall of the abdomen the celiac ganglion A contains cell bodies of postganglionic parasympathetic neurons B is synapsed upon by neurons in the posterior vagal trunk C is synapsed upon by neurons in the greater splanchnic nerve D contains sensory cell bodies of lumbar spinal nerves E contains cell bodies of neurons that cause an increase in the rate of peristasis

Explanation The celiac ganglion is one of the preaortic ganglia of the sympathetic nervous system It contains cell bodies of postganglionic sympathetic neurons The sympathetic splanchnic nerves contain preganglionic sympathetic neurons that pass through the sympathetic chain without synapsing These splanchnic nerves go to the preaortic ganglia to synapse The greater splanchnic nerve contains preganglionic neurons from spinal cord segments T5-T9 This nerve synapses in the celiac ganglion The nerve fibers in the vagal trunks are preganglionic parasympathetic fibers that go to the walls of the organs that they will innervate and synapse on postganglionic parasympathetic neurons in the walls of those organs Cell bodies of sensory neurons in the abdomen are found in the dorsal root ganglia or the sensory ganglia of the vagus nerve Sympathetic innervation decreases the rate of peristalsis parasympathetic innervation increases the rate of peristalsis

Which of the following pairs of arteries will allow blood to bypass an occlusion of the celiac trunk

A Left gastric artery-right gastric artery

B Left gastroepiploic artery-right gastroepiploic artery

C Superior pancreaticoduodenal artery-inferior pancreaticoduodenal artery

D Splenic artery-common hepatic artery

E Left gastric artery - proper hepatic artery

Explanation The anastoOlosis of a branch of the celiac trunk and a branch of the superior mesenteric artery will

provide collateral circulation around an occlusion of the celiac trunk Each of the other choices pair

branches of the celiac trunk therefore these will not provide collateral flow around the obstruction of the

celiac trunk The left gastric splenic and common hepatic arteries are direct branches of the celiac trunk

The right gastric artery is a branch of the proper hepatic artery which is a branch of the common hepatic artery The left gastroepiploic artery is a branch of the splenic artery The right gastroepiploic artery is a

branch of the gastroduodenal artery whlch is a branch of the common hepatic artery

Which of the following organs has appendices epiploica The

A sigmoid colon

Bjejunum

C duodenum

D stomach E esophagus

Explanation Appendices epiploica are characteristic of the colon Appendices epiploica are subserosal accumulations

of fat None of the organs of the gastrointestinal tract has appendices epiploica except the colon

Page 46: Chirag's Abdomen Review

Answer B Of the five choices onty the dscending olon is retroperiton~al aldwould be a lik ~ ( choice to be seen immediately a(~jilcent to t11e posterior abdominal middotn~L

Amwen D The SpltftlC ~-ein ourses posterior to the body of the panneas m its way tt drain into the superior mCSfttltlri( vein

Answcr B TILt glstrodllolticnal artery 1 direct hIamh of the comrootl hepatic artery courses immediately pt))iwri() to the duodenum and is slbject to erosion

Answer B Carcinoma of th pan middott3S in the 1tilt1 may compreampgt the portltil vein at irs orishygill The poTtai vcin is fomled when the splenic vein jQiaswith tfie superior meStllt eric vein The inferiot mesenteric vein joins the ~plenjc vein just priOT to tlli~ point at which the splenic joins the superior Jlleit1ltcri( vein Increescd venous presslu in the inferior mesenteric vein is a cause of emo hoid~

Answer C The- velltral pancreas wilich forms most of the head of the p ~ncr as develops in the ventral mes(ntery as antutgrowth of the hepatic diverticulum Th~ hepatic divershyticulull induding the biIJary appa~atus develops in tbe ventral mesentery of the foregut

Answer~ A The superior mesenteric ~in joins with the spienkvein to form the hepatic portal vciu

Answer D The structure at gttlK is the proper hepatic artery~ whkh suppUesoxygenated b middotood to the liver

MAKE SURE YOU KNOW the diff bw Rectus Sheath above and below the arcuate line

ABOVE

Aponeurosis of xiiltmal obllque musclo

Extemll f)biquw musde

Reotln ilbdomlnls musole S~in

Internal 9bliquQ mY~QI

AponeUfOsi$ of hJH$V~~S Lir9a a lb lbdolTlin~ musolo Tri OJV6 rUi

atldomlnis mUS(loe

Sub cutanlilous tiue (tatty ye r)

BElOW

A POrl lJfosis 01 etemal oblique muscle

Aponeul~)sis 01 Internal oblique mU$cl~

Anteriol lay~ of r~ltdus st~ath EXttom1 oblique rnu$cll

Rectus Jbdominis muscle Intoernal Aponeurc-sis of tra~fersU$ oblique muscle-

at-domlnis muscentl ~ Skio

Tra nsvitSus abdomioLs ml)ZClt

TralSVersaHs fascia Medial umQil iegtt1 1i9Jment -and folj

Uldchus Peritoneum (ir median Umbilj~al Suboutane ous

Extraprftone 11ascia

Ymbilimiddot~1 fold)

preu9poundiea1 fascia

tissue (fatty 4nd m~mbr3n(iUS layers)

o Above the arcuate line (A horizontal line 13 of the distance bw the umbilicus and the

pubic symphysis) -10 Aponeurosis divides into an AntPost Laminae

o The Ant Laminae joins EO and Post Laminae joins Trans Abdominis = Ant and Post

RECTUS SHEATH respectively

o BElOW the arcuate line - all 3 aponeurosis join ANTERIOR to rectus muscle to meet its

counterpart in the midline (linea Alba)

o Take away Msg - The abdomen is devoid of a posterior rectus sheath below the

arcuate line and is therefore more vulnerable to herniasinjuries

Question - A physician makes a deep incision in the patients midline immediately superior to

the pubic symphysis which of the following layers is his knife least likely to pass

Rectus Abdominis External Oblique Ant Rectus Sheath Posterior Rectus Sheath All of the

Above

Answer - All of the above None of the other answer choices are midline structures -LINEA

ALBA

Linea Alba has very poor blood supply - doesnt heal well after surgery Therefore this is a

common site for incisional hernias

a Spleen b Transverse colon c Descending colon d Stomach e Pleura

17 Meckels diverticulum is normally found 2 feet proximal from the

a Pyloric sphincter b Lower esophageal sphincter c Ileo-cecal valve d Middle valve of Huston e Anal valve

18 Ulcer in the posterior wall of the first part of the duodenum would erode ___ artery and would cause bleeding

a Left gastric b Right gastric c Hepatic artery proper d Gastroduodenal artery e Middle colic artery

19 An inflamed appendix is identified by a surgeon on the operation table by noting

a The appendicies epiploicae b The convergence of tenia c The artery of Drummond d The mesocolon e The mesosalphinx

20 The nerve which emerges through the psoas major is

a Femoral b Ilio-inguinal c Ilio-hypogastric d Pudendal e Subcostal

21 The right gonadal vein drains into the

a Azygos b Hemiazygos c Inferior Vena Cava d Right renal vein e Left renal vein

22 The hepatocytes in the liver is derived from

a Ectoderm b Endoderm c Mesoderm

d Neural ectoderm

23 Abscess in the lumbar vertebrae due to tuberculosis would spread to the adjacent muscle which is

a Psoas Major b Iliacus c Quadratus lumborum d Tranversus Abdominis

24 The anterior wall of the inguinal canal is formed by

a External oblique and transverses abdominis b External oblique and fascia transversalis c Internal oblique and external oblique d Internal oblique and transverses abdominis e Fascia transversalis and peritoneum

Meckels diverticulum is a result of which of the following developmental abnormalities shy

A Failure of the vitelline duct to close

B Failure of the herniated intestinal loop to retract into the abdomen

C Failure of the urachus to close

D Failure of the midgut to rotate

E Failure of the hepatic duct to close

Explanation

Meckels diverticulum is a result of the persistence of the proximal part of the vitelline duct This

diverticulum is usually found about 2 feet proximal to the ileocecal junction and is usually about 2 inches

long It is present in about 2 of the popUlation It may be the site of ectopic pancreatic tissue or gastric

mucosa and may develop inflammatory processes and ulcerations Acute Meckels diverticulitis

simulates appendicitis

Which of the following veins carries blood from the esophagus to the portal vein The

A right gastric vein

B left gastric vein c splenic vein D azygos vein

E left gastroepiploic vein

Explanation

The left gastric vein a direct branch of the portal vein drains blood from the lesser curvature of the

stomach and the inferior portion of the esophagus Because branches of the portal vein do not have

valves blood can flow in a retrograde path when there is an obstruction to flow through the portal system or liveL Rlooci Cln then flow from the nortl] vein thr()1Ph the left PRstric vein to the esonhlPlIS lno

through venous communications within the submucosa of the esophagus to esophageal veins that drain

into the azygos vein The increase in blood flow through the esophageal submucosal veins results in esophageal varices

On the posterior wall of the abdomen the celiac ganglion A contains cell bodies of postganglionic parasympathetic neurons B is synapsed upon by neurons in the posterior vagal trunk C is synapsed upon by neurons in the greater splanchnic nerve D contains sensory cell bodies of lumbar spinal nerves E contains cell bodies of neurons that cause an increase in the rate of peristasis

Explanation The celiac ganglion is one of the preaortic ganglia of the sympathetic nervous system It contains cell bodies of postganglionic sympathetic neurons The sympathetic splanchnic nerves contain preganglionic sympathetic neurons that pass through the sympathetic chain without synapsing These splanchnic nerves go to the preaortic ganglia to synapse The greater splanchnic nerve contains preganglionic neurons from spinal cord segments T5-T9 This nerve synapses in the celiac ganglion The nerve fibers in the vagal trunks are preganglionic parasympathetic fibers that go to the walls of the organs that they will innervate and synapse on postganglionic parasympathetic neurons in the walls of those organs Cell bodies of sensory neurons in the abdomen are found in the dorsal root ganglia or the sensory ganglia of the vagus nerve Sympathetic innervation decreases the rate of peristalsis parasympathetic innervation increases the rate of peristalsis

Which of the following pairs of arteries will allow blood to bypass an occlusion of the celiac trunk

A Left gastric artery-right gastric artery

B Left gastroepiploic artery-right gastroepiploic artery

C Superior pancreaticoduodenal artery-inferior pancreaticoduodenal artery

D Splenic artery-common hepatic artery

E Left gastric artery - proper hepatic artery

Explanation The anastoOlosis of a branch of the celiac trunk and a branch of the superior mesenteric artery will

provide collateral circulation around an occlusion of the celiac trunk Each of the other choices pair

branches of the celiac trunk therefore these will not provide collateral flow around the obstruction of the

celiac trunk The left gastric splenic and common hepatic arteries are direct branches of the celiac trunk

The right gastric artery is a branch of the proper hepatic artery which is a branch of the common hepatic artery The left gastroepiploic artery is a branch of the splenic artery The right gastroepiploic artery is a

branch of the gastroduodenal artery whlch is a branch of the common hepatic artery

Which of the following organs has appendices epiploica The

A sigmoid colon

Bjejunum

C duodenum

D stomach E esophagus

Explanation Appendices epiploica are characteristic of the colon Appendices epiploica are subserosal accumulations

of fat None of the organs of the gastrointestinal tract has appendices epiploica except the colon

Page 47: Chirag's Abdomen Review

MAKE SURE YOU KNOW the diff bw Rectus Sheath above and below the arcuate line

ABOVE

Aponeurosis of xiiltmal obllque musclo

Extemll f)biquw musde

Reotln ilbdomlnls musole S~in

Internal 9bliquQ mY~QI

AponeUfOsi$ of hJH$V~~S Lir9a a lb lbdolTlin~ musolo Tri OJV6 rUi

atldomlnis mUS(loe

Sub cutanlilous tiue (tatty ye r)

BElOW

A POrl lJfosis 01 etemal oblique muscle

Aponeul~)sis 01 Internal oblique mU$cl~

Anteriol lay~ of r~ltdus st~ath EXttom1 oblique rnu$cll

Rectus Jbdominis muscle Intoernal Aponeurc-sis of tra~fersU$ oblique muscle-

at-domlnis muscentl ~ Skio

Tra nsvitSus abdomioLs ml)ZClt

TralSVersaHs fascia Medial umQil iegtt1 1i9Jment -and folj

Uldchus Peritoneum (ir median Umbilj~al Suboutane ous

Extraprftone 11ascia

Ymbilimiddot~1 fold)

preu9poundiea1 fascia

tissue (fatty 4nd m~mbr3n(iUS layers)

o Above the arcuate line (A horizontal line 13 of the distance bw the umbilicus and the

pubic symphysis) -10 Aponeurosis divides into an AntPost Laminae

o The Ant Laminae joins EO and Post Laminae joins Trans Abdominis = Ant and Post

RECTUS SHEATH respectively

o BElOW the arcuate line - all 3 aponeurosis join ANTERIOR to rectus muscle to meet its

counterpart in the midline (linea Alba)

o Take away Msg - The abdomen is devoid of a posterior rectus sheath below the

arcuate line and is therefore more vulnerable to herniasinjuries

Question - A physician makes a deep incision in the patients midline immediately superior to

the pubic symphysis which of the following layers is his knife least likely to pass

Rectus Abdominis External Oblique Ant Rectus Sheath Posterior Rectus Sheath All of the

Above

Answer - All of the above None of the other answer choices are midline structures -LINEA

ALBA

Linea Alba has very poor blood supply - doesnt heal well after surgery Therefore this is a

common site for incisional hernias

a Spleen b Transverse colon c Descending colon d Stomach e Pleura

17 Meckels diverticulum is normally found 2 feet proximal from the

a Pyloric sphincter b Lower esophageal sphincter c Ileo-cecal valve d Middle valve of Huston e Anal valve

18 Ulcer in the posterior wall of the first part of the duodenum would erode ___ artery and would cause bleeding

a Left gastric b Right gastric c Hepatic artery proper d Gastroduodenal artery e Middle colic artery

19 An inflamed appendix is identified by a surgeon on the operation table by noting

a The appendicies epiploicae b The convergence of tenia c The artery of Drummond d The mesocolon e The mesosalphinx

20 The nerve which emerges through the psoas major is

a Femoral b Ilio-inguinal c Ilio-hypogastric d Pudendal e Subcostal

21 The right gonadal vein drains into the

a Azygos b Hemiazygos c Inferior Vena Cava d Right renal vein e Left renal vein

22 The hepatocytes in the liver is derived from

a Ectoderm b Endoderm c Mesoderm

d Neural ectoderm

23 Abscess in the lumbar vertebrae due to tuberculosis would spread to the adjacent muscle which is

a Psoas Major b Iliacus c Quadratus lumborum d Tranversus Abdominis

24 The anterior wall of the inguinal canal is formed by

a External oblique and transverses abdominis b External oblique and fascia transversalis c Internal oblique and external oblique d Internal oblique and transverses abdominis e Fascia transversalis and peritoneum

Meckels diverticulum is a result of which of the following developmental abnormalities shy

A Failure of the vitelline duct to close

B Failure of the herniated intestinal loop to retract into the abdomen

C Failure of the urachus to close

D Failure of the midgut to rotate

E Failure of the hepatic duct to close

Explanation

Meckels diverticulum is a result of the persistence of the proximal part of the vitelline duct This

diverticulum is usually found about 2 feet proximal to the ileocecal junction and is usually about 2 inches

long It is present in about 2 of the popUlation It may be the site of ectopic pancreatic tissue or gastric

mucosa and may develop inflammatory processes and ulcerations Acute Meckels diverticulitis

simulates appendicitis

Which of the following veins carries blood from the esophagus to the portal vein The

A right gastric vein

B left gastric vein c splenic vein D azygos vein

E left gastroepiploic vein

Explanation

The left gastric vein a direct branch of the portal vein drains blood from the lesser curvature of the

stomach and the inferior portion of the esophagus Because branches of the portal vein do not have

valves blood can flow in a retrograde path when there is an obstruction to flow through the portal system or liveL Rlooci Cln then flow from the nortl] vein thr()1Ph the left PRstric vein to the esonhlPlIS lno

through venous communications within the submucosa of the esophagus to esophageal veins that drain

into the azygos vein The increase in blood flow through the esophageal submucosal veins results in esophageal varices

On the posterior wall of the abdomen the celiac ganglion A contains cell bodies of postganglionic parasympathetic neurons B is synapsed upon by neurons in the posterior vagal trunk C is synapsed upon by neurons in the greater splanchnic nerve D contains sensory cell bodies of lumbar spinal nerves E contains cell bodies of neurons that cause an increase in the rate of peristasis

Explanation The celiac ganglion is one of the preaortic ganglia of the sympathetic nervous system It contains cell bodies of postganglionic sympathetic neurons The sympathetic splanchnic nerves contain preganglionic sympathetic neurons that pass through the sympathetic chain without synapsing These splanchnic nerves go to the preaortic ganglia to synapse The greater splanchnic nerve contains preganglionic neurons from spinal cord segments T5-T9 This nerve synapses in the celiac ganglion The nerve fibers in the vagal trunks are preganglionic parasympathetic fibers that go to the walls of the organs that they will innervate and synapse on postganglionic parasympathetic neurons in the walls of those organs Cell bodies of sensory neurons in the abdomen are found in the dorsal root ganglia or the sensory ganglia of the vagus nerve Sympathetic innervation decreases the rate of peristalsis parasympathetic innervation increases the rate of peristalsis

Which of the following pairs of arteries will allow blood to bypass an occlusion of the celiac trunk

A Left gastric artery-right gastric artery

B Left gastroepiploic artery-right gastroepiploic artery

C Superior pancreaticoduodenal artery-inferior pancreaticoduodenal artery

D Splenic artery-common hepatic artery

E Left gastric artery - proper hepatic artery

Explanation The anastoOlosis of a branch of the celiac trunk and a branch of the superior mesenteric artery will

provide collateral circulation around an occlusion of the celiac trunk Each of the other choices pair

branches of the celiac trunk therefore these will not provide collateral flow around the obstruction of the

celiac trunk The left gastric splenic and common hepatic arteries are direct branches of the celiac trunk

The right gastric artery is a branch of the proper hepatic artery which is a branch of the common hepatic artery The left gastroepiploic artery is a branch of the splenic artery The right gastroepiploic artery is a

branch of the gastroduodenal artery whlch is a branch of the common hepatic artery

Which of the following organs has appendices epiploica The

A sigmoid colon

Bjejunum

C duodenum

D stomach E esophagus

Explanation Appendices epiploica are characteristic of the colon Appendices epiploica are subserosal accumulations

of fat None of the organs of the gastrointestinal tract has appendices epiploica except the colon

Page 48: Chirag's Abdomen Review

a Spleen b Transverse colon c Descending colon d Stomach e Pleura

17 Meckels diverticulum is normally found 2 feet proximal from the

a Pyloric sphincter b Lower esophageal sphincter c Ileo-cecal valve d Middle valve of Huston e Anal valve

18 Ulcer in the posterior wall of the first part of the duodenum would erode ___ artery and would cause bleeding

a Left gastric b Right gastric c Hepatic artery proper d Gastroduodenal artery e Middle colic artery

19 An inflamed appendix is identified by a surgeon on the operation table by noting

a The appendicies epiploicae b The convergence of tenia c The artery of Drummond d The mesocolon e The mesosalphinx

20 The nerve which emerges through the psoas major is

a Femoral b Ilio-inguinal c Ilio-hypogastric d Pudendal e Subcostal

21 The right gonadal vein drains into the

a Azygos b Hemiazygos c Inferior Vena Cava d Right renal vein e Left renal vein

22 The hepatocytes in the liver is derived from

a Ectoderm b Endoderm c Mesoderm

d Neural ectoderm

23 Abscess in the lumbar vertebrae due to tuberculosis would spread to the adjacent muscle which is

a Psoas Major b Iliacus c Quadratus lumborum d Tranversus Abdominis

24 The anterior wall of the inguinal canal is formed by

a External oblique and transverses abdominis b External oblique and fascia transversalis c Internal oblique and external oblique d Internal oblique and transverses abdominis e Fascia transversalis and peritoneum

Meckels diverticulum is a result of which of the following developmental abnormalities shy

A Failure of the vitelline duct to close

B Failure of the herniated intestinal loop to retract into the abdomen

C Failure of the urachus to close

D Failure of the midgut to rotate

E Failure of the hepatic duct to close

Explanation

Meckels diverticulum is a result of the persistence of the proximal part of the vitelline duct This

diverticulum is usually found about 2 feet proximal to the ileocecal junction and is usually about 2 inches

long It is present in about 2 of the popUlation It may be the site of ectopic pancreatic tissue or gastric

mucosa and may develop inflammatory processes and ulcerations Acute Meckels diverticulitis

simulates appendicitis

Which of the following veins carries blood from the esophagus to the portal vein The

A right gastric vein

B left gastric vein c splenic vein D azygos vein

E left gastroepiploic vein

Explanation

The left gastric vein a direct branch of the portal vein drains blood from the lesser curvature of the

stomach and the inferior portion of the esophagus Because branches of the portal vein do not have

valves blood can flow in a retrograde path when there is an obstruction to flow through the portal system or liveL Rlooci Cln then flow from the nortl] vein thr()1Ph the left PRstric vein to the esonhlPlIS lno

through venous communications within the submucosa of the esophagus to esophageal veins that drain

into the azygos vein The increase in blood flow through the esophageal submucosal veins results in esophageal varices

On the posterior wall of the abdomen the celiac ganglion A contains cell bodies of postganglionic parasympathetic neurons B is synapsed upon by neurons in the posterior vagal trunk C is synapsed upon by neurons in the greater splanchnic nerve D contains sensory cell bodies of lumbar spinal nerves E contains cell bodies of neurons that cause an increase in the rate of peristasis

Explanation The celiac ganglion is one of the preaortic ganglia of the sympathetic nervous system It contains cell bodies of postganglionic sympathetic neurons The sympathetic splanchnic nerves contain preganglionic sympathetic neurons that pass through the sympathetic chain without synapsing These splanchnic nerves go to the preaortic ganglia to synapse The greater splanchnic nerve contains preganglionic neurons from spinal cord segments T5-T9 This nerve synapses in the celiac ganglion The nerve fibers in the vagal trunks are preganglionic parasympathetic fibers that go to the walls of the organs that they will innervate and synapse on postganglionic parasympathetic neurons in the walls of those organs Cell bodies of sensory neurons in the abdomen are found in the dorsal root ganglia or the sensory ganglia of the vagus nerve Sympathetic innervation decreases the rate of peristalsis parasympathetic innervation increases the rate of peristalsis

Which of the following pairs of arteries will allow blood to bypass an occlusion of the celiac trunk

A Left gastric artery-right gastric artery

B Left gastroepiploic artery-right gastroepiploic artery

C Superior pancreaticoduodenal artery-inferior pancreaticoduodenal artery

D Splenic artery-common hepatic artery

E Left gastric artery - proper hepatic artery

Explanation The anastoOlosis of a branch of the celiac trunk and a branch of the superior mesenteric artery will

provide collateral circulation around an occlusion of the celiac trunk Each of the other choices pair

branches of the celiac trunk therefore these will not provide collateral flow around the obstruction of the

celiac trunk The left gastric splenic and common hepatic arteries are direct branches of the celiac trunk

The right gastric artery is a branch of the proper hepatic artery which is a branch of the common hepatic artery The left gastroepiploic artery is a branch of the splenic artery The right gastroepiploic artery is a

branch of the gastroduodenal artery whlch is a branch of the common hepatic artery

Which of the following organs has appendices epiploica The

A sigmoid colon

Bjejunum

C duodenum

D stomach E esophagus

Explanation Appendices epiploica are characteristic of the colon Appendices epiploica are subserosal accumulations

of fat None of the organs of the gastrointestinal tract has appendices epiploica except the colon

Page 49: Chirag's Abdomen Review

d Neural ectoderm

23 Abscess in the lumbar vertebrae due to tuberculosis would spread to the adjacent muscle which is

a Psoas Major b Iliacus c Quadratus lumborum d Tranversus Abdominis

24 The anterior wall of the inguinal canal is formed by

a External oblique and transverses abdominis b External oblique and fascia transversalis c Internal oblique and external oblique d Internal oblique and transverses abdominis e Fascia transversalis and peritoneum

Meckels diverticulum is a result of which of the following developmental abnormalities shy

A Failure of the vitelline duct to close

B Failure of the herniated intestinal loop to retract into the abdomen

C Failure of the urachus to close

D Failure of the midgut to rotate

E Failure of the hepatic duct to close

Explanation

Meckels diverticulum is a result of the persistence of the proximal part of the vitelline duct This

diverticulum is usually found about 2 feet proximal to the ileocecal junction and is usually about 2 inches

long It is present in about 2 of the popUlation It may be the site of ectopic pancreatic tissue or gastric

mucosa and may develop inflammatory processes and ulcerations Acute Meckels diverticulitis

simulates appendicitis

Which of the following veins carries blood from the esophagus to the portal vein The

A right gastric vein

B left gastric vein c splenic vein D azygos vein

E left gastroepiploic vein

Explanation

The left gastric vein a direct branch of the portal vein drains blood from the lesser curvature of the

stomach and the inferior portion of the esophagus Because branches of the portal vein do not have

valves blood can flow in a retrograde path when there is an obstruction to flow through the portal system or liveL Rlooci Cln then flow from the nortl] vein thr()1Ph the left PRstric vein to the esonhlPlIS lno

through venous communications within the submucosa of the esophagus to esophageal veins that drain

into the azygos vein The increase in blood flow through the esophageal submucosal veins results in esophageal varices

On the posterior wall of the abdomen the celiac ganglion A contains cell bodies of postganglionic parasympathetic neurons B is synapsed upon by neurons in the posterior vagal trunk C is synapsed upon by neurons in the greater splanchnic nerve D contains sensory cell bodies of lumbar spinal nerves E contains cell bodies of neurons that cause an increase in the rate of peristasis

Explanation The celiac ganglion is one of the preaortic ganglia of the sympathetic nervous system It contains cell bodies of postganglionic sympathetic neurons The sympathetic splanchnic nerves contain preganglionic sympathetic neurons that pass through the sympathetic chain without synapsing These splanchnic nerves go to the preaortic ganglia to synapse The greater splanchnic nerve contains preganglionic neurons from spinal cord segments T5-T9 This nerve synapses in the celiac ganglion The nerve fibers in the vagal trunks are preganglionic parasympathetic fibers that go to the walls of the organs that they will innervate and synapse on postganglionic parasympathetic neurons in the walls of those organs Cell bodies of sensory neurons in the abdomen are found in the dorsal root ganglia or the sensory ganglia of the vagus nerve Sympathetic innervation decreases the rate of peristalsis parasympathetic innervation increases the rate of peristalsis

Which of the following pairs of arteries will allow blood to bypass an occlusion of the celiac trunk

A Left gastric artery-right gastric artery

B Left gastroepiploic artery-right gastroepiploic artery

C Superior pancreaticoduodenal artery-inferior pancreaticoduodenal artery

D Splenic artery-common hepatic artery

E Left gastric artery - proper hepatic artery

Explanation The anastoOlosis of a branch of the celiac trunk and a branch of the superior mesenteric artery will

provide collateral circulation around an occlusion of the celiac trunk Each of the other choices pair

branches of the celiac trunk therefore these will not provide collateral flow around the obstruction of the

celiac trunk The left gastric splenic and common hepatic arteries are direct branches of the celiac trunk

The right gastric artery is a branch of the proper hepatic artery which is a branch of the common hepatic artery The left gastroepiploic artery is a branch of the splenic artery The right gastroepiploic artery is a

branch of the gastroduodenal artery whlch is a branch of the common hepatic artery

Which of the following organs has appendices epiploica The

A sigmoid colon

Bjejunum

C duodenum

D stomach E esophagus

Explanation Appendices epiploica are characteristic of the colon Appendices epiploica are subserosal accumulations

of fat None of the organs of the gastrointestinal tract has appendices epiploica except the colon

Page 50: Chirag's Abdomen Review

through venous communications within the submucosa of the esophagus to esophageal veins that drain

into the azygos vein The increase in blood flow through the esophageal submucosal veins results in esophageal varices

On the posterior wall of the abdomen the celiac ganglion A contains cell bodies of postganglionic parasympathetic neurons B is synapsed upon by neurons in the posterior vagal trunk C is synapsed upon by neurons in the greater splanchnic nerve D contains sensory cell bodies of lumbar spinal nerves E contains cell bodies of neurons that cause an increase in the rate of peristasis

Explanation The celiac ganglion is one of the preaortic ganglia of the sympathetic nervous system It contains cell bodies of postganglionic sympathetic neurons The sympathetic splanchnic nerves contain preganglionic sympathetic neurons that pass through the sympathetic chain without synapsing These splanchnic nerves go to the preaortic ganglia to synapse The greater splanchnic nerve contains preganglionic neurons from spinal cord segments T5-T9 This nerve synapses in the celiac ganglion The nerve fibers in the vagal trunks are preganglionic parasympathetic fibers that go to the walls of the organs that they will innervate and synapse on postganglionic parasympathetic neurons in the walls of those organs Cell bodies of sensory neurons in the abdomen are found in the dorsal root ganglia or the sensory ganglia of the vagus nerve Sympathetic innervation decreases the rate of peristalsis parasympathetic innervation increases the rate of peristalsis

Which of the following pairs of arteries will allow blood to bypass an occlusion of the celiac trunk

A Left gastric artery-right gastric artery

B Left gastroepiploic artery-right gastroepiploic artery

C Superior pancreaticoduodenal artery-inferior pancreaticoduodenal artery

D Splenic artery-common hepatic artery

E Left gastric artery - proper hepatic artery

Explanation The anastoOlosis of a branch of the celiac trunk and a branch of the superior mesenteric artery will

provide collateral circulation around an occlusion of the celiac trunk Each of the other choices pair

branches of the celiac trunk therefore these will not provide collateral flow around the obstruction of the

celiac trunk The left gastric splenic and common hepatic arteries are direct branches of the celiac trunk

The right gastric artery is a branch of the proper hepatic artery which is a branch of the common hepatic artery The left gastroepiploic artery is a branch of the splenic artery The right gastroepiploic artery is a

branch of the gastroduodenal artery whlch is a branch of the common hepatic artery

Which of the following organs has appendices epiploica The

A sigmoid colon

Bjejunum

C duodenum

D stomach E esophagus

Explanation Appendices epiploica are characteristic of the colon Appendices epiploica are subserosal accumulations

of fat None of the organs of the gastrointestinal tract has appendices epiploica except the colon